Sunteți pe pagina 1din 965

Module

1
Objectives and Methods
of Analysis and Design,
and Properties of
Concrete and Steel
Version 2 CE IIT, Kharagpur
Lesson
1
Objectives and Methods
of Analysis and Design
Version 2 CE IIT, Kharagpur
Instructional Objectives:
At the end of this lesson, the student should be able to:

• state the four objectives of the design of reinforced concrete structures,


• name the three methods of design of concrete structure and identify the best
method of design,
• state the basis of the analysis of the structure,
• express the design loads in terms of characteristic loads in limit state and
working stress methods,
• define the characteristic load,
• name the different loads, forces and effects to be considered in the design,
• state the basis of determining the combination of different loads acting on the
structure

1.1.1 Introduction
Reinforced concrete, as a composite material, has occupied a special
place in the modern construction of different types of structures due to its several
advantages. Italian architect Ponti once remarked that concrete liberated us from
the rectangle. Due to its flexibility in form and superiority in performance, it has
replaced, to a large extent, the earlier materials like stone, timber and steel.
Further, architect's scope and imaginations have widened to a great extent due
to its mouldability and monolithicity. Thus, it has helped the architects and
engineers to build several attractive shell forms and other curved structures.
However, its role in several straight line structural forms like multistoried frames,
bridges, foundations etc. is enormous.

The design of these modern reinforced concrete structures may appear to


be highly complex. However, most of these structures are the assembly of
several basic structural elements such as beams, columns, slabs, walls and
foundations (Anim. 1.1.1). Accordingly, the designer has to learn the design of
these basic reinforced concrete elements. The joints and connections are then
carefully developed.

Anim. 1.1.1

Design of reinforced concrete structures started in the beginning of last


century following purely empirical approach. Thereafter came the so called
rigorous elastic theory where the levels of stresses in concrete and steel are
limited so that stress-deformations are taken to be linear. However, the limit state
method, though semi-empirical approach, has been found to be the best for the
design of reinforced concrete structures (see sec. 1.1.3.1 also). The constraints
and applicabilities of both the methods will be discussed later.

Version 2 CE IIT, Kharagpur


1.1.2 Objectives of the Design of Reinforced Concrete
Structures
Every structure has got its form, function and aesthetics. Normally, we
consider that the architects will take care of them and the structural engineers will
be solely responsible for the strength and safety of the structure. However, the
roles of architects and structural engineers are very much interactive and a
unified approach of both will only result in an "Integrated" structure, where every
material of the total structure takes part effectively for form, function, aesthetics,
strength as well as safety and durability. This is possible when architects have
some basic understanding of structural design and the structural engineers also
have the basic knowledge of architectural requirements.

Both the engineer and the architect should realize that the skeletal
structure without architecture is barren and mere architecture without the
structural strength and safety is disastrous. Safety, here, includes consideration
of reserve strength, limited deformation and durability. However, some basic
knowledge of architectural and structural requirements would facilitate to
appreciate the possibilities and limitations of exploiting the reinforced concrete
material for the design of innovative structures.

Before proceeding to the design, one should know the objectives of the
design of concrete structures. The objectives of the design are as follows:

1.1.2.1 The structures so designed should have an acceptable probability


of performing satisfactorily during their intended life.

This objective does not include a guarantee that every structure must
perform satisfactorily during its intended life. There are uncertainties in the design
process both in the estimation of the loads likely to be applied on the structure
and in the strength of the material. Moreover, full guarantee would only involve
more cost. Thus, there is an acceptable probability of performance of structures
as given in standard codes of practices of different countries.

1.1.2.2 The designed structure should sustain all loads and deform within
limits for construction and use.

Anim. 1.1.2 Anim. 1.1.3 Anim. 1.1.4

Adequate strengths and limited deformations are the two requirements of


the designed structure. The structure should have sufficient strength and the
deformations must be within prescribed limits due to all loads during construction
and use as seen in Anim. 1.1.2. Animation 1.1.3 shows the structure having
insufficient strength of concrete which fails in bending compression with the
increase of load, though the deformation of the structure is not alarming. On the

Version 2 CE IIT, Kharagpur


other hand, Anim. 1.1.4 shows another situation where the structure, having
sufficient strength, deforms excessively. Both are undesirable during normal
construction and use.

However, sometimes structures are heavily loaded beyond control. The


structural engineer is not responsible to ensure the strength and deformation
within limit under such situation. The staircases in residential buildings during
festival like marriage etc., roof of the structures during flood in the adjoining area
or for buildings near some stadium during cricket or football matches are some of
the examples when structures get overloaded. Though, the structural designer is
not responsible for the strength and deformations under these situations, he,
however, has to ensure that the failure of the structures should give sufficient
time for the occupants to vacate. The structures, thus, should give sufficient
warning to the occupants and must not fail suddenly.

1.1.2.3 The designed structures should be durable.

The materials of reinforced concrete structures get affected by the


environmental conditions. Thus, structures having sufficient strength and
permissible deformations may have lower strength and exhibit excessive
deformations in the long run. The designed structures, therefore, must be
checked for durability. Separate checks for durability are needed for the steel
reinforcement and concrete. This will avoid problems of frequent repairing of the
structure.

1.1.2.4 The designed structures should adequately resist to the effects of


misuse and fire.

Structures may be misused to prepare fire works, store fire works, gas and
other highly inflammable and/or explosive chemicals. Fire may also take place as
accidents or as secondary effects during earthquake by overturning kerosene
stoves or lantern, electrical short circuiting etc. Properly designed structures
should allow sufficient time and safe route for the persons inside to vacate the
structures before they actually collapse.

How to fulfill the objectives?

All the above objectives can be fulfilled by understanding the strength and
deformation characteristics of the materials used in the design as also their
deterioration under hostile exposure. Out of the two basic materials concrete and
steel, the steel is produced in industries. Further, it is available in form of
standard bars and rods of specific diameters. However, sample testing and
checking are important to ensure the quality of these steel bars or rods. The
concrete, on the other hand, is prepared from several materials (cement, sand,

Version 2 CE IIT, Kharagpur


coarse aggregate, water and admixtures, if any). Therefore, it is important to
know the characteristic properties of each of the materials used to prepare
concrete. These materials and the concrete after its preparation are also to be
tested and checked to ensure the quality. The necessary information regarding
the properties and characteristic strength of these materials are available in the
standard codes of practices of different countries. It is necessary to follow these
clearly defined standards for materials, production, workmanship and
maintenance, and the performance of structures in service.

1.1.3 Method of Design


Three methods of design are accepted in cl. 18.2 of IS 456:2000 (Indian
Standard Plain and Reinforced Concrete - Code of Practice, published by the
Bureau of Indian Standards, New Delhi). They are as follows:

1.1.3.1 Limit state method

The term “Limit states” is of continental origin where there are three limit
states - serviceability / crack opening / collapse. For reasons not very clear, in
English literature limit state of collapse is termed as limit state.
As mentioned in sec. 1.1.1, the semi-empirical limit state method of design
has been found to be the best for the design of reinforced concrete members.
More details of this method are explained in Module 3 (Lesson 4). However,
because of its superiority to other two methods (see sections 2.3.2 and 2.3.3 of
Lesson 3), IS 456:2000 has been thoroughly updated in its fourth revision in 2000
taking into consideration the rapid development in the field of concrete
technology and incorporating important aspects like durability etc. This standard
has put greater emphasis to limit state method of design by presenting it in a full
section (section 5), while the working stress method has been given in Annex B
of the same standard. Accordingly, structures or structural elements shall
normally be designed by limit state method.

1.1.3.2 Working stress method

This method of design, considered as the method of earlier times, has


several limitations. However, in situations where limit state method cannot be
conveniently applied, working stress method can be employed as an alternative.
It is expected that in the near future the working stress method will be completely
replaced by the limit state method. Presently, this method is put in Annex B of IS
456:2000.

Version 2 CE IIT, Kharagpur


1.1.3.3 Method based on experimental approach

The designer may perform experimental investigations on models or full


size structures or elements and accordingly design the structures or elements.
However, the four objectives of the structural design (sec. 1.1.2) must be
satisfied when designed by employing this approach. Moreover, the engineer-in-
charge has to approve the experimental details and the analysis connected
therewith.

Though the choice of the method of design is still left to the designer as
per cl. 18.2 of IS 456:2000, the superiority of the limit state method is evident
from the emphasis given to this method by presenting it in a full section (Section
5), while accommodating the working stress method in Annex B of IS 456:2000,
from its earlier place of section 6 in IS 456:1978. It is expected that a gradual
change over to the limit state method of design will take place in the near future
after overcoming the inconveniences of adopting this method in some situations.

1.1.4 Analysis of Structures

Structures when subjected to external loads (actions) have internal


reactions in the form of bending moment, shear force, axial thrust and torsion in
individual members. As a result, the structures develop internal stresses and
undergo deformations. Essentially, we analyse a structure elastically replacing
each member by a line (with EI values) and then design the section using
concepts of limit state of collapse. Figure 1.1.1 explains the internal and external
reactions of a simply supported beam under external loads. The external loads to
be applied on the structures are the design loads and the analyses of structures
are based on linear elastic theory (vide cl. 22 of IS 456:2000).

1.1.5 Design Loads


The design loads are determined separately for the two methods of design
as mentioned below after determining the combination of different loads.

1.1.5.1 In the limit state method, the design load is the characteristic load
with appropriate partial safety factor (vide sec. 2.3.2.3 for partial safety
factors).

Version 2 CE IIT, Kharagpur


1.1.5.2 In the working stress method, the design load is the characteristic
load only.

What is meant by characteristic load?

Characteristic load (cl. 36.2 of IS 456:2000) is that load which has a


ninety-five per cent probability of not being exceeded during the life of the
structure.

Version 2 CE IIT, Kharagpur


The various loads acting on structures consist of dead loads, live loads,
wind or earthquake loads etc. These are discussed in sec. 1.1.6. However, the
researches made so far fail to estimate the actual loads on the structure.
Accordingly, the loads are predicted based on statistical approach, where it is
assumed that the variation of the loads acting on structures follows the normal
distribution (Fig. 1.1.2). Characteristic load should be more than the
average/mean load. Accordingly,

Characteristic load = Average/mean load + K (standard deviation for load)

The value of K is assumed such that the actual load does not exceed the
characteristic load during the life of the structure in 95 per cent of the cases.

1.1.6Loads and Forces


The following are the different types of loads and forces acting on the
structure. As mentioned in sec. 1.1.5, their values have been assumed based on
earlier data and experiences. It is worth mentioning that their assumed values as
stipulated in IS 875 have been used successfully.

1.1.6.1 Dead loads

Anims. 1.1.5 a and

Version 2 CE IIT, Kharagpur


b

These are the self weight of the structure to be designed (see Anim.
1.1.5a). Needless to mention that the dimensions of the cross section are to be
assumed initially which enable to estimate the dead loads from the known unit
weights of the materials of the structure. The accuracy of the estimation thus
depends on the assumed values of the initial dimensions of the cross section.
The values of unit weights of the materials are specified in Part 1 of IS 875.

1.1.6.2 Imposed loads

They are also known as live loads (Anim. 1.1.5a) and consist of all loads
other than the dead loads of the structure. The values of the imposed loads
depend on the functional requirement of the structure. Residential buildings will
have comparatively lower values of the imposed loads than those of school or
office buildings. The standard values are stipulated in Part 2 of IS 875.

1.1.6.3 Wind loads

These loads (Anim. 1.1.5a) depend on the velocity of the wind at the
location of the structure, permeability of the structure, height of the structure etc.
They may be horizontal or inclined forces depending on the angle of inclination of
the roof for pitched roof structures. They can even be suction type of forces
depending on the angle of inclination of the roof or geometry of the buildings
(Anim. 1.1.5b). Wind loads are specified in Part 3 of IS 875.

1.1.6.4 Snow loads

These are important loads for structures located in areas having snow fall,
which gets accumulated in different parts of the structure depending on
projections, height, slope etc. of the structure (Anim. 1.1.6). The standard values
of snow loads are specified in Part 4 of IS 875.

Anim. 1.1.6

1.1.6.5 Earthquake forces

Anim. 1.1.7

Earthquake generates waves which move from the origin of its location
(epicenter) with velocities depending on the intensity and magnitude of the
earthquake. The impact of earthquake on structures depends on the stiffness of
the structure, stiffness of the soil media, height and location of the structure etc.
(Anim. 1.1.7). Accordingly, the country has been divided into several zones
depending on the magnitude of the earthquake. The earthquake forces are

Version 2 CE IIT, Kharagpur


prescribed in IS 1893. Designers have adopted equivalent static load approach
or spectral method.

1.1.6.6 Shrinkage, creep and temperature effects

Anim. 1.1.8, 9 and


10

Shrinkage, creep and temperature (high or low) may produce stresses and
cause deformations like other loads and forces (Anim. 1.1.8, 9 and 10). Hence,
these are also considered as loads which are time dependent. The safety and
serviceability of structures are to be checked following the stipulations of cls.
6.2.4, 5 and 6 of IS 456:2000 and Part 5 of IS 875.

1.1.6.7 Other forces and effects

It is difficult to prepare an exhaustive list of loads, forces and effects


coming onto the structures and affecting the safety and serviceability of them.
However, IS 456:2000 stipulates the following forces and effects to be taken into
account in case they are liable to affect materially the safety and serviceability of
the structures. The relevant codes as mentioned therein are also indicated below:

• Foundation movement (IS 1904) (Fig. 1.1.3)


• Elastic axial shortening

Version 2 CE IIT, Kharagpur


• Soil and fluid pressures (vide IS 875 - Part 5)
• Vibration
• Fatigue
• Impact (vide IS 875 - Part 5)
• Erection loads (Please refer to IS 875 - Part 2) (Fig. 1.1.4)
• Stress concentration effect due to point of application of load and the
like.

1.1.6.8 Combination of loads

Design of structures would have become highly expensive in order to


maintain their serviceability and safety if all types of forces would have acted on
all structures at all times. Accordingly, the concept of characteristic loads has
been accepted to ensure that in at least 95 per cent of the cases, the
characteristic loads considered will be higher than the actual loads on the
structure. However, the characteristic loads are to be calculated on the basis of
average/mean load of some logical combinations of all the loads mentioned in
sec. 1.1.6.1 to 7. These logical combinations are based on (i) the natural
phenomena like wind and earthquake do not occur simultaneously, (ii) live loads
on roof should not be present when wind loads are considered; to name a few. IS
875 Part 5 stipulates the combination of loads to be considered in the design of
structures.

1.1.7 Practice Questions and Problems with Answers


Q.1: Show two reasons why concrete is superior to stone, timber and steel?

A.1: (i) Stone, timber and steel cannot be fitted to any mould, but concrete
during its green stage can fit to any mould.

(ii) Structures made of stone, timber and steel have several joints, but
different elements of concrete structures can be cast monolithically.

Q.2: Define integrated structure.

A.2: Integrated structure is one where each part of the structure satisfies both
the structural requirements of strength and serviceability and architectural
requirements of aesthetics and functionality.

Q.3: State four objectives of the design of reinforced concrete structure.

A.3: Properly designed reinforced concrete structures should:

(i) have acceptable probability of performing satisfactorily during their


intended life,

Version 2 CE IIT, Kharagpur


(ii) sustain all loads with limited deformations during construction and
use,

(iii) be durable,

(iv) adequately resist the effects of misuse and fire.

Q.4: How to fulfil the four objectives of the design of reinforced concrete
structures?

A.4: The four objectives can be fulfilled by:

(i) understanding the strength and deformation characteristics of


concrete and steel,

(ii) following the clearly defined standards for materials, production,


workmanship and maintenance, and use of structures in service,

(iii) adopting measures needed for durability.

Q.5: What are the three methods of design of reinforced concrete structural
elements?

A.5: The three methods are:

(i) limit state method,


(ii) working stress method,
(iii) method based on experimental approach

Q.6: Which of the three methods is the best?

A.6: Limit state method is the best of the three methods when clearly
applicable.

Q.7: What is the basis of the analysis of structures to be designed?

A.7: The basis of the analysis is the employment of linear elastic theory.

Q.8: How to estimate the design loads in (i) limit state method, and (ii) working
stress method?

A.8: (i) In limit state method,

Version 2 CE IIT, Kharagpur


Design loads = Characteristic loads multiplied by the partial safety factor
for loads

(ii) In working stress method,

Design loads = Characteristic loads

Q.9: Define characteristic load.

A.9: Characteristic load is that load which has a ninety-five per cent probability
of not being exceeded during the life of the structure.

Q.10: What are the main (i) loads, (ii) forces and (iii) effects to be considered
while designing the structures?

A.10: (i) The main loads are:

(a) Dead loads


(b) Imposed loads or live loads
(c) Wind loads
(d) Snow loads
(e) Erection loads

(ii) The main force is:

(a) Earthquake force

(iii) The main effects are:

(a) Shrinkage, creep and temperature effects


(b) Foundation movements
(c) Elastic axial shortening
(d) Soil and fluid pressures
(e) Vibration
(f) Fatigue
(g) Impact
(h) Stress concentration effects due to application of point loads

Q.11: What are the basis of combining different loads for the design?

A.11: Natural phenomenon and common sense are the basis of selecting the
combination different loads acting on the structure while designing.

Version 2 CE IIT, Kharagpur


1.1.8 References
1. Reinforced Concrete Limit State Design, 6th Edition, by Ashok K. Jain,
Nem Chand & Bros, Roorkee, 2002.
2. Limit State Design of Reinforced Concrete, 2nd Edition, by P.C.Varghese,
Prentice-Hall of India Pvt. Ltd., New Delhi, 2002.
3. Advanced Reinforced Concrete Design, by P.C.Varghese, Prentice-Hall of
India Pvt. Ltd., New Delhi, 2001.
4. Reinforced Concrete Design, 2nd Edition, by S.Unnikrishna Pillai and
Devdas Menon, Tata McGraw-Hill Publishing Company Limited, New
Delhi, 2003.
5. Limit State Design of Reinforced Concrete Structures, by P.Dayaratnam,
Oxford & I.B.H. Publishing Company Pvt. Ltd., New Delhi, 2004.
6. Reinforced Concrete Design, 1st Revised Edition, by S.N.Sinha, Tata
McGraw-Hill Publishing Company. New Delhi, 1990.
7. Reinforced Concrete, 6th Edition, by S.K.Mallick and A.P.Gupta, Oxford &
IBH Publishing Co. Pvt. Ltd. New Delhi, 1996.
8. Behaviour, Analysis & Design of Reinforced Concrete Structural Elements,
by I.C.Syal and R.K.Ummat, A.H.Wheeler & Co. Ltd., Allahabad, 1989.
9. Reinforced Concrete Structures, 3rd Edition, by I.C.Syal and A.K.Goel,
A.H.Wheeler & Co. Ltd., Allahabad, 1992.
10. Textbook of R.C.C, by G.S.Birdie and J.S.Birdie, Wiley Eastern Limited,
New Delhi, 1993.
11. Design of Concrete Structures, 13th Edition, by Arthur H. Nilson, David
Darwin and Charles W. Dolan, Tata McGraw-Hill Publishing Company
Limited, New Delhi, 2004.
12. Concrete Technology, by A.M.Neville and J.J.Brooks, ELBS with
Longman, 1994.
13. Properties of Concrete, 4th Edition, 1st Indian reprint, by A.M.Neville,
Longman, 2000.
14. Reinforced Concrete Designer’s Handbook, 10th Edition, by C.E.Reynolds
and J.C.Steedman, E & FN SPON, London, 1997.
15. Indian Standard Plain and Reinforced Concrete – Code of Practice (4th
Revision), IS 456: 2000, BIS, New Delhi.
16. Design Aids for Reinforced Concrete to IS: 456 – 1978, BIS, New Delhi.

1.1.9 Test 1 with Solutions


Maximum Marks = 50, Maximum Time = 30 minutes

Answer all questions.

TQ.1: Tick the correct answer: (4 x 7 = 28 marks)

(i) Properly designed concrete structures should

Version 2 CE IIT, Kharagpur


(a) sustain all loads likely to come during next 50 years

(b) sustain all loads and deformations without collapse or any damage

(c) sustain all loads with limited deformations during construction and use

(d) sustain characteristic loads and deformations during the next 50 years

A.TQ.1: (i): (c)

(ii) In the limit state method, the design load is

(a) the characteristic load

(b) the ultimate load

(c) the characteristic load divided by the partial safety factor for loads

(d) the characteristic load multiplied by the partial safety for loads

A.TQ.1: (ii): (d)

(iii) In the limit state method, the basis of the analysis of structure is

(a) linear elastic theory

(b) non-linear theory

(c) plastic method of analysis

(d) involving fracture mechanics

A.TQ.1: (iii): (a)

(iv) The characteristic load is

(a) the load at first crack

(b) that load which has a probability of ninety-five per cent of not being exceeded
during the life of the structure

(c) the ultimate collapse load

(d) the ultimate collapse load multiplied by the partial safety factor for loads

Version 2 CE IIT, Kharagpur


A.TQ.1: (iv): (b)

TQ.2: Name the main (i) loads, (ii) forces and (iii) effects to be considered while
designing the structures.
(5 + 1 + 8 = 14 marks)
(5 marks for five loads, 1 mark for one force and 8 marks for eight effects)

A.TQ.2: See A.10 of sec. 1.1.7.

TQ.3: State the basis of determining the combination of different loads acting on
the structure.
(8 marks)

A.TQ.3: See A.11 of sec. 1.1.7.

1.1.10 Summary of this Lesson


This lesson explains the four objectives of the design of reinforced
concrete structures. It also mentions the three methods of design and states
about the superiority of the limit state method. The basis of the analysis of
structure is discussed. The terminologies like design loads, characteristic loads
etc. are defined and explained. The different loads, forces and effects to be
considered have been listed and discussed. Finally, the basis of the combination
of different loads to be considered is also explained.

Version 2 CE IIT, Kharagpur


Module
1
Objectives and Methods
of Analysis and Design,
and Properties of
Concrete and Steel
Version 2 CE IIT, Kharagpur
Lesson
2
Properties of Concrete
and Steel
Version 2 CE IIT, Kharagpur
Instructional Objectives:
At the end of this lesson, the student should be able to:

• know the properties of concrete in respect of strength, deformation and


durability

• know the properties of steel used as reinforcement in concrete structures

• understand the importance of quality control, inspection and testing of


concrete and steel in several steps from its basic preparation to the removal
of formwork after the construction

• recommend the acceptance of good concrete based on sample test of


specimens, core tests, load test and non-destructive tests

1.2.1 Introduction
It is essential that the designer has to acquire a fair knowledge of the
materials to be used in the design of reinforced concrete structure. This lesson
summarises the characteristic properties of concrete and steel, the two basic
materials used for the design. This summary, though not exhaustive, provides the
minimum information needed for the design.

1.2.2 Properties of Concrete


Plain concrete is prepared by mixing cement, sand (also known as fine
aggregate), gravel (also known as coarse aggregate) and water with specific
proportions. Mineral admixtures may also be added to improve certain properties
of concrete. Thus, the properties of concrete regarding its strength and
deformations depend on the individual properties of cement, sand, gravel, water
and admixtures. Clauses 5 and 6 of IS 456:2000 stipulate the standards and
requirements of the individual material and concrete, respectively. Plain concrete
after preparation and placement needs curing to attain strength. However, plain
concrete is very good in compression but weak in tension. That is why steel is
used as reinforcing material to make the composite sustainable in tension also.
Plain concrete, thus when reinforced with steel bars in appropriate locations is
known as reinforced concrete.

The strength and deformation characteristics of concrete thus depend on


the grade and type of cement, aggregates, admixtures, environmental conditions
and curing. The increase of strength with its age during curing is considered to be
marginal after 28 days. Blended cements (like fly ash cement) have slower rate
of strength gain than ordinary Portland cement as recognized by code,

Version 2 CE IIT, Kharagpur


Depending on several factors during its preparation, placement and curing,
concrete has a wide range of compressive strength and the material is graded on
the basis of its compressive strength on 28th day also known as "characteristic
strength" as defined below while discussing various strength and deformation
properties.

(a) Characteristic strength property

Characteristic strength is defined as the strength below which not more


than five per cent of the test results are expected to fall. Concrete is graded on
the basis of its characteristic compressive strength of 150 mm size cube at 28
days and expressed in N/mm2. The grades are designated by one letter M (for
mix) and a number from 10 to 80 indicating the characteristic compressive
strength (fck) in N/mm2. As per IS 456 (Table 2), concrete has three groups as (i)
ordinary concrete (M 10 to M 20), (ii) standard concrete (M 25 to M 55) and (iii)
high strength concrete (M 60 to M 80). The size of specimen for determining
characteristic strength may be different in different countries.

(b) Other strengths of concrete

In addition to its good compressive strength, concrete has flexural and


splitting tensile strengths too. The flexural and splitting tensile strengths are
obtained as described in IS 516 and IS 5816, respectively. However, the
following expression gives an estimation of flexural strength (fcr) of concrete from
its characteristic compressive strength (cl. 6.2.2)

fcr = 0.7 fc k in N/mm 2


(1.1)

(c) Elastic deformation of concrete

Version 2 CE IIT, Kharagpur


Figure 1.2.1 shows a typical stress-strain curve of concrete in compression,
where

Ec = initial tangent modulus at the origin, also known as short term static
modulus

Es = secant modulus at A

Et = tangent modulus at A

εe = elastic strain at A

εi = inelastic strain at A

It is seen that the initial tangent modulus is much higher than Et (tangent
modulus at A). Near the failure, the actual strain consists of both εe and εi (elastic
and inelastic respectively) components of strain. The initial tangent modulus Ec in
N/mm2 is estimated from

Ec = 5000 fck (1.2)

where fck = characteristic compressive strength of concrete at 28 days

The initial tangent modulus Ec is also known as short term static modulus of
elasticity of concrete in N/mm2 and is used to calculate the elastic deflections.

(d) Shrinkage of concrete

Shrinkage is the time dependent deformation, generally compressive in


nature. The constituents of concrete, size of the member and environmental
conditions are the factors on which the total shrinkage of concrete depends.
However, the total shrinkage of concrete is most influenced by the total amount
of water present in the concrete at the time of mixing for a given humidity and
temperature. The cement content, however, influences the total shrinkage of
concrete to a lesser extent. The approximate value of the total shrinkage strain
for design is taken as 0.0003 in the absence of test data (cl. 6.2.4.1).

Version 2 CE IIT, Kharagpur


(e) Creep of concrete

Creep is another time dependent deformation of concrete by which it


continues to deform, usually under compressive stress. The creep strains recover
partly when the stresses are released. Figure 1.2.2 shows the creep recovery in
two parts. The elastic recovery is immediate and the creep recovery is slow in
nature.

Thus, the long term deflection will be added to the short term deflection to get
the total deflection of the structure. Accordingly, the long term modulus Ece or the
effective modulus of concrete will be needed to include the effect of creep due to
permanent loads. The relationship between Ece and Ec is obtained as follows:

εc = fc/Ec
(1.3)

where εc = short term strain at the age of loading at a stress value of fc

εcr = ultimate creep strain

ε cr
θ = creep coefficient = (cl. 6.2.5.1 of IS 456)
εc
(1.4)

The values of θ on 7th, 28th and 365th day of loading are 2.2, 1.6 and 1.1
respectively.

Version 2 CE IIT, Kharagpur


fc
Then the total strain = εc + εcr =
E ce
(1.5)

where Ece = effective modulus of concrete

From the above Eq. (1.5), we have

fc ε c Ec Ec
Ece = = =
ε c +ε c r ε c +ε c r 1+ θ
(1.6)

The effective modulus of Ece of concrete is used only in the calculation of creep
deflection.

It is seen that the value of creep coefficient θ (Eq. 1.4) is reducing with the
age of concrete at loading. It may also be noted that the ultimate creep strain εcr
does not include short term strain εc. The creep of concrete is influenced by

• Properties of concrete
• Water/cement ratio
• Humidity and temperature of curing
• Humidity during the period of use
• Age of concrete at first loading
• Magnitude of stress and its duration
• Surface-volume ratio of the member

(f) Thermal expansion of concrete

The knowledge of thermal expansion of concrete is very important as it is


prepared and remains in service at a wide range of temperature in different
countries having very hot or cold climates. Moreover, concrete will be having its
effect of high temperature during fire. The coefficient of thermal expansion
depends on the nature of cement, aggregate, cement content, relative humidity
and size of the section. IS 456 stipulates (cl. 6.2.6) the values of coefficient of
thermal expansion for concrete / oC for different types of aggregate.

1.2.3Workability and Durability of Concrete


Workability and durability of concrete are important properties to be
considered. The relevant issues are discussed in the following:

(a) Concrete mix proportioning

Version 2 CE IIT, Kharagpur


The selected mix proportions of cement, aggregates (fine and coarse) and
water ensure:

• the workability of fresh concrete,


• required strength, durability and surface finish when concrete is hardened.

Recently more than forty per cent of concrete poured world over would contain
admixtures.

(b) Workability

It is the property which determines the ease and homogeneity with which
concrete can be mixed, placed, compacted and finished. A workable concrete will
not have any segregation or bleeding. Segregation causes large voids and hence
concrete becomes less durable. Bleeding results in several small pores on the
surface due to excess water coming up. Bleeding also makes concrete less
durable. The degree of workability of concrete is classified from very low to very
high with the corresponding value of slump in mm (cl. 7 of IS 456).

(c) Durability of concrete

A durable concrete performs satisfactorily in the working environment


during its anticipated exposure conditions during service. The durable concrete
should have low permeability with adequate cement content, sufficient low free
water/cement ratio and ensured complete compaction of concrete by adequate
curing. For more information, please refer to cl. 8 of IS 456.

(d) Design mix and nominal mix concrete

In design mix, the proportions of cement, aggregates (sand and gravel),


water and mineral admixtures, if any, are actually designed, while in nominal mix,
the proportions are nominally adopted. The design mix concrete is preferred to
the nominal mix as the former results in the grade of concrete having the
specified workability and characteristic strength (vide cl. 9 of IS 456).

(e) Batching

Mass and volume are the two types of batching for measuring cement,
sand, coarse aggregates, admixtures and water. Coarse aggregates may be
gravel, grade stone chips or other man made aggregates. The quantities of
cement, sand, coarse aggregates and solid admixtures shall be measured by
mass. Liquid admixtures and water are measured either by volume or by mass
(cl. 10 of IS 456).

1.2.4 Properties of Steel


Version 2 CE IIT, Kharagpur
As mentioned earlier in sec. 1.2.2, steel is used as the reinforcing material
in concrete to make it good in tension. Steel as such is good in tension as well as
in compression. Unlike concrete, steel reinforcement rods are produced in steel
plants. Moreover, the reinforcing bars or rods are commercially available in some
specific diameters. Normally, steel bars up to 12 mm in diameter are designated
as bars which can be coiled for transportation. Bars more than 12 mm in
diameter are termed as rods and they are transported in standard lengths.

Like concrete, steel also has several types or grades. The four types of
steel used in concrete structures as specified in cl. 5.6 of IS 456 are given below:

(i) Mild steel and medium tensile steel bars conforming to IS 432 (Part 1)

(ii) High yield strength deformed (HYSD) steel bars conforming to IS 1786

(iii) Hard-drawn steel wire fabric conforming to IS 1566

(iv) Structural steel conforming to Grade A of IS 2062.

Mild steel bars had been progressively replaced by HYSD bars and subsequently
TMT bars are promoted in our country. The implications of adopting different
kinds of blended cement and reinforcing steel should be examined before
adopting.

Stress-strain curves for reinforcement

Version 2 CE IIT, Kharagpur


Figures 1.2.3 and 1.2.4 show the representative stress-strain curves for
steel having definite yield point and not having definite yield point, respectively.
The characteristic yield strength fy of steel is assumed as the minimum yield
stress or 0.2 per cent of proof stress for steel having no definite yield point. The
modulus of elasticity of steel is taken to be 200000 N/mm2.

For mild steel (Fig. 1.2.3), the stress is proportional to the strain up to the
yield point. Thereafter, post yield strain increases faster while the stress is
assumed to remain at constant value of fy.

For cold-worked bars (Fig. 1.2.4), the stress is proportional to the strain up
to a stress of 0.8 fy. Thereafter, the inelastic curve is defined as given below:

Stress Inelastic strain


0.80 fy Nil
0.85 fy 0.0001
0.90 fy 0.0003
0.95 fy 0.0007
0.975 fy 0.0010
1.00 fy 0.0020

Version 2 CE IIT, Kharagpur


Linear interpolation is to be done for intermediate values. The two grades of cold-
worked bars used as steel reinforcement are Fe 415 and Fe 500 with the values
of fy as 415 N/mm2 and 500 N/mm2, respectively.

Considering the material safety factor γm (vide sec. 2.3.2.3 of Lesson 3) of


steel as 1.15, the design yield stress (fyd) of both mild steel and cold worked bars
is computed from

fyd = fy / γm
(1.7)

Accordingly, the representative stress-strain curve for the design is


obtained by substituting fyd for fy in Figs. 1.2.3 and 1.2.4 for the two types of steel
with or without the definite yield point, respectively.

1.2.5 Other Important Factors


The following are some of the important factors to be followed properly as
per the stipulations in IS 456 even for the design mix concrete with materials free
from impurities in order to achieve the desired strength and quality of concrete.
The relevant clause numbers of IS 456 are also mentioned as ready references
for each of the factors.

(a) Mixing (cl. 10.3)

Concrete is mixed in a mechanical mixer at least for two minutes so as to


have uniform distribution of the materials having uniform colour and consistency.

(b) Formwork (cl. 11)

Properly designed formwork shall be used to maintain its rigidity during


placing and compaction of concrete. It should prevent the loss of slurry from the
concrete. The stripping time of formwork should be such that the concrete attains
strength of at least twice the stress that the concrete may be subjected at the
time of removing the formwork. As a ready reference IS 456 specifies the
minimum period before striking formwork.

There is a scope for good design of formwork system so that stripping off
is efficient without undue shock to concrete and facilitating reuse of formwork.

(c) Assembly of reinforcement (cl. 12)

The required reinforcement bars for the bending moment, shear force and
axial thrust are to be accommodated together and proper bar bending schedules
shall be prepared. The reinforcement bars should be placed over blocks,
spacers, supporting bars etc. to maintain their positions so that they have the

Version 2 CE IIT, Kharagpur


required covers. High strength deformed steel bars should not be re-bent. The
reinforcement bars should be assembled to have proper flow of concrete without
obstruction or segregation during placing, compacting and vibrating.

(d) Transporting, placing, compaction and curing (cl. 13)

Concrete should be transported to the formwork immediately after mixing


to avoid segregation, loss of any of the ingredients, mixing of any foreign matter
or loss of workability. Proper protections should be taken to prevent evaporation
loss of water in hot weather and loss of heat in cold weather.

To avoid rehandling, concrete should be deposited very near to the final


position of its placing. The compaction should start before the initial setting time
and should not be disturbed once the initial setting has started. While placing
concrete, reinforcement bars should not be displaced and the formwork should
not be moved.

The compaction of concrete using only mechanical vibrators is very


important, particularly around the reinforcement, embedded fixtures and the
corners of the formwork to prevent honeycomb type of concreting. Excessive
vibration leads to segregation.

Proper curing prevents loss of moisture from the concrete and maintains a
satisfactory temperature regime. In moist curing, the exposed concrete surface is
kept in a damp or wet condition by ponding or covering with a layer of sacking,
canvas, hessian etc. and kept constantly wet for a period of 7-14 days depending
on the type of cement and weather conditions. Blended cement needs extended
curing. In some situations, polyethylene sheets or similar impermeable
membranes may be used to cover the concrete surface closely to prevent
evaporation.

(e) Sampling and strength of designed concrete mix (cl. 15)

Random samples of concrete cubes shall be cast from fresh concrete,


cured and tested at 28 days as laid down in IS 516. Additional tests on beams for
modulus of rupture at 3 or 7 days, or compressive strength tests at 7 days shall
also be conducted. The number of samples would depend on the total quantity of
concrete as given in cl. 15.2.2 and there should be three test specimens in each
sample for testing at 28 days, and additional tests at 3 or 7 days.

(f) Acceptance criteria (cl. 16)

Concrete should be considered satisfactory when both the mean strength


of any group of four consecutive test results and any individual test result of
compressive strength and flexural strength comply the limits prescribed in IS 456.

Version 2 CE IIT, Kharagpur


(g) Inspection and testing of structures (cl. 17)

Inspection of the construction is very important to ensure that it complies


the design. Such inspection should follow a systematic procedure covering
materials, records, workmanship and construction.

All the materials of concrete and reinforcement are to be tested following


the relevant standards. It is important to see that the design and detailing are
capable of execution maintaining a standard with due allowance for the
dimensional tolerances. The quality of the individual parts of the structure should
be verified. If needed, suitable quality assurance schemes should be used. The
concrete should be inspected immediately after the removal of formwork to
remove any defective work before concrete has hardened.

Standard core tests (IS 516) are to be conducted at three or more points
to represent the whole concrete work in case of any doubt regarding the grade of
concrete during inspection either due to poor workmanship or unsatisfactory
results on cube strength obtained following the standard procedure. If the
average equivalent cube strength of cores is equal to at least 85 per cent of the
cube strength of that grade of concrete at that age and each of the individual
cores has strength of at least 75 per cent, then only the concrete represented by
the core test is considered acceptable. For unsatisfactory core test results, load
tests should be conducted for the flexural members and proper analytical
investigations should be made for non-flexural members.

Such load tests should be done as soon as possible after expiry of 28


days from the date of casting of the flexural members subjected to full dead load
and 1.25 times the imposed load for 24 hours and then the imposed load shall be
removed. The maximum deflection of the member during 24 hours under
imposed load in mm should be less than 40 l2/D, where l is the effective span in
m and D is the overall depth of the member in mm. For members showing more
deflection, the recovery of the deflection within 24 hours of removal of the
imposed load has to be noted. If the recovery is less than 75 per cent of the
deflection under imposed load, the test should be repeated after a lapse of 72
hours. The structure is considered unacceptable if the recovery is less than 80
per cent.

There are further provisions of conducting non-destructive tests like


ultrasonic pulse velocity (UPV), rebound hammer, probe penetration, pull out and
maturity, as options to core tests or to supplement the data obtained from a
limited number of cores. However, it is important that the acceptance criteria shall
be agreed upon prior to these non-destructive testing. There are reports that UPV
tests conducted three days after casting after removal of side formwork give very
dependable insight about the quality of concrete.

1.2.6 Concluding Remarks

Version 2 CE IIT, Kharagpur


The reinforced concrete consisting of plain concrete and steel
reinforcement opened a new vista fulfilling the imaginations of architect with a
unified approach of the architect and structural engineer. This has been made
possible due to mouldability and monolithicity of concrete in addition to its
strength in both tension and compression when reinforced with steel. However,
concrete is produced by mixing cement, sand, gravel, water and mineral
admixtures, if needed. Therefore, the final strength of concrete depends not only
on the individual properties of its constituent materials, but also on the
proportions of the material and the manner in which it is prepared, transported,
placed, compacted and cured. Moreover, durability of the concrete is also largely
influenced by all the steps of its preparation.

Steel reinforcement though produced in steel plants and made available in


form of bars and rods of specific diameter also influences the final strength of
reinforced concrete by its quality and durability due to environmental effects.

Concrete cover provides the protective environment to embedded steel


from rusting that would need presence of both oxygen and moisture. Not only the
extent of cover but the quality of cover is important for this reason.

Accordingly, inspection of concrete work, sample testing of specimens,


core tests, load tests and non-destructive tests are very important to maintain the
quality, strength and durability of reinforced concrete structures. Moreover, it is
equally important to remove small defects or make good of it after removing the
formwork before it has thoroughly hardened.

Thus, starting from the selection of each constitutive material to the


satisfactory construction of the structure, the designer's responsibility will only
produce the desired concrete structure which will satisfy the functional
requirements as well as will have its aesthetic values exploiting all the good
properties of this highly potential material.

1.2.7 Practice Questions and Problems with Answers


Q.1: What are the constituent materials of plain concrete?

A.1: The constituent materials of plain concrete are cement, sand (fine
aggregate), gravel (coarse aggregate), water and mineral admixtures in
some special cases.

Q.2: Define characteristic strength fck of concrete.

Version 2 CE IIT, Kharagpur


A.2: Characteristic strength of concrete is defined as the compressive strength
of 150 mm size cube at 28 days and expressed in N/mm2 below which not
more than five per cent of the test results are expected to fall.

Q.3: How and when the characteristic compressive strength fck is determined?

A.3: Characteristic compressive strength is determined by conducting


compressive strength tests on specified number of 150 mm concrete cubes
at 28 days after casting. It is expressed in N/mm2.

Q.4: What do the symbols M and 20 mean for grade M 20 concrete?

A.4: The symbol M refers to mix and the number 20 indicates that the
characteristic strength fck of grade M 20 is 20 N/mm2.

Q.5: Express the relation between flexural strength (fcr) and characteristic
compression strength fck of concrete.

A.5: The generally accepted relation is: fcr = 0.7 f ck where fcr and fck are in
2
N/mm .

Q.6: Draw stress-strain curve of concrete and show the following:

(a) Initial tangent modulus Ec, (b) Secant modulus Es at any point A on the
stress-strain curve, (c) Tangent modulus Et at A and (d) elastic and
inelastic strain components of the total strain at A.

A.6: Please refer to Fig. 1.2.1

Q.7: Express the short term static modulus Ec in terms of the characteristic
compressive strength fck of concrete.

A.7: The suggested expression is : Ec = 5000 f where Ec and fck are in


ck
N/mm2.

Q.8: State the approximate value of total shrinkage strain of concrete to be


taken for the design purpose and mention the relevant clause no. of IS
code.

A.8: As per cl. 6.2.4.1 of IS 456:2000, the approximate value of total shrinkage
strain of concrete is to be taken as 0.0003.

Q.9: Define creep coefficient θ of concrete and express the relation between the
effective modulus Ece, short term static modulus Ec and creep coefficient θ
of concrete.

Version 2 CE IIT, Kharagpur


A.9: Creep coefficient θ is the ratio of ultimate creep strain εcr and short term
strain at the age of loading (θ = εcr/εc).

E
The required relation is Ece = c . {The derivation of Eq. 1.6 is given in
1+ θ
sec. 1.2.2 part (e)}.

Q.10: Define workability of concrete.

A.10: Workability of concrete is the property which determines the ease and
homogeneity with which concrete can be mixed, placed, compacted and
finished.

Q.11: Differentiate between design mix and nominal mix concrete.

A.11: In design mix, the proportions of cement, aggregates (sand and gravel),
water and mineral admixtures, if any are actually determined by actual
design to have a desired strength. In nominal mix, however, these
proportions are nominally adopted.

Q.12: What are the different types of batching in mixing the constituent materials
of concrete and name the type of batching to be adopted for different
materials?

A.12: Mass and volume are the two types of batching. The quantities of cement,
aggregates (sand and gravel) and solid admixtures shall be measured by
mass batching. Liquid admixtures and water are measured either by mass
or volume batching.

Q.13: Differentiate between steel bars and rods.

A.13: Bars are steel bars of diameter up to 12 mm which are coiled during
transportation. Rods are steel bars of diameter greater than 12 mm and
cannot be coiled. They are transported in standard lengths.

Q.14: Name the types of steel and their relevant IS standards to be used as
reinforcement in concrete.

A.14: Please refer to sec. 1.2.4(i), (ii), (iii) and (iv).

Q.15: Draw stress-strain curve of steel bars with or without definite yield point
and indicate the yield stress fy of them.

Version 2 CE IIT, Kharagpur


A.15: Please refer to Figs. 1.2.3 and 1.2.4. For steel bars of Fig. 1.2.3, fy = 250
N/mm2 and for steel bars of Fig. 1.2.4, fy = 415 and 500 N/mm2 for the two
different grades.

Q.16: What are the criteria of properly mixed concrete and how to achieve
them?

A.16: Properly mixed concrete will have uniform distribution of materials having
uniform colour and consistency.

These are achieved by mixing the constituent materials in a mechanical


mixer at least for two minutes or such time till those qualities are achieved.

Q.17: What should be the expected strength of concrete structure at the time of
removal of formwork?

A.17: The concrete at the time of removing the formwork should have strength
of at least twice the stress that it may be subjected to at the time of removal
of formwork.

Q.18: Name the sample tests to be performed for checking the strength of
concrete.

A.18: The main test to be performed is 150 mm cube strength at 28 days made
of fresh concrete and cured. Additional tests should also be conducted on
150 mm cubes at 7 days and beam tests to determine modulus of rupture
at 3 or 7 days. There should be at least 3 or more samples of such
specimens to represent the entire concrete work. Each sample should have
at least three specimens for conducting each of the above-mentioned tests.

Q.19: Mention the specific acceptance criteria of the sample tests of cubes and
beams.

A.19: Concrete should be considered satisfactory when both the mean strength
determined from any group of four consecutive test results and any
individual test result of compressive and flexural strength tests comply the
prescribed limits of cl. 16 of IS 456.

Q.20: When is it essential to conduct standard core test?

A.20: Standard core tests are needed if the inspection of concrete work raises
doubt regarding the grade of concrete either due to poor workmanship or
unsatisfactory cube strength results performed following standard
procedure.

Q.21: When do you consider core test results as satisfactory?

Version 2 CE IIT, Kharagpur


A.21: The core test results are considered satisfactory if:
(i) the average equivalent cube strength of the cores is at least 85 per cent
of the cube strength of the grade of concrete at that age, and
(ii) each of the individual cores has strength of at least 75 per cent of the
concrete cube strength at that age.

Q.22: What are to be done for unsatisfactory core test results?

A.22: Load tests are to be conducted for the flexural members and analytical
investigations are to be performed for non-flexural members.

Q.23: Prescribe the loading conditions and age of structure for conducting load
tests.

A.23: Load tests are to be conducted as soon as possible after expiry of 28 days
from the date of casting. The flexural member is subjected to full dead load
and 1.25 times the imposed load for 24 hours and then the imposed load
has to be removed.

Q.24: When do you conclude the load tests as satisfactory?

A.24: Load tests are considered satisfactory if the maximum deflection in mm of


the member during 24 hours under load is less than 40 l2/D, where l =
effective span in m and D = overall depth of the member in mm.

For members showing more deflection, the recovery of the deflection within
24 hours of removal of the imposed load has to be noted. If the recovery is
less than 75 per cent of the deflection under imposed load, the tests should
be repeated after a lapse of 72 hours. The structure is considered
unacceptable if the recovery is less than 80 per cent.

Q.25: Name the acceptable non-destructive tests to be performed on structures.

A.25: The acceptable non-destructive tests are ultrasonic pulse velocity, rebound
hammer, probe penetration, pull out and maturity.

1.2.8 References
1. Reinforced Concrete Limit State Design, 6th Edition, by Ashok K. Jain,
Nem Chand & Bros, Roorkee, 2002.
2. Limit State Design of Reinforced Concrete, 2nd Edition, by P.C.Varghese,
Prentice-Hall of India Pvt. Ltd., New Delhi, 2002.
3. Advanced Reinforced Concrete Design, by P.C.Varghese, Prentice-Hall of
India Pvt. Ltd., New Delhi, 2001.

Version 2 CE IIT, Kharagpur


4. Reinforced Concrete Design, 2nd Edition, by S.Unnikrishna Pillai and
Devdas Menon, Tata McGraw-Hill Publishing Company Limited, New
Delhi, 2003.
5. Limit State Design of Reinforced Concrete Structures, by P.Dayaratnam,
Oxford & I.B.H. Publishing Company Pvt. Ltd., New Delhi, 2004.
6. Reinforced Concrete Design, 1st Revised Edition, by S.N.Sinha, Tata
McGraw-Hill Publishing Company. New Delhi, 1990.
7. Reinforced Concrete, 6th Edition, by S.K.Mallick and A.P.Gupta, Oxford &
IBH Publishing Co. Pvt. Ltd. New Delhi, 1996.
8. Behaviour, Analysis & Design of Reinforced Concrete Structural Elements,
by I.C.Syal and R.K.Ummat, A.H.Wheeler & Co. Ltd., Allahabad, 1989.
9. Reinforced Concrete Structures, 3rd Edition, by I.C.Syal and A.K.Goel,
A.H.Wheeler & Co. Ltd., Allahabad, 1992.
10. Textbook of R.C.C, by G.S.Birdie and J.S.Birdie, Wiley Eastern Limited,
New Delhi, 1993.
11. Design of Concrete Structures, 13th Edition, by Arthur H. Nilson, David
Darwin and Charles W. Dolan, Tata McGraw-Hill Publishing Company
Limited, New Delhi, 2004.
12. Concrete Technology, by A.M.Neville and J.J.Brooks, ELBS with
Longman, 1994.
13. Properties of Concrete, 4th Edition, 1st Indian reprint, by A.M.Neville,
Longman, 2000.
14. Reinforced Concrete Designer’s Handbook, 10th Edition, by C.E.Reynolds
and J.C.Steedman, E & FN SPON, London, 1997.
15. Indian Standard Plain and Reinforced Concrete – Code of Practice (4th
Revision), IS 456: 2000, BIS, New Delhi.
16. Design Aids for Reinforced Concrete to IS: 456 – 1978, BIS, New Delhi.

1.2.9 Test 2 with Solutions


Maximum Marks = 50, Maximum Time = 30 minutes

Answer all questions. Each question carries five marks.

TQ.1: Define characteristic strength (fck) of concrete

A.TQ.1: See Ans. 2 of sec. 1.2.7.

TQ.2: How and when the characteristic compressive strength (fck) is


determined?

A.TQ.2: See Ans. 3 of sec. 1.2.7.

TQ.3: What do the symbols M and 20 mean for grade M 20 concrete?

Version 2 CE IIT, Kharagpur


A.TQ.3: See Ans. 4 of sec. 1.2.7.

TQ.4: Express the short term static modulus (Ec) in terms of the characteristic
compressive strength (fck) of concrete.

A.TQ.4: See Ans. 7 of sec. 1.2.7.

TQ.5: Define creep coefficient θ of concrete and express the relation between
the effective modulus (Ece), short term static modulus (Ec) and creep
coefficient (θ) of concrete.

A.TQ.5: See Ans. 9 of sec. 1.2.7.

TQ.6: Differentiate between design mix and nominal mix concrete.

A.TQ.6: See Ans. 11 of sec. 1.2.7.

TQ.7: Draw stress-strain curve of steel bars with or without definite yield point
and indicate the yield stress fy of them.

A.TQ.7: See Ans. 15 of sec. 1.2.7.

TQ.8: Mention the specific acceptance criteria of the sample tests of cubes and
beams.

A.TQ.8: See Ans. 19 of sec. 1.2.7.

TQ.9: When do you consider core test results as satisfactory?

A.TQ.9: See Ans. 21 of sec. 1.2.7.

TQ.10: Prescribe the loading conditions and age of structure for conducting load
tests.

A.TQ.10: See Ans. 23 of sec. 1.2.7.

1.2.10 Summary of this Lesson


Properties of concrete and steel are essential to be thoroughly known and
understood by the designer. These properties in respect of strength, deformation
and durability are summarised in this lesson. The importance of quality control,
inspection and testing are emphasised starting from the basic preparation of
concrete to the removal of formwork after the construction. The
recommendations of Indian Standards are discussed regarding the acceptance of

Version 2 CE IIT, Kharagpur


good concrete based on sample tests of specimens, core tests and other non-
destructive tests.

Version 2 CE IIT, Kharagpur


Module
2
Philosophies of Design
by Limit State Method
Version 2 CE IIT, Kharagpur
Lesson
3
Philosophies of Design
by Limit State Method
Version 2 CE IIT, Kharagpur
Instructional Objectives:
At the end of this lesson, the student should be able to:

• categorically state the four common steps to be followed in any method of


design,

• define the limit states,

• identify and differentiate the different limit states,

• state if the structures are to be designed following all the limit states,

• explain the concept of separate partial safety factors for loads and material
strengths depending on the limit state being considered,

• justify the "size effect" of concrete in its strength,

• name the theory for the analysis of structures to be designed by limit states,

• name the method of analysis of statically indeterminate beams and frames,


and slabs spanning in two direction at right angles,

• justify the need to redistribute the moments in statically indeterminate beams


and frames,

• identify four reasons to justify the design of structures or parts of the structure
by limit state method.

2.3.1 Introduction
In any method of design, the following are the common steps to be followed:

(i) To assess the dead loads and other external loads and forces likely to
be applied on the structure,

(ii) To determine the design loads from different combinations of loads,

(iii) To estimate structural responses (bending moment, shear force, axial


thrust etc.) due to the design loads,

(iv) To determine the cross-sectional areas of concrete sections and


amounts of reinforcement needed.

Version 2 CE IIT, Kharagpur


Many of the above steps have lot of uncertainties. Estimation of loads and
evaluation of material properties are to name a few. Hence, some suitable factors
of safety should be taken into consideration depending on the degrees of such
uncertainties.

Limit state method is one of the three methods of design as per IS


456:2000. The code has put more emphasis on this method by presenting it in a
full section (Section 5), while accommodating the working stress method in
Annex B of the code (IS 456). Considering rapid development in concrete
technology and simultaneous development in handling problems of uncertainties,
the limit state method is a superior method where certain aspects of reality can
be explained in a better manner.

2.3.2 Limit State Method


2.3.2.1 What are limit states?

Limit states are the acceptable limits for the safety and serviceability
requirements of the structure before failure occurs. The design of structures by
this method will thus ensure that they will not reach limit states and will not
become unfit for the use for which they are intended. It is worth mentioning that
structures will not just fail or collapse by violating (exceeding) the limit states.
Failure, therefore, implies that clearly defined limit states of structural usefulness
has been exceeded.

Limit state of collapse was found / detailed in several countries in


continent fifty years ago. In 1960 Soviet Code recognized three limit states: (i)
deformation, (ii) cracking and (iii) collapse.

2.3.2.2 How many limit states are there?

Version 2 CE IIT, Kharagpur


There are two main limit states: (i) limit state of collapse and (ii) limit state
of serviceability (see Fig. 2.3.1).

(i) Limit state of collapse deals with the strength and stability of structures
subjected to the maximum design loads out of the possible combinations of
several types of loads. Therefore, this limit state ensures that neither any part nor
the whole structure should collapse or become unstable under any combination
of expected overloads.

(ii) Limit state of serviceability deals with deflection and cracking of


structures under service loads, durability under working environment during their
anticipated exposure conditions during service, stability of structures as a whole,
fire resistance etc.

All relevant limit states have to be considered in the design to ensure


adequate degree of safety and serviceability. The structure shall be designed on
the basis of the most critical limit state and shall be checked for other limit states
(see Fig. 2.3.2).

Version 2 CE IIT, Kharagpur


2.3.2.3 Partial safety factors

The characteristic values of loads as discussed in sec. 1.1.5 are based on


statistical data. It is assumed that in ninety-five per cent cases the characteristic
loads will not be exceeded during the life of the structures (Fig. 2.3.3). However,
structures are subjected to overloading also. Hence, structures should be
designed with loads obtained by multiplying the characteristic loads with suitable
factors of safety depending on the nature of loads or their combinations, and the
limit state being considered. These factors of safety for loads are termed as
partial safety factors (γf) for loads. Thus, the design loads are calculated as

(Design load Fd) = (Characteristic load F) (Partial safety factor for load γf)
(2.1)

Respective values of γf for loads in the two limit states as given in Table 18 of IS
456 for different combinations of loads are furnished in Table 2.1.

Version 2 CE IIT, Kharagpur


Table 2.1 Values of partial safety factor γf for loads

Load combinations Limit state of collapse Limit state of


serviceability
(for short term effects
only)

DL IL WL DL IL WL
DL + IL 1.5 1.0 1.0 1.0 -
DL + WL 1.5 or - 1.5 1.0 - 1.0
0.91)
DL + IL + WL 1.2 1.0 0.8 0.8
NOTES:

1 While considering earthquake effects, substitute EL for WL.

2 For the limit states of serviceability, the values of γf given in this table are applicable for
short term effects. While assessing the long term effects due to creep the dead load and
that part of the live load likely to be permanent may only be considered.
1)
This value is to be considered when stability against overturning or stress reversal is
critical.

Version 2 CE IIT, Kharagpur


Similarly, the characteristic strength of a material as obtained from the
statistical approach is the strength of that material below which not more than five
per cent of the test results are expected to fall (see Fig. 2.3.4). However, such
characteristic strengths may differ from sample to sample also. Accordingly, the
design strength is calculated dividing the characteristic strength further by the
partial safety factor for the material (γm), where γm depends on the material and
the limit state being considered. Thus,

Characteristic strength of the material f


Design strength of the material f =
d Partial safety factor of the material γ
m
(2.2)

Both the partial safety factors are shown schematically in Fig. 2.3.5.

Clause 36.4.2 of IS 456 states that γm for concrete and steel should be
taken as 1.5 and 1.15, respectively when assessing the strength of the structures
or structural members employing limit state of collapse. However, when
assessing the deflection, the material properties such as modulus of elasticity
should be taken as those associated with the characteristic strength of the

Version 2 CE IIT, Kharagpur


material. It is worth mentioning that partial safety factor for steel (1.15) is
comparatively lower than that of concrete (1.5) because the steel for
reinforcement is produced in steel plants and commercially available in specific
diameters with expected better quality control than that of concrete.

Further, in case of concrete the characteristic strength is calculated on the


basis of test results on 150 mm standard cubes. But the concrete in the structure
has different sizes. To take the size effect into account, it is assumed that the
concrete in the structure develops a strength of 0.67 times the characteristic
strength of cubes. Accordingly, in the calculation of strength employing the limit
state of collapse, the characteristic strength (fck) is first multiplied with 0.67 (size
effect) and then divided by 1.5 (γm for concrete) to have 0.446 fck as the
maximum strength of concrete in the stress block.

2.3.3 Analysis
Analysis of structure has been briefly mentioned in sec. 1.1.4 earlier.
Herein, the analysis of structure, in the two limit states (of collapse and of
serviceability), is taken up. In the limit state of collapse, the strength and stability
of the structure or part of the structure are ensured. The resistances to bending
moment, shear force, axial thrust, torsional moment at every section shall not be
less than their appropriate values at that section due to the probable most
unfavourable combination of the design loads on the structure. Further, the
structure or part of the structure should be assessed for rupture of one or more
critical sections and buckling due to elastic or plastic instability considering the
effects of sway, if it occurs or overturning.

Linear elastic theory is recommended in cl. 22 of IS 456 to analyse the


entire structural system subjected to design loads. The code further stipulates the
adoption of simplified analyses for frames (cl. 22.4) and for continuous beams (cl.
22.5). For both the limit states the material strengths should be taken as the
characteristic values in determining the elastic properties of members. It is worth
mentioning that the statically indeterminate structures subjected to design loads
will have plastic hinges at certain locations as the loads increase beyond the
characteristic loads. On further increase of loads, bending moments do not
increase in the locations of plastic hinges as they are already at the full capacities
of bending moments. However, these plastic hinges undergo more rotations and
the moments are now received by other sections which are less stressed. This
phenomenon continues till the plastic hinges reach their full rotation capacities to
form a mechanism of collapse of the structure. This is known as the redistribution
of moments (Figs. 2.3.6 and 2.3.7). The theory and numerical problems of
“Redistribution of moments” are presented elaborately in Lesson 38.

Version 2 CE IIT, Kharagpur


Version 2 CE IIT, Kharagpur
The design of structure, therefore, should also ensure that the less
stressed sections can absorb further moments with a view to enabling the
structure to rotate till their full capacities. This will give sufficient warming to the
users before the structures collapse. Accordingly, there is a need to redistribute
moments in continuous beams and frames. Clause 37.1.1 stipulates this
provision and the designer has to carry out the redistribution by satisfying the
stipulated conditions there.

The analysis of slabs spanning in two directions at right angles should be


performed by employing yield line theory or any other acceptable method. IS
456:2000 has illustrated alternative provisions for the simply supported and
restrained slabs spanning in two directions in Annex D along with Tables 26 and
27 giving bending moment coefficients of these slabs for different possible
boundary conditions. These provisions enable to determine the reinforcement

Version 2 CE IIT, Kharagpur


needed for bending moments in two directions and torsional reinforcement
wherever needed.

2.3.4 Concluding Remarks


The limit state method is based on a stochastic process where the design
parameters are determined from observations taken over a period of time. The
concept of separate partial safety factors for loads and material strengths are
based on statistical and probabilistic grounds. These partial safety factors for the
material strengths are determined on the basis of reliability of preparations of
concrete and reinforcement. The overloading of structure has been kept in mind
while specifying the partial safety factors of loads.

The stress block of structures or parts of structure designed on the basis


of limit state method subjected to the designed loads or collapse loads
represents the stress-strain diagram at the defined states of collapse and satisfy
the requirements of strength and stability. Simultaneous checking of these
structures or parts of them for the limit state of serviceability ensures the
deflection and cracking to remain within their limits. Thus, this method is more
rational and scientific.

2.3.5 Practice Questions and Problems with Answers


Q.1: List the common steps of design of structures by any method of design.

A.1: The four steps are listed in section 2.3.1 under (i) to (iv).

Q.2: What are limit states?

A.2: See sec. 2.3.2.1

Q.3: How many limit states are there? Should a structure be designed following
all the limit states?

A.3: See sec. 2.3.2.2

Q.4: Define partial safety factors of load and material. Write the expressions to
determine the design load and design strength of the material from their
respective characteristic values employing the corresponding partial safety
factors.

A.4: See sec. 2.3.2.3 and Eqs.2.1 and 2.2

Q.5: What is size effect of concrete? What is its role in determining the material
strength of concrete?

Version 2 CE IIT, Kharagpur


A.5: The characteristic strength of concrete is determined from the results of
tests conducted on cube specimens of 150 mm dimension. The
dimensions of concrete structures or in members of structure are
different and widely varying. This has an effect on the strength of concrete
in the structure. This is known as size effect.

Due to the size effect, the characteristic strength of concrete is reduced to


2/3 of its value and then further divided by the partial safety factor of the
concrete (γm = 1.5) to get the design strength of concrete (fd). Thus,

0.67 f
( 2/ 3 ) ck
f = (f ) = = 0.446 f
d ck 1.5 1.5 ck

Q.6: Which theory should be employed for the analysis of structural system to
be designed element wise, by limit state method?

A.6: Linear elastic theory should be employed for the analysis of structural
system subjected to design loads.

Q.7: Justify the need to do the redistribution of moments in statically


indeterminate structures.

A.7: Statically indeterminate structures will have plastic hinges formed when
loads increase from the characteristic values. These locations where
plastic hinges are formed will undergo rotations at constant moment when
the sections of lower stresses will receive the additional moments due to
further increase of loads. This process will continue till sufficient plastic
hinges are formed to have a mechanism of collapse (see Figs. 2.3.6 and
2.3.7).

The structures should have such a provision to avoid sudden failure at the
failure of one critical section. The comparatively lower stressed sections,
therefore, should be designed taking the redistribution of moments into
account.

Q.8: What are the analytical methods for the design of simply supported and
restrained slabs?

A.8: Yield line theory or any other acceptable method of analysis can be
employed for these slabs. Alternatively, the method illustrated in Annex D
of IS 456:2000 can also be used for the slabs spanning in two
perpendicular directions.

Q.9: Give four reasons to justify the design of structures by limit state method.

Version 2 CE IIT, Kharagpur


A.9: The four reasons are:

(i) Concept of separate partial safety factors of loads of different


combinations in the two limit state methods.

(ii) Concept of separate partial safety factors of materials depending on


their quality control during preparation. Thus, γm for concrete is 1.5
and the same for steel is 1.15. This is more logical than one
arbitrary value in the name of safety factor.

(iii) A structure designed by employing limit state method of collapse


and checked for other limit states will ensure the strength and
stability requirements at the collapse under the design loads and
also deflection and cracking at the limit state of serviceability. This
will help to achieve the structure with acceptable probabilities that
the structure will not become unfit for the use for which it is
intended.

(iv) The stress block represents in a more realistic manner when the
structure is at the collapsing stage (limit state of collapse) subjected
to design loads.

2.3.6 References
1. Reinforced Concrete Limit State Design, 6th Edition, by Ashok K. Jain,
Nem Chand & Bros, Roorkee, 2002.
2. Limit State Design of Reinforced Concrete, 2nd Edition, by P.C.Varghese,
Prentice-Hall of India Pvt. Ltd., New Delhi, 2002.
3. Advanced Reinforced Concrete Design, by P.C.Varghese, Prentice-Hall of
India Pvt. Ltd., New Delhi, 2001.
4. Reinforced Concrete Design, 2nd Edition, by S.Unnikrishna Pillai and
Devdas Menon, Tata McGraw-Hill Publishing Company Limited, New
Delhi, 2003.
5. Limit State Design of Reinforced Concrete Structures, by P.Dayaratnam,
Oxford & I.B.H. Publishing Company Pvt. Ltd., New Delhi, 2004.
6. Reinforced Concrete Design, 1st Revised Edition, by S.N.Sinha, Tata
McGraw-Hill Publishing Company. New Delhi, 1990.
7. Reinforced Concrete, 6th Edition, by S.K.Mallick and A.P.Gupta, Oxford &
IBH Publishing Co. Pvt. Ltd. New Delhi, 1996.
8. Behaviour, Analysis & Design of Reinforced Concrete Structural Elements,
by I.C.Syal and R.K.Ummat, A.H.Wheeler & Co. Ltd., Allahabad, 1989.
9. Reinforced Concrete Structures, 3rd Edition, by I.C.Syal and A.K.Goel,
A.H.Wheeler & Co. Ltd., Allahabad, 1992.
10. Textbook of R.C.C, by G.S.Birdie and J.S.Birdie, Wiley Eastern Limited,
New Delhi, 1993.

Version 2 CE IIT, Kharagpur


11. Design of Concrete Structures, 13th Edition, by Arthur H. Nilson, David
Darwin and Charles W. Dolan, Tata McGraw-Hill Publishing Company
Limited, New Delhi, 2004.
12. Concrete Technology, by A.M.Neville and J.J.Brooks, ELBS with
Longman, 1994.
13. Properties of Concrete, 4th Edition, 1st Indian reprint, by A.M.Neville,
Longman, 2000.
14. Reinforced Concrete Designer’s Handbook, 10th Edition, by C.E.Reynolds
and J.C.Steedman, E & FN SPON, London, 1997.
15. Indian Standard Plain and Reinforced Concrete – Code of Practice (4th
Revision), IS 456: 2000, BIS, New Delhi.
16. Design Aids for Reinforced Concrete to IS: 456 – 1978, BIS, New Delhi.

2.3.7 Test 3 with Solutions


Maximum Marks = 50, Maximum Time = 30 minutes

Answer all questions.

TQ.1: List the common steps of design of structures by any method of design.
(8 marks)

A.TQ.1: See Ans. 1 of sec. 2.3.5.

TQ.2: What are limit states?


(6 marks)

A.TQ.2: See Ans. 2 of sec. 2.3.5.

TQ.3: Draw schematic figures to explain (i) the different limit states and (ii) use
of the limit states to design a structure. (6 +
6 = 12 marks)
A.TQ.3: The schematic figures are Figs. 2.3.1 and 2.3.2 and answers are in sec.
2.3.2.2

TQ.4: What is size effect of concrete? What is its role in determining the material
strength of concrete?
(6 marks)

A.TQ.4: See Ans. 5 of sec. 2.3.5.

TQ.5: Justify the need to do the redistribution of moments in statically


indeterminate structures.
(10 marks)

A.TQ.5: See Ans. 7 of sec. 2.3.5.

Version 2 CE IIT, Kharagpur


TQ.6: Give four reasons to justify the design of structures by limit state method.
(8 marks)

A.TQ.6: See Ans. 9 of sec. 2.3.5.

2.3.8 Summary of this Lesson


This lesson defines and identifies different limit states. It explains the
concept and defines the partial safety factors for loads and material strengths
depending on the limit states to be considered in the design. It explains the
theory for the analysis of structures to be carried out by the designer. Further, the
redistribution of moments in statically indeterminate beams and frames is
explained. Finally, the use of limit state method for the design of reinforced
concrete structures has been justified.

Version 2 CE IIT, Kharagpur


Module
3
Limit State of Collapse -
Flexure (Theories and
Examples)

Version 2 CE IIT, Kharagpur


Lesson
4
Computation of
Parameters of
Governing Equations

Version 2 CE IIT, Kharagpur


Instructional Objectives:
At the end of this lesson, the student should be able to:

• identify the primary load carrying mechanisms of reinforced concrete beams


and slabs,

• name three different types of reinforced concrete beam with their specific
applications,

• identify the parameters influencing the effective widths of T and L-beams,

• differentiate between one-way and two-way slabs,

• state and explain the significance of six assumptions of the design,

• draw the stress-strain diagrams across the depth of a cross-section of


rectangular beam,

• write the three equations of equilibrium,

• write and derive the expressions of total compression and tension forces C
and T, respectively.

3.4.1 Introduction

Version 2 CE IIT, Kharagpur


Reinforced concrete beams and slabs carry loads primarily by bending
(Figs. 3.4.1 to 3). They are, therefore, designed on the basis of limit state of
collapse in flexure. The beams are also to be checked for other limit states of
shear and torsion. Slabs under normal design loadings (except in bridge decks
etc.) need not be provided with shear reinforcement. However, adequate
torsional reinforcement must be provided wherever needed.

Version 2 CE IIT, Kharagpur


Version 2 CE IIT, Kharagpur
This lesson explains the basic governing equations and the computation of
parameters required for the design of beams and one-way slabs employing limit
state of collapse in flexure. There are three types of reinforced concrete beams:

(i) Singly or doubly reinforced rectangular beams (Figs. 3.4.4 to 7)


(ii) Singly or doubly reinforced T-beams (Figs. 3.4.8 to 11)
(iii) Singly or doubly reinforced L-beams (Figs. 3.4.12 to 15)

Version 2 CE IIT, Kharagpur


During construction of reinforced concrete structures, concrete slabs and beams
are cast monolithic making the beams a part of the floor deck system. While
bending under positive moments near midspan, bending compression stresses at
the top are taken by the rectangular section of the beams above the neutral axis
and the slabs, if present in T or L-beams (Figs. 3.4.4, 5, 8, 9, 12 and 13).
However, under the negative moment over the support or elsewhere, the bending
compression stresses are at the bottom and the rectangular sections of
rectangular, T and L-beams below the neutral axis only resist that compression
(Figs. 3.4.6, 7, 10, 11, 14 and 15). Thus, in a slab-beam system the beam will be
Version 2 CE IIT, Kharagpur
considered as rectangular for the negative moment and T for the positive
moment. While for the intermediate spans of slabs the beam under positive
moment is considered as T, the end span edge beam is considered as L-beam if
the slab is not projected on both the sides of the beam. It is worth mentioning that
the effective width of flange of these T or L-beams is to be determined which
depends on:

Version 2 CE IIT, Kharagpur


Version 2 CE IIT, Kharagpur
(a) if it is an isolated or continuous beam
(b) the distance between points of zero moments in the beam
(c) the width of the web
(d) the thickness of the flange

Reinforced concrete slabs are classified as one-way or two-way


depending on if they are spanning in one or two directions (Figs. 3.4.16 and 17).
As a guideline, slabs whose ratio of longer span (ly) to the shorter span (lx) is
more than two are considered as one-way slabs. One-way slabs also can be
designed following the procedure of the design of beams of rectangular cross-
section. Again, slabs may be isolated or continuous also.

Version 2 CE IIT, Kharagpur


3.4.2 Assumptions

Version 2 CE IIT, Kharagpur


The following are the assumptions of the design of flexural members (Figs.
3.4.18 to 20) employing limit state of collapse:

(i) Plane sections normal to the axis remain plane after bending.

This assumption ensures that the cross-section of the member does not
warp due to the loads applied. It further means that the strain at any point on the
cross-section is directly proportional to its distance from the neutral axis.

(ii) The maximum strain in concrete at the outer most compression fibre is
taken as 0.0035 in bending (Figs. 3.4.19 and 20).

This is a clearly defined limiting strain of concrete in bending compression


beyond which the concrete will be taken as reaching the state of collapse. It is
very clear that the specified limiting strain of 0.0035 does not depend on the
strength of concrete.

(iii) The acceptable stress-strain curve of concrete is assumed to be parabolic


as shown in Fig. 1.2.1 of Lesson 2.

The maximum compressive stress-strain curve in the structure is obtained


by reducing the values of the top parabolic curve (Figs. 21 of IS 456:2000) in two
stages. First, dividing by 1.5 due to size effect and secondly, again dividing by
1.5 considering the partial safety factor of the material. The middle and bottom
curves (Fig. 21 of IS 456:2000) represent these stages. Thus, the maximum
compressive stress in bending is limited to the constant value of 0.446 fck for the
strain ranging from 0.002 to 0.0035 (Figs. 3.4.19 and 20, Figs. 21 and 22 of IS
456:2000).

Version 2 CE IIT, Kharagpur


(iv) The tensile strength of concrete is ignored.

Concrete has some tensile strength (very small but not zero). Yet, this
tensile strength is ignored and the steel reinforcement is assumed to resist the
tensile stress. However, the tensile strength of concrete is taken into account to
check the deflection and crack widths in the limit state of serviceability.

(v) The design stresses of the reinforcement are derived from the
representative stress-strain curves as shown in Figs. 1.2.3 and 4 of Lesson 2 and
Figs. 23A and B of IS 456:2000, for the type of steel used using the partial safety
factor γm as 1.15.

In the reinforced concrete structures, two types of steel are used: one with
definite yield point (mild steel, Figs. 1.2.3 of Lesson 2 and Figs. 23B of IS
456:2000) and the other where the yield points are not definite (cold work
deformed bars). The representative stress-strain diagram (Fig. 1.2.4 of Lesson 2
and Fig. 23A of IS 456:2000) defines the points between 0.8 fy and 1.0 fy in case
of cold work deformed bars where the curve is inelastic.

(vi) The maximum strain in the tension reinforcement in the section at failure
shall not be less than fy/(1.15 Es) + 0.002, where fy is the characteristic strength
of steel and Es = modulus of elasticity of steel (Figs. 3.4.19 and 20).

This assumption ensures ductile failure in which the tensile reinforcement


undergoes a certain degree of inelastic deformation before concrete fails in
compression.

3.4.3 Singly Reinforce Rectangular Beams


Figure 3.4.18 shows the singly reinforced rectangular beam in flexure. The
following notations are used (Figs. 3.4.19 and 20):

Ast = area of tension steel


b = width of the beam
C = total compressive force of concrete
d = effective depth of the beam
L = centre to centre distance between supports
P = two constant loads acting at a distance of L/3 from the two supports
of the beam
T = total tensile force of steel
xu = depth of neutral axis from the top compression fibre

3.4.4 Equations of Equilibrium


The cross-sections of the beam under the applied loads as shown in Fig.
3.4.18 has three types of combinations of shear forces and bending moments: (i)
Version 2 CE IIT, Kharagpur
only shear force is there at the support and bending moment is zero, (ii) both
bending moment (increasing gradually) and shear force (constant = P) are there
between the support and the loading point and (iii) a constant moment (= PL/3) is
there in the middle third zone i.e. between the two loads where the shear force is
zero (Fig. 1.1.1 of Lesson 1). Since the beam is in static equilibrium, any cross-
section of the beam is also in static equilibrium. Considering the cross-section in
the middle zone (Fig. 3.4.18) the three equations of equilibrium are the following
(Figs. 3.4.19 and 20):

(i) Equilibrium of horizontal forces: Σ H = 0 gives T = C


(3.1)

(ii) Equilibrium of vertical shear forces: Σ V = 0


(3.2)

This equation gives an identity 0 = 0 as there is no shear in the middle


third zone of the beam.

(iii) Equilibrium of moments: Σ M = 0,


(3.3)

This equation shows that the applied moment at the section is fully resisted by
moment of the resisting couple T a = C a , where a is the operating lever arm
between T and C (Figs. 3.4.19 and 20).

3.4.5 Computations of C and T

Version 2 CE IIT, Kharagpur


Figures 3.4.21a and b present the enlarged view of the compressive part
of the strain and stress diagrams. The convex parabolic part of the stress block of
Fig. 3.4.21b is made rectangular by dotted lines to facilitate the calculations
adding another concave parabolic stress zone which is really non-existent as
marked by hatch in Fig. 3.4.21b.

The different compressive forces C, C1, C2 and C3 and distances x1 to x5


and xu as marked in Fig. 3.4.21b are explained in the following:

C = Total compressive force of concrete = C1 + C2

C1 = Compressive force of concrete due to the constant stress of 0.446


fck and up to a depth of x3 from the top fibre

C2 = Compressive force of concrete due to the convex parabolic stress


block of values ranging from zero at the neutral axis to 0.446 fck at a
distance of x3 from the top fibre

C3 = Compressive force of concrete due to the concave parabolic stress


block (actually non-existent) of values ranging from 0.446 fck at the
neutral axis to zero at a distance of x3 from the top fibre

x1 = Distance of the line of action of C1 from the top compressive fibre

x2 = Distance of the line of action of C (= C1 + C2) from the top


compressive fibre

x3 = Distance of the fibre from the top compressive fibre, where the strain
= 0.002 and stress = 0.446 fck

x4 = Distance of the line of action of C2 from the top compressive fibre

x5 = Distance of the line of action of C3 from the top compressive fibre

xu = Distance of the neutral axis from the top compressive fibre.

From the strain triangle of Fig. 3.4.21a, we have

xu − x3 0.002 4
= = = 0.57 , giving
xu 0.0035 7

x3 = 0.43 xu
(3.4)

Since C1 is due to the constant stress acting from the top to a distance of x3, the
distance x1 of the line of action of C1 is:

Version 2 CE IIT, Kharagpur


x1 = 0.5 x3 = 0.215 xu
(3.5)

From Fig. 3.4.21a:

3
x5 = x3 + (xu - x3) = 0.43 xu + 0.75(0.57 xu)
4

or x5 = 0.86 xu
(3.6)

The compressive force C1 due to the rectangular stress block is:

C1 = b x3(0.446 fck) = 0.191 b xu fck


(3.7)

The compressive force C2 due to parabolic stress block is:

2
C2 = b (xu - x3) (0.446 fck) = 0.17 b xu fck
3
(3.8)

Adding C1 and C2, we have

C = C1 + C2 = 0.361 b xu fck = 0.36 b xu fck (say)


(3.9)

The non-existent compressive force C3 due to parabolic (concave) stress block


is:

1
C3 = b (xu - x3) (0.446 fck) = 0.085 b xu fck
3
(3.10)

Now, we can get x4 by taking moment of C2 and C3 about the top fibre as follows:

xu − x3
C2(x4) + C3 (x5) = (C2 + C3) (x3 + )
2

which gives x4 = 0.64 xu


(3.11)

Similarly, x2 is obtained by taking moment of C1 and C2 about the top fibre as


follows:

C1(x1) + C2(x4) = C(x2)

which gives x2 = 0.4153 xu


Version 2 CE IIT, Kharagpur
or x2 = 0.42 xu (say).
(3.12)

Thus, the required parameters of the stress block (Fig. 3.4.19) are

C = 0.36 b xu fck (3.9)

x2 = 0.42 xu (3.12)

and lever arm = (d - x2) = (d - 0.42 xu)


(3.13)

The tensile force T is obtained by multiplying the design stress of steel with the
area of steel. Thus,

fy
T = ( ) Ast = 0.87 f y Ast
1.15
(3.14)

3.4.6 Practice Questions and Problems with Answers


Q.1: How do the beams and slabs primarily carry the transverse loads ?

A.1: The beams and slabs carry the transverse loads primarily by bending.

Q.2: Name three different types of reinforced concrete beams and their specific
applications.

A.2: They are:

(i) Singly reinforced and doubly reinforced rectangular beams - used in


resisting negative moments in intermediate spans of continuous
beams over the supports or elsewhere in slab-beam monolithic
constructions, and positive moments in midspan of isolated or
intermediate spans of beams with inverted slab (monolithic)
constructions and lintels.

(ii) Singly reinforced and doubly reinforced T-beams - used in resisting


positive moments in isolated or intermediate spans (midspan) in
slab-beam monolithic constructions and negative moments over the
support for continuous spans with inverted slab (monolithic)
constructions.

Version 2 CE IIT, Kharagpur


(iii) Singly reinforced and doubly reinforced L-beams - Same as (ii)
above except that these are for end spans instead of intermediate
spans.

Q.3: Name four parameters which determine the effective widths of T and L-
beams.

A.3: The four parameters are:

(i) isolated or continuous beams,


(ii) the distance between points of zero moments in the beam,
(iii) the breadth of the web,
(iv) the thickness of the flange.

Q.4: Differentiate between one-way and two-way slabs.

A.4: One-way slab spans in one direction and two-way slab spans in both the
directions. Slabs whose ratio of longer span (ly) to shorter span (lx) is more
than 2 are called one-way. Slabs of this ratio up to 2 are called two-way
slabs.

Q.5: State and explain the significance of the six assumptions of design of
flexural members employing limit state of collapse.

A.5: Sec. 3.4.2 gives the full answer.

Q.6: Draw a cross-section of singly reinforced rectangular beam and show the
strain and stress diagrams.

A.6: Fig. 3.4.19.

Q.7: Write the three equations of equilibrium needed to design the reinforced
concrete beams.

A.7: Vide sec. 3.4.4 and Eqs. 3.1 to 3.

Q.8: Write the final expression of the total compressive force C and tensile
force T for a rectangular reinforced concrete beam in terms of the
designing parameters.

A.8: Eq. 3.9 for C and Eq. 3.14 for T.

Version 2 CE IIT, Kharagpur


3.4.7 References
1. Reinforced Concrete Limit State Design, 6th Edition, by Ashok K. Jain,
Nem Chand & Bros, Roorkee, 2002.
2. Limit State Design of Reinforced Concrete, 2nd Edition, by P.C.Varghese,
Prentice-Hall of India Pvt. Ltd., New Delhi, 2002.
3. Advanced Reinforced Concrete Design, by P.C.Varghese, Prentice-Hall of
India Pvt. Ltd., New Delhi, 2001.
4. Reinforced Concrete Design, 2nd Edition, by S.Unnikrishna Pillai and
Devdas Menon, Tata McGraw-Hill Publishing Company Limited, New
Delhi, 2003.
5. Limit State Design of Reinforced Concrete Structures, by P.Dayaratnam,
Oxford & I.B.H. Publishing Company Pvt. Ltd., New Delhi, 2004.
6. Reinforced Concrete Design, 1st Revised Edition, by S.N.Sinha, Tata
McGraw-Hill Publishing Company. New Delhi, 1990.
7. Reinforced Concrete, 6th Edition, by S.K.Mallick and A.P.Gupta, Oxford &
IBH Publishing Co. Pvt. Ltd. New Delhi, 1996.
8. Behaviour, Analysis & Design of Reinforced Concrete Structural Elements,
by I.C.Syal and R.K.Ummat, A.H.Wheeler & Co. Ltd., Allahabad, 1989.
9. Reinforced Concrete Structures, 3rd Edition, by I.C.Syal and A.K.Goel,
A.H.Wheeler & Co. Ltd., Allahabad, 1992.
10. Textbook of R.C.C, by G.S.Birdie and J.S.Birdie, Wiley Eastern Limited,
New Delhi, 1993.
11. Design of Concrete Structures, 13th Edition, by Arthur H. Nilson, David
Darwin and Charles W. Dolan, Tata McGraw-Hill Publishing Company
Limited, New Delhi, 2004.
12. Concrete Technology, by A.M.Neville and J.J.Brooks, ELBS with
Longman, 1994.
13. Properties of Concrete, 4th Edition, 1st Indian reprint, by A.M.Neville,
Longman, 2000.
14. Reinforced Concrete Designer’s Handbook, 10th Edition, by C.E.Reynolds
and J.C.Steedman, E & FN SPON, London, 1997.
15. Indian Standard Plain and Reinforced Concrete – Code of Practice (4th
Revision), IS 456: 2000, BIS, New Delhi.
16. Design Aids for Reinforced Concrete to IS: 456 – 1978, BIS, New Delhi.

3.4.8 Test 4 with Solutions


Maximum Marks = 50, Maximum Time = 30 minutes

Answer all questions.

TQ.1: Tick the correct answer: (4 x 5 = 20


marks)

Version 2 CE IIT, Kharagpur


(i) Beams and slabs carry the transverse loads primarily by

(a) truss action


(b) balance of shear action
(c) bending
(d) slab-beam interaction

A.TQ.1: (i): (c)

(ii) The ratio of longer span (ly) to shorter span (lx) of a two-way slab is

(a) up to 2
(b) more than 2
(c) equal to 1
(d) more than 1

A.TQ.1: (ii): (a)

(iii) An inverted T-beam is considered as a rectangular beam for the design

(a) over the intermediate support of a continuous beam where the bending
moment is negative

(b) at the midspan of a continuous beam where the bending moment is


positive

(c) at the point of zero bending moment

(d) over the support of a simply supported beam

A.TQ.1: (iii): (b)

(iv) The maximum strain in the tension reinforcement in the section at failure
shall be

(a) more than fy /(1.15 Es) + 0.002

(b) equal to 0.0035

(c) more than fy /Es + 0.002

(d) less than fy /(1.15 Es) + 0.002

A.TQ.1: (iv): (d)

Version 2 CE IIT, Kharagpur


TQ.2: Draw a cross-section of singly reinforced rectangular beam and show the
strain and stress diagrams.
(10)

A.TQ.2: Fig. 3.4.19

TQ.3: Name four parameters which determine the effective widths of T and L-
beams. (6)

A.TQ.3: The four parameters are:

(i) isolated or continuous beams,


(ii) the distance between points of zero moments in the beam,
(iii) the breadth of the web,
(iv) the thickness of the flange.

TQ.4: Derive the final expressions of the total compressive force C and tensile
force T for a rectangular reinforced concrete beam in terms of the
designing parameters.
(10 +
4 = 14)
A.TQ.4: Section 3.4.5 is the full answer.

3.4.9 Summary of this Lesson


Lesson 4 illustrates the primary load carrying principle in a slab-beam
structural system subjected to transverse loadings. It also mentions three
different types of singly and doubly reinforced beams normally used in
construction. Various assumptions made in the design of these beams employing
limit state of collapse are explained. The stress and strain diagrams of a singly
reinforced rectangular beam are explained to write down the three equations of
equilibrium. Finally, the computations of the total compressive and tensile forces
are illustrated.

Version 2 CE IIT, Kharagpur


Module
3
Limit State of Collapse -
Flexure (Theories and
Examples)

Version 2 CE IIT, Kharagpur


Lesson
5
Determination of
Neutral Axis Depth and
Computation of Moment
of Resistance
Version 2 CE IIT, Kharagpur
Instructional Objectives:
At the end of this lesson, the student should be able to:

• determine the depth of the neutral axis for a given cross-section with known
value of Ast and grades of steel and concrete,

• write and derive the expressions of xu,max, pt,lim, Mu,lim/bd2 and state the
influences of grades of steel and concrete on them separately,

• derive the corresponding expression of Mu when xu < xu,max, xu = xu,max and xu


> xu,max,

• finally take a decision if xu > xu,max.

3.5.1 Introduction
After learning the basic assumptions, the three equations of equilibrium
and the computations of the total compressive and tensile forces in Lesson 4, it is
now required to determine the depth of neutral axis (NA) and then to estimate the
moment of resistance of the beams. These two are determined using the two
equations of equilibrium (Eqs. 3.1 and 3.3). It has been explained that the depth
of neutral axis has important role to estimate the moment of resistance.
Accordingly, three different cases are illustrated in this lesson.

3.5.2 Computation of the Depth of Neutral Axis xu

From Eqs. 3.1, 9 and 14, we have

0.87 fy Ast = 0.36 b xu fck


(3.15)

0.87 f y Ast
or xu =
0.36 b f ck
(3.16)

We can also write:

xu 0.87 f y Ast
=
d 0.36 b d f ck
(3.17)

Version 2 CE IIT, Kharagpur


xu
Substituting the expression of from Eq. 3.17 into Eq. 3.13 of Lesson 4, we
d
have

⎧⎪ ⎛ 0.87 f y Ast ⎞⎫⎪


lever arm = d ⎨1 − 0.42 ⎜ ⎟⎬
⎪⎩ ⎜ 0.36 f b d ⎟⎪
⎝ ck ⎠⎭

⎛ Ast f y ⎞
= d ⎜⎜1 − 1.015 ⎟

⎝ f ck b d ⎠
(3.18)

Ignoring multiplying factor 1.015 in Eq. 3.18, we have

⎛ Ast f y ⎞
lever arm = d ⎜⎜1 − ⎟
⎝ f ck b d ⎟⎠

(3.19)

3.5.3 Limiting Value of xu (= xu, max)


Should there be a limiting or maximum value of xu?

xu
Equation 3.17 reveals that increases with the increase of percentage
d
Ast
of steel reinforcement for fixed values of fy and fck. Thus, the depth of the
bd
neutral axis xu will tend to reach the depth of the tensile steel. But, that should
not be allowed. However, let us first find out that value of xu which will satisfy
assumptions (ii) and (vi) of sec. 3.4.2 and designate that by xu, max for the
present, till we confirm that xu should have a limiting value.

Version 2 C IIT, Kharagpur


Figure 3.5.1 presents the strain diagrams for the three cases: (i) when xu = xu, max
; (ii) when xu is less than xu, max and (iii) when xu is greater than xu, max . The
following discussion for the three cases has the reference to Fig. 3.5.1.

(i) When xu = xu, max

The compressive strain at the top concrete fibre = 0.0035 and the tensile
⎛ 0.87 f y ⎞
strain at the level of steel = ⎜ + 0.002 ⎟ . Thus, it satisfies the assumptions (ii)
⎜ E ⎟
⎝ s ⎠
and (vi) of sec. 3.4.2.

(ii) When xu is less than xu, max

There are two possibilities here:

Version 2 CE IIT, Kharagpur


(a) If the compressive strain at the top fibre = 0.0035, the tensile strain is
⎛ 0.87 f y ⎞
more than ⎜ + 0.002 ⎟ . Thus, this possibility satisfies the two assumptions (ii)
⎜ E ⎟
⎝ s ⎠
and (vi) of sec. 3.4.2.

⎛ 0.87 f y ⎞
(b) When the steel tensile strain is ⎜ + 0.002 ⎟ , the compressive
⎜ E ⎟
⎝ s ⎠
concrete strain is less than 0.0035. Here also, both the assumptions (ii) and (vi)
of sec. 3.4.2 are satisfied.

(iii) When xu is more than xu, max

There are two possibilities here:

(a) When the top compressive strain reaches 0.0035, the tensile steel
⎛ 0.87 f y ⎞
strain is less than ⎜ + 0.002 ⎟⎟ . It violets the assumption (vi) though
⎜ E
⎝ s ⎠
assumption (ii) of sec. 3.4.2 is satisfied.

⎛ 0.87 f y ⎞
(b) When the steel tensile strain is ⎜ + 0.002 ⎟ , the compressive strain
⎜ E ⎟
⎝ s ⎠
of concrete exceeds 0.0035. Thus, this possibility violets assumption (ii) though
assumption (vi) is satisfied.

The above discussion clearly indicates that the depth of xu should not
become more than xu, max. Therefore, the depth of the neutral axis has a limiting
or maximum value = xu, max. Accordingly, if the Ast provided yields xu > xu, max, the
section has to be redesigned.

Since xu depends on the area of steel, we can calculate Ast, lim from Eq.
3.17.

From Eq. 3.17 (using xu = xu, max and Ast = Ast, lim), we have

xu ,max 0.87 f y Ast ,lim


=
d 0.36 b d f ck

Ast ,lim ⎛ 0.36 f ck xu ,max ⎞


or (100) = pt,lim = ⎜ ⎟ (100) (3.20)
bd ⎜ 0.87 f d ⎟
⎝ y ⎠

xu ,max
In the above equation can be obtained from the strain diagram of Fig.
d
3.5.1 as follows:

Version 2 CE IIT, Kharagpur


xu ,max 0.0035
=
d 0.87 f y
+ 0.0055
Es
(3.21)

xu ,max
3.5.4 Values of d
and pt, lim

Equation 3.20 shows that the values of pt, lim depend on both the grades
x
of steel and concrete, while Eq. 3.21 reveals that u ,max depends on the grade of
d
steel alone and not on the grade of concrete at all. The respective values of pt, lim
for the three grades of steel and the three grades of concrete are presented in
x
Table 3.1. Similarly, the respective values of u ,max for three grades of steel are
d
presented in Table 3.2.

Table 3.1 Values of pt, lim

fck in N/mm2 fy = 250 N/mm2 fy = 415 N/mm2 fy = 500 N/mm2

20 1.76 0.96 0.76


25 2.20 1.19 0.94
30 2.64 1.43 1.13

xu ,max
Table 3.2 Values of
d
2
fy in N/mm 250 415 500

xu ,max 0.531 = 0.53 (say) 0.479 = 0.48 (say) 0.456 = 0.46 (say)
d

A careful study of Tables 3.1 and 3.2 reveals the following:

(i) The pt, lim increases with lowering the grade of steel for a particular
grade of concrete. The pt, lim, however, increases with increasing the grade of
concrete for a specific grade of steel.

(ii) The maximum depth of the neutral axis xu,max increases with lowering
the grade of steel. That is more area of the section will be utilized in taking the
compression with lower grade of steel.

3.5.5 Computation of Mu
Equation 3.3 of Lesson 4 explains that Mu can be obtained by multiplying
the tensile force T or the compressive force C with the lever arm. The
expressions of C, lever arm and T are given in Eqs. 3.9, 3.13 (also 3.19) and

Version 2 CE IIT, Kharagpur


3.14 respectively of Lesson 4. Section 3.5.3 discusses that there are three
possible cases depending on the location of xu. The corresponding expressions
of Mu are given below for the three cases:

(i) When xu < xu, max

Figure 3.5.1 shows that when concrete reaches 0.0035, steel has started
0.87 f y
flowing showing ductility (Strain > + 0.002 ). So, the computation of Mu is to
Es
be done using the tensile force of steel in this case. Therefore, using Eqs. 3.13
and 3.14 of Lesson 4, we have

Mu = T (lever arm) = 0.87 fy Ast (d - 0.42 xu)


(3.22)

Substituting the expressions of T and lever arm from Eqs. 3.14 of Lesson
4 and 3.19 respectively we get,

⎛ Ast f y ⎞
Mu = 0.87 f y Ast d ⎜⎜1 − ⎟
⎝ f ck b d ⎟⎠

(3.23)

(ii) When xu = xu, max

0.87 f y
From Fig. 3.5.1, it is seen that steel just reaches the value of + 0.002
Es
and concrete also reaches its maximum value. The strain of steel can further
increase but the reaching of limiting strain of concrete should be taken into
consideration to determine the limiting Mu as Mu, lim here. So, we have

Mu, lim = C (lever arm)

Substituting the expressions of C and lever arm from Eqs. 3.9 of Lesson 4 and
3.19 respectively, we have

xu ,max ⎛ xu ,max ⎞
M u ,lim = 0.36 ⎜1 − 0.42 ⎟ b d 2 f ck
d ⎜ d ⎟
⎝ ⎠
(3.24)

(iii) When xu > xu, max

In this case, it is seen from Fig. 3.5.1 that when concrete reaches the
0.87 f y
strain of 0.0035, tensile strain of steel is much less than ( + 0.002 ) and any
Es
further increase of strain of steel will mean failure of concrete, which is to be

Version 2 CE IIT, Kharagpur


0.87 f y
avoided. On the other hand, when steel reaches + 0.002 , the strain of
Es
concrete far exceeds 0.0035. Hence, it is not possible. Therefore, such design is
avoided and the section should be redesigned.

However, in case of any existing reinforced concrete beam where xu > xu,
max, the moment of resistance Mu for such existing beam is calculated by
restricting xu to xu, max only and the corresponding Mu will be as per the case when
xu = xu, max.

3.5.6 Computation of Limiting Moment of Resistance Factor


Equation 3.24 shows that a particular rectangular beam of given
dimensions of b and d has a limiting capacity of Mu, lim for a specified grade of
concrete. The limiting moment of resistance factor R,lim (= Mu,lim/bd2) can be
established from Eq. 3.24 as follows:

M u ,lim xu ,max ⎛ xu ,max ⎞


R, lim = = 0.36 ⎜1 − 0.42 ⎟ f ck
2 d ⎜ d ⎟
bd ⎝ ⎠
(3.25)

It is seen that the limiting moment of resistance factor R,lim depends on


xu ,max x
and fck. Since u ,max depends on the grade of steel fy, we can say that
d d
R,lim depends on fck and fy. Table 3.3 furnishes the values of R,lim for three grades
of concrete and three grades of steel.

M u ,lim
Table 3.3 Limiting values of R,lim = 2
factors (in N/mm2)
bd
2 2
fck in N/mm fy = 250 N/mm fy = 415 N/mm2 fy = 500 N/mm2

20 2.98 2.76 2.66


25 3.73 3.45 3.33
30 4.47 4.14 3.99

A study of Table 3.3 reveals that the limiting moment of resistance factor
R,lim increases with higher grade of concrete for a particular grade of steel. It is
also seen that this factor increases with lowering the grade of steel for a
particular grade of concrete. The increase of this factor due to higher grade of
concrete is understandable. However, such increase of the factor with lowering
the grade of steel is explained below:

Version 2 CE IIT, Kharagpur


xu ,max
Lowering the grade of steel increases the (vide Table 3.2) and this
d
xu ,max
enhanced increases Mu as seen from Eq. 3.24. However, one may argue
d
xu ,max ⎛ xu ,max ⎞
that Eq. 3.24 has two terms: and ⎜1 − 0.42 ⎟ .
d ⎜ d ⎟⎠

⎛ xu ,max ⎞
, ⎜⎜1 − 0.42 ⎟ is decreasing. Then how
xu ,max
With the increase of
d
⎝ d ⎟⎠
xu ,max
do we confirm that the product is increasing with the increase of ? Actual
d
computation will reveal the fact. Otherwise, it can be further explained from Table
3.1 that as the grade of steel is lowered for a particular grade of concrete, the pt,
lim gets increased. Therefore, amount of steel needed to have Mu, lim with lower
x
grade of steel is higher. Thus, higher amount of steel and higher values of u ,max
d
M u ,lim
show higher factor with the lowering of grade of steel for a particular grade
b d2
of concrete (see Table 3.3).

3.5.7 Practice Questions and Problems with Answers


Q.1: Which equation is needed to determine the depth of the neutral axis?

A.1: Eq. 3.16 or 3.17.

Q.2: How to find the lever arm?

A.2: Eq. 3.13 of Lesson 4 or Eq. 3.19.

Q.3: Should there be limiting or maximum value of up? If so, why? What is the
equation to find the maximum value of xu? How to find Ast, lim for such
case?

A.3: Sec. 3.5.3 is the complete answer.

Q.4: State the effects of grades of concrete and steel separately on pt, lim and
xu ,max
.
d

A.4: (i) pt, lim (see Eq. 3.20 and Table 3.1)

(a) pt, lim increases with lowering the grade of steel for a particular grade of
concrete

Version 2 CE IIT, Kharagpur


(b) pt, lim increases with increasing the grade of concrete for a particular
grade of steel

xu ,max
(ii) (see Eq. 3.21 and Table 3.2)
d

xu ,max
(a) increases with lowering the grade of steel
d

xu ,max
(b) is independent on the grade of concrete.
d

Q.5: Write the corresponding expression of Mu when (i) xu < xu, max; and (ii) xu =
xu, max

A.5: (i) Eq. 3.23, when xu < xu, max

(ii) Eq. 3.24, when xu = xu, max

Q.6: What is to be done if xu > xu, max and why ?

A.6: When xu > xu, max, the section has to be redesigned as this does not
ensure ductile failure of the beam.

M u ,lim
Q.7: Write the expression of limiting moment of resistance factor R,lim = .
b d2
What is its unit?

M u ,lim xu ,max ⎛ xu ,max ⎞


A.7: R, lim = = 0.36 ⎜1 − 0.42 ⎟ f ck (Eq. 3.25). Its unit is
b d2 d ⎜ d ⎟
⎝ ⎠
N/mm2.

Q.8: State separately the effects of grades of concrete and steel on the limiting
M
moment of resistance factor R,lim = u ,lim
2
.
bd

A.8: (i) R,lim increases with increasing the grade of concrete for a particular
grade of steel (see Table 3.3).

(ii) R,lim increases with lowering the grade of steel for a particular grade of
concrete (see Table 3.3).

Version 2 CE IIT, Kharagpur


3.5.8 References
1. Reinforced Concrete Limit State Design, 6th Edition, by Ashok K. Jain,
Nem Chand & Bros, Roorkee, 2002.
2. Limit State Design of Reinforced Concrete, 2nd Edition, by P.C.Varghese,
Prentice-Hall of India Pvt. Ltd., New Delhi, 2002.
3. Advanced Reinforced Concrete Design, by P.C.Varghese, Prentice-Hall of
India Pvt. Ltd., New Delhi, 2001.
4. Reinforced Concrete Design, 2nd Edition, by S.Unnikrishna Pillai and
Devdas Menon, Tata McGraw-Hill Publishing Company Limited, New
Delhi, 2003.
5. Limit State Design of Reinforced Concrete Structures, by P.Dayaratnam,
Oxford & I.B.H. Publishing Company Pvt. Ltd., New Delhi, 2004.
6. Reinforced Concrete Design, 1st Revised Edition, by S.N.Sinha, Tata
McGraw-Hill Publishing Company. New Delhi, 1990.
7. Reinforced Concrete, 6th Edition, by S.K.Mallick and A.P.Gupta, Oxford &
IBH Publishing Co. Pvt. Ltd. New Delhi, 1996.
8. Behaviour, Analysis & Design of Reinforced Concrete Structural Elements,
by I.C.Syal and R.K.Ummat, A.H.Wheeler & Co. Ltd., Allahabad, 1989.
9. Reinforced Concrete Structures, 3rd Edition, by I.C.Syal and A.K.Goel,
A.H.Wheeler & Co. Ltd., Allahabad, 1992.
10. Textbook of R.C.C, by G.S.Birdie and J.S.Birdie, Wiley Eastern Limited,
New Delhi, 1993.
11. Design of Concrete Structures, 13th Edition, by Arthur H. Nilson, David
Darwin and Charles W. Dolan, Tata McGraw-Hill Publishing Company
Limited, New Delhi, 2004.
12. Concrete Technology, by A.M.Neville and J.J.Brooks, ELBS with
Longman, 1994.
13. Properties of Concrete, 4th Edition, 1st Indian reprint, by A.M.Neville,
Longman, 2000.
14. Reinforced Concrete Designer’s Handbook, 10th Edition, by C.E.Reynolds
and J.C.Steedman, E & FN SPON, London, 1997.
15. Indian Standard Plain and Reinforced Concrete – Code of Practice (4th
Revision), IS 456: 2000, BIS, New Delhi.
16. Design Aids for Reinforced Concrete to IS: 456 – 1978, BIS, New Delhi.

3.5.9 Test 5 with Solutions


Maximum Marks = 50, Maximum Time = 30 minutes

Answer all questions.

TQ.1: Tick the correct answer: (4 x 5 =


20)
Version 2 CE IIT, Kharagpur
(i) The depth of the neutral axis is calculated from the known area of steel and it
should be

(a) less than 0.5 times the full depth of the beam

(b) more than 0.5 times the effective depth of the beam

(c) less than or equal to limiting value of the neutral axis depth

(d) less than 0.43 times the effective depth of the beam

A.TQ.1: (i): (c)

(ii) For a particular grade of concrete and with lowering the grade of steel, the
pt,lim

(a) increases

(b) decreases

(c) sometimes increases and sometimes decreases

(d) remains constant

A.TQ.1: (ii): (a)

(iii) For a particular grade of steel and with increasing the grade of concrete, the
pt,lim

(a) decreases

(b) increases

(c) remains constant

(d) sometimes increases and sometimes decreases

A.TQ.1: (iii): (b)

(iv) Which of the statements is correct?

(a) xu,max /d is independent of grades of concrete and steel

(b) xu,max /d is independent of grade of steel but changes with grade of


steel

Version 2 CE IIT, Kharagpur


(c) xu,max /d changes with the grade of concrete and steel

(d) xu,max /d is independent of the grade of concrete and changes with


the grade of steel

A.TQ.1: (iv): (d)

TQ.2: Derive the expression of determining the depth of neutral axis and lever
arm of a singly reinforced rectangular beam with known quantity of tension
steel. (10)

A.TQ.2: Section 3.5.2 is the complete answer.

TQ.3: Establish the expressions of the moment of resistance of a singly


reinforced rectangular beam when (i) xu < xu,max , (ii) xu = xu,max and (iii)
xu > xu,max.

(8+7=15)

A.TQ.3: Section 3.5.5 is the complete answer.

TQ.4: Derive the expression of limiting moment of resistance factor and explain
how it is influenced by the grades of concrete and steel.
(5)

A.TQ.4: Section 3.5.6 is the full answer.

3.5.10 Summary of this Lesson


Understanding the various assumptions of the design, stress-strain
diagrams of concrete and steel, computations of the total compressive and
tensile forces and the equations of equilibrium in Lesson 4, this lesson illustrates
the applications of the two equations of equilibrium. Accordingly, the depth of
neutral axis and the moment of resistance of the beam can be computed with the
expressions derived in this lesson. Different cases that arise due to different
values of the depth of neutral axis are discussed to select the particular
expression of the moment of resistance. Further, the expressions of determining
the limiting values of percentage of steel, depth of neutral axis, moment of
resistance and moment of resistance factor are established. The influences of
grades of concrete and steel on them are also illustrated.

Version 2 CE IIT, Kharagpur


Module
3
Limit State of Collapse -
Flexure (Theories and
Examples)

Version 2 CE IIT, Kharagpur


Lesson
6
Numerical Problems on
Singly Reinforced
Rectangular Beams
Version 2 CE IIT, Kharagpur
Instructional Objectives:
At the end of this lesson, the student should be able to:

• identify the main two types of problems of singly reinforced rectangular


sections,

• name the inputs and outputs of the two types of problems,

• state the specific guidelines of assuming the breadth, depths, area of steel
reinforcement, diameter of the bars, grade of concrete and grade of steel,

• determine the depth of the neutral axis for specific dimensions of beam
(breadth and depth) and amount of reinforcement,

• identify the beam with known dimensions and area of steel if it is under-
reinforced or over-reinforced,

• apply the principles to design a beam.

3.6.1 Types of Problems


Two types of problems are possible: (i) design type and (ii) analysis type.
In the design type of problems, the designer has to determine the dimensions b,
d, D, Ast (Fig. 3.6.1) and other detailing of reinforcement, grades of concrete and
steel from the given design moment of the beam. In the analysis type of the
problems, all the above data will be known and the designer has to find out the
moment of resistance of the beam. Both the types of problems are taken up for
illustration in the following two lessons.

Version 2 CE IIT, Kharagpur


3.6.2 Design Type of Problems
The designer has to make preliminary plan lay out including location of the
beam, its span and spacing, estimate the imposed and other loads from the given
functional requirement of the structure. The dead loads of the beam are
estimated assuming the dimensions b and d initially. The bending moment, shear
force and axial thrust are determined after estimating the different loads. In this
illustrative problem, let us assume that the imposed and other loads are given.
Therefore, the problem is such that the designer has to start with some initial
dimensions and subsequently revise them, if needed. The following guidelines
are helpful to assume the design parameters initially.

3.6.2.1 Selection of breadth of the beam b

Normally, the breadth of the beam b is governed by: (i) proper housing of
reinforcing bars and (ii) architectural considerations. It is desirable that the width
of the beam should be less than or equal to the width of its supporting structure
like column width, or width of the wall etc. Practical aspects should also be kept
in mind. It has been found that most of the requirements are satisfied with b as
150, 200, 230, 250 and 300 mm. Again, width to overall depth ratio is normally
kept between 0.5 and 0.67.

3.6.2.2 Selection of depths of the beam d and D

The effective depth has the major role to play in satisfying (i) the strength
requirements of bending moment and shear force, and (ii) deflection of the beam.
The initial effective depth of the beam, however, is assumed to satisfy the
deflection requirement depending on the span and type of the reinforcement. IS
456 stipulates the basic ratios of span to effective depth of beams for span up to
10 m as (Clause 23.2.1)

Cantilever 7
Simply supported 20
Continuous 26

For spans above 10 m, the above values may be multiplied with 10/span
in metres, except for cantilevers where the deflection calculations should be
made. Further, these ratios are to be multiplied with the modification factor
depending on reinforcement percentage and type. Figures 4 and 5 of IS 456 give
the different values of modification factors. The total depth D can be determined
by adding 40 to 80 mm to the effective depth.

3.6.2.3 Selection of the amount of steel reinforcement Ast

The amount of steel reinforcement should provide the required tensile


force T to resist the factored moment Mu of the beam. Further, it should satisfy

Version 2 CE IIT, Kharagpur


the minimum and maximum percentages of reinforcement requirements also.
The minimum reinforcement As is provided for creep, shrinkage, thermal and
other environmental requirements irrespective of the strength requirement. The
minimum reinforcement As to be provided in a beam depends on the fy of steel
and it follows the relation: (cl. 26.5.1.1a of IS 456)

As 0.85
=
bd fy
(3.26)

The maximum tension reinforcement should not exceed 0.04 bD (cl. 26.5.1.1b of
IS 456), where D is the total depth.

Besides satisfying the minimum and maximum reinforcement, the amount


of reinforcement of the singly reinforced beam should normally be 75 to 80% of
0.87 f y
pt, lim. This will ensure that strain in steel will be more than ( + 0.002) as
Es
the design stress in steel will be 0.87 fy. Moreover, in many cases, the depth
required for deflection becomes more than the limiting depth required to resist Mu,
lim. Thus, it is almost obligatory to provide more depth. Providing more depth also
helps in the amount of the steel which is less than that required for Mu, lim. This
helps to ensure ductile failure. Such beams are designated as under-reinforced
beams.

3.6.2.4 Selection of diameters of bar of tension reinforcement

Reinforcement bars are available in different diameters such as 6, 8, 10,


12, 14, 16, 18, 20, 22, 25, 28, 30, 32, 36 and 40 mm. Some of these bars are
less available. The selection of the diameter of bars depends on its availability,
minimum stiffness to resist while persons walk over them during construction,
bond requirement etc. Normally, the diameters of main tensile bars are chosen
from 12, 16, 20, 22, 25 and 32 mm.

3.6.2.5 Selection of grade of concrete

Besides strength and deflection, durability is a major factor to decide on


the grade of concrete. Table 5 of IS 456 recommends M 20 as the minimum
grade under mild environmental exposure and other grades of concrete under
different environmental exposures also.

3.6.2.6 Selection of grade of steel

Normally, Fe 250, 415 and 500 are in used in reinforced concrete work.
Mild steel (Fe 250) is more ductile and is preferred for structures in earthquake
zones or where there are possibilities of vibration, impact, blast etc.

Version 2 CE IIT, Kharagpur


3.6.3Design Problem 3.1

Design a simply supported reinforced concrete rectangular beam (Fig.


3.6.2) whose centre to centre distance between supports is 8 m and supported
on brick walls of 300 mm thickness. The beam is subjected to imposed loads of
7.0 kN/m.

3.6.4 Solution by Direct Computation Method


The unknowns are b, d, D, Ast, grade of steel and grade of concrete. It is
worth mentioning that these parameters have to satisfy different requirements
and they also are interrelated. Accordingly, some of them are to be assumed
which subsequently may need revision.

3.6.4.1 Grades of steel and concrete

Let us assume Fe 415 and M 20 are the grades of steel and concrete
respectively. As per clause 6.1.2 and Table 5 of IS 456, minimum grade of
concrete is M 20 for reinforced concrete under mild exposure (durability
requirement).

3.6.4.2 Effective span Leff

Clause 22.2(a) of IS 456 recommends that the effective span is the lower
of (i) clear span plus effective depth and (ii) centre to centre distance between
two supports. Here, the clear span is 7700 mm. Thus

Version 2 CE IIT, Kharagpur


(i) Clear span + d = 7700 + 400 (assuming d = 400 from the specified ratio of
span to effective depth as 20 and mentioned in
the next section)

(ii) Centre to centre distance between two supports = 8000 mm.

Hence, Leff = 8000 mm

3.6.4.3 Percentage of steel reinforcement pt

The percentage of steel reinforcement to be provided is needed to


determine the modification factor which is required to calculate d. As mentioned
earlier in sec. 3.6.2.3, it is normally kept at 75 to 80 per cent of pt, lim. Here, pt,
lim = 0.96 (vide Table 3.1 of Lesson 5). So, percentage of steel to be provided is
assumed = 0.75 (0.96) = 0.72.

3.6.4.4 Effective depth d

As per clause 23.2.1 of IS 456, the basic value of span to effective depth
ratio here is 20. Further, Fig. 4 of IS 456 presents the modification factor which
will be multiplied with the basic span to effective depth ratio. This modification
factor is determined on the value of fs where

Area of cross-section of steel required


f s = 0.58 f y
Area of cross-section of steel provided

= 0.58 fy (assuming that the Ast provided is the same as Ast required)

= 0.58 (415) = 240.7 N/mm2.

From Fig. 4 of IS 456, the required modification factor is found to be 1.1 for fs =
240.7 N/mm2 and percentage of steel = 0.72. So, the span to effective depth ratio
= 22 as obtained by multiplying 20 with 1.1. Accordingly, the effective depth =
8000/22 = 363.63 mm, say 365 mm. Since this value of d is different from the d
assumed at the beginning, let us check the effective span as lower of (i) 7700 +
365 and (ii) 8000 mm. Thus, the effective span remains at 8000 mm. Adding 50
mm with the effective depth of 365 mm (assuming 50 mm for cover etc.), the
total depth is assumed to be 365 + 50 = 415 mm.

3.6.4.5 Breadth of the beam b

Let us assume b = 250 mm to get b/D = 250/415 = 0.6024, which is


acceptable as the ratio of b/D is in between 0.5 and 0.67.

3.6.4.6 Dead loads, total design loads Fd and bending moment

Version 2 CE IIT, Kharagpur


With the unit weight of reinforced concrete as 25 kN/m3 (cl. 19.2.1 of IS
456):

Dead load of the beam = 0.25 (0.415) (25) kN/m = 2.59 kN/m

Imposed loads = 7.00 kN/m

Thus, total load = 9.59 kN/m, which gives factored load Fd as 9.59 (1.5)
(partial safety factor for dead load and imposed load as 1.5) = 14.385 kN/m. We
have, therefore, Mu = Factored bending moment = 14.385 (8) = 115.08 kNm.

3.6.4.7 Checking of effective depth d

It is desirable to design the beam as under-reinforced so that the ductility


is ensured with steel stress reaching the design value. Let us now determine the
limiting effective depth when xu = xu, max and the factored moment Mu = Mu, lim =
115.08 kNm from Eq. 3.24 of Lesson 5.

xu, max ⎧ xu, max ⎫


M u, lim = 0.36 ⎨1 − 0.42
2
⎬ b d f ck
d ⎩ d ⎭
(3.24)

xu, max
Table 3.2 of Lesson 5 gives = 0.479 for fy = 415 N/mm2. Thus:
d

(115.08) 106 Nmm = 0.36(0.479) [1 - 0.42(0.479)] b d2 (20)

which gives d = 408.76 mm

So, let us revise d = 410 mm from the earlier value of 365 mm to have the total
depth = 410 + 50 = 460 mm.

3.6.4.8 Area of Steel Ast

The effective depth of the beam has been revised to 408.76 mm from the
limiting moment carrying capacity of the beam. Increasing that depth to 410 also
has raised the Mu, lim of the beam from the design factored moment of 115.08
kNm. Therefore, the area of steel is to be calculated from the moment equation
(Eq. 3.23 of Lesson 5), when steel is ensured to reach the design stress fd =
0.87 (415) = 361.05 N/mm2.

⎧ Ast f y ⎫
Mu = 0.87 f y Ast d ⎨1 − ⎬
⎩ f ck b d ⎭
(3.23)

Version 2 CE IIT, Kharagpur


Here, all but Ast are known. However, this will give a quadratic equation of Ast and
one of the values, the lower one, will be provided in the beam. The above
equation gives:

⎧ Ast ( 415 ) ⎫
115.08 (10 6 ) Nmm = 0.87 ( 415 ) Ast ( 410 ) ⎨1 − ⎬ Nmm
⎩ 20 ( 250 )( 410 ) ⎭

= 148030.5 Ast - 29.96715 Ast2

or Ast2 - 4939.759 Ast + 3840205 = 0

which gives

Ast = 966.5168 mm2 or 3973.2422 mm2

The values of xu determined from Eq. 3.16 of Lesson 5 are 193.87 mm and
796.97 mm respectively, when Ast = 966.5168 mm2 and 3973.2422 mm2. It is
seen that the value of xu with lower value of Ast is less than xu,max (= 216 mm).
However, the value of xu with higher value of Ast (= 3973.2422 mm2) is more than
the value of xu,max (= 0.48 d = 216 mm), which is not permissible as it exceeds
the total depth of the beam (= 460 mm). In some problems, the value of xu may
be less than the total depth of the beam, but it shall always be more than xu,max.
The beam becomes over-reinforced. Therefore, the lower value of the area of
steel is to be accepted as the tensile reinforcement out of the two values
obtained from the solution of the quadratic equation involving Ast.

Accepting the lower value of Ast = 966.5168 mm2, the percentage of steel
becomes

966.5168 (100)
= 0.9429 per cent
250 (410)

This percentage is higher than the initially assumed percentage as 0.72. By


providing higher effective depth, this can be maintained as shown below.

3.6.4.9 Increase of effective depth and new Ast

Increasing the effective depth to 450 mm from 410 mm, we have from Eq.
3.23 of Lesson 5,

Version 2 CE IIT, Kharagpur


⎧ Ast ( 415 ) ⎫
115.08 (10 6 ) = 0.87 (415) Ast (450) ⎨1 − ⎬
⎩ 20 (250) (450) ⎭

= 162472.5 Ast - 29.967148 Ast2

or Ast2 - 5421.6871 Ast + 3840205.2 = 0

or Ast = 0.5 {5421.6871 ± 3746.1808}

The lower value of Ast now becomes 837.75315 which gives the percentage of
Ast as

837.75315 (100)
= 0.7446 , which is close to earlier assumed percentage of
250 ( 450)
0.72.

Therefore, let us have d = 450 mm, D = 500 m, b = 250 mm and Ast =


837.75315 mm2 for this beam.

For any design problem, this increase of depth is obligatory to satisfy the
deflection and other requirements. Moreover, obtaining Ast with increased depth
employing moment equation (Eq. 3.23 of Lesson 5) as illustrated above, results
in under-reinforced beam ensuring ductility.

3.6.4.10 Further change of Ast due to increased dead load

However, increasing the total depth of the beam to 500 mm from earlier
value of 415 mm has increased the dead load and hence, the design moment Mu.
This can be checked as follows:

The revised dead load = 0.25 (0.5) (25) = 3.125 kN/m

Imposed loads = 7.00 kN/m

Total factored load Fd = 1.5(10.125) = 15.1875 kN/m

Mu = 15.1875 (8) = 121.5 kNm

The limiting moment that this beam can carry is obtained from using Mu, lim/bd2
factor as 2.76 from Table 3.3 of of Lesson 5. Thus,

Mu, lim = (2.76) bd2 = (2.76) (250) (450)2 Nmm

Version 2 CE IIT, Kharagpur


= 139.72 kNm > (Mu = 121.5 kNm)

Hence, it is under-reinforced beam.

Equation 3.23 of Lesson 5 is now used to determine the Ast for Mu =


121.5 kNm

⎧ Ast f y ⎫
Mu = 0.87 f y Ast d ⎨1 − ⎬
⎩ f ck b d ⎭
(3.23)
⎧ Ast (415) ⎫
or 121.5 (10 6 ) = 0.87 (415) Ast (450) ⎨1 − ⎬
⎩ 20 (250) (450) ⎭

= 162472.5 Ast - 29.96715 Ast2

or Ast = 0.5 {5421.6867 ± 3630.0038} = 895.84145 mm2

895.84 (100)
The steel reinforcement is = 0.7963 per cent which is 83 per cent
250 ( 450)
of pt,lim.

So, we have the final parameters as b = 250 m, d = 450 mm, D = 500 mm,
Ast = 895.84 mm2. A selection of 2-20 T bars and 2-14 T bars gives the Ast =
935 mm2 (Fig. 3.6.3). Though not designed, Fig. 3.6.3 shows the holder bars
and stirrups also.

Version 2 CE IIT, Kharagpur


3.6.4.11 Summary of steps

Table 3.4 presents the complete solution of the problem in eleven steps.
Six columns of the table indicate (i) parameters assumed/determined, (ii) if they
need revision, (iii) final parameters, (iv) major requirements of the parameter, (v)
reference section numbers, and (vi) reference source material.

Table 3.4 Steps of the illustrative problem

Ste Assumed/ If Final Major Referen Reference


p determined need(s) paramet requirement of ce source
parameter( revision er(s) the parameter(s) section Material(s)
s) (ii) (iii) number (vi)
(i) (iv) (v)
1 fck, fy No fck, fy Durability for fck 3.6.4.1 cl.6.1.2, cl.
and ductility for fy 8 and Table
5 of IS 456
2 d Yes No c/c span 3.6.4.2 cl. 23.2 of
d=
20 IS 456
3 Leff Yes No Boundary 3.6.4.2 cl.22.2 of
conditions IS 456
4 p = Ast/bd No Yes Ductility (p = 75 to 3.6.4.3 Table 3.1 of
80% of pt, lim) Lesson 5
for pt, lim
5 d Yes No Control of 3.6.4.4 cl.23.2 of
deflection IS 456
6 D, b Yes for b Economy 3.6.4.5 D = d + (40
D to 80 mm)
b = (0.5 to
0.67)D
7 Fd, Mu Yes No Strength 3.6.4.6 Strength of
material
books
8 d Yes No Limiting depth 3.6.4.7 Eq. 3.24 of
considering Lesson 5
Mu = Mu,lim
9 Ast Yes No Strength 3.6.4.8 Eq. 3.23 of
Lesson 5
10 d, D, Ast No d, D, Leff Under-reinforced 3.6.4.9 D = d + 50
Eq. 3.23 of
Lesson 5
11 Ast No Ast Strength 3.6.4.10 Eq. 3.23 of
Lesson 5

Version 2 CE IIT, Kharagpur


3.6.5 Use of Design Aids
From the solution of the illustrative numerical problems, it is clear that b, d,
D and Ast are having individual requirements and they are mutually related. Thus,
any design problem has several possible sets of these four parameters. After
getting one set of values, obtaining the second set, however, involves the same
steps as those of the first one. The steps are simple but time consuming and
hence, the designer may not have interest to compare between several sets of
these parameters. The client, contractor or the architect may request for
alternatives also. Thus, there is a need to get several sets of these four
parameters as quickly as possible. One way is to write a computer program
which also may restrict average designer not having a computer. Bureau of
Indian Standard (BIS), New Delhi published SP-16, Design Aids for Reinforced
Concrete to IS: 456, Special Publication No. 16, which is very convenient to get
several sets of these values quickly.

SP-16 provides both charts (graphs) and tables explaining their use with
illustrative examples. On top left or right corner of these charts and tables, the
governing parameters are provided for which that chart/table is to be used.

3.6.6 Solution by using Design Aids Charts (SP-16)


The initial dimension of effective depth d of Design Problem 3.1 is
modified from 400 mm to 410 mm first to satisfy the deflection and other
requirements and then to 450 mm as the final dimension. While using only the
charts or tables of SP-16, the final results as obtained for this problem by direct
calculation method will not be available. So, we will assume the percentage of
steel as 0.75 (0.96) = 0.72 initially.

3.6.6.1 Effective depth d

Chart 22 of SP-16 for fy = 415 N/mm2 and fck = 20 N/mm2 gives maximum
ratio of span to effective depth as 21.5 when the percentage of steel assumed =
0.75 (0.96) = 0.72. Thus, we get effective depth d = 8000/21.5 = 372.09 mm with
d = 372.09 mm and effective span Leff = 8000 mm. Total depth D = 372.09 + 50 =
422.09 = 425 mm (say).

3.6.6.2 Breadth b and factored moment Mu

Here also b = 250 mm is assumed and accordingly,

Dead load = 0.25 (0.425) (25) = 2.66 kN/m

Imposed loads = 7.00 kN/m

Total factored load, Fd = 1.5 (9.66) = 14.50 kN/m

Version 2 CE IIT, Kharagpur


Factored bending moment = (14.5) (8) = 116.00 kN/m

3.6.6.3 Checking of effective depth d and area of steel Ast

Chart 14 of SP-16 is for fck = 20 N/mm2, fy = 415 N/mm2 and d varying


from 300 to 550 mm. For this problem, Mu per metre width of the beam =
464 kNm/m . For the percentage of reinforcement = 0.72, chart 14 gives d = 460
mm and then D = 510 mm. Area of steel reinforcement 0.72 (25) (460)/100 =
828 mm2.

As in the earlier problem, the increased dead load due to the increased D
to 510 mm is checked below:

Revised dead load = 0.25 (0.51) (25) = 3.188 kN/m

Imposed loads = 7.000 kN/m

Total factored load Fd = 1.5 (10.188) = 15.282 kN/m

Factored moment Mu = 15.282 (8) = 122.256 kN/m

Mu per metre width of the beam = 122.256/0.25 = 489.02 kNm/m.

Chart 14 of Sp-16 gives the effective depth of the beam d = 472 mm and
D = 475 + 50 = 525 mm assuming d = 475 mm.

Ast required = (0.72/100) (250) (475) = 855.0 mm2

Thus, we have b = 250 mm, d = 475 mm, D = 525 mm and Ast = 855 mm2

3.6.7 Solution by using Design Aids Tables (SP-16)


3.6.7.1 Effective depth d

Tables 1 to 4 of SP-16 present pt for different values of Mu/bd2 covering a


wide range of fy and fck. Table 2 is needed for this problem.

To have more confidence while employing this method, we are starting


with the effective depth d as 400 mm as in the direct computational method. The
total depth D is (400 + 50) mm = 450 mm. The breadth b of the beam is taken
as 250 mm.

3.6.7.2 Factored load and bending moment

Dead load = 0.25 (0.45) (25) = 2.8125 kN/m

Version 2 CE IIT, Kharagpur


Imposed loads = 7.00 kN/m

Factored load Fd = 1.5 (2.8125 + 7.00) = 14.71875 kN/m

Factored bending moment Mu = 14.71875 (8) = 117.75 kNm

Mu 117.75 (10 6 )
= = 2.94375
b d2 250 (400) (400)

3.6.7.3 Use of Tables of SP-16

Mu
Table 2 of SP-16 shows that is restricted up to 2.76 when pt =
b d2
0.955, i.e. the limiting condition. So, increasing the effective depth by another 50
mm to have D = 500 m, the total factored moment as calculated in sec. 3.6.4.10
is 121.5 kNm,

Mu 121.5 (10 6 )
Now, = = 2.4
b d2 250 (450) (450)

From Table 2 of SP-16, the corresponding pt becomes 0.798.

Therefore, Ast = 0.01 (0.798) (250) (450) = 897.75 mm2

3.6.8 Comparison of Results of Three Methods


Results of this problem by three methods: (i) direct computation method,
(ii) use of charts of SP-16 and (iii) use of tables of SP-16 are summarised for the
purpose of comparison. The tabular summary includes the last two values of d
and Ast. Other parameters (b, fck and fy) are remaining constants in all the three
methods.

Table 3.5 Comparison of d and Ast by three methods

Cycle Direct computation Use of charts of SP-16 Use of tables of SP-


method 16
d (mm) Ast (mm2) d (mm) Ast (mm2) d (mm) Ast (mm2)
1 410 966.5168 460 828 400 Not
possible
2 450 895.84145 475 855 450 897.75
(2-20+2-14 (2-20 + 2- (2-20+2-14
= 935 12 = 854 = 935
mm2) mm2) mm2)

Version 2 CE IIT, Kharagpur


3.6.9 Other Alternatives using Charts and Tables of SP-16
Any alternative solution of d will involve computations of factored loads Fd
and bending moment Mu. Thereafter, Eq. 3.23 of Lesson 5 has to be solved to
get the value of Ast by direct computation method. On the other hand, it is very
simple to get the Ast with the help of either charts or tables of SP-16 from the
value of factored bending moment. Some alternatives are given below in Table
3.6 by the use of tables of SP-16.

In sec. 3.6.7.3, it is observed that an effective depth of 400 mm is not


acceptable. Hence, the effective depth is increased up to 450 mm at intervals of
10 mm and the corresponding Ast values are presented in Table 3.6. The width b
is kept as 250 mm and M 20 and Fe 415 are used for all the alternatives.

Mu
Table 3.6 Alternative values of d, D, Fd, Mu, , pt and Ast
b d2
Sl. d D Fd Mu Mu pt Ast
No. (mm) (mm) (kN/m) (kNm) bd 2 (%) (mm2)
2
(N/mm
)
1 410 460 14.825 118.5 2.8197 Not Not acceptable
acceptable
2 420 470 14.906 119.2 2.7041 976.5
2 5 0.93
3 430 480 15.0 120.0 2.596 0.88 946.0
0
4 440 490 15.09 2.4948 0.839 922.9
120.7
5
5 450 500 15.187 121.5 2.4 0.798 897.75
5

3.6.10 Advantages of using SP-16


The following are the advantages:

(i) Alternative sets of b, d and Ast are obtained very quickly.


(ii) The results automatically exclude those possibilities where the steel
reinforcement is inadmissible.

It has been mentioned that the reinforcement should be within 75 to 80 per


cent of limiting reinforcement to ensure ductile failure. The values of charts and
tables are given up to the limiting reinforcement. Hence, the designer should be
careful to avoid the reinforcement up to the limiting amount. Moreover, these
charts and tables can be used for the design of slabs also. Therefore, the values

Version 2 CE IIT, Kharagpur


are also taking care of the minimum reinforcement of slabs. The minimum
reinforcement of beams are higher than that of slabs. Accordingly, the designer
should also satisfy the requirement of minimum reinforcement for beams while
using SP-16.

It is further suggested to use the tables than the charts as the values of
the charts may have personal error while reading from the charts. Tabular values
have the advantage of numerical, which avoid personal error. Moreover,
intermediate values can also be evaluated by linear interpolation.

3.6.11 Practice Questions and Problems with Answers


Q.1: Mention the necessary input data and unknowns to be determined for the
two types of problems of singly reinforce beams.

A.1: (i) The input data for the design type of problems are layout plan, imposed
loads, grades of steel and concrete. The unknowns to be determined are
b, d, D, Ast and Leff.

(ii) The input data for the analysis type of problem are b, d, D, Ast, Leff,
grades of concrete and steel. The unknowns to be determined are Mu and
service imposed loads.

Q.2: State specific guidelines to select the initial dimensions/amount/grade of


the following parameters before designing the reinforced concrete beams:

(i) b, (ii) d, (iii) D, (iv) Ast, (v) diameter of reinforcing bars, (vi) grade of
concrete and (vii) grade of steel.

A.2: Sections 3.6.2.1 to 6 cover the answers.

Q.3: Name the three methods of solution of the design of reinforced concrete
beam problems.

A.3: The three methods are: (i) Direct computation method, (ii) Use of charts of
SP-16 and (iii) Use of tables of SP-16

Q.4: Determine the imposed loads and the tensile steel Ast,lim of the singly
reinforced rectangular beam shown in Figs. 3.6.2 and 4 of L = 8.0 m
simply supported, thickness of brick wall = 300 mm, width b = 300 mm,
effective depth d = 550 mm, total depth D = 600 mm, grade of concrete =
M 20 and grade of steel = Fe 500. Use (i) direct computation method, (ii)
design chart of SP-16 and (iii) design table of SP-16.

Version 2 CE IIT, Kharagpur


A.4: (i) Direct computation method:

The limiting moment of resistance Mu,lim is obtained from Eq. 3.24 as


follows

xu,lim ⎧ xu,lim ⎫
M u, lim = 0.36 ⎨1 − 0.42
2
⎬ b d f ck
d ⎩ d ⎭

Here, xu,lim /d = 0.46 (cl. 38.1, Note of IS 456:2000)

Hence, Mu,lim = 0.36 (0.46) {1 - 0.42 (0.46)} (300) (550) (500) (20)
Nmm

= 22,04,50,00,000 Nmm

Tensile steel Ast,lim is obtained from Eq. 3.23 as follows:

⎧ Ast ,lim f y ⎫
M u ,lim = 0.87 f y Ast ,lim d ⎨1 − ⎬
⎩ f ck b d ⎭
(3.23)

Denoting the unknown Ast,lim as A, we get:

A2 - 6600 A + 6081379.31 = 0

Solving the above equation, the lower value of A is the Ast,lim which is
equal to 1107.14 mm2

Version 2 CE IIT, Kharagpur


(ii) Use of chart of SP-16:

Using chart 17 of SP-16 for Mu,lim /b = 220.45/0.3 = 734.833 kNm/m, we


get the reinforcement percentage 100(Ast,lim )/bd = 0.67.

So, Ast,lim = 0.67 (300) (550)/100 = 1105 mm2

(iii) Use of table of SP-16:

Table 2 of SP-16 for Mu,lim /bd2 = 220.45/300 (0.55) (0.55) = 2.4292


N/mm2, we get the reinforcement percentage by linear interpolation as:

0.669 + (0.007) (0.0092)/(0.02) = 0.67222.

Hence, Ast,lim = 0.67222 (300) (550)/(100) = 1109.16 mm2

Comparison of results:

Method Ast,lim (mm2)

(i) 1107.14
(ii) 1105.00
(iii) 1109.16

Imposed loads:

The total load W per metre can be obtained from

W = 8 (Mu,lim ) / L2eff

Where, Leff is the lower of (i) 7700 + 550 or (ii) 8000 mm (cl. 22.2a of IS
456:2000)

Using Leff = 8000 mm and Mu,lim = 220.45 kNm

We get the total load W = 220.45/8 = 27.556 kN/m

The dead load of the beam = 0.3 (0.6) (25) = 4.5 kN/m

Hence, the imposed loads = 27.556 - 4.5 = 23.056 kN/m

Version 2 CE IIT, Kharagpur


3.6.12 References
1. Reinforced Concrete Limit State Design, 6th Edition, by Ashok K. Jain,
Nem Chand & Bros, Roorkee, 2002.
2. Limit State Design of Reinforced Concrete, 2nd Edition, by P.C.Varghese,
Prentice-Hall of India Pvt. Ltd., New Delhi, 2002.
3. Advanced Reinforced Concrete Design, by P.C.Varghese, Prentice-Hall of
India Pvt. Ltd., New Delhi, 2001.
4. Reinforced Concrete Design, 2nd Edition, by S.Unnikrishna Pillai and
Devdas Menon, Tata McGraw-Hill Publishing Company Limited, New
Delhi, 2003.
5. Limit State Design of Reinforced Concrete Structures, by P.Dayaratnam,
Oxford & I.B.H. Publishing Company Pvt. Ltd., New Delhi, 2004.
6. Reinforced Concrete Design, 1st Revised Edition, by S.N.Sinha, Tata
McGraw-Hill Publishing Company. New Delhi, 1990.
7. Reinforced Concrete, 6th Edition, by S.K.Mallick and A.P.Gupta, Oxford &
IBH Publishing Co. Pvt. Ltd. New Delhi, 1996.
8. Behaviour, Analysis & Design of Reinforced Concrete Structural Elements,
by I.C.Syal and R.K.Ummat, A.H.Wheeler & Co. Ltd., Allahabad, 1989.
9. Reinforced Concrete Structures, 3rd Edition, by I.C.Syal and A.K.Goel,
A.H.Wheeler & Co. Ltd., Allahabad, 1992.
10. Textbook of R.C.C, by G.S.Birdie and J.S.Birdie, Wiley Eastern Limited,
New Delhi, 1993.
11. Design of Concrete Structures, 13th Edition, by Arthur H. Nilson, David
Darwin and Charles W. Dolan, Tata McGraw-Hill Publishing Company
Limited, New Delhi, 2004.
12. Concrete Technology, by A.M.Neville and J.J.Brooks, ELBS with
Longman, 1994.
13. Properties of Concrete, 4th Edition, 1st Indian reprint, by A.M.Neville,
Longman, 2000.
14. Reinforced Concrete Designer’s Handbook, 10th Edition, by C.E.Reynolds
and J.C.Steedman, E & FN SPON, London, 1997.
15. Indian Standard Plain and Reinforced Concrete – Code of Practice (4th
Revision), IS 456: 2000, BIS, New Delhi.
16. Design Aids for Reinforced Concrete to IS: 456 – 1978, BIS, New Delhi.

3.6.13 Test 6 with Solutions


Maximum Marks = 50, Maximum Time = 30 minutes

Answer all questions.

TQ.1: State specific guidelines to select the initial dimensions/amount/grade of


the following parameters before designing the reinforced concrete beams:

Version 2 CE IIT, Kharagpur


(i) b, (ii) d, (iii) D, (iv) Ast, (v) diameter of reinforcing bars, (vi) grade of
concrete and (vii) grade of steel. (6 x 5 =
30 marks)

A.TQ.1: See secs. 3.6.2.1 to 6.

TQ.2: State the advantages of using SP-16 than employing direct computation
method in the design of a beam.
(15 marks)
A.TQ.2: See sec. 3.6.10 (except the last para).

TQ.3: Why the use of tables of SP-16 is better than the use of chart ?
(5 marks)

A.TQ.3: See sec. 3.6.10 (last para only).

3.6.14 Summary of this Lesson


Explaining the two types of problems and giving the necessary guidelines
of the preliminary selection of the parameters, this lesson illustrates step by step
method of solving design type of problems employing (i) direct computation
method, (ii) use of charts of SP-16 and (iii) use of tables of SP-16. The results of
a specific problem are compared. The advantages of using SP-16 in general and
the superiority of using the tables of SP-16 to the charts are also discussed.

Version 2 CE IIT, Kharagpur


Module
3
Limit State of Collapse -
Flexure (Theories and
Examples)

Version 2 CE IIT, Kharagpur


Lesson
7
Numerical Problems on
Singly Reinforced
Rectangular Beams
(Continued)
Version 2 CE IIT, Kharagpur
Instructional Objectives:
At the end of this lesson, the student should be able to:

• Apply the principles to analyse a given cross-section of a beam with specific


reinforcement to determine its moment of resistance.

3.7.1 Introduction
This lesson explains the determination of moment of resistance of given
singly reinforced rectangular beam sections with the help of illustrative analysis
type of problem. The numerical problem is solved by (i) direct computation
method, (ii) using charts of SP-16 and (iii) using tables of SP-16. Step by step
solutions illustrate the procedure clearly.

3.7.2 Analysis Type of Problems


It may be required to estimate the moment of resistance and hence the
service load of a beam already designed earlier with specific dimensions of b, d
and D and amount of steel reinforcement Ast. The grades of concrete and steel
are also known. In such a situation, the designer has to find out first if the beam
is under-reinforced or over-reinforced. The following are the steps to be followed
for such problems.

3.7.2.1 xu, max

The maximum depth of the neutral axis xu, max is determined from Table
3.2 of Lesson 5 using the known value of fy.

3.7.2.2 xu

The depth of the neutral axis for the particular beam is determined from
Eq. 3.16 of Lesson 5 employing the known values of fy, fck, b and Ast.

3.7.2.3 Mu and service imposed loads

The moment of resistance Mu is calculated for the three different cases as


follows:

(a) If xu < xu, max, the beam is under-reinforced and Mu is obtained from
Eq. 3.22 of Lesson 5.

(b) If xu = xu, max, the Mu is obtained from Eq. 3.24 of Lesson 5.

Version 2 CE IIT, Kharagpur


(c) If xu > xu, max, the beam is over-reinforced for which xu is taken as xu,
max and then Mu is calculated from Eq. 3.24 of Lesson 5, using xu = xu,
max.

With the known value of Mu, which is the factored moment, the total
factored load can be obtained from the boundary condition of the beam. The total
service imposed loads is then determined dividing the total factored load by
partial safety factor for loads (= 1.5). The service imposed loads are then
obtained by subtracting the dead load of the beam from the total service loads.

3.7.3 Analysis Problems 3.2 and 3.3


Determine the service imposed loads of two simply supported beam of
same effective span of 8 m (Figs. 3.7.1 and 2) and same cross-sectional
dimensions, but having two different amounts of reinforcement. Both the beams
are made of M 20 and Fe 415.

3.7.4 Solution by Direct Computation Method - Problem 3.2


Given data: b = 300 mm, d = 550 mm, D = 600 mm, Ast = 1256 mm2 (4-20
T), Leff = 8 m and boundary condition = simply supported (Fig. 3.7.1).

3.7.4.1 xu, max

From Table 3.2 of Lesson 5, we get xu, max = 0.479 d = 263.45 mm

Version 2 CE IIT, Kharagpur


3.7.4.2 xu

0.87 f y Ast
xu =
0.36 b f ck
(3.16)

0.87 (415) (1256)


=
0.36 (300) (20)

= 209.94385 mm < xu, max = (263.45 mm)

Hence, the beam is under-reinforced.

3.7.4.3 Mu and service imposed loads

For xu < xu, max , we have

Mu = 0.87 fy Ast (d - 0.42 xu)


(3.22)

= 0.87 (415) (1256) {550 - 0.42(209.94385)}

= 209.4272 kNm

8M 8 (209.4272)
Total factor load F = u =
d
L2 8 (8)
eff

= 26.1784 kN/m

Fd 26.1784
Total service load = = = 17.452266 kN/m
1.5 1.5

Dead load of the beam = 0.3 (0.6) (25) = 4.5 kN/m

Hence, service imposed loads = (17.452266 - 4.5) kN/m

= 12.952266 kN/m

Version 2 CE IIT, Kharagpur


3.7.5 Solution by Direct Computation Method - Problem 3.3

Given data: b = 300 mm, d = 550 mm, D = 600 mm, Ast = 1658 mm2 (4-20 T
+ 2-16 T), Leff = 8 m and boundary conditions = simply supported (Fig. 3.7.2)

3.7.5.1 xu, max

From Table 3.2 of Lesson 5, we get xu, max = 0.479 d = 263.45 mm

3.7.5.2 xu

0.87 f y Ast
xu =
0.36 b f ck
(3.16)

0.87 ( 415) (1658)


=
0.36 (300) (20)

= 277.13924 mm > xu, max = (263.45 mm)

Hence, the beam is over-reinforced.

3.7.5.3 Mu and service imposed loads

For xu > xu,max, we have

xu, max xu, max


Mu = 0.36 (1 − 0.42 ) b d 2 f ck (3.24)
d d

Version 2 CE IIT, Kharagpur


= 0.36 (0.479) {1 - 0.42(0.479)} (300) (550) (550) (20) Nmm

= 250.01356 kNm

If we use Eq. 3.22 using xu = xu,max, for Mu

Mu = 0.87 fy Ast (d - 0.42 xu, max)


(3.22)

Then, (Mu)steel = 0.87 (415) (1658) {550 - 0.42 (263.45)} Nmm

= 263.00468 kNm > (Mu)concrete (= 250.01356 kNm)

The higher Mu as obtained from steel is not true because the entire amount of
steel (1658 mm2) cannot yield due to over-reinforcing. Prior to that, concrete fails
at 250.01356 kNm. However, we can get the same of Mu as obtained from Eq.
3.24 of Lesson 5, if we can find out how much Ast is needed to have xu = 263.45
mm. From Eq. 3.16 of Lesson 5, we can write:

0.36 b f ck xu
( Ast ) needed for xu = 263.45 =
0.87 f y

0.36 (300) ( 20) (263.45)


=
0.87 (415)

= 1576.1027 mm2

If we use this value for Ast in Eq. 3.22 of Lesson 5, we get

(M u ) = 0.87 ( 415) (1576.1027 ) {550 − 0.42 ( 263.45 )}

= 250.0135 (same as obtained from Eq. 3.24).

From the factored moment Mu = 250.01356 kNm, we have:

8 Mu 8 (250.01356)
Total factored load = Fd = = = 31.251695
L2eff 8 (8)
kN/m

31.251695
Total service load = = 20.834463 kN/m
1 .5

Version 2 CE IIT, Kharagpur


Now, Dead load of the beam = 0.3 (0.6) (25) = 4.5 kN/m

Hence, service imposed loads = 20.834463 - 4.5 = 16.334463 kN/m

3.7.6 Solution by Design Chart - Problems 3.2 and 3.3


For the two problems with known b, d, D, Ast, grade of concrete and grade
of steel, chart 14 of SP-16 is applicable. From the effective depth d and
A 100
percentage of reinforcement pt (= st ) , the chart is used to find Mu per metre
bd
width. Multiplying Mu per meter width with b, we get Mu for the beam. After that,
the service imposed load is calculated using the relation

8 Mu
Service imposed load = - Dead load
(1.5) L2
(3.27)

The results of the two problems are furnished in Table 3.7.

Table 3.7 Results of Problems 3.2 and 3.3 (Chart of SP-16)

Prob- Ast (mm2) d, b Mu Mu 8 Mu Dead Service Remarks


lem pt (%) (mm) (kNm/m) (kNm) 1.5 L2 load impose
(Chart (kN/m) (kN/m) d loads
14) (kN/m)
3.2 1256 550, 700 210 17.5 4.5 13 Mu per m of
0.7612 300 700 is well
within the
chart, hence
under-
reinforced.
3.3 1658 550, 820 246 20.5 4.5 16 Maximum
1.205818 300 Mu = 820, so
1 over-
reinforced.

Version 2 CE IIT, Kharagpur


3.7.7 Solution by Design Tables - Problems 3.2 and 3.3
Table 2 of SP-16 presents the value of reinforcement percentage for
different combinations of fy and (Mu/bd2) for M-20. Here, from the known values
of p and fy, the corresponding values of Mu/bd2 are determined. These in turn
give Mu of the beam. Then the service imposed load can be obtained using Eq.
3.27 as explained in the earlier section (sec. 3.7.6). The results of the two
problems are presented in Table 3.8.

Table 3.8 Results of Problems 3.2 and 3.3 (Table of SP-16)

Prob- Ast (mm2) d, b fy Mu Mu 8 Mu Dead Service Re


(N/mm2
2
lem pt (%) (mm) bd (kNm) 1.5 L2 load impose mar
) (N/mm2 (kN/ d loads ks
) m) (kN/m)
from
Table 2
3.2 1256 550, 415 2.3105 209.6778 17.47315 4.5 12.973 *
0.7612 300 4 5 155
3.3 1658 500, 415 2.76 250.47 20.8725 4.5 16.372 **
1.205818 300 5
1

Mu (0.7612 − 0.757)
* Linear interpolated = 2.30 + (2.32 − 2.30) = 2.3105
b d2 (0.765 − 0.757)

** pt = 1.2058181 is not admissible, i.e. over-reinforced. So at pt = 0.955, Mu/bd2


=2.76.

3.7.8 Comparison of Results of Three Methods


The values of Mu and service imposed loads of the under-reinforced and
over-reinforced problems (Problems 3.2 and 3.3), computed by three methods,
are presented in Table 3.9.

Version 2 CE IIT, Kharagpur


Table 3.9 Comparison of results of Problems 3.2 and 3.3

Mu (kNm) Service imposed loads (kN/m)


Proble
m Direct Chart of Table of Direct Chart of Table of
computatio SP-16 SP-16 computatio SP-16 SP-16
n n
3.2 209.4272 210 209.6778 12.9522 13 12.97315
4 5

3.3 250.01356 246 250.47 16.334463 16 16.3725

3.7.9 Practice Questions and Problems with Answers


Q.1: Determine the moments of resistance Mu and service imposed loads on a
simply supported beam of effective span 10.0 m with b = 300mm, d = 500
mm, D = 550 mm and grades of concrete and steel are M20 and Fe500,
respectively for the two different cases employing (a) direct computation
method and (b) using charts and tables of SP-16: (i) when Ast is minimum
acceptable and (ii) when Ast is maximum acceptable (Fig. 3.7.3).

A.1: Given data: b = 300 mm, d = 500 mm, D = 550 mm, Leff = 10.0 m, fck =
20 N/mm2 and fy = 500 N/mm2.

(a) Direct computation method:

Case (i) When Ast is minimum acceptable (Eq. 3.26 of Lesson 6)


0.85 b d
Minimum As =
fy
(3.26)

Version 2 CE IIT, Kharagpur


So, As =
0.5 (300) (500)
mm2 = 255 mm2. Providing 4-12T gives Ast = 452
500
mm2.

Equation 3.16 of Lesson 5 gives the depth of the neutral axis xu:

0.87 f y Ast 0.87 (500) (452)


xu = = = 91.03 mm
0.36 b f ck 0.36 (300) (20)

Table 3.2 of Lesson 5 gives xu, max = 0.46(500) = 230 mm.

The beam is, therefore, under-reinforced (as xu < xu, max).

Equation 3.22 of Lesson 5 gives the Mu as follows:

Mu = 0.87 fy Ast (d - 0.42 xu)


(3.22)

= 0.87(500) (452) {500 - 0.42(91.03)} Nmm

= 90.79 kNm

8 Mu 8 (90.79)
Total factored load = = = 7.26 kN/m
L2eff 100

The dead load of the beam = 0.3 (0.55) (25) = 4.125 kN/m

So, the service imposed loads = {(Total factored load)/(Load factor)} - (Dead
load)

= 7.26/1.5 - 4.125 = 0.715 kN/m

This shows that the beam can carry maximum service imposed loads, 17 per
cent of its dead load only, when the acceptable minimum tensile reinforcement is
452 mm2 (4 bars of 12 mm diameter).

Case (ii) when Ast is maximum acceptable:

To ensure ductile failure, it is essential that the acceptable maximum


tensile reinforcement should be between 75 and 80 per cent of pt, lim and not as
given in clause 26.5.1.1.(b), i.e. 0.04 bD. Thus, here the maximum acceptable pt
should be between 0.57 and 0.61 per cent (as pt, lim = 0.76 from Table 3.1 of
Lesson 5). However, let us start with 0.76 per cent as the span is relatively large

Version 2 CE IIT, Kharagpur


and keeping in mind that while selecting the bar diameter, it may get reduced to
some extent.

So, (Ast)max = 0.76 (300) (500)/100 = 1140 mm2

Selecting 4-16 and 2-12 mm diameter bars, we get Ast = 1030 mm2 when pt
becomes 0.67 per cent. So, the maximum acceptable tensile reinforcement is
1030 mm2.

0.87 f y Ast 0.87 (500) (1030)


xu = = = 207.43 mm (Eq. 3.16 of Lesson 5)
0.36 b f ck 0.36 (300) (20)

xu < xu, max (as xu, max = 230 mm; see Case (i) of this problem).

Therefore, Mu is obtained from Eq. 3.22 of Lesson 5 as,

Mu = 0.87 fy Ast (d - 0.42 xu)


(3.22)

= 0.87(500) (1030) {500 - 0.42(207.43)} Nmm

= 184.99 kNm

8 Mu 8 (184.99)
Total factored load = = = 14.8 kN/m
L2eff 100

With dead load = 4.125 kN/m (see Case (i) of this problem), we have:

Service imposed load = 14.8/1.5 - 4.125 = 5.74 kN/m

This beam, therefore, is in a position to carry service imposed loads of 5.74


kN/m, about 40% higher than its own dead load.

(b) Using chart and tables of SP-16:

Tables 3.10 and 3.11 present the results using charts and tables respectively
of SP-16. For the benefit of the reader the different steps are given below
separately for the use of chart and table respectively for the minimum acceptable
reinforcement.

The steps using chart of SP-16 are given below:

Version 2 CE IIT, Kharagpur


Step 1: With the given fck, fy and d, choose the particular chart. Here, for fck =
20 N/mm2, fy = 500 N/mm2 and d = 500 mm, the needed chart no. is
17.

Step 2: Chart 17 shows minimum pt = 0.13% which gives Ast = 195 mmm2

Step 3: Provide bars of 4-12 mm diameter, which give pt = 0.301%

Step 4: For pt = 0.301, Chart 17 shows Mu = 300 kNm per metre width, which
gives Mu = 300 (0.3) = 90 kNm for the beam.

Step 5: The service imposed loads are calculated as follows:

8 Mu
Service imposed loads = 2
− 4.125 = 0.675 kN/m, using
L (1.5)
eff

the dead load of the beam as 4.125 kN/m (see case (i) of this problem).

Step 6: The capacity of the beam is to carry 0.675 kN/m which is (0.675/4.125)
100 = 16.36%.

Table 3.10 Results of Q.1 using chart of SP-16

Sl. Chart No. Minimum/ Provided (pt) Mu Service Service


No Min/Max Maximum bars provided (kNm/m, imposed imposed
. pt and Ast (No, mm (%) kNm) loads loads in
(%, mm2) diameter) (kN/m) (%)
1 17 0.13, 4-12 0.301 300, 0.675 16.36
(Minimum) 195 90
2 17 0.76, 4-16 + 0.67 610, 5.635 136.61
(Maximum) 1140 2-12 183

Similar calculations are done for the maximum acceptable reinforcement. The
steps are given below:

Step 1: With the given fck = 20 N/mm2, fy = 500 N/mm2, Table 2 of SP-16 is
selected.

Step 2: (pt)min = 0.07 from Table 2, which gives Ast = 105 mmm2

Step 3: Provide bars of 4-12 mm diameter, which give pt = 0.301%

Step 4: For (pt)provided, we get (Mu/bd2) from Table 2 of SP-16 by linear


interpolation as follows:

Version 2 CE IIT, Kharagpur


Mu (1.25 − 1.2) (0.301 − 0.298)
= 1.20 + = 1.204
b d2 (0.312 − 0.298)

Hence, Mu = 1.204 (300) (500)2 = 90.3 kNm

Step 5: Same as Step 5 while using chart of SP-16.

Step 6: Same as Step 6 while using chart of SP-16.

Table 3.11 Results of Q.1 using table of SP-16

Sl. Table No. Minimum/ Provided (pt) Mu Service Service


No. Min/Max Maximum bars provided b d2 imposed imposed
pt and Ast (No, mm (%) (kNm/m, loads loads in
diameter) kNm) (kN/m) (%)
(%, mm2)
1 2 0.07, 4-12 0.301 1.204, 0.691 16.75
(Minimum) 105 90.3

2 2 0.755, 4-16 + 0.67 2.42, 5.555 134.66


(Maximum 1132 2-12 181.5
)

3.7.10 References
1. Reinforced Concrete Limit State Design, 6th Edition, by Ashok K. Jain,
Nem Chand & Bros, Roorkee, 2002.
2. Limit State Design of Reinforced Concrete, 2nd Edition, by P.C.Varghese,
Prentice-Hall of India Pvt. Ltd., New Delhi, 2002.
3. Advanced Reinforced Concrete Design, by P.C.Varghese, Prentice-Hall of
India Pvt. Ltd., New Delhi, 2001.
4. Reinforced Concrete Design, 2nd Edition, by S.Unnikrishna Pillai and
Devdas Menon, Tata McGraw-Hill Publishing Company Limited, New
Delhi, 2003.
5. Limit State Design of Reinforced Concrete Structures, by P.Dayaratnam,
Oxford & I.B.H. Publishing Company Pvt. Ltd., New Delhi, 2004.
6. Reinforced Concrete Design, 1st Revised Edition, by S.N.Sinha, Tata
McGraw-Hill Publishing Company. New Delhi, 1990.
7. Reinforced Concrete, 6th Edition, by S.K.Mallick and A.P.Gupta, Oxford &
IBH Publishing Co. Pvt. Ltd. New Delhi, 1996.
8. Behaviour, Analysis & Design of Reinforced Concrete Structural Elements,
by I.C.Syal and R.K.Ummat, A.H.Wheeler & Co. Ltd., Allahabad, 1989.

Version 2 CE IIT, Kharagpur


9. Reinforced Concrete Structures, 3rd Edition, by I.C.Syal and A.K.Goel,
A.H.Wheeler & Co. Ltd., Allahabad, 1992.
10. Textbook of R.C.C, by G.S.Birdie and J.S.Birdie, Wiley Eastern Limited,
New Delhi, 1993.
11. Design of Concrete Structures, 13th Edition, by Arthur H. Nilson, David
Darwin and Charles W. Dolan, Tata McGraw-Hill Publishing Company
Limited, New Delhi, 2004.
12. Concrete Technology, by A.M.Neville and J.J.Brooks, ELBS with
Longman, 1994.
13. Properties of Concrete, 4th Edition, 1st Indian reprint, by A.M.Neville,
Longman, 2000.
14. Reinforced Concrete Designer’s Handbook, 10th Edition, by C.E.Reynolds
and J.C.Steedman, E & FN SPON, London, 1997.
15. Indian Standard Plain and Reinforced Concrete – Code of Practice (4th
Revision), IS 456: 2000, BIS, New Delhi.
16. Design Aids for Reinforced Concrete to IS: 456 – 1978, BIS, New Delhi.

3.7.11 Test 7 with Solutions


Maximum Marks = 50, Maximum Time = 30 minutes

TQ.1: Determine the moment of resistance for the beams shown in Figs. 3.7.4
and 3.7.5 using M 20 and Fe 250 by direct computation and using charts
and tables of SP-16.

A.TQ.1: Case A: TQ.1 A of Fig. 3.7.4

(i) Direct computation method

Version 2 CE IIT, Kharagpur


Ast = 1963 mm2 (4-25 φ)

x u , max = 0.53 (450) = 238.5 mm (Table 3.2 of Lesson 5 gives 0.53)

0.87 f y Ast 0.87 (250) (1963)


xu = = = 197.66 (See Eq. 3.16 of Lesson 5)
0.36 b f ck 0.36 (300) (20)

So, xu < x u , max shows that it is under-reinforced section for which Mu is


obtained from Eq. 3.22 of Lesson 5

Mu = 0.87 fy Ast (d - 0.42 xu)


(3.22)

= 0.87(250) (1963) {450 - 0.42(197.66)}

= 156.68 kNm

(ii) Chart of SP-16

Ast = 1963 mm2

Ast (100) 1963 (100)


pt = = = 1.45
bd 300 (450)

From chart 11 of SP-16, when pt = 1.45, d = 450, we get

Mu per metre width = 522 kNm/m

Mu = 522 (0.3) = 156.6 kNm

(iii) Table of SP-16

Table 2 of SP-16 is for M-20 and Fe250. At pt = 1.451, we get

Mu
= 2.58 N/mm2
bd2

So, Mu = 2.58 (300) (450) (450) (10-6) kNm = 156.74 kNm

The three values of Mu are close to each other.

Version 2 CE IIT, Kharagpur


A.TQ.1: Case B: TQ.1 B of Fig. 3.7.5

(i) Direct computation method

Ast = 1963 + 981 = 2944 mm2 (6-25 φ)

0.87 f y Ast 0.87 (250) (2944)


xu = = = 296.44 mm (See Eq. 3.16 of
0.36 b f ck 0.36 (300) (20)
Lesson 5)

x u , max = 0.53 (450) = 238.5 mm (Table 3.2 of Lesson 5 gives 0.53)

So, xu > xu, max and the beam is over-reinforced. In such a situation, we
take xu = xu, max = 238.5 mm. The Mu will be calculated from Eq. 3.24 of
Lesson 5.

xu , max xu , max
Mu = M u , lim = 0.36 (1 − 0.42 ) b d 2 f ck
d d
(3.24)

= 0.36 (0.53) {1 - 0.42 (0.53)} (300) (450)2 (20) Nmm

= 180.22 kNm

(ii) Chart of SP-16

2944 (100)
pt = = 2.18 . In chart 11 (for M 20 and Fe 250), maximum
300 (450)
admissible pt is 1.75 and for this pt when d = 450, Mu = 600 kNm/m.

Version 2 CE IIT, Kharagpur


So, Mu = 600 (0.3) = 180 kNm

(iii) Table of SP-16

Table 2 (for M 20 and Fe 250) has the maximum pt = 1.76 and at that
value, (Mu/bd2) = 2.98. This gives

Mu = 2.98 (300) (450)2 (10-6) = 181.03 kNm. Here also the three values of
Mu are close to each other.

3.7.12 Summary of this Lesson


This lesson explains the use of equations derived in Lesson 5 for the
analysis type of problems. The three methods (i) direct computation, (ii) use of
charts of SP-16 and (iii) use of tables of SP-16 are illustrated through the step by
step solutions of numerical problems. Their results are compared to show the
closeness of them.

Version 2 CE IIT, Kharagpur


Module
4
Doubly Reinforced
Beams – Theory and
Problems

Version 2 CE IIT, Kharagpur


Lesson
8
Doubly Reinforced
Beams – Theory

Version 2 CE IIT, Kharagpur


Instructional Objectives:
At the end of this lesson, the student should be able to:

• explain the situations when doubly reinforced beams are designed,


• name three cases other than doubly reinforced beams where compression
reinforcement is provided,
• state the assumptions of analysis and design of doubly reinforced beams,
• derive the governing equations of doubly reinforced beams,
• calculate the values of fsc from (i) d'/d and (ii) calculating the strain of the
compression reinforcement,
• state the minimum and maximum amounts of Asc and Ast in doubly
reinforced beams,
• state the two types of numerical problems of doubly reinforced beams,
• name the two methods of solving the two types of problems, and
• write down the steps of the two methods for each of the two types of
problems.

4.8.1 Introduction

Version 2 CE IIT, Kharagpur


Concrete has very good compressive strength and almost negligible
tensile strength. Hence, steel reinforcement is used on the tensile side of
concrete. Thus, singly reinforced beams reinforced on the tensile face are good
both in compression and tension. However, these beams have their respective
limiting moments of resistance with specified width, depth and grades of concrete
and steel. The amount of steel reinforcement needed is known as Ast,lim. Problem
will arise, therefore, if such a section is subjected to bending moment greater
than its limiting moment of resistance as a singly reinforced section.

There are two ways to solve the problem. First, we may increase the depth
of the beam, which may not be feasible in many situations. In those cases, it is
possible to increase both the compressive and tensile forces of the beam by
providing steel reinforcement in compression face and additional reinforcement in
tension face of the beam without increasing the depth (Fig. 4.8.1). The total
compressive force of such beams comprises (i) force due to concrete in
compression and (ii) force due to steel in compression. The tensile force also has
two components: (i) the first provided by Ast,lim which is equal to the compressive
force of concrete in compression. The second part is due to the additional steel in
tension - its force will be equal to the compressive force of steel in compression.
Such reinforced concrete beams having steel reinforcement both on tensile and
compressive faces are known as doubly reinforced beams.

Doubly reinforced beams, therefore, have moment of resistance more than


the singly reinforced beams of the same depth for particular grades of steel and
concrete. In many practical situations, architectural or functional requirements
may restrict the overall depth of the beams. However, other than in doubly
reinforced beams compression steel reinforcement is provided when:

(i) some sections of a continuous beam with moving loads undergo


change of sign of the bending moment which makes compression
zone as tension zone or vice versa.

(ii) the ductility requirement has to be followed.

(iii) the reduction of long term deflection is needed.

It may be noted that even in so called singly reinforced beams there would be
longitudinal hanger bars in compression zone for locating and fixing stirrups.

4.8.2 Assumptions
(i) The assumptions of sec. 3.4.2 of Lesson 4 are also applicable here.

(ii) Provision of compression steel ensures ductile failure and hence,


the limitations of x/d ratios need not be strictly followed here.

Version 2 CE IIT, Kharagpur


(iii) The stress-strain relationship of steel in compression is the same
as that in tension. So, the yield stress of steel in compression is
0.87 fy.

4.8.3 Basic Principle

As mentioned in sec. 4.8.1, the moment of resistance Mu of the doubly


reinforced beam consists of (i) Mu,lim of singly reinforced beam and (ii) Mu2
because of equal and opposite compression and tension forces (C2 and T2) due
to additional steel reinforcement on compression and tension faces of the beam
(Figs. 4.8.1 and 2). Thus, the moment of resistance Mu of a doubly reinforced
beam is

Version 2 CE IIT, Kharagpur


Mu = Mu,lim + Mu2 (4.1)

The Mu,lim is as given in Eq. 3.24 of Lesson 5, i.e.,

xu ,max xu ,max
Mu,lim = 0.36 ( ) (1 − 0.42 ) b d 2 f ck (4.2)
d d

Also, Mu,lim can be written from Eq. 3.22 of Lesson 5, using xu = xu, max, i.e.,

Mu, lim = 0.87 Ast, lim fy (d - 0.42 xu, max)

xu , max
= 0.87 pt, lim (1 - 0.42 ) b d2 fy (4.3)
d

The additional moment Mu2 can be expressed in two ways (Fig. 4.8.2):
considering (i) the compressive force C2 due to compression steel and (ii) the
tensile force T2 due to additional steel on tension face. In both the equations, the
lever arm is (d - d'). Thus, we have

M u2 = Asc ( f sc − f cc ) (d − d ' ) (4.4)

M u2 = Ast 2 ( 0.87 f y ) ( d − d ' ) (4.5)

where Asc = area of compression steel reinforcement

fsc = stress in compression steel reinforcement

fcc = compressive stress in concrete at the level of centroid of


compression steel reinforcement

Ast2 = area of additional steel reinforcement

Since the additional compressive force C2 is equal to the additional tensile force
T2, we have
Asc (fsc - fcc) = Ast2 (0.87 fy)
(4.6)

Any two of the three equations (Eqs. 4.4 - 4.6) can be employed to determine Asc
and Ast2.

The total tensile reinforcement Ast is then obtained from:

Ast = Ast1 + Ast 2 (4.7)

Version 2 CE IIT, Kharagpur


bd M u , lim
where Ast1 = p t , lim = (4.8)
100 0.87 f y ( d − 0.42 xu , max )

4.8.4 Determination of fsc and fcc


It is seen that the values of fsc and fcc should be known before calculating
Asc. The following procedure may be followed to determine the value of fsc and
fcc for the design type of problems (and not for analysing a given section). For
the design problem the depth of the neutral axis may be taken as xu,max as
shown in Fig. 4.8.2. From Fig. 4.8.2, the strain at the level of compression steel
reinforcement εsc may be written as

d'
ε sc = 0.0035 (1 − ) (4.9)
xu , max

The stress in compression steel fsc is corresponding to the strain εsc of Eq. 4.9
and is determined for (a) mild steel and (b) cold worked bars Fe 415 and 500 as
given below:

(a) Mild steel Fe 250

The strain at the design yield stress of 217.39 N/mm2 (fd = 0.87 fy ) is
0.0010869 (= 217.39/Es). The fsc is determined from the idealized stress-strain
diagram of mild steel (Fig. 1.2.3 of Lesson 2 or Fig. 23B of IS 456) after
computing the value of εsc from Eq. 4.9 as follows:

(i) If the computed value of εsc ≤ 0.0010869, fsc = εsc Es = 2 (105) εsc

(ii) If the computed value of εsc > 0.0010869, fsc = 217.39 N/mm2.

(b) Cold worked bars Fe 415 and Fe 500

The stress-strain diagram of these bars is given in Fig. 1.2.4 of Lesson 2


and in Fig. 23A of IS 456. It shows that stress is proportional to strain up to a
stress of 0.8 fy. The stress-strain curve for the design purpose is obtained by
substituting fyd for fy in the figure up to 0.8 fyd. Thereafter, from 0.8 fyd to fyd,
Table A of SP-16 gives the values of total strains and design stresses for Fe 415
and Fe 500. Table 4.1 presents these values as a ready reference here.

Version 2 CE IIT, Kharagpur


Table 4.1 Values of fsc and εsc

Stress level Fe 415 Fe 500

Strain εsc Stress fsc Strain εsc Stress fsc


(N/mm2) (N/mm2)
0.80 fyd 0.00144 288.7 0.00174 347.8
0.85 fyd 0.00163 306.7 0.00195 369.6
0.90 fyd 0.00192 324.8 0.00226 391.3
0.95 fyd 0.00241 342.8 0.00277 413.0
0.975 fyd 0.00276 351.8 0.00312 423.9
1.0 fyd 0.00380 360.9 0.00417 434.8
Linear interpolation may be done for intermediate values.

The above procedure has been much simplified for the cold worked bars by
presenting the values of fsc of compression steel in doubly reinforced beams for
different values of d'/d only taking the practical aspects into consideration. In
most of the doubly reinforced beams, d'/d has been found to be between 0.05
and 0.2. Accordingly, values of fsc can be computed from Table 4.1 after
determining the value of εsc from Eq. 4.9 for known values of d'/d as 0.05, 0.10,
0.15 and 0.2. Table F of SP-16 presents these values of fsc for four values of d'/d
(0.05, 0.10, 0.15 and 0.2) of Fe 415 and Fe 500. Table 4.2 below, however,
includes Fe 250 also whose fsc values are computed as laid down in sec.
4.8.4(a) (i) and (ii) along with those of Fe 415 and Fe 500. This table is very
useful and easy to determine the fsc from the given value of d'/d. The table also
includes strain values at yield which are explained below:
(i) The strain at yield of Fe 250 =
Design Yield Stress 250
= = 0.0010869
Es 1.15 (200000)

Here, there is only elastic component of the strain without any inelastic strain.

Design Yield Stress


(ii) The strain at yield of Fe 415 = Inelastic Strain +
Es

415
= 0.002 + = 0.0038043
1.15 (200000)

500
(iii) The strain at yield of Fe 500 = 0.002 + = 0.0041739
1.15 (200000)

Version 2 CE IIT, Kharagpur


Table 4.2 Values of fsc for different values of d'/d

fy d'/d Strain at
(N/mm2) 0.05 0.10 0.15 0.20 yield
250 217.4 217.4 217.4 217.4 0.0010869
415 355 353 342 329 0.0038043
500 412 412 395 370 0.0041739

4.8.5 Minimum and maximum steel


4.8.5.1 In compression

There is no stipulation in IS 456 regarding the minimum compression steel


in doubly reinforced beams. However, hangers and other bars provided up to
0.2% of the whole area of cross section may be necessary for creep and
shrinkage of concrete. Accordingly, these bars are not considered as
compression reinforcement. From the practical aspects of consideration,
therefore, the minimum steel as compression reinforcement should be at least
0.4% of the area of concrete in compression or 0.2% of the whole cross-sectional
area of the beam so that the doubly reinforced beam can take care of the extra
loads in addition to resisting the effects of creep and shrinkage of concrete.

The maximum compression steel shall not exceed 4 per cent of the whole
area of cross-section of the beam as given in cl. 26.5.1.2 of IS 456.

4.8.5.2 In tension

As stipulated in cl. 26.5.1.1(a) and (b) of IS 456, the minimum amount of


tensile reinforcement shall be at least (0.85 bd/fy) and the maximum area of
tension reinforcement shall not exceed (0.04 bD).

It has been discussed in sec. 3.6.2.3 of Lesson 6 that the singly reinforced
beams shall have Ast normally not exceeding 75 to 80% of Ast,lim so that xu
remains less than xu,max with a view to ensuring ductile failure. However, in the
case of doubly reinforced beams, the ductile failure is ensured with the presence
of compression steel. Thus, the depth of the neutral axis may be taken as xu, max
if the beam is over-reinforced. Accordingly, the Ast1 part of tension steel can go
up to Ast, lim and the additional tension steel Ast2 is provided for the additional
moment Mu - Mu, lim. The quantities of Ast1 and Ast2 together form the total Ast,
which shall not exceed 0.04 bD.

Version 2 CE IIT, Kharagpur


4.8.6 Types of problems and steps of solution
Similar to the singly reinforced beams, the doubly reinforced beams have
two types of problems: (i) design type and (ii) analysis type. The different steps of
solutions of these problems are taken up separately.

4.8.6.1 Design type of problems

In the design type of problems, the given data are b, d, D, grades of


concrete and steel. The designer has to determine Asc and Ast of the beam from
the given factored moment. These problems can be solved by two ways: (i) use
of the equations developed for the doubly reinforced beams, named here as
direct computation method, (ii) use of charts and tables of SP-16.

(a) Direct computation method

Step 1: To determine Mu, lim and Ast, lim from Eqs. 4.2 and 4.8,
respectively.

Step 2: To determine Mu2, Asc, Ast2 and Ast from Eqs. 4.1, 4.4, 4.6 and
4.7, respectively.

Step 3: To check for minimum and maximum reinforcement in


compression and tension as explained in sec. 4.8.5.

Step 4: To select the number and diameter of bars from known values of
Asc and Ast.

(b) Use of SP table

Tables 45 to 56 present the pt and pc of doubly reinforced sections for


d'/d = 0.05, 0.10, 0.15 and 0.2 for different fck and fy values against Mu /bd2.
The values of pt and pc are obtained directly selecting the proper table with
known values of Mu/bd2 and d'/d.

4.8.6.2 Analysis type of problems

In the analysis type of problems, the data given are b, d, d', D, fck, fy, Asc
and Ast . It is required to determine the moment of resistance Mu of such beams.
These problems can be solved: (i) by direct computation method and (ii) by using
tables of SP-16.

(a) Direct computation method

Step 1: To check if the beam is under-reinforced or over-reinforced.

Version 2 CE IIT, Kharagpur


First, xu,max is determined assuming it has reached limiting stage using
xu, max
coefficients as given in cl. 38.1, Note of IS 456. The strain of tensile steel
d
ε c (d - xu, max )
εst is computed from ε st = and is checked if εst has reached the
xu, max
yield strain of steel:

fy
ε st at yield = + 0.002
1.15 (E)

The beam is under-reinforced or over-reinforced if εst is less than or more than


the yield strain.

Step 2: To determine Mu,lim from Eq. 4.2 and Ast,lim from the pt, lim given
in Table 3.1 of Lesson 5.

Step 3: To determine Ast2 and Asc from Eqs. 4.7 and 4.6, respectively.

Step 4: To determine Mu2 and Mu from Eqs. 4.4 and 4.1, respectively.

(b) Use of tables of SP-16

As mentioned earlier Tables 45 to 56 are needed for the doubly reinforced


beams. First, the needed parameters d'/d, pt and pc are calculated. Thereafter,
Mu/bd2 is computed in two stages: first, using d'/d and pt and then using d'/d and
pc . The lower value of Mu is the moment of resistance of the beam.

4.8.7 Practice Questions and Problems with Answers


Q.1: When do we go for doubly reinforced beams ?

A.1: The depth of the beams may be restricted for architectural and/or
functional requirements. Doubly reinforced beams are designed if such
beams of restricted depth are required to resist moment more that its Mu,
lim.

Q.2: Name three situations other than doubly reinforced beams, where the
compression reinforcement is provided.

A.2: Compression reinforcement is provided when:

(i) Some sections of a continuous beam with moving loads undergo


change of sign of the bending moment which makes compression
zone as tension zone,

Version 2 CE IIT, Kharagpur


(ii) the ductility requirement has to be satisfied,

(iii) the reduction of long term deflection is needed.

Q.3: State the assumptions of the analysis and design of doubly reinforced
beams.

A.3: See sec. 4.8.2 (i), (ii) and (iii).

Q.4: Derive the governing equations of a doubly reinforced beam.

A.4: See sec. 4.8.3

Q.5: How do you determine fsc of mild steel and cold worked bars and fcc?

A.5: See sec. 4.8.4

Q.6: State the minimum and maximum amounts of Asc and Ast in doubly
reinforced beams.

A.6: See sec. 4.8.5

Q.7: State the two types of problems of doubly reinforced beams specifying the
given data and the values to be determined in the two types of problems.

A.7: The two types of problems are:

(i) Design type of problems and


(ii) Analysis type of problems

(i) Design type of problems:

The given data are b, d, D, fck, fy and Mu . It is required to determine Asc


and Ast.

(ii) Analysis type of problems:

The given data are b, d, D, fck, fy, Asc and Ast. It is required to
determine the Mu of the beam.

Q.8: Name the two methods of solving the two types of problems.

A.8: The two methods of solving the two types of problems are:

(i) Direct computation method, and


(ii) Use of tables of SP-16.

Version 2 CE IIT, Kharagpur


Q.9: Write down the steps of the solution by the two methods of each of the two
types of problems.

A.9: (A) For the design type of problems:

(i) See sec. 4.8.6.1(a) for the steps of direct computation method, and

(ii) See sec. 4.8.6.1(b) for the steps of using the tables of SP-16

(B) For the analysis type of problems:

(i) See sec. 4.8.6.2 (a) for the steps of direct computation method, and

(ii) See sec. 4.8.6.2 (b) for the steps of using the tables of SP-16.

4.8.8 References
1. Reinforced Concrete Limit State Design, 6th Edition, by Ashok K. Jain,
Nem Chand & Bros, Roorkee, 2002.
2. Limit State Design of Reinforced Concrete, 2nd Edition, by P.C.Varghese,
Prentice-Hall of India Pvt. Ltd., New Delhi, 2002.
3. Advanced Reinforced Concrete Design, by P.C.Varghese, Prentice-Hall of
India Pvt. Ltd., New Delhi, 2001.
4. Reinforced Concrete Design, 2nd Edition, by S.Unnikrishna Pillai and
Devdas Menon, Tata McGraw-Hill Publishing Company Limited, New
Delhi, 2003.
5. Limit State Design of Reinforced Concrete Structures, by P.Dayaratnam,
Oxford & I.B.H. Publishing Company Pvt. Ltd., New Delhi, 2004.
6. Reinforced Concrete Design, 1st Revised Edition, by S.N.Sinha, Tata
McGraw-Hill Publishing Company. New Delhi, 1990.
7. Reinforced Concrete, 6th Edition, by S.K.Mallick and A.P.Gupta, Oxford &
IBH Publishing Co. Pvt. Ltd. New Delhi, 1996.
8. Behaviour, Analysis & Design of Reinforced Concrete Structural Elements,
by I.C.Syal and R.K.Ummat, A.H.Wheeler & Co. Ltd., Allahabad, 1989.
9. Reinforced Concrete Structures, 3rd Edition, by I.C.Syal and A.K.Goel,
A.H.Wheeler & Co. Ltd., Allahabad, 1992.
10. Textbook of R.C.C, by G.S.Birdie and J.S.Birdie, Wiley Eastern Limited,
New Delhi, 1993.
11. Design of Concrete Structures, 13th Edition, by Arthur H. Nilson, David
Darwin and Charles W. Dolan, Tata McGraw-Hill Publishing Company
Limited, New Delhi, 2004.
12. Concrete Technology, by A.M.Neville and J.J.Brooks, ELBS with
Longman, 1994.
13. Properties of Concrete, 4th Edition, 1st Indian reprint, by A.M.Neville,
Longman, 2000.

Version 2 CE IIT, Kharagpur


14. Reinforced Concrete Designer’s Handbook, 10th Edition, by C.E.Reynolds
and J.C.Steedman, E & FN SPON, London, 1997.
15. Indian Standard Plain and Reinforced Concrete – Code of Practice (4th
Revision), IS 456: 2000, BIS, New Delhi.
16. Design Aids for Reinforced Concrete to IS: 456 – 1978, BIS, New Delhi.

4.8.9 Test 8 with Solutions


Maximum Marks = 50, Maximum Time = 30 minutes

Answer all questions.

TQ.1: Derive the governing equations of a doubly reinforced beam.


(10 marks)

A.TQ.1: See sec. 4.8.3

TQ.2: State the two types of problems of doubly reinforced beams specifying the
given data and the values to be determined in the two type of problems.
(8 marks)

A.TQ.2: The two types of problems are:

(i) Design type of problems and


(ii) Analysis type of problems

(i) Design type of problems:

The given data are b, d, D, fck, fy and Mu . It is required to determine Asc


and Ast.

(ii) Analysis type of problems:

The given data are b, d, D, fck, fy, Asc and Ast. It is required to
determine the Mu of the beam.

TQ.3: Write down the steps of the solution by the two methods of each of the two
types of problems.
(8 marks)
A.TQ.3: (A) For the design type of problems:

(i) See sec. 4.8.6.1(a) for the steps of direct computation method, and

(ii) See sec. 4.8.6.1(b) for the steps of using the tables of SP-16

(B) For the analysis type of problems:

Version 2 CE IIT, Kharagpur


(i) See sec. 4.8.6.2 (a) for the steps of direct computation method, and

(ii) See sec. 4.8.6.2 (b) for the steps of using the tables of SP-16.

TQ.4: How do you determine fsc of mild steel and cold worked bars and fcc?
(8 marks)
A.TQ.4: See sec. 4.8.4

TQ.5: State the assumptions of the analysis and design of doubly reinforced
beams.
(8 marks)
A.TQ.5: See sec. 4.8.2 (i), (ii) and (iii).

TQ.6: Name three situations other than doubly reinforced beams, where the
compression reinforcement is provided.
(8 marks)
A.TQ.6: Compression reinforcement is provided when:

(i) Some sections of a continuous beam with moving loads undergo


change of sign of the bending moment which makes compression
zone as tension zone,

(ii) the ductility requirement has to be satisfied,

(iii) the reduction of long term deflection is needed.

4.8.10 Summary of this Lesson


Lesson 8 derives the governing equations of the doubly reinforced beams
explaining different assumptions and situations when they are needed. The
methods of determination of compressive stress in steel are illustrated. The two
types of problems and the different steps of solution of them by two different
methods are explained.

Version 2 CE IIT, Kharagpur


Module
4
Doubly Reinforced
Beams – Theory and
Problems

Version 2 CE IIT, Kharagpur


Lesson
9
Doubly Reinforced
Beams – Theory

Version 2 CE IIT, Kharagpur


Instructional Objectives:
At the end of this lesson, the student should be able to:

• design the amounts of compression and tensile reinforcement if the b, d, d',


fck, fy and Mu are given, and
• determine the moment of resistance of a beam if b, d, d', fck, fy, Asc and Ast are
given.

4.9.1 Introduction
This lesson illustrates the application of the theory of doubly reinforced
beams in solving the two types of problems mentioned in Lesson 8. Both the
design and analysis types of problems are solved by (i) direct computation
method, and (ii) using tables of SP-16. The step by step solution of the problems
will help in understanding the theory of Lesson 8 and its application.

4.9.2 Numerical problems


4.9.2.1 Problem 4.1

Design a simply supported beam of effective span 8 m subjected to imposed


loads of 35 kN/m. The beam dimensions and other data are: b = 300 mm, D =
700 mm, M 20 concrete, Fe 415 steel (Fig. 4.9.1). Determine fsc from d'/d as
given in Table 4.2 of Lesson 8.

Version 2 CE IIT, Kharagpur


(a) Solution by direct computation method

Dead load of the beam = 0.3 (0.7) (25) = 5.25 kN/m

Imposed loads (given) = 35.00 kN/m

Total loads = 5.25 + 35.00 = 40.25 kN/m

wl 2 ( 1.5 ) ( 40.25 ) ( 8 )( 8 )
Factored bending moment = (1.5 ) = = 482.96 kNm
8 8

Assuming d' = 70 mm, d = 700 - 70 = 630 mm

xu, max
= 0.48 gives xu, max = 0.48 (630) = 302.4 mm
d

Step 1: Determination of Mu, lim and Ast, lim

xu, max xu, max


M u, lim = 0.36 ( ) (1 - 0.42 ) b d 2 f ck
d d
(4.2)

= 0.36(0.48) {1 – 0.42 (0.48)} (300) (630)2 (20) (10-6) kNm

= 328.55 kNm

M u, lim
Ast, lim = (6.8)
0.87 f y (d - 0.42 xu, max )

328.55 (10 6 ) Nmm


So, Ast1 = = 1809.14 mm 2
0.87 (415) { 630 − 0.42 (0.48) 630 }

Step 2: Determination of Mu2, Asc, Ast2 and Ast

(Please refer to Eqs. 4.1, 4.4, 4.6 and 4.7 of Lesson 8.)

M u 2 = M u - M u, lim = 482.96 - 328.55 = 154.41 kNm

Here, d'/d = 70/630 = 0.11

From Table 4.2 of Lesson 8, by linear interpolation, we get,

Version 2 CE IIT, Kharagpur


353 - 342
f sc = 353 - = 350.8 N/mm2
5

M u2 154.41 (10 6 ) Nmm


Asc = = = 806.517 mm 2
( f sc - f cc ) (d - d' ) {350.8 − 0.446 (20)} (630 - 70) N/mm

Asc ( f sc - f cc ) 806.517 (350.8 - 8.92)


Ast 2 = = = 763.694 mm 2
0.87 f y (0.87) (415)

Ast = Ast1 + Ast 2 = 1809.14 + 783.621 = 2572.834 mm 2

Step 3: Check for minimum and maximum tension and compression steel.

(vide sec.4.8.5 of Lesson 8)

(i) In compression:

0.2
(a) Minimum Asc = (300) (700) = 420 mm 2
100

4
(b) Maximum Asc = (300) (700) = 8400 mm 2
100

Thus, 420 mm2 < 806.517 mm2 < 8400 mm2 . Hence, o.k.

(ii) In tension:

0.85 b d 0.85 (300) (630)


(a) Minimum Ast = = 387.1 mm 2
fy 415

4
(b) Maximum Ast = (300) (700) = 8400 mm 2
100

Here, 387.1 mm2 < 2572.834 mm2 < 8400 mm2 . Hence, o.k.

Step 4: Selection of bar diameter and numbers.

(i) for Asc: Provide 2-20 T + 2-12 T (= 628 + 226 = 854 mm2)

(ii) for Ast: Provide 4-25 T + 2-20 T (= 1963 + 628 = 2591 mm2)

Version 2 CE IIT, Kharagpur


It may be noted that Ast is provided in two layers in order to provide adequate
space for concreting around reinforcement. Also the centroid of the tensile bars is
at 70 mm from bottom (Fig. 4.9.1).

(b) Solution by use of table of SP-16

Mu 482.96 (10 6 ) 70
For this problem, = = 4.056 and d'/d = = 0.11.
bd 2
300 (630) 2
630

Mu
Table 50 of SP-16 gives pt and pc for = 4 and 4.1 and d'/d = 0.1 and 0.15.
b d2
The required pt and pc are determined by linear interpolation. The values are
presented in Table 4.3 to get the final pt and pc of this problem.

Table 4.3 Calculation of pt and pc

Mu d ' / d = 0.1 d ' / d = 0.15 d ' / d = 0.11


b d2
pt 1.337 1.360 0.023 (0.01)
1.337 + = 1.342
4.0 0.05
pc 0.401 0.437 0.036 (0.01)
0.433 + = 0.408
0.05
pt 1.368 1.392 0.024 (0.01)
1.368 + = 1.373
0.05
4.1 pc 0.433 0.472 0.039 (0.01)
0.433 + = 0.441
0.05
pt Not NA 0.031 (0.056)
1.342 + = 1.3594
Applicable 0 .1
4.056 (NA)
pc NA NA 0.033 (0.056)
0.408 + = 0.426
0 .1

1.3594 (300) (630)


So, Ast = = 2569.26 m 2
100

0.426 (300) (630)


and Asc = = 805.14 mm 2
100

These values are close to those obtained by direct computation method where
Ast = 2572.834 mm2 and Asc = 806.517 mm2. Thus, by using table of SP-16 we

Version 2 CE IIT, Kharagpur


get the reinforcement very close to that of direct computation method. Hence,
provide

(i) for Asc: 2-20 T + 2-12 T (= 628 + 226 = 854 mm2)

(ii) for Ast: 4-25 T + 2-20 T (= 1963 + 628 = 2591 mm2)

4.9.2.2 Problem 4.2

Determine the ultimate moment capacity of the doubly reinforced beam of b =


350 mm, d' = 60 mm, d = 600 mm, Ast = 2945 mm2 (6-25 T), Asc = 1256 mm2
(4-20 T), using M 20 and Fe 415 (Fig.4.9.2). Use direct computation method only.

Solution by direct computation method

Step 1: To check if the beam is under-reinforced or over-reinforced.

xu, max = 0.48 (600) = 288 mm

ε c (d - xu, max ) 0.0035 (600 - 288)


ε st = = = 0.00379
xu, max 288

Version 2 CE IIT, Kharagpur


fy 415
Yield strain of Fe 415 = + 0.002 = + 0.002
1.15 ( E s ) (1.15) (2) (10 5 )

= 0.0038 > 0.00379.

Hence, the beam is over-reinforced.

Step 2: To determine Mu,lim and Ast,lim


(vide Eq. 4.2 of Lesson 8 and Table 3.1 of Lesson 5)

xu, max xu, max


M u , lim = 0.36 ( ) ( 1 - 0.42 ) b d 2 f ck
d d

= 0.36 (0.48) {1 - 0.42 (0.48)} (350) (600)2 (20) (10-6) kNm

= 347.67 kNm

From Table 3.1 of Lesson 5, for fck = 20 N/mm2 and fy = 415 N/mm2,

0.96 (350) (600)


Ast , lim = = 2016 mm 2
100

Step 3: To determine Ast2 and Asc


(vide Eqs.4.7 and 4.6 of Lesson 8)

Ast2 = Ast - Ast, lim = 2945 - 2016 = 929 mm2

The required Asc will have the compression force equal to the tensile force as
given by 929 mm2 of Ast2.

Ast 2 (0.87 f y )
So, Asc =
( f sc - f cc )

For fsc let us calculate εsc: (vide Eq. 4.9 of Lesson 8)

0.0035 ( xu, max - d' ) 0.0035 (288 - 60)


ε sc = = = 0.002771
xu, max 288

Table 4.1 of Lesson 8 gives:

(360.9 - 351.8) (0.002771 - 0.002760)


f sc = 351.8 + = 351.896 N/mm 2
(0.00380 − 0.00276)

Version 2 CE IIT, Kharagpur


929 (0.87) (415)
So, Asc = = 977.956 mm 2
{351.89 − 0.446 (20)}

Step 4: To determine Mu2, Mu and Ast


(Please refer to Eqs. 4.4 and 4.1 of Lesson 8)

M u 2 = Asc ( f sc - f cc ) (d - d' )

= 977.956 {351.896 - 0.446 (20)} (600 - 60) (10-6) kNm

= 181.12 kNm

Mu = Mu, lim + Mu2 = 347.67 + 181.12 = 528.79 kNm

Therefore, with Ast = Ast, lim + Ast2 = 2016 + 929 = 2945 mm2 the required
Asc = 977.956 mm2 (much less than the provided 1256 mm2). Hence, o.k.

4.9.3 Practice Questions and Problems with Answers


Q.1: Design a doubly reinforced beam (Fig. 4.9.3) to resist Mu = 375 kNm
when b = 250 mm, d = 500 mm, d' = 75 mm, fck = 30 N/mm2 and fy =
500 N/mm2, using (i) direct computation method and (ii) using table of SP-
16.

Version 2 CE IIT, Kharagpur


A.1: (A) Solution by direct computation method:

From the given data

xu ,max xu ,max
Mu, lim = 0.36 ( ) (1 − 0.42 ) b d 2 f ck
d d

= 0.36 (0.46) {1 - 0.42 (0.46)} (250) (500)2 (30) (10-6) kNm

= 250.51 kNm

Using the value of pt = 1.13 from Table 3.1 of Lesson 5 for fck = 30 N/mm2
and fy = 500 N/mm2,

1.13 (250) (500)


Ast, lim = = 1412.5 mm 2
100

Mu2 = 375 - 250.51 = 124.49 kNm

From Table 4.2 of Lesson 8, for d'/d = 75/500 = 0.15 and fy = 500 N/mm2
, we get fsc = 395 N/mm2

M u2 124.49 (10 6 )
Asc = = = 767.56 mm 2
( f sc - f cc ) (d - d' ) {395 − 0.446 (30)} (500 - 75)

Asc ( f sc - f cc ) 767.56 {395 - 0.446 (30)}


Ast 2 = = = 673.37 mm 2
0.87 f y 0.87 (500)

Ast = Ast ,lim + Ast 2 = 1412.5 + 673.37 = 2085.87 mm 2

Alternatively: (use of Table 4.1 of Lesson 8 to determine fsc from εsc)

xu, max = 0.46 (500) = 230 mm

0.0035 (230 - 75) 0.0035 (155)


ε sc = = = 0.002359
230 230

From Table 4.1

Version 2 CE IIT, Kharagpur


(413.0 - 391.3) (0.002359 - 0.00226)
f sc = 391.3 + = 395.512 N/mm 2
(0.00277 - 0.00226)

M u2 124.49 (10 6 )
Asc = = = 766.53 mm 2
( f sc - f cc ) (d - d' ) {395.512 − 0.446 (30)} (500 - 75)

Asc ( f sc - f cc ) 766.53 (382.132)


Ast 2 = = = 673.369 mm 2
0.87 f y 0.87 (500)

Ast = Ast , lim + Ast 2 = 1412.5 + 673.369 = 2085.869 mm 2

Check for minimum and maximum Ast and Asc

0.85 b d 0.85 (250) (500)


(i) Minimum Ast = = 212.5 mm 2
fy 500

(ii) Maximum Ast = 0.04 b D = 0.04 (250) (575) = 5750 mm 2

0.2 b D 0.2 (250) (575)


(iii) Minimum Ast = = 287.5 mm 2
100 100

(iv) Maximum Ast = 0.04 b D = 0.04 (250) (575) = 5750 mm 2

Hence, the areas of reinforcement satisfy the requirements.

So, provide (i) 6-20 T + 2-12 T = 1885 + 226 = 2111 mm2 for Ast

(ii) 4-16 T = 804 mm2 for Asc

(B) Solution by use of table of SP-16

From the given data, we have

Mu 375 (10 6 )
= = 6.0
b d2 250 (500) 2

Version 2 CE IIT, Kharagpur


d'/d = 75/500 = 0.15

Table 56 of SP-16 gives: pt = 1.676 and pc = 0.619

(1.676) (250) (500)


So, Ast = = 2095 mm 2
100

(0.619 (250) (500)


and Asc = = 773.75 mm 2
100

These values are close to those of (A). Hence, provide 6-20 T + 2-12 T as
Ast and 4-16 T as Asc.

Q.2: Determine the moment of resistance of the doubly reinforced beam (Fig.
4.9.4) with b = 300 mm, d = 600 mm, d' = 90 mm, fck = 30 N/mm2, fy =
500 N/mm2, Asc = 2236 mm2 (2-32 T + 2-20 T), and Ast = 4021 mm2 (4-
32 T + 4-16 T). Use (i) direct computation method and (ii) tables of SP-16.

A.2: (i) Solution by direct computation method:

xu, max = 0.46 (600) = 276 mm

Version 2 CE IIT, Kharagpur


0.0035 (600 - 276)
ε st = = 0.0041086
276

εyield = 0.00417. So εst < εyield i.e. the beam is over-reinforced.

For d'/d = 0.15 and fy = 500 N/mm2, Table 4.2 of Lesson 8 gives: fsc = 395
N/mm2 and with fck = 30 N/mm2 , Table 3.1 of Lesson 5 gives pt, lim =
1.13.

1.13 (300) (600)


Ast ,lim = = 2034 mm 2
100

xu ,max xu ,max
Mu,lim = 0.36 ( ) (1 − 0.42 ) b d 2 f ck
d d

= 0.36 (0.46) {1 - 0.42 (0.46)} (300) (600)2 (30) (10-6) kNm

= 432.88 kNm

Ast2 = 4021 - 2034 = 1987 mm2

Ast 2 (0.87) f y 1987 (0.87) (500)


( Asc ) required = = = 2264.94 mm 2 > 2236 mm 2
( f sc - f cc ) {395 − 0.446 (30)}

So, Ast2 of 1987 mm2 is not fully used. Let us determine Ast2 required
when Asc = 2236 mm2.

Asc ( f sc - f cc ) 2236 {395 - 0.446 (30)}


Ast 2 = = = 1961.61 mm 2
0.87 f y (0.87) (500)

Ast = Ast, lim + Ast2 = 2034 + 1961.61 = 3995.61 mm2 < 4021 mm2.

Hence, o.k.

With Ast2 = 1961.61 mm2, Mu2 = Ast2 (0.87 fy ) (d - d')

= 1961.61 (0.87) (500) (600 - 75) (10-6) kNm = 447.98268 kNm

Again, when Asc = 2236 mm2 (as provided)

Version 2 CE IIT, Kharagpur


M u 2 = Asc ( f sc - f cc ) (d - d' )

= 2236 {395 - 0.446 (30)} (600 - 75) (10-6) kNm = 447.9837 kNm

Mu = Mu, lim + Mu2 = 432.88 + 447.98 (Mu2 is taken the lower of the
two)

= 880.86 kNm

Hence, the moment of resistance of the beam is 880.86 kNm.

Alternatively fsc can be determined from Table 4.1 of Lesson 8.

Using the following from the above:

xu, max = 276 mm

Ast,lim = 2034 mm2

Mu,lim = 432.88 kNm

Ast2 = 1987 mm2

To find (Asc)required

0.0035 (276 - 90)


ε st = = 0.00236
276

Table 4.1 of Lesson 8 gives:

(413 - 391.3) (0.00236 - 0.00226)


f sc = 391.3 + = 395.55 N/mm 2
(0.00277 - 0.00226)

Ast 2 (0.87) f y 1987 (0.87) (500)


( Asc ) required = =
( f sc - f cc ) {395.55 − 0.446 (30)}

= 2261.68 mm 2 > 2236 mm 2

So, it is not o.k.

Let us determine Ast2 required when Asc = 2236 mm2.

Version 2 CE IIT, Kharagpur


Asc ( f sc - f cc ) 2236 {395.55 - 0.446 (30)}
Ast 2 = = = 1964.44 mm 2
0.87 f y (0.87) (500)

Ast = Ast, lim + Ast2 = 2034 + 1964.44 = 3998.44 mm2 < 4021 mm2.

So, o.k.

Mu2 (when Ast2 = 1964.44 mm2) = Ast2 (0.87 fy ) (d - d')

= 1964.44 (0.87) (500) (600 - 75) (10-6) kNm

= 448.63 kNm

For Asc = 2236 mm2,

M u 2 = Asc ( f sc - f cc ) (d - d' )

= 2236 {(395.55 - 0.446 (30)} (600 - 75) (10-6) kNm

= 2236 (382.17) (525) (10-6) kNm

= 448.63 kNm

Both the Mu2 values are the same. So,

Mu = Mu,lim + Mu2 = 432.88 + 448.63

= 881.51 kNm

Here, the Mu = 881.51 kNm.

(ii) Solution by using table of SP-16

From the given data:

4021 (100)
pt = = 2.234
300 (600)

2236 (100)
pc = = 1.242
300 (600)

d'/d = 0.15

Version 2 CE IIT, Kharagpur


Table 56 of SP-16 is used first considering d'/d = 0.15 and pt = 2.234,
and secondly, considering d'/d = 0.15 and pc = 1.242. The calculated
values of pc and Mu/bd2 for the first and pt and Mu/bd2 for the second
cases are presented below separately. Linear interpolation has been
done.

(i) When d'/d = 0.15 and pt = 2.234

Mu (8.1 - 8.0) (2.234 - 2.218)


= 8.00 + = 8.06
bd 2
(2.245 - 2.218)

(1.266 - 1.235) (0.016)


p c = 1.235 + = 1.253 > 1.242
(0.027)

So, this is not possible.

(ii) When d'/d = 0.15 and pc = 1.242

Mu (8.1 - 8.0) (1.242 - 1.235)


= 8.00 + = 8.022
bd 2
(1.266 - 1.235)

(2.245 - 2.218) (1.242 - 1.235)


pt = 2.218 + = 2.224 < 2.234
(1.266 − 1.235)

So, Mu = 8.022 (300) (600)2 (10-6)= 866.376 kNm.

Hence, o.k.

4.9.4 References
1. Reinforced Concrete Limit State Design, 6th Edition, by Ashok K. Jain,
Nem Chand & Bros, Roorkee, 2002.
2. Limit State Design of Reinforced Concrete, 2nd Edition, by P.C.Varghese,
Prentice-Hall of India Pvt. Ltd., New Delhi, 2002.
3. Advanced Reinforced Concrete Design, by P.C.Varghese, Prentice-Hall of
India Pvt. Ltd., New Delhi, 2001.
4. Reinforced Concrete Design, 2nd Edition, by S.Unnikrishna Pillai and
Devdas Menon, Tata McGraw-Hill Publishing Company Limited, New
Delhi, 2003.

Version 2 CE IIT, Kharagpur


5. Limit State Design of Reinforced Concrete Structures, by P.Dayaratnam,
Oxford & I.B.H. Publishing Company Pvt. Ltd., New Delhi, 2004.
6. Reinforced Concrete Design, 1st Revised Edition, by S.N.Sinha, Tata
McGraw-Hill Publishing Company. New Delhi, 1990.
7. Reinforced Concrete, 6th Edition, by S.K.Mallick and A.P.Gupta, Oxford &
IBH Publishing Co. Pvt. Ltd. New Delhi, 1996.
8. Behaviour, Analysis & Design of Reinforced Concrete Structural Elements,
by I.C.Syal and R.K.Ummat, A.H.Wheeler & Co. Ltd., Allahabad, 1989.
9. Reinforced Concrete Structures, 3rd Edition, by I.C.Syal and A.K.Goel,
A.H.Wheeler & Co. Ltd., Allahabad, 1992.
10. Textbook of R.C.C, by G.S.Birdie and J.S.Birdie, Wiley Eastern Limited,
New Delhi, 1993.
11. Design of Concrete Structures, 13th Edition, by Arthur H. Nilson, David
Darwin and Charles W. Dolan, Tata McGraw-Hill Publishing Company
Limited, New Delhi, 2004.
12. Concrete Technology, by A.M.Neville and J.J.Brooks, ELBS with
Longman, 1994.
13. Properties of Concrete, 4th Edition, 1st Indian reprint, by A.M.Neville,
Longman, 2000.
14. Reinforced Concrete Designer’s Handbook, 10th Edition, by C.E.Reynolds
and J.C.Steedman, E & FN SPON, London, 1997.
15. Indian Standard Plain and Reinforced Concrete – Code of Practice (4th
Revision), IS 456: 2000, BIS, New Delhi.
16. Design Aids for Reinforced Concrete to IS: 456 – 1978, BIS, New Delhi.

4.9.5 Test 9 with Solutions


Maximum Marks = 50, Maximum Time = 30 minutes

Answer all questions.

TQ.1: Design a simply supported beam of effective span 8 m subjected to


imposed loads of 35 kN/m. The beam dimensions and other data are: b =
300 mm, D = 700 mm, M 20 concrete, Fe 415 steel (Fig. 4.9.1).
Determine fsc from strain εsc as given in Table 4.1 of Lesson 8.

A.TQ.1: This problem is the same as Problem 4.1 in sec. 4.9.2.1 except that here
the fsc is to be calculated using Table 4.1 instead of Table 4.2.

Step 1: Here, the Step 1 will remain the same as that of Problem 4.1.

Step 2: Determination of Mu2, Asc, Ast2 and Ast

M u 2 = M u - M u, lim = 482.96 - 328.55 = 154.41 kNm

Version 2 CE IIT, Kharagpur


From strain triangle: (Fig. 4.8.2 of Lesson 8)

0.0035 (302.4 - 70)


ε sc = = 0.00269
302.4

(351.8 - 342.8)
f sc (from Table 4.1 of Lesson 8) = 342.8 + (0.00269 - 0.00241)
(0.00276 - 0.00241)

= 350 N/mm 2

M u2 154.41 (10 6 )
Asc = = = 808.41 mm 2
( f sc - f cc ) (d - d' ) {350 − 0.446 (20)} (630 - 70) N/mm

Asc ( f sc - f cc ) 808.41 (341.08)


Ast 2 = = = 763.696 mm 2
0.87 f y (0.87) (415)

Ast = Ast1 + Ast 2 = 1809 .14 + 763.696 = 2572 .836 mm 2

Asc = 808.41 mm2

Steps 3 & 4 will also remain the same as those of Problem 4.1.

Hence, provide 2-20 T + 2-12 T (854 mm2) as Asc and 4-25 T + 2-20 T (2591
mm2) as Ast .

TQ.2: Determine the ultimate moment capacity of the doubly reinforced beam of
b = 350 mm, d' = 60 mm, d = 600 mm, Ast = 2945 mm2 (6-25 T), Asc =
1256 mm2 (4-20 T), using M 20 and Fe 415 (Fig. 4.9.2). Use table of SP-16
only.

A.TQ.2: Solution by using table of SP-16

This problem is the same as that of Problem 4.2 of sec. 4.9.2.2, which has
been solved by direct computation method. Here, the same is to be solved
by using SP-16.

The needed parameters are:

d'/d = 60/600 = 0.1

Ast (100) 2945 (100)


pt = = = 1.402
bd 350(600)

Version 2 CE IIT, Kharagpur


Asc (100) 1256 (100)
pc = = = 0.5981
bd 350 (600)

Here, we need to use Table 50 for fck = 20 N/mm2 and fy = 415 N/mm2. The
table gives values of Mu/bd2 for (I) d'/d and pt and (ii) d'/d and pc. So, we
will consider both the possibilities and determine Mu.

(i) Considering Table 50 of SP-16 when d'/d = 0.1 and pt = 1.402:

Interpolating the values of Mu/bd2 at pt = 1.399 and 1.429, we get

⎛ Mu ⎞ (4.3 - 4.2) (1.402 - 1.399)


⎜ ⎟ = 4.2 + = 4.21
⎝ b d ⎠ pt = 1.402
2
(1.429 - 1.399)

the corresponding ( pc ) pt = 1.402 = 0.466 +


(0.498 - 0.466) (1.402 - 1.399)
= 0.4692
(1.429 − 1.399)

But, pc provided is 0.5981 indicates that extra compression reinforcement has


been used.

So, we get

M u = 4.21 b d 2 = (4.21) (350) (600) 2 (10 -6 ) = 530.46 kNm when Ast =


2 2 2
2945 mm and Asc = 985.32 mm , i.e. 270.69 mm (= 1256 - 985.32) of
compression steel is extra.

(ii) Considering d'/d = 0.1 and pc = 0.5981, we get by linear interpolation

⎛ Mu ⎞ (4.7 - 4.6) (0.5981 - 0.595)


⎜ ⎟ = 4.6 + = 4.61
⎝ b d ⎠ pc = 0.5981
2
(0.628 - 0.595)

the corresponding pt is:

( pt ) pc = 0.5981 = 1.522 +
(1.533 - 1.522) (0.5981 - 0.595)
= 1.5231
(0.628 − 0.595)

The provided pt = 1.402 indicates that the tension steel is insufficient by 254.31
mm2 as shown below:

Amount of additional Ast still required =

Version 2 CE IIT, Kharagpur


(1.5231 - 1.402) (350) (600)
= 254.31 mm 2
100

If this additional steel is provided, then the Mu of this beam becomes:

Mu = 4.61 b d2 = 4.61 (350) (600)2 (10-6) kNm = 580.86 kNm

The above two results show that the moment of resistance of this beam is
the lower of the two. So, Mu = 530.46 kNm. By direct computation the Mu =
528.79 kNm. The two results are in good agreement.

4.9.6 Summary of this Lesson


This lesson presents solutions of four numerical problems covering both
design and analysis types. These problems are solved by two methods: (i) direct
computation method and (ii) using table of SP-16. Two problems are illustrated
in the lesson and the other two are given in the practice problem and test of this
lesson. The solutions will help in understanding the step by step application of
the theory of doubly reinforced beams given in Lesson 8.

Version 2 CE IIT, Kharagpur


Module
5
Flanged Beams – Theory
and Numerical
Problems

Version 2 CE IIT, Kharagpur


Lesson
10
Flanged Beams – Theory
Version 2 CE IIT, Kharagpur
Instructional Objectives:
At the end of this lesson, the student should be able to:

• identify the regions where the beam shall be designed as a flanged and
where it will be rectangular in normal slab beam construction,

• define the effective and actual widths of flanged beams,

• state the requirements so that the slab part is effectively coupled with the
flanged beam,

• write the expressions of effective widths of T and L-beams both for


continuous and isolated cases,

• derive the expressions of C, T and Mu for four different cases depending


on the location of the neutral axis and depth of the flange.

5.10.1 Introduction

Version 2 CE IIT, Kharagpur


Reinforced concrete slabs used in floors, roofs and decks are mostly cast
monolithic from the bottom of the beam to the top of the slab. Such rectangular
beams having slab on top are different from others having either no slab
(bracings of elevated tanks, lintels etc.) or having disconnected slabs as in some
pre-cast systems (Figs. 5.10.1 a, b and c). Due to monolithic casting, beams and
a part of the slab act together. Under the action of positive bending moment, i.e.,
between the supports of a continuous beam, the slab, up to a certain width
greater than the width of the beam, forms the top part of the beam. Such beams
having slab on top of the rectangular rib are designated as the flanged beams -
either T or L type depending on whether the slab is on both sides or on one side
of the beam (Figs. 5.10.2 a to e). Over the supports of a continuous beam, the
bending moment is negative and the slab, therefore, is in tension while a part of
the rectangular beam (rib) is in compression. The continuous beam at support is
thus equivalent to a rectangular beam (Figs. 5.10.2 a, c, f and g).

Version 2 CE IIT, Kharagpur


Version 2 CE IIT, Kharagpur
Version 2 CE IIT, Kharagpur
The actual width of the flange is the spacing of the beam, which is the
same as the distance between the middle points of the adjacent spans of the
slab, as shown in Fig. 5.10.2 b. However, in a flanged beam, a part of the width
less than the actual width, is effective to be considered as a part of the beam.
This width of the slab is designated as the effective width of the flange.

5.10.2 Effective Width

Version 2 CE IIT, Kharagpur


5.10.2.1 IS code requirements

The following requirements (cl. 23.1.1 of IS 456) are to be satisfied to


ensure the combined action of the part of the slab and the rib (rectangular part of
the beam).

(a) The slab and the rectangular beam shall be cast integrally or they shall be
effectively bonded in any other manner.

(b) Slabs must be provided with the transverse reinforcement of at least 60 per
cent of the main reinforcement at the mid span of the slab if the main
reinforcement of the slab is parallel to the transverse beam (Figs. 5.10.3 a and
b).

The variation of compressive stress (Fig. 5.10.4) along the actual width of
the flange shows that the compressive stress is more in the flange just above the
rib than the same at some distance away from it. The nature of variation is
complex and, therefore, the concept of effective width has been introduced. The
effective width is a convenient hypothetical width of the flange over which the
compressive stress is assumed to be uniform to give the same compressive

Version 2 CE IIT, Kharagpur


force as it would have been in case of the actual width with the true variation of
compressive stress.

5.10.2.2 IS code specifications

Clause 23.1.2 of IS 456 specifies the following effective widths of T and


L-beams:

(a) For T-beams, the lesser of

(i) bf = lo/6 + bw + 6 Df

(ii) bf = Actual width of the flange

(b) For isolated T-beams, the lesser of

lo
(i) bf = + bw
(l o /b) + 4

(ii) bf = Actual width of the flange

(c) For L-beams, the lesser of

(i) bf = lo/12 + bw + 3 Df

(ii) bf = Actual width of the flange

(d) For isolated L-beams, the lesser of

0.5 l o
(i) bf = + bw
(l o /b) + 4

(ii) bf = Actual width of the flange

where bf = effective width of the flange,

lo = distance between points of zero moments in the beam, which is the


effective span for simply supported beams and 0.7 times the effective
span for continuous beams and frames,

bw = beadth of the web,

Df = thickness of the flange,

and b = actual width of the flange.

Version 2 CE IIT, Kharagpur


5.10.3 Four Different Cases
The neutral axis of a flanged beam may be either in the flange or in the
web depending on the physical dimensions of the effective width of flange bf,
effective width of web bw, thickness of flange Df and effective depth of flanged
beam d (Fig. 5.10.4). The flanged beam may be considered as a rectangular
beam of width bf and effective depth d if the neutral axis is in the flange as the
concrete in tension is ignored. However, if the neutral axis is in the web, the
compression is taken by the flange and a part of the web.

All the assumptions made in sec. 3.4.2 of Lesson 4 are also applicable for
the flanged beams. As explained in Lesson 4, the compressive stress remains
constant between the strains of 0.002 and 0.0035. It is important to find the depth
h of the beam where the strain is 0.002 (Fig. 5.10.5 b). If it is located in the web,
the whole of flange will be under the constant stress level of 0.446 fck. The

Version 2 CE IIT, Kharagpur


following gives the relation of Df and d to facilitate the determination of the
depth h where the strain will be 0.002.

From the strain diagram of Fig. 5.10.5 b:

0.002 x -h
= u
0.0035 xu

h 3
or = = 0.43
xu 7
(5.1)

when xu = xu , max , we get

3
h = xu , max = 0.227 d , 0.205 d and 0.197 d , for Fe 250, Fe 415 and Fe
7
500, respectively. In general, we can adopt, say

h/d = 0.2 (5.2)

The same relation is obtained below from the values of strains of concrete and
steel of Fig. 5.10.5 b.

ε st d - xu
=
εc xu

d ε + εc
or = st (5.3)
xu εc

Dividing Eq. 5.1 by Eq. 5.3

h 0.0015
= (5.4)
d ε st + 0.0035

Using ε st = (0.87 f y / Es ) + 0.002 in Eq. 5.4, we get h/d = 0.227, 0.205 and
0.197 for Fe 250, Fe 415 and Fe 500 respectively, and we can adopt h/d = 0.2
(as in Eq. 5.2).

Thus, we get the same Eq. 5.2 from Eq. 5.4,

h/d = 0.2 (5.2)

Version 2 CE IIT, Kharagpur


It is now clear that the three values of h are around 0.2 d for the three
grades of steel. The maximum value of h may be Df, at the bottom of the flange
where the strain will be 0.002, if Df /d = 0.2. This reveals that the thickness of the
flange may be considered small if Df /d does not exceed 0.2 and in that case, the
position of the fibre of 0.002 strain will be in the web and the entire flange will be
under a constant compressive stress of 0.446 fck .

On the other hand, if Df is > 0.2 d, the position of the fibre of 0.002 strain
will be in the flange. In that case, a part of the slab will have the constant stress
of 0.446 fck where the strain will be more than 0.002.

Thus, in the balanced and over-reinforced flanged beams (when


xu = xu , max ), the ratio of Df /d is important to determine if the rectangular stress
block is for the full depth of the flange (when Df /d does not exceed 0.2) of for a
part of the flange (when Df /d > 0.2). Similarly, for the under-reinforced flanged
beams, the ratio of Df /xu is considered in place of Df /d. If Df /xu does not exceed
0.43 (see Eq. 5.1), the constant stress block is for the full depth of the flange. If
Df /xu > 0.43, the constant stress block is for a part of the depth of the flange.

Based on the above discussion, the four cases of flanged beams are as
follows:

Version 2 CE IIT, Kharagpur


(i) Neutral axis is in the flange (xu < Df ), (Fig. 5.10.6 a to c)

Version 2 CE IIT, Kharagpur


Version 2 CE IIT, Kharagpur
(ii) Neutral axis is in the web and the section is balanced (xu = xu,max >
Df), (Figs. 5.10.7 and 8 a to e)

It has two situations: (a) when Df /d does not exceed 0.2, the
constant stress block is for the entire depth of the flange (Fig. 5.10.7), and
(b) when Df /d > 0.2, the constant stress block is for a part of the depth of
flange (Fig. 5.10.8).

Version 2 CE IIT, Kharagpur


Version 2 CE IIT, Kharagpur
(iii) Neutral axis is in the web and the section is under-reinforced (xu,max
> xu > Df), (Figs. 5.10.9 and 10 a to e)

Version 2 CE IIT, Kharagpur


This has two situations: (a) when Df /xu does not exceed 0.43, the
full depth of flange is having the constant stress (Fig. 5.10.9), and (b)
when Df /xu > 0.43, the constant stress is for a part of the depth of flange
(Fig. 5.10.10).

(iv) Neutral axis is in the web and the section is over-reinforced (xu >
xu,max> Df), (Figs. 5.10.7 and 8 a to e)

As mentioned earlier, the value of xu is then taken as xu,max when


xu> xu,max. Therefore, this case also will have two situations depending on
Df /d not exceeding 0.2 or > 0.2 as in (ii) above. The governing
equations of the four different cases are now taken up.

5.10.4 Governing Equations


The following equations are only for the singly reinforced T-beams.
Additional terms involving Mu,lim, Mu2, Asc , Ast1 and Ast2 are to be included from
Eqs. 4.1 to 4.8 of sec. 4.8.3 of Lesson 8 depending on the particular case.
Applications of these terms are explained through the solutions of numerical
problems of doubly reinforced T-beams in Lessons 11 and 12.

5.10.4.1 Case (i): When the neutral axis is in the flange (xu < Df ), (Figs.
5.10.6 a to c)

Concrete below the neutral axis is in tension and is ignored. The steel
reinforcement takes the tensile force (Fig. 5.10.6). Therefore, T and L-beams are
considered as rectangular beams of width bf and effective depth d. All the
equations of singly and doubly reinforced rectangular beams derived in Lessons
4 to 5 and 8 respectively, are also applicable here.

5.10.4.2 Case (ii): When the neutral axis is in the web and the section is
balanced (xu,max > Df ), (Figs. 5.10.7 and 8 a to e)

(a) When Df /d does not exceed 0.2, (Figs. 5.10.7 a to e)

As explained in sec. 5.10.3, the depth of the rectangular portion of the


stress block (of constant stress = 0.446 fck) in this case is greater than Df (Figs.
5.10.7 a, b and c). The section is split into two parts: (i) rectangular web of width
bw and effective depth d, and (ii) flange of width (bf - bw) and depth Df (Figs.
5.10.7 d and e).

Total compressive force = Compressive force of rectangular beam of width bw


and depth d + Compressive force of rectangular
flange of width (bf - bw) and depth Df .

Thus, total compressive force

Version 2 CE IIT, Kharagpur


C = 0.36 fck bw xu, max + 0.45 fck (bf - bw) Df (5.5)

(Assuming the constant stress of concrete in the flange as 0.45 fck in place of
0.446 fck ,as per G-2.2 of IS 456), and the tensile force

T = 0.87 fy Ast (5.6)

The lever arm of the rectangular beam (web part) is (d - 0.42 xu, max) and the
same for the flanged part is (d - 0.5 Df ).

So, the total moment = Moment due to rectangular web part + Moment due to
rectangular flange part

or Mu = 0.36 fck bw xu, max (d - 0.42 xu, max ) + 0.45 fck (bf - bw) Df (d - Df /2)

or Mu = 0.36(xu, max /d){1 - 0.42( xu, max/d)} fck bw d2 + 0.45 fck(bf - bw) Df(d - Df
/2)
(5.7)
Equation 5.7 is given in G-2.2 of IS 456.

(b) When Df /d > 0.2, (Figs. 5.10.8 a to e)

In this case, the depth of rectangular portion of stress block is within the
flange (Figs. 5.10.8 a, b and c). It is assumed that this depth of constant stress
(0.45 fck) is yf, where

yf = 0.15 xu, max + 0.65 Df, but not greater than Df


(5.8)

The above expression of yf is derived in sec. 5.10.4.5.

As in the previous case (ii a), when Df /d does not exceed 0.2, equations
of C, T and Mu are obtained from Eqs. 5.5, 6 and 7 by changing Df to yf. Thus,
we have (Figs. 5.10.8 d and e)

C = 0.36 fck bw xu, max + 0.45 fck (bf - bw) yf (5.9)

T = 0.87 fy Ast
(5.10)

The lever arm of the rectangular beam (web part) is (d - 0.42 xu, max ) and the
same for the flange part is (d - 0.5 yf ). Accordingly, the expression of Mu is as
follows:

Version 2 CE IIT, Kharagpur


Mu = 0.36(xu, max /d){1 - 0.42( xu, max/d)} fck bw d2 + 0.45 fck(bf - bw) yf(d - yf
/2)

(5.11)

5.10.4.3 Case (iii): When the neutral axis is in the web and the section is
under-reinforced (xu > Df ), (Figs. 5.10.9 and 10 a to e)

(a) When Df / xu does not exceed 0.43, (Figs. 5.10.9 a to e)

Since Df does not exceed 0.43 xu and h (depth of fibre where the strain
is 0.002) is at a depth of 0.43 xu, the entire flange will be under a constant stress
of 0.45 fck (Figs. 5.10.9 a, b and c). The equations of C, T and Mu can be
written in the same manner as in sec. 5.10.4.2, case (ii a). The final forms of the
equations are obtained from Eqs. 5.5, 6 and 7 by replacing xu, max by xu. Thus,
we have (Figs. 5.10.9 d and e)

C = 0.36 fck bw xu + 0.45 fck (bf - bw) Df


(5.12)

T = 0.87 fy Ast
(5.13)

Mu = 0.36(xu /d){1 - 0.42( xu /d)} fck bw d2 + 0.45 fck(bf - bw) Df (d - Df /2)


(5.14)

(b) When Df / xu > 0.43, (Figs. 5.10.10 a to e)

Since Df > 0.43 xu and h (depth of fibre where the strain is 0.002) is at
a depth of 0.43 xu, the part of the flange having the constant stress of 0.45 fck is
assumed as yf (Fig. 5.10.10 a, b and c). The expressions of yf , C, T and Mu
can be written from Eqs. 5.8, 9, 10 and 11 of sec. 5.10.4.2, case (ii b), by
replacing xu,max by xu. Thus, we have (Fig. 5.10.10 d and e)

yf = 0.15 xu + 0.65 Df, but not greater than Df


(5.15)

C = 0.36 fck bw xu + 0.45 fck (bf - bw) yf


(5.16)

T = 0.87 fy Ast
(5.17)

Mu = 0.36(xu /d){1 - 0.42( xu /d)} fck bw d2 + 0.45 fck(bf - bw) yf (d - yf /2)


(5.18)

Version 2 CE IIT, Kharagpur


5.10.4.4 Case (iv): When the neutral axis is in the web and the section is
over-reinforced (xu > Df ), (Figs. 5.10.7 and 8 a to e)

For the over-reinforced beam, the depth of neutral axis xu is more than
xu, max as in rectangular beams. However, xu is restricted up to xu,max. Therefore,
the corresponding expressions of C, T and Mu for the two situations (a) when
Df / d does not exceed 0.2 and (b) when Df / d > 0.2 are written from Eqs. 5.5
to 5.7 and 5.9 to 5.11, respectively of sec. 5.10.4.2 (Figs. 5.10.7 and 8). The
expression of yf for (b) is the same as that of Eq. 5.8.

(a) When Df /d does not exceed 0.2 (Figs. 5.10.7 a to e)

The equations are:

C = 0.36 fck bw xu, max + 0.45 fck (bf - bw) Df (5.5)

T = 0.87 fy Ast (5.6)

Mu = 0.36(xu, max /d){1 - 0.42( xu, max/d)} fck bw d2 + 0.45 fck(bf - bw) Df(d - Df
/2)
(5.7)

(b) When Df /d > 0.2 (Figs. 5.10.8 a to e)

yf = 0.15 xu, max + 0.65 Df, but not greater than Df


(5.8)

C = 0.36 fck bw xu, max + 0.45 fck (bf - bw) yf (5.9)

T = 0.87 fy Ast
(5.10)

Mu = 0.36(xu, max /d){1 - 0.42( xu, max/d)} fck bw d2 + 0.45 fck(bf - bw) yf(d - yf
/2)

(5.11)

It is clear from the above that the over-reinforced beam will not have
additional moment of resistance beyond that of the balanced one. Moreover, it
will prevent steel failure. It is, therefore, recommended either to re-design or to
go for doubly reinforced flanged beam than designing over-reinforced flanged
beam.

Version 2 CE IIT, Kharagpur


5.10.4.5 Derivation of the equation to determine yf , Eq. 5.8, Fig. 5.10.11

Whitney's stress block has been considered to derive Eq. 5.8. Figure
5.10.11 shows the two stress blocks of IS code and of Whitney.

yf = Depth of constant portion of the stress block when Df /d > 0.2. As yf


is a function of xu and Df and let us assume

yf = A xu + B Df
(5.19)

where A and B are to be determined from the following two conditions:

(i) yf = 0.43 xu , when Df = 0.43 xu


(5.20)

(ii) yf = 0.8 xu , when Df = xu


(5.21)

Using the conditions of Eqs. 5.20 and 21 in Eq. 5.19, we get A = 0.15 and B =
0.65. Thus, we have

yf = 0.15 xu + 0.65 Df
(5.8)

5.10.5 Practice Questions and Problems with Answers


Q.1: Why do we consider most of the beams as T or L-beams between the
supports and rectangular beams over the support of continuous span?

Version 2 CE IIT, Kharagpur


A.1: Sec. 5.10.1, first paragraph.

Q.2: Draw cross-section of a beam with top slab and show the actual width and
effective width of the T-beam.

A.2: Fig. 5.10.2 b.

Q.3: State the requirements with figures as per IS 456 which ensure the
combined action of the part of the slab and the rib of flanged beams.

A.3: Sec. 5.10.2.1(a) and (b), Figure 5.10.3 (a and b).

Q.4: Define “effective width” of flanged beams.

A.4: Effective width is an imaginary width of the flange over which the
compressive stress is assumed to be uniform to give the same
compressive force as it would have been in case of the actual width with
the true variation of compressive stress (Fig. 5.10.4 of text).

Q.5: Write the expressions of effective widths of T and L-beams and isolated
beams.

A.5: Sec. 5.10.2.2.

Q.6: Name the four different cases of flanged beams.

A.6: The four different cases are:

(i) When the neutral axis is in the flange (xu < Df) (discussed in sec.
5.10.4.1).

(ii) When the neutral axis is in the web and the section is balanced (xu,max >
Df). It has two situations: (a) when Df /d does not exceed 0.2 and (b)
when Df /d > 0.2 (discussed in sec. 5.10.4.2).

(iii) When the neutral axis is in the web and the section is under-reinforced
(xu,max > xu > Df). It has two situations: (a) when Df /xu does not exceed
0.43 and (b) when Df /xu > 0.43 (discussed in sec. 5.10.4.3).

(iv) When the neutral axis is in the web and the section is over-reinforced
(xu > xu,max> Df). It has two situations: (a) when Df /d does not exceed
0.2 and (b) when Df /d > 0.2 (discussed in sec. 5.10.4.4).

Q.7: (a) Derive the following equation:

yf = 0.15 xu,max + 0.65 Df

Version 2 CE IIT, Kharagpur


(b) State when this equation is to be used.

(c) What is the limiting value of yf ?

A.7: (a) For derivation of the equation, see sec. 5.10.4.5.

(b) This equation gives the depth of flange over which the stress is
constant at 0.45 fck (i.e. strain is more than 0.002) when the neutral axis is
in web. This occurs when Df /d > 0.2 for balanced beam and when Df /xu
> 0.43 for under-reinforced beams.

(c) Limiting value of yf is Df.

5.10.6 References
1. Reinforced Concrete Limit State Design, 6th Edition, by Ashok K. Jain,
Nem Chand & Bros, Roorkee, 2002.
2. Limit State Design of Reinforced Concrete, 2nd Edition, by P.C.Varghese,
Prentice-Hall of India Pvt. Ltd., New Delhi, 2002.
3. Advanced Reinforced Concrete Design, by P.C.Varghese, Prentice-Hall of
India Pvt. Ltd., New Delhi, 2001.
4. Reinforced Concrete Design, 2nd Edition, by S.Unnikrishna Pillai and
Devdas Menon, Tata McGraw-Hill Publishing Company Limited, New
Delhi, 2003.
5. Limit State Design of Reinforced Concrete Structures, by P.Dayaratnam,
Oxford & I.B.H. Publishing Company Pvt. Ltd., New Delhi, 2004.
6. Reinforced Concrete Design, 1st Revised Edition, by S.N.Sinha, Tata
McGraw-Hill Publishing Company. New Delhi, 1990.
7. Reinforced Concrete, 6th Edition, by S.K.Mallick and A.P.Gupta, Oxford &
IBH Publishing Co. Pvt. Ltd. New Delhi, 1996.
8. Behaviour, Analysis & Design of Reinforced Concrete Structural Elements,
by I.C.Syal and R.K.Ummat, A.H.Wheeler & Co. Ltd., Allahabad, 1989.
9. Reinforced Concrete Structures, 3rd Edition, by I.C.Syal and A.K.Goel,
A.H.Wheeler & Co. Ltd., Allahabad, 1992.
10. Textbook of R.C.C, by G.S.Birdie and J.S.Birdie, Wiley Eastern Limited,
New Delhi, 1993.
11. Design of Concrete Structures, 13th Edition, by Arthur H. Nilson, David
Darwin and Charles W. Dolan, Tata McGraw-Hill Publishing Company
Limited, New Delhi, 2004.
12. Concrete Technology, by A.M.Neville and J.J.Brooks, ELBS with
Longman, 1994.
13. Properties of Concrete, 4th Edition, 1st Indian reprint, by A.M.Neville,
Longman, 2000.
14. Reinforced Concrete Designer’s Handbook, 10th Edition, by C.E.Reynolds
and J.C.Steedman, E & FN SPON, London, 1997.

Version 2 CE IIT, Kharagpur


15. Indian Standard Plain and Reinforced Concrete – Code of Practice (4th
Revision), IS 456: 2000, BIS, New Delhi.
16. Design Aids for Reinforced Concrete to IS: 456 – 1978, BIS, New Delhi.

5.10.7 Test 10 with Solutions


Maximum Marks = 50, Maximum Time = 30 minutes

Answer all questions.

TQ.1: Why do we consider most of the beams as T or L- beams between the


supports and rectangular beams over the support of continuous span?
(5 marks)
A.TQ.1: Sec. 5.10.1, first paragraph.

TQ.2: Define “effective width” of flanged beams.


(5 marks)
A.TQ.2: Effective width is a convenient hypothetical width of the flange over
which the compressive stress is assumed to be uniform to give the same
compressive force as it would have been in case of the actual width with
the true variation of compressive stress (Fig. 5.10.4 of text).

TQ.3: State the requirements with figures as per IS 456 which ensure the
combined action of the part of the slab and the rib of flanged beams.
(10 marks)
A.TQ.3: Sec. 5.10.2.1(a) and (b), Figure 5.10.3 (a and b).

TQ.4: Write the expressions of effective widths of T and L-beams and isolated
beams.
(10 marks)
A.TQ.4: Sec. 5.10.2.2.

TQ.5: Name the four different cases of flanged beams.


(10 marks)
A.TQ.5: The four different cases are

(i) When the neutral axis is in the flange (xu < Df) (discussed in sec.
5.10.4.1).

(ii) When the neutral axis is in the web and the section is balanced. It has
two situations: (a) when Df /d does not exceed 0.2 and (b) when Df /d
> 0.2 (discussed in sec. 5.10.4.2).

Version 2 CE IIT, Kharagpur


(iii) When the neutral axis is in the web and the section is under-
reinforced. It has two situations: (a) when Df /xu does not exceed 0.43
and (b) when Df /xu > 0.43 (discussed in sec. 5.10.4.3).

(iv) When the neutral axis is in the web and the section is over-reinforced.
It has two situations: (a) when Df /d does not exceed 0.2 and (b) when
Df /d > 0.2 (discussed in sec. 5.10.4.4).

TQ.6: (a) Derive the following equation:

yf = 0.15 xu,max + 0.65 Df

(b) State when this equation is to be used.

(c) What is the limiting value of yf ?


(5 + 3 + 2 = 10 marks)
A.TQ.6: (a) For derivation of the equation, see sec. 5.10.4.5.

(b) This equation gives the depth of flange over which the stress is
constant at 0.45 fck (i.e. strain is more than 0.002) when the neutral axis is
in web. This occurs when Df /d > 0.2 for balanced beam and when Df /xu
> 0.43 for under-reinforced beams.

(c) Limiting value of yf is Df.

5.10.8 Summary of this Lesson


This lesson illustrates the practical situations when slabs are cast
integrally with the beams to form either T and L-beams or rectangular beams.
The concept of effective width of the slab to form a part of the beam has been
explained. The requirements as per IS 456 have been illustrated so that the
considered part of the slab may become effective as a beam. Expressions of
effective widths for different cases of T and L-beams are given. Four sets of
governing equations for determining C, T and Mu are derived for four different
cases. These equations form the basis of analysis and design of singly and
doubly reinforced T and L- beams.

Version 2 CE IIT, Kharagpur


Module
5
Flanged Beams – Theory
and Numerical
Problems

Version 2 CE IIT, Kharagpur


Lesson
11
Flanged Beams –
Numerical Problems
Version 2 CE IIT, Kharagpur
Instructional Objectives:
At the end of this lesson, the student should be able to:

• identify the two types of numerical problems – analysis and design types,
• apply the formulations to analyse the capacity of a flanged beam,
• determine the limiting moment of resistance quickly with the help of tables
of SP-16.

5.11.1 Introduction
Lesson 10 illustrates the governing equations of flanged beams. It is now
necessary to apply them for the solution of numerical problems. Two types of
numerical problems are possible: (i) Analysis and (ii) Design types. This lesson
explains the application of the theory of flanged beams for the analysis type of
problems. Moreover, use of tables of SP-16 has been illustrated to determine the
limiting moment of resistance of sections quickly for the three grades of steel.
Besides mentioning the different steps of the solution, numerical examples are
also taken up to explain their step-by-step solutions.

5.11.2 Analysis Type of Problems


The dimensions of the beam bf, bw, Df, d, D, grades of concrete and steel
and the amount of steel Ast are given. It is required to determine the moment of
resistance of the beam.

Step 1: To determine the depth of the neutral axis xu

The depth of the neutral axis is determined from the equation of


equilibrium C = T. However, the expression of C depends on the location of
neutral axis, Df /d and Df /xu parameters. Therefore, it is required to assume first
that the xu is in the flange. If this is not the case, the next step is to assume xu in
the web and the computed value of xu will indicate if the beam is under-
reinforced, balanced or over-reinforced.

Version 2 CE IIT, Kharagpur


Other steps:

After knowing if the section is under-reinforced, balanced or over-


reinforced, the respective parameter Df/d or Df/xu is computed for the under-
reinforced, balanced or over-reinforced beam. The respective expressions of C,
T and Mu, as established in Lesson 10, are then employed to determine their
values. Figure 5.11.1 illustrates the steps to be followed.

Version 2 CE IIT, Kharagpur


5.11.3 Numerical Problems (Analysis Type)

Ex.1: Determine the moment of resistance of the T-beam of Fig. 5.11.2. Given
data: bf = 1000 mm, Df = 100 mm, bw = 300 mm, cover = 50 mm, d = 450
mm and Ast = 1963 mm2 (4- 25 T). Use M 20 and Fe 415.

Step 1: To determine the depth of the neutral axis xu

Assuming xu in the flange and equating total compressive and tensile


forces from the expressions of C and T (Eq. 3.16 of Lesson 5) as the T-beam
can be treated as rectangular beam of width bf and effective depth d, we get:

0.87 f y Ast 0.87 (415) (1963)


xu = = = 98.44 mm < 100 mm
0.36 b f f ck 0.36 (1000) (20)

So, the assumption of xu in the flange is correct.

xu, max for the balanced rectangular beam = 0.48 d = 0.48 (450) = 216
mm.

It is under-reinforced since xu < xu,max.

Step 2: To determine C, T and Mu

From Eqs. 3.9 (using b = bf) and 3.14 of Lesson 4 for C and T and Eq.
3.23 of Lesson 5 for Mu, we have:

Version 2 CE IIT, Kharagpur


C = 0.36 bf xu fck (3.9)

= 0.36 (1000) (98.44) (20) = 708.77 kN

T = 0.87 fy Ast
(3.14)

= 0.87 (415) (1963) = 708.74 kN

Ast f y
M u = 0.87 f y Ast d (1 - ) (3.23)
f ck b f d
(1963) (415)
= 0.87 (415) (1963) (450) {1 - } = 290.06 kNm
(20) (1000) (450)

This problem belongs to the case (i) and is explained in sec. 5.10.4.1 of Lesson
10.

Ex.2: Determine Ast,lim and Mu,lim of the flanged beam of Fig. 5.11.3. Given
data are: bf = 1000 mm, Df = 100 mm, bw = 300 mm, cover = 50 mm and d
= 450 mm. Use M 20 and Fe 415.

Step 1: To determine Df/d ratio

For the limiting case xu = xu,max = 0.48 (450) = 216 mm > Df. The
ratio Df/d is computed.

Version 2 CE IIT, Kharagpur


Df/d = 100/450 = 0.222 > 0.2

Hence, it is a problem of case (ii b) and discussed in sec. 5.10.4.2 b of Lesson


10.

Step 2: Computations of yf , C and T

First, we have to compute yf from Eq.5.8 of Lesson 10 and then employ


Eqs. 5.9, 10 and 11 of Lesson 10 to determine C, T and Mu, respectively.

yf = 0.15 xu,max + 0.65 Df = 0.15 (216) + 0.65 (100) = 97.4 mm. (from
Eq. 5.8)

C = 0.36 fck bw xu,max + 0.45 fck (bf - bw) yf


(5.9)

= 0.36 (20) (300) (216) + 0.45 (20) (1000 - 300) (97.4) = 1,080.18 kN.

T = 0.87 fy Ast = 0.87 (415) Ast


(5.10)

Equating C and T, we have

(1080.18) (1000) N
Ast = = 2,991.77 mm 2
0.87 (415) N/mm 2

Provide 4-28 T (2463 mm2) + 3-16 T (603 mm2) = 3,066 mm2

Step 3: Computation of Mu

xu, max xu, max


Mu, lim = 0.36( ) {1 - 0.42( )} fck bw d 2
d d
+ 0.45 fck (bf - bw ) yf (d - y f /2) (5.11)
= 0.36 (0.48) {1 - 0.42 (0.48)} (20) (300) (450)2
+ 0.45 (20) (1000 - 300) (97.4) (450 - 97.4/2) = 413.87 kNm

Version 2 CE IIT, Kharagpur


Ex.3: Determine the moment of resistance of the beam of Fig. 5.11.4 when Ast
= 2,591 mm2 (4- 25 T and 2- 20 T). Other parameters are the same as those of
Ex.1: bf = 1,000 mm, Df = 100 mm, bw = 300 mm, cover = 50 mm and d =
450 mm. Use M 20 and Fe 415.

Step 1: To determine xu

Assuming xu to be in the flange and the beam is under-reinforced, we


have from Eq. 3.16 of Lesson 5:

0.87 f y Ast 0.87 (415) (2591)


xu = = = 129.93 mm > 100 mm
0.36 b f f ck 0.36 (1000) (20)

Since xu > Df, the neutral axis is in web. Here, Df/d = 100/450 = 0.222 > 0.2.
So, we have to substitute the term yf from Eq. 5.15 of Lesson 10, assuming Df /
xu > 0.43 in the equation of C = T from Eqs. 5.16 and 17 of sec. 5.10.4.3 b of
Lesson 10. Accordingly, we get:

0.36 fck bw xu + 0.45 fck (bf - bw) yf = 0.87 fy Ast

or 0.36 (20) (300) (xu) + 0.45 (20) (1000 - 300) {0.15 xu + 0.65 (100)}

= 0.87 (415) (2591)

or xu = 169.398 mm < 216 mm (xu,max = 0.48 xu = 216 mm)

Version 2 CE IIT, Kharagpur


So, the section is under-reinforced.

Step 2: To determine Mu

Df /xu = 100/169.398 = 0.590 > 0.43

This is the problem of case (iii b) of sec. 5.10.4.3 b. The corresponding equations
are Eq. 5.15 of Lesson 10 for yf and Eqs. 5.16 to 18 of Lesson 10 for C, T and
Mu, respectively. From Eq. 5.15 of Lesson 10, we have:

yf = 0.15 xu + 0.65 Df = 0.15 (169.398) + 0.65 (100) = 90.409 mm

From Eq. 5.18 of Lesson 10, we have

Mu = 0.36(xu /d){1 - 0.42( xu /d)} fck bw d2 + 0.45 fck(bf - bw) yf (d - yf /2)

or Mu = 0.36 (169.398/450) {1 - 0.42 (169.398/450)} (20) (300) (450) (450)

+ 0.45 (20) (1000 - 300) (90.409) (450 - 90.409/2)

= 138.62 + 230.56 = 369.18 kNm.

Ex.4: Determine the moment of resistance of the flanged beam of Fig. 5.11.5
with Ast = 4,825 mm2 (6- 32 T). Other parameters and data are the same as
those of Ex.1: bf = 1000 mm, Df = 100 mm, bw = 300 mm, cover = 50 mm
and d = 450 mm. Use M 20 and Fe 415.

Version 2 CE IIT, Kharagpur


Step 1: To determine xu

Assuming xu in the flange of under-reinforced rectangular beam we have


from Eq. 3.16 of Lesson 5:

0.87 f y Ast 0.87 (415) (4825)


xu = = = 241.95 mm > D f
0.36 b f f ck 0.36 (1000) (20)

Here, Df/d = 100/450 = 0.222 > 0.2. So, we have to determine yf from Eq.
5.15 and equating C and T from Eqs. 5.16 and 17 of Lesson 10.

yf = 0.15 xu + 0.65 Df (5.15)

0.36 fck bw xu + 0.45 fck (bf - bw) yf = 0.87 fy Ast (5.16 and
5.17)

or 0.36 (20) (300) (xu) + 0.45 (20) (1000 - 300) {0.15 xu + 0.65 (100)}

= 0.87 (415) (4825)

or 2160 xu + 945 xu = - 409500 + 1742066

or xu = 1332566/3105 = 429.17 mm

xu,max = 0.48 (450) = 216 mm

Since xu > xu,max, the beam is over-reinforced. Accordingly.

xu = xu, max = 216 mm.

Step 2: To determine Mu

This problem belongs to case (iv b), explained in sec.5.10.4.4 b of Lesson


10. So, we can determine Mu from Eq. 5.11 of Lesson 10.

Mu = 0.36(xu, max /d){1 - 0.42(xu, max /d)} fck bw d2 + 0.45 fck(bf - bw) yf (d - yf
/2)
(5.11)

where yf = 0.15 xu, max + 0.65 Df = 97.4 mm


(5.8)

From Eq. 5.11, employing the value of yf = 97.4 mm, we get:

Mu = 0.36 (0.48) {1 - 0.42 (0.48)} (20) (300) (450) (450)

Version 2 CE IIT, Kharagpur


+ 0.45 (20) (1000 - 300) (97.4) (450 - 97.4/2)

= 167.63 + 246.24 = 413.87 kNm

It is seen that this over-reinforced beam has the same Mu as that of the
balanced beam of Example 2.

5.11.4 Summary of Results of Examples 1-4


The results of four problems (Exs. 1-4) are given in Table 5.1 below. All
the examples are having the common data except Ast.

Table 5.1 Results of Examples 1-4 (Figs. 5.11.2 – 5.11.5)

Ex. Ast Case Section Mu Remarks


No. (mm2) No. (kNm)
1 1,963 (i) 5.10.4.1 290.06 xu = 98.44 mm < xu, max (= 216
mm),
xu < Df (= 100 mm),
Under-reinforced, (NA in the
flange).
2 3,066 (ii b) 5.10.4.2 413.87 xu = xu, max = 216 mm,
(b) Df /d = 0.222 > 0.2,
Balanced, (NA in web).
3 2,591 (iii b) 5.10.4.3 369.18 xu = 169.398 mm < xu, max(= 216
(b) mm),
Df /xu = 0.59 > 0.43,
Under-reinforced, (NA in the
web).
4 4,825 (iv b) 5.10.4.4 413.87 xu = 241.95 mm > xu, max (= 216
(b) mm),
Df /d = 0.222 > 0.2,
Over-reinforced, (NA in web).

It is clear from the above table (Table 5.1), that Ex.4 is an over-reinforced
flanged beam. The moment of resistance of this beam is the same as that of
balanced beam of Ex.2. Additional reinforcement of 1,759 mm2 (= 4,825 mm2 –
3,066 mm2) does not improve the Mu of the over-reinforced beam. It rather
prevents the beam from tension failure. That is why over-reinforced beams are to
be avoided. However, if the Mu has to be increased beyond 413.87 kNm, the
flanged beam may be doubly reinforced.

5.11.5 Use of SP-16 for the Analysis Type of Problems


Using the two governing parameters (bf /bw) and (Df /d), the Mu,lim of
balanced flanged beams can be determined from Tables 57-59 of SP-16 for the
Version 2 CE IIT, Kharagpur
three grades of steel (250, 415 and 500). The value of the moment coefficient
Mu,lim /bwd2fck of Ex.2, as obtained from SP-16, is presented in Table 5.2 making
linear interpolation for both the parameters, wherever needed. Mu,lim is then
calculated from the moment coefficient.

Table 5.2 Mu,lim of Example 2 using Table 58 of SP-16

Parameters: (i) bf /bw = 1000/300 = 3.33


(ii) Df /d = 100/450 = 0.222

(Mu,lim /bw d2 fck) in N/mm2


Df /d bf /bw
3 4 3.33
0.22 0.309 0.395
0.23 0.314 0.402
0.222 0.31* 0.3964* 0.339*
* by linear interpolation

M u , lim
So, from Table 5.2, = 0.339
bw d 2 f ck

Mu,lim = 0.339 bw d2 fck = 0.339 (300) (450) (450) (20) 10-6 = 411.88
kNm

Mu,lim as obtained from SP-16 is close to the earlier computed value of Mu,lim =
413.87 kNm (see Table 5.1).

5.11.6 Practice Questions and Problems with Answers

Version 2 CE IIT, Kharagpur


Q.1: Determine the moment of resistance of the simply supported doubly
reinforced flanged beam (isolated) of span 9 m as shown in Fig. 5.11.6.
Assume M 30 concrete and Fe 500 steel.

A.1: Solution of Q.1:

lo 9000
Effective width bf = + bw = + 300 = 1200 mm
(l o /b) + 4 (9000/1500) + 4

Step 1: To determine the depth of the neutral axis

Assuming neutral axis to be in the flange and writing the equation C = T,


we have:
0.87 fy Ast = 0.36 fck bf xu + (fsc Asc – fcc Asc)

Here, d ' / d = 65/600 = 0.108 = 0.1 (say). We, therefore, have fsc = 353
N/mm2 .

From the above equation, we have:

0.87 (500) (6509) - {(353) (1030) - 0.446 (30) (1030)}


xu = = 191.48 mm > 120 mm
0.36 (30) (1200)
So, the neutral axis is in web.

Df /d = 120/600 = 0.2

Assuming Df /xu < 0.43, and Equating C = T

0.87 fy Ast = 0.36 fck bw xu + 0.446 fck (bf – bw) Df + (fsc – fcc) Asc

0.87 (500) (6509) - 1030{353 - 0.446 (30)}- 0.446 (30) (1200 - 300) (120)
xu =
0.36 ( 30 ) ( 300 )
= 319.92 > 276 mm (xu ,max = 276 mm)

So, xu = xu,max = 276 mm (over-reinforced beam).

Df /xu = 120/276 = 0.4347 > 0.43

Let us assume Df /xu > 0.43. Now, equating C = T with yf as the depth of
flange having constant stress of 0.446 fck. So, we have:

yf = 0.15 xu + 0.65 Df = 0.15 xu + 78

0.36 fck bw xu + 0.446 fck (bf – bw) yf + Asc (fsc – fcc) = 0.87 fy Ast

Version 2 CE IIT, Kharagpur


0.36 (30) (300) xu + 0.446 (30) (900) (0.15 xu + 78)

= 0.87 (500) (6509) – 1030 {353 – 0.446 (30)}

or xu = 305.63 mm > xu,max. (xu,max = 276 mm)

The beam is over-reinforced. Hence, xu = xu,max = 276 mm. This is a problem of


case (iv), and we, therefore, consider the case (ii) to find out the moment of
resistance in two parts: first for the balanced singly reinforced beam and then for
the additional moment due to compression steel.

Step 2: Determination of xu,lim for singly reinforced flanged beam

Here, Df /d = 120/600 = 0.2, so yf is not needed. This is a problem of case (ii


a) of sec. 5.10.4.2 of Lesson 10. Employing Eq. 5.7 of Lesson 10, we have:

Mu,lim = 0.36 (xu,max /d) {1 – 0.42 (xu,max /d)} fck bw d2

+ 0.45 fck (bf – bw) Df (d – Df /2)

= 0.36(0.46) {1 – 0.42(0.46)} (30) (300) (600) (600)


+ 0.45(30) (900) (120) (540)

= 1,220.20 kNm

M u ,lim
Ast ,lim =
0.87 f y d {1 - 0.42 (xu,max / d )}
(1220.20) (106 )
= = 5,794.6152 mm 2
( 0.87 ) ( 500 ) ( 600 ) ( 0.8068 )

Step 3: Determination of Mu2

Total Ast = 6,509 mm2, Ast,lim = 5,794.62 mm2

Ast2 = 714.38 mm2 and Asc = 1,030 mm2

It is important to find out how much of the total Asc and Ast2 are required
effectively. From the equilibrium of C and T forces due to additional steel
(compressive and tensile), we have:

(Ast2) (0.87) (fy) = (Asc) (fsc)

If we assume Asc = 1,030 mm2

Version 2 CE IIT, Kharagpur


1030 (353)
Ast 2 = = 835.84 mm 2 > 714.38 mm 2 , (714.38 mm2 is the total
0.87 (500)
Ast2 provided). So, this is not possible.

Now, using Ast2 = 714.38 mm2 , we get Asc from the above equation.

(714.38) (0.87) (500)


Asc = = 880.326 < 1,030 mm 2 , (1,030 mm2 is
353
the total Asc provided).

M u 2 = Asc f sc (d - d ' ) = (880.326) (353) (600 - 60) = 167.807 kNm

Total moment of resistance = Mu,lim + Mu2 = 1,220.20 + 167.81 = 1,388.01


kNm

Total Ast required = Ast,lim + Ast2 = 5,794.62 + 714.38 = 6,509.00 mm2 ,


(provided Ast = 6,509 mm2)

Asc required = 880.326 mm2 (provided 1,030 mm2).

5.11.7 References
1. Reinforced Concrete Limit State Design, 6th Edition, by Ashok K. Jain,
Nem Chand & Bros, Roorkee, 2002.
2. Limit State Design of Reinforced Concrete, 2nd Edition, by P.C.Varghese,
Prentice-Hall of India Pvt. Ltd., New Delhi, 2002.
3. Advanced Reinforced Concrete Design, by P.C.Varghese, Prentice-Hall of
India Pvt. Ltd., New Delhi, 2001.
4. Reinforced Concrete Design, 2nd Edition, by S.Unnikrishna Pillai and
Devdas Menon, Tata McGraw-Hill Publishing Company Limited, New
Delhi, 2003.
5. Limit State Design of Reinforced Concrete Structures, by P.Dayaratnam,
Oxford & I.B.H. Publishing Company Pvt. Ltd., New Delhi, 2004.
6. Reinforced Concrete Design, 1st Revised Edition, by S.N.Sinha, Tata
McGraw-Hill Publishing Company. New Delhi, 1990.
7. Reinforced Concrete, 6th Edition, by S.K.Mallick and A.P.Gupta, Oxford &
IBH Publishing Co. Pvt. Ltd. New Delhi, 1996.
8. Behaviour, Analysis & Design of Reinforced Concrete Structural Elements,
by I.C.Syal and R.K.Ummat, A.H.Wheeler & Co. Ltd., Allahabad, 1989.
9. Reinforced Concrete Structures, 3rd Edition, by I.C.Syal and A.K.Goel,
A.H.Wheeler & Co. Ltd., Allahabad, 1992.
10. Textbook of R.C.C, by G.S.Birdie and J.S.Birdie, Wiley Eastern Limited,
New Delhi, 1993.

Version 2 CE IIT, Kharagpur


11. Design of Concrete Structures, 13th Edition, by Arthur H. Nilson, David
Darwin and Charles W. Dolan, Tata McGraw-Hill Publishing Company
Limited, New Delhi, 2004.
12. Concrete Technology, by A.M.Neville and J.J.Brooks, ELBS with
Longman, 1994.
13. Properties of Concrete, 4th Edition, 1st Indian reprint, by A.M.Neville,
Longman, 2000.
14. Reinforced Concrete Designer’s Handbook, 10th Edition, by C.E.Reynolds
and J.C.Steedman, E & FN SPON, London, 1997.
15. Indian Standard Plain and Reinforced Concrete – Code of Practice (4th
Revision), IS 456: 2000, BIS, New Delhi.
16. Design Aids for Reinforced Concrete to IS: 456 – 1978, BIS, New Delhi.

5.11.8 Test 11 with Solutions


Maximum Marks = 50, Maximum Time = 30 minutes

Answer all questions.

TQ.1: Determine Mu,lim of the flanged beam of Ex. 2 (Fig. 5.11.3) with the help of
SP-16 using (a) M 20 and Fe 250, (b) M 20 and Fe 500 and (c) compare the
results with the Mu,lim of Ex. 2 from Table 5.2 when grades of concrete and steel
are M 20 and Fe 415, respectively. Other data are: bf = 1000 mm, Df = 100
mm, bw = 300 mm, cover = 50 mm and d = 450 mm.
(10 X 3 = 30 marks)
A.TQ.1: From the results of Ex. 2 of sec. 5.11.5 (Table 5.2), we have:

Parameters: (i) bf /bw = 1000/300 = 3.33


(ii) Df /d = 100/450 = 0.222

For part (a): When Fe 250 is used, the corresponding table is Table 57 of SP-
16. The computations are presented in Table 5.3 below:

Table 5.3 (Mu,lim /bw d2 fck) in N/mm2 Of TQ.1 (PART a for M 20 and Fe 250)

(Mu,lim /bw d2 fck) in N/mm2


Df /d bf /bw
3 4 3.33
0.22 0.324 0.411
0.23 0.330 0.421
0.222 0.3252* 0.413* 0.354174*
• by linear interpolation

Mu,lim /bw d2 fck = 0.354174 = 0.354 (say)

Version 2 CE IIT, Kharagpur


So, Mu,lim = (0.354) (300) (450) (450) (20) N mm = 430.11 kNm

For part (b): When Fe 500 is used, the corresponding table is Table 59 of SP-
16. The computations are presented in Table 5.4 below:

Table 5.4 (Mu,lim /bw d2 fck) in N/mm2 Of TQ.1 (PART b for M 20 and Fe 500)

(Mu,lim /bw d2 fck) in N/mm2


Df /d bf /bw
3 4 3.33
0.22 0.302 0.386
0.23 0.306 0.393
0.222 0.3028* 0.3874* 0.330718*
* by linear interpolation

Mu,lim /bw d2 fck = 0.330718 = 0.3307 (say)

So, Mu,lim = (0.3307) (300) (450) (450) (20) mm = 401.8 kNm

For part (c): Comparison of results of this problem with that of Table 5.2 (M 20
and Fe
415) is given below in Table 5.5.

Table 5.5 Comparison of results of Mu,lim

Sl. Grade of Steel Mu,lim (kNm)


No.
1 Fe 250 430.11
2 Fe 415 411.88
3 Fe 500 401.80

It is seen that Mu,lim of the beam decreases with higher grade of steel for a
particular grade of concrete.

TQ.2: With the aid of SP-16, determine separately the limiting moments of
resistance and the limiting areas of steel of the simply supported isolated,
singly reinforced and balanced flanged beam of Q.1 as shown in Fig.
5.11.6 if the span = 9 m. Use M 30 concrete and three grades of steel, Fe
250, Fe 415 and Fe 500, respectively. Compare the results obtained
above with that of Q.1 of sec. 5.11.6, when balanced.
(15 + 5 = 20 marks)

A.TQ.2: From the results of Q.1 sec. 5.11.6, we have:

Parameters: (i) bf /bw = 1200/300 = 4.0


(ii) Df /d = 120/600 = 0.2

Version 2 CE IIT, Kharagpur


For Fe 250, Fe 415 and Fe 500, corresponding tables are Table 57, 58 and 59,
respectively of SP-16. The computations are done accordingly. After computing
the limiting moments of resistance, the limiting areas of steel are determined as
explained below. Finally, the results are presented in Table 5.6 below:

M u ,lim
Ast ,lim =
0.87 f y d {1 - 0.42 (xu,max / d )}

Table 5.6 Values of Mu,lim in N/mm2 Of TQ.2

Grade of Fe / Q.1 of (Mu,lim/bw d2 fck) Mu,lim (kNm) Ast,lim (mm2)


sec. 5.11.6 (N/mm2 )
Fe 250 0.39 1, 263.60 12,455.32
Fe 415 0.379 1, 227.96 7,099.78
Fe 500 0.372 1, 205.28 5,723.76
Q.1 of sec. 5.11.6 (Fe 1, 220.20 5,794.62
415)

The maximum area of steel allowed is .04 b D = (.04) (300) (660) = 7,920
2
mm . Hence, Fe 250 is not possible in this case.

5.11.9 Summary of this Lesson


This lesson mentions about the two types of numerical problems (i)
analysis and (ii) design types. In addition to explaining the steps involved in
solving the analysis type of numerical problems, several examples of analysis
type of problems are illustrated explaining all steps of the solutions both by direct
computation method and employing SP-16. Solutions of practice and test
problems will give readers the confidence in applying the theory explained in
Lesson 10 in solving the numerical problems.

Version 2 CE IIT, Kharagpur


Module
5
Flanged Beams – Theory
and Numerical
Problems

Version 2 CE IIT, Kharagpur


Lesson
12
Flanged Beams –
Numerical Problems
(Continued)
Version 2 CE IIT, Kharagpur
Instructional Objectives:
At the end of this lesson, the student should be able to:

• identify the two types of problems – analysis and design types,

• apply the formulations to design the flanged beams.

5.12.1 Introduction
Lesson 10 illustrates the governing equations of flanged beams and
Lesson 11 explains their applications for the solution of analysis type of
numerical problems. It is now necessary to apply them for the solution of design
type, the second type of the numerical problems. This lesson mentions the
different steps of the solution and solves several numerical examples to explain
their step-by-step solutions.

5.12.2 Design Type of Problems


We need to assume some preliminary dimensions of width and depth of
flanged beams, spacing of the beams and span for performing the structural
analysis before the design. Thus, the assumed data known for the design are:
Df, bw, D, effective span, effective depth, grades of concrete and steel and
imposed loads.

There are four equations: (i) expressions of compressive force C, (ii)


expression of the tension force T, (iii) C = T and (iv) expression of Mu in terms
of C or T and the lever arm {M = (C or T ) (lever arm)}. However, the relative
dimensions of Df, D and xu and the amount of steel (under-reinforced, balanced
or over-reinforced) influence the expressions. Accordingly, the respective
equations are to be employed assuming a particular situation and, if necessary,
they need to be changed if the assumed parameters are found to be not
satisfactory. The steps of the design problems are as given below.

Step 1: To determine the factored bending moment Mu

Step 2: To determine the Mu,lim of the given or the assumed section

The beam shall be designed as under-reinforced, balanced or doubly


reinforced if the value of Mu is less than, equal to or more than Mu,lim. The
design of over-reinforced beam is to be avoided as it does not increase the
bending moment carrying capacity beyond Mu,lim either by increasing the depth
or designing a doubly reinforced beam.

Version 2 CE IIT, Kharagpur


Step 3: To determine xu, the distance of the neutral axis, from the
expression of Mu

Here, it is necessary to assume first that xu is in the flange. Later on, it


may be necessary to calculate xu if the value is found to be more than Df . This is
to be done assuming first that Df /xu < 0.43 and then Df /xu > 0.43 separately.

Step 4: To determine the area(s) of steel

For doubly reinforced beams Ast = Ast,lim + Ast2 and Asc are to be
obtained, while only Ast is required to be computed for under-reinforced and
balanced beams. These are calculated employing C = T (for Ast and Ast, lim) and
the expression of Mu2 to calculate Ast2 and Asc.

Step 5: It may be necessary to check the xu and Ast once again after Step
4

It is difficult to prescribe all the relevant steps of design problems.


Decisions are to be taken judiciously depending on the type of problem. For the
design of a balanced beam, it is necessary to determine the effective depth in
Step 3 employing the expression of bending moment Mu. For such beams and
for under-reinforced beams, it may be necessary to estimate the Ast
approximately immediately after Step 2. This value of Ast will facilitate to
determine xu.

5.12.3 Numerical Problems


Four numerical examples are solved below explaining the steps involved
in the design problems.

Version 2 CE IIT, Kharagpur


Ex.5: Design the simply supported flanged beam of Fig. 5.12.1, given the
following: Df = 100 mm, D = 750 mm, bw = 350 mm, spacing of beams =
4000 mm c/c, effective span = 12 m, cover = 90 mm, d = 660 mm and
imposed loads = 5 kN/m2. Fe 415 and M 20 are used.

Solution:

Step 1: Computation of factored bending moment

Weight of slab per m2 = (0.1) (1) (1) (25) = 2.5 kN/m2


So, Weight of slab per m = (4) (2.5) = 10.00 kN/m
Dead loads of web part of the beam = (0.35) (0.65) (1) (25) = 5.6875
kN/m
Imposed loads = (4) (5) = 20 kN/m

Total loads = 30 + 5.6875 = 35.6875 kN/m

(35.6875) (12) (12)


Factored Bending moment = (1.5) = 963.5625 kNm
8

Step 2: Computation of xu,lim

Effective width of flange =(lo/6) + bw+ 6 Df = (12000/6) + 350 + 600 = 2,950


mm.

xu,max = 0.48 d = 0.48 (660) = 316.80 mm. This shows that the neutral
axis is in the web of this beam.

Df /d = 100/660 = 0.1515 < 0.2, and

Df /xu = 100/316.8 = 0.316 < 0.43

The expression of Mu,lim is obtained from Eq. 5.7 of Lesson 10 (case ii a of sec.
5.10.4.2) and is as follows:

Mu,lim = 0.36(xu,max /d){1 - 0.42 (xu,max /d)} fck bw d2 + 0.45 fck (bf - bw) Df (d -
Df /2)

= 0.36(0.48) {1 - 0.42(0.48)} (20) (350) (650) (650)

+ 0.45 (20) (2950 - 350) (100) (660 - 50) = 1,835.43 kNm

The design moment Mu = 963.5625 kNm is less than Mu,lim. Hence, one under-
reinforced beam can be designed.

Version 2 CE IIT, Kharagpur


Step 3: Determination of xu

Since the design moment Mu is almost 50% of Mu,lim, let us assume the neutral
axis to be in the flange. The area of steel is to be calculated from the moment
equation (Eq. 3.23 of Lesson 5), when steel is ensured to reach the design stress
fd = 0.87 (415) = 361.05 N/mm2. It is worth mentioning that the term b of Eq. 3.23
of Lesson 5 is here bf as the T- beam is treated as a rectangular beam when the
neutral axis is in the flange.

⎧ Ast f y ⎫
Mu = 0.87 f y Ast d ⎨1 − ⎬
⎩ f ck b d ⎭
(3.23)

Here, all but Ast are known. However, this will give a quadratic equation of Ast
and the lower one of the two values will be provided in the beam. The above
equation gives:

Ast2 - 93831.3253 Ast + 379416711.3 = 0

which gives the lower value of Ast as:

Ast = 4,234.722097 mm2. The reason of selecting the lower value of Ast
is explained in sec 3.6.4.8 of Lesson 6 in the solution of Design Problem 3.1.

Then, employing Eq. 3.16 of Lesson 5, we get

0.87 f y Ast
xu =
0.36 b f ck
(3.16)

or xu = 71.98 mm.

Again, employing Eq. 3.24 of Lesson 5, we can determine xu first and then Ast
from Eq. 3.16 or 17 of Lesson 5, as explained in the next step.

Eq. 3.24 of Lesson 5 gives:

Mu = 0.36(xu /d) {1 - 0.42(xu /d)} fck bf d2

= 0.36 (xu) {1 - 0.42 (xu /d)} fck bf d

963.5625 (106) = 0.36 (xu) {1 - 0.42 (xu /660)} (20) (2950) (660)

or xu = 72.03 mm.

Version 2 CE IIT, Kharagpur


The two values of xu are the same. It is thus seen that, the value of xu can be
determined either first finding the value of Ast, from Eq. 3.23 of Lesson 5 or
directly from Eq. 3.24 of Lesson 5 first and then the value of Ast can be
determined.

Step 4: Determination of Ast

Equating C = T, we have from Eq. 3.17 of Lesson 5:

xu 0.87 f y Ast
=
d 0.36 f ck b f d

0.36 f ck b f xu 0.36 (20) (2950) (72.03)


Ast = = = 4,237.41 mm 2
0.87 f y 0.87 (415)

Minimum Ast = (0.85/fy) bw d = (0.85/415) (350) (660) = 473.13 mm2

Maximum Ast = 0.04 bw D = (0.04) (350) (660) = 9,240 mm2

Hence, Ast = 4,237.41 mm2 is o.k.

Provide 6 - 28 T (= 3694 mm2) + 2-20 T (= 628 mm2) to have total Ast = 4,322
mm2.

Ex.6: Design a beam in place of the beam of Ex.5 (Fig. 5.12.1) if the imposed
loads are increased to 12 kN/m2. Other data are: Df = 100 mm, bw = 350 mm,
spacing of beams = 4000 mm c/c, effective span = 12 m simply supported and
cover = 90 mm. Use Fe 415 and M 20.

Solution: As in Ex.5, bf = 2,950 mm.

Step 1: Computation of factored bending moment

Weight of slab/m2 = 2.5 kN/m2 (as in Ex.1)

Imposed loads = 12.0 kN/m2 (given)

Total loads = 14.5 kN/m2

Total weight of slab and imposed loads = 14.5 (4) = 58.0 kN/m

Dead loads of the beam = 0.65 (0.35) (25) = 5.6875 kN/m

Total loads = 63.6875 kN/m

Version 2 CE IIT, Kharagpur


1.5 (63.6875) (12) (12)
(Mu)factored = = 1,719.5625 kNm
8

Step 2: Determination of Mu,lim

Mu,lim of the beam of Ex.5 = 1,835.43 kNm. The factored moment of this
problem (1,719.5625 kNm) is close to the value of Mu,lim of the section.

Step 3: Determination of d

Assuming Df /d < 0.2, we have from Eq. 5.7 of Lesson 10,

Mu = 0.36(xu,max /d){1 - 0.42 (xu,max /d)} fck bw d2 + 0.45 fck (bf - bw) Df (d - Df
/2)

1719.5625 (106) = 0.36(0.48) {1 - 0.42(0.48)} (20) (350) d2

+ 0.45 (2600) (20) (100) (d - 50)

Solving the above equation, we get d = 624.09 mm, giving total depth =
624.09 + 90 = 715 mm (say).

Since the dead load of the beam is reduced due to decreasing the depth
of the beam, the revised loads are calculated below:

Loads from the slab = 58.0 kN/m

Dead loads (revised) = 0.615 (0.35) (25) = 5.38125 kN/m

Total loads = 63.38125 kN/m

1.5 (63.38125) (12) (12)


( M u )factored = = 1,711.29 kNm
8

Approximate value of Ast:

Mu 1711.29 (106 )
Ast = = = 8,243.06 mm 2
Df 0.87 (415) (625 - 50)
0.87 f y (d - )
2

Version 2 CE IIT, Kharagpur


Step 4: Determination of Ast (Fig. 5.12.2)

xu = xu,max = 0.48 (625) = 300 mm

Equating T and C (Eq. 5.5 of Lesson 10), we have:

0.87 fy Ast = 0.36 xu,max bw fck + 0.45 fck (bf - bw) Df

0.36 (300) (350) (20) + 0.45 (20) (2600) (100)


or Ast = = 8,574.98 mm 2
0.87 (415)

Maximum Ast = 0.04 b D = 0.04 (350) (715) = 10,010.00 mm2

Minimum Ast = (0.85/fy) bw d = (0.85/415) (350) (625) = 448.05 mm2

Hence, Ast = 8,574.98 mm2 is o.k.

So, provide 8-36 T + 2-18 T = 8143 + 508 = 8,651 mm2

Step 5: Determination of xu

Using Ast = 8,651 mm2 in the expression of T = C (Eq. 5.5 of Lesson 10),
we have:

0.87 fy Ast = 0.36 xu bw fck + 0.45 fck (bf - bw) Df

0.87 f y Ast - 0.45 f ck (b f - bw )


or xu =
0.36 bw f ck

Version 2 CE IIT, Kharagpur


0.87 (415) (8651) - 0.45 (20) (2600) (100)
= = 310.89 > xu ,max (= 300 mm)
0.36 (350) (20)

So, Ast provided is reduced to 8-36 + 2-16 = 8143 + 402 = 8,545 mm2.
Accordingly,

0.87 (415) (8545) - 0.45 (20) (2600) (100)


xu = = 295.703 mm < xu ,max (= 300 mm)
0.36 (350) (20)

Step 6: Checking of Mu

Df /d = 100/625 = 0.16 < 0.2

Df /xu = 100/215.7 = 0.33 < 0.43. Hence, it is a problem of case (iii a)


and Mu can be obtained from Eq. 5.14 of Lesson 10.

So, Mu = 0.36(xu /d) {1 - 0.42(xu /d)} fck bf d2 + 0.45 fck (bf - bw) (Df) (d - Df /2)

= 0.36 (295.703/625) {1 - 0.42 (295.703/625)} (20) (350) (625) (625)

+ 0.45 (20) (2600) (100) (625 - 50)

= 1,718.68 kNm > (Mu)design (= 1,711.29 kNm)

Hence, the design is o.k.

Ex.7: Determine the tensile reinforcement Ast of the flanged beam of Ex.5 (Fig.
5.12.1) when the imposed loads = 12 kN/m2. All other parameters are the same
as those of Ex.5: Df = 100 mm, D = 750 mm, bw = 350 mm, spacing of beams =
4000 mm c/c, effective span = 12 m, simply supported, cover = 90 mm and d =
660 mm. Use Fe 415 and M 20.

Solution:

Step 1: Computation of factored bending moment Mu

Dead loads of the slab (see Ex.5) = 2.5 kN/m2

Imposed loads = 12.0 kN/m2

Total loads = 14.5 kN/m2

Loads/m = 14.5 (4) = 58.0 kN/m

Dead loads of beam = 0.65 (0.35) (25) = 5.6875 kN/m

Version 2 CE IIT, Kharagpur


Total loads = 63.6875 kN/m

Factored Mu = (1.5) (63.6875) (12) (12)/8 = 1,719.5625 kNm.

Step 2: Determination of Mu,lim

From Ex.5, the Mu,lim of this beam = 1,835.43 kNm. Hence, this beam
shall be designed as under-reinforced.

Step 3: Determination of xu

Assuming xu to be in the flange, we have from Eq. 3.24 of Lesson 5 and


considering b = bf,

Mu = 0.36xu {1 - 0.42(xu /d)} fck bf d

1719.5625 (106) = 0.36 xu {1 - 0.42 (xu /660)} (20) (2950) (550)

Solving, we get xu = 134.1 > 100 mm

So, let us assume that the neutral axis is in the web and Df /xu < 0.43, from Eq.
5.14 of Lesson 10 (case iii a of sec. 5.10.4.3), we have:

Mu = 0.36(xu /d) {1 - 0.42(xu /d)} fck bw d2 + 0.45 fck (bf - bw) (Df) (d - Df /2)

= 0.36 xu {1 - 0.42 (xu /660)} (20) (350) (660)

+ 0.45 (20) (2600) (100) (660 - 50)

Substituting the value of Mu = 1,719.5625 kNm in the above equation and


simplifying,

xu2 - 1571.43 xu + 276042 = 0

Solving, we have xu = 201.5 mm

Df /xu = 100/201.5 = 0.496 > 0.43.

So, we have to use Eq. 5.15 and 5.18 of Lesson 10 for yf and Mu (case iii b of
sec. 5.10.4.3). Thus, we have:

Mu = 0.36 xu {1 - 0.42( xu /d)} fck bw d + 0.45 fck (bf - bw) yf (d - yf /2)

where, yf = (0.15 xu + 0.65 Df)

So, Mu = 0.36 xu {1 - 0.42 (xu/660)} (20) (350) (660)

Version 2 CE IIT, Kharagpur


+ 0.45 (20) (2600) (0.15 xu + 65) (660 - 0.075 xu - 32.5)

or 1719.5625 (106) = 3.75165 (106) xu - 795.15 xu2 + 954.4275 (106)

Solving, we get xu = 213.63 mm.

Df /xu = 100/213.63 = 0.468 > 0.43.


Hence, o.k.

Step 4: Determination of Ast

Equating C = T from Eqs. 5.16 and 5.17 of Lesson 10 (case iii b of sec.
5.10.4.3), we have:

0.87 fy Ast = 0.36 fck bw xu + 0.45 fck (bf – bw) yf

where, yf = 0.15 xu + 0.65 Df

Here, using xu = 213.63 mm, Df = 100 mm, we get

yf = 0.15 (213.63) + 0.65 (100) = 97.04 mm

0.36 (20) (350) (213.63) + 0.45 (20) (2600) (97.04)


So, Ast = = 7,780.32 mm2
0.87 (415)

Minimum Ast = (0.85/fy) (bw) (d) = 0.85 (350) (660)/(415) = 473.13 mm2

Maximum Ast = 0.04 bw D = 0.04 (350) (750) = 10,500 mm2

Hence, Ast = 7,780.32 mm2 is o.k.

Version 2 CE IIT, Kharagpur


Provide 6-36 T + 3-28 T (6107 + 1847 = 7,954 mm2). Please refer to Fig.
5.12.3.

Step 5: Checking of xu and Mu using Ast = 7,954 mm2

From T = C (Eqs. 5.16 and 5.17 of Lesson 10), we have

0.87 fy Ast = 0.36 fck bw xu + 0.45 fck (bf – bw) yf

where, yf = 0.15 xu + 0.65 Df

or 0.87 (415) (7954) = 0.36 (20) (350) xu + 0.45 (20) (2600) (0.15 xu + 0.65
Df)

or xu = 224.01 mm

Df /xu = 100/224.01 = 0.446 > 0.43. Accordingly, employing Eq. 5.18 of


Lesson 10 (case iii b of sec. 5.10.4.3), we have:

So, Mu = 0.36 xu {1 - 0.42( xu /d)} fck bw d + 0.45 fck (bf - bw) yf (d - yf /2)

= 0.36 (224.01){1 - 0.42 (224.01/660)} (20) (350) (660)

+ 0.45 (20) (2600) {(0.15) 224.01 + 65} {(660) - 0.15 (112) - 32.5}

= 1,779.439 kNm > 1,719.5625 kNm


Hence, o.k.

Version 2 CE IIT, Kharagpur


Ex.8: Design the flanged beam of Fig. 5.12.4, given in following: Df = 100 mm, D
= 675 mm, bw = 350 mm, spacing of beams = 4000 mm c/c, effective span = 12
m simply supported, cover = 90 mm, d = 585 mm and imposed loads = 12 kN/m2.
Use Fe 415 and M 20.

Step 1: Computation of factored bending moment, Mu

Weight of slab/m2 = (0.1) (25) = 2.5 kN/m2

Imposed loads = 12.0 kN/m2

Total loads = 14.5 kN/m2

Total weight of slab + imposed loads/m = 14.5 (4) = 58 kN/m

Dead loads of beam = 0.575 (0.35) (25) = 5.032 kN/m

Total loads = 63.032 kN/m

Factored Mu = (1.5) (63.032) (12) (12)/8 = 1,701.87 kNm

Step 2: Determination of Mu,lim

Assuming the neutral axis to be in the web, Df /xu < 0.43 and Df /d = 100 /
585 = 0.17 < 0.2, we consider the case (ii a) of sec. 5.10.4.2 of Lesson 10 to get
the following:

Mu,lim = 0.36 (xu,max /d) {1 – 0.42 (xu,max /d)} fck bw d2

+ 0.45 fck (bf – bw) Df (d – Df /2)

= 0.36(0.48) {1 – 0.42 (0.48)} (20) (350) (585) (585)

+ 0.45(20) (2600) (100) (585 – 50) = 1,582.4 kNm

Since, factored Mu > Mu,lim, the beam is designed as doubly reinforced.

Mu2 = Mu - Mu,lim = 1701.87 - 1582.4 = 119.47 kNm

Version 2 CE IIT, Kharagpur


Step 3: Determination of area of steel

Ast,lim is obtained equating T = C (Eqs. 5.5 and 6 of Lesson 10).

0.87 fy (Ast,lim) = 0.36 bw (xu,max /d) d fck + 0.45 fck (bf - bw) Df

0.36 bw ( xu, max / d ) d f ck + 0.45 f ck (b f - bw ) D f


or Ast ,lim =
0.87 f y

0.36 (350) (0.48) (585) (20) + 0.45 (20) (2600) (100)


= = 8,440.98
0.87 (415)
mm2

M u2
Asc = (Eq. 4.4 of Lesson 8).
( f sc - f cc ) (d - d ' )

where fsc = 353 N/mm2 for d'/d = 0.1

fcc = 0.446 fck = 0.446 (20) = 8.92 N/mm2

Mu2 = 119.47 (106) Nmm

d' = 58.5 mm

d = 585 mm

Version 2 CE IIT, Kharagpur


Using the above values in the expression of Asc (Eq. 4.4 of Lesson 8), we get

Asc = 659.63 mm2

Asc ( f sc - f cc )
Ast 2 = (Eqs. 4.4 and 4.5 of Lesson 8).
0.87 f y

Substituting the values of Asc, fsc, fcc and fy we get

Ast2 = 628.48 mm2

Total Ast = Ast,lim + Ast2 = 8,440.98 + 628.48 = 9,069.46 mm2

Maximum Ast = 0.04 bw D = 0.04 (350) (675) = 9,450 mm2

and minimum Ast = (0.85/fy) bw d = (0.85/415) (350) (585) = 419.37 mm2

Hence, Ast = 9, 069.46 mm2 is o.k.

Provide 8-36 T + 3-20 T = 8143 + 942 = 9,085 mm2 for Ast and 1-20 + 2-16
= 314 + 402 = 716 mm2 for Asc (Fig. 5.12.5).

Step 4: To check for xu and Mu (Fig. 5.12.5)

Assuming xu in the web and Df /xu < 0.43 and using T = C (case ii a of sec.
5.10.4.2 of Lesson 10 with additional compression force due to compression
steel), we have:

0.87 fy Ast = 0.36 bw xu fck + 0.45 (bf - bw) fck Df + Asc (fsc - fcc)

or 0.87 (415) (9085) = 0.36 (350) xu (20) + 0.45 (2600) (20) (100)

+ 716 {353 - 0.45 (20)}

This gives xu = 275.33 mm.

xu,max = 0.48 (d) = 0.48 (585) = 280.8 mm.

So, xu < xu,max, Df /xu = 100/275.33 = 0.363 < 0.43

and Df /d = 100/585 = 0.17 < 0.2.

The assumptions, therefore, are correct. So, Mu can be obtained from Eq. 5.14
of sec. 5.10.4.3 of Lesson 10 with additional moment due to compression steel,
as given below:

Version 2 CE IIT, Kharagpur


So, Mu = 0.36 bw xu fck (d - 0.42 xu) + 0.45 (bf - bw) fck Df (d - Df /2)

+ Asc (fsc - fcc) (d - d')

= 0.36 (350) (275.33) (20) {585 - 0.42 (275.33)}

+ 0.45 (2600) (20) (100) (585 - 50) + 716 (344) (585 - 58.5)

= 325.66 + 1251.9 + 129.67 = 1,707.23 kNm

Factored moment = 1,701.87 kNm < 1,707.23 kNm. Hence, o.k.

5.12.4 Practice Questions and Problems with Answers

Q.1: Determine the steel reinforcement of a simply supported flanged beam


(Fig. 5.12.6) of Df = 100 mm, D = 700 mm, cover = 50 mm, d = 650
mm, bw = 300 mm, spacing of the beams = 4,000 mm c/c, effective span
= 10 m and imposed loads = 10 kN/m2. Use M 20 and Fe 415.

A.1: Solution:

Step 1: Computation of (Mu)factored

Weight of slab = (0.1) (25) = 2.5 kN/m2


Imposed loads = 10.0 kN/m2
_________
12.5 kN/m2

Version 2 CE IIT, Kharagpur


Total loads per m = (12.5) (4) = 50 kN/m

Dead loads of beam = (0.3) (0.6) (25) = 4.50 kN/m

Total loads = 54.50 kN/m

Factored Mu = (1.5) (54.50) (10) (10)/8 = 1,021.87 kNm

Step 2: Determination of Mu,lim

Effective width of the flange bf = lo/6 + bw + 6 Df = (10,000/6) + 300 + 600 =


2,567 mm.
xu,max = 0.48 d = 0.48 (650) = 312 mm

Hence, the balanced neutral axis is in the web of the beam.

Df /d = 100/650 = 0.154 < 0.2

Df /xu = 100/312 = 0.32 < 0.43

So, the full depth of flange is having a stress of 0.446 fck. From Eq. 5.7 of Lesson
10 (case ii a of sec. 5.10.4.2), we have,

Mu,lim = 0.36 (xu,max /d) {1 – 0.42 (xu,max /d)} fck bw d2

+ 0.45 fck (bf – bw) Df (d – Df /2)

= 0.36(0.48) {1 – 0.42(0.48)} (20) (300) (650) (650)

+ 0.45(20) (2267) (100) (650 – 50)

= 1573.92 kNm > Mu (= 1021.87 kNm)

So, the beam will be under-reinforced one.

Step 3: Determination of xu

Assuming xu is in the flange, we have from Eq. 3.24 of Lesson 5


(rectangular beam when b = bf ).

Mu = 0.36 (xu /d) {1 – 0.42 (xu /d)} fck bf d2

= 0.36 xu {1 – 0.42 (xu /d)} fck bf d

1021.87 (106) = 0.36 xu {1 – 0.42(xu/650)} (20) (2567) (650)

Version 2 CE IIT, Kharagpur


xu = 89.55 mm2 < 100 mm (Hence, the neutral axis is in the flange.)

Step 4: Determination of Ast

Equating C = T, we have from Eq. 3.17 of Lesson 5:

xu 0.87 f y Ast
=
d 0.36 f ck b f d

0.36 f ck b f xu 0.36 (20) (2567) (89.55)


or Ast = = = 4,584.12 mm 2
0.87 f y 0.87 (415)

0.85 (300) (650)


Minimum Ast = (0.85/f y ) (bw ) d = = 399.39 mm 2
415

Maximum Ast = 0.04 bw D = (0.04) (300) (700) = 8,400 mm2

So, Ast = 4,584.12 mm2 is o.k.

Provide 6-28 T + 2-25 T = 3694 + 981 = 4,675 mm2 (Fig. 5.12.6).

5.12.5 References
1. Reinforced Concrete Limit State Design, 6th Edition, by Ashok K. Jain,
Nem Chand & Bros, Roorkee, 2002.
2. Limit State Design of Reinforced Concrete, 2nd Edition, by P.C.Varghese,
Prentice-Hall of India Pvt. Ltd., New Delhi, 2002.
3. Advanced Reinforced Concrete Design, by P.C.Varghese, Prentice-Hall of
India Pvt. Ltd., New Delhi, 2001.
4. Reinforced Concrete Design, 2nd Edition, by S.Unnikrishna Pillai and
Devdas Menon, Tata McGraw-Hill Publishing Company Limited, New
Delhi, 2003.
5. Limit State Design of Reinforced Concrete Structures, by P.Dayaratnam,
Oxford & I.B.H. Publishing Company Pvt. Ltd., New Delhi, 2004.
6. Reinforced Concrete Design, 1st Revised Edition, by S.N.Sinha, Tata
McGraw-Hill Publishing Company. New Delhi, 1990.
7. Reinforced Concrete, 6th Edition, by S.K.Mallick and A.P.Gupta, Oxford &
IBH Publishing Co. Pvt. Ltd. New Delhi, 1996.
8. Behaviour, Analysis & Design of Reinforced Concrete Structural Elements,
by I.C.Syal and R.K.Ummat, A.H.Wheeler & Co. Ltd., Allahabad, 1989.
9. Reinforced Concrete Structures, 3rd Edition, by I.C.Syal and A.K.Goel,
A.H.Wheeler & Co. Ltd., Allahabad, 1992.
10. Textbook of R.C.C, by G.S.Birdie and J.S.Birdie, Wiley Eastern Limited,
New Delhi, 1993.

Version 2 CE IIT, Kharagpur


11. Design of Concrete Structures, 13th Edition, by Arthur H. Nilson, David
Darwin and Charles W. Dolan, Tata McGraw-Hill Publishing Company
Limited, New Delhi, 2004.
12. Concrete Technology, by A.M.Neville and J.J.Brooks, ELBS with
Longman, 1994.
13. Properties of Concrete, 4th Edition, 1st Indian reprint, by A.M.Neville,
Longman, 2000.
14. Reinforced Concrete Designer’s Handbook, 10th Edition, by C.E.Reynolds
and J.C.Steedman, E & FN SPON, London, 1997.
15. Indian Standard Plain and Reinforced Concrete – Code of Practice (4th
Revision), IS 456: 2000, BIS, New Delhi.
16. Design Aids for Reinforced Concrete to IS: 456 – 1978, BIS, New Delhi.

5.12.6 Test 12 with Solutions


Maximum Marks = 50, Maximum Time = 30 minutes

Answer all questions.

TQ.1: Determine the steel reinforcement Ast of the simply supported flanged
beam of Q.1 (Fig. 5.12.6) having Df = 100 mm, D = 700 mm, cover = 50
mm, d = 650 mm, bw = 300 mm, spacing of the beams = 4,000 mm
c/c, effective span = 12 m and imposed loads = 10 kN/m2. Use M 20 and
Fe 415.

Version 2 CE IIT, Kharagpur


A.TQ.1: Solution:

Step 1: Computation of (Mu)factored

Total loads from Q.1 of sec. 5.12.4 = 54.50 kN/m

Factored Mu = (1.5) (54.50) (12) (12)/8 = 1,471.5 kNm

Step 2: Determination of Mu,lim

Effective width of flange = lo/6 + bw + 6 Df

= (12000/6) + 300 + 600 = 2,900 mm (Fig. 5.12.7)

xu,max = 0.48 d = 0.48 (650) = 312 mm

Hence, the balanced neutral axis is in the web.

Df /d = 100/650 = 0.154 < 0.2

Df /xu = 100/312 = 0.32 < 0.43

So, the full depth of flange is having constant stress of 0.446 fck. From Eq. 5.7 of
Lesson 10 (case ii a of sec. 5.10.4.2), we have

Mu,lim = 0.36 (xu,max /d) {1 – 0.42 (xu,max /d)} fck bw d2

+ 0.45 fck (bf – bw) Df (d – Df /2)

= 0.36(0.48) {1 – 0.42(0.48)} (20) (300) (650) (650)

+ 0.45(20) (2600) (100) (650 – 50) = 1,753.74 kNm > 1,471.5


kNm

So, the beam will be under-reinforced.

Step 3: Determination of xu

Assuming xu to be in the flange, we have from Eq. 3.24 of Lesson 5 (singly


reinforced rectangular beam when b = bf ):

Mu = 0.36 xu {1 – 0.42 (xu /d)} fck bf d

or 1471.5 (106) = 0.36 (xu) {1 – 0.42 (xu /650)} (20) (2900) (650)

or xu2 – 1547.49 xu + 167.81 (103) = 0

Version 2 CE IIT, Kharagpur


Solving, we have xu = 117.34 mm > 100 mm

So, neutral axis is in the web.

Assuming Df /xu < 0.43, we have from Eq. 5.14 of Lesson 10 (case iii a of sec.
5.10.4.3),

Mu = 0.36 xu {1 – 0.42 (xu /d)} fck bw d + 0.45 fck (bf – bw) Df (d – Df /2)

= 0.36 xu {1 – 0.42 (xu/650)} (20) (300) (650)

+ 0.45(20) (2600) (100) (650 – 50)

or xu2 – 1547.62 xu + 74404.7 = 0

Solving, we have xu = 49.67 < 100 mm

However, in the above when it is assumed that the neutral axis is in the
flange xu is found to be 117.34 mm and in the second trial when xu is assumed in
the web xu is seen to be 49.67 mm. This indicates that the full depth of the
flange will not have the strain of 0.002, neutral axis is in the web and Df /xu is
more than 0.43. So, we have to use Eq. 5.18 of Lesson 10, with the introduction
of yf from Eq. 5.15 of Lesson 10.

Assuming Df /xu > 0.43, from Eqs. 5.15 and 5.18 of Lesson 10 (case iii b
of sec. 5.10.4.3), we have:

Mu = 0.36 xu {1 – 0.42 (xu /d)} fck bw d + 0.45 fck (bf – bw) yf (d – yf /2)

where, yf = (0.15 xu + 0.65 Df)

So, Mu = 0.36 xu {1 – 0.42 (xu/650)} (20) (300) (650)

+ 0.45(20) (2600) (0.15 xu + 0.65) (650 – 0.075 xu – 0.325 xu)

or, 1471.5 (106) = - 1170.45 xu2 + 3.45735 xu + 939.2175 (106)

Solving, we get xu = 162.9454 mm. This shows that the assumption of Df /xu >
0.43 is correct as Df /xu = 100 / 162.9454 = 0.614.

Step 4: Determination of Ast

Equating C = T from Eqs. 5.16 and 5.17 of Lesson 10 (case iii b of sec.
5.10.4.3), we have

Version 2 CE IIT, Kharagpur


0.87 fy Ast = 0.36 fck bw xu + 0.45 fck (bf – bw) yf

0.36 (20) (30) (162.9454) + 0.45 (20) (2600) {0.15 (162.9454) + 65}
or Ast =
0.87 (415)

= 974.829 + 5,796.81 = 6,771.639 mm2

Minimum Ast = (0.85/fy) (bw) (d) = 0.85 (300) (650)/415 = 399.39 mm2

Maximum Ast = 0.04 (bw) (D) = 0.04 (300) (700) = 8,400 mm2

So, Ast = 6,771.639 is o.k.

Provide 2-36 T + 6-32 T = 2035 + 4825 = 6,860 mm2 > 6,771.639 mm2
(Fig. 5.12.7).

5.12.7 Summary of this Lesson


This lesson explains the steps involved in solving the design type of
numerical problems. Further, several examples of design type of numerical
problems are illustrated explaining the steps of their solutions. Solutions of
practice problems and test problems will give the readers confidence in applying
the theory explained in Lesson 10 in solving the numerical problems.

Version 2 CE IIT, Kharagpur


Module
6
Shear, Bond,
Anchorage,
Development Length
and Torsion
Version 2 CE IIT, Kharagpur
Lesson
13
Limit State of Collapse
in Shear
Version 2 CE IIT, Kharagpur
Instructional Objectives:

At the end of this lesson, the student should be able to:

• name and explain the three different failure modes of reinforced concrete
beams under the combined effects of bending moment and shear force,
• define nominal shear stress τv of rectangular and T-beams of uniform and
varying depths under the combined effects of bending moment and shear
force,
• name the two parameters on which the design shear strength of concrete
depends,
• find out the maximum shear stress of concrete beams τcmax with shear
reinforcement,
• locate the critical sections for shear in beams,
• explain when and why do we consider enhanced shear strength of concrete,
• explain why the minimum shear reinforcement is provided in any beam,
• determine the amount of minimum shear reinforcement to be provided in
any beam,
• specify the three different ways of providing shear reinforcement in a beam,
• design the shear reinforcement in a beam for each of the three methods
mentioned above,
• design the shear reinforcement closed to the support of a beam,
• specify the conditions to be satisfied for the curtailment of tension
reinforcement when designing shear reinforcement,
• place the vertical stirrups in a beam.

6.13.1 Introduction

This lesson explains the three failure modes due to shear force in beams
and defines different shear stresses needed to design the beams for shear. The
critical sections for shear and the minimum shear reinforcement to be provided in
beams are mentioned as per IS 456. The design of shear reinforcement has
been illustrated in Lesson 14 through several numerical problems including the
curtailment of tension reinforcement in flexural members.

Version 2 CE IIT, Kharagpur


6.13.2 Failure Modes due to Shear

Bending in reinforced concrete beams is usually accompanied by shear,


the exact analysis of which is very complex. However, experimental studies
confirmed the following three different modes of failure due to possible
combinations of shear force and bending moment at a given section (Figs.
6.13.1a to c):

(i) Web shear (Fig. 6.13.1a)


(ii) Flexural tension shear (Fig. 6.13.1b)
(iii) Flexural compression shear (Fig. 6.13.1c)

Web shear causes cracks which progress along the dotted line shown in
Fig. 6.13.1a. Steel yields in flexural tension shear as shown in Fig. 6.13.1b, while
concrete crushes in compression due to flexural compression shear as shown in
Fig. 6.13.1c. An in-depth presentation of the three types of failure modes is
beyond the scope here. Only the salient points needed for the routine design of
beams in shear are presented here.

Version 2 CE IIT, Kharagpur


6.13.3 Shear Stress
The distribution of shear stress in reinforced concrete rectangular, T and
L-beams of uniform and varying depths depends on the distribution of the normal
stress. However, for the sake of simplicity the nominal shear stress τv is
considered which is calculated as follows (IS 456, cls. 40.1 and 40.1.1):

Version 2 CE IIT, Kharagpur


(i) In beams of uniform depth (Figs. 6.13.2a and b):

Vu
τv =
bd
(6.1)

where Vu = shear force due to design loads,

b = breadth of rectangular beams and breadth of the web bw for flanged


beams, and

d = effective depth.

(ii) In beams of varying depth:

Mu
Vu ± tanβ
τv = d
bd
(6.2)

where τv, Vu, b or bw and d are the same as in (i),

Mu = bending moment at the section, and

β = angle between the top and the bottom edges.

The positive sign is applicable when the bending moment Mu decreases


numerically in the same direction as the effective depth increases, and the
negative sign is applicable when the bending moment Mu increases numerically
in the same direction as the effective depth increases.

6.13.4 Design Shear Strength of Reinforced Concrete


Recent laboratory experiments confirmed that reinforced concrete in
beams has shear strength even without any shear reinforcement. This shear
strength (τc) depends on the grade of concrete and the percentage of tension
steel in beams. On the other hand, the shear strength of reinforced concrete with
the reinforcement is restricted to some maximum value τcmax depending on the
grade of concrete. These minimum and maximum shear strengths of reinforced
concrete (IS 456, cls. 40.2.1 and 40.2.3, respectively) are given below:

6.13.4.1 Design shear strength without shear reinforcement (IS 456, cl.
40.2.1)

Version 2 CE IIT, Kharagpur


Table 19 of IS 456 stipulates the design shear strength of concrete τc for
different grades of concrete with a wide range of percentages of positive tensile
steel reinforcement. It is worth mentioning that the reinforced concrete beams
must be provided with the minimum shear reinforcement as per cl. 40.3 even
when τv is less than τc given in Table 6.1.

Table 6.1 Design shear strength of concrete, τc in N/mm2

Grade of concrete
(100 As /b M 20 M 25 M 30 M 35 M40 and
d) above
≤ 0.15 0.28 0.29 0.29 0.29 0.30
0.25 0.36 0.36 0.37 0.37 0.38
0.50 0.48 0.49 0.50 0.50 0.51
0.75 0.56 0.57 0.59 0.59 0.60
1.00 0.62 0.64 0.66 0.67 0.68
1.25 0.67 0.70 0.71 0.73 0.74
1.50 0.72 0.74 0.76 0.78 0.79
1.75 0.75 0.78 0.80 0.82 0.84
2.00 0.79 0.82 0.84 0.86 0.88
2.25 0.81 0.85 0.88 0.90 0.92
2.50 0.82 0.88 0.91 0.93 0.95
2.75 0.82 0.90 0.94 0.96 0.98
≥ 3.00 0.82 0.92 0.96 0.99 1.01

In Table 6.1, As is the area of longitudinal tension reinforcement which


continues at least one effective depth beyond the section considered except at
support where the full area of tension reinforcement may be used provided the
detailing is as per IS 456, cls. 26.2.2 and 26.2.3.

6.13.4.2 Maximum shear stress τcmax with shear reinforcement (cls. 40.2.3,
40.5.1 and 41.3.1)

Table 20 of IS 456 stipulates the maximum shear stress of reinforced


concrete in beams τcmax as given below in Table 6.2. Under no circumstances,
the nominal shear stress in beams τv shall exceed τcmax given in Table 6.2 for
different grades of concrete.

Table 6.2 Maximum shear stress, τcmax in N/mm2

Grade of M 20 M 25 M 30 M 35 M 40 and
concrete above

Τcmax, 2.8 3.1 3.5 3.7 4.0


N/mm2

Version 2 CE IIT, Kharagpur


6.13.5 Critical Section for Shear

Clauses 22.6.2 and 22.6.2.1 stipulate the critical section for shear and are
as follows:

For beams generally subjected to uniformly distributed loads or where the


principal load is located further than 2d from the face of the support, where d is
the effective depth of the beam, the critical sections depend on the conditions of
supports as shown in Figs. 6.13.3 a, b and c and are mentioned below.

(i) When the reaction in the direction of the applied shear introduces tension
(Fig. 6.13.3a) into the end region of the member, the shear force is to be
computed at the face of the support of the member at that section.

(ii) When the reaction in the direction of the applied shear introduces
compression into the end region of the member (Figs. 6.13.3b and c), the shear
force computed at a distance d from the face of the support is to be used for the
design of sections located at a distance less than d from the face of the support.
The enhanced shear strength of sections close to supports, however, may be
considered as discussed in the following section.

Version 2 CE IIT, Kharagpur


6.13.6 Enhanced Shear Strength of Sections Close to
Supports (cl. 40.5 of IS 456)

Figure 6.13.4 shows the shear failure of simply supported and cantilever
beams without shear reinforcement. The failure plane is normally inclined at an
angle of 30o to the horizontal. However, in some situations the angle of failure is
more steep either due to the location of the failure section closed to a support or
for some other reasons. Under these situations, the shear force required to
produce failure is increased.

Such enhancement of shear strength near a support is taken into account


by increasing the design shear strength of concrete to (2dτc/av) provided that the
design shear stress at the face of the support remains less than the value of
τcmax given in Table 6.2 (Table 20 of IS 456). In the above expression of the
enhanced shear strength

d = effective depth of the beam,

τc = design shear strength of concrete before the enhancement as given in


Table 6.1 (Table 19 of IS 456),

av = horizontal distance of the section from the face of the support (Fig.
6.13.4).

Similar enhancement of shear strength is also to be considered for


sections closed to point loads. It is evident from the expression (2dτc /av) that
when av is equal to 2d, the enhanced shear strength does not come into picture.
Further, to increase the effectivity, the tension reinforcement is recommended to
be extended on each side of the point where it is intersected by a possible failure
plane for a distance at least equal to the effective depth, or to be provided with
an equivalent anchorage.

Version 2 CE IIT, Kharagpur


6.13.7 Minimum Shear Reinforcement (cls. 40.3, 26.5.1.5
and 26.5.1.6 of IS 456)
Minimum shear reinforcement has to be provided even when τv is less
than τc given in Table 6.1 as recommended in cl. 40.3 of IS 456. The amount of
minimum shear reinforcement, as given in cl. 26.5.1.6, is given below.

The minimum shear reinforcement in the form of stirrups shall be provided


such that:

Asv 0.4

b sv 0.87 f y
(6.3)

where Asv = total cross-sectional area of stirrup legs effective in shear,

sv = stirrup spacing along the length of the member,

b = breadth of the beam or breadth of the web of the web of flanged


beam bw, and

fy = characteristic strength of the stirrup reinforcement in N/mm2 which


shall not be taken greater than 415 N/mm2.

The above provision is not applicable for members of minor structural


importance such as lintels where the maximum shear stress calculated is less
than half the permissible value.

The minimum shear reinforcement is provided for the following:

(i) Any sudden failure of beams is prevented if concrete cover bursts


and the bond to the tension steel is lost.

(ii) Brittle shear failure is arrested which would have occurred without
shear reinforcement.

(iii) Tension failure is prevented which would have occurred due to


shrinkage, thermal stresses and internal cracking in beams.

(iv) To hold the reinforcement in place when concrete is poured.

(v) Section becomes effective with the tie effect of the compression
steel.

Version 2 CE IIT, Kharagpur


Further, cl. 26.5.1.5 of IS 456 stipulates that the maximum spacing of
shear reinforcement measured along the axis of the member shall not be more
than 0.75 d for vertical stirrups and d for inclined stirrups at 45o, where d is the
effective depth of the section. However, the spacing shall not exceed 300 mm in
any case.

6.13.8 Design of Shear Reinforcement (cl. 40.4 of IS 456)


When τv is more than τc given in Table 6.1, shear reinforcement shall be
provided in any of the three following forms:

(a) Vertical stirrups,


(b) Bent-up bars along with stirrups, and
(c) Inclined stirrups.

In the case of bent-up bars, it is to be seen that the contribution towards


shear resistance of bent-up bars should not be more than fifty per cent of that of
the total shear reinforcement.

The amount of shear reinforcement to be provided is determined to carry a


shear force Vus equal to

Vus = Vu – τc b d
(6.4)

where b is the breadth of rectangular beams or bw in the case of flanged


beams.

The strengths of shear reinforcement Vus for the three types of shear
reinforcement are as follows:

(a) Vertical stirrups:

0.87 f y Asv d
Vus =
sv
(6.5)

(b) For inclined stirrups or a series of bars bent-up at different cross-sections:

0.87 f y Asv d
Vus = (sinα + cosα )
sv
(6.6)

Version 2 CE IIT, Kharagpur


(c) For single bar or single group of parallel bars, all bent-up at the same cross-
section:

Vus = 0.87 f y Asv s v sinα


(6.7)

where Asv = total cross-sectional area of stirrup legs or bent-up bars


within a distance sv,

sv = spacing of stirrups or bent-up bars along the length of the member,

τv = nominal shear stress,

τc = design shear strength of concrete,

b = breadth of the member which for the flanged beams shall be taken as
the breadth of the web bw,

fy = characteristic strength of the stirrup or bent-up reinforcement which


shall not be taken greater than 415 N/mm2,

α = angle between the inclined stirrup or bent-up bar and the axis of the
member, not less than 45o, and

d = effective depth.

The following two points are to be noted:

(i) The total shear resistance shall be computed as the sum of the
resistance for the various types separately where more than one
type of shear reinforcement is used.

(ii) The area of stirrups shall not be less than the minimum specified in
cl. 26.5.1.6.

6.13.9 Shear Reinforcement for Sections Close to Supports


As stipulated in cl. 40.5.2 of IS 456, the total area of the required shear
reinforcement As is obtained from:

As = av b (τv – 2d τc /av)/0.87 fy

and ≥ 0.4 av b/0.87 fy


(6.8)

Version 2 CE IIT, Kharagpur


For flanged beams, b will be replaced by bw, the breadth of the web of
flanged beams.

This reinforcement should be provided within the middle three quarters of


av, where av is less than d, horizontal shear reinforcement will be effective than
vertical.

Alternatively, one simplified method has been recommended in cl. 40.5.3


of IS 456 and the same is given below.

The following method is for beams carrying generally uniform load or


where the principal load is located further than 2d from the face of support. The
shear stress is calculated at a section a distance d from the face of support. The
value of τc is calculated in accordance with Table 6.1 and appropriate shear
reinforcement is provided at sections closer to the support. No further check for
shear at such sections is required.

6.13.10 Curtailment of Tension Reinforcement in


Flexural Members (cl. 26.2.3.2 of IS 456)
Curtailment of tension reinforcement is done to provide the required
reduced area of steel with the reduction of the bending moment. However, shear
force increases with the reduction of bending moment. Therefore, it is necessary
to satisfy any one of following three conditions while terminating the flexural
reinforcement in tension zone:

(i) The shear stress τv at the cut-off point should not exceed two-thirds of
the permitted value which includes the shear strength of the web reinforcement.
Accordingly,

τv ≤ (2/3) (τc + Vus /b d)

or Vus ≥ (1.5 τv - τc) b d


(6.9)

(ii) For each of the terminated bars, additional stirrup area should be
provided over a distance of three-fourth of effective depth from the cut-off point.
The additional stirrup area shall not be less than 0.4 b s/fy, where b is the breadth
of rectangular beams and is replaced by bw, the breadth of the web for flanged
beams, s = spacing of additional stirrups and fy is the characteristic strength of
stirrup reinforcement in N/mm2. The value of s shall not exceed d/(8 βb), where
βb is the ratio of area of bars cut-off to the total area of bars at that section, and d
is the effective depth.

Version 2 CE IIT, Kharagpur


(iii) For bars of diameters 36 mm and smaller, the continuing bars provide
double the area required for flexure at the cut-off point. The shear stress should
not exceed three-fourths that permitted. Accordingly,

τv ≤ (3/4) (τc + Vus /b d)

or Vus ≥ (1.33 τv - τc) b d


(6.10)

In the above expression b is the breadth of the rectangular beams which will be
bw in the case of flanged beams.

6.13.11 Placement of Stirrups

The stirrups in beams shall be taken around the outer-most tension and
compression bars. In T and L-beams, the stirrups will pass around longitudinal
bars located close to the outer face of the flange. In the rectangular beams, two
holder bars of diameter 10 or 12 mm are provided if there is no particular need
for compression reinforcement (Fig. 6.13.5).

6.13.12 Practice Questions and Problems with Answers


Q.1: Name and explain the three different failure modes of reinforced concrete
beams under the combined effects of bending moment and shear force.

Version 2 CE IIT, Kharagpur


A.1: Sec. 6.13.2

Q.2: Define nominal shear stress τv of rectangular and T-beams of (i) uniform
depth and (ii) varying depth subjected to bending moment and shear
force.

A.2: Sec. 6.13.3

Q.3: What is meant by “Design shear strength of concrete τc” ?

A.3: Sec. 6.13.4

Q.4: On what parameters τc of beams without shear reinforcement depends ?


How do you get τc for different grades of concrete ?

A.4: τc depends on (i) grade of concrete and (ii) percentage of tensile steel in
the beam.

Table 19 of cl. 40.2.1 of IS 456 gives the values of τc and the same table is
presented in Table 6.1 of sec. 6.13.4.1 of this lesson.

Q.5: How do you know the maximum shear stress of concrete beams τcmax
with shear reinforcement ?

A.5: Sec. 6.13.4.2

Q.6: How do you determine the critical sections for shear in a beam ?

A.6: Sec. 6.13.5

Q.7: When and why do we consider enhanced shear strength of concrete ?

A.7: Sec. 6.13.6

Q.8: How do we determine the minimum shear reinforcement in rectangular


and T-beams ? Why do we provide the minimum shear reinforcement ?

A.8: Sec. 6.13.7

Q.9: What are the three different ways to provide shear reinforcement ? Explain
the method of design of each of them.

A.9: Sec. 6.13.8

Version 2 CE IIT, Kharagpur


Q.10: How do we design the shear reinforcement close to the support of a beam
?

A.10: Sec. 6.13.9

Q.11: State the conditions to be satisfied for the curtailment of tension


reinforcement when designing the shear reinforcement.

A.11: Sec. 6.13.10

Q.12: How do we place the vertical stirrups in a beam ?

A.12: Sec. 6.13.11

6.13.13 References:
1. Reinforced Concrete Limit State Design, 6th Edition, by Ashok K. Jain,
Nem Chand & Bros, Roorkee, 2002.
2. Limit State Design of Reinforced Concrete, 2nd Edition, by P.C.Varghese,
Prentice-Hall of India Pvt. Ltd., New Delhi, 2002.
3. Advanced Reinforced Concrete Design, by P.C.Varghese, Prentice-Hall of
India Pvt. Ltd., New Delhi, 2001.
4. Reinforced Concrete Design, 2nd Edition, by S.Unnikrishna Pillai and
Devdas Menon, Tata McGraw-Hill Publishing Company Limited, New
Delhi, 2003.
5. Limit State Design of Reinforced Concrete Structures, by P.Dayaratnam,
Oxford & I.B.H. Publishing Company Pvt. Ltd., New Delhi, 2004.
6. Reinforced Concrete Design, 1st Revised Edition, by S.N.Sinha, Tata
McGraw-Hill Publishing Company. New Delhi, 1990.
7. Reinforced Concrete, 6th Edition, by S.K.Mallick and A.P.Gupta, Oxford &
IBH Publishing Co. Pvt. Ltd. New Delhi, 1996.
8. Behaviour, Analysis & Design of Reinforced Concrete Structural Elements,
by I.C.Syal and R.K.Ummat, A.H.Wheeler & Co. Ltd., Allahabad, 1989.
9. Reinforced Concrete Structures, 3rd Edition, by I.C.Syal and A.K.Goel,
A.H.Wheeler & Co. Ltd., Allahabad, 1992.
10. Textbook of R.C.C, by G.S.Birdie and J.S.Birdie, Wiley Eastern Limited,
New Delhi, 1993.
11. Design of Concrete Structures, 13th Edition, by Arthur H. Nilson, David
Darwin and Charles W. Dolan, Tata McGraw-Hill Publishing Company
Limited, New Delhi, 2004.
12. Concrete Technology, by A.M.Neville and J.J.Brooks, ELBS with
Longman, 1994.
13. Properties of Concrete, 4th Edition, 1st Indian reprint, by A.M.Neville,
Longman, 2000.
14. Reinforced Concrete Designer’s Handbook, 10th Edition, by C.E.Reynolds
and J.C.Steedman, E & FN SPON, London, 1997.

Version 2 CE IIT, Kharagpur


15. Indian Standard Plain and Reinforced Concrete – Code of Practice (4th
Revision), IS 456: 2000, BIS, New Delhi.
16. Design Aids for Reinforced Concrete to IS: 456 – 1978, BIS, New Delhi.

6.13.14 Test 13 with Solutions


Maximum Marks = 50, Maximum Time = 30 minutes

Answer all questions.

TQ.1: Define nominal shear stress τv of rectangular and T-beams of (i)


uniform depth and (ii) varying depth subjected to bending moment and
shear force.
(5
marks)

A.TQ.1: Sec. 6.13.3

TQ.2: How do you determine the critical sections for shear in a beam ? (5
marks)

A.TQ.2: Sec. 6.13.5

TQ.3: When and why do we consider enhanced shear strength of concrete? (5


marks)

A.TQ.3: Sec. 6.13.6

TQ.4: How do we determine the minimum shear reinforcement in rectangular


and T-beams? Why do we provide the minimum shear reinforcement ?
(5 marks)

A.TQ.4: Sec. 6.13.7

TQ.5: What are the three different ways to provide shear reinforcement ?
Explain the method of design of each of them. (5 marks)

A.TQ.5: Sec. 6.13.8

TQ.6: How do we design the shear reinforcement close to the support of a beam?
(5 marks)

A.TQ.6: Sec. 6.13.9

TQ.7: State the conditions to be satisfied for the curtailment of tension


reinforcement when designing the shear reinforcement. (5 marks)

Version 2 CE IIT, Kharagpur


A.TQ.7: Sec. 6.13.10

6.13.15 Summary of this Lesson


Learning the different failure modes, the shear stresses and the design
procedure of beams subjected to shear, this lesson explains the design
procedure with special reference to curtailment of tension reinforcement in
flexural members. Solutions of problems as illustrated in Lesson 14 and given in
the practice problems and test, students will be thoroughly conversant with the
design of rectangular and T-beams subjected to shear following the limit state of
collapse as recommended by IS 456..

Version 2 CE IIT, Kharagpur


Module
6
Shear, Bond,
Anchorage,
Development Length
and Torsion
Version 2 CE IIT, Kharagpur
Lesson
14
Limit State of Collapse
in Shear – Numerical
Problems
Version 2 CE IIT, Kharagpur
Instructional Objectives:
At the end of this lesson, the student should be able to:

• solve specific numerical problems of rectangular and T-beams for the


complete design of shear reinforcement as per the stipulations of IS 456.

6.14.1 Introduction
Lesson 13 explains the three failure modes due to shear force in beams
and defines different shear stresses needed to design the beams for shear. The
critical sections for shear and the minimum shear reinforcement to be provided in
beams are mentioned as per IS 456. In this lesson, the design of shear
reinforcement has been illustrated through several numerical problems including
the curtailment of tension reinforcement in flexural members.

6.14.2 Numerical Problems

Problem 1:

Determine the shear reinforcement of the simply supported beam of


effective span 8 m whose cross-section is shown in Fig. 6.14.1. Factored shear
force is 250 kN. Use M 20 and Fe 415.

Version 2 CE IIT, Kharagpur


Solution 1:

Here, Ast = 2-25T + 2-20T gives the percentage of tensile reinforcement

100 (1609)
p = = 1.43
250 (450)

From Table 6.1 of Lesson 13, τc = 0.67 + 0.036 = 0.706 N/mm2 (by linear
interpolation).

Employing Eq. 6.1 of Lesson 13,

Vu 250 (10 3 )
τv = = = 2.22 N/mm 2 and τcmax = 2.8 N/mm2 (from Table
bd 250 (450)
6.2 of Lesson 13).

Hence, τc < τv < τcmax. So, shear reinforcement is needed for the shear force
(Eq. 6.4 of Lesson 13).

Vus = Vu – τc b d = 250 – 0.706 (250) (450) (10-3) = 170.575 kN

Providing 8 mm, 2 legged vertical stirrups, we have

Asv = 2 (50) = 100 mm2

Hence, spacing of the stirrups as obtained from Eq. 6.5 of Lesson 13:

0.87 f y Asv d 0.87 (415) (100) (450)


sv = = = 95.25 mm, say 95 mm.
Vus 170575

For 10 mm, 2 legged vertical stirrups, (Asv = 157 mm2), spacing

0.87 (415) (157) (450)


sv = = 149.54 mm
170575

According to cl. 26.5.1.5 of IS 456, the maximum spacing of the stirrups = 0.75
d = 0.75 (450) = 337.5 mm = 300 mm (say).

Minimum shear reinforcement (cl. 26.5.1.6 of IS 456) is obtained from (Eq.6.3 of


sec. 6.13.7 of Lesson 13):

Version 2 CE IIT, Kharagpur


Asv 0.4

(b) s v 0.87 ( f y )

0.4 b s v 0.4 (250) (145)


From the above, Asv(minimum) = = = 40.16 mm 2 .
0.87 f y 0.87 (415)

So, we select 10 mm, 2 legged stirrups @ 145 mm c/c.

Problem 2:

Design the bending and shear reinforcement of the tapered cantilever


beam of width b = 300 mm and as shown in Fig. 6.14.2 using M 20 and Fe 415
(i) without any curtailment of bending reinforcement and (ii) redesign the bending
and shear reinforcement if some of the bars are curtailed at section 2-2 of the
beam. Use SP-16 for the design of bending reinforcement.

Solution 2:

75 (3.5) (3.5)
M u at section 1 - 1 = = 459.375 kNm
2

Mu 459.375 (10 6 )
2
= = 6.125 N/mm 2
bd 300 (500) (500)

Version 2 CE IIT, Kharagpur


Solution 1:

Here, Ast = 2-25T + 2-20T gives the percentage of tensile reinforcement

100 (1609)
p = = 1.43
250 (450)

From Table 6.1 of Lesson 13, τc = 0.67 + 0.036 = 0.706 N/mm2 (by linear
interpolation).

Employing Eq. 6.1 of Lesson 13,

Vu 250 (10 3 )
τv = = = 2.22 N/mm 2 and τcmax = 2.8 N/mm2 (from Table
bd 250 (450)
6.2 of Lesson 13).

Hence, τc < τv < τcmax. So, shear reinforcement is needed for the shear force
(Eq. 6.4 of Lesson 13).

Vus = Vu – τc b d = 250 – 0.706 (250) (450) (10-3) = 170.575 kN

Providing 8 mm, 2 legged vertical stirrups, we have

Asv = 2 (50) = 100 mm2

Hence, spacing of the stirrups as obtained from Eq. 6.5 of Lesson 13:

0.87 f y Asv d 0.87 (415) (100) (450)


sv = = = 95.25 mm, say 95 mm.
Vus 170575

For 10 mm, 2 legged vertical stirrups, (Asv = 157 mm2), spacing

0.87 (415) (157) (450)


sv = = 149.54 mm
170575

According to cl. 26.5.1.5 of IS 456, the maximum spacing of the stirrups = 0.75
d = 0.75 (450) = 337.5 mm = 300 mm (say).

Minimum shear reinforcement (cl. 26.5.1.6 of IS 456) is obtained from (Eq.6.3 of


sec. 6.13.7 of Lesson 13):

Version 2 CE IIT, Kharagpur


Reinforcing bars of 3-28T + 1-16T give 2048 mm2.

Asc at section 2-2 = (0.686) (400) (300)/100 = 823.2 mm2.

Reinforcing bars of 2-20T + 2-12T give 854 mm2.

Though it is better to use 4-28T as Ast and 2-20T + 2-16 as Asc with proper
curtailment from the practical aspects of construction, here the bars are selected
to have areas close to the requirements for the academic interest only.

Figure 6.14.4 shows the reinforcement at sec. 2-2.

Case (i): No curtailment (all bars of bending reinforcement are continued):

Bending moment Mu at section 2-2 = 234.375 kNm

Shear force Vu at section 2-2 = 187.5 kN

Effective depth d at section 2-2 = 450 – 50 = 400 mm, and

Width b = 300 mm

tanβ = (550 – 200)/3500 = 0.1

Clause 40.1.1 of IS 456 gives (Eq.6.2 of sec. 6.13.3 of Lesson 13):

Version 2 CE IIT, Kharagpur


Mu
Vu − tanβ
d (10 3 ) {187.5 - 234.375 (0.1)/0.4}
τv = = = 1.074 N/mm 2
bd (300) (400)

(Here, the negative sign is used as the bending moment increases numerically in
the same direction as the effective depth increases.)

Continuing 4-28T and 3-16T bars (= 3066 mm2), we get

p = 3066 (100)/300 (400) = 2.555 %

Table 6.1 of Lesson 13 gives τc = 0.82 N/mm2 < τv (= 1.074 N/mm2). Hence,
shear reinforcement is needed for shear force obtained from Eq. 6.4 of Lesson
13:

Vus = Vu - τv b d = 1.875 – 0.82 (300) (400) (10-3) kN = 89.1 kN

From cl.40.4 of IS 456, we have (Eq.6.5 of sec. 6.13.8 of Lesson 13),

0.87 f y Asv d
Vus =
sv

where Asv = 100 mm2 for 8 mm, 2 legged vertical stirrups. This gives sv =
162.087 mm (f y = 415 N/mm2). IS 456, cl. 26.5.1.6 gives the spacing
considering minimum shear reinforcement (Eq.6.3 of sec. 6.13.7 of Lesson 13):

0.87 f y Asv
sv ≤
0.4 b

or sv ≤ 300.875 mm

Hence, provide 8 mm, 2 legged vertical stirrups @ 150 mm c/c, as shown in Fig.
6.14.3.

Case (ii): With curtailment of bars:

Clause 26.2.3.2 of IS 456 stipulates that any one of the three conditions is
to be satisfied for the termination of flexural reinforcement in tension zone (see
sec. 6.13.10 of Lesson 13). Here, two of the conditions are discussed.

(a) Condition (i):

2 V
τv ≤ (τ c + us ), which gives Eq. 6.9 of Lesson 13 as
3 bd
Version 2 CE IIT, Kharagpur
Vus ≥ (1.5 τv – τc) b d

After the curtailment, at section 2-2 Ast = 2048 mm2 (3-28T + 1-16T bars), gives
p = 2048 (100)/300 (400) ≅ 1.71 %. Table 6.1 of Lesson 13 gives τc = 0.7452
N/mm2 when p = 1.71% (making liner interpolation). Now from Eq. 6.2 of Lesson
13:

Mu
Vu − tanβ
d (10 3 ) {187.5 - 234.375 (0.1)/0.4}
τv = = = 1.074 N/mm 2
bd (300) (400)

So, Vus = {1.5 (1.074) – 0.7452} (300) (400) (10-3) kN = 103.896 kN

which gives the spacing of stirrups (Eq. 6.5):

0.87 f y Asv d
sv ≤
Vus

Using 8 mm, 2 legged vertical stirrups (Asv = 100 mm2), we have:

sv = 0.87 (415) (100) (400)/(103.896) (103) = 139.005 mm

Hence, provide 8 mm, 2 legged vertical stirrups @ 130 mm c/c, as shown in Fig.
6.14.4.

Version 2 CE IIT, Kharagpur


(b) Condition (ii):

Additional stirrup area for a distance of 0.75 d {= 0.75 (400) = 300 mm} =
0.4 b s/fy, where spacing s is not greater than (d/8βb), where βb = cut off bar
area/total bar area = 2048/3066 = 0.67. Since, additional stirrups are of lower
diameter, mild steel bars are preferred with fy = 250 N/mm2. Maximum spacing s
= d/8βb = 400/8 (0.67) = 75 mm. Excess area = 0.4 b s/fy = 0.4 (300) (75)/250 =
36 mm2.

Provide 6 mm, 2 legged mild steel vertical stirrups (56 mm2) @ 75 mm c/c
for a distance of 300 mm, i.e., five numbers of stirrups (additional), as shown in
Fig. 6.14.5.

Problem 3:

Design the flexural and shear reinforcement of the simply supported T-


beam (Fig. 6.14.6) of effective span 8 m placed @ 4.2 m c/c and subjected to a
total factored load of 150 kN/m. Use M 30, Fe 415 and SP-16 tables for the
design of flexural reinforcement.

Solution 3:

Design of flexural reinforcement with SP-16:

Effective width of flange bf = lo/6 + bw + 6 Df = 8000/6 + 300 + 6(120) = 2353


mm > 2100 mm (breadth of the web plus half the sum of the clear distance to the
adjacent beams on either side).

So, bf = 2100 mm.


(Mu)factored = 150 (8) (8)/(8) = 1200 kNm, at the mid-span.

Version 2 CE IIT, Kharagpur


(Vu)factored = 150 (8)/(2) = 600 kN, at the support.

The bending moment and shear force diagrams are shown in Fig. 6.14.7. At the
mid-span

Mu 1200 (106 )
= = 0.37
bw d 2 f ck 300 (600) (600) (30)

Df /d = 120/600 = 0.2 and bf /bw = 2100/300 = 7


Table 58 of SP-16 (fy = 415 N/mm2) gives:

Version 2 CE IIT, Kharagpur


M u ,lim
= 0.62 (when bf /bw = 7.0 and Df /d = 0.2) > 0.37 in
b d 2 f ck
this case.

Hence, o.k.

Mu 1200 (10 6 )
Ast = = = 6155 mm 2
0.87 f y (d - d f /2) 0.87 (415) (540)

Provide 7-32T + 1-28T (= 6245 mm2) bars at mid-span and up to section 5-5 (Fig.
6.14.8, sec. 5-5). The flexural reinforcement is cranked up and the reinforcement
diagrams are shown at five sections in Fig. 6.14.8.

Version 2 CE IIT, Kharagpur


Design of shear reinforcement:

The details of calculations are shown below for the section 1-1 in six
steps. Results of all four sections are presented in Table 6.3.

Step 1:

Ast at section 1-1 is determined (= 3217 mm2 = 4-32T) from Fig. 6.14.7 to
calculate p = Ast (100)/bw d = 3217 (100)/300 (600) = 1.79%. From Table 6.1 of
Lesson 13, τc is determined for p = 1.79% as 0.81 N/mm2. Table 6.2 of Lesson
13 gives τc,max for M 30 = 3.5 N/mm2.

Step 2:

Vu is 600 kN at section 1-1 from Fig.6.14.7 to get τv from Eq.6.1 of Lesson


13 as:

Vu (600) (10 3 )
τv = = = 3.33 N/mm 2
bw d (300) (600)

Here, τv is less than τcmax.

Step 3:

The magnitude of shear force for which shear reinforcement is needed


(say, Vreinf.) is determined from Fig. 6.14.7. For section 1-1, from Eq. 6.4 of
Lesson 13, Vreinf = Vu – τc b d = 600 – 0.81 (300) (600) = 454.2 kN.

The magnitude of shear force taken by bent up bar(s) is obtained from Eq.
6.7 of Lesson 13, Vbent = 0.87 fy Asv sinα = 0.87 (415) (804) (1/√2) (10-3) = 206.5
kN. This force should not be greater than 0.5 (Vreinf), which, at this section, is
227.1 kN (vide sec. 6.13.8 of Lesson 13).

The magnitude of the shear force for the design of vertical stirrup = Vus =
Vreinf – Vbent = 454.2 – 206.5 = 247.7 kN.

Step 4:

Assume the diameter of vertical stirrup bars as 10 mm, 2 legged (Asv =


157 mm2). The spacing of vertical stirrups sv = 0.87 fy Asv d/Vus = 0.87 (415)
(157) (600)/247.7 (103) = 137.3 mm c/c. Please refer to Eq.6.5 of Lesson 13.

Version 2 CE IIT, Kharagpur


Step 5:

Check sv considering minimum shear reinforcement from cl. 26.5.1.6 of IS


456 as (see Eq.6.3 of Lesson 13):

sv ≤ 0.87 fy Asv /0.4 bw ≤ 0.87 (415) (157) /0.4 (300) ≤ 472 mm

Further, cl. 26.5.1.5 stipulates the maximum spacing = 0.75 d on 300 mm. Here,
the maximum spacing = 300 mm.

Step 6:

So, provide 10 mm, 2 legged stirrups @ 135 mm c/c.

The results of all the sections are given below:

Table 6.3 Design of stirrups using 10 mm 2 legged vertical stirrups, (τcmax = 3.5
N/mm2)

Step Values of Sec. 1-1 Sec. 2-2 Sec. 3-3 Sec. 4-4
Ast (mm2) 3,217 4,021 4,825 5,629
1 p (%) 1.79 2.23 2.68 3.13
τc (N/mm2) 0.81 0.877 0.93 0.96
Vu (kN) 600 525 450 375
2 τv (N/mm2) 3.33 2.92 2.50 2.08

Vreinf (kN) 454.2 367.14 282.6 202.2


3 Vbent (kN) 206.5* 206.5 206.5 157.24**
< 0.5 Vreinf < 183.57 < 141.3 < 101.1
Vus (kN) 247.7 183.57 141.3 101.1
4 sv (mm) 137.3 185.27 240.7 336.4
5 Min sv (mm) ≤ 300 ≤ 300 ≤ 300 ≤ 300
6 Provide sv (mm) 135 180 240 300

* diameter of bent up bar = 32 mm

** diameter of bent up bar = 28 mm

To avoid several spacings for the practical consideration, provide stirrups @ 130
mm c/c for first 1 m, @ 230 mm for next 1 m and then @ 300 mm up to the mid-
span in a symmetric manner (Fig. 6.14.8, secs. 1-1 to 5-5).

Version 2 CE IIT, Kharagpur


6.14.3 Practice Questions and Problems with Answers
Q.1: Check if shear reinforcement is needed for the beam shown in Fig. 6.14.9.
If so, design the shear reinforcement using M 20 and mild steel (Fe 250).
The tensile reinforcement is of Fe 415. Factored shear force = 300 kN.

A.1:

Ast = 4-25T = 1963 mm2

p = 1963/300(500) = 1.31%

τc from Table 6.1 of sec. 6.13.4.1 of Lesson 13 = 0.68 N/mm2

From Table 6.2 of sec. 6.13.4.2 of Lesson 13 τcmax = 2.8 N/mm2

τv = Vu /b d (Eq.6.1 of Lesson 13) = 300000/300(500) = 2.0 N/mm2

Since τc < τv < τcmax, shear reinforcement is needed. From Eq.6.4 of Lesson
13:

Vus = Vu – τc b d = 300 – 0.68(300)(500)(10-3) = 198 kN

Alternative 1: Providing 10 mm, 2 legged stirrups (Asv = 157 mm2), the


spacing sv = 0.87(250)(157)(500)/198000 = 86.23 mm. Provide 10 mm, 2 legged
stirrups @ 85 mm c/c. Please refer to Eq.6.5 of Lesson 13. The required area of
stirrups spaced @ 85 mm c/c to satisfy the minimum shear reinforcement (cl.

Version 2 CE IIT, Kharagpur


26.5.1.6 of IS 456) is obtained from Eq. 6.3 of Lesson 13 as: Asv =
0.4(300)(85)/0.87(250) = 46.89 mm2 < 157 mm2 .

Alternative 2: Providing 12 mm, 2 legged stirrups (Asv = 226 mm2), the


spacing sv = 0.87(250)(226)(500)/198000 = 124.13 mm. Provide 12 mm, 2
legged stirrups @ 120 mm c/c. The required area of stirrups spaced @ 120 mm
c/c to satisfy the minimum shear reinforcement (cl. 26.5.1.6 of IS 456) is obtained
from Eq. 6.3 of Lesson 13 as: Asv = 0.4(300)(120)/0.87(250) = 66.21 mm2 <
226 mm2 .

Further, the maximum spacing (cl. 26.5.1.5 of IS 456 and sec. 6.13.7 of
Lesson 13) = 0.75 d = 0.75(500) = 375 mm.

Hence, both are possible, though 12 mm @ 120 mm c/c is desirable since the
other spacing of 85 mm c/c is very close (Fig. 6.14.9).

6.14.4 References:
1. Reinforced Concrete Limit State Design, 6th Edition, by Ashok K. Jain,
Nem Chand & Bros, Roorkee, 2002.
2. Limit State Design of Reinforced Concrete, 2nd Edition, by P.C.Varghese,
Prentice-Hall of India Pvt. Ltd., New Delhi, 2002.
3. Advanced Reinforced Concrete Design, by P.C.Varghese, Prentice-Hall of
India Pvt. Ltd., New Delhi, 2001.
4. Reinforced Concrete Design, 2nd Edition, by S.Unnikrishna Pillai and
Devdas Menon, Tata McGraw-Hill Publishing Company Limited, New
Delhi, 2003.
5. Limit State Design of Reinforced Concrete Structures, by P.Dayaratnam,
Oxford & I.B.H. Publishing Company Pvt. Ltd., New Delhi, 2004.
6. Reinforced Concrete Design, 1st Revised Edition, by S.N.Sinha, Tata
McGraw-Hill Publishing Company. New Delhi, 1990.
7. Reinforced Concrete, 6th Edition, by S.K.Mallick and A.P.Gupta, Oxford &
IBH Publishing Co. Pvt. Ltd. New Delhi, 1996.
8. Behaviour, Analysis & Design of Reinforced Concrete Structural Elements,
by I.C.Syal and R.K.Ummat, A.H.Wheeler & Co. Ltd., Allahabad, 1989.
9. Reinforced Concrete Structures, 3rd Edition, by I.C.Syal and A.K.Goel,
A.H.Wheeler & Co. Ltd., Allahabad, 1992.
10. Textbook of R.C.C, by G.S.Birdie and J.S.Birdie, Wiley Eastern Limited,
New Delhi, 1993.
11. Design of Concrete Structures, 13th Edition, by Arthur H. Nilson, David
Darwin and Charles W. Dolan, Tata McGraw-Hill Publishing Company
Limited, New Delhi, 2004.
12. Concrete Technology, by A.M.Neville and J.J.Brooks, ELBS with
Longman, 1994.

Version 2 CE IIT, Kharagpur


13. Properties of Concrete, 4th Edition, 1st Indian reprint, by A.M.Neville,
Longman, 2000.
14. Reinforced Concrete Designer’s Handbook, 10th Edition, by C.E.Reynolds
and J.C.Steedman, E & FN SPON, London, 1997.
15. Indian Standard Plain and Reinforced Concrete – Code of Practice (4th
Revision), IS 456: 2000, BIS, New Delhi.
16. Design Aids for Reinforced Concrete to IS: 456 – 1978, BIS, New Delhi.

6.14.5 Test 14 with Solutions


Maximum Marks = 50, Maximum Time = 30 minutes

Answer all questions.

TQ. 1: The T-beam of Fig. 6.14.10 has a factored shear force of 400 kN.
Determine the diameter and spacing of vertical stirrups at a section
where two 25 mm diameter bent up bears are also available for the
shear resistance. Use M 20 and Fe 415.
(50
marks)

A.TQ. 1:

Ast = 4-25 φ = 1963 mm2

Version 2 CE IIT, Kharagpur


p = 1963(100)/300(550) = 1.19%

τc = 0.658 N/mm2 (Table 6.1 of Lesson 13)

Vu = 400 kN

τv (Eq.6.1 of Lesson 13) = 400/165 = 2.43 N/mm2 < τcmax of 2.8


2
N/mm .

Hence, o.k.

Vreinf. = Vu – τc b d = 400 – 0.658(300)(0.55) = 291.43 kN (please


refer to Eq.6.4 of Lesson 13)

Vbent (2-25 mm diameter) = 0.87 fy Asv sinα = 0.87(415)(981)(1/√2)

= 250.48 kN,

subjected to the maximum value of 0.5(Vreinf.) = 145.71 kN (see sec.


6.13.8 of Lesson 13)

Vus = 145.72 kN

Using 10 mm, 2 legged vertical stirrups (Asv = 157 mm2), the spacing,
obtained from Eq.6.5 of Lesson 13, sv = 0.87 fy Asv d/Vus = 0.87 (415)
(157) (550)/145720 = 213.95 mm c/c.

For the minimum shear of reinforcement as per cl. 26.5.1.6 of IS 456,


using 10 mm, 2 legged vertical stirrups, the spacing as obtained from
Eq.6.3 of Lesson 13:

sv ≤ 0.87 fy Asv /0.4 bw ≤ 0.87 (415) (157) /0.4 (300) ≤ 472 mm

Again, cl. 26.5.1.5 of IS 456 stipulates the maximum spacing = 0.75 d =


0.75(550) = 412.5 mm (see sec. 6.13.7 of Lesson 13).

Hence, provide 10 mm, 2 legged vertical stirrups @ 210 mm c/c (Fig.


6.14.10).

6.14.6 Summary of this Lesson


Learning the different failure modes, the shear stresses and the design
procedure of beams subjected to shear in Lesson 13, this lesson explains the
design through several numerical problems with special reference to curtailment
of tension reinforcement in flexural members. Solution of problems given in the

Version 2 CE IIT, Kharagpur


practice problems and test, students will be thoroughly conversant with the
design of rectangular and T-beams subjected to shear following the limit state of
collapse.

Version 2 CE IIT, Kharagpur


Module
6
Shear, Bond,
Anchorage,
Development Length
and Torsion
Version 2 CE IIT, Kharagpur
Lesson
15
Bond, Anchorage,
Development Length
and Splicing
Version 2 CE IIT, Kharagpur
Instruction Objectives:
At the end of this lesson, the student should be able to:

• understand the importance of bond and why is it essential to provide


between steel and concrete,

• explain the development length,

• understand the need for anchoring the tensile bars,

• justify the superiority of deformed bars to smooth mild steel bars,

• define the design bond stress,

• understand the reason for different values of design bond stresses of plain
bars and deformed bars in tension and compression,

• determine the development length of a single bar and bars bundled in


contact,

• derive the expression to check the development length of bars in tension,

• specify the salient points of anchoring bars in tension, compression and


shear,

• determine the bearing stress and check the same,

• take adequate precautions when the direction of reinforcement changes,

• specify the salient points of splicing and welding of reinforcement to make


it longer,

• apply the theory for designing beams in different situations as may arise.

6.15.1 Introduction
The bond between steel and concrete is very important and essential so
that they can act together without any slip in a loaded structure. With the perfect
bond between them, the plane section of a beam remains plane even after
bending. The length of a member required to develop the full bond is called the
anchorage length. The bond is measured by bond stress. The local bond stress
varies along a member with the variation of bending moment. The average value

Version 2 CE IIT, Kharagpur


throughout its anchorage length is designated as the average bond stress. In our
calculation, the average bond stress will be used.

Thus, a tensile member has to be anchored properly by providing


additional length on either side of the point of maximum tension, which is known
as ‘Development length in tension’. Similarly, for compression members also, we
have ‘Development length Ld in compression’.

It is worth mentioning that the deformed bars are known to be superior to


the smooth mild steel bars due to the presence of ribs. In such a case, it is
needed to check for the sufficient development length Ld only rather than
checking both for the local bond stress and development length as required for
the smooth mild steel bars. Accordingly, IS 456, cl. 26.2 stipulates the
requirements of proper anchorage of reinforcement in terms of development
length Ld only employing design bond stress τbd.

6.15.2 Design Bond Stress τbd


(a) Definition

The design bond stress τbd is defined as the shear force per unit nominal
surface area of reinforcing bar. The stress is acting on the interface between bars
and surrounding concrete and along the direction parallel to the bars.

This concept of design bond stress finally results in additional length of a


bar of specified diameter to be provided beyond a given critical section. Though,
the overall bond failure may be avoided by this provision of additional
development length Ld, slippage of a bar may not always result in overall failure
of a beam. It is, thus, desirable to provide end anchorages also to maintain the
integrity of the structure and thereby, to enable it carrying the loads. Clause 26.2
of IS 456 stipulates, “The calculated tension or compression in any bar at any
section shall be developed on each side of the section by an appropriate
development length or end anchorage or by a combination thereof.”

(b) Design bond stress – values

Section 6.15.1 mentions that the local bond stress varies along the length
of the reinforcement while the average bond stress gives the average value
throughout its development length. This average bond stress is still used in the
working stress method and IS 456 has mentioned about it in cl. B-2.1.2.
However, in the limit state method of design, the average bond stress has been
designated as design bond stress τbd and the values are given in cl. 26.2.1.1.
The same is given below as a ready reference.

Version 2 CE IIT, Kharagpur


Table 6.4 τbd for plain bars in tension

Grade of M 20 M 25 M 30 M 35 M 40 and
concrete above
Design
Bond 1.2 1.4 1.5 1.7 1.9
Stress τbd
in N/mm2

For deformed bars conforming to IS 1786, these values shall be increased by 60


per cent. For bars in compression, the values of bond stress in tension shall be
increased by 25 per cent.

6.15.3 Development Length

(a) A single bar

Figure 6.15.1(a) shows a simply supported beam subjected to uniformly


distributed load. Because of the maximum moment, the Ast required is the
maximum at x = L/2. For any section 1-1 at a distance x < L/2, some of the

Version 2 CE IIT, Kharagpur


tensile bars can be curtailed. Let us then assume that section 1-1 is the
theoretical cut-off point of one bar. However, it is necessary to extend the bar for
a length Ld as explained earlier. Let us derive the expression to determine Ld of
this bar.

Figure 6.15.1(b) shows the free body diagram of the segment AB of the
bar. At B, the tensile force T trying to pull out the bar is of the value T = (π φ 2
σs /4), where φ is the nominal diameter of the bar and σs is the tensile stress in
bar at the section considered at design loads. It is necessary to have the
resistance force to be developed by τbd for the length Ld to overcome the
tensile force. The resistance force = π φ (Ld) (τbd). Equating the two, we get

π φ (Ld) (τbd) = (π φ 2 σs /4)


(6.11)

Equation 6.11, thus gives

φ σs
Ld =
4 τ bd
(6.12)

The above equation is given in cl. 26.2.1 of IS 456 to determine the development
length of bars.

The example taken above considers round bar in tension. Similarly, other
sections of the bar should have the required Ld as determined for such sections.
For bars in compression, the development length is reduced by 25 per cent as
the design bond stress in compression τbd is 25 per cent more than that in
tension (see the last lines below Table 6.4). Following the same logic, the
development length of deformed bars is reduced by 60 per cent of that needed
for the plain round bars. Tables 64 to 66 of SP-16 present the development
lengths of fully stressed plain and deformed bars (when σs = 0.87 fy) both under
tension and compression. It is to be noted that the consequence of stress
concentration at the lugs of deformed bars has not been taken into consideration.

(b) Bars bundled in contact

The respective development lengths of each of the bars for two, three or
four bars in contact are determined following the same principle. However, cl.
26.2.1.2 of IS 456 stipulates a simpler approach to determine the development
length directly under such cases and the same is given below:

Version 2 CE IIT, Kharagpur


“The development length of each bar of bundled bars shall be that for the
individual bar, increased by 10 per cent for two bars in contact, 20 per cent for
three bars in contact and 33 per cent for four bars in contact.”

However, while using bundled bars the provision of cl. 26.1.1 of IS 456
must be satisfied. According to this clause:

• In addition to single bar, bars may be arranged in pairs in contact or in


groups of three or four bars bundled in contact.

• Bundled bars shall be enclosed within stirrups or ties to ensure the bars
remaining together.

• Bars larger than 32 mm diameter shall not be bundled, except in columns.

Curtailment of bundled bars should be done by terminating at different


points spaced apart by not less than 40 times the bar diameter except for
bundles stopping at support (cl. 26.2.3.5 of IS 456).

6.15.4 Checking of Development Lengths of Bars in Tension


The following are the stipulation of cl. 26.2.3.3 of IS 456.

(i) At least one-third of the positive moment reinforcement in simple members


and one-fourth of the positive moment reinforcement in continuous members
shall be extended along the same face of the member into the support, to a
length equal to Ld/3.

(ii) Such reinforcements of (i) above shall also be anchored to develop its design
stress in tension at the face of the support, when such member is part of the
primary lateral load resisting system.

(iii) The diameter of the positive moment reinforcement shall be limited to a


diameter such that the Ld computed for σs = fd in Eq. 6.12 does not exceed the
following:

M1
( Ld ) when σ s = fd ≤ + Lo
V
(6.13)

where M1 = moment of resistance of the section assuming all reinforcement at


the section to be stressed to fd,

fd = 0.87 fy,

V = shear force at the section due to design loads,


Version 2 CE IIT, Kharagpur
Lo = sum of the anchorage beyond the centre of the support and the
equivalent anchorage value of any hook or mechanical anchorage
at simple support. At a point of inflection, Lo is limited to the
effective depth of the member or 12 φ , whichever is greater, and

φ = diameter of bar.

It has been further stipulated that M1/V in the above expression may be
increased by 30 per cent when the ends of the reinforcement are confined by a
compressive reaction.

6.15.5 Derivation of the Limiting Ld (Eq. 6.13)

At the face of simple support and at the points of inflection in continuous


beams, the tensile capacity to be developed is normally small although the rate
of change of tensile stress in the bars is high. These bond stresses are
designated as flexural bond stress.

Figure 6.15.2(a) shows the tensile bar AB near the support of a beam and
Fig. 6.15.2(b) explains the stresses and forces in the free body diagram of the

Version 2 CE IIT, Kharagpur


segment CD of the bar. The tensile force at C (TC) is greater than that at D (TD).
Considering z as the lever arm (distance between the centres of gravity of tensile
and compressive force), we have

MC = TC (z)

(6.14)
MD = TD (z)

Again TC - TD = πφ (dx) τ bd
(6.15)

From Eqs.6.14 and 6.15,

MC - MD
= πφτ bd (dx )
z

which gives

dM
= πφ z τ bd
dx

V
or τ bd =
πφz
(6.16)

Equation 6.16 gives the flexural bond stress in the tension reinforcement
at any section. If there are N bars of equal size, we have

V
τ bd =
z ( ∑ 0)
(6.17)

where ∑ 0 = N π φ = total perimeter of all bars in tension at the section.

Again, for N bars of equal diameter, we get from Eq. 6.11:

N π φ Ld τ bd = Ast σ s = Ast 0.87 f y (when σ s = 0.87 f y ),

which gives:

Version 2 CE IIT, Kharagpur


0.87 f y Ast
τ bd =
Ld (∑ 0)
(6.18)

Equations 6.17 and 6.18 give:

0.87 f y Ast V
=
Ld (∑ 0) z (∑ 0)

M1
or Ld =
V
(6.19)

where M 1 = 0.87 f y Ast z is the moment of resistance at the section and

V = shear force at the section

From Eq.6.19, we find that the ratio of M1/V at the section must be equal to or
greater than Ld if the design bond stress τbd is to be restricted within limit. The
stipulation of additional Lo in the expression of Eq.6.13 is for additional safety.
The meaning of Lo has been mentioned in sec. 6.15.4 (iii).

6.15.6 Anchoring Reinforcing Bars


Section 6.15.2(a) mentions that the bars may be anchored in combination
of providing development length to maintain the integrity of the structure. Such
anchoring is discussed below under three sub-sections for bars in tension,
compression and shear respectively, as stipulated in cl. 26.2.2 of IS 456.

Version 2 CE IIT, Kharagpur


(a) Bars in tension (cl. 26.2.2.1 of IS 456)

The salient points are:

• Derformed bars may not need end anchorages if the development length
requirement is satisfied.

• Hooks should normally be provided for plain bars in tension.

• Standard hooks and bends should be as per IS 2502 or as given in Table


67 of SP-16, which are shown in Figs.6.15.3 a and b.

• The anchorage value of standard bend shall be considered as 4 times the


diameter of the bar for each 45o bend subject to a maximum value of 16
times the diameter of the bar.

• The anchorage value of standard U-type hook shall be 16 times the


diameter of the bar.

(b) Bars in compression (cl. 26.2.2.2 of IS 456)

Here, the salient points are:

Version 2 CE IIT, Kharagpur


• The anchorage length of straight compression bars shall be equal to its
development length as mentioned in sec. 6.15.3.

• The development length shall include the projected length of hooks, bends
and straight lengths beyond bends, if provided.

(c) Bars in shear (cl. 26.2.2.4 of IS 456)

The salient points are:

• Inclined bars in tension zone will have the development length equal to
that of bars in tension and this length shall be measured from the end of
sloping or inclined portion of the bar.

Version 2 CE IIT, Kharagpur


• Inclined bars in compression zone will have the development length equal
to that of bars in tension and this length shall be measured from the mid-
depth of the beam.

• For stirrups, transverse ties and other secondary reinforcement, complete


development length and anchorage are considered to be satisfied if
prepared as shown in Figs.6.15.4.

6.15.7 Bearing Stresses at Bends (cl. 26.2.2.5 of IS 456)


The bearing stress inside a bend is to be calculated from the expression:

Fbt
Bearing stress =

(6.20)

where Fbt = tensile force due to design loads in a bar or group of bars,

r = internal radius of the bend, and

φ = size of the bar or bar of equivalent area in bundled bars.

The calculated bearing stress of Eq.6.20 shall not exceed the following:

1.5 f ck
Calculated bearing stress >/
1 + 2 φ /a
(6.21)

where fck = characteristic cube strength of concrete

a = center to center distance between bars or groups of bars


perpendicular to the plane of the bend. For bars adjacent to the face
of the member, a shall be taken as cover plus size of the bar φ .

6.15.8 Change in Direction of Reinforcement (cl. 26.2.2.6 of


IS 456)
In some situations, the change in direction of tension or compression
reinforcement induces a resultant force. This force may have a tendency to split
the concrete and, therefore, should be taken up by additional links or stirrups.
Normally, this aspect is taken care while detailing of bars is carried out.

Version 2 CE IIT, Kharagpur


6.15.9 Reinforcement Splicing (cl. 26.2.5 of IS 456)
Reinforcement is needed to be joined to make it longer by overlapping
sufficient length or by welding to develop its full design bond stress. They should
be away from the sections of maximum stress and be staggered. IS 456 (cl.
26.2.5) recommends that splices in flexural members should not be at sections
where the bending moment is more than 50 per cent of the moment of resistance
and not more than half the bars shall be spliced at a section.

(a) Lap Splices (cl. 26.2.5.1 of IS 456)

The following are the salient points:

• They should be used for bar diameters up to 36 mm.

• They should be considered as staggered if the centre to centre distance of


the splices is at least 1.3 times the lap length calculated as mentioned
below.

• The lap length including anchorage value of hooks for bars in flexural
tension shall be Ld or 30 φ , whichever is greater. The same for direct
tension shall be 2Ld or 30 φ , whichever is greater.

• The lap length in compression shall be equal to Ld in compression but


not less than 24 φ .

• The lap length shall be calculated on the basis of diameter of the smaller
bar when bars of two different diameters are to be spliced.

• Lap splices of bundled bars shall be made by splicing one bar at a time
and all such individual splices within a bundle shall be staggered.

(b) Strength of Welds (cl. 26.2.5.2 of IS 456)

The strength of welded splices and mechanical connections shall be


taken as 100 per cent of the design strength of joined bars for compression
splices.

For tension splices, such strength of welded bars shall be taken as 80 per
cent of the design strength of welded bars. However, it can go even up to 100
per cent if welding is strictly supervised and if at any cross-section of the member
not more than 20 per cent of the tensile reinforcement is welded. For mechanical
connection of tension splice, 100 per cent of design strength of mechanical
connection shall be taken.

Version 2 CE IIT, Kharagpur


6.15.10 Numerical Problems

Problem 1:

Determine the anchorage length of 4-20T reinforcing bars going into the
support of the simply supported beam shown in Fig. 6.15.5. The factored shear
force Vu = 280 kN, width of the column support = 300 mm. Use M 20 concrete
and Fe 415 steel.

Solution 1:

Table 6.4 gives

τbd for M 20 and Fe 415 (with 60% increased) = 1.6(1.2) = 1.92 N/mm2

Eq.6.12 gives

φ σs 0.87(415) φ
Ld = = (when σ s = 0.87 f y ) = 47.01φ ...... (1)
4 τ bd 4(1.92)

Eq.6.13 gives

M1
( Ld ) when σ s = f d ≤ + Lo
V

Here, to find M1, we need xu

Version 2 CE IIT, Kharagpur


0.87 f y Ast 0.87(415) (1256)
xu = = = 209.94 mm
0.36 f ck b 0.36(20) (300)

xu,max = 0.48(500) = 240 mm

Since xu < xu,max ; M1 = 0.87 fy Ast (d – 0.42 xu)

or M1 = 0.87(415) (1256) {500 – 0.42(209.94)} = 187.754 kNm

and V = 280 kN

We have from Eq.6.13 above, with the stipulation of 30 per cent increase
assuming that the reinforcing bars are confined by a compressive reaction:

M1
Ld ≤ 1.3 ( ) + Lo ...... (2)
V

From Eqs.(1) and (2), we have

M1
47.01φ ≤ 1.3 ( ) + Lo
V

187.754(10 6 )
or 47.01φ ≤ 1.3 { }; if Lo is assumed as zero.
280(10 3 )

or φ ≤ 18.54 mm

Therefore, 20 mm diameter bar does not allow Lo = 0.

Determination of Lo:

M1
1.3 ( ) + Lo ≥ 47.01φ
V

M1 187754
Minimum Lo = 47.01φ - 1.3 ( ) = 47.01(20) - 1.3( ) = 68.485 mm
V 280

So, the bars are extended by 100 mm to satisfy the requirement as shown in
Fig.6.15.6.

Version 2 CE IIT, Kharagpur


6.15.11 Practice Questions and Problems with Answers
Q.1: Explain the importance of the bond and why is it essential to provide
between steel and concrete in beams?

A.1: Sec. 6.15.1 para 1

Q.2: Define the design bond stress τ bd .

A.2: Sec. 6.15.2(a)

Q.3: State the percentage increase/decrease of design bond stress of deformed


bars in tension and compression with reference to the respective values of
plain bars.

A.3: Three lines below Table 6.4 of sec. 6.15.2(b)

Q.4: Derive the expression of determining the development length of a single


bar in tension. State the changes, if any, for the compression bars.

A.4: Sec. 6.15.3(a).

Q.5: How would you determine the development lengths of bars when two,
three or four bars are bundled in contact? State the salient points of the
stipulations of IS 456 in this respect.

A.5: Sec. 6.15.3(b)

Version 2 CE IIT, Kharagpur


Q.6: Derive the limiting value of the development length for bars in tension
having both bending moment and shear force. Explain the role of
additional length Lo.

A.6: Sec. 6.15.5 and the final form is given in Eq.6.13.

Q.7: State the salient points of the stipulations of IS 456 regarding anchoring
reinforcing bars in tension, compression and shear, respectively.

A.7: Sec. 6.15.6(a), (b) and (c) are the respective answers.

Q.8: Write down the expressions of calculated bearing stress at bends and its
limiting value.

A.8: Eqs.6.20 and 6.21 are the answers.

Q.9: State the additional measure to be taken when the reinforcing bars change
the direction.

A.9: Sec. 6.15.8.

Q.10: State the salient points of splicing and welding of reinforcing bars.

A.10: Sec. 6.15.9(a) and (b), respectively, are the answers.

Q.11: Check the bond requirement of the continuous beam of Fig.6.15.7 if the
factored shear force is 200 kN at the point of inflection. Assume M 20 and
Fe 415.

Version 2 CE IIT, Kharagpur


A.11: Table 6.4 gives τbd for M 20 and Fe 415 as 1.92 N/mm2 (see Solution 1 of
Problem 1 of sec. 6.15.10).

Eq.6.12 gives

φ σs 0.87(415) φ
Ld = = = 47.01φ ...... (3)
4 τ bd 4(1.92)

Eq.6.13 gives

M1
( Ld ) when σ s = f d ≤ + Lo
V

To find M1, we need xu

0.87 f y Ast 0.87(415) (942)


xu = = = 157.46 mm
0.36 f ck b 0.36(20) (300)

xu,max = 0.48(400) = 192 mm

Since xu < xu,max ; M1 = 0.87 fy Ast (d – 0.42 xu)

= 0.87(415) (942) {400 – 0.42(157.46)} = 113.55 kNm ….. (4)

From Eqs. (3) and (4), we have

M1
47.01φ ≤ ( ) + Lo ...... (5)
V

Version 2 CE IIT, Kharagpur


At the point of inflection Lo is the maximum of d (= 400 mm) or 12φ ( = 240 mm).

With Lo = 400 mm, we can write from Eq.(5)

113.55(10 6 )
47.01φ ≤ { } + 400
200(10 3 )

which gives φ ≤ 20.58 mm.

Thus, use of φ = 20 mm satisfies the bond requirement.

6.15.12 References:
1. Reinforced Concrete Limit State Design, 6th Edition, by Ashok K. Jain,
Nem Chand & Bros, Roorkee, 2002.
2. Limit State Design of Reinforced Concrete, 2nd Edition, by P.C.Varghese,
Prentice-Hall of India Pvt. Ltd., New Delhi, 2002.
3. Advanced Reinforced Concrete Design, by P.C.Varghese, Prentice-Hall of
India Pvt. Ltd., New Delhi, 2001.
4. Reinforced Concrete Design, 2nd Edition, by S.Unnikrishna Pillai and
Devdas Menon, Tata McGraw-Hill Publishing Company Limited, New
Delhi, 2003.
5. Limit State Design of Reinforced Concrete Structures, by P.Dayaratnam,
Oxford & I.B.H. Publishing Company Pvt. Ltd., New Delhi, 2004.
6. Reinforced Concrete Design, 1st Revised Edition, by S.N.Sinha, Tata
McGraw-Hill Publishing Company. New Delhi, 1990.
7. Reinforced Concrete, 6th Edition, by S.K.Mallick and A.P.Gupta, Oxford &
IBH Publishing Co. Pvt. Ltd. New Delhi, 1996.
8. Behaviour, Analysis & Design of Reinforced Concrete Structural Elements,
by I.C.Syal and R.K.Ummat, A.H.Wheeler & Co. Ltd., Allahabad, 1989.
9. Reinforced Concrete Structures, 3rd Edition, by I.C.Syal and A.K.Goel,
A.H.Wheeler & Co. Ltd., Allahabad, 1992.
10. Textbook of R.C.C, by G.S.Birdie and J.S.Birdie, Wiley Eastern Limited,
New Delhi, 1993.
11. Design of Concrete Structures, 13th Edition, by Arthur H. Nilson, David
Darwin and Charles W. Dolan, Tata McGraw-Hill Publishing Company
Limited, New Delhi, 2004.
12. Concrete Technology, by A.M.Neville and J.J.Brooks, ELBS with
Longman, 1994.
13. Properties of Concrete, 4th Edition, 1st Indian reprint, by A.M.Neville,
Longman, 2000.
14. Reinforced Concrete Designer’s Handbook, 10th Edition, by C.E.Reynolds
and J.C.Steedman, E & FN SPON, London, 1997.

Version 2 CE IIT, Kharagpur


15. Indian Standard Plain and Reinforced Concrete – Code of Practice (4th
Revision), IS 456: 2000, BIS, New Delhi.
16. Design Aids for Reinforced Concrete to IS: 456 – 1978, BIS, New Delhi.

6.15.13 Test 15 with Solutions


Maximum Marks = 50, Maximum Time = 30 minutes

Answer all questions.

TQ.1: Derive the expression of determining the development length of a single


bar in tension. State the changes, if any, for the compression bars.
(10 marks)

A.TQ.1: Sec. 6.15.3(a).

TQ.2: Derive the limiting value of the development length for bars in tension
having both bending moment and shear force. Explain the role of
additional length Lo.
(10 marks)

A.TQ.2: Sec. 6.15.5 and the final form is given in Eq.6.13.

TQ.3: State the salient points of the stipulations of IS 456 regarding anchoring
reinforcing bars in tension, compression and shear, respectively.
(10 marks)

A.TQ.3: Sec. 6.15.6(a), (b) and (c) are the respective answers.

TQ.4: Design the main tensile reinforcement of a singly reinforced rectangular


beam of width 300 mm and effective span of 8 m carrying a total factored
load of 40 kN/m using M 20 and Fe 415 with the help of SP-16. Check the
development length at the support if 50 per cent of the reinforcing bars are
continued to the support. Assume width of the support as 300 mm.
(20 marks)

A.TQ.4: Design of beam using SP-16:

Factored mid-span bending moment = 40(8)(8)/8 = 320 kNm

Factored shear at the support = 40(8)/2 = 160 kN

Required moment of resistance per metre width = 320/0.3 = 1067


kNm/m

Version 2 CE IIT, Kharagpur


Chart 15 of SP-16 gives d = 63.25 cm and percentage of reinforcement =
0.9.

Using the total depth D = 700 mm and effective depth as 650 mm, we get
the area of main tensile reinforcement at mid-span = 0.9(bd)/100 =
0.9(300)(650)/100 = 1755 mm2.

Let us provide 6-20T of area = 1885 mm2.

Three 20T bars (50 per cent) are continued at the support. The value of
τ bd for M 20 and Fe 415 is obtained from Table 6.4 with 60 per cent
increased as 1.92 N/mm2. Eq.6.12 gives

Ld = φ σ s /4(τ bd ) = (20)(0.87)(415)/4(1.92) = 940.23 mm


(6)

Eq.6.13 gives ( Ld ) when σ s = f d ≤ ( M 1 / V ) + Lo . To find M1, we need xu as


given below:

xu = (0.87 fy Ast)/(0.36 fck b)

= 0.87(415)(942.5)/(0.36)(20)(300)

= 157.54 mm < xu,max {= 0.48(650) = 312 mm}

So,

M1 = 0.87 fy Ast (d – 0.42 xu)

= 0.87(415)(942.5){(650) – 0.42(157.54)}

= 198.67 kNm

We have from Eq.6.13 with the stipulation of increase of 30 per cent


assuming that the reinforcing bars are confined by a compressive
reaction:

Ld ≤ 1.3(M1/V) + Lo
(7)

From Eqs.6 and 7 above, we have

940.23 ≤ 1.3(M1/V) + Lo
(8)

Version 2 CE IIT, Kharagpur


Here, 1.3(M1/V) = 1.3(198.67)/160 = 1.614 m

Hence, Lo is not required as Ld is satisfied when Lo = 0 (sec. Eq.8).

6.15.14 Summary of this Lesson


This lesson explains the importance of bond between steel and concrete
which is essential for the two materials to act together. To ensure the bond,
additional length of the reinforcing bars is to be provided, which is known as the
development length and calculated on the basis of design bond stresses. The
design bond stress has been defined in this lesson and the values of the design
bond stresses for different grades of concrete are given for mild steel plain and
ribbed steel bars. The expression of determination of development length has
been derived. The stipulations of IS 456 regarding anchoring bars in tension,
compression and shear are discussed. Numerical problems solved in the lesson
and other examples of practice problem and test question explain the
calculations of satisfying the requirements of reinforcing bars under different
situations with respect to bond.

Version 2 CE IIT, Kharagpur


Module
6
Shear, Bond,
Anchorage,
Development Length
and Torsion
Version 2 CE IIT, Kharagpur
Lesson
16
Torsion in Beams -
Limit State of Collapse
Version 2 CE IIT, Kharagpur
Instruction Objectives:
At the end of this lesson, the student should be able to:

• identify the beams and frames subjected to torsion,


• name and explain the two types of torsion,
• state the basis of approach of design for combined bending, shear and
torsion as per IS 456,
• select the critical section for the design,
• determine the equivalent shear and moment from the given factored bending
moment, shear and torsional moment,
• define equivalent nominal shear stress,
• state when do we provide minimum shear reinforcement in beams subjected
to combined bending moment, shear and torsional moment,
• state when do we provide both longitudinal and transverse reinforcement in
beams subjected to combined bending moment, shear and torsional moment,
• state when do we provide tensile, compressive and side face reinforcement,
respectively, in beams subjected to combined bending shear and torsional
moment,
• design the beams subjected to combined bending, shear and torsional
moment as per IS 456.

6.16.1 Introduction
This lesson explains the presence of torsional moment along with bending
moment and shear in reinforced concrete members with specific examples. The
approach of design of such beams has been explained mentioning the critical
section to be designed. Expressing the equivalent shear and bending moment,
this lesson illustrates the step by step design procedure of beam under combined
bending, shear and torsion. The requirements of IS 456 regarding the design are
also explained. Numerical problems have been solved to explain the design of
beams under combined bending, shear and torsion.

Version 2 CE IIT, Kharagpur


6.16.2 Torsion in Reinforced Concrete Members

On several situations beams and slabs are subjected to torsion in addition


to bending moment and shear force. Loads acting normal to the plane of bending
will cause bending moment and shear force. However, loads away from the
plane of bending will induce torsional moment along with bending moment and
shear. Space frames (Fig.6.16.1a), inverted L-beams as in supporting sunshades
and canopies (Fig.6.16.1b), beams curved in plan (Fig.6.16.1c), edge beams of
slabs (Fig.6.16.1d) are some of the examples where torsional moments are also
present.

Skew bending theory, space-truss analogy are some of the theories


developed to understand the behaviour of reinforced concrete under torsion
combined with bending moment and shear. These torsional moments are of two
types:

(i) Primary or equilibrium torsion, and


(ii) Secondary or compatibility torsion.

The primary torsion is required for the basic static equilibrium of most of
the statically determinate structures. Accordingly, this torsional moment must be
considered in the design as it is a major component.

The secondary torsion is required to satisfy the compatibility condition


between members. However, statically indeterminate structures may have any of

Version 2 CE IIT, Kharagpur


the two types of torsions. Minor torsional effects may be ignored in statically
indeterminate structures due to the advantage of having more than one load path
for the distribution of loads to maintain the equilibrium. This may produce minor
cracks without causing failure. However, torsional moments should be taken into
account in the statically indeterminate structures if they are of equilibrium type
and where the torsional stiffness of the members has been considered in the
structural analysis. It is worth mentioning that torsion must be considered in
structures subjected to unsymmetrical loadings about axes.

Clause 41 of IS 456 stipulates the above stating that, "In structures,


where torsion is required to maintain equilibrium, members shall be designed for
torsion in accordance with 41.2, 41.3 and 41.4. However, for such indeterminate
structures where torsion can be eliminated by releasing redundant restraints, no
specific design for torsion is necessary, provided torsional stiffness is neglected
in the calculation of internal forces. Adequate control of any torsional cracking is
provided by the shear reinforcement as per cl. 40".

6.16.3 Analysis for Torsional Moment in a Member


The behaviour of members under the effects of combined bending, shear
and torsion is still a subject of extensive research.

We know that the bending moments are distributed among the sharing
members with the corresponding distribution factors proportional to their bending
stiffness EI/L where E is the elastic constant, I is the moment of inertia and L
is the effective span of the respective members. In a similar manner, the torsional
moments are also distributed among the sharing members with the
corresponding distribution factors proportional to their torsional stiffness GJ/L,
where G is the elastic shear modulus, J is polar moment of inertia and L is
the effective span (or length) of the respective members.

The exact analysis of reinforced concrete members subjected to torsional


moments combined with bending moments and shear forces is beyond the scope
here. However, the codal provisions of designing such members are discussed
below.

6.16.4 Approach of Design for Combined Bending, Shear


and Torsion as per IS 456
As per the stipulations of IS 456, the longitudinal and transverse
reinforcements are determined taking into account the combined effects of
bending moment, shear force and torsional moment. Two impirical relations of
equivalent shear and equivalent bending moment are given. These fictitious
shear force and bending moment, designated as equivalent shear and equivalent
bending moment, are separate functions of actual shear and torsion, and actual

Version 2 CE IIT, Kharagpur


bending moment and torsion, respectively. The total vertical reinforcement is
designed to resist the equivalent shear Ve and the longitudinal reinforcement is
designed to resist the equivalent bending moment Me1 and Me2, as explained
in secs. 6.16.6 and 6.16.7, respectively. These design rules are applicable to
beams of solid rectangular cross-section. However, they may be applied to
flanged beams by substituting bw for b. IS 456 further suggests to refer to
specialist literature for the flanged beams as the design adopting the code
procedure is generally conservative.

6.16.5 Critical Section (cl. 41.2 of IS 456)


As per cl. 41.2 of IS 456, sections located less than a distance d from the
face of the support is to be designed for the same torsion as computed at a
distance d, where d is the effective depth of the beam.

6.16.6 Shear and Torsion


(a) The equivalent shear, a function of the actual shear and torsional
moment is determined from the following impirical relation:

Ve = Vu + 1.6(Tu/b)
(6.22)

where Ve = equivalent shear,

Vu = actual shear,

Tu = actual torsional moment,

b = breadth of beam.

(b) The equivalent nominal shear stress τ ve is determined from:

τ ve = ( Ve / bd )
(6.23)

However, τ ve shall not exceed τ c max given in Table 20 of IS 456 and Table 6.2
of Lesson 13.

(c) Minimum shear reinforcement is to be provided as per cl. 26.5.1.6 of


IS 456, if the equivalent nominal shear stress τ ve obtained from Eq.6.23 does
not exceed τ c given in Table 19 of IS 456 and Table 6.1 of Lesson 13. .

Version 2 CE IIT, Kharagpur


(d) Both longitudinal and transverse reinforcement shall be provided as
per cl. 41.4 and explained below in sec. 6.16.7, if τ ve exceeds τ c given in Table
19 of IS 456 and Table 6.1 of Lesson 13 and is less than τ c max , as mentioned in
(b) above.

6.16.7 Reinforcement in Members subjected to Torsion


(a) Reinforcement for torsion shall consist of longitudinal and transverse
reinforcement as mentioned in sec. 6.16.6(d).

(b) The longitudinal flexural tension reinforcement shall be determined to


resist an equivalent bending moment Me1 as given below:

Me1 = Mu + Mt
(6.24)

where Mu = bending moment at the cross-section, and

Mt = (Tu/1.7) {1 + (D/b)}
(6.25)

where Tu = torsional moment,

D = overall depth of the beam, and

b = breadth of the beam.

(c) The longitudinal flexural compression reinforcement shall be provided


if the numerical value of Mt as defined above in Eq.6.25 exceeds the numerical
value of Mu. Such compression reinforcement should be able to resist an
equivalent bending moment Me2 as given below:

Me2 = Mt - Mu
(6.26)

The Me2 will be considered as acting in the opposite sense to the moment Mu.

Version 2 CE IIT, Kharagpur


(d) The transverse reinforcement consisting of two legged closed loops
(Fig.6.16.2) enclosing the corner longitudinal bars shall be provided having an
area of cross-section Asv given below:

Tu sv Vu sv
Asv = +
b1 d1( 0.87 f y ) 2.5 d1( 0.87 f y )
(6.27)

However, the total transverse reinforcement shall not be less than the following:

Asv ≥ (τ ve - τ c ) b s v /(0.87 f y )
(6.28)

where Tu = torsional moment,

Vu = shear force,

sv = spacing of the stirrup reinforcement,

b1 = centre to centre distance between corner bars in the direction of the


width,

d1 = centre to centre distance between corner bars,

Version 2 CE IIT, Kharagpur


b = breadth of the member,

fy = characteristic strength of the stirrup reinforcement,

τ ve = equivalent shear stress as specified in Eqs.6.22 and 6.23, and

τ c = shear strength of concrete as per Table 19 of IS 456 and Table 6.1


of Lesson 13.

6.16.8 Requirements of Reinforcement


Beams subjected to bending moment, shear and torsional moment should
satisfy the following requirements:

(a) Tension reinforcement (cl. 26.5.1.1 of IS 456)

The minimum area of tension reinforcement should be governed by

As /(bd) = 0.85/fy
(6.29)

where As = minimum area of tension reinforcement,

b = breadth of rectangular beam or breadth of web of T-beam,

d = effective depth of beam,

fy = characteristic strength of reinforcement in N/mm2.

The maximum area of tension reinforcement shall not exceed 0.04 bD,
where D is the overall depth of the beam.

(b) Compression reinforcement (cl. 26.5.1.2 of IS 456)

The maximum area of compression reinforcement shall not exceed 0.04


bD. They shall be enclosed by stirrups for effective lateral restraint.

(c) Side face reinforcement (cls. 26.5.1.3 and 26.5.1.7b)

Beams exceeding the depth of 750 mm and subjected to bending moment


and shear shall have side face reinforcement. However, if the beams are having
torsional moment also, the side face reinforcement shall be provided for the
overall depth exceeding 450 mm. The total area of side face reinforcement shall
be at least 0.1 per cent of the web area and shall be distributed equally on two
faces at a spacing not exceeding 300 mm or web thickness, whichever is less.

Version 2 CE IIT, Kharagpur


(d) Transverse reinforcement (cl. 26.5.1.4 of IS 456)

The transverse reinforcement shall be placed around the outer-most


tension and compression bars. They should pass around longitudinal bars
located close to the outer face of the flange in T- and I-beams.

(e) Maximum spacing of shear reinforcement (cl. 26.5.1.5 of IS 456)

The centre to centre spacing of shear reinforcement shall not be more


than 0.75 d for vertical stirrups and d for inclined stirrups at 45o, but not
exceeding 300 mm, where d is the effective depth of the section.

(f) Minimum shear reinforcement (cl. 26.5.1.6 of IS 456)

This has been discussed in sec. 6.13.7 of Lesson 13 and the governing
equation is Eq.6.3 of Lesson 13.

(g) Distribution of torsion reinforcement (cl. 26.5.1.7 of IS 456)

The transverse reinforcement shall consist of rectangular close stirrups


placed perpendicular to the axis of the member. The spacing of stirrups shall not
be more than the least of x1, (x1 + y1)/4 and 300 mm, where x1 and y1 are the
short and long dimensions of the stirrups (Fig.6.16.2).

Longitudinal reinforcements should be placed as close as possible to the


corners of the cross-section.

(h) Reinforcement in flanges of T- and L-beams (cl. 26.5.1.8 of IS 456)

For flanges in tension, a part of the main tensile reinforcement shall be


distributed over the effective flange width or a width equal to one-tenth of the
span, whichever is smaller. For effective flange width greater than one-tenth of
the span, nominal longitudinal reinforcement shall be provided to the outer
portion of the flange.

Version 2 CE IIT, Kharagpur


6.16.9 Numerical Problems
Problem 1

Determine the reinforcement required of a ring beam (Fig.6.16.3) of b =


400 mm, d = 650 mm, D = 700 mm and subjected to factored Mu = 200 kNm,
factored Tu = 50 kNm and factored Vu = 100 kN. Use M 20 and Fe 415 for the
design.

Solution 1

The solution of the problem is illustrated in seven steps below.

Step 1: Check for the depth of the beam

From Eq.6.22, we have the equivalent shear

Ve = Vu + 1.6(Tu/b) = 100 + 1.6(50/0.4) = 300 kN

From Eq.6.2.3, the equivalent shear stress

τ ve = ( Ve / bd ) = 300/(0.4)(0.65) = 1.154 N/mm2

From Table 6.2 of Lesson 13 (Table 20 of IS 456), τ c max = 2.8 N/mm2. Hence,
the section does not need any revision.

Step 2: Check if shear reinforcement shall be required.

Assuming percentage of tensile steel as 0.5, Table 6.1 of Lesson 13


(Table 19 of IS 456) gives τ c = 0.48 N/mm2 < τ ve < τ c max . So, both
longitudinal and transverse reinforcement shall be required.

Version 2 CE IIT, Kharagpur


Step 3: Longitudinal tension reinforcement

From Eqs.6.24 and 6.25, we have,

Me1 = Mu + Mt = Mu + (Tu/1.7) {1 + (D/b)}

= 200 + (50/1.7) {1 + (700/400)} = 200 + 80.88 = 280.88 kNm

Me1/bd2 = (280.88)(106)/(400)(650)(650) = 1.66 N/mm2

From Table 2 of SP-16, corresponding to Mu/bd2 = 1.66 N/mm2, we have by


linear interpolation pt = 0.5156. So,

Ast = 0.5156(400)(650)/100 = 1340.56 mm2.

Provide 2-25T and 2-16T = 981 + 402 = 1383 mm2. This gives percentage of
tensile reinforcement = 0.532, for which τ c from Table 6.1 of Lesson 13 is 0.488
N/mm2.

From Eq.6.29, minimum percentage of tension reinforcement =


(0.85/fy)(100) = 0.205 and from sec.6.16.8, the maximum percentage of tension
reinforcement is 4.0. So, 2-25T and 2-16T bars satisfy the requirements (Fig.
6.16.4).

Version 2 CE IIT, Kharagpur


Step 4: Longitudinal compression reinforcement

Here, in this problem, the numerical value of Mt (= 80.88 kNm) is less


than that of Mu (200 kNm). So, as per sec. 6.16.7c, longitudinal compression
reinforcement shall not be required.

Step 5: Longitudinal side face reinforcement

Side face reinforcement shall be provided as the depth of the beam


exceeds 450 mm. Providing 2-10 mm diameter bars (area = 157 mm2) at the
mid-depth of the beam and one on each face (Fig.6.16.4), the total area required
as per sec.6.16.8c, 0.1(400)(300)/100 = 120 mm2 < 157 mm2. Hence o.k.

Step 6: Transverse reinforcement

Providing two legged, 10 mm diameter stirrups (area = 157 mm2), we have


(Fig.6.16.5)

d1 = 700 - 50 - 50 = 600 mm

b1 = 400 - 2(25 + 10 + 12.5) = 305 mm

From Eq.6.27, we have

0.87 f y Asv /sv = (Tu / b1 d1 ) + ( Vu / 2.5 d1 )

Version 2 CE IIT, Kharagpur


Using the numerical values of Tu, b1, d1 and Vu, we have

0.87 f y Asv /sv = 339.89 N/mm …. (1)

Again from Eq.6.28, we have

0.87 f y Asv /sv ≥ (1.154 - 0.48) 400 ≥ 269.6 N/mm …. (2)

So, Eq.(1) is governing and we get for 2 legged 10 mm stirrups (Asv = 157 mm2),

sv = 0.87(415)(157)/339.89 = 166.77 mm

Step 7: Check for sv

Figure 6.16.4 shows the two legged 10 mm diameter stirrups for which x1
= 340 mm and y1 = 628.5 mm. The maximum spacing sv should be the least of
x1, (x1 + y1)/4 and 300 mm (Figs. 6.16.4 and 5).

Here, x1 = 340 mm, (x1 + y1)/4 = 242.12 mm. So, provide 2 legged 10
mm T stirrups @ 160 mm c/c.

6.16.10 Practice Questions and Problems with Answers


Q.1: Explain the situations when torsional moments remain present in beams
and frames.

A.1: The first paragraph of sec. 6.16.2.

Q.2: Explain and differentiate between primary and secondary types of torsion.

A.2: Third, fourth and fifth paragraphs of sec. 6.16.2.

Q.3: Write expressions of equivalent shear and equivalent bending moment.

A.3: Eqs.6.22, 24 and 25.

Q.4: When do you provide minimum shear reinforcement in beam subjected to


bending moment, shear and torsional moment?

A.4: Sec. 6.16.6(c)

Version 2 CE IIT, Kharagpur


Q.5: Explain the situations when do you provide longitudinal tension,
compression and side face reinforcement in beam subjected to bending
moment, shear and torsional moment.

A.5: Sec. 6.16.7a, b and c parts.

Q.6: Illustrate the steps of designing transverse reinforcement in beams


subjected to bending moment, shear and torsional moment.

A.6: Sec. 6.16.7d.

Q.7: A reinforced concrete rectangular beam (Fig.6.16.6) of b = 300 mm, d =


600 mm and D = 650 mm is subjected to factored shear force Vu = 70 kN
in one section. Assuming the percentage of tensile reinforcement as 0.5 in
that section, determine the factored torsional moment that the section can
resist if (a) no additional reinforcement for torsion is provided, (b)
maximum steel for torsion is provided in that section, and (c) determine
the reinforcement needed for the case (b). Assume M 30 concrete, Fe 500
for longitudinal and Fe 415 for transverse reinforcing steel bars.

A.7:
(a) When no additional reinforcement for torsion is provided in that
section.

For M 30 concrete with 0.5 per cent tensile reinforcement, Table 6.1 of
Lesson 13 (Table 19 of IS 456) gives τ c = 0.5 N/mm2 and Table 6.2 of Lesson
13 (Table 20 of IS 456 gives τ c max = 3.5 N/mm2.

Version 2 CE IIT, Kharagpur


Since no additional reinforcement for torsion will be provided, we will take
τ ve = τ c = 0.5 N/mm2. From Eq.6.23, we have

Ve = (τ ve ) bd = 0.5(300)(600) N = 90 kN

From Eq.6.22, we have

Tu = (Ve - Vu) (b/1.6) = (90 - 70) (0.3/1.6) = 3.75 kNm

So, that section of the beam can resist factored torsional moment of 3.75 kNm if
no additional reinforcement is provided.

(b) When maximum steel is provided for torsion in that section.

In this case, τ ve = τ cmax = 3.5 N/mm2. Using this value in Eq.6.23, we get

Ve = (τ ve ) bd = (3.5)(300)(600) N = 630 kN

From Eq.6.22, we get

Tu = (Ve - Vu) (b/1.6) = (630 - 70) (0.3/1.6) = 105 kNm

This section, therefore, can resist a factored torsional moment of 105 kNm when
maximum torsional reinforcement is provided.

(c) Determination of maximum torsional reinforcement for case (b)

Step 1: Determination of the Mu when 0.5 per cent tensile reinforcement is


assumed.

From Table 4 of SP-16, for Fe 500 and M 30 with pt = 0.5 per cent, we
have Mu = (1.993)(300)(600)2 Nmm = 215.244 kNm (using linear interpolation).

Version 2 CE IIT, Kharagpur


Step 2: Tension and compression reinforcement

From Eqs.6.24 and 6.25, we get

Me1 = Mu + Mt = Mu + (Tu/1.7) {1 + (D/b)}

= 215.244 + (105/1.7){1 + (650/300)}= 215.244 + 195.588 =


410.832 kNm

Here, the numerical value of Mt (= 195.588 kNm) is less than that of Mu (=


215.244 kNm). So, no compression reinforcement is needed.

Table 4 of SP-16 is used to determine tension reinforcement with Mu/bd2


= 410.832/(0.3)(0.6)(0.6) = 3.804 N/mm. From Table 4, we get pt = 1.064 by
linear interpolation, which gives Ast = 1.064(300)(600)/100 = 1915.2 mm2.
Provide 4-25T (1963 mm2) as shown in Fig.6.16.7. Now, pt =
1963(100)/(300)(600) = 1.09, for which τ c = 0.678 N/mm (Table 6.1 of Lesson
2

13).

Step 3: Side face reinforcement

Since the depth of the beam exceeds 450 mm, we provide side face
reinforcement with two 10 mm bars (area = 157 mm2) near the mid-depth of the
beam, one on each side to get the spacing of the bar 280 mm (Fig.6.16.7). Area
required to satisfy = 0.1(300)(280)/(100) = 84 mm2 < 157 mm2, hence, o.k.

Version 2 CE IIT, Kharagpur


Step 4: Transverse reinforcement

Assuming 2 legged 12 mm dia stirrup (area = 226 mm2) of Fe 415, we


have from Fig.6.16.8, d1 = 557 mm and b1 = 201 mm. From Eq.6.27,

0.87 f y Asv Tu Vu
= +
sv b1 d1 2.5 d1

= {105(106)/(201)(557)} + {70(103)/(2.5)(557)}

= 988.13 N/mm

From Eq.6.28 the least value of the above is ( τ ve - τ c ) b = (3.5 - 0.678)(300) =


846.6 Nm2. So, from 0.87 fy Asv/sv = 988.13 N/mm, we get

sv = 0.87(415)(226)/988.13 = 82.57 mm

Step 5: Check for sv

Figure 6.16.8 shows the stirrups of 12 mm diameter two legged for which
x1 = 238 mm and y1 = 587.5 mm. The maximum spacing should be the least of
x1, (x1 + y1)/4 and 300 mm.

Here, x1 = 238 mm, (x1 + y1)/4 = 206.375 mm and 300 mm. So, the
spacing of 80 mm c/c is o.k. Provide 12 mm, 2 legged stirrups @ 80 mm c/c, as
shown in Fig.6.16.8.

Version 2 CE IIT, Kharagpur


6.16.11 References
References:

1. Reinforced Concrete Limit State Design, 6th Edition, by Ashok K. Jain,


Nem Chand & Bros, Roorkee, 2002.
2. Limit State Design of Reinforced Concrete, 2nd Edition, by P.C.Varghese,
Prentice-Hall of India Pvt. Ltd., New Delhi, 2002.
3. Advanced Reinforced Concrete Design, by P.C.Varghese, Prentice-Hall of
India Pvt. Ltd., New Delhi, 2001.
4. Reinforced Concrete Design, 2nd Edition, by S.Unnikrishna Pillai and
Devdas Menon, Tata McGraw-Hill Publishing Company Limited, New
Delhi, 2003.
5. Limit State Design of Reinforced Concrete Structures, by P.Dayaratnam,
Oxford & I.B.H. Publishing Company Pvt. Ltd., New Delhi, 2004.
6. Reinforced Concrete Design, 1st Revised Edition, by S.N.Sinha, Tata
McGraw-Hill Publishing Company. New Delhi, 1990.
7. Reinforced Concrete, 6th Edition, by S.K.Mallick and A.P.Gupta, Oxford &
IBH Publishing Co. Pvt. Ltd. New Delhi, 1996.
8. Behaviour, Analysis & Design of Reinforced Concrete Structural Elements,
by I.C.Syal and R.K.Ummat, A.H.Wheeler & Co. Ltd., Allahabad, 1989.
9. Reinforced Concrete Structures, 3rd Edition, by I.C.Syal and A.K.Goel,
A.H.Wheeler & Co. Ltd., Allahabad, 1992.
10. Textbook of R.C.C, by G.S.Birdie and J.S.Birdie, Wiley Eastern Limited,
New Delhi, 1993.
11. Design of Concrete Structures, 13th Edition, by Arthur H. Nilson, David
Darwin and Charles W. Dolan, Tata McGraw-Hill Publishing Company
Limited, New Delhi, 2004.
12. Concrete Technology, by A.M.Neville and J.J.Brooks, ELBS with
Longman, 1994.
13. Properties of Concrete, 4th Edition, 1st Indian reprint, by A.M.Neville,
Longman, 2000.
14. Reinforced Concrete Designer’s Handbook, 10th Edition, by C.E.Reynolds
and J.C.Steedman, E & FN SPON, London, 1997.
15. Indian Standard Plain and Reinforced Concrete – Code of Practice (4th
Revision), IS 456: 2000, BIS, New Delhi.
16. Design Aids for Reinforced Concrete to IS: 456 – 1978, BIS, New Delhi.

6.16.12 Test 16 with Solutions


Maximum Marks = 50, Maximum Time = 30 minutes

Answer all questions. Each question carries five marks.

Version 2 CE IIT, Kharagpur


TQ.1: Explain and differentiate between primary and secondary types of
torsion.
(10
marks)

A.TQ.1: Third, fourth and fifth paragraphs of sec. 6.16.2.

TQ.2: Explain the situations when do you provide longitudinal tension,


compression and side face reinforcement in beam subjected to bending
moment, shear and torsional moment.
(10 marks)

A.TQ.2: Sec. 6.16.7a, b and c parts.

TQ.3: Illustrate the steps of designing transverse reinforcement in beams


subjected to bending moment, shear and torsional moment.
(10 marks)

A.TQ.3: Sec. 6.16.7d

TQ.4: The beam of Fig.6.16.9 has factored bending moment Mu = 70 kNm,


factored shear force Vu = 100 kN and factored torsional moment Tu = 60
kNm at one section. Design the reinforcement of that section assuming
that the torsion is fully taken by the web. Assume M 30 concrete, Fe 500
for longitudinal and Fe 415 for transverse reinforcing steel bars.
(20 marks)

A.TQ.4:

Version 2 CE IIT, Kharagpur


Step 1: Checking of depth of the beam

From Eq.6.22, the equivalent shear

Ve = Vu + 1.6(Tu/b) = 100 + 1.6(60/0.3) = 420 kN

From Eq.6.23, we get

τ ve = ( Ve / bd ) = 420/(0.3)(0.5) kN/m2 = 2.8 N/mm2

Table 6.2 of Lesson 13 gives τ c max = 3.5 N/mm2 > τ ve , So the depth is
satisfying.

Step 2: Check if shear reinforcement is required

Assuming percentage of tensile steel as 0.5, Table 6.1 of Lesson 13 gives


τ c = 0.5 N/mm2. Hence, both longitudinal and transverse reinforcements shall be
provided.

Step 3: Longitudinal tension reinforcement

From Eqs.6.24 and 6.25, we have,

Me1 = Mu + Mt = Mu + (Tu/1.7) {1 + (D/b)}

Version 2 CE IIT, Kharagpur


= 70 + (60/1.7) {1 + (550/300)} = 70 + 100 = 170 kNm.

Me1/bd2 = 170(106)/(300)(500)(500) = 2.267 N/mm2

Table 4 of SP-16 or M 30, Fe 500 and Mu/bd2 = 2.267 N/mm2, we have by


linear interpolation pt = 0.577.

Ast = 0.577(300)(500)/100 = 865.5 mm2.

Minimum Ast (Eq.6.29) = 0.85 bd/fy = 0.85(300)(500)/(500) = 255 mm2.

Maximum Ast = 0.04 bD = 0.04(300)(550) = 6600 mm2.

So, Ast = 865.5 mm2 is acceptable.

Provide 3-20T (area = 942 mm2) giving pt = (942)(100)/(300)(500) = 0.628%


(Fig. 6.16.10). Table 6.1 of Lesson 13 gives τ c = 0.546 N/mm2 (for pt =
0.628%).

Step 4: Longitudinal compression reinforcement

Here, the numerical value of Mt (= 100 kNm) is greater than that of Mu (=


70 kNm), as computed in Step 3. So, compression reinforcement shall be
provided for Me2 = Mt - Mu = 100 - 70 = 30 kNm. Me2/bd2 =
30(106)/(300)(500)(500) = 0.4 N/mm. Table 4 of SP-16 gives pc = 0.093 to have
compression steel reinforcement Asc = 0.093(300)(500)/100 = 139.5 mm2.
Maximum area compression steel = 0.04 bD = 0.04(300)(550) = 6600 mm2.
Hence, Asc = 139.5 mm2 is satisfying. Provide 2-12T (area = 226 mm2) as
compression steel (Fig. 6.16.10).

Step 5: Side face reinforcement

Since the depth of the beam exceeds 450 mm, provide 2-10 mm T
(area = 157 mm2) near the mid-depth, one on each side of the beam with
maximum spacing = 230 mm (Fig.6.16.9). Area required = 0.1(300)(230)/100 =
69 mm2 < 157 mm2. Hence, two 10 mmT bars as shown in Fig.6.16.10 is o.k.

Step 6: Transverse reinforcement

Using 10 mm T, 2 legged stirrups, we have d1 = 459 mm and b1 = 210


mm (Fig.6.16.10). From Eq.6.27, we have,

0.87 f y Asv Tu Vu
= +
sv b1 d1 2.5 d1

Version 2 CE IIT, Kharagpur


= 60(106)/(210)(459)} + 100(103)/(2.5)(459)}

= 709.61 N/mm2

However, Eq.6.28 gives the least value of 0.87 fy As/sv = ( τ ve - τ c ) b = (2.8 -


0.546)(300) = 676.2 N/mm. So, 0.87 fy Asv/sv = 709.61 gives

sv = 0.87 fy Asv/709.61 = 0.87(415)(157)/709.61 = 79.88 mm

Step 7: Checking of sv

Figure 6.16.11 shows x1 = 300 - 2(25 + 5) = 240 mm and y1 = 550 - 30 -


35 = 485 mm. Accordingly, the maximum spacing is the least of x1, (x1 + y1)/4 or
300 mm. Here, the least value is 181.25 mm. Hence sv = 76 mm is acceptable
(Fig.6.16.10).

6.16.13 Summary of this Lesson


This lesson explains the different situations when beams and frames are
subjected to combined bending, shear and torsional moment. Briefly discussing
about the analysis of such beams, design of beams as per IS 456 has been
illustrated with numerical examples. The requirements of IS 456 have been
specified. Solutions of practice and test problems will help the students in
understanding the theory and designing the beams subjected to combined
bending moment, shear and torsional moment.

Version 2 CE IIT, Kharagpur


Module
7
Limit State of
Serviceability
Version 2 CE IIT, Kharagpur
Lesson
17
Limit State of
Serviceability
Version 2 CE IIT, Kharagpur
Instruction Objectives:
At the end of this lesson, the student should be able to:

• explain the need to check for the limit state of serviceability after designing
the structures by limit state of collapse,

• differentiate between short- and long-term deflections,

• state the influencing factors to both short- and long-term deflections,

• select the preliminary dimensions of structures to satisfy the requirements


as per IS 456,

• calculate the short- and long-term deflections of designed beams.

7.17.1 Introduction
Structures designed by limit state of collapse are of comparatively smaller
sections than those designed employing working stress method. They, therefore,
must be checked for deflection and width of cracks. Excessive deflection of a
structure or part thereof adversely affects the appearance and efficiency of the
structure, finishes or partitions. Excessive cracking of concrete also seriously
affects the appearance and durability of the structure. Accordingly, cl. 35.1.1 of
IS 456 stipulates that the designer should consider all relevant limit states to
ensure an adequate degree of safety and serviceability. Clause 35.3 of IS 456
refers to the limit state of serviceability comprising deflection in cl. 35.3.1 and
cracking in cl. 35.3.2. Concrete is said to be durable when it performs
satisfactorily in the working environment during its anticipated exposure
conditions during service. Clause 8 of IS 456 refers to the durability aspects of
concrete. Stability of the structure against overturning and sliding (cl. 20 of IS
456), and fire resistance (cl. 21 of IS 456) are some of the other importance
issues to be kept in mind while designing reinforced concrete structures.

This lesson discusses about the different aspects of deflection of beams


and the requirements as per IS 456. In addition, lateral stability of beams is also
taken up while selecting the preliminary dimensions of beams. Other
requirements, however, are beyond the scope of this lesson.

7.17.2 Short- and Long-term Deflections


As evident from the names, short-term deflection refers to the immediate
deflection after casting and application of partial or full service loads, while the
long-term deflection occurs over a long period of time largely due to shrinkage

Version 2 CE IIT, Kharagpur


and creep of the materials. The following factors influence the short-term
deflection of structures:

(a) magnitude and distribution of live loads,


(b) span and type of end supports,
(c) cross-sectional area of the members,
(d) amount of steel reinforcement and the stress developed in the
reinforcement,
(e) characteristic strengths of concrete and steel, and
(f) amount and extent of cracking.

The long-term deflection is almost two to three times of the short-term


deflection. The following are the major factors influencing the long-term deflection
of the structures.

(a) humidity and temperature ranges during curing,


(b) age of concrete at the time of loading, and
(c) type and size of aggregates, water-cement ratio, amount of
compression reinforcement, size of members etc., which influence the
creep and shrinkage of concrete.

7.17.3 Control of Deflection


Clause 23.2 of IS 456 stipulates the limiting deflections under two heads
as given below:
(a) The maximum final deflection should not normally exceed span/250
due to all loads including the effects of temperatures, creep and shrinkage and
measured from the as-cast level of the supports of floors, roof and all other
horizontal members.

(b) The maximum deflection should not normally exceed the lesser of
span/350 or 20 mm including the effects of temperature, creep and shrinkage
occurring after erection of partitions and the application of finishes.

It is essential that both the requirements are to be fulfilled for every


structure.

7.17.4 Selection of Preliminary Dimensions


The two requirements of the deflection are checked after designing the
members. However, the structural design has to be revised if it fails to satisfy any
one of the two or both the requirements. In order to avoid this, IS 456
recommends the guidelines to assume the initial dimensions of the members
which will generally satisfy the deflection limits. Clause 23.2.1 stipulates different
span to effective depth ratios and cl. 23.3 recommends limiting slenderness of

Version 2 CE IIT, Kharagpur


beams, a relation of b and d of the members, to ensure lateral stability. They
are given below:

(A) For the deflection requirements

Different basic values of span to effective depth ratios for three different
support conditions are prescribed for spans up to 10 m, which should be modified
under any or all of the four different situations: (i) for spans above 10 m, (ii)
depending on the amount and the stress of tension steel reinforcement, (iii)
depending on the amount of compression reinforcement, and (iv) for flanged
beams. These are furnished in Table 7.1.

(B) For lateral stability

The lateral stability of beams depends upon the slenderness ratio and the
support conditions. Accordingly cl. 23.3 of IS code stipulates the following:

(i) For simply supported and continuous beams, the clear distance
between the lateral restraints shall not exceed the lesser of 60b or 250b2/d,
where d is the effective depth and b is the breadth of the compression face
midway between the lateral restraints.

(ii) For cantilever beams, the clear distance from the free end of the
cantilever to the lateral restraint shall not exceed the lesser of 25b or 100b2/d.

Table 7.1 Span/depth ratios and modification factors

Sl. Items Cantilever Simply Continuous


No. supported
1 Basic values of span to 7 20 26
effective depth ratio for
spans up to 10 m
2 Modification factors for Not applicable Multiply values of row 1 by
spans > 10 m as deflection 10/span in metres.
calculations
are to be
done.
3 Modification factors Multiply values of row 1 or 2 with the modification
depending on area and factor from Fig.4 of IS 456.
stress of steel
4 Modification factors Further multiply the earlier respective value with
depending as area of that obtained from Fig.5 of IS 456.
compression steel
5 Modification factors for (i) Modify values of row 1 or 2 as per Fig.6 of IS
flanged beams 456.
(ii) Further modify as per row 3 and/or 4 where

Version 2 CE IIT, Kharagpur


reinforcement percentage to be used on area
of section equal to bf d.

7.17.5 Calculation of Short-Term Deflection


Clause C-2 of Annex C of IS 456 prescribes the steps of calculating the
short-term deflection. The code recommends the usual methods for elastic
deflections using the short-term modulus of elasticity of concrete Ec and
effective moment of inertia Ieff given by the following equation:

Ir
I eff = ; but I r ≤ I eff ≤ I gr
1.2 - (M r / M )( z / d )( 1 − x / d )( bw / b )
(7.1)

where Ir = moment of inertia of the cracked section,

Mr = cracking moment equal to (fcr Igr)/yt , where fcr is the modulus of


rupture of concrete, Igr is the moment of inertia of the gross section
about the centroidal axis neglecting the reinforcement, and yt is the
distance from centroidal axis of gross section, neglecting the
reinforcement, to extreme fibre in tension,

M = maximum moment under service loads,

z = lever arm,

x = depth of neutral axis,

d = effective depth,

bw = breadth of web, and

b = breadth of compression face.

For continuous beams, however, the values of Ir, Igr and Mr are to be
modified by the following equation:

⎡X + X2 ⎤
X e = k1 ⎢ 1 ⎥ + (1 - k1 ) X o
⎣ 2 ⎦
(7.2)

where Xe = modified value of X,

X1, X2 = values of X at the supports,

Version 2 CE IIT, Kharagpur


Xo = value of X at mid span,

k1 = coefficient given in Table 25 of IS 456 and in Table 7.2 here, and

X = value of Ir, Igr or Mr as appropriate.

Table 7.2 Values of coefficient k1

k1 0.5 0.6 0.7 0.8 0.9 1.0 1.1 1.2 1.3 1.4
or
less
k2 0 0.03 0.08 0.16 0.30 0.50 0.73 0.91 0.97 1.0

Note: k2 is given by (M1 + M2)/(MF1 + MF2), where M1 and M2 = support


moments, and MF1 and MF2 = fixed end moments.

7.17.6 Deflection due to Shrinkage


Clause C-3 of Annex C of IS 456 prescribes the method of calculating the
deflection due to shrinkage α cs from the following equation:
α cs = k 3 ψ cs l 2
(7.3)

where k3 is a constant which is 0.5 for cantilevers, 0.125 for simply supported
members, 0.086 for members continuous at one end, and 0.063 for fully
continuous members; ψ cs is shrinkage curvature equal to k4 ε cs /D where ε cs is
the ultimate shrinkage strain of concrete. For ε cs , cl. 6.2.4.1 of IS 456
recommends an approximate value of 0.0003 in the absence of test data.

k 4 = 0.72( pt - p c ) / pt ≤ 1.0, for 0.25 ≤ pt - pc < 1.0


= 0.65( pt - p c ) / pt ≤ 1.0, for pt - pc ≥ 1.0
(7.4)

where pt = 100Ast/bd and pc = 100Asc/bd, D is the total depth of the section,


and l is the length of span.

Version 2 CE IIT, Kharagpur


7.17.7 Deflection Due to Creep
Clause C-4 of Annex C of IS 456 stipulates the following method of
calculating deflection due to creep. The creep deflection due to permanent loads
α cc ( perm ) is obtained from the following equation:

α cc ( perm ) = α 1cc ( perm ) - α 1(perm)


(7.5)

where α 1cc ( perm ) = initial plus creep deflection due to permanent loads obtained
using an elastic analysis with an effective modulus of
elasticity,

E ce = E c /(1 + θ ), θ being the creep coefficient, and

α 1( perm ) = short-term deflection due to permanent loads using Ec.

7.17.8 Numerical Problems

Problem 1:

Figures 7.17.1 and 2 present the cross-section and the tensile steel of a
simply supported T-beam of 8 m span using M 20 and Fe 415 subjected to dead

Version 2 CE IIT, Kharagpur


load of 9.3 kN/m and imposed loads of 10.7 kN/m at service. Calculate the short-
and long-term deflections and check the requirements of IS 456.

Solution 1:

Step 1: Properties of plain concrete section

Taking moment of the area about the bottom of the beam

(300)(600)(300) + (2234 - 300)(100)(550)


yt = = 429.48 mm
(300)(600) + (2234 - 300)(100)

300(429.48) 3 2234(170.52) 3 1934(70.52) 3


I gr = + - = (11.384) (10) 9 mm 4
3 3 3

This can also be computed from SP-16 as explained below:

Here, bf /bw = 7.45, Df /D = 0.17. Using these values in chart 88 of SP-16,


we get k1 = 2.10.

Igr = k1bw D3/12 = (2.10)(300)(600)3/12 = (11.384)(10)9 mm4

Step 2: Properties of the cracked section (Fig.7.17.2)

f cr = 0.7 f ck (cl. 6.2.2 of IS 456) = 3.13 N/mm2

Version 2 CE IIT, Kharagpur


Mr = fcr Igr /yt = 3.13(11.384)(10)9/429.48 = 82.96 kNm

Es = 200000 N/mm2

Ec = 5000 f ck (cl. 6.2.3.1 of IS 456) = 22360.68 N/mm2

m = Es /Ec = 8.94

Taking moment of the compressive concrete and tensile steel about the neutral
axis, we have (Fig.7.17.2)

bf x2/2 = m Ast (d – x) gives (2234)(x2/2) = (8.94)(1383)(550 – x)

or x2 + 11.07 x – 6087.92 = 0 . Solving the equation, we get x = 72.68 mm.

z = lever arm = d – x/3 = 525.77 mm

2234(72.68) 3
Ir = + 8.94(1383)(550 - 72.68) 2 = 3.106(10) 9 mm 4
3

M = wl2/8 = (9.3 + 10.7)(8)(8)/8 = 160 kNm

Ir
I eff = …. (Eq. 7.1)
M z x b
1.2 - r (1 - ) ( w )
M d d b

Ir
= = 0.875 I r . But I r ≤ I eff ≤ I gr
82.96 525.77 72.68 300
1.2 - ( )( ) (1 - )( )
160 550 550 2234

So, Ieff = Ir = 3.106(10)9 mm4.

Step 3: Short-term deflection (sec. 7.17.5)

E c = 5000 f ck (cl. 6.2.3.1 of IS 456) = 22360.68 N/mm2

Short-term deflection = (5/384) wl4/EcIeff

= (5)(20)(8)4(1012)/(384)(22360.68)(3.106)(109) = 15.358 mm
(1)

Version 2 CE IIT, Kharagpur


Step 4: Deflection due to shrinkage (sec. 7.17.6)

k 4 = 0.72( pt - pc ) / pt = 0.72(0.84) 0.84 = 0.6599

ψ cs = k 4 ε cs / D = (0.6599)(0 .0003)/600 = 3.2995(10) -7

k3 = 0.125 (from sec. 7.17.6)

α cs = k 3 ψ cs l 2 (Eq. 7.3) = (0.125)(3.2995)(10)-7(64)(106) = 2.64 mm


(2)

Step 5: Deflection due to creep (sec. 7.17.7)

Equation 7.5 reveals that the deflection due to creep α cc ( perm ) can be
obtained after calculating α 1cc ( perm ) and α 1( perm ) . We calculate α 1cc ( perm ) in the
next step.

Step 5a: Calculation of α 1cc ( perm )

Assuming the age of concrete at loading as 28 days, cl. 6.2.5.1 of IS 456


gives θ = 1.6. So, Ecc = Ec /(1 + θ ) = 22360.68/(1 + 1.6) = 8600.2615
N/mm2 and m = Es /Ecc = 200000/8600.2615 = 23.255

Step 5b: Properties of cracked section

Version 2 CE IIT, Kharagpur


Taking moment of compressive concrete and tensile steel about the
neutral axis (assuming at a distance of x from the bottom of the flange as
shown in Fig.7.17.3):

2234(100)(50 + x ) = (23.255)(1383)(450 - x ) or x = 12.92 mm

which gives x = 112.92 mm. Accordingly, z = lever arm = d – x/3 =


512.36 mm.

Ir = 2234(100)3/12 + 2234(100)(62.92)2 + 23.255(1383)(550 – 112.92)2

+ 300(12.92)3/3 = 7.214(109) mm4

Mr = 82.96 kNm (see Step 2)

M = wperm l2/8 = 9.3(8)(8)/8 = 74.4 kNm.

Ir
I eff = = 0.918 I r
82.96 512.36 112.92 300
(1.2) - ( )( ) (1 - )( )
74.4 550 550 2234

However, to satisfy Ir ≤ Ieff ≤ Igr, Ieff should be equal to Igr. So, Ieff = Igr =
11.384(109). For the value of Igr please see Step 1.

Step 5c: Calculation of α 1cc ( perm )

α 1cc ( perm ) = 5wl4/384(Ecc)(Ieff) =


5(9.3)(8)4(10)12/384(8600.2615)(11.384)(109)

= 5.066 mm
(3)

Step 5d: Calculation of α 1( perm )

α 1( perm ) = 5wl4/384(Ec)(Ieff) =
5(9.3)(8)4(10)12/384(22360.68)(11.384)(109)

= 1.948 mm
(4)

Step 5e: Calculation of deflection due to creep

α cc ( perm ) = α 1cc ( perm ) - α 1( perm )

Version 2 CE IIT, Kharagpur


= 5.066 – 1.948 = 3.118 mm
(5)

It is important to note that the deflection due to creep α cc ( perm ) can be


obtained even without computing α 1cc ( perm ) . The relationship of α cc ( perm ) and
α 1( perm ) is given below.
α cc ( perm ) = α 1cc ( perm ) - α 1( perm )

= {5wl4/384(Ec)(Ieff)} {(Ec /Ecc) – 1} = α 1( perm ) ( θ )

Hence, the deflection due to creep, for this problem is:

α cc ( perm ) = α 1( perm ) ( θ ) = 1.948(1.6) = 3.116 mm

Step 6: Checking of the requirements of IS 456

The two requirements regarding the control of deflection are given in sec.
7.17.3. They are checked in the following:

Step 6a: Checking of the first requirement

The maximum allowable deflection = 8000/250 = 32 mm

The actual final deflection due to all loads

= 15.358 (see Eq.1 of Step 3) + 2.64 (see Eq.2 of Step 4)

+ 3.118 (see Eq.5 of Step 5e) = 21.116 mm < 32 mm. Hence,


o.k.

Step 6b: Checking of the second requirement

The maximum allowable deflection is the lesser of span/350 or 20 mm.


Here, span/350 = 22.86 mm. So, the maximum allowable deflection = 20 mm.
The actual final deflection = 1.948 (see Eq.4 of Step 5d) + 2.64 (see Eq.2 of Step
4) + 3.118 (see Eq.5 of step 5e) = 7.706 mm < 20 mm. Hence, o.k.

Thus, both the requirements of cl.23.2 of IS 456 and as given in sec.


7.17.3 are satisfied.

Version 2 CE IIT, Kharagpur


7.17.9 Practice Questions and Problems with Answers
Q.1: Why is it essential to check the structures, designed by the limit state of
collapse, by the limit state of serviceability?

A.1: See sec. 7.17.1.

Q.2: Explain short- and long-term deflections and the respective influencing
factors of them.

A.2: See sec. 7.17.2.

Q.3: State the stipulations of IS 456 regarding the control of deflection.

A.3: See sec. 7.17.3.

Q.4: How would you select the preliminary dimensions of structures to satisfy (i)
the deflection requirements, and (ii) the lateral stability ?

A.4: See secs. 7.17.4 A for (i) and B for (ii).

Q.5: Check the preliminary cross-sectional dimensions of Problem 1 of sec.


7.17.8 (Fig.7.17.1) if they satisfy the requirements of control of deflection.
The spacing of the beam is 3.5 m c/c. Other data are the same as those of
Problem 1 of sec. 7.17.8.

A.5:

Step 1: Check for the effective width

bf = lo /6 + bw + 6Df or spacing of the beam, whichever is less.

Here, bf = (8000/6) + 300 + 6(100) = 2234 < 3500. Hence, bf = 2234 mm


is o.k.

Step 2: Check for span to effective depth ratio

(i) As per row 1 of Table 7.1, the basic value of span to effective depth ratio is
20.

(ii) As per row 2 of Table 7.1, the modification factor is 1 since the span 8 m <
10 m.

(iii) As per row 5 of Table 7.1, the modification factor for the flanged beam is to
be obtained from Fig. 6 of IS 456 for which the ratio of web width to flange width

Version 2 CE IIT, Kharagpur


= 300/2234 = 0.134. Figure 6 of IS 456 gives the modification factor as 0.8. So,
the revised span to effective depth ratio = 20(0.8) = 16.

(iv) Row 3 of Table 7.1 deals with the area and stress of tensile steel. At the
preliminary stage these values are to be assumed. However, for this problem the
area of steel is given as 1383 mm2 (2-25T + 2-16T), for which pt = Ast(100)/bf d =
1383(100)/(2234)(550) = 0.112.

fs = 0.58 fy (area of cross-section of steel required)/(area of cross-section of steel


provided) = 0.58(415)(1) = 240.7 (assuming that the provided steel is the
same as required, which is a rare case). Figure 4 of IS 456 gives the modification
factor as 1.8. So, the revised span to effective depth ratio = 16(1.8) = 28.8.

(v) Row 4 is concerning the amount of compression steel. Here, compression


steel is not there. So, the modification factor = 1.

Therefore, the final span to effective depth ratio = 28.8.

Accordingly, effective depth of the beam = 8000/28.8 = 277.8 mm < 550


mm.

Hence, the dimensions of the cross-section are satisfying the


requirements.

7.17.9 References
1. Reinforced Concrete Limit State Design, 6th Edition, by Ashok K. Jain,
Nem Chand & Bros, Roorkee, 2002.
2. Limit State Design of Reinforced Concrete, 2nd Edition, by P.C.Varghese,
Prentice-Hall of India Pvt. Ltd., New Delhi, 2002.
3. Advanced Reinforced Concrete Design, by P.C.Varghese, Prentice-Hall of
India Pvt. Ltd., New Delhi, 2001.
4. Reinforced Concrete Design, 2nd Edition, by S.Unnikrishna Pillai and
Devdas Menon, Tata McGraw-Hill Publishing Company Limited, New
Delhi, 2003.
5. Limit State Design of Reinforced Concrete Structures, by P.Dayaratnam,
Oxford & I.B.H. Publishing Company Pvt. Ltd., New Delhi, 2004.
6. Reinforced Concrete Design, 1st Revised Edition, by S.N.Sinha, Tata
McGraw-Hill Publishing Company. New Delhi, 1990.
7. Reinforced Concrete, 6th Edition, by S.K.Mallick and A.P.Gupta, Oxford &
IBH Publishing Co. Pvt. Ltd. New Delhi, 1996.
8. Behaviour, Analysis & Design of Reinforced Concrete Structural Elements,
by I.C.Syal and R.K.Ummat, A.H.Wheeler & Co. Ltd., Allahabad, 1989.
9. Reinforced Concrete Structures, 3rd Edition, by I.C.Syal and A.K.Goel,
A.H.Wheeler & Co. Ltd., Allahabad, 1992.

Version 2 CE IIT, Kharagpur


10. Textbook of R.C.C, by G.S.Birdie and J.S.Birdie, Wiley Eastern Limited,
New Delhi, 1993.
11. Design of Concrete Structures, 13th Edition, by Arthur H. Nilson, David
Darwin and Charles W. Dolan, Tata McGraw-Hill Publishing Company
Limited, New Delhi, 2004.
12. Concrete Technology, by A.M.Neville and J.J.Brooks, ELBS with
Longman, 1994.
13. Properties of Concrete, 4th Edition, 1st Indian reprint, by A.M.Neville,
Longman, 2000.
14. Reinforced Concrete Designer’s Handbook, 10th Edition, by C.E.Reynolds
and J.C.Steedman, E & FN SPON, London, 1997.
15. Indian Standard Plain and Reinforced Concrete – Code of Practice (4th
Revision), IS 456: 2000, BIS, New Delhi.
16. Design Aids for Reinforced Concrete to IS: 456 – 1978, BIS, New Delhi.

7.17.11 Test 17 with Solutions


Maximum Marks = 50, Maximum Time = 30 minutes

Answer all questions.

TQ.1: Explain short- and long-term deflections and the respective influencing
factors of them.
(10 marks)

A.TQ.1: See sec. 7.17.2.

Version 2 CE IIT, Kharagpur


TQ.2: Check the preliminary dimensions of a singly reinforced rectangular
cantilever beam of span 4 m (Fig.7.17.4) using M 20 and Fe 415.
(15 marks)

A.TQ.2: (i) From row 1 of Table 7.1, the basic value of span to effective depth
ratio is 7.

(ii) Modification factor for row 2 is 1 as this is a singly reinforced beam.

(iii) Assuming pt as 0.6 and area of steel to be provided is the same as


area of steel required, fs = 0.58(415(1) = 240.7 N/mm2. From Fig. 4 of IS 456,
the modification factor = 1.18. Hence, the revised span to effective depth ratio is
7(1.18) = 8.26.

(iv) Modification factors for rows 4 and 5 are 1 as there is no


compression steel and this being a rectangular beam. Hence, the preliminary
effective depth needed = 4000/8.26 = 484.26 mm < 550 mm. Hence, o.k.

TQ.3: Determine the tensile steel of the cantilever beam of TQ 2 (Fig. 7.17.4)
subjected to service imposed load of 11.5 kN/m using M 20 and Fe 415.
Use Sp-16 for the design. Calculate short- and long-term deflections and
check the requirements of IS 456 regarding the deflection.
(25 marks)

A.TQ.3: Determination of tensile steel of the beam using SP-16:

Dead load of the beam = 0.3(0.6)(25) kN/m = 4.5 kN/m

Service imposed loads = 11.5 kN/m

Total service load = 16.0 kN/m

Factored load = 16(1.5) = 24 kN/m

Mu = 24(4)(4)/2 = 192 kNm

For this beam of total depth 600 mm, let us assume d = 550 mm.

Mu /bd2 = 192/(0.3)(0.55)(0.55) = 2115.70 kN/m2

Table 2 of SP-16 gives the corresponding pt = 0.678 + 0.007(0.015)/0.02


= 0.683

Again, for Mu per metre run as 192/0.3 = 640 kNm/m, chart 15 of SP-16
gives pt = 0.68 when d = 550 mm.

Version 2 CE IIT, Kharagpur


With pt = 0.683, Ast = 0.683(300)(500)/100 = 1126.95 mm2. Provide 4-
20T to have 1256 mm2. This gives provided pt = 0.761%.

Calculation of deflection

Step 1: Properties of concrete section

yt = D/2 = 300 mm, Igr = bD3/12 = 300(600)3/12 = 5.4(109) mm4

Step 2: Properties of cracked section

fcr = 0.7 20 (cl. 6.2.2 of IS 456) = 3.13 N/mm2

yt = 300 mm

Mr = fcr Igr /yt = 3.13(5.4)(109)/300 = 5.634(107) Nmm

Es = 200000 N/mm2

Ec = 5000 f ck (cl. 6.2.3.1 of IS 456) = 22360.68 N/mm2

m = Es /Ec = 8.94

Taking moment of the compressive concrete and tensile steel about the neutral
axis (Fig.7.17.5):

300 x2/2 = (8.94)(1256)(550 – x) or x2 + 74.86 x – 41171.68 = 0

Version 2 CE IIT, Kharagpur


This gives x = 168.88 mm and z = d – x/3 = 550 – 168.88/3 = 493.71 mm.

Ir = 300(168.88)3/3 + 8.94(1256)(550 – 168.88)2 = 2.1126(109) mm4

M = wl2/2 = 20(4)(4)/2 = 160 kNm

Ir
I eff = = 1.02 I r = 2.1548 (10 9 ) mm 4
5.634 493.71 168.88
(1.2) - ( )( ) (1 - ) (1)
16 550 550

This satisfies Ir ≤ Ieff ≤ Igr. So, Ieff = 2.1548(109) mm4.

Step 3: Short-term deflection (sec. 7.17.5)

Ec = 22360.68 N/mm2 (cl. 6.2.3.1 of IS 456)

Short-term deflection = wl4/8EcIeff

= 20(44)(1012)/8(22360.68)(2.1548)(109) = 13.283 mm

So, short-term deflection = 13.283 mm


(1)

Step 4: Deflection due to shrinkage (sec. 7.17.6)

k 4 = 0.72( 0.761) / 0.761 = 0.664

ψ cs = k 4 ε cs / D = (0.664)(0. 0003)/600 = 3.32(10) -7

k3 = 0.5 (from sec. 7.17.6)

α cs = k 3 ψ cs l 2 = (0.5)(3.32)(10)-7(16)(106) = 2.656 mm
(2)

Step 5: Deflection due to creep (sec. 7.17.7)

Step 5a: Calculation of α 1cc ( perm )

Assuming the age of concrete at loading as 28 days, cl. 6.2.5.1 of IS 456


gives
Version 2 CE IIT, Kharagpur
θ = 1.6

So, Ecc = Ec /(1 + θ ) = 8600.2615 N/mm2

m = Es /Ecc = 200000/8600.2615 = 23.255

Step 5b: Properties of cracked section

From Fig.7.17.6, taking moment of compressive concrete and tensile steel


about the neutral axis, we have:

300 x2/2 = (23.255)(1256)(550 - x)

or x2 + 194.72 x – 107097.03 = 0

solving we get x = 244.072 mm

z = d – x/3 = 468.643 mm

Ir = 300(244.072)3/3 + (23.255)(1256)(550 – 468.643)2

= 1.6473(10)9 mm4

Version 2 CE IIT, Kharagpur


Mr = 5.634( 107) Nmm (see Step 2)

M = wperm l2/2 = 4.5(42)/2 = 36 kNm

Ir
I eff = = 2.1786 I r = 3.5888(10 9 ) mm 4
5.634 468.643 244.072
1.2 - ( )( ) (1 - ) (1)
3.6 550 550

Since this satisfies Ir ≤ Ieff ≤ Igr, we have, Ieff = 3.5888(109) mm4. For the value
of Igr please see Step 1.

Step 5c: Calculation of α 1cc ( perm )

α 1cc ( perm ) = (wperm)( l4)/(8Ecc Ieff) = 4.5(4)4(10)12/8(8600.2615)(3.5888)(109)

= 4.665 mm
(3)

Step 5d: Calculation of α 1( perm )

α 1( perm ) = (wperm)( l4)/(8Ec Ieff) = 4.5(4)4(10)12/8(22360.68)(3.5888)(109)

= 1.794 mm
(4)

Step 5e: Calculation of deflection due to creep

α cc ( perm ) = α 1cc ( perm ) - α 1( perm )

= 4.665 – 1.794 = 2.871 mm


(5)

Moreover: α cc ( perm ) = α 1cc ( perm ) ( θ ) gives α cc ( perm ) = 1.794(1.6) = 2.874 mm.

Step 6: Checking of the two requirements of IS 456

Step 6a: First requirement

Maximum allowable deflection = 4000/250 = 16 mm

The actual deflection = 13.283 (Eq.1 of Step 3) + 2.656 (Eq.2 of Step 4)

Version 2 CE IIT, Kharagpur


+ 2.871 (Eq.5 of Step 5e) = 18.81 > Allowable 16 mm.

Step 6b: Second requirement

The allowable deflection is lesser of span/350 or 20 mm. Here, span/350 =


11.428 mm is the allowable deflection. The actual deflection = 1.794 (Eq.4 of
Step 5d) + 2.656 (Eq.2 of Step 4) + 2.871 (Eq.5 of step 5e) = 7.321 mm <
11.428 mm.

Remarks:

Though the second requirement is satisfying, the first requirement is not


satisfying. However the extra deflection is only 2.81 mm, which can be made up
by giving camber instead of revising the section.

7.17.12 Summary of this Lesson


This lesson illustrates the importance of checking the structures for the
limit state of serviceability after designing by the limit state of collapse. The short-
and long-term deflections along with their respective influencing factors are
explained. The code requirements for the control of deflection and the necessary
guidelines for the selection of dimensions of cross-section are stated. Numerical
examples, solved as illustrative example and given in the practice problem and
test will help the students in understanding the calculations clearly for their
application in the design problems.

Version 2 CE IIT, Kharagpur


Module
8
Reinforced Concrete
Slabs
Version 2 CE IIT, Kharagpur
Lesson
18
One-way Slabs
Version 2 CE IIT, Kharagpur
Instructional Objectives:
At the end of this lesson, the student should be able to:

• state the names of different types of slabs used in construction,

• identify one-way and two-way slabs stating the limits of ly /lx ratios for one
and two-way slabs,

• explain the share of loads by the supporting beams of one- and two-way
slabs when subjected to uniformly distributed vertical loads,

• explain the roles of the total depth in resisting the bending moments,
shear force and in controlling the deflection,

• state the variation of design shear strength of concrete in slabs of different


depths with identical percentage of steel reinforcement,

• assume the depth of slab required for the control of deflection for different
support conditions,

• determine the positive and negative bending moments and shear force,

• determine the amount of reinforcing bars along the longer span,

• state the maximum diameter of a bar that can be used in a particular slab
of given depth,

• decide the maximum spacing of reinforcing bars along two directions of


one-way slab,

• design one-way slab applying the design principles and following the
stipulated guidelines of IS 456,

• draw the detailing of reinforcing bars of one-way slabs after the design.

Version 2 CE IIT, Kharagpur


8.18.1 Introduction

Version 2 CE IIT, Kharagpur


Slabs, used in floors and roofs of buildings mostly integrated with the
supporting beams, carry the distributed loads primarily by bending. It has been
mentioned in sec. 5.10.1 of Lesson 10 that a part of the integrated slab is
considered as flange of T- or L-beams because of monolithic construction.
However, the remaining part of the slab needs design considerations. These
slabs are either single span or continuous having different support conditions like
fixed, hinged or free along the edges (Figs.8.18.1a,b and c). Though normally
these slabs are horizontal, inclined slabs are also used in ramps, stair cases and
inclined roofs (Figs.8.18.2 and 3). While square or rectangular plan forms are
normally used, triangular, circular and other plan forms are also needed for
different functional requirements. This lesson takes up horizontal and rectangular
/square slabs of buildings supported by beams in one or both directions and
subjected to uniformly distributed vertical loadings.

The other types of slabs, not taken up in this module, are given below. All
these slabs have additional requirements depending on the nature and
magnitude of loadings in respective cases.

(a) horizontal or inclined bridge and fly over deck slabs carrying heavy
concentrated loads,

(b) horizontal slabs of different plan forms like triangular, polygonal or


circular,

(c) flat slabs having no beams and supported by columns only,

(d) inverted slabs in footings with or without beams,

(e) slabs with large voids or openings,

(f) grid floor and ribbed slabs.

Version 2 CE IIT, Kharagpur


8.18.2 One-way and Two-way Slabs

Figures 8.18.4a and b explain the share of loads on beams supporting


solid slabs along four edges when vertical loads are uniformly distributed. It is
evident from the figures that the share of loads on beams in two perpendicular
directions depends upon the aspect ratio ly /lx of the slab, lx being the shorter
span. For large values of ly, the triangular area is much less than the trapezoidal
area (Fig.8.18.4a). Hence, the share of loads on beams along shorter span will
gradually reduce with increasing ratio of ly /lx. In such cases, it may be said that
the loads are primarily taken by beams along longer span. The deflection profiles
of the slab along both directions are also shown in the figure. The deflection
profile is found to be constant along the longer span except near the edges for
the slab panel of Fig.8.18.4a. These slabs are designated as one-way slabs as
Version 2 CE IIT, Kharagpur
they span in one direction (shorter one) only for a large part of the slab when ly
/lx > 2.

On the other hand, for square slabs of ly /lx = 1 and rectangular slabs of
ly /lx up to 2, the deflection profiles in the two directions are parabolic
(Fig.8.18.4b). Thus, they are spanning in two directions and these slabs with ly /lx
up to 2 are designated as two-way slabs, when supported on all edges.

It would be noted that an entirely one-way slab would need lack of support
on short edges. Also, even for ly /lx < 2, absence of supports in two parallel
edges will render the slab one-way. In Fig. 8.18.4b, the separating line at 45
degree is tentative serving purpose of design. Actually, this angle is a function of
ly /lx .

This lesson discusses the analysis and design aspects of one-way slabs.
The two-way slabs are taken up in the next lesson.

8.18.3 Design Shear Strength of Concrete in Slabs


Experimental tests confirmed that the shear strength of solid slabs up to a
depth of 300 mm is comparatively more than those of depth greater than 300
mm. Accordingly, cl.40.2.1.1 of IS 456 stipulates the values of a factor k to be
multiplied with τ c given in Table 19 of IS 456 for different overall depths of slab.
Table 8.1 presents the values of k as a ready reference below:

Table 8.1 Values of the multiplying factor k

Overall 300 or 275 250 225 200 175 150 or


depth of more less
slab (mm)
k 1.00 1.05 1.10 1.15 1.20 1.25 1.30

Thin slabs, therefore, have more shear strength than that of thicker slabs.
It is the normal practice to choose the depth of the slabs so that the concrete can
resist the shear without any stirrups for slab subjected to uniformly distributed
loads. However, for deck slabs, culverts, bridges and fly over, shear
reinforcement should be provided as the loads are heavily concentrated in those
slabs. Though, the selection of depth should be made for normal floor and roof
slabs to avoid stirrups, it is essential that the depth is checked for the shear for
these slabs taking due consideration of enhanced shear strength as discussed
above depending on the overall depth of the slabs.

Version 2 CE IIT, Kharagpur


8.18.4 Structural Analysis
As explained in sec. 8.18.2, one-way slabs subjected to mostly uniformly
distributed vertical loads carry them primarily by bending in the shorter direction.
Therefore, for the design, it is important to analyse the slab to find out the
bending moment (both positive and negative) depending upon the supports.
Moreover, the shear forces are also to be computed for such slabs. These
internal bending moments and shear forces can be determined using elastic
method of analysis considering the slab as beam of unit width i.e. one metre
(Fig.8.18.1a). However, these values may also be determined with the help of the
coefficients given in Tables 12 and 13 of IS 456 in cl.22.5.1. It is worth
mentioning that these coefficients are applicable if the slab is of uniform cross-
section and subjected to substantially uniformly distributed loads over three or
more spans and the spans do not differ by more than fifteen per cent of the
longer span. It is also important to note that the average of the two values of the
negative moment at the support should be considered for unequal spans or if the
spans are not equally loaded. Further, the redistribution of moments shall not be
permitted to the values of moments obtained by employing the coefficients of
bending moments as given in IS 456.

For slabs built into a masonry wall developing only partial restraint, the
negative moment at the face of the support should be taken as Wl/24, where W
is the total design loads on unit width and l is the effective span. The shear
coefficients, given in Table 13 of IS 456, in such a situation, may be increased by
0.05 at the end support as per cl.22.5.2 of IS 456.

8.18.5 Design Considerations


The primary design considerations of both one and two-way slabs are
strength and deflection. The depth of the slab and areas of steel reinforcement
are to be determined from these two aspects. The detailed procedure of design
of one-way slab is taken up in the next section. However, the following aspects
are to be decided first.

(a) Effective span (cl.22.2 of IS 456)

The effective span of a slab depends on the boundary condition. Table 8.2
gives the guidelines stipulated in cl.22.2 of IS 456 to determine the effective span
of a slab.

Table 8.2 Effective span of slab (cl.22.2 of IS 456)

Sl.No. Support condition Effective span


1 Simply supported not built integrally Lesser of (i) clear span +
with its supports effective depth of slab, and (ii)
centre to centre of supports

Version 2 CE IIT, Kharagpur


2 Continuous when the width of the Do
support is < 1/12th of clear span
3 Continuous when the width of the (i) Clear span between the
support is > lesser of 1/12th of clear supports
span or 600 mm
(i) for end span with one end fixed (ii) Lesser of (a) clear span +
and the other end continuous or for half the effective depth of slab,
intermediate spans, and (b) clear span + half the
width of the discontinuous
(ii) for end span with one end free and support
the other end continuous,
(iii) The distance between the
(iii) spans with roller or rocker centres of bearings
bearings.
4 Cantilever slab at the end of a Length up to the centre of
continuous slab support
5 Cantilever span Length up to the face of the
support + half the effective
depth
6 Frames Centre to centre distance

(b) Effective span to effective depth ratio (cls.23.2.1a-e of IS 456)

The deflection of the slab can be kept under control if the ratios of
effective span to effective depth of one-way slabs are taken up from the
provisions in cl.23.2.1a-e of IS 456. These stipulations are for the beams and are
also applicable for one-way slabs as they are designed considering them as
beam of unit width. These provisions are explained in sec.3.6.2.2 of Lesson 6.

(c) Nominal cover (cl.26.4 of IS 456)

The nominal cover to be provided depends upon durability and fire


resistance requirements. Table 16 and 16A of IS 456 provide the respective
values. Appropriate value of the nominal cover is to be provided from these
tables for the particular requirement of the structure.

(d) Minimum reinforcement (cl.26.5.2.1 of IS 456)

Both for one and two-way slabs, the amount of minimum reinforcement in
either direction shall not be less than 0.15 and 0.12 per cents of the total cross-
sectional area for mild steel (Fe 250) and high strength deformed bars (Fe 415
and Fe 500)/welded wire fabric, respectively.

Version 2 CE IIT, Kharagpur


(e) Maximum diameter of reinforcing bars (cl.26.5.2.2)

The maximum diameter of reinforcing bars of one and two-way slabs shall
not exceed one-eighth of the total depth of the slab.

(f) Maximum distance between bars (cl.26.3.3 of IS 456)

The maximum horizontal distance between parallel main reinforcing bars


shall be the lesser of (i) three times the effective depth, or (ii) 300 mm. However,
the same for secondary/distribution bars for temperature, shrinkage etc. shall be
the lesser of (i) five times the effective depth, or (ii) 450 mm.

8.18.6 Design of One-way Slabs


The procedure of the design of one-way slab is the same as that of
beams. However, the amounts of reinforcing bars are for one metre width of the
slab as to be determined from either the governing design moments (positive or
negative) or from the requirement of minimum reinforcement. The different steps
of the design are explained below.

Step 1: Selection of preliminary depth of slab

The depth of the slab shall be assumed from the span to effective depth
ratios as given in section 3.6.2.2 of Lesson 6 and mentioned here in sec.8.18.5b.

Step 2: Design loads, bending moments and shear forces

The total factored (design) loads are to be determined adding the


estimated dead load of the slab, load of the floor finish, given or assumed live
loads etc. after multiplying each of them with the respective partial safety factors.
Thereafter, the design positive and negative bending moments and shear forces
are to be determined using the respective coefficients given in Tables 12 and 13
of IS 456 and explained in sec.8.18.4 earlier.

Step 3: Determination/checking of the effective and total depths of slabs

The effective depth of the slab shall be determined employing Eq.3.25 of


sec.3.5.6 of Lesson 5 and is given below as a ready reference here,

Mu,lim = R,lim bd2 …. (3.25)

where the values of R,lim for three different grades of concrete and three
different grades of steel are given in Table 3.3 of Lesson 5 (sec.3.5.6). The value
of b shall be taken as one metre.

Version 2 CE IIT, Kharagpur


The total depth of the slab shall then be determined adding appropriate
nominal cover (Table 16 and 16A of cl.26.4 of IS 456) and half of the diameter of
the larger bar if the bars are of different sizes. Normally, the computed depth of
the slab comes out to be much less than the assumed depth in Step 1. However,
final selection of the depth shall be done after checking the depth for shear force.

Step 4: Depth of the slab for shear force

Theoretically, the depth of the slab can be checked for shear force if the
design shear strength of concrete is known. Since this depends upon the
percentage of tensile reinforcement, the design shear strength shall be assumed
considering the lowest percentage of steel. The value of τ c shall be modified
after knowing the multiplying factor k from the depth tentatively selected for the
slab in Step 3. If necessary, the depth of the slab shall be modified.

Step 5: Determination of areas of steel

Area of steel reinforcement along the direction of one-way slab should be


determined employing Eq.3.23 of sec.3.5.5 of Lesson 5 and given below as a
ready reference.

Mu = 0.87 fy Ast d {1 – (Ast)(fy)/(fck)(bd)} …. (3.23)

The above equation is applicable as the slab in most of the cases is under-
reinforced due to the selection of depth larger than the computed value in Step 3.
The area of steel so determined should be checked whether it is at least the
minimum area of steel as mentioned in cl.26.5.2.1 of IS 456 and explained in
sec.8.18.5d.

Alternatively, tables and charts of SP-16 may be used to determine the


depth of the slab and the corresponding area of steel. Tables 5 to 44 of SP-16
covering a wide range of grades of concrete and Chart 90 shall be used for
determining the depth and reinforcement of slabs. Tables of SP-16 take into
consideration of maximum diameter of bars not exceeding one-eighth the depth
of the slab. Zeros at the top right hand corner of these tables indicate the region
where the percentage of reinforcement would exceed pt,lim. Similarly, zeros at
the lower left and corner indicate the region where the reinforcement is less than
the minimum stipulated in the code. Therefore, no separate checking is needed
for the allowable maximum diameter of the bars or the computed area of steel
exceeding the minimum area of steel while using tables and charts of SP-16.

The amount of steel reinforcement along the large span shall be the
minimum amount of steel as per cl.26.5.2.1 of IS 456 and mentioned in
sec.8.18.5d earlier.

Version 2 CE IIT, Kharagpur


Step 6: Selection of diameters and spacings of reinforcing bars (cls.26.5.2.2
and 26.3.3 of IS 456)

The diameter and spacing of bars are to be determined as per cls.26.5.2.2


and 26.3.3 of IS 456. As mentioned in Step 5, this step may be avoided when
using the tables and charts of SP-16.

8.18.7 Detailing of Reinforcement

Version 2 CE IIT, Kharagpur


Figures 8.18.5a and b present the plan and section 1-1 of one-way
continuous slab showing the different reinforcing bars in the discontinuous and
continuous ends (DEP and CEP, respectively) of end panel and continuous end
of adjacent panel (CAP). The end panel has three bottom bars B1, B2 and B3
and four top bars T1, T2, T3 and T4. Only three bottom bars B4, B5 and B6 are
shown in the adjacent panel. Table 8.3 presents these bars mentioning the
respective zone of their placement (DEP/CEP/CAP), direction of the bars (along
x or y) and the resisting moment for which they shall be designed or if to be
provided on the basis of minimum reinforcement. These bars are explained
below for the three types of ends of the two panels.

Table 8.3 Steel bars of one-way slab (Figs.8.18.5a and b)

Sl.No. Bars Panel Along Resisting moment


1 B1, B2 DEP x + 0.5 Mx for each,

2 B3 DEP y Minimum steel


3 B4, B5 CAP x + 0.5 Mx for each,

4 B6 CAP y Minimum steel


5 T1, T2 CEP x - 0.5 Mx for each,

6 T3 DEP x + 0.5 Mx
7 T4 DEP y Minimum steel

Notes: (i) DEP = Discontinuous End Panel


(ii) CEP = Continuous End Panel
(iii) CAP = Continuous Adjacent Panel

(i) Discontinuous End Panel (DEP)

• Bottom steel bars B1 and B2 are alternately placed such that B1 bars are
curtailed at a distance of 0.25 lx1 from the adjacent support and B2 bars
are started from a distance of 0.15lx1 from the end support. Thus, both
B1 and B2 bars are present in the middle zone covering 0.6lx1, each of
which is designed to resist positive moment 0.5Mx. These bars are along
the direction of x and are present from one end to the other end of ly.

• Bottom steel bars B3 are along the direction of y and cover the entire
span lx1 having the minimum area of steel. The first bar shall be placed
at a distance not exceeding s/2 from the left discontinuous support,
where s is the spacing of these bars in y direction.

• Top bars T3 are along the direction of x for resisting the negative
moment which is numerically equal to fifty per cent of positive Mx. These

Version 2 CE IIT, Kharagpur


bars are continuous up to a distance of 0.1lx1 from the centre of support
at the discontinuous end.

• Top bars T4 are along the direction of y and provided up to a distance of


0.1lx1 from the centre of support at discontinuous end. These are to
satisfy the requirement of minimum steel.

(ii) Continuous End Panel (CEP)

• Top bars T1 and T2 are along the direction of x and cover the entire ly.
They are designed for the maximum negative moment Mx and each has
a capacity of -0.5Mx. Top bars T1 are continued up to a distance of
0.3lx1, while T2 bars are only up to a distance of 0.15lx1.

• Top bars T4 are along y and provided up to a distance of 0.3lx1 from the
support. They are on the basis of minimum steel requirement.

(iii) Continuous Adjacent Panel (CAP)

• Bottom bars B4 and B5 are similar to B1 and B2 bars of (i) above.

• Bottom bars B6 are similar to B3 bars of (i) above.

Detailing is an art and hence structural requirement can be satisfied by more


than one mode of detailing each valid and acceptable.

8.18.8 Numerical Problems


(a) Problem 8.1

Design the one-way continuous slab of Fig.8.18.6 subjected to uniformly


distributed imposed loads of 5 kN/m2 using M 20 and Fe 415. The load of floor
finish is 1 kN/m2. The span dimensions shown in the figure are effective spans.
The width of beams at the support = 300 mm.

Version 2 CE IIT, Kharagpur


Solution of Problem 8.1

Step 1: Selection of preliminary depth of slab

The basic value of span to effective depth ratio for the slab having simple
support at the end and continuous at the intermediate is (20+26)/2 = 23 (cl.23.2.1
of IS 456).

Modification factor with assumed p = 0.5 and fs = 240 N/mm2 is obtained


as 1.18 from Fig.4 of IS 456.

Therefore, the minimum effective depth = 3000/23(1.18) = 110.54 mm. Let


us take the effective depth d = 115 mm and with 25 mm cover, the total depth
D = 140 mm.

Step 2: Design loads, bending moment and shear force

Dead loads of slab of 1 m width = 0.14(25) = 3.5 kN/m

Dead load of floor finish =1.0 kN/m

Factored dead load = 1.5(4.5) = 6.75 kN/m

Factored live load = 1.5(5.0) = 7.50 kN/m

Total factored load = 14.25 kN/m

Maximum moments and shear are determined from the coefficients given
in Tables 12 and 13 of IS 456.

Maximum positive moment = 14.25(3)(3)/12 = 10.6875 kNm/m

Maximum negative moment = 14.25(3)(3)/10 = 12.825 kNm/m

Maximum shear Vu = 14.25(3)(0.4) = 17.1 kN

Step 3: Determination of effective and total depths of slab

From Eq.3.25 of sec. 3.5.6 of Lesson 5, we have

Mu,lim = R,lim bd2 where R,lim is 2.76 N/mm2 from Table 3.3 of sec. 3.5.6
of Lesson 5. So, d = {12.825(106)/(2.76)(1000)}0.5 = 68.17 mm

Version 2 CE IIT, Kharagpur


Since, the computed depth is much less than that determined in Step 1, let
us keep D = 140 mm and d = 115 mm.

Step 4: Depth of slab for shear force

Table 19 of IS 456 gives τ c = 0.28 N/mm2 for the lowest percentage of


steel in the slab. Further for the total depth of 140 mm, let us use the coefficient
k of cl. 40.2.1.1 of IS 456 as 1.3 to get τ c = k τ c = 1.3(0.28) = 0.364 N/mm2.

Table 20 of IS 456 gives τ c max = 2.8 N/mm2. For this problem τ v = Vu /bd
= 17.1/115 = 0.148 N/mm2. Since, τ v < τ c < τ c max , the effective depth d = 115 mm
is acceptable.

Step 5: Determination of areas of steel

From Eq.3.23 of sec. 3.5.5 of Lesson 5, we have

Mu = 0.87 fy Ast d {1 – (Ast)(fy)/(fck)(bd)}

(i) For the maximum negative bending moment

12825000 = 0.87(415)(Ast)(115){1 – (Ast)(415)/(1000)(115)(20)}

or Ast2 - 5542.16 Ast + 1711871.646 = 0

Solving the quadratic equation, we have the negative Ast = 328.34 mm2

(ii) For the maximum positive bending moment

10687500 = 0.87(415) Ast(115) {1 – (Ast)(415)/(1000)(115)(20)}

or Ast2 - 5542.16 Ast + 1426559.705 = 0

Solving the quadratic equation, we have the positive Ast = 270.615 mm2

Alternative approach: Use of Table 2 of SP-16

(i) For negative bending moment

Mu/bd2 = 0.9697

Version 2 CE IIT, Kharagpur


Table 2 of SP-16 gives: ps = 0.2859 (by linear interpolation). So, the area of
negative steel = 0.2859(1000)(115)/100 = 328.785 mm2.

(ii) For positive bending moment

Mu/bd2 = 0.8081

Table 2 of SP-16 gives: ps = 0.23543 (by linear interpolation). So, the area of
positive steel = 0.23543(1000)(115)/100 = 270.7445 mm2.

These areas of steel are comparable with those obtained by direct


computation using Eq.3.23.

Distribution steel bars along longer span ly

Distribution steel area = Minimum steel area = 0.12(1000)(140)/100 = 168


2
mm . Since, both positive and negative areas of steel are higher than the
minimum area, we provide:

(a) For negative steel: 10 mm diameter bars @ 230 mm c/c for which Ast
= 341 mm2 giving ps = 0.2965.

(b) For positive steel: 8 mm diameter bars @ 180 mm c/c for which Ast =
279 mm2 giving ps = 0.2426

(c) For distribution steel: Provide 8 mm diameter bars @ 250 mm c/c for
which Ast (minimum) = 201 mm2.

Step 6: Selection of diameter and spacing of reinforcing bars

The diameter and spacing already selected in step 5 for main and
distribution bars are checked below:

For main bars (cl. 26.3.3.b.1 of IS 456), the maximum spacing is the
lesser of 3d and 300 mm i.e., 300 mm. For distribution bars (cl. 26.3.3.b.2 of IS
456), the maximum spacing is the lesser of 5d or 450 mm i.e., 450 mm. Provided
spacings, therefore, satisfy the requirements.

Maximum diameter of the bars (cl. 26.5.2.2 of IS 456) shall not exceed
140/8 = 17 mm is also satisfied with the bar diameters selected here.

Version 2 CE IIT, Kharagpur


Figure 8.18.7 presents the detailing of the reinforcement bars. The
abbreviation B1 to B3 and T1 to T4 are the bottom and top bars, respectively
which are shown in Fig.8.18.5 for a typical one-way slab.

The above design and detailing assume absence of support along short
edges. When supports along short edges exist and there is eventual clamping
top reinforcement would be necessary at shorter supports also.

8.18.9 Practice Questions and Problems with Answers


Q.1: State the names of different types of slabs used in construction.

A.1: See sec. 8.18.1.

Q.2: (a) State the limit of the aspect ratio of ly/lx of one- and two-way slabs.
(b) Explain the share of loads by the supporting beams in one- and two-
way slabs.

A.2: (a) The aspect ratio ly/lx (lx is the shorter one) is from 1 to 2 for two-way
slabs and beyond 2 for one-way slabs.

(b) See sec. 8.18.2.

Q.3: How to determine the design shear strength of concrete in slabs of


different depths having the same percentage of reinforcement?

A.3: See sec. 8.18.3.

Version 2 CE IIT, Kharagpur


Q.4: State span to depth ratios of one-way slabs for different support conditions
to be considered for the control of deflection.

A.4: See sec. 8.18.5b.

Q.5: State the minimum amounts of reinforcing bars to be provided in slabs?

A.5: See sec. 8.18.5d.

Q.6: State the maximum diameter of a bar to be used in slabs.

A.6: See sec. 8.18.5e.

Q.7: How do we determine the effective depth of a slab for a given factored
moment?

A.7: See sec. 8.18.6, Step 3, Eq.3.25.

Q.8: How do we determine the area of steel to be provided for a given


factored moment?

A.8: See sec. 8.18.6, Step 5, Eq.3.23.

Q.9: How do we determine the amount of steel in the longer span direction?

A.9: Minimum amount of steel shall be provided for temperature, shrinkage etc.
as per cl. 26.5.2.1 of IS 456. These are known as distribution bars.

Q.10: Design the cantilever panel of the one-way slab shown in Fig.8.18.8
subjected to uniformly distributed imposed loads 5 kN/m2 using M 20 and
Fe 415. The load of floor finish is 0.75 kN/m2. The span dimensions shown
in the figure are effective spans. The width of the support is 300 mm.

A.10:

Version 2 CE IIT, Kharagpur


Step 1: Selection of preliminary depth of slab

Basic value of span to depth ratio (cl. 23.2.1 of IS 456) = 7

Modification factor = 1.18 (see Problem 8.1)

Minimum effective depth = 1850/7(1.18) = 223.97 mm

Assume d = 225 mm and D = 250 mm.

Step 2: Design loads, bending moment and shear force

Factored dead loads = (1.5)(0.25)(25) = 9.375 kN/m

Factored load of floor finish = (1.5)(0.75) = 1.125 kN/m

Factored live loads = (1.5)(5) = 7.5 kN/m

Total factored loads = 18.0 kN/m

Maximum negative moment = 18.0(1.85)(1.85)(0.5) = 30.8025 kNm/m

Maximum shear force = 18.0(1.85) = 33.3 kN/m

Step 3: Determination of effective and total depths of slab

From Eq.3.25, Step 3 of sec. 8.18.6, we have

d = {30.8025(106)/2.76(103)}0.5 = 105.64 mm, considering the value of


R = 2.76 N/mm2 from Table 3.3 of sec. 3.5.5 of Lesson 5. This depth is less
than assumed depth of slab in Step 1. Hence, assume d = 225 mm and D =
250 mm.

Step 4: Depth of slab for shear force

Using the value of k = 1.1 (cl. 40.2.1.1 of IS 456) for the slab of 250 mm
depth, we have τ c (from Table 19 of IS 456) = 1.1(0.28) = 0.308 N/mm2. Table
20 of IS 456 gives τ c max = 2.8 N/mm2. Here, τ v = Vu / bd = 33.3/225 = 0.148 N/mm 2 .
The depth of the slab is safe in shear as τ v < τ c < τ c max .

Step 5: Determination of areas of steel (using table of SP-16)

Table 44 gives 10 mm diameter bars @ 200 mm c/c can resist 31.43


kNm/m > 30.8025 kNm/m. Fifty per cent of the bars should be curtailed at a

Version 2 CE IIT, Kharagpur


distance of larger of Ld or 0.5lx. Table 65 of SP-16 gives Ld of 10 mm bars =
470 mm and 0.5lx = 0.5(1850) = 925 mm from the face of the column. The
curtailment distance from the centre line of beam = 925 + 150 = 1075, say 1100
mm.

The above, however, is not admissible as the spacing of bars after the
curtailment exceeds 300 mm. So, we provide 10 mm @ 300 c/c and 8 mm @
300 c/c. The moment of resistance of this set is 34.3 kNm/m > 30.8025 kNm/m
(see Table 44 of SP-16).

Figure 8.18.9 presents the detailing of reinforcing bars of this problem.

8.18.10 References
1. Reinforced Concrete Limit State Design, 6th Edition, by Ashok K. Jain,
Nem Chand & Bros, Roorkee, 2002.
2. Limit State Design of Reinforced Concrete, 2nd Edition, by P.C.Varghese,
Prentice-Hall of India Pvt. Ltd., New Delhi, 2002.
3. Advanced Reinforced Concrete Design, by P.C.Varghese, Prentice-Hall of
India Pvt. Ltd., New Delhi, 2001.
4. Reinforced Concrete Design, 2nd Edition, by S.Unnikrishna Pillai and
Devdas Menon, Tata McGraw-Hill Publishing Company Limited, New
Delhi, 2003.
5. Limit State Design of Reinforced Concrete Structures, by P.Dayaratnam,
Oxford & I.B.H. Publishing Company Pvt. Ltd., New Delhi, 2004.
6. Reinforced Concrete Design, 1st Revised Edition, by S.N.Sinha, Tata
McGraw-Hill Publishing Company. New Delhi, 1990.
7. Reinforced Concrete, 6th Edition, by S.K.Mallick and A.P.Gupta, Oxford &
IBH Publishing Co. Pvt. Ltd. New Delhi, 1996.
Version 2 CE IIT, Kharagpur
8. Behaviour, Analysis & Design of Reinforced Concrete Structural Elements,
by I.C.Syal and R.K.Ummat, A.H.Wheeler & Co. Ltd., Allahabad, 1989.
9. Reinforced Concrete Structures, 3rd Edition, by I.C.Syal and A.K.Goel,
A.H.Wheeler & Co. Ltd., Allahabad, 1992.
10. Textbook of R.C.C, by G.S.Birdie and J.S.Birdie, Wiley Eastern Limited,
New Delhi, 1993.
11. Design of Concrete Structures, 13th Edition, by Arthur H. Nilson, David
Darwin and Charles W. Dolan, Tata McGraw-Hill Publishing Company
Limited, New Delhi, 2004.
12. Concrete Technology, by A.M.Neville and J.J.Brooks, ELBS with
Longman, 1994.
13. Properties of Concrete, 4th Edition, 1st Indian reprint, by A.M.Neville,
Longman, 2000.
14. Reinforced Concrete Designer’s Handbook, 10th Edition, by C.E.Reynolds
and J.C.Steedman, E & FN SPON, London, 1997.
15. Indian Standard Plain and Reinforced Concrete – Code of Practice (4th
Revision), IS 456: 2000, BIS, New Delhi.
16. Design Aids for Reinforced Concrete to IS: 456 – 1978, BIS, New Delhi.

8.18.11 Test 18 with Solutions


Maximum Marks = 50, Maximum Time = 30 minutes

Answer all questions.

TQ.1: (a) State the limit of the aspect ratio of ly/lx of one- and two-way slabs.
(b) Explain the share of loads by the supporting beams in one- and two-
way slabs.
(10 marks)

A.TQ.1: (a) The aspect ratio ly/lx (lx is the shorter one) is from 1 to 2 for two-way
slabs and beyond 2 for one-way slabs.

(b) See sec. 8.18.2.

TQ.2: How to determine the design shear strength of concrete in slabs of


different depths having the same percentage of reinforcement?
(10 marks)

A.TQ.2: See sec. 8.18.3.

TQ3: Determine the areas of steel, bar diameters and spacings in the two
directions of a simply supported slab of effective spans 3.5 m x 8 m
(Figs.8.18.10a and b) subjected to live loads of 4 kN/m2 and the load of

Version 2 CE IIT, Kharagpur


floor finish is 1 kN/m2. Use M 20 and Fe 415. Draw the diagram showing
the detailing of reinforcement. (30 marks)

A.TQ.3:

Version 2 CE IIT, Kharagpur


This is one-way slab as ly/lx = 8/3.5 = 2.285 > 2. The calculations are
shown in different steps below:

Step 1: Selection of preliminary depth of slab

Clause 23.2.1 stipulates the basic value of span to effective depth ratio of
20. Using the modification factor of 1.18 from Fig.4 of IS 456, with p = 0.5 per
cent and fs = 240 N/mm2, we have the span to effective depth ratio = 20(1.18) =
23.6.

So, the minimum effective depth of slab = 3500/23.6 = 148.305 mm. Let
us take d = 150 mm and D = 175 mm.

Step 2: Design loads, bending moment and shear force

Factored dead loads of slab = (1.5)(0.175)(25) = 6.5625 kN/m

Factored load of floor finish = (1.5)(1) = 1.5 kN/m

Factored live load = (1.5)(4) = 6.0 kN/m

Total factored load = 14.0625 kN/m

Maximum positive bending moment = 14.0625(3.5)(3.5)/8 = 21.533


kNm/m

Maximum shear force = 14.0625(3.5)(0.5) = 24.61 kN/m

Step 3: Determination/checking of the effective and total depths of slab

Using Eq.3.25 as explained in Step 3 of sec. 8.18.6, we have

d = {21.533(106)/(2.76)(103)}0.5 = 88.33 mm < 150 mm, as assumed in


Step 1. So, let us keep d = 150 mm and D = 175 mm.

Step 4: Depth of the slab for shear force

With the multiplying factor k = 1.25 for the depth as 175 mm (vide Table
8.1 of this lesson) and τ c = 0.28 N/mm2 from Table 19 of IS 456, we have τ c =
1.25(0.28) = 0.35 N/mm2.

Table 20 of IS 456 gives τ c max = 2.8 N/mm2. For this problem:


τ v = 24.61(1000 )/(1000)(1 50) = 0.1641 N/mm 2 .

Thus, the effective depth of slab as 150 mm is safe as τ v < τ c < τ c max .

Version 2 CE IIT, Kharagpur


Step 5: Determination of areas of steel

Table 41 of SP-16 gives 8 mm diameter bars @ 120 mm c/c have 22.26


kNm/m > 21.533 kNm/m. Hence, provide 8 mm T @ 120 mm c/c as main positive
steel bars along the short span of 3.5 mm.

The minimum amount of reinforcement (cl. 26.5.2.1 of IS 456) =


0.12(175)(1000)/100 = 210 mm2. Provide 6 mm diameter bars @ 120 mm c/c
(236 mm2) along the large span of 8m.

Figure 8.18.10b shows the detailing of reinforcing bars.

8.18.12 Summary of this Lesson


This lesson mentions the different types of slabs used in construction and
explains the differences between one and two-way slabs. Illustrating the
principles of design as strength and deflection, the methods of determining the
bending moments and shear forces are explained. The stipulated guidelines of
assuming the preliminary depth, maximum diameter of reinforcing bars, nominal
covers, spacing of reinforcements, minimum amount of reinforcing bars etc. are
illustrated. The steps of the design of one-way slabs are explained. Design
problems are solved to illustrate the application of design guidelines. Further, the
detailing of reinforcement bars are explained for typical one-way slab and for the
numerical problems solved in this lesson.

Version 2 CE IIT, Kharagpur


Module
8
Reinforced Concrete
Slabs
Version 2 CE IIT, Kharagpur
Lesson
19
Two-way Slabs
Version 2 CE IIT, Kharagpur
Instructional Objectives:
At the end of this lesson, the student should be able to:

• determine the shear force of two-way slabs subjected to uniformly


distributed loads,

• state the two types of two-way slabs mentioning the differences between
them,

• determine the preliminary depth of two-way slabs of different support


conditions from the span to effective depth ratios as given in IS 456,

• explain the provisions of torsion reinforcing bars at two types of corners of


a restrained two-way slab,

• design the two types of two-way slabs applying the different methods
explained in this lesson and draw the detailing of reinforcing bars.

8.19.1 Introduction
Lesson 18 explains the various types of slabs with different support
conditions, plan forms, horizontal/inclined etc. Moreover, sec. 8.18.2 of Lesson
18 illustrates the sharing of uniformly distributed loads to the supporting beams of
both one and two-way slabs including the profiles of deflection (Figs.8.18.4a and
b). It is, thus, understood that two-way slabs span in both directions having the
aspect ratio of ly/lx up to 2, considering lx as the shorter span. This lesson
presents the different aspects of analysis and design of two-way slabs. Many of
the stipulations of IS 456 are the same as those of one-way slabs. While
mentioning the common stipulations with their respective section in Lesson 18,
this lesson presents other relevant requirements regarding the analysis, design
and detailing of two-way slabs. Numerical problems are also solved to illustrate
the applications of the theory in the design of two-way slabs.

8.19.2 Two-way Slabs


Two-way slabs subjected mostly to uniformly distributed loads resist them
primarily by bending about both the axis. However, as in the one-way slab, the
depth of the two-way slabs should also be checked for the shear stresses to
avoid any reinforcement for shear. Moreover, these slabs should have sufficient
depth for the control deflection. Thus, strength and deflection are the
requirements of design of two-way slabs.

Version 2 CE IIT, Kharagpur


8.19.3 Design Shear Strength of Concrete
Design shear strength of concrete in two-way slabs is to be determined
incorporating the multiplying factor k from Table 8.1 of Lesson 18 in the same
manner as discussed in sec. 8.18.3 of Lesson 18.

8.19.4 Structural Analysis

8.19.4.1 Computation of shear force

Shear forces are computed following the procedure stated below with
reference to Fig.8.19.1.

The two-way slab of Fig. 8.19.1 is divided into two trapezoidal and two
triangular zones by drawing lines from each corner at an angle of 45o. The loads
of triangular segment A will be transferred to beam 1-2 and the same of
trapezoidal segment B will be beam 2-3. The shear forces per unit width of the
strips aa and bb are highest at the ends of strips. Moreover, the length of half the
strip bb is equal to the length of the strip aa. Thus, the shear forces in both strips
are equal and we can write,
Vu = W (lx/2)
(8.1)
where W = intensity of the uniformly distributed loads.

The nominal shear stress acting on the slab is then determined from

τ v = Vu / bd (8.2)

Version 2 CE IIT, Kharagpur


8.19.4.2 Computation of bending moments

Two-way slabs spanning in two directions at right angles and carrying


uniformly distributed loads may be analysed using any acceptable theory.
Pigeoud’s or Wester-guard’s theories are the suggested elastic methods and
Johansen’s yield line theory is the most commonly used in the limit state of
collapse method and suggested by IS 456 in the note of cl. 24.4. Alternatively,
Annex D of IS 456 can be employed to determine the bending moments in the
two directions for two types of slabs: (i) restrained slabs, and (ii) simply
supported slabs. The two methods are explained below:

(i) Restrained slabs

Restrained slabs are those whose corners are prevented from lifting due
to effects of torsional moments. These torsional moments, however, are not
computed as the amounts of reinforcement are determined from the computed
areas of steel due to positive bending moments depending upon the intensity of
torsional moments of different corners. This aspect has been explained in Step 7
of sec. 8.19.6. Thus, it is essential to determine the positive and negative
bending moments in the two directions of restrained slabs depending on the
various types of panels and the aspect ratio ly/lx.

Restrained slabs are considered as divided into two types of strips in each
direction: (i) one middle strip of width equal to three-quarters of the respective
length of span in either directions, and (ii) two edge strips, each of width equal to
one-eighth of the respective length of span in either directions. Figures 8.19.2a
and b present the two types of strips for spans lx and ly separately.

Version 2 CE IIT, Kharagpur


The maximum positive and negative moments per unit width in a slab are
determined from

M x = α x w l x2
(8.3)

M y = α x w l y2
(8.4)

where α x and α y are coefficients given in Table 26 of IS 456, Annex D, cl. D-


1.1. Total design load per unit area is w and lengths of shorter and longer spans
are represented by lx and ly, respectively. The values of α x and α y , given in
Table 26 of IS 456, are for nine types of panels having eight aspect ratios of ly/lx
from one to two at an interval of 0.1. The above maximum bending moments are
applicable only to the middle strips and no redistribution shall be made.

Tension reinforcing bars for the positive and negative maximum moments
are to be provided in the respective middle strips in each direction. Figure 8.19.2
shows the positive and negative coefficients α x and α y .

The edge strips will have reinforcing bars parallel to that edge following
the minimum amount as stipulated in IS 456.

The detailing of all the reinforcing bars for the respective moments and for
the minimum amounts as well as torsional requirements are discussed in sec.
8.19.7(i).

(ii) Simply supported slabs

The maximum moments per unit width of simply supported slabs, not
having adequate provision to resist torsion at corners and to prevent the corners
from lifting, are determined from Eqs.8.3 and 8.4, where α x and α y are the
respective coefficients of moments as given in Table 27 of IS 456, cl. D-2. The
notations Mx, My, w, lx and ly are the same as mentioned below Eqs.8.3 and
8.4 in (i) above.

The detailing of reinforcing bars for the respective moments is explained in


sec. 8.19.7(ii).

Version 2 CE IIT, Kharagpur


The coefficients α x and α y of simply supported two-way slabs are
derived from the Grashoff-Rankine formula which is based on the consideration
of the same deflection at any point P (Fig.8.19.3) of two perpendicular
interconnected strips containing the common point P of the two-way slab
subjected to uniformly distributed loads.

8.19.5 Design Considerations


The design considerations mentioned in sec. 8.18.5 of Lesson 18 in (a),
(c), (d), (e) and (f) are applicable for the two-way slabs also. However, the
effective span to effective depth ratio is different from those of one-way slabs.
Accordingly, this item for the two-way slabs is explained below.

Effective span to effective depth ratio (cl. 24.1 of IS 456)

The following are the relevant provisions given in Notes 1 and 2 of cl.
24.1.

• The shorter of the two spans should be used to determine the span to
effective depth ratio.

• For spans up to 3.5 m and with mild steel reinforcement, the span to
overall depth ratios satisfying the limits of vertical deflection for loads up to
3 kN/m2 are as follows:

Simply supported slabs 35

Continuous slabs 40

Version 2 CE IIT, Kharagpur


• The same ratios should be multiplied by 0.8 when high strength deformed
bars (Fe 415) are used in the slabs.

8.19.6 Design of Two-way Slabs


The procedure of the design of two-way slabs will have all the six steps
mentioned in sec. 8.18.6 for the design of one-way slabs except that the bending
moments and shear forces are determined by different methods for the two types
of slab.

While the bending moments and shear forces are computed from the
coefficients given in Tables 12 and 13 (cl. 22.5) of IS 456 for the one-way slabs,
the same are obtained from Tables 26 or 27 for the bending moment in the two
types of two-way slabs and the shear forces are computed from Eq.8.1 for the
two-way slabs.

Further, the restrained two-way slabs need adequate torsional reinforcing


bars at the corners to prevent them from lifting. There are three types of corners
having three different requirements. Accordingly, the determination of torsional
reinforcement is discussed in Step 7, as all the other six steps are common for
the one and two-way slabs.

Step 7: Determination of torsional reinforcement

Three types of corners, C1, C2 and C3, shown in Fig.8.19.4, have three
different requirements of torsion steel as mentioned below.

(a) At corner C1 where the slab is discontinuous on both sides, the


torsion reinforcement shall consist of top and bottom bars each with layers of bar
placed parallel to the sides of the slab and extending a minimum distance of one-
fifth of the shorter span from the edges. The amount of reinforcement in each of
the four layers shall be 75 per cent of the area required for the maximum mid-
span moment in the slab. This provision is given in cl. D-1.8 of IS 456.

Version 2 CE IIT, Kharagpur


(b) At corner C2 contained by edges over one of which is continuous, the
torsional reinforcement shall be half of the amount of (a) above. This provision is
given in cl. D-1.9 of IS 456.

(c) At corner C3 contained by edges over both of which the slab is


continuous, torsional reinforcing bars need not be provided, as stipulated in cl. D-
1.10 of IS 456.

8.19.7 Detailing of Reinforcement


As mentioned in sec. 8.19.6, Step 5 of sec. 8.18.6 explains the two
methods of determining the required areas of steel required for the maximum
positive and negative moments. The two methods are (i) employing Eq.3.23 as
given in Step 5 of sec. 8.18.6 or (ii) using tables and charts of SP-16. Thereafter,
Step 7 of sec. 8.19.6 explains the method of determining the areas steel for
corners of restrained slab depending on the type of corner. The detailing of
torsional reinforcing bars is explained in Step 7 of sec. 8.19.6. In the following,
the detailings of reinforcing bars for (i) restrained slabs and (ii) simply supported
slabs are discussed separately for the bars either for the maximum positive or
negative bending moments or to satisfy the requirement of minimum amount of
steel.

(i) Restrained slabs

The maximum positive and negative moments per unit width of the slab
calculated by employing Eqs.8.3 and 8.4 as explained in sec. 8.19.4.2(i) are
applicable only to the respective middle strips (Fig.8.19.2). There shall be no
redistribution of these moments. The reinforcing bars so calculated from the
maximum moments are to be placed satisfying the following stipulations of IS
456.

Version 2 CE IIT, Kharagpur


Version 2 CE IIT, Kharagpur
• Bottom tension reinforcement bars of mid-span in the middle strip shall
extent in the lower part of the slab to within 0.25l of a continuous edge, or
0.15l of a discontinuous edge (cl. D-1.4 of IS 456). Bars marked as B1,
B2, B5 and B6 in Figs.8.19.5 a and b are these bars.

• Top tension reinforcement bars over the continuous edges of middle strip
shall extend in the upper part of the slab for a distance of 0.15l from the
support, and at least fifty per cent of these bars shall extend a distance of
0.3l (cl. D-1.5 of IS 456). Bars marked as T2, T3, T5 and T6 in Figs.8.19.5
a and b are these bars.

• To resist the negative moment at a discontinuous edge depending on the


degree of fixity at the edge of the slab, top tension reinforcement bars
equal to fifty per cent of that provided at mid-span shall extend 0.1l into
the span (cl. D-1.6 of IS 456). Bars marked as T1 and T4 in Figs.8.19.5 a
and b are these bars.

Version 2 CE IIT, Kharagpur


• The edge strip of each panel shall have reinforcing bars parallel to that
edge satisfying the requirement of minimum amount as specified in sec.
8.18.15d of Lesson 18 (cl. 26.5.2.1 of IS 456) and the requirements for
torsion, explained in Step 7 of sec. 8.19.6 (cls. D-1.7 to D-1.10 of IS 456).
The bottom and top bars of the edge strips are explained below.

• Bottom bars B3 and B4 (Fig.8.19.5 a) are parallel to the edge along lx for
the edge strip for span ly, satisfying the requirement of minimum amount
of steel (cl. D-1.7 of IS 456).

• Bottom bars B7 and B8 (Fig.8.19.5 b) are parallel to the edge along ly for
the edge strip for span lx, satisfying the requirement of minimum amount
of steel (cl. D-1.7 of IS 456).

• Top bars T7 and T8 (Fig.8.19.5 a) are parallel to the edge along lx for the
edge strip for span ly, satisfying the requirement of minimum amount of
steel (cl. D-1.7 of IS 456).

• Top bars T9 and T10 (Fig.8.19.5 b) are parallel to the edge along ly for
the edge strip for span lx, satisfying the requirement of minimum amount
of steel (cl. D-1.7 of IS 456).

The detailing of torsion bars at corners C1 and C2 is explained in


Fig.8.19.7 of Problem 8.2 in sec. 8.19.8.

The above explanation reveals that there are eighteen bars altogether
comprising eight bottom bars (B1 to B8) and ten top bars (T1 to T10). Tables 8.4
and 8.5 present them separately for the bottom and top bars, respectively,
mentioning the respective zone of their placement (MS/LDES/ACES/BDES to
designate Middle Strip/Left Discontinuous Edge Strip/Adjacent Continuous Edge
Strip/Bottom Discontinuous Edge Strip), direction of the bars (along x or y), the
resisting moment for which they shall be determined or if to be provided on the
basis of minimum reinforcement clause number of IS 456 and Fig. No. For easy
understanding, plan views in (a) and (b) of Fig.8.19.5 show all the bars
separately along x and y directions, respectively. Two sections (1-1 and 2-2),
however, present the bars shown in the two plans. Torsional reinforcements are
not included in Tables 8.4 and 8.5 and Figs.8.19.5 a and b.

Table 8.4 Details of eight bottom bars

Sl.No. Bars Into Along Resisting Cl.No. of Fig.No.


Moment IS 456
1 B1, B2 MS x Max. + D-1.3,1.4 8.19.5a, c,
Mx d
2 B3 LDES x Min. D-1.7 8.19.5a, c
Steel

Version 2 CE IIT, Kharagpur


3 B4 ACES x Min. D-1.7 8.19.5a, c
Steel
4 B5, B6 MS y Max. + D-1.3,1.4 8.19.5b, c,
My d
5 B7 BDES y Min. D-1.7 8.19.5b, d
Steel
6 B8 ACES y Min. D-1.7 8.19.5b, d
Steel

Notes: (i) MS = Middle Strip


(ii) LDES = Left Discontinuous Edge Strip
(iii) ACES = Adjacent Continuous Edge Strip
(iv) BDES = Bottom Discontinuous Edge Strip

Table 8.5 Details of eight top bars

Sl.No. Bars Into Along Resisting Cl.No. of Fig.No.


Moment IS 456
1 T1 BDES x + 0.5 Mx D-1.6 8.19.5a,
d
2 T2, T3 ACES x - 0.5 Mx D-1.5 8.19.5a,
for each d
3 T4 LDES y + 0.5 My D-1.6 8.19.5b,
c
4 T5, T6 ACES y -0.5 My D-1.5 8.19.5b,
for each c
5 T7 LDES x Min. D-1.7 8.19.5a,
Steel c
6 T8 ACES x Min. D-1.7 8.19.5a,
Steel c
7 T9 LDES y Min. D-1.7 8.19.5b,
Steel d
8 T10 ACES y Min. D-1.7 8.19.5b,
Steel d

Notes: (i) MS = Middle Strip


(ii) LDES = Left Discontinuous Edge Strip
(iii) ACES = Adjacent Continuous Edge Strip
(iv) BDES = Bottom Discontinuous Edge Strip

Version 2 CE IIT, Kharagpur


(ii) Simply supported slabs

Figures 8.19.6 a, b and c present the detailing of reinforcing bars of simply


supported slabs not having adequate provision to resist torsion at corners and to
prevent corners from lifting. Clause D-2.1 stipulates that fifty per cent of the
tension reinforcement provided at mid-span should extend to the supports. The
remaining fifty per cent should extend to within 0.1lx or 0.1ly of the support, as
appropriate.

Version 2 CE IIT, Kharagpur


8.19.8 Numerical Problems
Problem 8.2

Design the slab panel 1 of Fig.8.19.7 subjected to factored live load of 8


kN/m2 in addition to its dead load using M 20 and Fe 415. The load of floor finish
is 1 kN/m2. The spans shown in figure are effective spans. The corners of the
slab are prevented from lifting.

Solution of Problem 8.2

Step 1: Selection of preliminary depth of slab

The span to depth ratio with Fe 415 is taken from cl. 24.1, Note 2 of IS
456 as 0.8 (35 + 40) / 2 = 30. This gives the minimum effective depth d =
4000/30 = 133.33 mm, say 135 mm. The total depth D is thus 160 mm.

Step 2: Design loads, bending moments and shear forces

Dead load of slab (1 m width) = 0.16(25) = 4.0 kN/m2

Version 2 CE IIT, Kharagpur


Dead load of floor finish (given) = 1.0 kN/m2

Factored dead load = 1.5(5) = 7.5 kN/m2

Factored live load (given) = 8.0 kN/m2

Total factored load = 15.5 kN/m2

The coefficients of bending moments and the bending moments Mx and


My per unit width (positive and negative) are determined as per cl. D-1.1 and
Table 26 of IS 456 for the case 4, “Two adjacent edges discontinuous” and
presented in Table 8.6. The ly / lx for this problem is 6/4 = 1.5.

Table 8.6 Maximum bending moments of Problem 8.2

For Short span Long span


αx Mx (kNm/m) αy My (kNm/m)
Negative moment at 0.075 18.6 0.047 11.66
continuous edge
Positive moment at 0.056 13.89 0.035 8.68
mid-span

Maximum shear force in either direction is determined from Eq.8.1


(Fig.8.19.1) as

Vu = w(lx/2) = 15.5 (4/2) = 31 kN/m

Step 3: Determination/checking of the effective depth and total depth of


slab

Using the higher value of the maximum bending moments in x and y


directions from Table 8.6, we get from Eq.3.25 of Lesson 5 (sec. 3.5.5):

Mu,lim = R,lim bd2

or d = [(18.6)(106)/{2.76(103)}] 1/2 = 82.09 mm,

where 2.76 N/mm2 is the value of R,lim taken from Table 3.3 of Lesson 5
(sec. 3.5.5). Since, this effective depth is less than 135 mm assumed in
Step 1, we retain d = 135 mm and D = 160 mm.

Version 2 CE IIT, Kharagpur


Step 4: Depth of slab for shear force

Table 19 of IS 456 gives the value of τ c = 0.28 N/mm2 when the lowest
percentage of steel is provided in the slab. However, this value needs to be
modified by multiplying with k of cl. 40.2.1.1 of IS 456. The value of k for the
total depth of slab as 160 mm is 1.28. So, the value of τ c is 1.28(0.28) =
0.3584 N/mm2.

Table 20 of IS 456 gives τ c max = 2.8 N/mm2. The computed shear stress
τ v = Vu/bd = 31/135 = 0.229 N/mm2.

Since, τ v < τ c < τ c max , the effective depth of the slab as 135 mm and
the total depth as 160 mm are safe.

Step 5: Determination of areas of steel

The respective areas of steel in middle and edge strips are to be


determined employing Eq.3.23 of Step 5 of sec. 8.18.6 of Lesson 18. However, in
Problem 8.1 of Lesson 18, it has been shown that the areas of steel computed
from Eq.3.23 and those obtained from the tables of SP-16 are in good
agreement. Accordingly, the areas of steel for this problem are computed from
the respective Tables 40 and 41 of SP-16 and presented in Table 8.7. Table 40
of SP-16 is for the effective depth of 150 mm, while Table 41 of SP-16 is for the
effective depth of 175 mm. The following results are, therefore, interpolated
values obtained from the two tables of SP-16.

Table 8.7 Reinforcing bars of Problem 8.2

Particulars Short span lx Long span ly


Table Mx Dia. & Table My Dia. &
No. (kNm/m) spacing No. (kNm/m) spacing
Top steel for 40,41 18.68 10 mm @ 40,41 8 mm @
negative > 18.6 200 mm c/c 12.314 200 mm
moment > 11.66 c/c
Bottom steel for 40,41 8 mm @ 40,41 9.20 8 mm @
positive moment 14.388 170 mm c/c > 8.68 250 mm
> 13.89 c/c

The minimum steel is determined from the stipulation of cl. 26.5.2.1 of IS


456 and is
As = (0.12/100)(1000)(160) = 192 mm2

Version 2 CE IIT, Kharagpur


and 8 mm bars @ 250 mm c/c (= 201 mm2) is acceptable. It is worth mentioning
that the areas of steel as shown in Table 8.7 are more than the minimum amount
of steel.

Step 6: Selection of diameters and spacings of reinforcing bars

The advantages of using the tables of SP-16 are that the obtained values
satisfy the requirements of diameters of bars and spacings. However, they are
checked as ready reference here. Needless to mention that this step may be
omitted in such a situation.

Maximum diameter allowed, as given in cl. 26.5.2.2 of IS 456, is 160/8 =


20 mm, which is more that the diameters used here.

The maximum spacing of main bars, as given in cl. 26.3.3(1) of IS 456, is


the lesser of 3(135) and 300 mm. This is also satisfied for all the bars.

The maximum spacing of minimum steel (distribution bars) is the lesser of


5(135) and 450 mm. This is also satisfied.

Version 2 CE IIT, Kharagpur


Figures 8.19.8 and 9 present the detailing of reinforcing bars.

Step 7: Determination of torsional reinforcement

Version 2 CE IIT, Kharagpur


Version 2 CE IIT, Kharagpur
Torsional reinforcing bars are determined for the three different types of
corners as explained in sec. 8.19.6 (Fig.8.19.4). The length of torsional strip is
4000/5 = 800 mm and the bars are to be provided in four layers. Each layer will
have 0.75 times the steel used for the maximum positive moment. The C1 type of
corners will have the full amount of torsional steel while C2 type of corners will
have half of the amount provided in C1 type. The C3 type of corners do not need
any torsional steel. The results are presented in Table 8.8 and Figs.8.19.10 a, b
and c.

Table 8.8 Torsional reinforcement bars of Problem 8.2

Type Dimensions along Bar diameter & No. of bars along Cl. no. of
x (mm) y (mm) spacing x y IS 456
C1 800 800 8 mm @ 5 5 D-1.8
200 mm c/c
C2 800 1600 8 mm @ 5 8 D-1.9
250 mm c/c
C2 1600 800 8 mm @ 8 5 D-1.9
250 mm c/c

8.19.9 Practice Questions and Problems with Answers


Q.1: How do you determine the shear force of a two-way slab subjected to
uniformly distributed loads?

Version 2 CE IIT, Kharagpur


A.1: See sec. 8.19.4.1.

Q.2: Name the two types of two-way slabs.

A.2: The two types of two-way slabs are: (i) restrained slabs and (ii) simply
supported slabs.

Q.3: What is the difference in the design of the two types of slabs of Q.2?

A.3: The restrained slabs are those whose corners are prevented from lifting
and accordingly, there are torsional reinforcing bars in the two types of
corners. The simply supported slabs do not have adequate provision to
resist torsion at corners and to prevent the corners from lifting. So,
torsional reinforcing bars are not provided in these slabs.

Q.4: State span to depth ratios of two-way slabs for different support
conditions to be considered for the control of deflection.

A.4: See sec. 8.19.5.

Q.5: Explain the provisions of torsional reinforcing bars in restrained type of two-
way slabs.

A.5: Step 7 of sec. 8.19.6.

Q.6:

Version 2 CE IIT, Kharagpur


Design a two-way simply supported slab of Fig.8.19.11, not having
adequate provision to resist torsion at corners and to prevent the corners
from lifting. The factored live load is 6 kN/m2 and the load of the floor finish
in 1 kN/m2. The spans shown in the figure are effective spans. Use M 20
and Fe 415. The width of the support is 300 mm.

A.6:

Step 1: Selection of preliminary depth of slab

As per cl.24.1, Note 2, the span to effective depth ratio = 0.8(35) = 28. The
minimum effective depth = d = 4200/28 = 150 mm and, therefore, D = 175 mm.

Step 2: Design loads, bending moments and shear forces

Factored dead load of the slab = 1.5(0.175)(25) = 6.5625 kN/m2

Factored load of floor finish = 1.5(1) = 1.5 kN/m2

Factored live loads = 6.0 kN/m2

Total factored loads = 14.0625 kN/m2

Version 2 CE IIT, Kharagpur


For this slab ly/lx = 5880/4200 = 1.4, Table 27 of IS 456 gives α x = 0.099
and α y = 0.051.

+ Mx = α x w lx2 = (0.099)(14.0625)(4.2)(4.2) = 24.558 kNm/m

+ My = α y w lx2 = (0.051)(14.0625)(4.2)(4.2) = 12.651 kNm/m

Vu = 0.5 w lx = 0.5(14.0625)(4.2) = 29.531 kN/m

Step 3: Determination/checking of the effective depth and total depth of


slab

d = {24.558(103)/2.76}0.5 = 94.328 mm < 150 mm

So, we keep d = 150 mm and D = 175 mm.

Step 4: Depth of slab for shear forces

Table 19 of IS 456 gives τ c = 0.28 N/mm2. Clause 40.2.1.1 of IS 456


gives k = 1.25 for D = 175 mm. So, τ c = (1.25)(0.28) = 0.35 N/mm2.

Table 20 of IS 456 gives τ c max = 2.8 N/mm2. For this problem τ v = Vu/bd
= 29.531/150 = 0.1968 N/mm2. Since τ v < τ c < τ c max , the depth is safe.

Step 5: Determination of areas of steel

The positive steel in the two directions and the minimum steel are
furnished below in Table 8.9. These are the results obtained from the use of
Table 41 of SP-16.

Table 8.9 Reinforcing bars of Problem Q.6

Particulars Short span lx Long span ly


SP Mx Dia. & SP My Dia. &
Table (kNm/m) spacing Table (kNm/m) spacing
No. No.
Positive steel 41 26.40 8 mm @ 41 12.93 6 mm @
> 24.558 100 mm > 12.651 120 mm
c/c (503 c/c (236
mm2) mm2)
Minimum 96 Min. 6 mm @ 96 Min. steel 6 mm @
steel steel 120 mm 120 mm
= 0.12(1750) c/c (236 c/c (236

Version 2 CE IIT, Kharagpur


= 210 mm2 mm2) > mm2) >
210 mm2 210 mm2

Figures 8.19.12a and b show the detailing of reinforcing bars.

8.19.10 References
1. Reinforced Concrete Limit State Design, 6th Edition, by Ashok K. Jain,
Nem Chand & Bros, Roorkee, 2002.
2. Limit State Design of Reinforced Concrete, 2nd Edition, by P.C.Varghese,
Prentice-Hall of India Pvt. Ltd., New Delhi, 2002.
3. Advanced Reinforced Concrete Design, by P.C.Varghese, Prentice-Hall of
India Pvt. Ltd., New Delhi, 2001.
4. Reinforced Concrete Design, 2nd Edition, by S.Unnikrishna Pillai and
Devdas Menon, Tata McGraw-Hill Publishing Company Limited, New
Delhi, 2003.
5. Limit State Design of Reinforced Concrete Structures, by P.Dayaratnam,
Oxford & I.B.H. Publishing Company Pvt. Ltd., New Delhi, 2004.
6. Reinforced Concrete Design, 1st Revised Edition, by S.N.Sinha, Tata
McGraw-Hill Publishing Company. New Delhi, 1990.

Version 2 CE IIT, Kharagpur


7. Reinforced Concrete, 6th Edition, by S.K.Mallick and A.P.Gupta, Oxford &
IBH Publishing Co. Pvt. Ltd. New Delhi, 1996.
8. Behaviour, Analysis & Design of Reinforced Concrete Structural Elements,
by I.C.Syal and R.K.Ummat, A.H.Wheeler & Co. Ltd., Allahabad, 1989.
9. Reinforced Concrete Structures, 3rd Edition, by I.C.Syal and A.K.Goel,
A.H.Wheeler & Co. Ltd., Allahabad, 1992.
10. Textbook of R.C.C, by G.S.Birdie and J.S.Birdie, Wiley Eastern Limited,
New Delhi, 1993.
11. Design of Concrete Structures, 13th Edition, by Arthur H. Nilson, David
Darwin and Charles W. Dolan, Tata McGraw-Hill Publishing Company
Limited, New Delhi, 2004.
12. Concrete Technology, by A.M.Neville and J.J.Brooks, ELBS with
Longman, 1994.
13. Properties of Concrete, 4th Edition, 1st Indian reprint, by A.M.Neville,
Longman, 2000.
14. Reinforced Concrete Designer’s Handbook, 10th Edition, by C.E.Reynolds
and J.C.Steedman, E & FN SPON, London, 1997.
15. Indian Standard Plain and Reinforced Concrete – Code of Practice (4th
Revision), IS 456: 2000, BIS, New Delhi.
16. Design Aids for Reinforced Concrete to IS: 456 – 1978, BIS, New Delhi.

8.19.11 Test 19 with Solutions


Maximum Marks = 50, Maximum Time = 30 minutes

Answer all questions.

TQ.1: Explain the provisions of torsional reinforcing bars in restrained type of


two-way slabs.
(20 marks)

A.TQ.1: Step 7 of sec. 8.19.6.

TQ.2: Design the interior panel (Panel 2) of Problem 8.2 (Fig.8.19.7). Other data
are the same as those of Problem 8.2.
(30 marks)

A.TQ.2: Let us keep the effective and total depths of the slab as 135 mm and
160 mm, respectively (see Problem 8.2). The total factored load = 15.5
kN/m2 (see Problem 8.2). The coefficients of bending moments and the
bending moments Mx and My (positive and negative) per unit width are
determined as per cl. D-1.1 and Table 26 of IS 456 for the case 1 (interior
panel) and presented in Table 8.10. The ly/lx for this problem is 1.5.

Version 2 CE IIT, Kharagpur


Table 8.10 Bending moments of Problem TQ.2

For Short span Long span


αx Mx (kNm/m) αy My (kNm/m)
Negative moment at 0.053 13.144 0.032 7.936
continuous edge
Positive moment at 0.041 10.168 0.024 5.952
mid-span

The maximum shear force in either direction is the same as that of


Problem 8.2 = 31 kN/m.

Since the bending moments are much less than those of Problem 8.2, the
effective depth of 135 m and total depth of 160 mm are safe.

In Step 4 of solution of Problem 8.2, this depth has been found to be safe
in shear. So, the depths 135 mm and 160 mm are safe.

Step 5: Determination of areas of steel

The areas of steel using the table of SP-16 are presented in Table 8.11.
The maximum diameter and spacing of bars are not needed to check separately
as the results obtained from tables of SP-16 already take into consideration
these aspects.

Table 8.11 Reinforcing bars of Problem TQ.2

Particulars Short span lx Long span ly


Table Mx Dia. & Table My Dia. &
No. (kNm/m) spacing No. (kNm/m) spacing
Top steel for 40,41 13.618 8T@ 40 9.2 8T@*
negative > 180 mm c/c > 7.936 250 mm
moment 13.144 c/c
Bottom steel for 40,41 8T@ 40 9.2 8T@*
positive moment 10.758 230 mm c/c > 5.952 250 mm
> c/c
10.168
* Note: The areas of bars are selected to satisfy the minimum amount of steel.

The minimum steel will be the same as that of Problem 8.2 i.e., 8 mm
diameter @ 250 mm c/c.

Since this is an internal panel, torsional reinforcing bars are not needed at
any of the four corners.

Version 2 CE IIT, Kharagpur


The detailing of the bars can be drawn following the same of Problem 8.2
shown in Figs.8.19.8 and 9.

8.19.12 Summary of this Lesson


This lesson explains the differences of the methods of computing shear
force and bending moments of the restrained and simply supported two-way
slabs. The design methods of the two types of slabs are explained avoiding the
common steps of the design of one-way slabs as explained in Lesson 18.
Separate diagrams for the detailing of reinforcing bars are presented to illustrate
them. Torsional bars at the two types of corners of restrained slabs are
illustrated. Numerical examples are solved to design and detail the
reinforcements of both types of two-way slabs. Solutions of practice problems
and test problems will help in understanding the complete design and detailing of
the two types of two-way slabs. It would be seen that detailing would need great
care and thoroughness.

Version 2 CE IIT, Kharagpur


Module
9
Staircases
Version 2 CE IIT, Kharagpur
Lesson
20
Types and Design of
Staircases
Version 2 CE IIT, Kharagpur
Instructional Objectives:

At the end of this lesson, the student should be able to:

• classify the different types of staircases based on geometrical


configurations,

• name and identify the different elements of a typical flight,

• state the general guidelines while planning a staircase,

• determine the dimensions of trade, riser, depth of slab etc. of a staircase,

• classify the different staircases based on structural systems,

• explain the distribution of loadings and determination of effective spans of


stairs,

• analyse different types of staircases including the free-standing staircases


in a simplified manner,

• design the different types of staircases as per the stipulations of IS 456.

9.20.1 Introduction
Staircase is an important component of a building providing access to
different floors and roof of the building. It consists of a flight of steps (stairs) and
one or more intermediate landing slabs between the floor levels. Different types
of staircases can be made by arranging stairs and landing slabs. Staircase, thus,
is a structure enclosing a stair. The design of the main components of a
staircase-stair, landing slabs and supporting beams or wall – are already covered
in earlier lessons. The design of staircase, therefore, is the application of the
designs of the different elements of the staircase.

Version 2 CE IIT, Kharagpur


9.20.2 Types of Staircases

Figures 9.20.1a to e present some of the common types of staircases


based on geometrical configurations:

(a) Single flight staircase (Fig. 9.20.1a)

Version 2 CE IIT, Kharagpur


(b) Two flight staircase (Fig. 9.20.1b)
(c) Open-well staircase (Fig. 9.20.1c)
(d) Spiral staircase (Fig. 9.20.1d)
(e) Helicoidal staircase (Fig. 9.20.1e)

Architectural considerations involving aesthetics, structural feasibility and


functional requirements are the major aspects to select a particular type of the
staircase. Other influencing parameters of the selection are lighting, ventilation,
comfort, accessibility, space etc.

9.20.3 A Typical Flight

Figures 9.20.2a to d present plans and sections of a typical flight of


different possibilities. The different terminologies used in the staircase are given
below:

Version 2 CE IIT, Kharagpur


(a) Tread: The horizontal top portion of a step where foot rests
(Fig.9.20.2b) is known as tread. The dimension ranges from 270 mm for
residential buildings and factories to 300 mm for public buildings where large
number of persons use the staircase.

(b) Nosing: In some cases the tread is projected outward to increase the
space. This projection is designated as nosing (Fig.9.20.2b).

(c) Riser: The vertical distance between two successive steps is termed
as riser (Fig.9.20.2b). The dimension of the riser ranges from 150 mm for public
buildings to 190 mm for residential buildings and factories.

(d) Waist: The thickness of the waist-slab on which steps are made is
known as waist (Fig.9.20.2b). The depth (thickness) of the waist is the minimum
thickness perpendicular to the soffit of the staircase (cl. 33.3 of IS 456). The
steps of the staircase resting on waist-slab can be made of bricks or concrete.

(e) Going: Going is the horizontal projection between the first and the last
riser of an inclined flight (Fig.9.20.2a).

The flight shown in Fig.9.20.2a has two landings and one going. Figures
9.2b to d present the three ways of arranging the flight as mentioned below:

(i) waist-slab type (Fig.9.20.2b),


(ii) tread-riser type (Fig.9.20.2c), or free-standing staircase, and
(iii) isolated tread type (Fig.9.20.2d).

9.20.4 General Guidelines


The following are some of the general guidelines to be considered while
planning a staircase:

• The respective dimensions of tread and riser for all the parallel steps
should be the same in consecutive floor of a building.

• The minimum vertical headroom above any step should be 2 m.

• Generally, the number of risers in a flight should be restricted to twelve.

• The minimum width of stair (Fig.9.20.2a) should be 850 mm, though it is


desirable to have the width between 1.1 to 1.6 m. In public building,
cinema halls etc., large widths of the stair should be provided.

Version 2 CE IIT, Kharagpur


9.20.5 Structural Systems

Different structural systems are possible for the staircase, shown in Fig.
9.20.3a, depending on the spanning direction. The slab component of the stair
spans either in the direction of going i.e., longitudinally or in the direction of the
steps, i.e., transversely. The systems are discussed below:

(A) Stair slab spanning longitudinally

Here, one or more supports are provided parallel to the riser for the slab
bending longitudinally. Figures 9.20.3b to f show different support arrangements
of a two flight stair of Fig.9.20.3a:

Version 2 CE IIT, Kharagpur


(i) Supported on edges AE and DH (Fig.9.20.3b)
(ii) Clamped along edges AE and DH (Fig.9.20.3c)
(iii) Supported on edges BF and CG (Fig.9.20.3d)
(iv) Supported on edges AE, CG (or BF) and DH (Fig.9.20.3e)
(v) Supported on edges AE, BF, CG and DH (Fig.9.20.3f)

Cantilevered landing and intermediate supports (Figs.9.20.3d, e and f) are


helpful to induce negative moments near the supports which reduce the positive
moment and thereby the depth of slab becomes economic.

In the case of two flight stair, sometimes the flight is supported between
the landings which span transversely (Figs.9.20.4a and b). It is worth mentioning
that some of the above mentioned structural systems are statically determinate
while others are statically indeterminate where deformation conditions have to
taken into account for the analysis.

Version 2 CE IIT, Kharagpur


Longitudinal spanning of stair slab is also possible with other
configurations including single flight, open-well helicoidal and free-standing
staircases.

(B) Stair slab spanning transversely

Here, either the waist slabs or the slab components of isolated tread-slab
and trade-riser units are supported on their sides or are cantilevers along the
width direction from a central beam. The slabs thus bend in a transverse vertical
plane. The following are the different arrangements:

(i) Slab supported between two stringer beams or walls (Fig.9.20.5a)


(ii) Cantilever slabs from a spandreal beam or wall (Fig.9.20.5b)
(iii) Doubly cantilever slabs from a central beam (Fig.9.20.5c)

Version 2 CE IIT, Kharagpur


9.20.6 Effective Span of Stairs
The stipulations of clause 33 of IS 456 are given below as a ready
reference regarding the determination of effective span of stair. Three different
cases are given to determine the effective span of stairs without stringer beams.

(i) The horizontal centre-to-centre distance of beams should be


considered as the effective span when the slab is supported at top and bottom
risers by beams spanning parallel with the risers.

(ii) The horizontal distance equal to the going of the stairs plus at each
end either half the width of the landing or one meter, whichever is smaller when
the stair slab is spanning on to the edge of a landing slab which spans parallel
with the risers. See Table 9.1 for the effective span for this type of staircases
shown in Fig.9.20.3a.

Table 9.1 Effective span of stairs shown in Fig.9.20.3a

Sl. No. x y Effective span in metres


1 <1m <1m G+x+y
2 <1m ≥ 1m G+x+1
3 ≥ 1m <1m G+y+1
4 ≥ 1m ≥ 1m G+1+1

Note: G = Going, as shown in Fig. 9.20.3a

9.20.7 Distribution of Loadings on Stairs

Version 2 CE IIT, Kharagpur


Figure 9.20.6 shows one open-well stair where spans partly cross at right
angle. The load in such stairs on areas common to any two such spans should
be taken as fifty per cent in each direction as shown in Fig.9.20.7. Moreover, one
150 mm strip may be deducted from the loaded area and the effective breadth of
the section is increased by 75 mm for the design where flights or landings are
embedded into walls for a length of at least 110 mm and are designed to span in
the direction of the flight (Fig.9.20.7).

9.20.8 Structural Analysis


Most of the structural systems of stair spanning longitudinally or
transversely are standard problems of structural analysis, either statically
determinate or indeterminate. Accordingly, they can be analysed by methods of
analysis suitable for a particular system. However, the rigorous analysis is
difficult and involved for a trade-riser type or free standing staircase where the
slab is repeatedly folded. This type of staircase has drawn special attraction due
to its aesthetic appeal and, therefore, simplified analysis for this type of staircase
spanning longitudinally is explained below. It is worth mentioning that certain
idealizations are made in the actual structures for the applicability of the
simplified analysis. The designs based on the simplified analysis have been
found to satisfy the practical needs.

Version 2 CE IIT, Kharagpur


Figure 9.20.8a shows the simply supported trade-riser staircase. The
uniformly distributed loads are assumed to act at the riser levels (Fig.9.20.8b).
The bending moment and shear force diagrams along the treads and the bending
moment diagram along the risers are shown in Figs.9.20.8c, d and e,
respectively. The free body diagrams of CD, DE and EF are shown in
Figs.9.20.8f, g and h, respectively. It is seen that the trade slabs are subjected to
varying bending moments and constant shear force (Fig.9.20.8f). On the other
hand the riser slabs are subjected to a constant bending moment and axial force
Version 2 CE IIT, Kharagpur
(either compressive or tensile). The assumption is that the riser and trade slabs
are rigidly connected. It has been observed that both trade and riser slabs may
be designed for bending moment alone as the shear stresses in trade slabs and
axial forces in riser slabs are comparatively low. The slab thickness of the trade
and risers should be kept the same and equal to span/25 for simply supported
and span/30 for continuous stairs.

Figure 9.20.9a shows an indeterminate trade-riser staircase. Here, the


analysis can be done by adding the effect of the support moment MA
(Fig.9.20.9b) with the results of earlier simply supported case. However, the
value of MA can be determined using the moment-area method. The free body
diagrams of two vertical risers BC and DE are show in Figs.9.20.9c and d,
respectively.

9.20.9 Illustrative Examples


Two typical examples of waist-slab and trade-riser types spanning
longitudinally are taken up here to illustrate the design.

Version 2 CE IIT, Kharagpur


Example 9.1:

Design the waist-slab type of the staircase of Fig.9.20.10. Landing slab A


is supported on beams along JK and PQ, while the waist-slab and landing slab B
are spanning longitudinally as shown in Fig.9.20.10. The finish loads and live
loads are 1 kN/m2 and 5 kN/m2, respectively. Use riser R = 160 mm, trade T =
270 mm, concrete grade = M 20 and steel grade = Fe 415.

Solution:

With R = 160 mm and T = 270 mm, the inclined length of each step =
{(160)2 + (270)2}½ = 313.85 mm.

(A) Design of going and landing slab B

Step 1: Effective span and depth of slab

Version 2 CE IIT, Kharagpur


The effective span (cls. 33.1b and c) = 750 + 2700 + 1500 + 150 = 5100
mm. The depth of waist slab = 5100/20 = 255 mm. Let us assume total depth of
250 mm and effective depth = 250 – 20 – 6 = 224 mm (assuming cover = 20 mm
and diameter of main reinforcing bar = 12 mm). The depth of landing slab is
assumed as 200 mm and effective depth = 200 – 20 – 6 = 174 mm.

Step 2: Calculation of loads (Fig.9.20.11, sec. 1-1)

(i) Loads on going (on projected plan area)

(a) Self-weight of waist-slab = 25(0.25)(313.85)/270 = 7.265 kN/m2

(b) Self-weight of steps = 25(0.5)(0.16) = 2.0 kN/m2

(c) Finishes (given) = 1.0 kN/m2

(d) Live loads (given) = 5.0 kN/m2

Total = 15.265 kN/m2

Total factored loads = 1.5(15.265) = 22.9 kN/m2

(ii) Loads on landing slab A (50% of estimated loads)

(a) Self-weight of landing slab = 25(0.2) = 5 kN/m2

(b) Finishes (given) = 1 kN/m2

(c) Live loads (given) = 5 kN/m2

Total = 11 kN/m2

Factored loads on landing slab A = 0.5(1.5)(11) = 8.25 kN/m2

(iii) Factored loads on landing slab B = (1.5)(11) = 16.5 kN/m2

The loads are drawn in Fig.9.20.11.

Step 3: Bending moment and shear force (Fig. 9.20.11)

Total loads for 1.5 m width of flight = 1.5{8.25(0.75) + 22.9(2.7) +


16.5(1.65)}

= 142.86 kN

Version 2 CE IIT, Kharagpur


VC = 1.5{8.25(0.75)(5.1 – 0.375) + 22.9(2.7)(5.1 – 0.75 – 1.35)

+ 16.5(1.65)(1.65)(0.5)}/5.1 = 69.76 kN

VD = 142.86 – 69.76 = 73.1 kN

The distance x from the left where shear force is zero is obtained from:

x = {69.76 – 1.5(8.25)(0.75) + 1.5(22.9)(0.75)}/(1.5)(22.9) = 2.51 m

The maximum bending moment at x = 2.51 m is

= 69.76(2.51) – (1.5)(8.25)(0.75)(2.51 – 0.375)

- (1.5)(22.9)(2.51 – 0.75)(2.51 – 0.75)(0.5) = 102.08 kNm.

For the landing slab B, the bending moment at a distance of 1.65 m from
D

= 73.1(1.65) – 1.5(16.5)(1.65)(1.65)(0.5) = 86.92 kNm

Step 4: Checking of depth of slab

From the maximum moment, we get d = {102080/2(2.76)}½ = 135.98


mm < 224 mm for waist-slab and < 174 mm for landing slabs. Hence, both the
depths of 250 mm and 200 mm for waist-slab and landing slab are more than
adequate for bending.

For the waist-slab, τ v = 73100/1500(224) = 0.217 N/mm2. For the waist-


slab of depth 250 mm, k = 1.1 (cl. 40.2.1.1 of IS 456) and from Table 19 of IS
456, τ c = 1.1(0.28) = 0.308 N/mm2. Table 20 of IS 456, τ c max = 2.8 N/mm2.
Since τ v < τ c < τ c max , the depth of waist-slab as 250 mm is safe for shear.

For the landing slab, τ v = 73100/1500(174) = 0.28 N/mm2. For the


landing slab of depth 200 mm, k = 1.2 (cl. 40.2.1.1 of IS 456) and from Table 19
of IS 456, τ c = 1.2(0.28) = 0.336 N/mm2 and from Table 20 of IS 456, τ c max = 2.8
N/mm2. Here also τ v < τ c < τ c max , so the depth of landing slab as 200 mm is
safe for shear.

Step 5: Determination of areas of steel reinforcement

Version 2 CE IIT, Kharagpur


(i) Waist-slab: Mu/bd2 = 102080/(1.5)224(224) = 1.356 N/mm2. Table 2 of SP-
16 gives p = 0.411.

The area of steel = 0.411(1000)(224)/(100) = 920.64 mm2. Provide 12


mm diameter @ 120 mm c/c (= 942 mm2/m).

(ii) Landing slab B: Mu/bd2 at a distance of 1.65 m from VD (Fig. 9.20.11) =


86920/(1.5)(174)(174) = 1.91 N/mm2. Table 2 of SP-16 gives: p = 0.606. The
area of steel = 0.606(1000) (174)/100 = 1054 mm2/m. Provide 16 mm diameter
@ 240 mm c/c and 12 mm dia. @ 240 mm c/c (1309 mm2) at the bottom of
landing slab B of which 16 mm bars will be terminated at a distance of 500 mm
from the end and will continue up to a distance of 1000 mm at the bottom of waist
slab (Fig. 9.20.12).

Distribution steel: The same distribution steel is provided for both the slabs as
calculated for the waist-slab. The amount is = 0.12(250) (1000)/100 = 300
mm2/m. Provide 8 mm diameter @ 160 mm c/c (= 314 mm2/m).

Step 6: Checking of development length and diameter of main bars

Development length of 12 mm diameter bars = 47(12) = 564 mm, say


600 mm and the same of 16 mm dia. Bars = 47(16) = 752 mm, say 800 mm.

(i) For waist-slab

Version 2 CE IIT, Kharagpur


M1 for 12 mm diameter @ 120 mm c/c (= 942 mm2) = 942(102.08)/920.64
= 104.44 kNm. With V (shear force) = 73.1 kN, the diameter of main bars ≤
{1.3(104440)/73.1}/47 ≤ 39.5 mm. Hence, 12 mm diameter is o.k.

(ii) For landing-slab B

M1 for 16 mm diameter @ 120 mm c/c (= 1675 mm2) =


1675(102.08)/1650.88 = 103.57 kNm. With V (shear force) = 73.1 kN, the
diameter of main bars ≤ {1.3(103570)/73.1}/47 = 39.18 mm. Hence, 16 mm
diameter is o.k.

The reinforcing bars are shown in Fig.9.20.12 (sec. 1-1).

(B) Design of landing slab A

Step 1: Effective span and depth of slab

The effective span is lesser of (i) (1500 + 1500 + 150 + 174), and (ii) (1500
+ 1500 + 150 + 300) = 3324 mm. The depth of landing slab = 3324/20 = 166 mm,
< 200 mm already assumed. So, the depth is 200 mm.

Step 2: Calculation of loads (Fig.9.20.13)

The following are the loads:

(i) Factored load on landing slab A(see Step 2 of A @ 50%) = 8.25 kN/m2

(ii) Factored reaction VC (see Step 3 of A) = 69.76 kN as the total load of


one flight

(iii) Factored reaction VC from the other flight = 69.76 kN

Thus, the total load on landing slab A

= (8.25)(1.5)(3.324) + 69.76 + 69.76 = 180.65 kN

Version 2 CE IIT, Kharagpur


Due to symmetry of loadings, VE = VF = 90.33 kN. The bending moment is
maximum at the centre line of EF.

Step 3: Bending moment and shear force (width = 1500 mm)

Maximum bending moment = (180.65)(3.324)/8 = 75.06 kNm

Maximum shear force = 0.5(180.65) = 90.33 kN

Step 4: Checking of depth of slab

In Step 3 of A, it has been observed that 135.98 mm is the required depth


for bending moment = 102.08 kNm. So, the depth of 200 mm is safe for this
bending moment of 75.06 kNm. However, a check is needed for shear force.

τ v = 90330/1500(174) = 0.347 N/mm2 > 0.336 N/mm2

The above value of τ c = 0.336 N/mm2 for landing slab of depth 200 mm has
been obtained in Step 4 of A. However, here τ c is for the minimum tensile steel
in the slab. The checking of depth for shear shall be done after determining the
area of tensile steel as the value of τ v is marginally higher.

Step 5: Determination of areas of steel reinforcement

For Mu/bd2 = 75060/(1.5)(174)(174) = 1.65 N/mm2, Table 2 of SP-16


gives p = 0.512.

The area of steel = (0.512)(1000)(174)/100 = 890.88 mm2/m. Provide


12 mm diameter @ 120 mm c/c (= 942 mm2/m). With this area of steel p =
942(100)/1000(174) = 0.541.

Version 2 CE IIT, Kharagpur


Distribution steel = The same as in Step 5 of A i.e., 8 mm diameter @ 160
mm c/c.

Step 6: Checking of depth for shear

Table 19 and cl. 40.2.1.1 gives: τ c = (1.2)(0.493) = 0.5916 N/mm2. τ v =


0.347 N/mm2 (see Step 3 of B) is now less than τ c (= 0.5916 N/mm2). Since, τ v
< τ c < τ c max , the depth of 200 mm is safe for shear.

The reinforcing bars are shown in Fig. 9.20.14.

Example 9.2:

Design a trade-riser staircase shown in Fig.9.20.15 spanning


longitudinally. Landing slabs are supported on beams spanning transversely. The
dimensions of riser and trade are 160 mm and 270 mm, respectively. The finish
loads and live loads are 1 kN/m2 and 5 kN/m2, respectively. Use M 20 and Fe
415.

Solution:

The distribution of loads on landings common to two spans perpendicular


to each other shall be done as per cl. 33.2 of IS 456 (50% in each direction),
since the going is supported on landing slabs which span transversely. The
effective span in the longitudinal direction shall be taken as the distance between
two centre lines of landings.

(A) Design of going

Version 2 CE IIT, Kharagpur


Step 1: Effective span and depth of slab

Figure 9.20.16 shows the arrangement of the landings and going. The
effective span is 4200 mm. Assume the thickness of trade-riser slab = 4200/25 =
168 mm, say 200 mm. The thickness of landing slab is also assumed as 200
mm.

Step 2: Calculation of loads (Fig. 9.20.17)

Version 2 CE IIT, Kharagpur


The total loads including self-weight, finish and live loads on projected
area of going (1500 mm x 2465 mm) is first determined to estimate the total
factored loads per metre run.

(i) Self-weight of going

(a) Nine units of (0.2)(0.36)(1.5) @ 25(9) = 24.3 kN

(b) One unit of (0.27)(0.36)(1.5) @ 25(1) = 3.645 kN

(c) Nine units of (0.07)(0.2)(1.5) @ 25(9) = 4.725 kN

(ii) Finish loads @ 1 kN/m2 = (1.5)(2.465)(1) = 3.6975 kN

(iii) Live loads @ 5 kN/m2 = (1.5)(2.465)(5) = 18.4875 kN

Total = 54.855 kN

Factored loads per metre run = 1.5(54.855)/2.465 = 33.38 kN/m

(iv) Self-weight of landing slabs per metre run = 1.5(0.2)(25) = 7.5 kN/m

(v) Live loads on landings = (1.5)(5) = 7.5 kN/m

(vi) Finish loads on landings = (1.5)(1) = 1.5 kN/m

Total = 16.5 kN/m

Factored loads = 1.5(16.5) = 24.75 kN/m

Due to common area of landings only 50 per cent of this load should be
considered. So, the loads = 12.375 kN/m. The loads are shown in Fig.9.20.17.

Step 3: Bending moment and shear force

Total factored loads = 33.38(2.465) + 12.375(0.85 + 0.885) = 103.75 kN

VC = {12.375(0.85)(4.2 – 0.425) + 33.38(2.465)(0.885 + 1.2325)

+ 12.375(0.885)(0.885)(0.5)}/4.2 = 52.09 kN

VD = 103.75 – 52.09 = 51.66 kN

The distance x from the left support where shear force is zero is now
determined:

Version 2 CE IIT, Kharagpur


52.09 – 12.375(0.85) – 32.38(x – 0.85) = 0

or x = {52.09 – 12.375(0.85) + 33.38(0.85)}/33.38 = 2.095 m

Maximum factored bending moment at x = 2.095 m is

52.09(2.095) – 12.375(0.85)(2.095 – 0.425) – 33.38(2.095 – 0.85)(2.095 –


0.85)(0.5)

= 65.69 kNm

Step 4: Checking of depth of slab

From the maximum bending moment, we have

d = {(65690)/(1.5)(2.76)}½ = 125.97 mm < 174 mm

From the shear force Vu = VA, we get τ v = 52090/(1500)(174) = 0.199


N/mm2. From cl. 40.2.1.1 and Table 19 of IS 456, we have τ c = (1.2)(0.28) =
0.336 N/mm2. Table 20 of IS 456 gives τ c max = 2.8 N/mm2. Since, τ v < τ c < τ c max ,
the depth of 200 mm is accepted.

Step 5: Determination of areas of steel reinforcement

Mu/bd2 = 65.69(106)/(1500)(174)(174) = 1.446 N/mm2

Version 2 CE IIT, Kharagpur


Table 2 of SP-16 gives, p = 0.4416, to have Ast = 0.4416(1000)(174)/100 =
768.384 mm2/m. Provide 12 mm diameter bars @ 140 mm c/c (= 808 mm2) in
form of closed ties (Fig.9.20.18).

Distribution bars: Area of distribution bars = 0.12(1000)(200)/100 = 240


2
mm /m.

Provide 8 mm diameter bars @ 200 mm c/c. The reinforcing bars are shown in
Fig. 9.20.18.

(B) Design of landing slab

Step 1: Effective span and depth of slab

With total depth D = 200 mm and effective depth d = 174 mm, the
effective span (cl. 22.2a) = lesser of (1500 + 150 + 1500 + 174) and (1500 + 150
+ 1500 + 300) = 3324 mm.

Step 2: Calculation of loads (Fig.9.20.19)

(i) Factored load of landing slab A = 50% of Step 2 (iv to vi) @ 12.375 kN/m =
12.375(3.324 = 41.1345 kN

(ii) Factored reaction VC from one flight (see Step 3) = 52.09 kN

(iii) Factored reaction VC from other flight = 52.09 kN

Total factored load = 145.32 kN. Due to symmetry of loads, VG = VH =


72.66 kN. The bending moment is maximum at the centre line of GH.

Step 3: Bending moment and shear force (width b = 1500 mm)

Maximum bending moment = 145.32(3.324)/8 = 60.38 kNm

Version 2 CE IIT, Kharagpur


Maximum shear force VG = VH = 145.32/2 = 72.66 kN

Step 4: Checking of depth of slab

From bending moment: d = {60380/(1.5)(2.76)}½ = 120.77 mm < 174


mm. Hence o.k.

From shear force: τ v = 72660/(1500)(174) = 0.278 N/mm2

From Step 4 of A: τ c = 0.336 N/mm2, τ c max = 2.8 N/mm2. Hence, the


depth is o.k. for shear also.

Step 5: Determination of areas of steel reinforcement

Mu/bd2 = 60380/(1.5)(174)(174) = 1.33 N/mm2

Table 2 of SP-16 gives, p = 0.4022. So, Ast = 0.4022(1000)(174)/100 = 699.828


mm2. Provide 12 mm diameter bars @ 160 mm c/c (= 707 mm2).

Distribution steel area = (0.12/100)(1000)(200) = 240 mm2

Provide 8 mm diameter @ 200 mm c/c (= 250 mm2).

Step 6: Checking of development length

The moment M1 for 12 mm @ 160 mm c/c (707 mm2) =


(707/699.828)60.38 = 60.998 kNm.

Version 2 CE IIT, Kharagpur


The shear force V = 72.66 kN. The diameter of the bar should be
less/equal to {(1.3)(60.998)(106)/72.66(103)}/47 = 23.2 mm. Hence 12 mm
diameter bars are o.k.

Use Ld = 47(12) = 564 mm, = 600 mm (say). The reinforcing bars are
shown in Fig. 9.20.20.

9.20.10 Practice Questions and Problems with Answers


Q.1: Name five types of staircases based on geometrical configurations.

A.1: See sec. 9.20.2.

Q.2: Draw a typical flight and show: (a) trade, (b) nosing, (c) riser, (d) waist and
(e) going.

A.2: Figures 9.20.2a, b and c (sec. 9.20.3)

Q.3: Mention four general considerations for the design of a staircase.

A.3: See sec. 9.20.4.

Q.4: Draw schematic diagrams of different types of staircases based on different


structural systems.

A.4: Figures 9.20.3a, b, c, d, e; Figs.9.20.4a and b; Figs.9.20.5a, b and c


(sec.9.20.5).

Q.5: Explain the method of determining the effective spans of stairs.

A.5: See sec. 9.20.6 (Table 9.1 also).

Q.6: Explain the distribution of loadings of open-well stairs and for those where
the landings are embedded in walls.

A.6: See sec. 9.20.7.

Q.7: Illustrate the simplified analysis of longitudinally spanning free-standing


staircases.

A.7: See sec. 9.20.8.

Version 2 CE IIT, Kharagpur


Q.8: Design the open-well staircase of Fig.9.20.21. The dimensions of risers and
trades are 160 mm and 270 mm, respectively. The finish loads and live
loads are 1 kN/m2 and 5 kN/m2, respectively. Landing A has a beam at the
edge while other landings (B and C) have brick walls. Use concrete of grade
M 20 and steel of grade Fe 415.

A.8:

Solution:

In this case landing slab A is spanning longitudinally along sec. 11 of


Fig.9.20.21. Landing slab B is common to spans of sec. 11 and sec. 22, crossing
at right angles. Distribution of loads on landing slab B shall be made 50 per cent
in each direction (cl. 33.2 of IS 456). The effective span for sec. 11 shall be from
the centre line of edge beam to centre line of brick wall, while the effective span
for sec. 22 shall be from the centre line of landing slab B to centre line of landing
slab C (cl. 33.1b of IS 456).

(A) Design of landing slab A and going (sec. 11 of Fig.9.20.21)

Step 1: Effective span and depth of slab

Version 2 CE IIT, Kharagpur


The effective span = 150 + 2000 + 1960 + 1000 = 5110 mm. The depth of
waist slab is assumed as 5110/20 = 255.5 mm, say 250 mm. The effective depth
= 250 – 20 – 6 = 224 mm. The landing slab is also assumed to have a total depth
of 250 mm and effective depth of 224 mm.

Step 2: Calculation of loads (Fig.9.20.22)

(i) Loads on going (on projected plan area)

(a) Self weight of waist slab = 25(0.25)(313.85/270) = 7.265 kN/m2

(b) Self weight of steps = 25(0.5)(0.16) = 2.0 kN/m2

(c) Finish loads (given) = 1.0 kN/m2

(d) Live loads (given) = 5.0 kN/m2

Total = 15.265 kN/m2

So, the factored loads = 1.5(15.265) = 22.9 kN/m2

(ii) Landing slab A

(a) Self weight of slab = 25(0.25) = 6.25 kN/m2

(b) Finish loads = 1.00 kN/m2

(c) Live loads = 5.00 kN/m2

Total = 12.25 kN/m2

Factored loads = 1.5(12.25) = 18.375 kN/m2

(iii) Landing slab B = 50 per cent of loads of landing slab A = 9.187 kN/m2

Version 2 CE IIT, Kharagpur


The total loads of (i), (ii) and (iii) are shown in Fig.9.22.

Total loads (i) going = 22.9(1.96)(2) = 89.768 kN

Total loads (ii) landing slab A = 18.375(2.15)(2) = 79.013 kN

Total loads (iii) landing slab B = 9.187(1.0)(2) = 18.374 kN

Total loads = 187.155 kN

The loads are shown in Fig. 9.20.22.

Step 3: Bending moment and shear force (width = 2.0 m, Fig. 9.20.22)

VP = {79.013(5.11 – 1.075) + 89.768(5.11 – 3.13) + 18.374(0.5)}/5.11

= 98.97 kN

VJ = 187.155 – 98.97 = 88.185 kN

The distance x where the shear force is zero is obtained from:

98.97 – 79.013 – 22.9(2)(x – 2.15) = 0

or x = 2.15 + (98.97 – 79.013)/22.9(2) = 2.586 m

Maximum bending moment at x = 2.586 m (width = 2 m)

= 98.97(2.586) – 79.013 – (22.9)(2)(0.436)(0.436)(0.5) = 161.013 kNm

Maximum shear force = 98.97 kN

Step 4: Checking of depth

From the maximum moment d = {161.013(103)/2(2.76)}½ = 170.8 mm <


224 mm. Hence o.k.

From the maximum shear force, τ v = 98970/2000(224) = 0.221 N/mm2.


For the depth of slab as 250 mm, k = 1.1(cl. 40.2.1.1 of IS 456) and τ c =
1.1(0.28) = 0.308 N/mm2 (Table 19 of IS 456). τ c max = 2.8 N/mm2 (Table 20 of IS
456). Since, τ v < τ c < τ c max , the depth of slab as 250 mm is safe.

Step 5: Determination of areas of steel reinforcement

Version 2 CE IIT, Kharagpur


Mu/bd2 = 161.013(103)/2(224)(224) = 1.60 N/mm2. Table 2 of SP-16
gives p = 0.494, to have Ast = 0.494(1000)(224)/100 = 1106.56 mm2/m. Provide
12 mm diameter bars @ 100 mm c/c (= 1131 mm2/m) both for landings and waist
slab.

Distribution reinforcement = 0.12(1000)(250)/100 = 300 mm2/m. Provide


8 mm diameter @ 160 mm c/c (= 314 mm2).

Step 6: Checking of development length

Development length of 12 mm diameter bars 7(12) = 564 mm. Provide


Ld = 600 mm.

For the slabs M1 for 12 mm diameter @ 100 mm c/c =


(1131)(161.013)/1106.56 = 164.57 kNm. Shear force = 98.97 kN. Hence, 47 φ ≤
1.3(164.57)/98.97 ≤ 2161.67 mm or the diameter of main bar φ ≤ 45.99 mm.
Hence, 12 mm diameter is o.k. The reinforcing bars are shown in Fig.9.20.23.

(B) Design of landing slabs B and C and going (sec. 22 of Fig.9.20.21)

Step 1: Effective span and depth of slab


The effective span from the centre line of landing slab B to the centre line
of landing slab C = 1000 + 1960 + 1000 = 3960 mm. The depths of waist slab
and landing slabs are maintained as 250 mm like those of sec. 11.

Version 2 CE IIT, Kharagpur


Step 2: Calculation of loads (Fig.9.20.24)

(i) Loads on going (Step 2(i) of A) = 22.9 kN/m2

(ii) Loads on landing slab B (Step 2(iii)) = 9.187 kN/m2

(iii) Loads on landing slab C (Step 2(iii)) = 9.187 kN/m2

Total factored loads are:

(i) Going = 22.9(1.96)(2) = 89.768 kN

(ii) Landing slab A = 9.187(1.0)(2) = 18.374 kN

(iii) Landing slab B = 9.187(1.0)(2) = 18.374 kN

Total = 126.506 kN

The loads are shown in Fig.9.20.24.

Step 3: Bending moment and shear force (width = 2.0 m, Fig.9.20.24)

The total load is 126.506 kN and symmetrically placed to give VG = VH =


63.253 kN. The maximum bending moment at x = 1.98 m (centre line of the span
3.96 m = 63.253(1.98) – 18.374(1.98 – 0.5) – 22.9(2)(0.98)(0.98)(0.5) = 76.05
kNm. Maximum shear force = 63.253 kN.

Since the maximum bending moment and shear force are less than those
of the other section (maximum moment = 161.013 kNm and maximum shear
force = 98.97 kN), the depth of 250 mm here is o.k. Accordingly, the amount of
reinforcing bars are determined.

Step 4: Determination of areas of steel reinforcement

Version 2 CE IIT, Kharagpur


Mu/bd2 = 76.05(103)/2(224)(224) = 0.76 N/mm2. Table 2 of SP-16 gives
p = 0.221. The area of steel = (0.221)(1000)(224)/100 = 495.04 mm2. Providing
12 mm diameter @ 220 mm c/c gives 514 mm2, however let us provide 12 mm
diameter @ 200 mm c/c (565 mm2) as it is easy to detail with 12 mm diameter @
100 mm c/c for the other section. Distribution bars are same as for sec. 11 i.e., 8
mm diameter @ 160 mm c/c.

Step 5: Checking of development length

For the slab reinforcement 12 mm dia. @ 200 mm c/c, M1 =


(565)(76.05)/495.04 = 86.80 kNm, V = 63.25 kN. So, the diameter of main bar φ
≤ {(1.3)(86.80)(103)/(63.25)}/47, i.e., ≤ 37.96 mm. Hence, 12 mm diameter bars
are o.k. Distribution steel shall remain the same as in sec. 11, i.e., 8 mm
diameter @ 160 mm c/c.

The reinforcing bars are shown in Fig.9.20.25. Figures 9.20.23 and


9.20.25 show the reinforcing bars considered separately. However, it is worth
mentioning that the common areas (landing B and C) will have the bars of larger
areas of either section eliminating the lower bars of other section.

9.20.11 References
1. Reinforced Concrete Limit State Design, 6th Edition, by Ashok K. Jain,
Nem Chand & Bros, Roorkee, 2002.
Version 2 CE IIT, Kharagpur
2. Limit State Design of Reinforced Concrete, 2nd Edition, by P.C.Varghese,
Prentice-Hall of India Pvt. Ltd., New Delhi, 2002.
3. Advanced Reinforced Concrete Design, by P.C.Varghese, Prentice-Hall of
India Pvt. Ltd., New Delhi, 2001.
4. Reinforced Concrete Design, 2nd Edition, by S.Unnikrishna Pillai and
Devdas Menon, Tata McGraw-Hill Publishing Company Limited, New
Delhi, 2003.
5. Limit State Design of Reinforced Concrete Structures, by P.Dayaratnam,
Oxford & I.B.H. Publishing Company Pvt. Ltd., New Delhi, 2004.
6. Reinforced Concrete Design, 1st Revised Edition, by S.N.Sinha, Tata
McGraw-Hill Publishing Company. New Delhi, 1990.
7. Reinforced Concrete, 6th Edition, by S.K.Mallick and A.P.Gupta, Oxford &
IBH Publishing Co. Pvt. Ltd. New Delhi, 1996.
8. Behaviour, Analysis & Design of Reinforced Concrete Structural Elements,
by I.C.Syal and R.K.Ummat, A.H.Wheeler & Co. Ltd., Allahabad, 1989.
9. Reinforced Concrete Structures, 3rd Edition, by I.C.Syal and A.K.Goel,
A.H.Wheeler & Co. Ltd., Allahabad, 1992.
10. Textbook of R.C.C, by G.S.Birdie and J.S.Birdie, Wiley Eastern Limited,
New Delhi, 1993.
11. Design of Concrete Structures, 13th Edition, by Arthur H. Nilson, David
Darwin and Charles W. Dolan, Tata McGraw-Hill Publishing Company
Limited, New Delhi, 2004.
12. Concrete Technology, by A.M.Neville and J.J.Brooks, ELBS with
Longman, 1994.
13. Properties of Concrete, 4th Edition, 1st Indian reprint, by A.M.Neville,
Longman, 2000.
14. Reinforced Concrete Designer’s Handbook, 10th Edition, by C.E.Reynolds
and J.C.Steedman, E & FN SPON, London, 1997.
15. Indian Standard Plain and Reinforced Concrete – Code of Practice (4th
Revision), IS 456: 2000, BIS, New Delhi.
16. Design Aids for Reinforced Concrete to IS: 456 – 1978, BIS, New Delhi.

9.20.12 Test 20 with Solutions


Maximum Marks = 50, Maximum Time = 1 hour

Answer all questions.

TQ.1: Draw a typical flight and show: (a) trade, (b) nosing, (c) riser, (d) waist and
(e) going.
(5 marks)

A.TQ.1: Figures 9.20.2a, b and c (sec. 9.20.3)

Version 2 CE IIT, Kharagpur


TQ.2: Draw schematic diagrams of different types of staircases based on
different structural systems.
(10 marks)

A.TQ.2: Figures 9.20.3a, b, c, d, e; Figs.9.20.4a and b; Figs.9.20.5a, b and c


(sec.9.20.5).

TQ.3: Illustrate the simplified analysis of longitudinally spanning free-standing


staircases.
(10 marks)

A.TQ.3: See sec. 9.20.8.

TQ.4: Design the staircase of illustrative example 9.1 of Fig.9.20.10 if supported


on beams along KQ and LR only making both the landings A and B as
cantilevers. Use the finish loads = 1 kN/m2, live loads = 5 kn/m2, riser R
= 160 mm, trade T = 270 mm, grade of concrete = M 20 and grade of
steel = Fe 415.
(25 marks)
A.TQ.4:

Version 2 CE IIT, Kharagpur


Solution:

The general arrangement is shown in Fig.9.20.26. With R = 160 mm and


T = 270 mm, the inclined length of each step = {(160)2 + (270)2}½ = 313.85 mm.
The structural arrangement is that the going is supported from beams along KQ
and LR and the landings A and B are cantilevers.

Step 1: Effective span and depth of slab

As per cl. 33.1a of IS 456, the effective span of going = 3000 mm and as
per cl. 22.2c of IS 456, the effective length of cantilever landing slabs = 1350
mm. The depth of waist slab and landing is kept at 200 mm (greater of 3000/20
and 1350/7). The effective depth = 200 – 20 – 6 = 174 mm.

Step 2: Calculation of loads

(i) Loads on going (on projected plan area)

(a) Self weight of waist-slab = 25(0.20)(313.85)/270 = 6.812 kN/m2

(b) Self weight of steps = 25(0.5)(0.16) = 2.0 kN/m2

(c) Finishes (given) = 1.0 kN/m2

(d) Live loads (given) = 5.0 kN/m2

Total: 14.812 kN/m2

Total factored loads = 1.5(14.812) = 22.218 kN/m2

(ii) Loads on landing slabs A and B

(a) Self weight of landing slabs = 25(0.2) = 5 kN/m2

(b) Finishes (given) = 1 kN/m2

(c) Live loads (given) = 5 kN/m2

Total: 11 kN/m2

Total factored loads = 1.5(11) = 16.5 kN/m2. The total loads are shown
in Fig.9.20.27.

Version 2 CE IIT, Kharagpur


Step 3: Bending moments and shear forces

Here, there are two types of loads: (i) permanent loads consisting of self-
weights of slabs and finishes for landings and self-weights of slab, finishes and
steps for going, and (ii) live loads. While the permanent loads will be acting
everywhere all the time, the live loads can have several cases. Accordingly, five
different cases are listed below. The design moments and shear forces will be
considered taking into account of the values in each of the cases,. The different
cases are (Fig.9.20.28):

(i) Permanent loads on going and landing slabs

(ii) Live loads on going and landing slabs

(iii) Live loads on landing slab A only

(iv) Live loads on going only

(v) Live loads on landing slabs A and B only

The results of VQ, VR, negative bending moment at Q and positive


bending moment at T (Fig.9.20.27) are summarized in Table 9.2. It is seen from
Table 9.2 that the design moments and shear forces are as follows:

(a) Positive bending moment = 25.195 kNm at T for load cases (i) and (iv).

(b) Negative bending moment = -22.553 kNm at Q for load cases (i) and
(ii).

(c) Maximum shear force = 83.4 kN at Q and R for load cases (i) and (ii).

Table 9.2 Values of reaction forces and bending moments for different
cases of loadings (Example: TQ.4, Figs. 9.20.26 to 9.20.28)

Case VQ (kN) VR (kN) Negative Positive


moment at Q moment at T
(kNm) (kNm)
Permanent loads
on going and +51.34 +51.34 -12.302 +12.535
landings
Live loads on
going and +32.06 +32.06 -10.251 +2.404
landings
Live loads on +18.605 -3.417 -10.251 -5.13
landing A only
Live loads on +16.875 +16.875 0 +2.66

Version 2 CE IIT, Kharagpur


going only
Live loads on +15.1875 +15.1875 -10.251 -10.251
landings A and B
only
Critical positive +68.215 +68.215 -12.302 +25.195
M (i) + (iv)
Critical negative +83.40 +83.40 -22.553 +14.939
M (i) + (ii)

Step 4: Checking of depth of slab

The depth is checked for the positive moment of 25.195 kNm as the two
depths are the same. The effective depth of slab d = {25.195(106)/1500(2.76)}½
= 78 mm < 174 mm. Hence o.k.

The nominal shear stress τ v = 83400/1500(174) = 0.3195 N/mm2. Using


the value of k = 1.2 (cl. 40.2.11 of IS 456) and from Table 19 of IS 456, we get
τ c = 1.2(0.28) = 0.336 N/mm2 and τ c max = 2.8 N/mm2 from Table 20 of IS 456.
Since, τ v < τ c < τ c max , the depth of 200 m is safe against shear.

Step 5: Determination of areas of steel reinforcement

Version 2 CE IIT, Kharagpur


(i) Waist slab: Mu/bd2 = 25195/1.5(174)(174) = 0.555 N/mm2. Table 2 of
SP-16 gives: p = 0.165. Accordingly, Ast = 0.165(1000)(174)/100 = 287.1
mm2/m. Provide 8 mm dia. bars @ 150 mm c/c (= 335 mm2 ).

(ii) Landing slab: Since the difference of positive and negative bending
moments is not much, same reinforcement bars i.e., 8 mm diameter @ 150 mm
c/c is used as positive and negative steel bars of waist and landing slabs.

Distribution bars: 0.12(200)(1000)/100 = 240 mm2/m. The same bar i.e., 8


mm diameter @ 150 mm c/c can be used as distribution bar. The extra amount is
useful to take care of change of ending moments due to different cases of
loadings.

The reinforcement bars are shown in Fig.9.20.29.

9.20.13 Summary of this Lesson


This lesson explains the different types of staircases based on geometrical
considerations. The different terminologies commonly used in a typical flight are
mentioned. Important guidelines to be considered at the planning stage of the
staircase are discussed. The classification of staircases based on structural
system is explained. The distribution of loadings, determination of effective spans
and selection of preliminary discussions of trade, riser and depth of slabs are
illustrated. Simplified analysis procedures of staircases including free-standing
staircase are explained. Several numerical problems are solved in the illustrative
examples, practice problem and test, which will help to understand the
applications of all the guidelines and the design of different types of staircases.

Version 2 CE IIT, Kharagpur


Module
10
Compression Members
Version 2 CE IIT, Kharagpur
Lesson
21
Definitions,
Classifications,
Guidelines and
Assumptions
Version 2 CE IIT, Kharagpur
Instructional Objectives:

At the end of this lesson, the student should be able to:

• define effective length, pedestal, column and wall,

• classify the columns based on types of reinforcement, loadings and


slenderness ratios,

• identify and explain the functions of bracing in a braced column,

• determine the minimum and maximum percentage of longitudinal


reinforcement,

• determine the minimum numbers and diameter of bars in rectangular and


circular columns,

• determine the longitudinal reinforcement in a pedestal,

• determine the type, pitch and diameter of lateral ties of columns after
determining the longitudinal steel,

• state the assumptions in the design of compression member by limit state


of collapse,

• determine the strain distribution lines of a compression member subjected


to axial load with or without the moments about one or both the axes,

• explain the need of the minimum eccentricity to be considered in the


design of compression members.

10.21.1 Introduction

Compression members are structural elements primarily subjected to axial


compressive forces and hence, their design is guided by considerations of
strength and buckling. Figures 10.21.1a to c show their examples: pedestal,
column, wall and strut. While pedestal, column and wall carry the loads along its
length l in vertical direction, the strut in truss carries loads in any direction. The
letters l, b and D represent the unsupported vertical length, horizontal lest
lateral dimension, width and the horizontal longer lateral dimension, depth. These
compression members may be made of bricks or reinforced concrete. Herein,
reinforced concrete compression members are only discussed.

Version 2 CE IIT, Kharagpur


Version 2 CE IIT, Kharagpur
This module is intended to explain the definition of some common
terminologies and to illustrate the design of compression members and other
related issues. This lesson, however, explain the definitions and classifications of
columns depending on different aspects. Further, the recommendations of IS 456
to be followed in the design are discussed regarding the longitudinal and lateral
reinforcing bars. The assumptions made in the design of compression member
by limit sate of collapse are illustrated.

10.21.2 Definitions
(a) Effective length: The vertical distance between the points of inflection
of the compression member in the buckled configuration in a plane is termed as
effective length le of that compression member in that plane. The effective
length is different from the unsupported length l of the member, though it
depends on the unsupported length and the type of end restraints. The relation
between the effective and unsupported lengths of any compression member is

le = k l
(10.1)

where k is the ratio of effective to the unsupported lengths. Clause 25.2 of IS


456 stipulates the effective lengths of compression members (vide Annex E of IS
456). This parameter is needed in classifying and designing the compression
members.

(b) Pedestal: Pedestal is a vertical compression member whose effective


length le does not exceed three times of its least horizontal dimension b (cl.
26.5.3.1h, Note). The other horizontal dimension D shall not exceed four times
of b (Fig.10.21.1a).

(c) Column: Column is a vertical compression member whose


unsupported length l shall not exceed sixty times of b (least lateral dimension),
if restrained at the two ends. Further, its unsupported length of a cantilever
column shall not exceed 100b2/D, where D is the larger lateral dimension which
is also restricted up to four times of b (vide cl. 25.3 of IS 456 and Fig.10.21.1b).

(d) Wall: Wall is a vertical compression member whose effective height


Hwe to thickness t (least lateral dimension) shall not exceed 30 (cl. 32.2.3 of IS
456). The larger horizontal dimension i.e., the length of the wall L is more than
4t (Fig.10.21.1c).

Version 2 CE IIT, Kharagpur


10.21.3 Classification of Columns Based on Types of
Reinforcement

Version 2 CE IIT, Kharagpur


Based on the types of reinforcement, the reinforced concrete columns are
classified into three groups:

(i) Tied columns: The main longitudinal reinforcement bars are enclosed
within closely spaced lateral ties (Fig.10.21.2a).

(ii) Columns with helical reinforcement: The main longitudinal


reinforcement bars are enclosed within closely spaced and continuously wound
spiral reinforcement. Circular and octagonal columns are mostly of this type
(Fig.10.21.2b).

(iii) Composite columns: The main longitudinal reinforcement of the


composite columns consists of structural steel sections or pipes with or without
longitudinal bars (Fig.20.21.2c and d).

Out of the three types of columns, the tied columns are mostly common
with different shapes of the cross-sections viz. square, rectangular, T-, L-, cross
etc. Helically bound columns are also used for circular or octagonal shapes of
cross-sections. Architects prefer circular columns in some specific situations for
the functional requirement. This module, accordingly takes up these two types
(tied and helically bound) of reinforced concrete columns.

Version 2 CE IIT, Kharagpur


10.21.4 Classification of Columns Based on Loadings

Version 2 CE IIT, Kharagpur


Columns are classified into the three following types based on the
loadings:

Version 2 CE IIT, Kharagpur


(i) Columns subjected to axial loads only (concentric), as shown in
Fig.20.21.3a.

(ii) Columns subjected to combined axial load and uniaxial bending, as


shown in Fig.10.21.3b.

(iii) Columns subjected to combined axial load and bi-axial bending, as


shown in Fig.10.21.3c.

Figure 10.21.4 shows the plan view of a reinforced concrete rigid frame
having columns and inter-connecting beams in longitudinal and transverse
directions. From the knowledge of structural analysis it is well known that the
bending moments on the left and right of columns for every longitudinal beam will
be comparable as the beam is continuous. Similarly, the bending moments at the
two sides of columns for every continuous transverse beam are also comparable
(neglecting small amounts due to differences of l1, l2, l3 and b1, b2, b3, b4).
Therefore, all internal columns (C1a to C1f) will be designed for axial force only.
The side columns (C2a to C2j) will have axial forces with uniaxial bending
moment, while the four corner columns (C3a to C3d) shall have axial forces with
bi-axial bending moments. Thus, all internal columns (C1a to C1f), side columns

Version 2 CE IIT, Kharagpur


(C2a to C2j) and corner columns (C3a to C3d) are the columns of type (i), (ii) and
(iii), respectively.

It is worth mentioning that pure axial forces in the inside columns is a rare
case. Due to rigid frame action, lateral loadings and practical aspects of
construction, there will be bending moments and horizontal shear in all the inside
columns also. Similarly, side columns and corner columns will have the column
shear along with the axial force and bending moments in one or both directions,
respectively. The effects of shear are usually neglected as the magnitude is very
small. Moreover, the presence of longitudinal and transverse reinforcement is
sufficient to resist the effect of column shear of comparatively low magnitude.
The effect of some minimum bending moment, however, should be taken into
account in the design even if the column is axially loaded. Accordingly, cls. 39.2
and 25.4 of IS 456 prescribes the minimum eccentricity for the design of all
columns. In case the actual eccentricity is more than the minimum, that should
be considered in the design.

10.21.5 Classification of Columns Based on Slenderness


Ratios
Columns are classified into the following two types based on the
slenderness ratios:

(i) Short columns

(ii) Slender or long columns

Version 2 CE IIT, Kharagpur


Figure 10.21.5 presents the three modes of failure of columns with
different slenderness ratios when loaded axially. In the mode 1, column does not
undergo any lateral deformation and collapses due to material failure. This is
known as compression failure. Due to the combined effects of axial load and
moment a short column may have material failure of mode 2. On the other hand,
a slender column subjected to axial load only undergoes deflection due to beam-
column effect and may have material failure under the combined action of direct
load and bending moment. Such failure is called combined compression and
bending failure of mode 2. Mode 3 failure is by elastic instability of very long
column even under small load much before the material reaches the yield
stresses. This type of failure is known as elastic buckling.

The slenderness ratio of steel column is the ratio of its effective length le
to its least radius of gyration r. In case of reinforced concrete column, however,
IS 456 stipulates the slenderness ratio as the ratio of its effective length le to its
least lateral dimension. As mentioned earlier in sec. 10.21.2(a), the effective
length le is different from the unsupported length, the rectangular reinforced
concrete column of cross-sectional dimensions b and D shall have two
effective lengths in the two directions of b and D. Accordingly, the column may
have the possibility of buckling depending on the two values of slenderness
ratios as given below:

Slenderness ratio about the major axis = lex/D

Slenderness ratio about the minor axis = ley/b

Based on the discussion above, cl. 25.1.2 of IS 456 stipulates the


following:

A compression member may be considered as short when both the


slenderness ratios lex/D and ley/b are less than 12 where lex = effective length
in respect of the major axis, D = depth in respect of the major axis, ley = effective
length in respect of the minor axis, and b = width of the member. It shall
otherwise be considered as a slender compression member.

Further, it is essential to avoid the mode 3 type of failure of columns so


that all columns should have material failure (modes 1 and 2) only. Accordingly,
cl. 25.3.1 of IS 456 stipulates the maximum unsupported length between two
restraints of a column to sixty times its least lateral dimension. For cantilever
columns, when one end of the column is unrestrained, the unsupported length is
restricted to 100b2/D where b and D are as defined earlier.

Version 2 CE IIT, Kharagpur


10.21.6 Braced and unbraced columns

It is desirable that the columns do not have to resist any horizontal loads
due to wind or earthquake. This can be achieved by bracing the columns as in
the case of columns of a water tank or tall buildings (Figs.10.21.6a and b).
Lateral tie members for the columns of water tank or shear walls for the columns
of tall buildings resist the horizontal forces and these columns are called braced
columns. Unbraced columns are supposed to resist the horizontal loads also.
The bracings can be in one or more directions depending on the directions of the
lateral loads. It is worth mentioning that the effect of bracing has been taken into
account by the IS code in determining the effective lengths of columns (vide
Annex E of IS 456).

10.21.7 Longitudinal Reinforcement


The longitudinal reinforcing bars carry the compressive loads along with
the concrete. Clause 26.5.3.1 stipulates the guidelines regarding the minimum
and maximum amount, number of bars, minimum diameter of bars, spacing of
bars etc. The following are the salient points:

(a) The minimum amount of steel should be at least 0.8 per cent of the
gross cross-sectional area of the column required if for any reason the provided
area is more than the required area.

Version 2 CE IIT, Kharagpur


(b) The maximum amount of steel should be 4 per cent of the gross cross-
sectional area of the column so that it does not exceed 6 per cent when bars
from column below have to be lapped with those in the column under
consideration.

(c) Four and six are the minimum number of longitudinal bars in
rectangular and circular columns, respectively.

(d) The diameter of the longitudinal bars should be at least 12 mm.

(e) Columns having helical reinforcement shall have at least six


longitudinal bars within and in contact with the helical reinforcement. The bars
shall be placed equidistant around its inner circumference.

(f) The bars shall be spaced not exceeding 300 mm along the periphery
of the column.

(g) The amount of reinforcement for pedestal shall be at least 0.15 per
cent of the cross-sectional area provided.

10.21.8 Transverse Reinforcement


Transverse reinforcing bars are provided in forms of circular rings,
polygonal links (lateral ties) with internal angles not exceeding 135o or helical
reinforcement. The transverse reinforcing bars are provided to ensure that every
longitudinal bar nearest to the compression face has effective lateral support
against buckling. Clause 26.5.3.2 stipulates the guidelines of the arrangement of
transverse reinforcement. The salient points are:

(a) Transverse reinforcement shall only go round corner and alternate


bars if the longitudinal bars are not spaced more than 75 mm on either side
(Fig.10.21.7).

Version 2 CE IIT, Kharagpur


(b) Longitudinal bars spaced at a maximum distance of 48 times the
diameter of the tie shall be tied by single tie and additional open ties for in
between longitudinal bars (Fig.10.21.8).

(c) For longitudinal bars placed in more than one row (Fig.10.21.9): (i)
transverse reinforcement is provided for the outer-most row in accordance with
(a) above, and (ii) no bar of the inner row is closer to the nearest compression
face than three times the diameter of the largest bar in the inner row.

(d) For longitudinal bars arranged in a group such that they are not in
contact and each group is adequately tied as per (a), (b) or (c) above, as

Version 2 CE IIT, Kharagpur


appropriate, the transverse reinforcement for the compression member as a
whole may be provided assuming that each group is a single longitudinal bar for
determining the pitch and diameter of the transverse reinforcement as given in
sec.10.21.9. The diameter of such transverse reinforcement should not, however,
exceed 20 mm (Fig.10.21.10).

10.21.9 Pitch and Diameter of Lateral Ties


(a) Pitch: The maximum pitch of transverse reinforcement shall be the
least of the following:

(i) the least lateral dimension of the compression members;

(ii) sixteen times the smallest diameter of the longitudinal


reinforcement bar to be tied; and

(iii) 300 mm.

(b) Diameter: The diameter of the polygonal links or lateral ties shall be
not less than one-fourth of the diameter of the largest longitudinal bar, and in no
case less than 6 mm.

10.21.10 Helical Reinforcement


(a) Pitch: Helical reinforcement shall be of regular formation with the turns
of the helix spaced evenly and its ends shall be anchored properly by providing
one and a half extra turns of the spiral bar. The pitch of helical reinforcement
shall be determined as given in sec.10.21.9 for all cases except where an
increased load on the column is allowed for on the strength of the helical
reinforcement. In such cases only, the maximum pitch shall be the lesser of 75
mm and one-sixth of the core diameter of the column, and the minimum pitch
shall be the lesser of 25 mm and three times the diameter of the steel bar
forming the helix.

(b) Diameter: The diameter of the helical reinforcement shall be as


mentioned in sec.10.21.9b.

10.21.11 Assumptions in the Design of Compression


Members by Limit State of Collapse
It is thus seen that reinforced concrete columns have different
classifications depending on the types of reinforcement, loadings and
slenderness ratios. Detailed designs of all the different classes are beyond the
scope here. Tied and helically reinforced short and slender columns subjected to
axial loadings with or without the combined effects of uniaxial or biaxial bending

Version 2 CE IIT, Kharagpur


will be taken up. However, the basic assumptions of the design of any of the
columns under different classifications are the same. The assumptions (i) to (v)
given in sec.3.4.2 of Lesson 4 for the design of flexural members are also
applicable here. Furthermore, the following are the additional assumptions for the
design of compression members (cl. 39.1 of IS 456).

(i) The maximum compressive strain in concrete in axial compression


is taken as 0.002.

(ii) The maximum compressive strain at the highly compressed


extreme fibre in concrete subjected to axial compression and
bending and when there is no tension on the section shall be
0.0035 minus 0.75 times the strain at the least compressed
extreme fibre.

Version 2 CE IIT, Kharagpur


The assumptions (i) to (v) of section 3.4.2 of Lesson 4 and (i) and (ii)
mentioned above are discussed below with reference to Fig.10.21.11a to c
presenting the cross-section and strain diagrams for different location of the
neutral axis.

The discussion made in sec. 3.4.2 of Lesson 4 regarding the assumptions


(i), (iii), (iv) and (v) are applicable here also. Assumption (ii) of sec.3.4.2 is also
applicable here when kD, the depth of neutral axis from the highly compressed
right edge is within the section i.e., k < 1. The corresponding strain profile IN in
Fig.10.21.11b is for particular value of P and M such that the maximum
compressive strain is 0.0035 at the highly compressed right edge and tensile
strain develops at the opposite edge. This strain profile is very much similar to
that of a beam in flexure of Lesson 4.

The additional assumption (i) of this section refers to column subjected


axial load P only resulting compressive strain of maximum (constant) value of
0.002 and for which the strain profile is EF in Fig.10.21.11b. The neutral axis is at
infinity (outside the section).

Extending the assumption of the strain profile IN (Fig.10.21.11b), we can


draw another strain profile IH (Fig.10.21.11c) having maximum compressive
strain of 0.0035 at the right edge and zero strain at the left edge. This strain
profile 1H along with EF are drawn in Fig.10.21.11c to intersect at V. From the
two similar triangles EVI and GHI, we have

EV/GH = 0.0015/0.0035 = 3/7, which gives

EV = 3D/7
(10.2)

The point V, where the two profiles intersect is assumed to act as a fulcrum for
the strain profiles when the neutral axis lies outside the section. Another strain
profile JK drawn on this figure passing through the fulcrum V and whose neutral
axis is outside the section. The maximum compressive strain GJ of this profile is
related to the minimum compressive strain HK as explained below.

GJ = GI – IJ = GI – 0.75 HK, as we can write IJ in term of HK from two


similar triangles JVI and HVK:

IJ/HK = VE/VF = 0.75.

The value of the maximum compressive strain GJ for the profile JK is,
therefore, 0.0035 minus 0.75 times the strain HK on the least compressed edge.
This is the assumption (ii) of this section (cl. 39.1b of IS 456).

Version 2 CE IIT, Kharagpur


10.21.12 Minimum Eccentricity
Section 10.21.4 illustrates that in practical construction, columns are rarely
truly concentric. Even a theoretical column loaded axially will have accidental
eccentricity due to inaccuracy in construction or variation of materials etc.
Accordingly, all axially loaded columns should be designed considering the
minimum eccentricity as stipulated in cl. 25.4 of IS 456 and given below
(Fig.10.21.3c)

ex min ≥ greater of )l/500 + D/30) or 20 mm

(10.3)

ey min ≥ greater of )l/500 + b/30) or 20 mm

where l, D and b are the unsupported length, larger lateral dimension and least
lateral dimension, respectively.

10.21.13 Practice Questions and Problems with Answers


Q.1: Define effective length, pedestal, column and wall.

A.1: See sec. 10.21.2.

Q.2: Classify the columns based on types of reinforcement.

A.2: See sec. 10.21.3

Q.3: Classify the columns based on loadings.

A.3: See sec. 10.21.4.

Q.4: Classify the columns based on slenderness ratios.

A.4: See Sec. 10.21.5

Q.5: Explain braced and unbraced columns.

A.5: See sec. 10.21.6.

Q.6: Answer the following:

(a) What are the minimum and maximum amounts of longitudinal


reinforcement in a column?

Version 2 CE IIT, Kharagpur


(b) What are the minimum numbers of longitudinal bars in rectangular and
circular columns?

(c) What is the amount of longitudinal reinforcement in a pedestal?

(d) What is the maximum pitch of transverse reinforcement in a column?

(e) What is the diameter of lateral ties in a column?

A.6: (a) 0.8% and 4%

(b) 4 and 6

(c) 0.15% of cross-sectional area of the pedestal

(d) See sec. 10.21.9(a)

(e) See sec. 10.21.9(b).

Q.7: Explain the assumptions of determining the strain distribution lines in a


column subjected to axial force and biaxial bending.

A.7: See sec. 10.21.11(i) and (ii).

Q.8: State the minimum eccentricity of a rectangular column for designing.

A.8: See sec. 10.21.12.

10.21.14 References
1. Reinforced Concrete Limit State Design, 6th Edition, by Ashok K. Jain,
Nem Chand & Bros, Roorkee, 2002.
2. Limit State Design of Reinforced Concrete, 2nd Edition, by P.C.Varghese,
Prentice-Hall of India Pvt. Ltd., New Delhi, 2002.
3. Advanced Reinforced Concrete Design, by P.C.Varghese, Prentice-Hall of
India Pvt. Ltd., New Delhi, 2001.
4. Reinforced Concrete Design, 2nd Edition, by S.Unnikrishna Pillai and
Devdas Menon, Tata McGraw-Hill Publishing Company Limited, New
Delhi, 2003.
5. Limit State Design of Reinforced Concrete Structures, by P.Dayaratnam,
Oxford & I.B.H. Publishing Company Pvt. Ltd., New Delhi, 2004.
6. Reinforced Concrete Design, 1st Revised Edition, by S.N.Sinha, Tata
McGraw-Hill Publishing Company. New Delhi, 1990.
7. Reinforced Concrete, 6th Edition, by S.K.Mallick and A.P.Gupta, Oxford &
IBH Publishing Co. Pvt. Ltd. New Delhi, 1996.

Version 2 CE IIT, Kharagpur


8. Behaviour, Analysis & Design of Reinforced Concrete Structural Elements,
by I.C.Syal and R.K.Ummat, A.H.Wheeler & Co. Ltd., Allahabad, 1989.
9. Reinforced Concrete Structures, 3rd Edition, by I.C.Syal and A.K.Goel,
A.H.Wheeler & Co. Ltd., Allahabad, 1992.
10. Textbook of R.C.C, by G.S.Birdie and J.S.Birdie, Wiley Eastern Limited,
New Delhi, 1993.
11. Design of Concrete Structures, 13th Edition, by Arthur H. Nilson, David
Darwin and Charles W. Dolan, Tata McGraw-Hill Publishing Company
Limited, New Delhi, 2004.
12. Concrete Technology, by A.M.Neville and J.J.Brooks, ELBS with
Longman, 1994.
13. Properties of Concrete, 4th Edition, 1st Indian reprint, by A.M.Neville,
Longman, 2000.
14. Reinforced Concrete Designer’s Handbook, 10th Edition, by C.E.Reynolds
and J.C.Steedman, E & FN SPON, London, 1997.
15. Indian Standard Plain and Reinforced Concrete – Code of Practice (4th
Revision), IS 456: 2000, BIS, New Delhi.
16. Design Aids for Reinforced Concrete to IS: 456 – 1978, BIS, New Delhi.

10.21.15 Test 21 with Solutions


Maximum Marks = 50, Maximum Time = 30 minutes

Answer all questions carrying equal marks.

TQ.1: Define effective length, pedestal, column and wall.

A.TQ.1: See sec. 10.21.2.

TQ.2: Classify the columns separately based on loadings and slenderness


ratios.

A.TQ.2: See secs. 10.21.4 and 5.

TQ.3: Explain braced and unbraced columns.

A.TQ.3: See sec. 10.21.6.

TQ.4: Answer the following:

(a) What are the minimum and maximum amounts of longitudinal


reinforcement in a column?

(b) What are the minimum numbers of longitudinal bars in rectangular


and circular columns?

Version 2 CE IIT, Kharagpur


(c) What is the amount of longitudinal reinforcement in a pedestal?

(d) What is the maximum pitch of transverse reinforcement in a column?

(e) What is the diameter of lateral ties in a column?

A.TQ.4: (a) 0.8% and 4%

(b) 4 and 6

(c) 0.15% of cross-sectional area of the pedestal

(d) See sec. 10.21.9(a)

(e) See sec. 10.21.9(b).

TQ.5: Explain the assumptions of determining the strain distribution lines in a


column subjected to axial force and biaxial bending.

A.TQ.5: See sec. 10.21.11(i) and (ii).

10.21.16 Summary of this Lesson


This lesson defines the effective length, pedestal, column and wall. Three
different classifications of columns based on types of reinforcement, loadings
slenderness ratio are explained. The need and functions of bracings are
illustrated. The guidelines of IS 456 are discussed regarding the types,
arrangement, minimum numbers and diameter of bars, pitch and other aspects of
longitudinal and transverse reinforcement of columns. The assumptions needed
for the design of compression members are illustrated. The determination of
strain distribution lines are explained depending on the location of the neutral
axis. The need for considering the minimum eccentricity and its amount are
explained.

Version 2 CE IIT, Kharagpur


Module
10
Compression Members
Version 2 CE IIT, Kharagpur
Lesson
22
Short Axially Loaded
Compression Members
Version 2 CE IIT, Kharagpur
Instructional Objectives:
At the end of this lesson, the student should be able to:

• state additional assumptions regarding the strengths of concrete and steel


for the design of short axially loaded columns,

• specify the values of design strengths of concrete and steel,

• derive the governing equation for the design of short and axially loaded
tied columns,

• derive the governing equation for the design of short and axially loaded
spiral columns,

• derive the equation to determine the pitch of helix in spiral columns,

• apply the respective equations to design the two types of columns by


direct computation,

• use the charts of SP-16 to design these two types of columns subjected to
axial loads as per IS code.

10.22.1 Introduction
Tied and helically bound are the two types of columns mentioned in
sec.10.21.3 of Lesson 21. These two types of columns are taken up in this
lesson when they are short and subjected to axially loads. Out of several types of
plan forms, only rectangular and square cross-sections are covered in this lesson
for the tied columns and circular cross-section for the helically bound columns.
Axially loaded columns also need to be designed keeping the provision of
resisting some moments which normally is the situation in most of the practical
columns. This is ensured by checking the minimum eccentricity of loads applied
on these columns as stipulated in IS 456. Moreover, the design strengths of
concrete and steel are further reduced in the design of such columns. The
governing equations of the two types of columns and the equation for
determining the pitch of the helix in continuously tied column are derived and
explained. The design can be done by employing the derived equation i.e., by
direct computation or by using the charts of SP-16. Several numerical examples
are solved to explain the design of the two types of columns by direct
computation and using the charts of SP-16.

Version 2 CE IIT, Kharagpur


10.22.2 Further Assumptions Regarding the Strengths of
Concrete and Steel
All the assumptions required for the derivation of the governing equations
are given in sec.10.21.11 of Lesson 21. The stress-strain diagrams of mild steel
(Fe 250) and cold worked deformed bars (Fe 415 and Fe 500) are given in
Figs.1.2.3 and 4, respectively of Lesson 2. The stress block of compressive part
of concrete is given in Fig.3.4.1.9 of Lesson 4, which is used in the design of
beam by limit state of collapse. The maximum design strength of concrete is
shown as constant at 0.446 fck when the strain ranges from 0.002 to 0.0035. The
maximum design stress of steel is 0.87 fy.

Sections 10.21.4 and 12 of Lesson 21 explain that all columns including


the short axially loaded columns shall be designed with a minimum eccentricity
(cls. 25.4 and 39.2 of IS 456). Moreover, the design strengths of concrete and
steel are further reduced to 0.4 fck and 0.67 fy, respectively, to take care of the
minimum eccentricity of 0.05 times the lateral dimension, as stipulated in cl.39.3
of IS 456. It is noticed that there is not attempt at strain compatibility. Also the
phenomenon of creep has not been directly considered.

ex min ≥ greater of (l/500 + D/30) or 20 mm

(10.3)
ey min ≥ greater of (l/500 + b/30) or 20 mm

The maximum values of lex/D and ley/b should not exceed 12 in a short column
as per cl.25.1.2 of IS 456. For a short column, when the unsupported length l =
lex (for the purpose of illustration), we can assume l = 12 D (or 12b when b is
considered). Thus, we can write the minimum eccentricity = 12D/500 + D/30 =
0.057D, which has been taken as 0.05D or 0.05b as the maximum amount of
eccentricity of a short column.

It is, therefore, necessary to keep provision so that the short columns can
resist the accidental moments due to the allowable minimum eccentricity by
lowering the design strength of concrete by ten per cent from the value of
0.446fck, used for the design of flexural members. Thus, we have the design
strength of concrete in the design of short column as (0.9)(0.446fck) = 0.4014fck,
say 0.40 fck. The reduction of the design strength of steel is explained below.

For mild steel (Fe 250), the design strength at which the strain is 0.002 is
fy/1.15 = 0.87fy. However, the design strengths of cold worked deformed bars (Fe
415 and Fe 500) are obtained from Fig.1.2.4 of Lesson 2 or Fig.23A of IS 456.
Table A of SP-16 presents the stresses and corresponding strains of Fe 415 and
Fe 500. Use of Table A of SP-16 is desirable as it avoids error while reading from
figures (Fig.1.2.4 or Fig.23A, as mentioned above). From Table A of SP-16, the

Version 2 CE IIT, Kharagpur


corresponding design strengths are obtained by making linear interpolation.
These values of design strengths for which the strain is 0.002 are as follows:

(i) Fe 415: {0.9fyd + 0.05fyd(0.002 – 0.00192)/(0.00241 – 0.00192)} = 0.908fyd =


0.789fy

(ii) Fe 500: {0.85fyd + 0.05fyd(0.002 – 0.00195)/(0.00226 – 0.00195)} = 0.859fyd =


0.746fy

A further reduction in each of three values is made to take care of the


minimum eccentricity as explained for the design strength of concrete. Thus, the
acceptable design strength of steel for the three grades after reducing 10 per
cent from the above mentioned values are 0.783fy, 0.710fy and 0.671fy for Fe
250, Fe 415 and Fe 500, respectively. Accordingly, cl. 39.3 of IS 456 stipulates
0.67fy as the design strength for all grades of steel while designing the short
columns. Therefore, the assumed design strengths of concrete and steel are
0.4fck and 0.67fy, respectively, for the design of short axially loaded columns.

10.22.3 Governing Equation for Short Axially Loaded Tied


Columns
Factored concentric load applied on short tied columns is resisted by
concrete of area Ac and longitudinal steel of areas Asc effectively held by lateral
ties at intervals (Fig.10.21.2a of Lesson 21). Assuming the design strengths of
concrete and steel are 0.4fck and 0.67fy, respectively, as explained in sec.
10.22.2, we can write

Pu = 0.4fck Ac + 0.67fy Asc


(10.4)

where Pu = factored axial load on the member,

fck = characteristic compressive strength of the concrete,

Ac = area of concrete,

fy = characteristic strength of the compression reinforcement, and

Asc = area of longitudinal reinforcement for columns.

The above equation, given in cl. 39.3 of IS 456, has two unknowns Ac and Asc to
be determined from one equation. The equation is recast in terms of Ag, the
gross area of concrete and p, the percentage of compression reinforcement
employing

Version 2 CE IIT, Kharagpur


Asc = pAg/100
(10.5)

Ac = Ag(1 – p/100)
(10.6)

Accordingly, we can write

Pu/Ag = 0.4fck + (p/100) (0.67fy – 0.4fck)


(10.7)

Equation 10.7 can be used for direct computation of Ag when Pu, fck and fy are
known by assuming p ranging from 0.8 to 4 as the minimum and maximum
percentages of longitudinal reinforcement. Equation 10.4 also can be employed
to determine Ag and p in a similar manner by assuming p. This method has been
illustrated with numerical examples and is designated as Direct Computation
Method.

On the other hand, SP-16 presents design charts based on Eq.10.7. Each
chart of charts 24 to 26 of SP-16 has lower and upper sections. In the lower
section, Pu/Ag is plotted against the reinforcement percentage p(= 100As/Ag) for
different grades of concrete and for a particular grade of steel. Thus, charts 24 to
26 cover the three grades of steel with a wide range of grades of concrete. When
the areas of cross-section of the columns are known from the computed value of
Pu/Ag, the percentage of reinforcement can be obtained directly from the lower
section of the chart. The upper section of the chart is a plot of Pu/Ag versus Pu
for different values of Ag. For a known value of Pu, a horizontal line can be drawn
in the upper section to have several possible Ag values and the corresponding
Pu/Ag values. Proceeding vertically down for any of the selected Pu/Ag value, the
corresponding percentage of reinforcement can be obtained. Thus, the combined
use of upper and lower sections of the chart would give several possible sizes of
the member and the corresponding Asc without performing any calculation. It is
worth mentioning that there may be some parallax error while using the charts.
However, use of chart is very helpful while deciding the sizes of columns at the
preliminary design stage with several possible alternatives.

Another advantage of the chart is that, the amount of compression


reinforcement obtained from the chart are always within the minimum and
maximum percentages i.e., from 0.8 to 4 per cent. Hence, it is not needed to
examine if the computed area of steel reinforcement is within the allowable range
as is needed while using Direct Computation Method. This method is termed as
SP-16 method while illustrating numerical examples.

Version 2 CE IIT, Kharagpur


10.22.4 Governing Equation of Short Axially Loaded
Columns with Helical Ties
Columns with helical reinforcement take more load than that of tied
columns due to additional strength of spirals in contributing to the strength of
columns. Accordingly, cl. 39.4 recommends a multiplying factor of 1.05 regarding
the strength of such columns. The code further recommends that the ratio of
volume of helical reinforcement to the volume of core shall not be less than 0.36
(Ag/Ac – 1) (fck/fy), in order to apply the additional strength factor of 1.05 (cl.
39.4.1). Accordingly, the governing equation of the spiral columns may be written
as

Pu = 1.05(0.4 fck Ac + 0.67 fy Asc)


(10.8)

All the terms have been explained in sec.10.22.3.

Earlier observations of several investigators reveal that the effect of


containing holds good in the elastic stage only and it gets lost when spirals reach
the yield point. Again, spirals become fully effective after spalling off the concrete
cover over the spirals due to excessive deformation. Accordingly, the above two
points should be considered in the design of such columns. The first point is
regarding the enhanced load carrying capacity taken into account by the
multiplying factor of 1.05. The second point is maintaining specified ratio of
volume of helical reinforcement to the volume of core, as specified in cl.39.4.1
and mentioned earlier.

The second point, in fact, determines the pitch p of the helical


reinforcement, as explained below with reference to Fig.10.21.2b of Lesson 21.

Volume of helical reinforcement in one loop = π ( Dc - φ sp ) a sp


(10.9)

Volume of core = (π / 4) Dc2 p


(10.10)

where Dc = diameter of the core (Fig.10.21.2b)

φ sp = diameter of the spiral reinforcement (Fig.10.21.2b)

asp = area of cross-section of spiral reinforcement

p = pitch of spiral reinforcement (Fig.10.21.2b)

To satisfy the condition of cl.39.4.1 of IS 456, we have

Version 2 CE IIT, Kharagpur


{π ( Dc - φ sp ) a sp } /{(π / 4) Dc2 p} ≥ 0.36( Ag / Ac - 1) ( f ck / f y )

which finally gives

p ≤ 11.1(Dc - φ sp ) a sp f y /( D 2 - Dc2 ) f ck
(10.11)

Thus, Eqs.10.8 and 11 are the governing equations to determine the diameter of
column, pitch of spiral and area of longitudinal reinforcement. It is worth
mentioning that the pitch p of the spiral reinforcement, if determined from
Eq.10.11, automatically satisfies the stipulation of cl.39.4.1 of IS 456. However,
the pitch and diameter of the spiral reinforcement should also satisfy cl. 26.5.3.2
of IS 456:2000.

10.22.5 Illustrative Examples


Problem 1:

Design the reinforcement in a column of size 400 mm x 600 mm subjected


to an axial load of 2000 kN under service dead load and live load. The column
has an unsupported length of 4.0 m and effectively held in position and
restrained against rotation in both ends. Use M 25 concrete and Fe 415 steel.

Solution 1:

Step 1: To check if the column is short or slender

Given l = 4000 mm, b = 400 mm and D = 600 mm. Table 28 of IS 456 = lex = ley =
0.65(l) = 2600 mm. So, we have

lex/D = 2600/600 = 4.33 < 12

ley/b = 2600/400 = 6.5 < 12

Hence, it is a short column.

Step 2: Minimum eccentricity

ex min = Greater of (lex/500 + D/30) and 20 mm = 25.2 mm

ey min = Greater of (ley/500 + b/30) and 20 mm = 20 mm

0.05 D = 0.05(600) = 30 mm > 25.2 mm (= ex min)

Version 2 CE IIT, Kharagpur


0.05 b = 0.05(400) = 20 mm = 20 mm (= ey min)

Hence, the equation given in cl.39.3 of IS 456 (Eq.10.4) is applicable for the
design here.

Step 3: Area of steel

Fro Eq.10.4, we have

Pu = 0.4 fck Ac + 0.67 fy Asc …. (10.4)

3000(103) = 0.4(25){(400)(600) – Asc} + 0.67(415) Asc

which gives,

Asc = 2238.39 mm2

Provide 6-20 mm diameter and 2-16 mm diameter rods giving 2287 mm2 (>
2238.39 mm2) and p = 0.953 per cent, which is more than minimum percentage
of 0.8 and less than maximum percentage of 4.0. Hence, o.k.

Step 4: Lateral ties

The diameter of transverse reinforcement (lateral ties) is determined from


cl.26.5.3.2 C-2 of IS 456 as not less than (i) φ /4 and (ii) 6 mm. Here, φ = largest
bar diameter used as longitudinal reinforcement = 20 mm. So, the diameter of
bars used as lateral ties = 6 mm.

The pitch of lateral ties, as per cl.26.5.3.2 C-1 of IS 456, should be not
more than the least of

(i) the least lateral dimension of the column = 400 mm

(ii) sixteen times the smallest diameter of longitudinal reinforcement


bar to be tied = 16(16) = 256 mm

(iii) 300 mm

Version 2 CE IIT, Kharagpur


Let us use p = pitch of lateral ties = 250 mm. The arrangement of
longitudinal and transverse reinforcement of the column is shown in Fig. 10.22.1.

Problem 2:

Design the column of Problem 1 employing the chart of SP-16.

Solution 2:

Steps 1 and 2 are the same as those of Problem 1.

Step 3: Area of steel

Pu/Ag = 3000(103)/(600)(400) = 12.5 N/mm2

From the lower section of Chart 25 of SP-16, we get p = 0.95% when Pu/Ag =
12.5 N/mm2 and concrete grade is M 25. This gives Asc = 0.95(400)(600)/100 =
2288 mm2. The results of both the problems are in good agreement. Marginally
higher value of Asc while using the chart is due to parallax error while reading the
value from the chart. Here also, 6-20 mm diameter bars + 2-16 mm diameter
bars (Asc provided = 2287 mm2) is o.k., though it is 1 mm2 less.

Step 4 is the same as that of Problem 1. Figure 10.22.1, thus, is also the
figure showing the reinforcing bars (longitudinal and transverse reinforcement) of
this problem (same column as that of Problem 1).

Problem 3:

Design a circular column of 400 mm diameter with helical reinforcement


subjected to an axial load of 1500 kN under service load and live load. The
column has an unsupported length of 3 m effectively held in position at both ends
but not restrained against rotation. Use M 25 concrete and Fe 415 steel.

Version 2 CE IIT, Kharagpur


Solution 3:

Step 1: To check the slenderness ratio

Given data are: unsupported length l = 3000 mm, D = 400 mm. Table 28
of Annex E of IS 456 gives effective length le = l = 3000 mm. Therefore, le/D = 7.5
< 12 confirms that it is a short column.

Step 2: Minimum eccentricity

emin = Greater of (l/500 + D/30) or 20 mm = 20 mm

0.05 D = 0.05(400) = 20 mm

As per cl.39.3 of IS 456, emin should not exceed 0.05D to employ the equation
given in that clause for the design. Here, both the eccentricities are the same.
So, we can use the equation given in that clause of IS 456 i.e., Eq.10.8 for the
design.

Step 3: Area of steel

From Eq.10.8, we have

Pu = 1.05(0.4 fck Ac + 0.67 fy Asc) … (10.8)

Ac = Ag – Asc = 125714.29 - Asc

Substituting the values of Pu, fck, Ag and fy in Eq.10.8,

1.5(1500)(103) = 1.05{0.4(25)(125714.29 – Asc) + 0.67(415) Asc}

we get the value of Asc = 3304.29 mm2. Provide 11 nos. of 20 mm diameter bars
(= 3455 mm2) as longitudinal reinforcement giving p = 2.75%. This p is between
0.8 (minimum) and 4 (maximum) per cents. Hence o.k.

Step 4: Lateral ties

It has been mentioned in sec.10.22.4 that the pitch p of the helix


determined from Eq.10.11 automatically takes care of the cl.39.4.1 of IS 456.
Therefore, the pitch is calculated from Eq.10.11 selecting the diameter of helical
reinforcement from cl.26.5.3.2 d-2 of IS 456. However, automatic satisfaction of
cl.39.4.1 of IS 456 is also checked here for confirmation.

Diameter of helical reinforcement (cl.26.5.3.2 d-2) shall be not less than


greater of (i) one-fourth of the diameter of largest longitudinal bar, and (ii) 6 mm.

Version 2 CE IIT, Kharagpur


Therefore, with 20 mm diameter bars as longitudinal reinforcement, the diameter
of helical reinforcement = 6 mm.

From Eq.10.11, we have

Pitch of helix p ≤ 11.1(Dc - φ sp ) asp fy/(D2 - Dc2 ) f ck … (10.11)

where Dc = 400 – 40 – 40 = 320 mm, φ sp = 6 mm, asp = 28 mm2, fy = 415


N/mm2, D = 400 mm and fck = 25 N/mm2.

So, p ≤ 11.1(320 – 6) (28) (415)/(4002 – 3202) (25) ≤ 28.125 mm

As per cl.26.5.3.2 d-1, the maximum pitch is the lesser of 75 mm and 320/6 =
53.34 mm and the minimum pitch is lesser of 25 mm and 3(6) = 18 mm. We
adopt pitch = 25 mm which is within the range of 18 mm and 53.34 mm. So,
provide 6 mm bars @ 25 mm pitch forming the helix.

Checking of cl. 39.4.1 of IS 456

The values of helical reinforcement and core in one loop are obtained from
Eqs.10.8 and 9, respectively. Substituting the values of Dc, φ sp , asp and pitch p in
the above two equations, we have

Volume of helical reinforcement in one loop = 27632 mm3 and

Volume of core in one loop = 2011428.571 mm3.

Their ratio = 27632/2011428.571 = 0.0137375

0.36(Ag/Ac – 1) (fck/fy) = 0.012198795

It is, thus, seen that the above ratio (0.0137375) is not less than 0.36(Ag/Ac – 1)
(fck/fy).

Version 2 CE IIT, Kharagpur


Hence, the circular column of diameter 400 mm has eleven longitudinal
bars of 20 mm diameter and 6 mm diameter helix with pitch p = 25 mm. The
reinforcing bars are shown in Fig.10.22.2.

10.22.6 Practice Questions and Problems with Answers


Q.1: State and explain the values of design strengths of concrete and steel to be
considered in the design of axially loaded short columns.

A.1: See sec. 10.22.2.

Q.2: Derive the governing equation for determining the dimensions of the
column and areas of longitudinal bars of an axially loaded short tied
column.

A.2: See sec. 10.22.2.

Q.3: Derive the governing equation for determining the diameter and areas of
longitudinal bars of an axially loaded circular spiral short column.

A.3: First and second paragraph of sec. 10.22.4.

Q.4: Derive the expression of determining the pitch of helix in a short axially
loaded spiral column which satisfies the requirement of IS 456.

A.4: See third paragraph onwards up to the end of sec. 10.22.4.

Q.5: Design a short rectangular tied column of b = 300 mm having the maximum
amount of longitudinal reinforcement employing the equation given in

Version 2 CE IIT, Kharagpur


cl.39.3 of IS 456, to carry an axial load of 1200 kN under service dead
load and live load using M 25 and Fe 415. The column is effectively held
in position at both ends and restrained against rotation at one end.
Determine the unsupported length of the column.

A.5:

Step 1: Dimension D and area of steel Asc

Substituting the values of Pu = 1.5(1200) = 1800 kN and Asc = 0.04(300)D


in Eq.10.4, we have

1800(103) = 0.4(25)(300D)(1 – 0.04) + 0.67(415)(0.04)(300D)

we get D = 496.60 mm. Use 300 mm x 500 mm column.

Asc = 0.04(300)(500) = 6000 mm2, provide 4-36 mm diameter + 4-25 mm


diameter bars to give 4071 + 1963 = 6034 mm2 > 6000 mm2.

Step 2: Lateral ties

Diameter of lateral ties shall not be less than the larger of (i) 36/4 = 9 mm
and (ii) 6 mm. Use 10 mm diameter bars as lateral ties.

Pitch of the lateral ties p shall not be more than the least of (i) 300 mm, (ii)
16(25) = 400 mm and (iii) 300 mm.

So, provide 10 mm diameter bars @ 300 mm c/c. The reinforcement bars


are shown in Fig.10.22.3.

Version 2 CE IIT, Kharagpur


The centre to centre distance between two corner longitudinal bas along
500 mm direction is 500 – 2(4) + 10 + 18) = 364 mm which is less than 48
(diameter of lateral tie). Hence, the arrangement is satisfying Fig.9 of cl. 26.5.3.2
b-2 of IS 456.

Step 3: Unsupported length

As per the stipulation in cl. 25.1.2 of IS 456, the column shall be


considered as short if lex = 12(D) = 6000 mm and ley = 12(300) = 3600 mm. For
the type of support conditions given in the problem, Table 28 of IS 456 gives
unsupported length is the least of (i) l = lex/0.8 = 6000/0.8 = 7500 mm and (ii)
ley/0.8 = 3600/0.8 = 4500 mm. Hence, the unsupported length of the column is
4.5 m if the minimum eccentricity clause (cl. 39.3) is satisfied, which is checked
in the next step.

Step 4: Check for minimum eccentricity

According to cl. 39.4 of IS 456, the minimum eccentricity of 0.05b or 0.05D


shall not exceed as given in cl. 25.4 of IS 456. Thus, we have

(i) 0.05(500) = l/500 + 500/30 giving l = 4165 mm

(ii) 0.05(300) = l/500 + 300/10 giving l = 2500 mm

Therefore, the column shall have the unsupported length of 2.5 m.

Q.6: (a) Suggest five alternative dimensions of square short column with the
minimum longitudinal reinforcement to carry a total factored axial load of
3000 kN using concrete of grades 20, 25, 30, 35 and 40 and Fe 415.
Determine the respective maximum unsupported length of the column if it
is effectively held in position at both ends but not restrained against
rotation. Compare the given factored load of the column with that obtained
by direct computation for all five alternative columns.

(b) For each of the five alternative sets of dimensions obtained in (a),
determine the maximum factored axial load if the column is having
maximum longitudinal reinforcement (i) employing SP-16 and (ii) by direct
computation.

Version 2 CE IIT, Kharagpur


A.6:

Solution of Part (a):

Step 1: Determination of Ag and column dimensions b (= D)

Chart 25 of SP-16 gives all the dimensions of five cases. The two input
data are Pu = 3000 kN and 100 As/Ag = 0.8. In the lower section of Chart 25, one
horizontal line AB is drawn starting from A where p = 0.8 (Fig.10.22.4) to meet
the lines for M 20, 25, 30, 35 and 40 respectively. In Fig.10.22.4, B is the meeting
point for M 20 concrete. Separate vertical lines are drawn from these points of
intersection to meet another horizontal line CD from the point C where Pu = 3000
kN in the upper section of the figure. The point D is the intersecting point. D
happens to be on line when Ag = 3000 cm2. Otherwise, it may be in between two
liens with different values of Ag. For M 20, Ag = 3000 cm2. However, in case the
point is in between two lines with different values of Ag, the particular Ag has to
be computed by linear interpolation. Thus, all five values of Ag are obtained.

The dimension b = D = 300000 = 550 mm. Other four values are


obtained similarly. Table 10.1 presents the values of Ag and D along with other
results as explained below.

Version 2 CE IIT, Kharagpur


Step 2: Unsupported length of each column

The unsupported length l is determined from two considerations:

(i) Clause 25.1.2 of IS 456 mentions that the maximum effective length lex
is 12 times b or D (as b = D here for a square column). The unsupported length is
related to the effective length depending on the type of support. In this problem
Table 28 of IS 456 stipulates l = lex. Therefore, maximum value of l = 12 D.

(ii) The minimum eccentricity of cl. 39.3 should be more than the same as
given in cl. 25.4. Assuming them to be equal, we get l/500 + D/30 = D/20, which
gives l = 8.33D. For the column using M 20 and Fe 415, the unsupported length
= 8.33(550) = 4581 mm. All unsupported lengths are presented in Table 10.1
using the equation

l = 8.33 D
(1)

Step 3: Area of longitudinal steel

Step 1 shows that the area provided for the first case is 550 mm x 550 mm
= 302500 mm2, slightly higher than the required area of 300000 mm2 for the
practical aspects of construction. However, the minimum percentage of the
longitudinal steel is to the calculated as 0.8 per cent of area required and not
area provided (vide cl. 26.5.3.1 b of IS 456). Hence, for this case Asc =
0.8(300000)/100 = 2400 mm2. Provide 4-25 mm diameter + 4-12 mm diameter
bars (area = 1963 + 452 = 2415 mm2). Table 10.1 presents this and other areas
of longitudinal steel obtained in a similar manner.

Step 4: Factored load by direct computation

Equation 10.4 is employed to calculate the factored load by determining Ac


from Ag and Ast. With a view to comparing the factored loads, we will use the
values of Ag as obtained from the chart and not the Ag actually provided. From
Eq.10.4, we have

Pu from direct computation = 0.4(fck)(0.992 Ag) + 0.67(fy)(0.008)Ag

or Pu = Ag(0.3968 fck + 0.00536 fy)


(2)

For the first case when Ag = 300000 mm2, fck = 20 N/mm2, and fy = 415 N/mm2,
Eq.(2) gives Pu = 3048.12 kN. This value and other values of factored loads
obtained from the direct computation are presented in Table 10.1.

Version 2 CE IIT, Kharagpur


Table 10.1 Results of Q.6a (Minimum Longitudinal Steel), given factored Pu =
3000 kN

Gross area of Area of steel (Asc)


Concret concrete (Ag) b=D Pu by direct l
e grade Require Provide (cm) Require Provide Bars computation (m)
d (cm2) d (cm2) d (cm2) d (cm2)
M 20 3000 3025 55 24 24.15 4-25 + 3048.12 4.58
4-12 1
M 25 2500 2500 50 20 20.60 4-20 + 3036.10 4.16
4-16 5
M 30 2200 2209 47 17.60 17.85 2.25 + 3108.25 3.91
4-16 5
M 35 1800 1806 42.5 14.40 14.57 2-28 + 2900.23 3.54
2-12 0
M 40 1600 1600 40 12.80 13.06 2-20 + 2895.42 3.33
6-12 2

Solution of Part (b):

Step 1: Determination of Pu

Due to the known dimensions of the column section, the Ag is now known.
With known Ag and reinforcement percentage 100As/Ag as 4 per cent, the
factored Pu shall be determined. For the first case, when b = D = 550 mm, Ag =
302500 mm2. In Chart 25, we draw a horizontal line EF from E, where 100As/Ag =
4 in the lower section of the chart (see Fig.10.22.4) to meet the M 20 line at F.
Proceeding vertically upward, the line FG intersects the line Ag = 302500 at G. A
horizontal line towards left from G, say GH, meets the load axis at H where Pu =
5600 kN. Similarly, Pu for other sets are determined and these are presented in
Table 10.2, except for the last case when M 40 is used, as this chart has ended
at p = 3.8 per cent.

Step 2: Area of longitudinal steel

The maximum area of steel, 4 per cent of gross area of column =


0.04(550)(550) = 12100 mm2. Provide 12-36 mm diameter bars to have the
actual area of steel = 12214 mm2 > 12100 mm2, as presented in Table 10.2.

Step 3: Factored Pu from direct computation

From Eq,10.4, as in Step 4 of the solution of Part (a) of this question, we


have

Pu = 0.4 fck (Ag – Asc) + 0.67 fy Asc


(3)

Version 2 CE IIT, Kharagpur


Substituting the values of Ag and Asc actually provided, we get the maximum Pu
of the same column when the longitudinal steel is the maximum. For the first
case when Ag = 302500 mm2, Asc = 12214 mm2, fck = 20 N/mm2 and fy = 415
N/mm2, we get Pu = 5718.4 kN. This value along with other four values are
presented in Table 10.2.

Remarks:

Tables 10.1 and 10.2 reveal that two sets of results obtained from charts
of SP-16 and by direct computation methods are in good agreement. However,
values obtained from the chart are marginally different from those obtained by
direct computation both on the higher and lower sides. These differences are
mainly due to personal error (parallax error) while reading the values with eye
estimation from the chart.

Table 10.2 Results of Q.6(b) (Maximum Longitudinal Steel) given the respective
Ag

Concret b = Gross Area of steel (Asc) Pu = Factored load


e grade D concret Require Provide Bars SP-chart Direct
(cm) e area d (cm2) d (cm2) (No. (kN) Computatio
(As) ) n
(cm2) (kN)
M 20 55 3025 121 122.14 12- 5600 5718.4
36
M 25 50 2500 100 101.06 8-36 5200 5208.9
+ 4-
25
M 30 47 2209 88.36 88.97 8-32 5000 5017.8
+ 4-
28
M 35 42.5 1806.25 72.25 73.69 12- 4500 4474.5
28
M 40 40 1600 64 64.46 8-28 Not 4249.2
+ 4- availabl
32 e

Q.7: Design a short, helically reinforced circular column with minimum amount of
longitudinal steel to carry a total factored axial load of 3000 kN with the
same support condition as that of Q.6, using M 25 and Fe 415. Determine
its unsupported length. Compare the results of the dimension and area of
longitudinal steel with those of Q.6(a) when M 25 and Fe 415 are used.

A.7:

Step 1: Diameter of helically reinforced circular column

Version 2 CE IIT, Kharagpur


As per cl. 39.4 of IS 456, applicable for short spiral column, we get from
Eq.10.8

Pu = 1.05(0.4 fck Ac + 0.67 fy Asc) …. (10.8)

Given data are: Pu = 3000 kN, Ac = π /4 (D2)(0.992), Asc = 0.008( π /4) D2, fck = 25
N/mm2 and fy = 415 N/mm2. So, we have

3000(103) = 1.05(12.1444)( π /4)D2

giving D = 547.2 mm and Ag = 235264.2252 mm2. Provide diameter of 550 mm.

Step 2: Area of longitudinal steel

Providing 550 mm diameter, the required Ag has been exceeded. Clause


26.5.3.1b stipulates that the minimum amount of longitudinal bar shall be
determined on the basis of area required and not area provided for any column.
Accordingly, the area of longitudinal steel = 0.008 Ag = 0.008(235264.2252) =
1882.12 mm2. Provide 6-20 mm diameter bars (area = 1885 mm2) as longitudinal
steel, satisfying the minimum number of six bar (cl. 26.5.3.1c of IS 456).

Step 3: Helical reinforcement

Minimum diameter of helical reinforcement is greater of (i) 20/4 or (ii) 6


mm. So, provide 6 mm diameter bars for the helical reinforcement (cl. 26.5.3.2d-
2 of IS 456). The pitch of the helix p is determined from Eq.10.11 as follows:

p ≤ 11.1(Dc - φ sp ) asp fy/(D2 - Dc2 ) fck …. (10.11)

Version 2 CE IIT, Kharagpur


Using Dc = 550 – 40 – 40 = 470 mm, φ sp = 6 mm, asp = 28 mm2, D = 550 mm,
fck = 25 N/mm2 and fy = 415 N/mm2, we get

p ≤ 11.1(470 – 6)(28)(415)/(5502 – 4702)(25) ≤ 29.34 mm

Provide 6 mm diameter bar @ 25 mm c/c as helix. The reinforcement bars are


shown in Fig. 10.22.5. Though use of Eq.10.11 automatically checks the
stipulation of cl. 39.4.1 of IS 456, the same is checked as a ready reference in
Step 4 below.

Step 4: Checking of cl. 39.4.1 of IS 456

Volume of helix in one loop = π (Dc - φ sp ) asp …. (10.9)

Volume of core = ( π /4) Dc2 (p) …. (10.10)

The ratio of Eq.10.9 and Eq.10.10 = 4(Dc - φ sp ) asp/ Dc2 p

= 4(470 – 6)(28)/(470)(470)(25) = 0.009410230874

This ratio should not be less than 0.36(Ag/Ac – 1)(fck/fy)

= 0.36{(D2/ Dc2 ) – 1)} (fck/fy) = 0.008011039177

Hence, the stipulation of cl. 39.4.1 is satisfied.

Step 5: Unsupported length

The unsupported length shall be the minimum of the two obtained from (i)
short column requirement given in cl. 25.1.2 of IS 456 and (ii) minimum
eccentricity requirement given in cls. 25.4 and 39.3 of IS 456. The two values are
calculated separately:

(i) l = le = 12D = 12(550) = 6600 mm

(ii) l/500 + D/30 = 0.05 D gives l = 4583.3 mm

So, the unsupported length of this column = 4.58 m.

Step 6: Comparison of results

Table 10.3 presents the results of required and actual gross areas of
concrete and area of steel bars, dimensions of column and number and diameter
of longitudinal reinforcement of the helically reinforced circular and the square
Version 2 CE IIT, Kharagpur
columns of Q.6(a) when M 20 and Fe 415 are used for the purpose of
comparison.

Table 10.3 Comparison of results of circular and square columns with minimum
longitudinal steel (Pu = 3000 kN, M 25, Fe 415)

Column Gross concrete area Area of steel


shape Required Provided Dimension Required Provided Bar dia.
and (cm2) (cm2) D (cm) (cm2) (cm2) and No.
Q.No. (mm,No.)
Circular 2352.64 2376.78 55 18.82 18.85 6-20
(Q.7)
Square 2500 2500 50 20 20.6 4-20 +
(Q.6(a) 4-16

10.22.7 References
1. Reinforced Concrete Limit State Design, 6th Edition, by Ashok K. Jain,
Nem Chand & Bros, Roorkee, 2002.
2. Limit State Design of Reinforced Concrete, 2nd Edition, by P.C.Varghese,
Prentice-Hall of India Pvt. Ltd., New Delhi, 2002.
3. Advanced Reinforced Concrete Design, by P.C.Varghese, Prentice-Hall of
India Pvt. Ltd., New Delhi, 2001.
4. Reinforced Concrete Design, 2nd Edition, by S.Unnikrishna Pillai and
Devdas Menon, Tata McGraw-Hill Publishing Company Limited, New
Delhi, 2003.
5. Limit State Design of Reinforced Concrete Structures, by P.Dayaratnam,
Oxford & I.B.H. Publishing Company Pvt. Ltd., New Delhi, 2004.
6. Reinforced Concrete Design, 1st Revised Edition, by S.N.Sinha, Tata
McGraw-Hill Publishing Company. New Delhi, 1990.
7. Reinforced Concrete, 6th Edition, by S.K.Mallick and A.P.Gupta, Oxford &
IBH Publishing Co. Pvt. Ltd. New Delhi, 1996.
8. Behaviour, Analysis & Design of Reinforced Concrete Structural Elements,
by I.C.Syal and R.K.Ummat, A.H.Wheeler & Co. Ltd., Allahabad, 1989.
9. Reinforced Concrete Structures, 3rd Edition, by I.C.Syal and A.K.Goel,
A.H.Wheeler & Co. Ltd., Allahabad, 1992.
10. Textbook of R.C.C, by G.S.Birdie and J.S.Birdie, Wiley Eastern Limited,
New Delhi, 1993.
11. Design of Concrete Structures, 13th Edition, by Arthur H. Nilson, David
Darwin and Charles W. Dolan, Tata McGraw-Hill Publishing Company
Limited, New Delhi, 2004.
12. Concrete Technology, by A.M.Neville and J.J.Brooks, ELBS with
Longman, 1994.
13. Properties of Concrete, 4th Edition, 1st Indian reprint, by A.M.Neville,
Longman, 2000.

Version 2 CE IIT, Kharagpur


14. Reinforced Concrete Designer’s Handbook, 10th Edition, by C.E.Reynolds
and J.C.Steedman, E & FN SPON, London, 1997.
15. Indian Standard Plain and Reinforced Concrete – Code of Practice (4th
Revision), IS 456: 2000, BIS, New Delhi.
16. Design Aids for Reinforced Concrete to IS: 456 – 1978, BIS, New Delhi.

10.22.8 Test 22 with Solutions


Maximum Marks = 50, Maximum Time = 30 minutes

Answer all questions carrying equal marks.

TQ.1: Derive the expression of determining the pitch of helix in a short axially
loaded spiral column which satisfies the requirement of IS 456.
(20 marks)

A.TQ.1: See third paragraph onwards up to the end of sec. 10.22.4.

TQ.2: Design a square, short tied column of b = D = 500 mm to carry a total


factored load of 4000 kN using M 20 and Fe 415. Draw the reinforcement
diagram.
(30 marks)

A.TQ.2:

Step 1: Area of longitudinal steel

Assuming p as the percentage of longitudinal steel, we have Ac =


(500)(500)(1 – 0.01p), Asc = (500)(500)(0.01p), fck = 20 N/mm2 and fy = 415
N/mm2. Using these values in Eq.10.4

Pu = 0.4 fck Ac + 0.67 fy Ast …. (10.4)

or 4000000 = 0.4(20)(250000)(1 – 0.01p) + 0.67(415)(250000)(0.01p)

we get p = 2.9624, which gives Asc = (2.9624)(500)(500)/100 = 7406 mm2. Use


4-36 + 4-25 + 4-22 mm diameter bars (4071 + 1963 + 1520) = 7554 mm2 > 7406
mm2 as longitudinal reinforcement.

Version 2 CE IIT, Kharagpur


Step 2: Lateral ties

Diameter of tie is the greater of (i) 36/4 and (ii) 6 mm. Provide 10 mm
diameter lateral ties.

The pitch of the lateral ties is the least of (i) 500 mm, (ii) 16(22) = 352 mm,
and (iii) 300 mm. Provide 10 mm diameter @ 300 mm c/c. The reinforcement
bars are shown in Fig.10.22.6. It is evident that the centre to centre distance
between two corner bars = 364 mm is less than 48 times the diameter of lateral
ties = 480 mm (Fig.9 of cl. 26.5.3.2b-2 of IS 456).

10.22.9 Summary of this Lesson


This lesson illustrates the additional assumptions made regarding the
strengths of concrete and steel for the design of short tied and helically
reinforced columns subjected to axial loads as per IS 456. The governing
equations for determining the areas of cross sections of concrete and longitudinal
steel are derived and explained. The equation for determining the pitch of the
helix for circular columns is derived. Several numerical examples are solved to
illustrate the applications of the derived equations and use of charts of SP-16 for
the design of both tied and helically reinforced columns. The results of the same
problem by direct computation are compared with those obtained by employing
the charts of SP-16 are compared. The differences of results, if any, are
discussed. Understanding the illustrative examples and solving the practice and
test problems will explain the applications of the equation and use of charts of
SP-16 for designing these two types of columns.

Version 2 CE IIT, Kharagpur


Module
10
Compression Members
Version 2 CE IIT, Kharagpur
Lesson
23
Short Compression
Members under
Axial Load with
Uniaxial Bending
Version 2 CE IIT, Kharagpur
Instructional Objectives:
At the end of this lesson, the student should be able to:

• draw the strain profiles for different locations of the depth of the neutral
axis,

• explain the behaviour of such columns for any one of the strain profiles,

• name and identify the three modes of failure of such columns,

• explain the interaction diagram and divide into the three regions indicating
three modes of failure,

• identify the three modes of failure from the depth of neutral axis,

• identify the three modes of failure from the eccentricity of the axial load,

• determine the area of compressive stress block, distance of the centroid of


the area of the compressive stress block from the highly compressed edge
when the neutral axis is within and outside the cross-section of the
column,

• determine the compressive stress of concrete and tensile/compressive


stress of longitudinal steel for any location of the neutral axis within or
outside the cross-section of the column,

• write the two equations of equilibrium,

• explain the need to recast the equations in non-dimensional form for their
use in the design of such columns.

10.23.1 Introduction
Short reinforced concrete columns under axial load with uniaxial bending
behave in a different manner than when it is subjected to axial load, though
columns subjected to axial load can also carry some moment that may appear
during construction or otherwise. The behaviour of such columns and the three
modes of failure are illustrated in this lesson. It is explained that the moment M,
equivalent to the load P with eccentricity e (= M/P), will act in an interactive
manner. A particular column with specific amount of longitudinal steel, therefore,
can carry either a purely axial load Po (when M = 0), a purely moment Mo (when
P = 0) or several pairs of P and M in an interactive manner. Hence, the needed
interaction diagram of columns, which is a plot of P versus M, is explained

Version 2 CE IIT, Kharagpur


discussing different positions of neutral axis, either outside or within the cross-
section of the column.

Depending on the position of the neutral axis, the column may or may not
have tensile stress to be taken by longitudinal steel. In the compression region
however, longitudinal steel will carry the compression load along with the
concrete as in the case of axially loaded column.

10.23.2 Behaviour of Short Columns under Axial Load and


Uniaxial Moment
Normally, the side columns of a grid of beams and columns are subjected
to axial load P and uniaxial moment Mx causing bending about the major axis xx,
hereafter will be written as M. The moment M can be made equivalent to the
axial load P acting at an eccentricity of e (= M/P). Let us consider a symmetrically
reinforced short rectangular column subjected to axial load Pu at an eccentricity
of e to have Mu causing failure of the column.

Figure 10.21.11b of Lesson 21 presents two strain profiles IN and EF. For
the strain profile IN, the depth of the neutral axis kD is less than D, i.e., neutral
axis is within the section resulting the maximum compressive strain of 0.0035 on
the right edge and tensile strains on the left of the neutral axis forming cracks.
This column is in a state of collapse for the axial force Pu and moment Mu for
which IN is the strain profile. Reducing the eccentricity of the load Pu to zero, we
get the other strain profile EF resulting in the constant compressive strain of
0.002, which also is another collapse load. This axial load Pu is different from the
other one, i.e., a pair of Pu and Mu, for which IN is the profile. For the strain profile
EF, the neutral axis is at infinity (k = α ).

Figure 10.21.11c of Lesson 21 presents the strain profile EF with two


more strain profiles IH and JK intersecting at the fulcrum point V. The strain
profile IH has the neutral axis depth kD = D, while other strain profile JK has kD >
D. The load and its eccentricity for the strain profile IH are such that the
maximum compressive strain reaches 0.0035 at the right edge causing collapse
of the column, though the strains throughout the depth is compressive and zero
at the left edge. The strain profile JK has the maximum compressive strain at the
right edge between 0.002 and 0.0035 and the minimum compressive strain at the
left edge. This strain profile JK also causes collapse of the column since the
maximum compressive strain at the right edge is a limiting strain satisfying
assumption (ii) of sec. 10.21.10 of Lesson 21.

The four strain profiles, IN, EF, JK and IH of Figs.10.21.11b and c,


separately cause collapse of the same column when subjected to four different
pairs of Pu and Mu. This shows that the column may collapse either due to a
uniform constant strain throughout (= 0.002 by EF) or due to the maximum
compressive strain at the right edge satisfying assumption (ii) of sec.10.21.10 of

Version 2 CE IIT, Kharagpur


Lesson 21 irrespective of the strain at the left edge (zero for IH and tensile for
IN). The positions of the neutral axis and the eccentricities of the load are widely
varying as follows:

(i) For the strain profile EF, kD is infinity and the eccentricity of the load is
zero.

(ii) For the strain profile JK, kD is outside the section (D < kD < α ), with
appropriate eccentricity having compressive strain in the section.

(iii) For the strain profile IH, kD is just at the left edge of the section (kD =
D), with appropriate eccentricity having zero and 0.0035 compressive
strains at the left and right edges, respectively.

(iv) For the strain profile IN, kD is within the section (kD < D), with
appropriate eccentricity having tensile strains on the left of the neutral
axis and 0.0035 compressive strain at the right edge.

Version 2 CE IIT, Kharagpur


It is evident that gradual increase of the eccentricity of the load Pu from zero is
changing the strain profiles from EF to JK, IH and then to IN. Therefore, we can
accept that if we increase the eccentricity of the load to infinity, there will be only
Mu acting on the column. Designating by Po as the load that causes collapse of
the column when acting alone and Mo as the moment that also causes collapse
when acting alone, we mark them in Fig.10.23.1 in the vertical and horizontal
axes. These two points are the extreme points on the plot of Pu versus Mu, any
point on which is a pair of Pu and Mu (of different magnitudes) that will cause
collapse of the same column having the neutral axis either outside or within the
column.

The plot of Pu versus Mu of Fig.10.23.1 is designated as interaction


diagram since any point on the diagram gives a pair of values of Pu and Mu
causing collapse of the same column in an interactive manner. Following the
same logic, several alternative column sections with appropriate longitudinal
steel bars are also possible for a particular pair of Pu and Mu. Accordingly for the
purpose of designing the column, it is essential to understand the different modes
of failure of columns, as given in the next section.

10.23.3 Modes of Failure of Columns


The two distinct categories of the location of neutral axis, mentioned in the
last section, clearly indicate the two types of failure modes: (i) compression
failure, when the neutral axis is outside the section, causing compression
throughout the section, and (ii) tension failure, when the neutral axis is within the
section developing tensile strain on the left of the neutral axis. Before taking up
these two failure modes, let us discuss about the third mode of failure i.e., the
balanced failure.

(A) Balanced failure

Under this mode of failure, yielding of outer most row of longitudinal steel
near the left edge occurs simultaneously with the attainment of maximum
compressive strain of 0.0035 in concrete at the right edge of the column. As a
result, yielding of longitudinal steel at the outermost row near the left edge and
crushing of concrete at the right edge occur simultaneously. The different yielding
strains of steel are determined from the following:

(i) For mild steel (Fe 250): ε y = 0.87fy/Es


(10.12)

(ii) For cold worked deformed bars: ε y = 0.87fy/Es + 0.002


(10.13)

Version 2 CE IIT, Kharagpur


The corresponding numerical values are 0.00109, 0.0038 and 0.00417 for Fe
250, Fe 415 and Fe 500, respectively. Such a strain profile is known a balanced
strain profile which is shown by the strain profile IQ in Fig.10.23.2b with a number
5. This number is shown in Fig.10.23.1 lying on the interaction diagram causing

Version 2 CE IIT, Kharagpur


collapse of the column. The depth of the neutral axis is designated as kbD and
shown in Fig.10.23.2b. The balanced strain profile IQ in Fig.10.23.2b also shows
the strain ε y whose numerical value would change depending on the grade of
steel as mentioned earlier. It is also important to observe that this balanced
profile IQ does not pass trough the fulcrum point V in Fig.10.23.2b, while other
profiles 1, 2 and 3 i.e., EF, LM and IN pass through the fulcrum point V as none
of them produce tensile strain any where in the section of the column. The
neutral axis depth for the balanced strain profile IQ is less than D, while the same
for the other three are either equal to or more than D.

To have the balanced strain profile IQ causing balanced failure of the


column, the required load and moment are designated as Pb and Mb ,
respectively and shown in Fig.10.23.1 as the coordinates of point 5. The
corresponding eccentricity of the load Pb is defined by the notation eb (= Mb/Pb).
The four parameters of the balanced failure are, therefore, Pb, Mb, eb and kb (the
coefficient of the neutral axis depth kbD).

(B) Compression failure

Compression failure of the column occurs when the eccentricity of the load
Pu is less than that of balanced eccentricity (e < eb) and the depth of the neutral
axis is more than that of balanced failure. It is evident from Fig.10.23.2b that
these strain profiles may develop tensile strain on the left of the neutral axis till
kD = D. All these strain profiles having 1 > k > kb will not pass through the
fulcrum point V. Neither the tensile strain of the outermost row of steel on the left
of the neutral axis reaches ε y .

On the other hand, all strain profiles having kD greater than D pass
through the fulcrum point V and cause compression failure (Fig.10.23.2b). The
loads causing compression failure are higher than the balanced load Pb having
the respective eccentricities less than that of the load of balanced failure. The
extreme strain profile is EF marked by 1 in Fig.10.23.2b. Some of these points
causing compression failure are shown in Fig.10.23.1 as 1, 2, 3 and 4 having k >
kb, either within or outside the section.

Three such strain profiles are of interest and need further elaboration. One
of them is the strain profile IH (Fig.10.23.2b) marked by point 3 (Fig.10.23.1) for
which kD = D. This strain profile develops compressive strain in the section with
zero strain at the left edge and 0.0035 in the right edge as explained in sec.
10.23.2. Denoting the depth of the neutral axis by D and eccentricity of the load
for this profile by eD, we observe that the other strain profiles LM and EF
(Fig.10.23.2b), marked by 2 and 1 in Fig.10.23.1, have the respective kD > D and
e < eD.

Version 2 CE IIT, Kharagpur


The second strain profile is EF (Fig.10.23.2b) marked by point 1 in
Fig.10.23.1 is for the maximum capacity of the column to carry the axial load Po
when eccentricity is zero and for which moment is zero and the neutral axis is at
infinity. This strain profile has also been discussed earlier in sec.10.23.2.

The third important strain profile LM, shown in Fig.10.23.2b and by point 2
in Figs.10.23.1 and 2, is also due to another pair of collapse Pm and Mm, having
the capacity to accommodate the minimum eccentricity of the load, which hardly
can be avoided in practical construction or for other reasons. The load Pm , as
seen from Fig.10.23.1, is less than Po and the column can carry Pm and Mm in an
interactive mode to cause collapse. Hence, a column having the capacity to carry
the truly concentric load Po (when M = 0) shall not be allowed in the design.
Instead, its maximum load shall be restricted up to Pm (< Po) along with Mm (due
to minimum eccentricity). Accordingly, the actual interaction diagram to be used
for the purpose of the design shall terminate with a horizontal line 22’ at point 2 of
Fig.10.23.1. Point 2 on the interaction diagram has the capacity of Pm with Mm
having eccentricity of em (= Mm/Pm) and the depth of the neutral axis is >> D
(Fig.10.23.2b).

It is thus seen that from points 1 to 5 (i.e., from compression failure to


balanced failure) of the interaction diagram of Fig.10.23.1, the loads are
gradually decreasing and the moments are correspondingly increasing. The
eccentricities of the successive loads are also increasing and the depths of
neutral axis are decreasing from infinity to finite but outside and then within the
section up to kbD at balanced failure (point 5). Moreover, this region of
compression failure can be subdivided into two zones: (i) zone from point 1 to
point 2, where the eccentricity of the load is less than the minimum eccentricity
that should be considered in the actual design as specified in IS 456, and (ii)
zone from point 2 to point 5, where the eccentricity of the load is equal to or more
than the minimum that is specified in IS 456. It has been mentioned also that the
first zone from point 1 to point 2 should be avoided in the design of column.

(C) Tension failure

Tension failure occurs when the eccentricity of the load is greater than the
balanced eccentricity eb. The depth of the neutral axis is less than that of the
balanced failure. The longitudinal steel in the outermost row on the left of the
neutral axis yields first. Gradually, with the increase of tensile strain, longitudinal
steel of inner rows, if provided, starts yielding till the compressive strain reaches
0.0035 at the right edge. The line IR of Fig.10.23.2b represents such a profile for
which some of the inner rows of steel bars have yielded and compressive strain
has reached 0.0035 at the right edge. The depth of the neutral axis is designated
by (kminD).

Version 2 CE IIT, Kharagpur


It is interesting to note that in this region of the interaction diagram (from 5
to 6 in Fig.10.23.1), both the load and the moment are found to decrease till point
6 when the column fails due to Mo acting alone. This important behaviour is
explained below starting from the failure of the column due to Mo alone at point 6
of Fig.10.23.1.

At point 6, let us consider that the column is loaded in simple bending to


the point (when M = Mo) at which yielding of the tension steel begins. Addition of
some axial compressive load P at this stage will reduce the previous tensile
stress of steel to a value less than its yield strength. As a result, it can carry
additional moment. This increase of moment carrying capacity with the increase
of load shall continue till the combined stress in steel due to additional axial load
and increased moment reaches the yield strength.

10.23.4 Interaction Diagram


It is now understood that a reinforced concrete column with specified
amount of longitudinal steel has different carrying capacities of a pair of Pu and
Mu before its collapse depending on the eccentricity of the load. Figure 10.23.1
represents one such interaction diagram giving the carrying capacities ranging
from Po with zero eccentricity on the vertical axis to Mo (pure bending) on the
horizontal axis. The vertical axis corresponds to load with zero eccentricity while
the horizontal axis represents infinite value of eccentricity. A radial line joining the
origin O of Fig.10.23.1 to point 2 represents the load having the minimum
eccentricity. In fact, any radial line represents a particular eccentricity of the load.
Any point on the interaction diagram gives a unique pair of Pu and Mu that causes
the state of incipient failure. The interaction diagram has three distinct zones of
failure: (i) from point 1 to just before point 5 is the zone of compression failure, (ii)
point 5 is the balanced failure and (iii) from point 5 to point 6 is the zone of
tension failure. In the compression failure zone, small eccentricities produce
failure of concrete in compression, while large eccentricities cause failure
triggered by yielding of tension steel. In between, point 5 is the critical point at
which both the failures of concrete in compression and steel in yielding occur
simultaneously.

The interaction diagram further reveals that as the axial force Pu becomes
larger the section can carry smaller Mu before failing in the compression zone.
The reverse is the case in the tension zone, where the moment carrying capacity
Mu increases with the increase of axial load Pu. In the compression failure zone,
the failure occurs due to over straining of concrete. The large axial force
produces high compressive strain of concrete keeping smaller margin available
for additional compressive strain line to bending. On the other hand, in the
tension failure zone, yielding of steel initiates failure. This tensile yield stress
reduces with the additional compressive stress due to additional axial load. As a

Version 2 CE IIT, Kharagpur


result, further moment can be applied till the combined stress of steel due to axial
force and increased moment reaches the yield strength.

Therefore, the design of a column with given Pu and Mu should be done


following the three steps, as given below:

(i) Selection of a trial section with assumed longitudinal steel,

(ii) Construction of the interaction diagram of the selected trial column


section by successive choices of the neutral axis depth from infinity
(pure axial load) to a very small value (to be found by trial to get P = 0
for pure bending),

(iii) Checking of the given Pu and Mu, if they are within the diagram.

We will discuss later whether the above procedure should be followed or


not. Let us first understand the corresponding compressive stress blocks of
concrete for the two distinct cases of the depth of the neutral axis: (i) outside the
cross-section and (ii) within the cross-section in the following sections.

10.23.5 Compressive Stress Block of Concrete when the


Neutral Axis Lies Outside the Section

Version 2 CE IIT, Kharagpur


Figure 10.23.3c presents the stress block for a typical strain profile JK
having neutral axis depth kD outside the section (k > 1). The strain profile JK in
Fig.10.23.3b shows that up to a distance of 3D/7 from the right edge (point AO),
the compressive strain is ≥ 0.002 and, therefore, the compressive stress shall
remain constant at 0.446fck. The remaining part of the column section of length
4D/7, i.e., up to the left edge, has reducing compressive strains (but not zero).
The stress block is, therefore, parabolic from AO to H which becomes zero at U
(outside the section). The area of the compressive stress block shall be obtained
subtracting the parabolic area between AO to H from the rectangular area
between G and H. To establish the expression of this area, it is essential to know
the equation of the parabola between AO and U, whose origin is at AO. The
positive coordinates of X and Y are measured from the point AO upwards and to
the left, respectively. Let us assume that the general equation of the parabola as

Version 2 CE IIT, Kharagpur


X = aY2 + bY + c
(10.14)

The values of a, b and c are obtained as follows:

(i) At Y = 0, X = 0, at the origin: gives c = 0

(ii) At Y = 0, dX/dY = 0, at the origin: gives b = 0

(iii) At Y = (kD – 3D/7), i.e., at point U, X = 0.446fck: gives a = 0.446fck/D2(k-


2
3/7) .

Therefore, the equation of the parabola is:

X = {0.446fck/D2(k – 3/7)2}Y2
(10.15)

The value of X at the point H (left edge of the column), g is now


determined from Eq.10.15 when Y = 4D/7, which gives

g = 0.446 fck {4/(7k – 3)}2


(10.16)

Hence, the area of the compressive stress block = 0.446 fck D [1 – (4/21){4/(7k –
3)}2]
= C1 fck D
(10.17)

where C1 = 0.446[1 – (4/21){4/(7k – 3)}2]


(10.18)

Equation 10.17 is useful to determine the area of the stress block for any
value of k > 1 (neutral axis outside the section) by substituting the value of C1
from Eq.10.18. The symbol C1 is designated as the coefficient for the area of the
stress block.

The position of the centroid of the compressive stress block is obtained by


dividing the moment of the stress block about the right edge by the area of the
stress block. The moment of the stress block is obtained by subtracting the
moment of the parabolic part between AO and H about the right edge from the
moment of the rectangular stress block of full depth D about the right edge. The
expression of the moment of the stress block about the right edge is:

0.446 fck D(D/2) – (1/3)(4D/7) 0.446 fck {4/(7k – 3)}2 {3D/7 + (3/4)(4D/7)}

Version 2 CE IIT, Kharagpur


= 0.446 fck D2 [(1/2) – (8/49){4/(7k – 3)}2]
(10.19)

Dividing Eq.10.19 by Eq.10.17, we get the distance of the centroid from


the right edge is:

D[(1/2) – (8/49){4/(7k – 3)}2]/[1 – (4/21){4/(7k – 3)}2]


(10.20)

= C2 D
(10.21)

where C2 is the coefficient for the distance of the centroid of the compressive
stress block of concrete measured from the right edge and is:

C2 = [(1/2) – (8/49){4/(7k – 3)}2]/[1 – (4/21){4/(7k – 3)}2]


(10.22)

Table 10.4 presents the values of C1 and C2 for different values of k greater than
1, as given in Table H of SP-16. For a specific depth of the neutral axis, k is
known. Using the corresponding values of C1 and C2 from Table 10.4, area of the
stress block of concrete and the distance of centroid from the right edge are
determined from Eqs.10.17 and 10.21, respectively.

Table 10.4 Stress block parameters C1 and C2 when the neutral axis is outside
the section

K C1 C2
1.00 0.361 0.416
1.05 0.374 0.432
1.10 0.384 0.443
1.20 0.399 0.458
1.30 0.409 0.468
1.40 0.417 0.475
1.50 0.422 0.480
2.00 0.435 0.491
2.50 0.440 0.495
3.00 0.442 0.497
4.00 0.444 0.499

It is worth mentioning that the area of the stress block is 0.446fckD and the
distance of the centroid from the right edge is 0.5D, when k is infinite. Values of
C1 and C2 at k = 4 are very close to those when k = ∞ . In fact, for the practical

Version 2 CE IIT, Kharagpur


interaction diagrams, it is generally adequate to consider values of k up to about
1.2.

10.23.6 Determination of Compressive Stress Anywhere in


the Section when the Neutral Axis Lies outside the
Section
The compressive stress of concrete at any point between G and AO of
Fig.10.23.3c is constant at 0.446fck as the strain in this zone is equal to or greater
than 0.002. So, we can write

fc = 0.446fck if 0.002 ≤ ε c ≤ 0.0035


(10.23)

However, compressive stress of concrete between AO and H is to be


determined using the equation of parabola. Let us determine the concrete stress
fc at a distance of Y from the origin AO. From Fig.10.23.3c, we have

fc = 0.446 fck - gc
(10.24)

where gc is as shown in Fig.10.23.3c and obtained from Eq.10.15. Thus, we get

fc = 0.446 fck – {0.446 fck/D2(k – 3/7)2}Y2

or fc = 0.446 fck {1 – Y2/(kD – 3D/7)2}


(10.25)

Designating the strain of concrete at this point by ε c (Fig.10.23.3b), we


have from similar triangles

ε c /0.002 = 1 – Y/(kD – 3D/7)

which gives

Y = {1 – ( ε /0.002)}(kD – 3D/7)
(10.26)

Substituting the value of Y from Eq.10.26 in Eq.10.25, we have

fc = 0.446 fck [2( ε c /0.002) - ( ε c /0.002)2], if 0 ≤ ε c < 0.002


(10.27)

Version 2 CE IIT, Kharagpur


10.23.7 Compressive Stress Block of Concrete when the
Neutral Axis is within the Section

Figure 10.23.4c presents the stress block for a typical strain profile IN
having neutral axis depth = kD within the section (k < 1). The strain profile IN in
Fig.10.23.4b shows that from a to AO, i.e., up to a distance of 3kD/7 from the
right edge, the compressive strain is ≥ 0.002 and, therefore, the compressive
Version 2 CE IIT, Kharagpur
stress shall remain constant at 0.446fck. From AO to U, i.e., for a distance of
4kD/7, the strain is reducing from 0.002 to zero and the stress in this zone is
parabolic as shown in Fig.10.23.4c. The area of the stress block shall be
obtained subtracting the parabolic area between AO and U from the total
rectangular area between G and U. As in the case when the neutral axis is
outside the section (sec.10.23.5), we have to establish the equation of the
parabola with AO as the origin and the positive coordinates X and Y are
measured from the point AO upwards for X and from the point AO to the left for
Y, as shown in Fig.10.23.4c. Proceeding in the same manner as in sec.10.23.5
and assuming the same equation of the parabola as in Eq.10.14, the values of a,
b and c are obtained as:

(i) At Y = 0, X = 0, at the origin: gives c = 0

(ii) At Y = 0, dX/dY = 0, at the origin: gives b = 0

(iii) At U, (i.e., at Y = 4kD/7), X = 0.446 fck: gives a = 0.446 fck/(4kD/7)2.

Therefore, the equation of the parabola OR is:

X = {0.446 fck/(4kD/7)2}Y2
(10.28)

The area of the stress block = 0.446 fck kD – (1/3) 0.446 fck (4kD/7) = 0.36 fck
kD, the same as obtained earlier in Eq.3.9 of Lesson 4 for flexural members.
Similarly, the distance of the centroid can be obtained by dividing the moment of
area of stress block about the right edge by the area of the stress block. The
result is the same as in Eq.3.12 for the flexural members. Therefore, we have

Area of the stress block = 0.36 fck kD


(10.29)

The distance of the centroid of the stress block from the right edge = 0.42kD
(10.30)

Thus, the values of C1 and C2 of Eqs.10.17 and 10.21, respectively, are


0.36 and 0.42 when the neutral axis is within the section. It is to be noted that the
coefficients C1 and C2 are multiplied by Dfck and D, respectively when the neutral
axis is outside the section. However, they are to be multiplied here, when the
neutral axis is within the section, by kDfck and kD, respectively.

It is further to note that though the expressions of the area of stress block
and the distance of the centroid of the stress block from the right edge are the
same as those for the flexural members, the important restriction of the maximum
depth of the neutral axis xumax in the flexural members is not applicable in case of
column. By this restriction, the compression failure of the flexural members is

Version 2 CE IIT, Kharagpur


avoided. In case of columns, compression failure is one of the three modes of
failure.

10.23.8 Determination of Compressive Stress Anywhere in


the Compressive Zone when the Neutral Axis is within
the Section
The compressive stress at any point between G and AO of Fig.10.23.4c is
constant at 0.446fck as the strain in this zone is equal to or greater than 0.002.
So, we can write
fc = 0.446 fck if 0.002 ≤ ε c ≤ 0.0035 ….
(10.23)

However, the compressive stress between AO and U is to be determined from


the equation of the parabola. Let us determine the compressive stress fci at a
distance of Y from the origin AO. From Fig.10.23.4c, we have

fc = 0.446 fck - gc
(10.31)

where gc as shown in Fig.10.23.4c, is obtained from Eq.10.28. Thus, we get,

fc = {0.446 fck – 0.446 fck (4kD/7)2}Y2


(10.32)

Designating the strain of concrete at this point by ε c (Fig.10.23.4b), we


have from similar triangles

ε c /0.002 = 1 – Y/(4kD/7), which gives

Y = {1 - ε c /0.002}(4kD/7)
(10.33)

Substituting the value of Y from Eq.10.33 in Eq.10.32, we get the same equation,
Eq.10.27 of sec.10.23.6, when the neutral axis is outside the section. Therefore,

fc = 0.446 fck [2( ε c /0.002) – ( ε c /0.002)2] …. (10.26)

From the point U to the left edge H of the cross-section of the column, the
compressive stress is zero. Thus, we have

fc = 0 if ε c ≤ 0

Version 2 CE IIT, Kharagpur


fc = 0.446 fck if ε c ≥ 0.002

fc = 0.446 fck{2( ε c /0.002) – ( ε c /0.002)2}, if 0 ≤ ε c < 0.002


(10.34)

10.23.9 Tensile and Compressive Stresses of Longitudinal


Steel
Stresses are compressive in all the six rows (A1 to A6 of Figs.10.23.3a
and c) of longitudinal steel provided in the column when the neutral axis depth
kD ≥ D. However, they are tensile on the left side of the neutral axis and
compressive on the right side of the neutral axis (Figs.10.23.4a and c) when kD <
D. These compressive or tensile stresses of longitudinal steel shall be calculated
from the strain ε si at that position of the steel which is obtained from the strain
profile considered for the purpose.

It should be remembered that the linear strain profiles are based on the
assumption that plane sections remain plane. Moreover, at the location of steel in
a particular row, the strain of steel ε si shall be the same as that in the adjacent
concrete ε ci . Thus, the strain of longitudinal steel can be calculated from the
particular strain profile if the neutral axis is within or outside the cross-section of
the column.

The corresponding stresses f si of longitudinal steel are determined from


the strain ε si (which is the same as that of ε ci in the adjacent concrete) from the
respective stress-strain diagrams of mild steel (Fig.1.2.3 of Lesson 2) and High
Yield Strength Deformed bars (Fig.1.2.4 of Lesson2). The values are
summarized in Table 10.5 below as presented in Table A of SP-16.

Table 10.5 Values of compressive or tensile f si from known values of ε si of


longitudinal steel (Fe 250, Fe 415 and Fe 500)

Fe 250 Fe 415 Fe 500


Strain Stress Strain Stress Strain Stress
ε si (N/mm2) ε si (N/mm2) ε si (N/mm2)
f si f si f si
< 0.00109 ε si (Es) < 0.00144 ε si (Es) < 0.00174 ε si (Es)
≥ 0.00109 217.5 0.00144 288.7 0.00174 347.8
(= 0.87 fy)
0.00163 306.7 0.00195 369.6
0.00192 324.8 0.00226 391.3
0.00241 342.8 0.00277 413.0

Version 2 CE IIT, Kharagpur


0.00276 351.8 0.00312 423.9
0.00380 360.9 0.00417 434.8

Notes: 1. Linear interpolation shall be done for intermediate values.


2. Strain at initial yield = fy/Es
3. Strain at final yield = fy/Es + 0.002

10.23.10 Governing Equations


A column subjected to Pu and Mu (= Pu e) shall satisfy the two equations of
equilibrium, viz., ∑V = 0 and ∑M = 0, taking moment of vertical forces about the
centroidal axis of the column. The two governing equation are, therefore,

Pu = Cc + Cs
(10.35)

Mu = Cc (appropriate lever arm) + Cs (appropriate lever arm)


(10.36)

where Cc = Force due to concrete in compression

Cs = Force due to steel either in compression when kD ≥ D or in tension


and compression when kD < D

However, two points are to be remembered while expanding the equation


∑V = 0. The first is that while computing the force of steel in compression, the
force of concrete that is not available at the location of longitudinal steel has to
be subtracted. The second point is that the total force of steel shall consist of the
summation of forces in every row of steel having different stresses depending on
the respective distances from the centroidal axis. These two points are also to be
considered while expanding the other equation ∑M = 0. Moreover, negative sign
should be used for the tensile force of steel on the left of the neutral axis when
kD < D.

It is now possible to draw the interaction diagram of a trial section for the
given values of Pu and Mu following the three steps mentioned in sec.10.23.4.
However, such an attempt should be avoided for the reason explained below.

It has been mentioned in sec.10.23.2 that any point on the interaction


diagram gives a pair of values of Pu and Mu causing collapse. On the other hand,
it is also true that for the given Pu and Mu, several sections are possible. Drawing
of interaction diagrams for all the trial sections is time consuming. Therefore, it is
necessary to recast the interaction diagram selecting appropriate non-
dimensional parameters instead of Pu versus Mu as has been explained in this
lesson. Non-dimensional interaction diagram has the advantage of selecting
alternative sections quickly for a given pair of Pu and Mu. It is worth mentioning

Version 2 CE IIT, Kharagpur


that all the aspects of the behaviour of column and the modes of failure shall
remain valid in constructing the more versatile non-dimensional interaction
diagram, which is taken up in Lesson 24.

10.23.11 Practice Questions and Problems with Answers


Q.1: Draw four typical strain profiles of a short, rectangular and symmetrically
reinforced concrete column causing collapse subjected to different pairs of
Pu and Mu when the depths of the neutral axis are (i) less than the depth of
column D, (ii) equal to the depth of column D, (iii) D < kD < ∞ and (iv) kD
= ∞ . Explain the behaviour of column for each of the four strain profiles.

A.1: See sec. 10.23.2.

Q.2: Name and explain the three modes of failures of short, rectangular and
symmetrically reinforced concrete columns subjected to axial load Pu
uniaxial moment Mu.

A.2: See sec.10.23.3.

Q.3: Draw a typical interaction diagram, and explain the three zones
representing three modes of failure of a short, rectangular and
symmetrically reinforced concrete column subjected to axial load Pu and
uniaxial moment Mu.

A.3: See sec.10.23.4.

Q.4: (a) Draw the compressive stress block of concrete of a short, rectangular
and symmetrically reinforced concrete column subjected to axial load Pu
and uniaxial moment Mu, when the neutral axis lies outside the section.

(b) Derive expressions of determining the area of the compressive stress


block of concrete and distance of the centroid of the compressive stress
block from the highly compressed right edge for a column of Q4(a).

A.4: See sec.10.23.5.

Q.5: Derive expression of determining the stresses anywhere within the section
of a column of Q4.

A.5: See sec.10.23.6.

Q.6: (a) Draw the compressive stress block of concrete of a short, rectangular
and symmetrically reinforced concrete column subjected to axial load
Pu and uniaxial moment Mu, when the neutral axis is within the section.

Version 2 CE IIT, Kharagpur


(b) Derive expressions of determining the area of the compressive stress
block of concrete and distance of the centroid of the compressive
stress block from the compressed right edge for a column of Q6(a).

A.6: See sec.10.23.7.

Q.7: Derive expression of determining the compressive stress in the


compression zone of a column of Q6.

A.7: See sec.10.23.8.

Q.8: Explain the principle of determining the stresses (both tensile and
compressive) of longitudinal steel of a short, rectangular and
symmetrically reinforced concrete column subjected to axial load Pu and
uniaxial moment Mu.

A.8: See sec.10.23.9.

Q.9: (a) Write the governing equations of equilibrium of a short, rectangular and
symmetrically reinforced concrete column subjected to axial load Pu
and uniaxial moment Mu.

(b) Would you use the equations of equilibrium for the design of a short,
rectangular and symmetrically reinforced concrete column for a given
pair of Pu and Mu? Justify your answer.

A.9: See sec.10.23.10.

10.23.12 References
1. Reinforced Concrete Limit State Design, 6th Edition, by Ashok K. Jain,
Nem Chand & Bros, Roorkee, 2002.
2. Limit State Design of Reinforced Concrete, 2nd Edition, by P.C.Varghese,
Prentice-Hall of India Pvt. Ltd., New Delhi, 2002.
3. Advanced Reinforced Concrete Design, by P.C.Varghese, Prentice-Hall of
India Pvt. Ltd., New Delhi, 2001.
4. Reinforced Concrete Design, 2nd Edition, by S.Unnikrishna Pillai and
Devdas Menon, Tata McGraw-Hill Publishing Company Limited, New
Delhi, 2003.
5. Limit State Design of Reinforced Concrete Structures, by P.Dayaratnam,
Oxford & I.B.H. Publishing Company Pvt. Ltd., New Delhi, 2004.
6. Reinforced Concrete Design, 1st Revised Edition, by S.N.Sinha, Tata
McGraw-Hill Publishing Company. New Delhi, 1990.
7. Reinforced Concrete, 6th Edition, by S.K.Mallick and A.P.Gupta, Oxford &
IBH Publishing Co. Pvt. Ltd. New Delhi, 1996.

Version 2 CE IIT, Kharagpur


8. Behaviour, Analysis & Design of Reinforced Concrete Structural Elements,
by I.C.Syal and R.K.Ummat, A.H.Wheeler & Co. Ltd., Allahabad, 1989.
9. Reinforced Concrete Structures, 3rd Edition, by I.C.Syal and A.K.Goel,
A.H.Wheeler & Co. Ltd., Allahabad, 1992.
10. Textbook of R.C.C, by G.S.Birdie and J.S.Birdie, Wiley Eastern Limited,
New Delhi, 1993.
11. Design of Concrete Structures, 13th Edition, by Arthur H. Nilson, David
Darwin and Charles W. Dolan, Tata McGraw-Hill Publishing Company
Limited, New Delhi, 2004.
12. Concrete Technology, by A.M.Neville and J.J.Brooks, ELBS with
Longman, 1994.
13. Properties of Concrete, 4th Edition, 1st Indian reprint, by A.M.Neville,
Longman, 2000.
14. Reinforced Concrete Designer’s Handbook, 10th Edition, by C.E.Reynolds
and J.C.Steedman, E & FN SPON, London, 1997.
15. Indian Standard Plain and Reinforced Concrete – Code of Practice (4th
Revision), IS 456: 2000, BIS, New Delhi.
16. Design Aids for Reinforced Concrete to IS: 456 – 1978, BIS, New Delhi.

10.23.13 Test 23 with Solutions


Maximum Marks = 50, Maximum Time = 30 minutes

Answer all questions.

TQ.1: Each of the following statements has four possible answers. Choose the
correct answer (2
x 5 = 20 marks)

(a) The designed axial load of a short column has the theoretical carrying
capacity before it collapses

(i) P = Po only as obtained from the interaction diagram on the vertical


axis.

(ii) P = Designed axial load with the code stipulated minimum eccentricity
only.

(iii) A pair of Pb and Mb only.

(iv) All of the above.

A.TQ.1a: (iv)

(b) A short column in compression failure due to an axial load Pu and


uniaxial moment Mu may have

Version 2 CE IIT, Kharagpur


(i) kD = 0 and e = 0

(ii) kD = ∞ and e = 0

(iii) kD = 0 and e = ∞

(iv) kD = ∞ and e = ∞

A.TQ.1b: (ii)

(c) The fulcrum of the strain profile of a short column is a point through
which

(i) The strain profiles causing compression failure will pass.

(ii) The strain profile causing balanced failure will pass.

(iii) The strain profiles having no tension and causing compression failure
will pass.

(iv) The strain profiles causing tension failure will pass.

A.TQ.1c: (iii)

(d) The maximum compressive strain of concrete in balanced failure of a


short column subjected to Pb and Mb is

(i) 0.0035

(ii) 0.0035 minus 0.75 times the tensile strain of steel

(iii) 0.002

(iv) None of the above

A.TQ.1d: (i)

TQ.2: (a) Draw the compressive stress block of concrete of a short, rectangular
and symmetrically reinforced concrete column subjected to axial load
Pu and uniaxial moment Mu, when the neutral axis is within the section.

(b) Derive expressions of determining the area of the compressive stress


block of concrete and distance of the centroid of the compressive
stress block from the compressed right edge for a column of TQ.2 (a).
(10 + 20 = 30)

Version 2 CE IIT, Kharagpur


A.TQ.2: See sec.10.23.7.

10.23.14 Summary of this Lesson


Illustrating the behaviour of short, rectangular and symmetrically
reinforced rectangular columns under axial load Pu and uniaxial bending Mu, this
lesson explains the three modes of failure and the interaction diagram of such
columns. The different possible strain profiles, and the compressive stress blocks
are drawn and explained when the neutral axis is within and outside the cross-
section of the column. Determination of compressive stresses of concrete and
tensile/compressive stresses of longitudinal steel are explained. The governing
equations of equilibrium are introduced to illustrate the need for recasting them in
non-dimensional form for the purpose of design of such columns.

Version 2 CE IIT, Kharagpur


Module
10
Compression Members
Version 2 CE IIT, Kharagpur
Lesson
24
Preparation of Design
Charts
Version 2 CE IIT, Kharagpur
Instructional Objectives:
At the end of this lesson, the student should be able to:

• identify a design chart and understand the differences between a design


chart and interaction diagram of P and M,

• name the major design parameters of short columns subjected to axial


loads and uniaxial bending,

• state the design parameters assumed before the design,

• state the design parameter actually designed for the column,

• explain the roles of each of the design parameters in increasing the


strength capacities of column,

• name the two non-dimensional design parameters to prepare the design


charts,

• derive the governing equations in four separate cases while preparing the
design charts,

• mention the various points at which the values of the two non-dimensional
parameters are determined to prepare the design charts,

• prepare the design chart of any short and rectangular column subjected to
axial loads and uniaxial moment.

10.24.1 Introduction
Lesson 23 illustrates the different steps of determining the capacities of a
short, rectangular, reinforced with steel bars, concrete column. Several pairs of
collapse strengths Pu and Mu are to be determined for a column with specific
percentage of longitudinal steel bars assuming different positions of the neutral
axis. A designer has to satisfy that each of the several pairs of Pu and Mu,
obtained from the structural analysis, is less than or equal to the respective
strengths in form of pairs of Pu and Mu obtained from determining the capacities
for several locations of the neutral axis. Thus, the design shall involve several
trials of a particular cross-section of a column for its selection.

On the other hand, it is also possible to prepare non-dimensional


interaction diagram selecting appropriate non-dimensional parameters. This
would help to get several possible cross-sections with the respective longitudinal

Version 2 CE IIT, Kharagpur


steel bars. This lesson explains the preparation of such non-dimensional
interaction diagrams which are also known as design charts.

Similar design charts of circular and other types of cross-sections can be


prepared following the same procedure as that of rectangular cross-section.
However, the stress block parameters, explained in Lesson 23, are to be
established separately by summing up the forces and moment of several strips
by dividing the cross-section of columns into the strips. This lesson is restricted
to columns of rectangular cross-section which are symmetrically reinforced.

10.24.2 Design Parameters


The following are the four major design parameters to be determined for
any column so that it has sufficient pairs of strengths (Pu and Mu) to resist all
critical pairs obtained from the analysis:

(i) dimensions b and D of the rectangular cross-section,

(ii) longitudinal steel reinforcing bars - percentage p, nature of distribution


(equally on two or four sides) and d'/D,

(iii) grades of concrete and steel, and

(iv) transverse reinforcement.

The roles and importance of each of the above four parameters are
elaborated below:

(i) Dimensions b and D of the rectangular cross-section

The strength of column depends on the two dimensions b and D.


However, preliminary dimensions of b and D are already assumed for the
analysis of structure, which are usually indeterminate statically. In the
subsequent redesign, these dimensions may be revised, if needed, inviting re-
analysis with the revised dimensions.

(ii) Longitudinal steel reinforcing bars

It is a very important consideration to utilise the total area of steel bars


effectively. The total area of steel, expressed in percentage p ranges from the
minimum 0.8 to the maximum 4 per cent of the gross area of the cross-section.
The bars may be distributed either equally on two sides or on all four sides
judiciously having two or multiple rows of steel bars. The strain profiles of
Fig.10.23.2 reveals that the rows of bars may be all in compression or both
compression and tension depending on the location of the neutral axis.
Accordingly, the total strength of the longitudinal bars is determined by adding all

Version 2 CE IIT, Kharagpur


the individual strengths of bars of different rows. The effective cover d', though
depends on the nominal cover, has to be determined from practical
considerations of housing all the steel bars.

(iii) Grades of concrete and steel

The dimensions b and D of the cross-section and the amount of


longitudinal steel bars depend on the grades of concrete and steel.

(iv) Transverse reinforcement

The transverse reinforcement, provided in form of lateral ties or spirals,


are important for the following advantages in

(a) preventing premature / local buckling of the longitudinal bars,

(b) improving ductility and strength by the effect of confinement of the core
concrete,

(c) holding the longitudinal bars in position during construction, and

(d) providing resistance against shear and torsion, if present.

However, the transverse reinforcement does not have a major contribution


in influencing the capacities of the column. Moreover, the design of transverse
reinforcement involves selection of bar diameter and spacing following the
stipulations in the design code. The bar diameter of the transverse reinforcement
also depends on the bar diameter of longitudinal steel. Accordingly, the
transverse reinforcement is designed after finalizing other parameters mentioned
above.

It is, therefore, clear that the design of columns mainly involves the
determination of percentage of longitudinal reinforcement p, either assuming or
knowing the dimensions b and D, grades of concrete and steel, distribution of
longitudinal bars in two or multiple rows and d'/D ratio from the analysis or
elsewhere. Needless to mention that any designed column should be able to
resist several critical pairs of Pu and Mu obtained from the analysis of the
structure. It is also a fact that several trials may be needed to arrive at the final
selection revising any or all the assumed parameters. Accordingly, the design
charts are prepared to give the results for the unknown parameter quickly
avoiding lengthy calculations after selecting appropriate non-dimensional
parameters.

Based on the above considerations and making the design simple, quick
and fairly accurate, the following are the two non-dimensional parameters:

Version 2 CE IIT, Kharagpur


For axial load: Pu/fckbD

For moment: Mu/fckbD2

The characteristic strength of concrete fck has been associated with the
non-dimensional parameters as the grade of concrete does not improve the
strength of the column significantly. The design charts prepared by SP-16 are
assuming the constant value of fck for M 20 to avoid different sets of design
charts for different grades of concrete. However, separate design charts are
presented in SP-16 for three grades of steel (Fe 250, Fe 415 and Fe 500), four
values of d'/D (0.05, 0.1, 0.15 and 0.2) and two types of distribution of
longitudinal steel (distributed equally on two and four sides). Accordingly there
are twenty-four design charts for the design of rectangular columns. Twelve
separate design charts are also presented in SP-16 for circular sections covering
the above mentioned three grades of steel and for values of d'/D ratio.

However, the unknown parameter p, the percentage of longitudinal


reinforcement has been modified to p/fck in all the design charts of SP-16, so that
for grades other than M 20, the more accurate value of p can be obtained by
multiplying the p/fck with the actual grade of concrete used in the design of that
column.

However, this lesson explains that it is also possible to prepare design


chart taking into consideration the actual grade of concrete. As mentioned earlier,
the design charts are prepared getting the pairs of values of Pu and Mu in non-
dimensional form from the equations of equilibrium for different locations of the
neutral axis. We now take up the respective non-dimensional equations for four
different cases as follows:

(a) When the neutral axis is at infinity, i.e., kD = ∞ , pure axial load is
applied on the column.

(b) When the neutral axis is outside the cross-section of the column, i.e.,
∞ > kD ≥ D.

(c) When the neutral axis is within the cross-section of the column, i.e.,
kD < D.

(d) When the column behaves like a steel beam.

10.24.3 Non-dimensional Equation of Equilibrium when k =


∞ , (Pure Axial Load)

Figures 10.23.2b and c of Lesson 23 present the strain profile EF and the
corresponding stress block for this case. As the load is purely axial, we need to

Version 2 CE IIT, Kharagpur


express the terms Cc and Cs of Eq.10.35 of sec.10.23.10 of Lesson 23. The total
compressive force due to concrete of constant stress of 0.446 fck is:

Cc = 0.446 fck b D
(10.37)

However, proper deduction shall be made for the compressive force of concrete
not available due to the replacement by steel bars while computing Cs.

The force of longitudinal steel bars in compression is now calculated. The


steel bars of area pbD/100 are subjected to the constant stress of fsc when the
strain is 0.002. Subtracting the compressive force of concrete of the same area
pbD/100, we have,

Cs = (pbD/100) (fsc - 0.446 fck)


(10.38)

Thus, we have from Eq.10.35 of sec.10.23.10 of Lesson 23 after substituting the


expressions of Cc and Cs from Eqs.10.37 and 10.38,

Pu = 0.446 fck b D + (pbD/100) (fsc - 0.446 fck)


(10.39)

Dividing both sides of Eq.10.39 by fck bD, we have

(Pu/fck bD) = 0.446 + (p/100 fck) (fsc - 0.446 fck)


(10.40)

Thus, Eq.10.40 is the only governing equation for this case to be considered.

10.24.4 Non-dimensional Equations of Equilibrium when


Neutral Axis is Outside the Section ( ∞ > kD ≥ D)
Figures 10.23.3b and c of Lesson 23 present the strain profile JK and the
corresponding stress block for this case. The expressions of Cc, Cs and
appropriate lever arms are determined to write the two equations of equilibrium
(Eqs.10.35 and 36) of Lesson 23. While computing Cc, the area of parabolic
stress block is determined employing the coefficient C1 from Table 10.4 of
Lesson 23. Similarly, the coefficient C2, needed to write the moment equation, is
obtained from Table 10.4 of Lesson 23. The forces and the corresponding lever
arms of longitudinal steel bars are to be considered separately and added for
each of the n rows of the longitudinal bars. Thus, we have the first equation as,

Version 2 CE IIT, Kharagpur


n
Pu = C1 fck bD + ∑i =1
( pi bD / 100) ( f si - f ci )

(10.41)

where C1 = coefficient for the area of stress block to be taken from Table 10.4
of Lesson 23,

pi = Asi/bD where Asi is the area of reinforcement in the ith row,

fsi = stress in the ith row of reinforcement, taken positive for compression
and negative for tension,

fci = stress in concrete at the level of the ith row of reinforcement, and

n = number of rows of reinforcement.

Here also, the deduction of the compressive force of concrete has been made for
the concrete replaced by the longitudinal steel bars.

Dividing both sides of Eq.10.41 by fckbD, we have

n
(Pu/fckbD) = C1 + ∑
i =1
( p i / 100 f ck ) ( f si - f ci )

(10.42)

Similarly, the moment equation (Eq.10.36) becomes,

n
Mu = C1 fckbD (D/2 - C2D) + ∑i =1
( pi bD / 100) ( f si - f ci ) y i

(10.43)

where C2 = coefficient for the distance of the centroid of the compressive stress
block of concrete measured from the highly compressed right edge
and is taken from Table 10.4 of Lesson 23, and

yi = the distance from the centroid of the section to the ith row of
reinforcement, positive towards the highly compressed right edge
and negative towards the least compressed left edge.

Dividing both sides of Eq.10.43 by fckbD2, we have

n
2
(Mu/fckbD ) = C1(0.5 - C2) + ∑
i =1
( p i / 100 f ck ) ( f si - f ci ) (yi/D)

(10.44)

Version 2 CE IIT, Kharagpur


Equations 10.42 and 10.44 are the two non-dimensional equations of
equilibrium in this case when ∞ < kD ≤ D.

10.24.5 Non-dimensional Equations of Equilibrium when the


Neutral Axis is within the Section (kD < D)
The strain profile IN and the corresponding stress block of concrete are
presented in Figs.10.23.4b and c for this case. Following the same procedure of
computing Cc, Cs and the respective lever arms, we have the first equation as

n
Pu = 0.36 fck kbD + ∑
i =1
( pi bD / 100) ( f si - f ci )

(10.45)

Dividing both sides of Eq.10.45 by fckbD, we have

n
Pu/fckbD = 0.36 k + ∑
i =1
( p i / 100 f ck ) ( f si - f ci )

(10.46)

and the moment equation (Eq.10.36) as

n
Mu = 0.36 fck kbD(0.5 - 0.42 k) D + ∑i =1
( p i bD / 100) ( f si - f ci ) (yi/D)

(10.47)

Dividing both sides of Eq.10.47 by fckbD2, we have

n
(Mu/fckbD2) = 0.36 k(0.5 - 0.42 k) + ∑i =1
( p i / 100 f ck ) ( f si - f ci ) (yi/D)

(10.48)

where k = Depth of the neutral axis/Depth of column, mentioned earlier in


sec.10.21.10 and Fig.10.21.11 of Lesson 21.

Equations 10.46 and 10.48 are the two non-dimensional equations of


equilibrium in this case.

10.24.6 Non-dimensional Equation of Equilibrium when the


Column Behaves as a Steel Beam
This is a specific situation when the column is subjected to pure moment
Mu = Mo only (Point 6 of the interaction diagram in Fig.10.23.1 of Lesson 23).

Version 2 CE IIT, Kharagpur


Since the column has symmetrical longitudinal steel on both sides of the
centroidal axis of the column, the column will resist the pure moment by yielding
of both tensile and compressive steel bars (i.e., fsi = 0.87 fy = fyd). Thus, we have
only one equation (Eq.10.36 of Lesson 23), which becomes

n
Mu = ∑
i =1
( pi bD / 100) (0.87 f y ) (yi/D)

(10.49)

Dividing both sides of Eq.10.49 by fck bD2, we have

n
(Mu/fckbD2) = ∑i =1
( pi / 100 f ck ) (0.87 f y ) (yi/D)

(10.50)

Equation 10.50 is the equation of equilibrium in this case.

10.24.7 Preparation of Design Charts


Design charts are prepared employing the equations of four different
cases as given in secs.10.24.3 to 6. The advantage of employing the equations
is that the actual grade of concrete can be taken into account, though it may not
be worthwhile to follow this accurately. However, preparation of interaction
diagram will help in understanding the behaviour of column with the change of
neutral axis depth for the four cases mentioned in sec.10.24.2. The step by step
procedure of preparing the design charts is explained below. It is worth
mentioning that the values of (Pu/fckbD) and (Mu/fckbD2) are determined
considering different locations of the neutral axis for the four cases mentioned in
sec.10.24.2.

Step 1: When the neutral axis is at infinity

The governing equation is Eq.10.40. The strain profile EF and the


corresponding stress block are in Fig.10.23.2b and c of Lesson 23, respectively.

Step 2: When the column is subjected to axial load considering minimum


eccentricity

Lesson 22 presents the design of short columns subjected to axial load


only considering minimum eccentricity as stipulated in cl.29.3 of IS 456,
employing Eq.10.4, which is as follows:

Pu = 0.4 fck b D + (pbD/100) (0.67 fy - 0.4 fck) …. (10.4)


Dividing both sides of Eq.10.4 by fck bD, we have

Version 2 CE IIT, Kharagpur


(Pu/fck bD) = 0.4 + (p/100 fck) (0.67 fy - 0.4 fck)
(10.51)

The Pu obtained from Eq.10.51 can also resist Mu as per cl.39.3 of IS 456.
From the stipulation of cl. 39.3 of IS 456 and considering the maximum value of
the minimum eccentricity as 0.05D, we have

Mu = (Pu) (0.05)D = 0.02 fck bD2 + (0.05 pbD2/100) (0.67 fy - 0.4 fck)

Dividing both sides of the above equation by fckbD2, we have

(Mu/fck bD2) = 0.02 + (0.05p/100 fck) (0.67 fy - 0.4 fck)


(10.52)

Equations 10.51 and 10.52 are the two equations to be considered in this
case.

Step 3: When the neutral axis is outside the section

Figures 10.23.3b and c of Lesson 23 present one strain profile JK and the
corresponding stress block, respectively, out of a large number of values of k
from 1 to infinity, only values up to about 1.2 are good enough to consider, as
explained in sec.10.23.5 of Lesson 23. Accordingly, we shall consider only one
point, where k = 1.1, in this case. With the help of Eqs.10.42 and 10.44, Table
10.4 for the values of C1 and C2, Table 10.5 for the values of fsi and Eq.10.23 or
Eq.10.27 for the values of fci, the non-dimensional parameters Pu/fck bD and
Mu/fck bD2 are determined.

Step 4: When the neutral axis is within the section

One representative strain profile IU and the corresponding stress block


are presented in Fig.10.23.4b and c, respectively, of Lesson 23. The following six
points of the interaction diagram are considered satisfactory for preparing the
design charts:

(a) Where the tensile stress of longitudinal steel is zero i.e., kD = D - d',

(b) Where the tensile stress of longitudinal steel is 0.4fyd = 0.4(0.87 fy),

(c) Where the tensile stress of longitudinal steel is) 0.8fyd = 0.8(0.87 fy) ,

(d) Where the tensile stress of longitudinal steel is fyd = 0.87fy and strain
= 0.87fy/Es, i.e., the initial yield point,

(e) Where the tensile stress of longitudinal steel is fyd = 0.87fy and strain
= 0.87fy/Es + 0.002, i.e., the final yield point,

Version 2 CE IIT, Kharagpur


(f) When the depth of the neutral axis is 0.25D.

For all six points, the respective strain profile and the corresponding stress
blocks can be drawn. Therefore, values of (Pu/fck bD) and (Mu/fck bD2) are
determined from Eqs.10.46 and 10.48, using Table 10.5 for fsc and Eq.10.34 for
fci.

Step 5: When the column behaves like a steel beam

As explained in sec.10.24.6, Eq.10.50 is used to compute Mu/fck bD2 in this


case.

Step 6: Preparation of design chart

The ten pairs of (Pu/fck bD) and (Mu/fck bD2) (one set each in steps 1, 2, 3
and 5 and six sets in step 4) can be plotted to prepare the desired design chart.

One illustrative example is taken up in the next section.

10.24.8 Illustrative Example


Problem 1:

Prepare a design chart for a rectangular column with 3 per cent


longitudinal steel distributed equally on two faces using M 25 and Fe 415, and
considering d'/D = 0.15.

Solution 1:

The solution of this problem is explained in six steps of the earlier section.

Step 1: When the neutral axis is at infinity

Figures 10.23.2b and c present the strain profile EF and the


corresponding stress block, respectively. Using the values of p = 3 per cent, fck =
25 N/mm2 and determining the value of fsc = 327.7388 N/mm2 (using linear
interpolation from the values of Table 10.5 of Lesson 23), we get the value of
(Pu/fck bD) from Eq.10.40 as

(Pu/fck bD) = 0.8259.

Step 2: When the column is subjected to axial load considering minimum


eccentricity
Using the value of p = 3 per cent, fck = 25 N/mm2 and fy = 415 N/mm2 in
Eqs.10.51 and 10.52 of sec.10.24.6, we have

Version 2 CE IIT, Kharagpur


(Pu/fck bD) = 0.7217

(Mu/fck bD2 ) = 0.0361

Step 3: When the neutral axis depth = 1.1 D

Figures 10.24.1a, b and c show the section of the column, strain profile JK
and the corresponding stress block, respectively, for this case. We use
Eqs.10.42 and 10.44 for determining the value of (Pu/fck bD) and (Mu/fck bD2 ) for

Version 2 CE IIT, Kharagpur


this case using k = 1.1, fck = 25 N/mm2, p1 = p2 = 1.5 , y1/D = 0.35 and y2/D = -
0.35. Values of C1, C2, fs1 and fs2, fc1 and fc2 are obtained from equations
mentioned in Step 3 of sec.10.24.6. The values of all the quantities are presented
in Table 10.6A, mentioning the source equation no., table no. etc. to get the two
non-dimensional parameters as given below:

(Pu/fck bD) = 0.67405


(Mu/fck bD2 ) = 0.06370

Version 2 CE IIT, Kharagpur


Version 2 CE IIT, Kharagpur
Version 2 CE IIT, Kharagpur
Version 2 CE IIT, Kharagpur
Version 2 CE IIT, Kharagpur
Step 4: When the neutral axis is within the section

In Step 4 of section 10.24.6, six different locations of neutral axis are


mentioned; five of them (a to e) are specified by the magnitude of fs2 (tensile) of
longitudinal steel and one of them is specified by the value of k = 0.25. The
values of all the quantities are presented in Tables 10.6A and B, mentioning the
source equation no., table no. etc.
Version 2 CE IIT, Kharagpur
Figures 10.24.2 to 10.24.7 present the respective strain profiles and the
corresponding stress block separately for all six different locations of the neutral
axis.

Table 10.6A Parameters and results of Problem 1 of Section 10.24.8

Given data: fck = 25 N/mm2, fy = 415 N/mm2, p = 3 per cent, p1 = p2 = 1.5 per
cent,
d’/D = 0.15
Note: Units of fsi, fsc and fci are in N/mm2, (-) minus sign indicates tensile strain
or stress.

Sl.No. Given k = 1.1 fs2 = 0 fs2 = -0.4 fyd fs2 = 0.8 fyd
Description
1 Sec. No. 10.24.7 10.24.7 10.24.7 10.24.7
2 Step No. 3 4 4 4
3 Fig. No. 10.24.1 10.24.2 10.24.3 10.24.4
4 εs1 = εc1 0.002829 0.00288 0.00275 0.00263
5 εs2 = εc2 0.000744 0.0 -0.00072 -0.00144
6 Table No. 10.5 10.5 10.5 10.5
of fsi and
fsc
7 fs1 352.407 352.871 351.669 348.392
8 fs2 148.914 0.0 -144.42 -288.84
9 fsc NA NA NA NA
10 Eq.Nos. of 10.23 and 10.34 10.34 10.34
fci 10.27
11 fc1 11.15 11.15 11.15 11.15
12 fc2 6.757 0.0 0.0 0.0
13 Table No. 10.4 NA NA NA
of C1 and
C2
14 C1 0.384 NA NA NA
15 C2 0.443 NA NA NA
16 y1/D +0.35 +0.35 +0.35 +0.35
17 y2/D -0.35 -0.35 -0.35 -0.35
18 k 1.1 0.85 0.7046 0.6017
19 Eq.No. of 10.42 10.46 10.46 10.46
Pu/fck bD
20 Pu/fck bD 0.6740 0.5110 0.3713 0.2457
21 Eq.No. of 10.44 10.48 10.48 10.48
Mu/fck bD2
22 Mu/fck bD2 0.0643 0.1155 0.1536 0.1850

Version 2 CE IIT, Kharagpur


Table 10.6B Parameters and results of Problem 1 of Section 10.24.8

Given data: fck = 25 N/mm2, fy = 415 N/mm2, p = 3 per cent, p1 = p2 = 1.5 per
cent,
d’/D = 0.15
Note: Units of fsi, fsc and fci are in N/mm2, (-) minus sign indicates tensile strain
or stress.

Sl.No. Given fs2 = - fyd fs2 = - fyd k = 0.25


Description (Initial yield) (Final yield)
1 Sec. No. 10.24.7 10.24.7 10.24.7
2 Step No. 4 4 4
3 Fig. No. 10.24.5 10.24.6 10.24.7
4 εs1 = εc1 0.00256 0.00221 0.0014
5 εs2 = εc2 -0.00180 -0.00380 -0.0084
6 Table No. 10.5 10.5 10.5
of fsi and
fsc
7 fs1 346.754 335.484 281.0
8 fs2 -361.05 -361.05 -361.05
9 fsc NA NA NA
10 Eq.Nos. of 10.34 10.34 10.34
fci
11 fc1 11.15 11.15 10.146
12 fc2 0.0 0.0 0.0
13 Table No. NA NA NA
of C1 and
C2
14 C1 NA NA NA
15 C2 NA NA NA
16 y1/D +0.35 +0.35 +0.35
17 y2/D -0.35 -0.35 -0.35
18 k 0.5607 0.4072 0.25
19 Eq.No. of 10.46 10.46 10.46
Pu/fck bD
20 Pu/fck bD 0.1866 0.1246 0.0353
21 Eq.No. of 10.48 10.48 10.48
Mu/fck bD2
22 Mu/fck bD2 0.1997 0.1921 0.1680

Step 5: When the column behaves like a steel beam

For this case, the parameter (Mu/fck bD2) is determined from Eq.10.50
using p1 = p2 = 1.5 per cent, fck = 25 N/mm2, fy = 415 N/mm2, y1/D = 0.35 and
y2/D = -0.35. Thus, we get

Version 2 CE IIT, Kharagpur


(Mu/fck bD2 ) = 0.15164

Step 6: Final results of design chart

The values of ten pairs of (Pu/fck bD) and (Mu/fck bD2) as obtained in steps
1 to 5 are presented in Sl. Nos. 1 to 10 of Table 10.6C. The design chart can be
prepared by plotting these values.

Table 10.6C Final values of Pu/fck bD and Mu/fck bD2 of Problem 1 of Section
10.24.8

Sl. No. Particulars about the point Pu/fck bD Mu/fck bD2


1 k=α 0.8259 0.0
2 Minimum eccentricity 0.7217 0.0361
3 k = 1.1 0.6740 0.0643
4 fs2 = 0 0.5110 0.1155
5 fs2 = (-)0.4 fyd 0.3713 0.1536
6 fs2 = (-)0.8 fyd 0.2457 0.1850
7 fs2 = (-) fyd 0.1866 0.1997
(Initial yield)
8 fs2 = (-) fyd 0.1246 0.1921
(Final yield)
9 k = 0.25 0.0353 0.1680
10 Steel Beam 0.0 0.1516

10.24.9 Practice Questions and Problems with Answers


Q.1: Why do we need to have non-dimensional design chart?

A.1: See sec. 10.24.1

Q.2: Name the different design parameters while designing a column. Mention
which one is the most important parameter.

A.2: See sec. 10.24.2.

Q.3: Prepare a design chart for a rectangular column within three per cent
longitudinal steel, equally distributed on two faces, using M 25 and Fe 250
and considering d'/D = 0.15.

.A.3: The solution of this problem is obtained following the same six steps of
Problem 1 of sec.10.24.8, except that the grade of steel here is Fe 250.
Therefore, the final results and all the parameters are presented in Table
10.7 avoiding explaining step by step again.

Version 2 CE IIT, Kharagpur


Table 10.7 Final values of Pu/fck bD and Mu/fck bD2 of Q.3 of Section 10.24.9

Sl. No. Particulars about the point Pu/fck bD Mu/fck bD2


1 k=α 0.6936 0.0
2 Minimum eccentricity 0.5890 0.0295
3 k = 1.1 0.5931 0.0354
4 fs2 = 0 0.4298 0.0871
5 fs2 = (-)0.4 fyd 0.3438 0.1113
6 fs2 = (-)0.8 fyd 0.2645 0.1323
7 fs2 = (-) fyd 0.2268 0.1421
(Initial yield)
8 fs2 = (-) fyd 0.1559 0.1395
(Final yield)
9 k = 0.25 0.0839 0.1248
10 Steel Beam 0.0 0.0914

10.24.10 References
1. Reinforced Concrete Limit State Design, 6th Edition, by Ashok K. Jain,
Nem Chand & Bros, Roorkee, 2002.
2. Limit State Design of Reinforced Concrete, 2nd Edition, by P.C.Varghese,
Prentice-Hall of India Pvt. Ltd., New Delhi, 2002.
3. Advanced Reinforced Concrete Design, by P.C.Varghese, Prentice-Hall of
India Pvt. Ltd., New Delhi, 2001.
4. Reinforced Concrete Design, 2nd Edition, by S.Unnikrishna Pillai and
Devdas Menon, Tata McGraw-Hill Publishing Company Limited, New
Delhi, 2003.
5. Limit State Design of Reinforced Concrete Structures, by P.Dayaratnam,
Oxford & I.B.H. Publishing Company Pvt. Ltd., New Delhi, 2004.
6. Reinforced Concrete Design, 1st Revised Edition, by S.N.Sinha, Tata
McGraw-Hill Publishing Company. New Delhi, 1990.
7. Reinforced Concrete, 6th Edition, by S.K.Mallick and A.P.Gupta, Oxford &
IBH Publishing Co. Pvt. Ltd. New Delhi, 1996.
8. Behaviour, Analysis & Design of Reinforced Concrete Structural Elements,
by I.C.Syal and R.K.Ummat, A.H.Wheeler & Co. Ltd., Allahabad, 1989.
9. Reinforced Concrete Structures, 3rd Edition, by I.C.Syal and A.K.Goel,
A.H.Wheeler & Co. Ltd., Allahabad, 1992.
10. Textbook of R.C.C, by G.S.Birdie and J.S.Birdie, Wiley Eastern Limited,
New Delhi, 1993.
11. Design of Concrete Structures, 13th Edition, by Arthur H. Nilson, David
Darwin and Charles W. Dolan, Tata McGraw-Hill Publishing Company
Limited, New Delhi, 2004.
12. Concrete Technology, by A.M.Neville and J.J.Brooks, ELBS with
Longman, 1994.
13. Properties of Concrete, 4th Edition, 1st Indian reprint, by A.M.Neville,
Longman, 2000.

Version 2 CE IIT, Kharagpur


14. Reinforced Concrete Designer’s Handbook, 10th Edition, by C.E.Reynolds
and J.C.Steedman, E & FN SPON, London, 1997.
15. Indian Standard Plain and Reinforced Concrete – Code of Practice (4th
Revision), IS 456: 2000, BIS, New Delhi.
16. Design Aids for Reinforced Concrete to IS: 456 – 1978, BIS, New Delhi.

10.24.11 Test 24 with Solutions


Maximum Marks = 50, Maximum Time = 30 minutes

Answer all questions.

TQ.1: Determine the parameters including the two non-dimensional


parameters, Pu and Mu of a rectangular reinforced concrete short
column of b = 370 mm, D = 530 mm, d'/D = 0.1 and having 8-25 mm
diameter bars as longitudinal steel distributed equally on two sides
using M 20 and Fe 415 for each of the following three cases:

(a) when fs2 = - 0.4 fyd

(b) when fs2 = - 0.8 fyd

(c) when fs2 = - fyd (at final yield)


(16 + 17 + 17 = 50)

A.TQ.1: This problem can be solved following the same procedure of explained
in Step 4b, c and d of sec.10.24.7. The step by step calculations are
not shown here and the final results are presented in Table 10.8.

Table 10.8 Parameters and results of TQ.1 of Section 10.24.11

Given data: fck = 20 N/mm2, fy = 415 N/mm2, b = 370 mm, D = 530 mm,
Longitudinal steel = 8-25 mm diameter equally distributed on
two sides, d'/D = 0.15

Sl.No. Given fs2 = -0.4 fyd fs2 = 0.8 fyd fs2 = - fyd
Description (Final yield)
1 Sec. No. 10.24.7 10.24.7 10.24.7
2 Step No. 4 4 4
3 Fig. No. 10.24.3 10.24.4 10.24.6
4 εs1 = εc1 0.0030 0.0029 0.0027
5 εs2 = εc2 -0.00072 -0.00144 -0.0038
6 Table No. 10.5 10.5 10.5
of fsi and
fsc
7 fs1 354.1702 353.468 349.956

Version 2 CE IIT, Kharagpur


8 fs2 -144.42 -288.84 -361.05
9 fsc NA NA NA
10 Eq.Nos. of 10.34 10.34 10.34
fci
11 fc1 11.15 11.15 11.15
12 fc2 0.0 0.0 0.0
13 Table No. NA NA NA
of C1 and
C2
14 C1 NA NA NA
15 C2 NA NA NA
16 y1/D +0.4 +0.4 +0.4
17 y2/D -0.4 -0.4 -0.4
18 k 0.7461 0.6371 0.4311
19 Eq.No. of 10.46 10.46 10.46
Pu/fck bD
20 Pu/fck bD 0.3690 0.2572 0.1452
21 Pu (kN) 1447.225 1008.792 569.568
22 Eq.No. of 10.48 10.48 10.48
Mu/fck bD2
23 Mu/fck bD2 0.1481 0.1799 0.1899
24 Mu (kNm) 307.777 374.125 394.779

10.24.12 Summary of this Lesson


This lesson explains the procedure of the preparation of design
charts of rectangular reinforced concrete short columns subjected to axial load
and uniaxial moment. Different positions of the neutral axis due to different pairs
of Pu and Mu give rise to different strain profiles and stress blocks. Accordingly,
the column may collapse when subjected to any pair of axial load and moment
exceeding the capacities of the column. Design charts are very much useful to
design the column avoiding lengthy numerical computations. Illustrative example,
practice and test problems will help in understanding each step of the procedure
to prepare the design chart.

Version 2 CE IIT, Kharagpur


Module
10
Compression Members
Version 2 CE IIT, Kharagpur
Lesson
25
Design of Short
Columns under
Axial Load with
Uniaxial Bending
Version 2 CE IIT, Kharagpur
Instructional Objectives:
At the end of this lesson, the student should be able to:

• state the two types of problems that can be solved using the design charts
of SP-16,

• mention the three sets of design charts specifying their parameters,

• state the approximations, limitations and usefulness of the design charts


of SP-16,

• mention the different steps of solving the analysis type of problems using
the design charts of SP-16,

• mention the different steps of solving the design type of problems using
the design charts of SP-16,

• apply the methods in solving both types of problems using the design
charts of SP-16.

10.25.1 Introduction
Lesson 24 explains the procedure of preparing the design charts of short
rectangular reinforced concrete columns under axial load with uniaxial bending. It
is also mentioned that similar design charts can be prepared for circular and
other types of cross-sections of columns by dividing the cross-section into
several strips. This lesson explains the design of rectangular and circular short
columns with the help of design charts.

It is known that the design of columns by direct computations involves


several trials and hence, time taking. On the other hand, design charts are very
useful in getting several alternative solutions quickly. Further, design charts are
also used for the analysis of columns for safety etc. However, there are
limitations of using the design charts, which are mentioned in this lesson. Several
numerical problems are solved in this lesson for the purpose of illustration
covering both analysis and design types of problems using the design charts of
SP-16.

Version 2 CE IIT, Kharagpur


10.25.2 Design Charts of SP-16
SP-16 has three sets of design charts prepared by following the procedure
explained in Lesson 24 for rectangular and circular types of cross-sections of
columns. The three sets are as follows:

(i) Charts 27 to 38 are the first set of twelve charts for rectangular columns
having symmetrical longitudinal steel bars in two rows (Fig.10.25.1) for three
grades of steel (Fe 250, Fe 415 and Fe 500) and each of them has four values of
d’/D ratios (0.05, 0.10, 0.15 and 0.20).

(ii) Charts 39 to 50 are the second set of twelve charts for rectangular
columns having symmetrical longitudinal steel bars (twenty numbers) distributed
equally on four sides (in six rows, Fig.10.25.2) for three grades of steel (Fe 250,
Fe 415 and Fe 500) and each of them has four values of d’/D ratios (0.05, 0.10,
0.15 and 0.20).

Version 2 CE IIT, Kharagpur


(iii) The third set of twelve charts, numbering from 51 to 62, are for circular
columns having eight longitudinal steel bars of equal diameter and uniformly
spaced circumferentially (Fig.10.25.3) for three grades of steel (Fe 250, Fe 415
and Fe 500) and each of them has four values of d’/D ratios (0.05, 0.10, 0.15 and
0.20).

All the thirty-six charts are prepared for M 20 grade of concrete only. This
is a justified approximation as it is not worthwhile to have separate design charts
for each grade of concrete.

10.25.3 Approximations and Limitations of Design Charts of


SP-16
(i) Approximations

The following are the approximations of the design charts of SP-16:

(a) Grade of concrete

As mentioned in the earlier section, all the design chars of SP-16 assume
the constant grade M 20 of concrete. However, each chart has fourteen plots
having different values of the parameter p/fck ranging from zero to 0.26 at an
interval of 0.02. The designer, thus, can make use of the actual grade of concrete
by multiplying the p/fck obtained from the plot with the actual fck for the particular
grade of concrete to partially compensate the approximation.

Version 2 CE IIT, Kharagpur


(b) The d’/D ratio

The three sets of charts have four fixed values of d’/D ratios (0.05, 0.10,
0.15 and 0.20). However, in the practical design, the d’/D ratio may be different
from those values. In such situations intermediate values are determined by
making linear interpolations.

(c) Equal distribution of twenty longitudinal steel bars on four sides of rectangular
columns

In spite of the above consideration, the design charts may be used without
significant error for any number of bars greater than eight provided the bars are
distributed equally on four sides.

(d) Longitudinal bars in circular columns

Though the design charts are prepared considering eight bars uniformly
placed circumferentially, they may generally be used for any number of bars
greater than six, uniformly placed circumferentially.

(ii) Limitations

The following are the limitations of the design chars of SP-16:

(a) Longitudinal bars equally distributed on four sides of rectangular columns

Twenty bars, when equally placed on four sides, are placed in six rows
avoiding any bar on the two axes. However, there will be bars on the axes for
odd number of rows. A very common type is the 6-bar arrangement (Fig.10.25.4).
Such arrangements, though symmetrical, are not covered in the design charts of
SP-16. In such cases, the designer has to make his own assumptions judiciously
in order to use the available charts of SP-16. Alternatively, he has to prepare the
actual design chart depending on the bar arrangement to get accurate results.

Version 2 CE IIT, Kharagpur


(b) Unsymmetrical arrangement of longitudinal bars in rectangular cross-sections

It is not covered in the charts.

(c) Non-uniform placing of longitudinal bars in circular cross-sections

It is not covered in the charts.

(d) Cross-sections other than rectangular or circular like, I, T, H, X etc.

These are not covered in the charts.

The items under b, c and d, though rare, should be taken care of by


preparing the respective design chars as and when needed.

(e) Concluding remarks

In spite of the above approximations and limitations, use of SP-16 has


several advantages even by making some more approximations if the charts are
not directly applicable. In the note of cl.39.5 of IS 456, the following is
recommended, which is worth reproducing:

“The design of members subject to combined axial load and uniaxial


bending will involve lengthy calculation by trial and error. In order to overcome
these difficulties interaction diagrams may be used. These have been prepared
and published by BIS in “SP-16 Design aids for reinforced concrete to IS 456’.”

Accordingly, the use of SP-16 is explained in the following sections for the
solutions of both analysis and design types of problems.

10.25.4 Use of Design Charts in the Analysis Type of


Problems
In many situations, it becomes necessary to assess the safety of a
column with known cross-section dimensions, and longitudinal and transverse
steel reinforcing bars. The objective is to examine if the column can resist some
critical values of Pu or Mu or pairs of Pu and Mu, as may be expected to be
applied on the column. This is done by comparing if the given values of pair of Pu
and Mu are less than the respective strength capacities pair of Pu and Mu. The
word “given” shall be used in the suffix of pairs of Pu and Mu to indicate that they
are the given values for which the column has to be examined. The strength
capacities of the column, either Pu or Mu alone or pair of Pu and Mu, will not have
any suffix. Thus, the designer shall assess

Version 2 CE IIT, Kharagpur


(pair of Pu and Mu)given < pair of Pu and Mu, as strength capacities
(10.53)

This type of problem is known as analysis type of problem. The three


steps are given below while using design charts of SP-16 for solving such
problems.

Step 1: Selection of the design chart

The designer has to select a particular design chart, specified by the chart
number, from the known value of d’/D and the grade of steel for circular columns;
and considering also the distribution of longitudinal steel bars equally on two or
four sides for the rectangular columns.

Step 2: Selection of the particular curve

The designer shall select the particular curve out of the family of fourteen
curves in the chart selected in Step 1. The selection of the curve shall be made
from the value of p/fck parameter which is known.

Step 3: Assessment of the column

This can be done in any of the three methods selecting two of the three
parameters as known and comparing the third parameter to satisfy Eq.10.53. The
parameters are Pu/fck bD, Mu/fck bD2 and p/fck for rectangular columns. For circular
columns the breadth b shall be replaced by D (the diameter of the column).

10.25.5 Use of Design Charts in the Design Type of


Problems
It is explained in sec.10.24.2 of Lesson 24 that the design of columns
mainly involves with the determination of percentage of longitudinal steel p, either
assuming or knowing the dimensions b and D, grades of concrete and steel,
distribution of longitudinal bars in two or multiple rows and d’/D from the analysis
or elsewhere. However, the designer has to confirm the assumed data or revise
them, if needed. The use of design charts of SP-16 is explained below in four
steps while designing columns:

Step 1: Selection of the design chart

As in step 1 of sec.10.25.4, the design chart is selected from the assumed


values of the parameter as explained in step 1 of sec.10.25.4. The only
difference is that, here the assumed parameter may be revised, if required.

Version 2 CE IIT, Kharagpur


Step 2: Determination of the percentage of longitudinal steel

The two parameters (Pu/fck bD) and (Mu/fck bD2) are known and the point A
is located on the design chart with these two coordinates (Fig.10.25.5). The point
may be like A1, on a particular curve of specified p/fck, or like A2, in between two
such curves having two values of p/fck, the difference between the two values of
p/fck is 0.02. In the first case, the corresponding p/fck is obtained directly as
specified on the curve. While, in the second case, liner interpolation is to be done
by drawing a line KL perpendicular to the two curves and passing through the
point A2.

The percentage of longitudinal steel is obtained by multiplying the p/fck, so


obtained, by the actual grade of concrete (which may be different from M 20
though the chart is prepared assuming M 20 only). Thus, percentage of
longitudinal steel,

p = (p/fck) (Actual fck)


(10.54)

This percentage of longitudinal steel (obtained from Eq.10.54) is a


tentative value and shall be confirmed after finalizing the assumed data, i.e., d’/D,
b, D etc.

Step 3: Design of transverse reinforcement

This should be done before confirming d’/D as the diameter of the lateral
tie has a role in finalizing d’. The design of transverse reinforcement shall be
done following the procedure explained in secs.10.21.8 and 10.21.9 of Lesson
21.

Version 2 CE IIT, Kharagpur


Step 4: Revision of the design, if required

If the value of d’/D changes in step 3 requiring any change of other


dimension etc., the repetition of steps 1 to 3 are needed. Otherwise, the design is
complete.

10.25.6 Illustrative Examples

Problem 1:

Figure 10.25.6 shows a rectangular short reinforced concrete column


using M 25 and Fe 415. Analyse the safety of the column when subjected to Pu =
1620 kN and Mu = 170 kNm.

Solution 1:

This is an analysis type of problems. The data given are: b = 300 mm, D =
450 mm, d’ = 56 mm, Asc = 4021 mm2 (20 bars of 16 mm diameter), fck = 25
N/mm2, fy = 415 N/mm2, Pu = 1620 kN and Mu = 170 kNm. So, we have d’/D =
56/450 = 0.1244, Pu/fckbD = 0.48, Mu/fckbD2 = 0.111934 and p/fck = 0.11914.

Step 1: Selection of design chart

From the given data: d’/D = 0.1244, fy = 415 N/mm2 and longitudinal steel
bars are equally distributed on four sides, the charts selected are 44 (for d’/D =

Version 2 CE IIT, Kharagpur


0.1) and 45 (for d’/D = 0.15). Linear interpolation has to done with the values
obtained from these two charts.

Step 2: Selection of the particular curve

From the given value of p/fck = 0.11914, the two curves having p/fck = 0.1
and 0.12 are selected from both the charts (No. 44 and 45). Here also, linear
interpolation has to be done.

Step 3: Assessment of the column

In order to assess the column, we select the two given parameters p/fck
and Pu/fckbd2 to determine the third parameter Mu/fckbD2 to compare its value
with the given parameter Mu/fckbD2. However, the value of Mu/fckbD2 is obtained
by doing linear interpolation two times: once with respect to p/fck and the second
time with respect to d’/D. The results are furnished in Table 10.9 below:

Table 10.9: Values of Mu/fckbD2 when (Pu/fckbD2)given = 0.48 and (p/fck)given =


0.11914; and d’/D = 0.1244

Sl. No. p/fck d’/D


0.1 0.15 0.1244
1 0.1 0.1* 0.09** 0.09512***
2 0.12 0.12* 0.107** 0.113656***
3 0.11914 0.1194*** 0.10649*** 0.1130941***

Note: * Values obtained from chart 44


** Values obtained from chart 45
*** Linearly interpolated values

Thus, the moment capacity of the column is obtained from the final value
of Mu/fckbD2 = 0.1130941 as

Mu = (0.1130941)(25)(300)(450)(450) Nmm = 171.762 kNm,

which is higher than the given Mu = 170 kNm. Hence, the column can be
subjected to the pair of given Pu and Mu as 1620 kN and 170 kNm, respectively.

Problem 2:

Design a short spiral column subjected to Pu = 2100 kN and Mu = 187.5


kNm using M 25 and Fe 415. The preliminary diameter of the column may be
taken as 500 mm.

Solution 2:

Version 2 CE IIT, Kharagpur


Step 1: Selection of design chart

With the given fy = 415 N/mm2 and assuming d’/D = 0.1, the chart
selected for this problem is Chart 56.

Step 2: Determination of the percentage of longitudinal steel

With the given fck = 25 N/mm2 and assuming the given D = 500 mm, we
have:

Pu/fckD2 = 2100000/25(500)(500) = 0.336, and

Mu/fckD3 = 187.5(106 )/25(500)(500)(500) = 0.06

The particular point A (Fig.10.25.5) having coordinates of Pu/fckD2 =


0.336 and Mu/fckD3 = 0.06 in Chart 56 gives: p/fck = 0.08. Hence, p = 0.08(25) =
2 per cent (see Eq.10.54).

Asc = 0.02( π )(500)(500)/4 = 3928.57 mm2

Provide 8-25 mm diameter bars to have Asc actually provided = 3927 mm2.
Marginally less amount of steel than required will be checked considering the
enhancement of strength for spiral columns as stipulated in cl.39.4 of IS 456.

Step 3: Design of transverse reinforcement

Version 2 CE IIT, Kharagpur


The diameter of the helical reinforcement is taken as 8 mm (> 25 mm/4).
The pitch p of the spiral is determined from Eq.10.11 of Lesson 22, which
satisfies the stipulation in cl.39.4.1 of IS 456. From Eq.10.11, we have the pitch
of the spiral p as:

p ≤ 11.1(Dc - φ sp ) asp fy/(D2 - Dc2 ) fck ….


(10.11)

where, Dc = 500 – 40 – 40 = 420 mm, D = 500 mm, fck = 25 N/mm2, fy =


415 N/mm2, φ sp = 8 mm and asp = 50 mm2.

Using the above values in Eq.10.11, we have p ≤ 25.716 mm. As per


cl.26.5.3.2d1, regarding the pitch of spiral: p >/ 420/6 (= 70 mm), p </ 25 mm
and p </ 24 mm. So, pitch of the spiral = 25 mm is o.k. Figure 10.25.7 presents
the cross-section with reinforcing bars of the column.

Step 4: Revision of the design, if required

Providing 25 mm diameter longitudinal steel bars and 8 mm diameter


spirals, we have d’ = 40 + 8 + 12.5 = 60.5 mm. This gives d’/D = 60.5/500 =
0.121. In step 1, d’/D is assumed as 0.1. So, the revision of the design is needed.

However, as mentioned in step 2, the area of steel required is not


provided and this may be offset considering the enhanced strength of the spiral
column, as stipulated in cl.39.4 of IS 456.

We, therefore, assess the strength of the designed column, when d’/D =
0.121 and Asc = 3927 mm2, if it can be subjected to Pu = 2100 kN and Mu = 187.5
kNm.

For the purpose of assessment, we determine the capacity Pu of the


column when Mu = 187.5 kNm. Further, the revised d’/D = 0.121 needs to
interpolate the values from Charts 56 (for d’/D = 0.1) and 57 (for d’/D = 0.15). The
value of p/fck = 0.08 and Mu/fckbD3 = 0.06. Table 10.10 presents the results.

Table 10.10: Value of Pu/fckbD2 when Mu/fckD3 = 0.06 and p/fck = 0.08

Sl.No. d’/D Pu/fckD2

1 0.1 0.336 (from Chart 56)


2 0.15 0.30 (from Chart 57)
3 0.121 0.32088 (Interpolated value)

From Table 10.10, thus, we get,

Version 2 CE IIT, Kharagpur


Pu/fckD2 = 0.32088, which gives Pu = (0.32088)(25)(500)(500) = 2005.5
kN.

Considering the enhanced strength as 1.05 times as per cl.39.4 of IS 456,


the actual capacity of this column is (1.05)(2005.5) = 2105 kN > 2100 kN.

Thus, the design is safe to carry Pu = 2100 kN and Mu = 187.5 kNm.

10.25.7 Practice Questions and Problems with Answers


Q.1: Name the two types of problems that can be solved using the design
charts of SP-16.

A.1: See sec. 10.25.1.

Q.2: Mention the three different sets of design charts available in SP-16
mentioning the number of charts and the parameters for their identification.

A.2: See sec. 10.25.2.

Q.3: State the approximations, limitations and usefulness of the design charts of
SP-16 in solving the analysis and design type of problems of short columns.

A.3: See sec. 10.25.3.

Q.4:

Version 2 CE IIT, Kharagpur


Assess the safety of the spiral column shown in Fig.10.25.8 using M 20 and
Fe 415 when subjected to Pu = 1200 kN and Mu = 64 kNm, considering the
enhanced strength of the spiral column.

A.4: In this problem, the given data are: D = 400 mm, d’ = 40 + 6 + 10 = 56 mm,
Asc = 2513 mm2 (8-20 mm diameter bars), fck = 20 N/mm2, fy = 415 N/mm2,
Pu = 1200 kN and Mu = 64 kNm.

Step 1: Selection of the design charts

With fy = 415 N/mm2 and d’/D = 56/400 = 0.14, we select two charts nos.
56 (for d’/D = 0.1) and 57 (for d’/D = 0.15). We have to interpolate the values
obtained from these two charts.

Step 2: Selection of the particular curve

From the given data we have p/fck = 0.0999488 ≅ 0.1. So, we select the
curve for p/fck = 0.1 in the two charts (Nos. 56 and 57).

Step 3: Assessment of the column

For the purpose of assessment, we select the two parameters p/fck and
Mu/fckD3 and determine the values of Pu/fckD2 from the two charts for interpolation.
The results are presented in Table 10.11 below.

Table 10.11: Values of p/fckD2 and Mu/fckD3 and p/fck = 0.1

Sl.No. d’/D Pu/fckD2

1 0.1 0.444 (from Chart 56)


2 0.15 0.422 (from Chart 57)
3 0.14 0.4264 (Interpolated value)

From Table 10.11, thus, we get Pu/fckD2 = 0.4264, which gives Pu = 1364.48 kN.

It may be noted that for more accuracy another set of values of d’/D = 0.08
is required. The interpolated value, thus obtained, shall be strictly applicable
when p/fck = 0.0999488. However, for all practical designs, such accuracy is not
required.

Further, as per cl.39.4 of IS 456, the enhanced capacity of the spiral


column = 1.05(1364.48) = 1432.704 kN, which is more than 1200 kN. It is also
seen that the column is safe even without considering the enhanced capacity as
the Pu = 1364.48 kN > 1200 kN.

Q.5:

Version 2 CE IIT, Kharagpur


Design a short square tied column to carry Pu = 2240 kN and Mu = 112
kNm using M 25 and Fe 415, and assuming the dimension b = D = 400
mm, as shown in Fig.10.25.9.

A.5: The data given are: b = D = 400 mm, Pu = 2240 kN, Mu = 112 kNm, fck =
25 N/mm2 and fy = 415 N/mm2.

Step 1: Selection of the design chart

With the given data of fy = 415 N/mm2 and assuming d’/D = 0.15, we have
to refer to Chart 45.

Step 2: Determination of percentage of longitudinal steel

Using the values of fck = 25 N/mm2 and assuming b = D = 400 mm as


given, we have Pu/fckD2 = 0.56 and Mu/fckD3 = 0.07.

From Chart 45, we get p/fck = 0.1, giving p = 2.5 per cent. Accordingly,

Asc = 2.5(400)(400)/100 = 4000 mm2. Provide 20 bars of 16 mm diameter


(Asc(provide) = 4021 mm2).

Version 2 CE IIT, Kharagpur


Step 3: Design of lateral tie

The arrangement of lateral tie shall be like Fig.18 of IS 456 as the


longitudinal bars are not spaced more than 75 mm on either side (cl.26.5.3.2b1
of IS 456). The pitch of the lateral tie of diameter 8 mm is kept at 250 mm c/c
satisfying the stipulation in cl.26.5.3.2c1 of IS 456. Figure 10.25.9 presents the
cross-section with reinforcing bars of the column.

Step 4: Revision of the design, if required

The value of d’ is now 56 mm which gives d’/D = 0.14. Accordingly, the


assumed value of d’/D in step 1 as 0.15 is not valid. So, we have to check the
capacity of the column interpolating the values when d’/D = 0.1 and 0.15 from
Charts 44 and 45, respectively. Further, the longitudinal steel provided gives
p/fck = 0.100525, which also is different from 0.1 as obtained in step 2 of this
problem. Though the difference is marginal, both the interpolations are done for
the academic interest and results are presented in Table 10.12 below. In
assessing the capacity of this column, we keep p/fck = 0.100125 and Pu/fckD2 =
0.56 as constants and determine the value of Mu/fckD3 by two linear
interpolations.

Table 10.12: Values of Mu/fckD3 when Pu/fckbD2 = 0.56 and p/fck = 0.10025

Sl. No. p/fck d’/D


0.1 0.15 0.14
1 0.1 0.1* 0.07** 0.072***
2 0.12 0.08* 0.09** 0.092***
3 0.100525 0.080525*** 0.070525*** 0.072525***

Note: * Values obtained from Chart 44


** Values obtained from Chart 45
*** Linearly interpolated values

So, the capacity of the column Mu = (0.072525)(25)(400)(400)(400) Nmm


= 116 kNm > 112 kNm.

Hence, the design of the column is o.k.

10.25.8 References
1. Reinforced Concrete Limit State Design, 6th Edition, by Ashok K. Jain,
Nem Chand & Bros, Roorkee, 2002.
2. Limit State Design of Reinforced Concrete, 2nd Edition, by P.C.Varghese,
Prentice-Hall of India Pvt. Ltd., New Delhi, 2002.

Version 2 CE IIT, Kharagpur


3. Advanced Reinforced Concrete Design, by P.C.Varghese, Prentice-Hall of
India Pvt. Ltd., New Delhi, 2001.
4. Reinforced Concrete Design, 2nd Edition, by S.Unnikrishna Pillai and
Devdas Menon, Tata McGraw-Hill Publishing Company Limited, New
Delhi, 2003.
5. Limit State Design of Reinforced Concrete Structures, by P.Dayaratnam,
Oxford & I.B.H. Publishing Company Pvt. Ltd., New Delhi, 2004.
6. Reinforced Concrete Design, 1st Revised Edition, by S.N.Sinha, Tata
McGraw-Hill Publishing Company. New Delhi, 1990.
7. Reinforced Concrete, 6th Edition, by S.K.Mallick and A.P.Gupta, Oxford &
IBH Publishing Co. Pvt. Ltd. New Delhi, 1996.
8. Behaviour, Analysis & Design of Reinforced Concrete Structural Elements,
by I.C.Syal and R.K.Ummat, A.H.Wheeler & Co. Ltd., Allahabad, 1989.
9. Reinforced Concrete Structures, 3rd Edition, by I.C.Syal and A.K.Goel,
A.H.Wheeler & Co. Ltd., Allahabad, 1992.
10. Textbook of R.C.C, by G.S.Birdie and J.S.Birdie, Wiley Eastern Limited,
New Delhi, 1993.
11. Design of Concrete Structures, 13th Edition, by Arthur H. Nilson, David
Darwin and Charles W. Dolan, Tata McGraw-Hill Publishing Company
Limited, New Delhi, 2004.
12. Concrete Technology, by A.M.Neville and J.J.Brooks, ELBS with
Longman, 1994.
13. Properties of Concrete, 4th Edition, 1st Indian reprint, by A.M.Neville,
Longman, 2000.
14. Reinforced Concrete Designer’s Handbook, 10th Edition, by C.E.Reynolds
and J.C.Steedman, E & FN SPON, London, 1997.
15. Indian Standard Plain and Reinforced Concrete – Code of Practice (4th
Revision), IS 456: 2000, BIS, New Delhi.
16. Design Aids for Reinforced Concrete to IS: 456 – 1978, BIS, New Delhi.

10.25.9 Test 25 with Solutions


Maximum Marks = 50, Maximum Time = 30 minutes

Answer all questions.

TQ.1: Mention the three different sets of design charts available in SP-16
mentioning the number of charts and the parameters for their
identification. (10 marks)

A.TQ.1: See sec. 10.25.2.

TQ.2: State the approximations, limitations and usefulness of the design charts
of SP-16 in solving the analysis and design type of problems of short
columns. (10 marks)

Version 2 CE IIT, Kharagpur


A.TQ.2: See sec. 10.25.3.

TQ.3:

Check the short square column of Fig.10.25.10 to carry Pu = 3250 kN and


Mu = 250 kNm using M 25 and Fe 415.
(30 marks)

A.TQ.3: Given data are: b = D = = 500 mm, Asc = 6283 mm2 (20 bars of 20 mm
diameter), fck = 25 N/mm2, fy = 415 N/mm2, Pu = 3250 kN and Mu = 250
kNm.

Step 1: Selection of design chart

From Fig.10.25.10, we get d’ = 65 mm giving d’/D = 0.13, and given fy =


415 N/mm2, we select Charts 44 (for d’/D = 0.1) and 45 (for d’/D = 0.15). We
have to interpolate the values to get the result when d’/D = 0.13.

Step 2: Selection of the particular curve

With p = 628300/(500)(500) = 2.5132 per cent, we get p/fck = 0.100528 ≅


0.1. Accordingly, the curve for p/fck = 0.1 is to be used in Charts 44 and 45.

Version 2 CE IIT, Kharagpur


Step 3: Assessment of the column

For the assessment, we keep Pu/fckD2 = 3250/25(500)(500) = 0.52 and


p/fck = 0.1 as constants to determine Mu/fckD3 from two charts. The results are
given in Table 10.13 below.

Table 10.13: Values of Mu/fckD3 when Pu/fckD2 = 0.52 and p/fck = 0.1

Sl.No. d’/D Mu/fckD3

1 0.1 0.09 (from Chart 44)


2 0.15 0.08 (from Chart 45)
3 0.13 0.084 (Interpolated value)

So, we get Mu/fckD3 = 0.084, giving Mu = (0.084)(25)(500)(500)(500) =


262.5 kNm > 250 kNm.

Hence, the column is safe to carry Pu = 3250 kN and Mu = 250 kNm.

10.25.10 Summary of this Lesson


This lesson explains the approximations, limitations and usefulness of the
three sets of design charts available in SP-16 for the purpose of solving analysis
and design types of reinforced concrete columns. The use of design charts has
been illustrated in several steps for the solution of both analysis and design types
of problems.

Several numerical problems in illustrative examples, practice problem and


test will help in understanding the use of design charts to solve the two types of
problems.

Version 2 CE IIT, Kharagpur


Module
10
Compression Members
Version 2 CE IIT, Kharagpur
Lesson
26
Short Compression
Members under
Axial Load with
Biaxial Bending
Version 2 CE IIT, Kharagpur
Instructional Objectives:

At the end of this lesson, the student should be able to:

• understand the behaviour of short columns under axial load and biaxial
bending,

• understand the concept of interaction surface,

• identify the load contour and interaction curves of Pu-Mu in a interaction


surface,

• mention the limitation of direct application of the interaction surface in


solving the problems,

• explain the simplified method of design and analysis of short columns


under axial load and biaxial bending,

• apply the IS code method in designing and analysing the reinforced


concrete short columns under axial load and biaxial bending.

10.26.1 Introduction
Beams and girders transfer their end moments into the corner columns of
a building frame in two perpendicular planes. Interior columns may also have
biaxial moments if the layout of the columns is irregular. Accordingly, such
columns are designed considering axial load with biaxial bending. This lesson
presents a brief theoretical analysis of these columns and explains the difficulties
to apply the theory for the design. Thereafter, simplified method, as
recommended by IS 456, has been explained with the help of illustrative
examples in this lesson.

Version 2 CE IIT, Kharagpur


10.26.2 Biaxial Bending

Figures 10.26.1a and b present column section under axial load and
uniaxial bending about the principal axes x and y, respectively. Figure 10.26.1c

Version 2 CE IIT, Kharagpur


presents the column section under axial load and biaxial bending. The
eccentricities ex and ey of Fig.10.26.1c are the same as those of Fig.10.26.1a (for
ex) and Fig.10.26.1b (for ey), respectively. Thus, the biaxial bending case (case
c) is the resultant of two uniaxial bending cases a and b. The resultant
eccentricity e, therefore, can be written as (see Fig.10.26.1c):

e = (e x2 + e y2 )1 / 2
(10.55)

Designating the moments of cases a, b and c by Mux, Muy and Mu, respectively,
we can write:

M u = ( M ux2 + M uy2 )1 / 2
(10.56)

and the resultant Mu is acting about an inclined axis, so that

tan θ = ex/ey = Muy/Mux


(10.57)

the angle of inclination θ is measured from y axis.

This inclined resultant axis shall also be the principal axis if the column
section including the reinforcing bars is axisymmetric. In such a situation, the
biaxial bending can be simplified to a uniaxial bending with the neutral axis
parallel to the resultant axis of bending.

The reinforced concrete column cross-sections are, in general, non-


axisymmetric with reference to the longitudinal axis and, therefore, the neutral
axis is not parallel to the resultant axis of bending ( θ is not equal to λ in
Fig.10.26.1c). Moreover, it is extremely laborious to find the location of the
neutral axis with successive trials. However, failure strain profile and stress block
can be drawn for a given location of the neutral axis. Figs.10.25.1d and e present
the strain profile and stress block, respectively, of the section shown in
Fig.10.25.1c.

Version 2 CE IIT, Kharagpur


10.26.3 Interaction Surface

Figure 10.26.2 can be visualised as a three-dimensional plot of Pu-Mux-


Muy, wherein two two-dimensional plots of Pu-Muy and Pu-Mus are marked as case
(a) and case (b), respectively. These two plots are the interaction curves for the
columns of Figs.10.26.1a and b, respectively. The envelope of several interaction
curves for different axes will generate the surface, known as interaction surface.

The interaction curve marked as case (c) in Fig.10.26.2, is for the column
under biaxial bending shown in Fig.10.26.1c. The corresponding axis of bending
is making an angle θ with the y axis and satisfies Eq.10.57. It has been
explained in Lesson 24 that a column subjected to a pair of P and M will be safe
if their respective values are less than Pu and Mu, given by its interaction curve.
Extending the same in the three-dimensional figure of interaction surface, it is
also acceptable that a column subjected to a set of Pu, Muy and Mux is safe if the
set of values lies within the surface. Since Pu is changing in the direction of z, let
us designate the moments and axial loads as mentioned below:

Muxz = design flexural strength with respect to major axis xx under biaxial
loading, when Pu = Puz,

Version 2 CE IIT, Kharagpur


Muyz = design flexural strength with respect to minor axis yy under biaxial
loading, when Pu = Puz,

Mux1 = design flexural strength with respect to major axis xx under uniaxial
loading, when Pu = Puz, and

Muy1 = design flexural strength with respect to minor axis yy under uniaxial
loading, when Pu = Puz.

The above notations are also shown in Fig.10.26.2.

All the interaction curves, mentioned above, are in planes perpendicular to


xy plane. However, the interaction surface has several curves parallel to xy
plane, which are planes of constant Pu. These curves are known as load contour,
one such load contour is shown in Fig.10.26.2, when Pu = Puz. Needless to
mention that the load is constant at all points of a load contour. These load
contour curves are also interaction curves depicting the interaction between the
biaxial bending capacities.

10.26.4 Limitation of Interaction Surface


The main difficulty in preparing an exact interaction surface is that the
neutral axis for the case (c) of Fig.10.26.1c will not, in general, be perpendicular
to the line joining the loading point Pu and the centre of the column
(Fig.10.26.1c). This will require several trials with c and λ , where c is the
distance of the neutral axis and λ angle made by the neutral axis with the x axis,
as shown in Fig.10.26.1c. Each trial will give a set of Pu, Mux and Muy. Only for a
particular case, the neutral axis will be perpendicular to the line joining the load
point Pu to the centre of the column. This search makes the process laborious.
Moreover, several trials with c and λ , giving different values of h (see
Fig.10.26.1c), may result in a failure surface with wide deviations, particularly as
the value of Pu will be increasing.

Accordingly, the design of columns under axial load with biaxial bending is
done by making approximations of the interaction surface. Different countries
adopted different approximate methods. Clause 39.6 of IS 456 recommends one
method based on Bresler's formulation, also known as "Load Contour Method",
which is taken up in the following section. (For more information, please refer to:
"Design Criteria for Reinforced Columns under Axial Load and Biaxial Bending",
by B. Bresler, J. ACI, Vol.32, No.5, 1960, pp.481-490).

10.26.5 IS Code Method for Design of Columns under Axial


Load and Biaxial Bending

Version 2 CE IIT, Kharagpur


IS 456 recommends the following simplified method, based on Bresler's
formulation, for the design of biaxially loaded columns. The relationship between
Muxz and Muyz for a particular value of Pu = Puz, expressed in non-dimensional
form is:

( M ux / M ux1 )α n + ( M uy / M uy1 )α n ≤ 1
(10.58)

where Mux and Muy = moments about x and y axes due to design loads, and

α n is related to Pu/Puz, (Fig.10.26.3), where

Puz = 0.45 fck Ac + 0.75 fy Asc

= 0.45 Ag + (0.75 fy - 0.45 fck) Asc


(10.59)

where Ag = gross area of the section, and

Asc = total area of steel in the section

Muxz, Muyz, Mux1 and Muy1 are explained in sec.10.26.3 earlier.

It is worth mentioning that the quantities Mux, Muy and Pu are due to
external loadings applied on the structure and are available from the analysis,
whereas Mux1, Muy1 and Puz are the capacities of the column section to be
considered for the design.

Equation 10.58 defines the shape of the load contour, as explained earlier
(Fig.10.26.2). That is why the method is also known as "Load Contour Method".
The exponent α n of Eq.10.58 is a constant which defines the shape of the load

Version 2 CE IIT, Kharagpur


contour and depends on the value of Pu. For low value of the axial load, the load
contour is approximated as a straight line and, in that case, α n = 1. On the other
hand, for high values of axial load, the load contour is approximated as a
quadrant of a circle, when α n = 2. For intermediate load values, the value of α n
lies between 1 and 2. Chart 64 of SP-16 presents the load contour and
Fig.10.26.3 presents the relationship between α n and Pu/Puz. The mathematical
relationship between α n and Pu/Puz is as follows:

α n = 1.0, when Pu/Puz ≤ 0.2

α n = 0.67 + 1.67 Pu/Puz, when 0.2 < (Pu/Puz) < 0.8

α n = 2.0, when (Pu/Puz) ≥ 0.8


(10.60)

10.26.6 Solution of Problems using IS Code Method


The IS code method, as discussed in sec.10.26.5, can be employed to
solve both the design and analysis types of problems. The only difference
between the design and analysis type of problems is that a trial section has to be
assumed including the percentage of longitudinal reinforcement in the design
problems. However, these data are available in the analysis type of problems.
Therefore, a guide line is given in this section for assuming the percentage of
longitudinal reinforcement for the design problem. Further, for both types of
problems, the eccentricities of loads are to be verified if they are more than the
corresponding minimum eccentricities, as stipulated in cl.25.4 of IS 456.
Thereafter, the relevant steps are given for the solution of the two types of
problems.

(a) Selection of trial section for the design type of problems

As mentioned in sec.10.24.2(i) of Lesson 24, the preliminary dimensions


are already assumed during the analysis of structure (mostly statically
indeterminate). Thus, the percentage of longitudinal steel is the one parameter to
be assumed from the given Pu, Mux, Muy, fck and fy. Pillai and Menon (Ref. No. 4)
suggested a simple way of considering a moment of approximately 15 per cent in
excess (lower percentage up to 5 per cent if Pu/Puz is relatively high) of the
resultant moment

M u = (1.15) ( M ux2 + M uy2 )1 / 2


(10.61)

as the uniaxial moment for the trial section with respect to the major principal axis
xx, if Mux ≥ Muy; otherwise, it should be with respect to the minor principal axis.
Version 2 CE IIT, Kharagpur
The reinforcement should be assumed to be distributed equally on four sides of
the section.

(b) Checking the eccentricities ex and ey for the minimum eccentricities

Clause 25.4 of IS 256 stipulates the amounts of the minimum


eccentricities and are given in Eq.10.3 of sec.10.21.11 of Lesson 21. However,
they are given below as a ready reference.

exmin ≥ greater of (l/500 + b/30) or 20 mm


…. (10.3)

eymin ≥ greater of (l/500 + D/30) or 20 mm

where l, b and D are the unsupported length, least lateral dimension and larger
lateral dimension, respectively. The clause further stipulates that for the biaxial
bending, it is sufficient to ensure that the eccentricity exceeding the minimum
value about one axis at a time.

(c) Steps for the solution of problems

The following are the steps for the solution of both analysis and design
types of problems while employing the method recommended by IS 456.

(i) Verification of eccentricities

It is to be done determining ex = Mux/Pu and ey = Muy/Pu from the given


data of Pu, Mux and Muy; and exmin and eymin from Eq.10.3 from the assumed b and
D and given l.

(ii) Assuming a trial section including longitudinal reinforcement

This step is needed only for the design type of problem, which is to be
done as explained in (a) above.

(iii) Determination of Mux1 and Muy1

Use of design charts should be made for this. Mux1 and Muy1,
corresponding to the given Pu, should be significantly greater than Mux and Muy,
respectively. Redesign of the section should be done if the above are not
satisfied for the design type of problem only.

(iv) Determination of Puz and α n

The values of Puz and α n can be determined from Eqs.10.59 and 10.60,
respectively. Alternatively, Puz can be obtained from Chart 63 of SP-16.

Version 2 CE IIT, Kharagpur


(v) Checking the adequacy of the section

This is done either using Eq.10.58 or using Chart 64 of SP-16.

10.26.7 Illustrative Example

Problem 1:

Design the reinforcement to be provided in the short column of Fig.10.26.4


is subjected to Pu = 2000 kN, Mux = 130 kNm (about the major principal axis) and
Muy = 120 kNm (about the minor principal axis). The unsupported length of the
column is 3.2 m, width b = 400 mm and depth D = 500 mm. Use M 25 and Fe
415 for the design.

Solution 1:

Step 1: Verification of the eccentricities

Given: l = 3200 mm, b = 400 mm and D = 500 mm, we have from Eq.10.3
of sec.10.26.6b, the minimum eccentricities are:

exmin = greater of (3200/500 + 400/30) and 20 mm = 19.73 mm or 20 mm = 20


mm

Version 2 CE IIT, Kharagpur


eymin = greater of (3200/500 + 500/30) and 20 mm = 23.07 mm or 20 mm =
23.07 mm

Again from Pu = 2000 kN, Mux = 130 kNm and Muy = 120 kNm, we have ex =
Mux/Pu = 130(106)/2000(103) = 65 mm and ey = Muy/Pu = 120(106)/2000(103) = 60
mm. Both ex and ey are greater than exmin and eymin, respectively.

Step 2: Assuming a trial section including the reinforcement

We have b = 400 mm and D = 500 mm. For the reinforcement,


Mu = 1.15 ( M ux2 + M uy2 )1 / 2 , from Eq.10.61 becomes 203.456 kNm. Accordingly, we
get

Pu/fckbD = 2000(103)/(25)(400)(500) = 0.4

Mu/fckbD2 = 203.456(106)/(25)(400)(500)(500) = 0.0814

Assuming d' = 60 mm, we have d'/D = 0.12. From Charts 44 and 45, the value of
p/fck is interpolated as 0.06. Thus, p = 0.06(25) = 1.5 per cent, giving Asc = 3000
mm2. Provide 12-20 mm diameter bars of area 3769 mm2, actual p provided =
1.8845 per cent. So, p/fck = 0.07538.

Step 3: Determination of Mux1 and Muy1

We have Pu/fckbD = 0.4 and p/fck = 0.07538 in step 2. Now, we get


Mux1/fckbD2 from chart corresponding to d' = 58 mm (Fig.10.26.4) i.e., d'/D =
0.116. We interpolate the values of Charts 44 and 45, and get Mux1/fckbD2 =
0.09044. So, Mux1 = 0.0944(25)(400)(500(500)(10-6) = 226.1 kNm.

For Mux1, d'/b = 58/400 = 0.145. In a similar manner, we get Muy1 =


0.0858(25)(400)(400)(500)(10-6) = 171.6 kNm.

As Mux1 and Muy1 are significantly greater than Mux and Muy, respectively,
redesign of the section is not needed.

Step 4: Determination of Puz and α n

From Eq.10.59, we have Puz = 0.45(25)(400)(500) + {0.75(415) -


0.45(25)}(3769) = 3380.7 kN.

Version 2 CE IIT, Kharagpur


Alternatively, Chart 63 may be used to find Puz as explained. From the upper
section of Chart 63, a horizontal line AB is drawn at p = 1.8845, to meet the Fe
415 line B (Fig.10.26.5). A vertical line BC is drawn from B to meet M 25 line at
C. Finally, a horizontal line CD is drawn from C to meet Puz/Ag at 17. This gives
Puz = 17(400)(500) = 3400 kN. The difference between the two values, 19.3 kN is
hardly 0.57 per cent, which is due to the error in reading the value from the chart.
However, any one of the two may be employed.

Now, the value of α n is obtained from Eq.10.60 for Pu/Puz = 2000/3380.7 =


0.5916, i.e., 0.2 < Pu/Puz < 0.8, which gives, α n = 0.67 + 1.67 (Pu/Puz) = 1.658.
Alternatively, α n may be obtained from Fig.10.26.3, drawn to scale.

Step 5: Checking the adequacy of the section

Using the values of Mux, Mux1, Muy, Muy1 and α n in Eq.10.58, we have
(130/226.1)1.658 + (120/171.6)1.658 = 0.9521 < 1.0. Hence, the design is safe.

Version 2 CE IIT, Kharagpur


Alternatively, Chart 64 may be used to determine the point (Mux/Mux1),
(Muy/Muy1) is within the curve of Pu/Puz = 0.5916 or not.

Here, Mux/Mux1 = 0.5749 and Muy/Muy1 = 0.6993. It may be seen that the
point is within the curve of Pu/Puz = 0.5916 of Chart 64 of SP-16.

Step 6: Design of transverse reinforcement

As per cl.26.5.3.2c of IS 456, the diameter of lateral tie should be > (20/4)
mm diameter. Provide 8 mm diameter bars following the arrangement shown in
Fig.10.26.4. The spacing of lateral tie is the least of :

(a) 400 mm = least lateral dimension of column,

(b) 320 mm = sixteen times the diameter of longitudinal reinforcement (20


mm),

(c) 300 mm

Accordingly, provide 8 mm lateral tie alternately @ 250 c/c (Fig.10.26.4).

10.26.8 Practice Questions and Problems with Answers


Q.1: Explain the behaviour of a short column under biaxial bending as the
resultant of two uniaxial bending.

A.1: See sec. 10.26.2

Q.2: Draw one interaction surface for a short column under biaxial bending and
show typical interaction curves and load contour curve. Explain the safety
of a column with reference to the interaction surface when the column is
under biaxial bending.

A.2: See sec.10.26.3 and Fig.10.26.2.

Q.3: Discuss the limitation of the interaction curve.

A.3: See sec.10.26.4.

Q.4: Illustrate the IS code method of design of columns under biaxial bending.

A.4: See sec.10.26.5.

Version 2 CE IIT, Kharagpur


Q.5:

Analyse the safety of the short column of unsupported length 3.2 m, b =


450 mm, D = 500 mm, as shown in Fig.10.26.6, having 12-16 mm
diameter bars as longitudinal reinforcement and 8 mm diameter bars as
lateral tie @ 250 mm c/c, when subjected to Pu = 1600 kN, Mux = 120 kNm
and Muy = 100 kNm. Use M 25 and Fe 415.

A.5:

Step 1: Verification of the eccentricities

From the given data: l = 3200 mm, b = 450 mm and D = 500 mm,

exmin = 3200/500 + 450/30 = 21.4 > 20 mm, so, 21.4 mm

eymin = 3200/500 + 5000/30 = 23.06 > 20 mm, so, 23.06 mm

ex = Mux/Pu = 120(103)/1600 = 75 mm

ey = Muy/Pu = 100(103)/1600 = 62.5 mm

So, the eccentricities ex and ey are >> exmin and eymin.

Step 2: Determination of Mux1 and Muy1

Version 2 CE IIT, Kharagpur


Given data are: b = 450 mm, D = 500 mm, fck = 25 N/mm2, fy = 415
N/mm , Pu = 1600 kN, Mux = 120 kNm, Muy = 100 kNm and Asc = 2412 mm2 (12-
2

16 mm diameter bars).

We have p = (100)(2412)/(450)(500) = 1.072 per cent, and d'/D = 56/500 =


0.112, d'/b = 56/450 = 0.124, Pu/fckbD = 1600/(25)(450)(500) = 0.2844 and p/fck =
1.072/25 = 0.043. We get Mux1/fckbD2 from Charts 44 and 45 as 0.09 and 0.08,
respectively. Linear interpolation gives Mux1/fckbD2 for d'/D = 0.112 as 0.0876.
Thus,

Mux1 = (0.0876)(25)(450)(500)(500) = 246.376 kNm

Similarly, interpolation of values (0.09 and 0.08) from Charts 44 and 45, we get
Muy1/fckdb2 = 0.085 for d'/b = 0.124. Thus

Muy1 = (0.085)(25)(500)(450)(450) = 215.156 kNm

Step 3: Determination of Puz and α n

From Eq.10.59, Puz = 0.45(25)(450)(500) + {0.75(415) - 0.45(25)}(2412) =


3254.85 kN. This gives Pu/Puz = 1600/3254.85 = 0.491574.

From Eq.10.60, α n = 0.67 + 1.67(Pu/Puz) = 0.67 + 1.67(0.491574) =


1.4909.

Step 4: Checking the adequacy of the section

From Eq.10.58, we have: (120/246.376)1.4909 + (100/215.156)1.4909 =


0.6612 < 1.

Hence, the section is safe to carry Pu = 1600 kN, Mux = 120 kNm and Muy
= 100 kNm.

10.26.9 References
1. Reinforced Concrete Limit State Design, 6th Edition, by Ashok K. Jain,
Nem Chand & Bros, Roorkee, 2002.
2. Limit State Design of Reinforced Concrete, 2nd Edition, by P.C.Varghese,
Prentice-Hall of India Pvt. Ltd., New Delhi, 2002.
3. Advanced Reinforced Concrete Design, by P.C.Varghese, Prentice-Hall of
India Pvt. Ltd., New Delhi, 2001.
4. Reinforced Concrete Design, 2nd Edition, by S.Unnikrishna Pillai and
Devdas Menon, Tata McGraw-Hill Publishing Company Limited, New
Delhi, 2003.

Version 2 CE IIT, Kharagpur


5. Limit State Design of Reinforced Concrete Structures, by P.Dayaratnam,
Oxford & I.B.H. Publishing Company Pvt. Ltd., New Delhi, 2004.
6. Reinforced Concrete Design, 1st Revised Edition, by S.N.Sinha, Tata
McGraw-Hill Publishing Company. New Delhi, 1990.
7. Reinforced Concrete, 6th Edition, by S.K.Mallick and A.P.Gupta, Oxford &
IBH Publishing Co. Pvt. Ltd. New Delhi, 1996.
8. Behaviour, Analysis & Design of Reinforced Concrete Structural Elements,
by I.C.Syal and R.K.Ummat, A.H.Wheeler & Co. Ltd., Allahabad, 1989.
9. Reinforced Concrete Structures, 3rd Edition, by I.C.Syal and A.K.Goel,
A.H.Wheeler & Co. Ltd., Allahabad, 1992.
10. Textbook of R.C.C, by G.S.Birdie and J.S.Birdie, Wiley Eastern Limited,
New Delhi, 1993.
11. Design of Concrete Structures, 13th Edition, by Arthur H. Nilson, David
Darwin and Charles W. Dolan, Tata McGraw-Hill Publishing Company
Limited, New Delhi, 2004.
12. Concrete Technology, by A.M.Neville and J.J.Brooks, ELBS with
Longman, 1994.
13. Properties of Concrete, 4th Edition, 1st Indian reprint, by A.M.Neville,
Longman, 2000.
14. Reinforced Concrete Designer’s Handbook, 10th Edition, by C.E.Reynolds
and J.C.Steedman, E & FN SPON, London, 1997.
15. Indian Standard Plain and Reinforced Concrete – Code of Practice (4th
Revision), IS 456: 2000, BIS, New Delhi.
16. Design Aids for Reinforced Concrete to IS: 456 – 1978, BIS, New Delhi.

10.26.10 Test 26 with Solutions


Maximum Marks = 50, Maximum Time = 30 minutes

Answer all questions.

Version 2 CE IIT, Kharagpur


TQ.1:

Analyse the safety of the short square column of unsupported length =


3.5 m, b = D = 500 mm, as shown in Fig.10.26.7, with 12-16 mm
diameter bars as longitudinal reinforcement and 8 mm diameter bars as
lateral tie @ 250 mm c/c, when subjected to Pu = 1800 kN, Mux = 160
kNm and Muy = 150 kNm.

A.TQ.1:

Step 1: Verification of the eccentricities

From the given data: l = 3500 mm, b = D = 500 mm, we have

emin in both directions (square column) = (3500/500) + (500/30) = 23.67


mm

ex = 160(103)/1800 = 88.88 mm and ey = 150(103)/1800 = 83.34 mm

Therefore, ex and ey >> emin.

Version 2 CE IIT, Kharagpur


Step 2: Determination of Mux1 and Muy1

We have the given data: b = D = 500 mm, fck = 25 N/mm2, fy = 415 N/mm2,
Pu = 1800 kN, Mux = 160 kNm, Muy = 150 kNm and Asc = 2412 mm2 (12-16 mm
diameter bars).

The percentage of longitudinal reinforcement p = 241200/(500)(500) =


0.9648 per cent, and d'/D = 56/500 = 0.112 and p/fck = 0.9648/25 = 0.03859.
Linear interpolation of values of Mux1/fckbD2 from Charts 44 and 45 for d'/D =
0.112 is obtained as 0.08. Thus,

Mux1 = (0.08)(25)(500)(500)(500) = 250 kNm

Muy1 = Mux1 = 250 kNm (square column)

Step 3: Determination of Puz and α n

From Eq.10.59,

Puz = 0.45(25)(500)(500) + {0.75(415) - 0.45(25)}(2415) = 3536.1 kN.

Pu/Puz = 1800/3536.1 = 0.509.

From Eq.10.60, α n = 0.67 + 1.67(0.509) = 1.52.

Step 4: Checking the adequacy of the section

From Eq.10.58, we have: (160/250)1.52 + (150/250)1.52 = 0.967 < 1.

Hence, the section can carry Pu = 1800 kN, Mux = 160 kNm and Muy = 150
kNm.

10.26.11 Summary of this Lesson


This lesson explains the behaviour of short columns under axial
load and biaxial bending with the help of interaction surface, visualised as a
three-dimensional plot of Pu-Mux-Muy. The interaction surface has a set of
interaction curves of Pu-Mu and another set of interaction curves of Muxz-Muyz at
constant Puz, also known as load contour. The design and analysis of short
columns are also explained with the help of derived equations and design charts
of SP-16. Numerical examples in the illustrative example, practice problems and
test will help in understanding the application of the theory in solving the analysis
and design types of problems of short columns under axial load and biaxial
bending.

Version 2 CE IIT, Kharagpur


Module
10
Compression Members
Version 2 CE IIT, Kharagpur
Lesson
27
Slender Columns
Version 2 CE IIT, Kharagpur
Instructional Objectives:
At the end of this lesson, the student should be able to:

• define a slender column,

• give three reasons for its increasing importance and popularity,

• explain the behaviour of slender columns loaded concentrically,

• explain the behaviour of braced and unbraced single column or a part of


rigid frame, bent in single or double curvatures,

• roles and importance of additional moments due to P- Δ effect and


moments due to minimum eccentricities in slender columns,

• identify a column if sway or nonsway type,

• understand the additional moment method for the design of slender


columns,

• apply the equations or use the appropriate tables or charts of SP-16 for
the complete design of slender columns as recommended by IS 456.

11.27.1 Introduction
Slender and short are the two types of columns classified on the basis of
slenderness ratios as mentioned in sec.10.21.5 of Lesson 21. Columns having
both lex/D and ley/b less than twelve are designated as short and otherwise, they
are slender, where lex and ley are the effective lengths with respect to major and
minor axes, respectively; and D and b are the depth and width of rectangular
columns, respectively. Short columns are frequently used in concrete structures,
the design of such columns has been explained in Lessons 22 to 26, loaded
concentrically or eccentrically about one or both axes. However, slender columns
are also becoming increasingly important and popular because of the following
reasons:

(i) the development of high strength materials (concrete and steel),

(ii) improved methods of dimensioning and designing with rational and


reliable design procedures,

Version 2 CE IIT, Kharagpur


(iii) innovative structural concepts – specially, the architect’s expectations
for creative structures.

Accordingly, this lesson explains first, the behaviour of slender elastic


columns loaded concentrically. Thereafter, reinforced concrete slender columns
loaded concentrically or eccentrically about one or both axes are taken up. The
design of slender columns has been explained and illustrated with numerical
examples for easy understanding.

10.27.2 Concentrically Loaded Columns


It has been explained in Lessons 22 to 26 that short columns fail by
reaching the respective stresses indicating their maximum carrying capacities.
On the other hand, the slender or long columns may fail at a much lower value of
the load when sudden lateral displacement of the member takes place between
the ends. Thus, short columns undergo material failure, while long columns may
fail by buckling (geometric failure) at a critical load or Euler’s load, which is much
less in comparison to that of short columns having equal area of cross-section.
The buckling load is termed as Euler’s load as Euler in 1744 first obtained the
value of critical load for various support conditions. For more information, please
refer to Additamentum, “De Curvis elasticis”, in the “Methodus inveiendi Lineas
Curvas maximi minimive proprietate gaudentes” Lausanne and Geneva, 1744.
An English translation of this work is given in Isis No.58, Vol.20, p.1, November
1933.

The general expression of the critical load Pcr at which a member will fail
by buckling is as follows:

Pcr = π2EI /(kl)2

where E is the Young’s modulus I is the moment of inertia about the axis of
bending, l is the unsupported length of the column and k is the coefficient whose
value depends on the degree of restraints at the supports. Expressing moment of
inertia I = Ar2, where A is the area of cross-section of the column and r is the
radius of gyration, the above equations can be written as,

Pcr = π2EA /(kl/r)2


(10.62)

Thus, Pcr of a particular column depends upon kl/r or slenderness ratio. It is worth
mentioning that kl is termed as effective length le of the column.

Version 2 CE IIT, Kharagpur


Figures 10.27.1 and 2 show two elastic slender columns having hinge
supports at both ends and fixed supports against rotation at both ends,

Version 2 CE IIT, Kharagpur


respectively. Figure 10.27.3 presents a column of real structure whose end
supports are not either hinged or fixed. It has supports partially restrained against
rotation by the top and bottom beams. Each of the three figures shows the
respective buckled shape, points of inflection PIs (points of zero moment), the
distance between the PIs and the value of k. All the three columns, having
supports at both ends, have the k values less than one or at most one. By
providing supports at both ends, one end of the column is prevented from
undergoing lateral movement or sidesway with respect to the other end.

Version 2 CE IIT, Kharagpur


Version 2 CE IIT, Kharagpur
However, cantilever columns are entirely free at one end, as shown in
Fig.10.27.4. Figure 10.27.5 shows another type of column, rotationally fixed at
both ends but one end can move laterally with respect to the other. Like that of
Fig.10.27.3, a real column, not hinged, fixed or entirely free but restrained by top
and bottom beams, where sideway can also take place. Each of these three
figures, like those of Figs.10.27.1 to 3, presents the respective buckled shape,
points of inflection (PIs), if any, the distance between the PIs and the value of k.
All these columns have the respective k values greater than one or at least one.

Version 2 CE IIT, Kharagpur


Figures 10.27.7 and 8 present two reinforced concrete portal frames, a
typical reinforced concrete rigid frame. Columns of Fig.10.27.7 are prevented
from sidesway and those of Fig.10.27.8 are not prevented from sidesway,
respectively, when subjected to concentric loadings. The buckled configuration of
the frame, prevented from sidesway (Fig.10.27.7) is similar to that of Fig.10.27.3,

Version 2 CE IIT, Kharagpur


except that the lower ends of the portal frame are hinged. One of the two points
of inflection (PIs) is at the lower end of the column, while the other PI is slightly
below the upper end of the column, depending on the degree of restraint. The
value of k for such a frame is thus less than 1. The critical load is, therefore,
slightly more than Pcr of the hinge-hinge column of Fig.10.27.1. The buckled
configuration of the other portal frame of Fig.10.27.8, where sidesway is not
prevented, is similar to the column of Fig.10.27.4 when it is made upside down,
except that the upper end is not fixed but partially restrained by the supporting
beam. In this case, the value of k exceeds 2, depending on the degree of
restraint. One of the two PIs is at the bottom of the column. The critical load of
the column of Fig.10.27.8 is much less than that of the column of Fig.10.27.1.

Table 10.14: Critical loads in terms of Pcr of hinge-hinge column and effective
lengths le = kl of elastic and reinforced concrete columns with different boundary
conditions and for a constant unsupported length l

Sl. Support conditions Critical load Effective length Fig. No.


No. Pcr le = kl
(A) Elastic single columns
1. Hinged at both ends, no Pcr l 10.27.1
sidesway
2. Fixed against rotation at 4Pcr 0.5 l 10.27.2
both ends – no sidesway
3. Partially restrained Between Pcr l > kl > l/2 10.27.3
against rotation by top and 4Pcr
and bottom cross-
beams, no sidesway
4. Fixed at one end and 0.25 Pcr 2 l, one PI is on 10.27.4
entirely free at other end imaginary
– sidesway not extension
prevented
5. Rotationally fixed at both Pcr l, one PI is on 10.27.5
ends – sidesway not imaginary
prevented extension
6. Partially restrained Between zero l < kl < α 10.27.6
against rotation at both and slightly
ends – sidesway not less than Pcr *
prevented
(B) Reinforced concrete columns
7. Hinged portal frame – no > Pcr kl < l 10.27.7
sidesway
8. Hinged portal frame – << Pcr kl > 2 l 10.27.8
sidesway not prevented

Notes: 1. Buckled shapes are half sine wave between two points of inflection
(PIs).

Version 2 CE IIT, Kharagpur


2. * The critical load is slightly less than Pcr of hinge-hinge column
(Sl.No.1), when cross-beams are very rigid compared to columns, i.e.,
the case under Sl.No.6 approaches the case under Sl.No.1.

The critical load is zero when cross-beams are very much


flexible compared to columns, i.e., the case under Sl.No.6 approaches
to hinge-hinge column of Sl.No.1, allowing sidesway. In that case, it
becomes unstable and hence, carries zero load.

Table 10.14 presents the critical load in terms of that of hinge-hinge


column Pcr and effective lengths le (equal to the distance between two points of
inflection PIs = kl) of elastic and reinforced concrete columns for a constant value
of the unsupported length l.

The stress-strain curve of concrete, as shown in Fig.1.2.1 of Lesson 2,


reveals that the initial tangent modulus of concrete Ec is much higher than Et
(tangent modulus at higher stress level). Taking this into account in Eq.10.62,
Fig.10.27.9 presents a plot of buckling load Pcr versus kl/r. It is evident from the
plot that the critical load is reducing with increasing slenderness ratio. For very
short columns, the limiting factored concentric load estimated from Eq.10.39 of
Lesson 24 will be found to be less than the critical load, determined from
Eq.10.62. The column, therefore, will fail by direct crushing and not by buckling.
We can also find out the limiting value of kl/r when the crushing load and the
buckling load are the same. The (kl/r)lim is shown in Fig.10.27.9. The limiting
value of kl/r also indicates that a column having kl/r more than (kl/r)lim will fail by

Version 2 CE IIT, Kharagpur


buckling, while columns having any value of kl/r less than (kl/r)lim will fail by
crushing of concrete.

The following are the observations of the discussions about the


concentrically loaded columns:

1. As the slenderness ratio kl/r increases, the strength of concentrically


loaded column decreases.

2. The effective length of columns either in single members or parts of


rigid frames is between 0.5l and l, if the columns are prevented from sidesway by
bracing or otherwise. The actual value depends on the degree of end restraints.

3. The effective length of columns either in single members or parts of


rigid frames is always greater than one, if the columns are not prevented from
sidesway. The actual value depends on the degree of end restraints.

4. The critical load of braced frame against sidesway is always


significantly larger than that of the unbraced frame.

Version 2 CE IIT, Kharagpur


10.27.3 Slender Columns under Axial Load and Uniaxial
Moment
(A) Columns bent in single curvature

Figure 10.27.10a shows a column bent in single curvature under axial load
P less than its critical load Pcr with constant moment Pe. The deflection profile
marked by dotted line is due to the constant moment. However, there will be
additional moment of Py at a distance z from the origin (at the bottom of column)
which will deflect the column further, as shown by the solid line. The constant
moment Pe and additional moment Py are shown in Fig.10.27.10b. Thus, the
total moment becomes

M = Mo + Py = P(e + y)
(10.63)

The maximum moment is P(e + Δ ) at the mid-height of the column. This, we can
write

Mmax = Mo + P Δ = P(e + Δ )
(10.64)

This is known as P - Δ effect.

Version 2 CE IIT, Kharagpur


Figure 10.27.11a shows another column whose bending is caused by a
transverse load H. The bending moment at a distance z from the origin (bottom
of the column) is Hz/2 causing deflection of the column marked by dotted line in
the figure. The axial load P, less than its critical load Pcr, causes additional
moment resulting in further deflection, marked by solid line in the figure. This
additional deflection produces additional moment of Py at a section z from the
origin. The two bending moment diagrams are shown in Fig.10.27.11b. Here
again, the total moment is

M = Mo + Py = Hz/2 + Py
(10.65)

The maximum moment at the mid-height of the column is

Mmax = Mo,max + P Δ = Hl/4 + P Δ


(10.66)

The total moment in Eqs.10.63 and 10.65 consists of the moment Mo that
acts in the presence of P and the additional moment caused by P (= Py). The
deflections y can be computed from yo, the deflections without the axial load
from the expression

y = yo[1/{1 – (P/Pcr)}]
(10.67)

From Eq.10.64, we have

Mmax = Mo + P Δ = Mo + P Δ o[1/{1 – (P/Pcr)}]


(10.68)

Equation 10.68 can be written as

1 + ψ (P / Pcr )
M max = M o
1 - (P / Pcr )
(10.69)

where ψ depends on the type of loading and generally varies between ± 0.20.
Since P/Pcr is always less than one, we can ignore ψ (P/Pcr) term of Eq.10.69, to
have

Mmax = Mo/{1 – (P/Pcr)}


(10.70)

where 1/{1 – (P/Pcr)} is the moment magnification factor. In both the cases above
(Figs.10.27.10 and 11), a direct addition of the maximum moment caused by

Version 2 CE IIT, Kharagpur


transverse load or otherwise, to the maximum moment caused by P gives the
total maximum moment as that is the most unfavourable situation. However, this
is not the case for situation taken up in the following.

(B) Columns bent in double curvature

Figure 10.27.12a shows a column subjected to equal end moment of


opposite signs. From the moment diagrams Mo and Py (Figs.10.27.12b and c), it
is clear that though Mo moments are maximum at the ends, the Py moments are
maximum at some distance from the ends. The total moment can be either as
shown in d or in e of Fig.10.27.12. In case of Fig.10.27.12d, the maximum
moment remains at the ends and in Fig.10.27.12e, the maximum moment is at
some distance from the ends, where Mo is comparatively smaller than Mo max at
the ends. Accordingly, the total maximum moment is moderately higher than Mo
max.

From the above, it is evident that the moment Mo will be magnified most
strongly if the section of Mo max coincides with the section of maximum value of y,
as in the case of column bent in single curvature of Figs.10.27.10 and 11.
Similarly, if the two moments are unequal but of same sign as in Fig.10.27.10,
the moment Mo will be magnified but not so much as in Fig.10.27.10. On the
other hand, if the unequal end moments are of opposite signs and cause bending
in double curvature, there will be little or no magnification of Mo moment.

This dependence of moment magnification on the relative magnitudes of


the two moments can be expressed by modifying the earlier Eq.10.70 as

Version 2 CE IIT, Kharagpur


Mmax = Mo Cm/{1 – (P/Pcr)}
(10.71)

where Cm = 0.6 + 0.4(M1/M2) ≥ 0.4


(10.72)

The moment M1 is smaller than M2 and M1/M2 is positive if the moments produce
single curvature and negative if they produce double curvature. It is further seen
from Eq.10.72 that Cm = 1, when M1 = M2 and in that case, Eq.10.71 becomes
the same as Eq.10.70.

For the column of Fig.10.27.12a, the deflections caused by Mo are


magnified when axial load P is applied. The deflection can be obtained from

y = yo [1/{1 – (P/4Pcr)}]
(10.73)

(C) Portal frame laterally unbraced and braced

Here, the sidesway can occur only for the entire frame simultaneously. A
fixed portal frame, shown in Fig.10.27.13a, is under horizontal load H and
compression force P. The moments due to H and P and the total moment
diagrams are shown in Fig.10.27.13b, c and d, respectively. The deformations of
the frame due to H are shown in Fig.10.27.13a by dotted curves, while the solid
curves are the magnified deformations. It is observed that the maximum values
of positive and negative Mo are at the ends of the column where the maximum

Version 2 CE IIT, Kharagpur


values of positive and negative moments due to P also occur. Thus, the total
moment shall be at the ends as the two effects are fully additive.

Figure 10.27.14a shows a fixed portal frame, laterally braced so that no


sidesway can occur. Figures 10.27.14b and c show the moments Mo and due to
P. It is seen that the maximum values of the two different moments do not occur
at the same location. As a result, the magnification of the moment either may not
be true or shall be small.

(D) Columns with different slenderness ratios

Version 2 CE IIT, Kharagpur


Figure 10.27.15 shows the interaction diagram of P and M at the mid-
height section of the column shown in Fig.10.27.10. Three loading paths OA, OB
and OC are also shown in the figure for three columns having the same cross-
sectional area and the eccentricity of loads but with different slenderness ratios.
The three columns are loaded with increasing P and M (at constant eccentricity)
up to failure. The loading path OA is linear indicating Δ = 0, i.e., for a very short
column. It should be noted that Δ should be theoretically zero only when either
the effective length or the eccentricity is zero. In a practical short column,
however, some lateral deflection shall be there, which, in turn will cause
additional moment not more than five per cent of the primary moment and may
be neglected. The loading path OA terminates at point A of the interaction
diagram, which shows the failure load Psc of the short column with moment Msc =
Psc e. The short column fails by crushing of concrete at the mid-height section.
This type of failure is designated as material failure, either a tension failure or a
compression failure depending on the location of the point A on the interaction
curve.

The load path OB is for a long column, where the deflection Δ caused by
increasing value of P is significant. Finally, the long column fails at load Plc and
moment Mlc = Plc(e + Δ ). The loading path OB further reveals that the secondary
moment Plc Δ is comparable to the primary moment Plc e. Moreover, the failure
load and the primary moment of the long column Plc and Plc e, respectively, are
less than those of the short column (Psc and Psc e, respectively), though both the
columns have the same cross-sectional areas and eccentricities but different
slenderness ratios. Here also, the mid-height section of the column undergoes
material failure, either a compression failure or a tension failure, depending on
the location of the point B on the interaction diagram.

Version 2 CE IIT, Kharagpur


The loading path OC, on the other hand, is for a very long column when
the lateral deflection Δ is so high that the slope of the path dP/dM at C is zero.
The column is so slender that the failure is due to buckling (instability) at a
comparatively much low value of the load Pcr, though this column has the same
cross-sectional area and the eccentricity of load as of the other two columns.
Such instability failure occurs for very slender columns, specially when they are
not braced.

The following points are summarised from the discussion made in


sec.10.27.3.

1. Additional deflections and moments are caused by the axial


compression force P in columns. The additional moments increase with the
increase of kl/r, when other parameters are equal.

2. Laterally braced compression members and bent in single curvature


have the same or nearby locations of the maxima of both Mo and Py. Thus, being
fully additive, they have large moment magnification.

3. Laterally braced compression members and bent in double curvature


have different locations of the maxima of both Mo and Py. As a result, the
moment magnification is either less or zero.

4. Members of frames not braced laterally, the maxima of Mo and Py


mostly occur at the ends of column and cause the maximum total moment at the
ends of columns only. Additional moments and additional deflections increase
with the increase of kl/r.

10.27.4 Effective Length of Columns


Annex E of IS 456 presents two figures (Figs.26 and 27) and a table
(Table 26) to estimate the effective length of columns in frame structures based
on a research paper, “Effective length of column in multistoreyed building” by
R.H. Wood in The Structural Engineer Journal, No.7, Vol.52, July 1974. Figure
26 is for columns in a frame with no sway, while Fig.27 is for columns in a frame
with sway. These two figures give the values of k (i.e., le/l) from two parameters
β1 and β 2 which are obtained from the following expression:

β = ∑K /∑K + ∑K
c c b

(10.74)

where Kc and Kb are flexural stiffnesses of columns and beams, respectively. The
quantities β1 and β 2 at the top and bottom joints A and B, respectively, are
determined by summing up the K values of members framing into a joint at top

Version 2 CE IIT, Kharagpur


and bottom, respectively. Thus β1 and β 2 for the frame shown in Fig.10.27.16 are
as follows:

β 1 = (Kc + Kct)/(Kc + Kct + Kb1 + Kb2)


(10.75)

β 2 = (Kc + Kcb)/(Kc + Kcb + Kb3 + Kb4)


(10.76)

However, assuming idealised conditions, the effective length in a given


plane may be assessed from Table 28 in Annex E of IS 456, for normal use.

10.27.5 Determination of Sway or No Sway Column


Clause E-2 of IS 456 recommends the stability index Q to determine if a
column is a no sway or sway type. The stability index Q is expressed as:

Q = ∑ Pu Δ u /Hu hz
(10.77)

where ∑ Pu = sum of axial loads on all columns in the storey,


Δu = elastically computed first-order lateral deflection,
Hu = total lateral force acting within the storey, and

Version 2 CE IIT, Kharagpur


hz = height of the storey.

The column may be taken as no sway type if the value of Q is ≤ 0.4,


otherwise, the column is considered as sway type.

10.27.6 Design of Slender Columns


The design of slender columns, in principle, is to be done following the
same procedure as those of short columns. However, it is essential to estimate
the total moment i.e., primary and secondary moments considering P- Δ effects.
These secondary moments and axial forces can be determined by second-order
rigorous structural analysis – particularly for unbraced frames. Further, the
problem becomes more involved and laborious as the principle of superposition
is not applicable in second-order analysis.

However, cl.39.7 of IS 456 recommends an alternative simplified method


of determining additional moments to avoid the laborious and involved second-
order analysis. The basic principle of additional moment method for estimating
the secondary moments is explained in the next section.

10.27.7 Additional Moment Method


In this method, slender columns should be designed for biaxial
eccentricities which include secondary moments (Py of Eq.10.63 and 10.65)
about major and minor axes. We first consider braced columns which are bent
symmetrically in single curvature and cause balanced failure i.e., Pu = Pub.

(A) Braced columns bent symmetrically in single curvature and undergoing


balanced failure

For braced columns bent symmetrically in single curvature, we have from


Eqs.10.63 and 10.65,

M = Mo + Py = Mo + P ea = Mo + Ma
(10.78)

where P is the factored design load Pu, M are the total factored design moments
Mux and Muy about the major and minor axes, respectively; Mo are the primary
factored moments Moux and Mouy about the major and minor axes, respectively;
Ma are the additional moments Max and May about the major and minor axes,
respectively and ea are the additional eccentricities eax and eay along the minor
and major axes, respectively. The quantities Mo and P of Eq.10.78 are known
and hence, it is required to determine the respective values of ea, the additional
eccentricities only.

Version 2 CE IIT, Kharagpur


Let us consider the columns of Figs.10.27.10 and 11 showing Δ as the
maximum deflection at the mid-height section of the columns. The column of
Fig.10.27.10, having a constant primary moment Mo, causes constant curvature
φ , while the column of Fig.10.27.11, having a linearly varying primary moment
with a maximum value of Mo max at the mid-height section of the column, has a
linearly varying curvature with the maximum curvature of φ max at the mid-height
section the column. The two maximum curvatures can be expressed in terms of
their respective maximum deflection Δ as follows:

The constant curvature (Fig.10.27.10) φ max = 8Δ/l e2


(10.79)

The linearly varying curvature (Fig.10.27.11) φ max = 12Δ/l e2


(10.80)

where le are the respective effective lengths kl of the columns. We, therefore,
consider the maximum φ as the average value lying in between the two values of
Eqs.10.79 and 80 as

φ max = 10Δ/l e2
(10.81)

Accordingly, the maximum additional eccentricities ea, which are equal to


the maximum deflections Δ , can be written as

ea = Δ = φ l e2 /10
(10.82)

Version 2 CE IIT, Kharagpur


Assuming the column undergoes a balanced failure when Pu = Pub, the
maximum curvature at the mid-height section of the column, shown in
Figs.10.27.17a and b, can be expressed as given below, assuming (i) the values
of ε c = 0.0035, ε st = 0.002 and d ′ / D = 0.1, and (ii) the additional moment
capacities are about eighty per cent of the total moment.

φ = eighty per cent of {(0.0035 + 0.002)/0.9D} (see Fig.10.27.17c)

or φ = 1/200D
(10.83)

Substituting the value of φ in Eq.10.82,

ea = D(le/D)2/2000
(10.84)

Therefore, the additional moment Ma can be written as,

Version 2 CE IIT, Kharagpur


Ma = Py = P Δ = Pea = (PD/2000) (le/D)2
(10.85)

Thus, the additional moments Max and May about the major and minor axes,
respectively, are:

Max = (PuD/2000) (lex/D)2


(10.86)

May = (Pub/2000) (ley/b)2


(10.87)

where Pu = axial load on the member,

lex = effective length in respect of the major axis,

ley = effective length in respect of the minor axis,

D = depth of the cross-section at right angles to the major axis, and

b = width of the member.

Clause 39.7.1 of IS 456 recommends the expressions of Eqs.10.86 and 87 for


estimating the additional moments Max and May for the design. These two
expressions of the additional moments are derived considering the columns to be
braced and bent symmetrically undergoing balanced failure. Therefore, proper
modifications are necessary for different situations like braced columns with
unequal end moments with the same or different signs, unbraced columns and
columns causing compression failure i.e., when Pu > Pub.

(B) Braced columns subjected to unequal primary moments at the two


ends

For braced columns without any transverse loads occurring in the height,
the primary maximum moment (Mo max of Eq.10.64), with which the additional
moments of Eqs.10.86 and 87 are to be added, is to be taken as:

Mo max = 0.4 M1 + 0.6 M2


(10.88)

and further Mo max ≥ 0.4 M2


(10.89)

where M2 is the larger end moment and M1 is the smaller end moment, assumed
to be negative, if the column is bent in double curvature.

Version 2 CE IIT, Kharagpur


To eliminate the possibility of total moment Mu max becoming less than M2
for columns bent in double curvature (see Fig.10.27.12) with M1 and M2 having
opposite signs, another condition has been imposed as

Mu max ≥ M2
(10.90)

The above recommendations are given in notes of cl.39.7.1 of IS 456.

(C) Unbraced columns

Unbraced frames undergo considerable deflection due to P- Δ effect. The


additional moments determined from Eqs.10.86 and 87 are to be added with the
maximum primary moment Mo max at the ends of the column. Accordingly, we
have

Mo max = M2 + Ma
(10.91)

The above recommendation is given in the notes of cl.39.7.1 of IS 456.

(D) Columns undergoing compression failure (Pu > Pub)

It has been mentioned in part A of this section that the expressions of


additional moments given by Eqs.10.86 and 10.87 are for columns undergoing
balanced failure (Fig.10.27.17). However, when the column causes compression
failure, the e/D ratio is much less than that of balanced failure at relatively high
axial loads. The entire section may be under compression causing much less
curvatures. Accordingly, additional moments of Eqs.10.86 and 10.87 are to be
modified by multiplying with the reduction factor k as given below:

(i) For Pu > Pubx: kax = (Puz – Pu)/(Puz – Pubx)


(10.92)

(ii) For Pu > Puby: kay = (Puz – Pu)/(Puz – Puby)


(10.93)

with a condition that kax and kay should be ≤ 1


(10.94)

where Pu = axial load on compression member

Puz is given in Eq.10.59 of Lesson 26 and is,

Puz = 0.45 fck Ac + 0.75 fy Ast … (10.59)

Version 2 CE IIT, Kharagpur


Pubx, Puby = axial loads with respect to major and minor axes,
respectively, corresponding to the condition of maximum compressive strain of
0.0035 in concrete and tensile strain of 0.002 in outermost layer of tension steel.

It is seen from Eqs.10.92 and 10.93 that the values of k (kax and kay) vary
linearly from zero (when Pu = Puz) to one (when Pu = Pub). Since Eqs.10.92 and
10.93 are not applicable for Pu < Pub, another condition has been imposed as
given in Eq.10.94.

The above recommendations are given in cl.39.7.1.1 of IS 456.

The following discussion is very important for the design of slender


columns.

Additional moment method is one of the methods of designing slender


columns as discussed in A to D of this section. This method is recommended in
cl.39.7 of IS 456 also. The basic concept here is to enhance the primary
moments by adding the respective additional moments estimated in a simple way
avoiding laborious and involved calculations of second-order structural analysis.
However, these primary moments under eccentric loadings should not be less
than the moments corresponding to the respective minimum eccentricity, as
stipulated in the code. Hence, the primary moments in such cases are to be
replaced by the minimum eccentricity moments. Moreover, all slender columns,
including those under axial concentric loadings, are also to be designed for
biaxial bending, where the primary moments are zero. In such cases, the total
moment consisting of the additional moment multiplied with the modification
factor, if any, in each direction should be equal to or greater than the respective
moments under minimum eccentricity conditions. As mentioned earlier, the
minimum eccentricity consideration is given in cl.25.4 of IS 456.

10.27.8 Illustrative Example


The following illustrative example is taken up to explain the design of
slender columns. The example has been solved in step by step using (i) the
equations of Lessons 21 to 27 and (ii) employing design charts and tables of SP-
16, to compare the results.

Version 2 CE IIT, Kharagpur


Problem 1:

Determine the reinforcement required for a braced column against


sidesway with the following data: size of the column = 350 x 450 mm
(Fig.10.27.18); concrete and steel grades = M 30 and Fe 415, respectively;
effective lengths lex and ley = 7.0 and 6.0 m, respectively; unsupported length l = 8
m; factored load Pu = 1700 kN; factored moments in the direction of larger
dimension = 70 kNm at top and 30 kNm at bottom; factored moments in the
direction of shorter dimension = 60 kNm at top and 30 kNm at bottom. The
column is bent in double curvature. Reinforcement will be distributed equally on
four sides.

Solution 1:

Step 1: Checking of slenderness ratios

lex/D = 7000/450 = 15.56 > 12,

ley/b = 6000/350 = 17.14 > 12.

Hence, the column is slender with respect to both the axes.

Step 2: Minimum eccentricities and moments due to minimum


eccentricities (Eq.10.3 of Lesson21)

ex min = l/500 + D/30 = 8000/500 + 450/30 = 31.0 > 20 mm

ey min = l/500 + b/30 = 8000/500 + 350/30 = 27.67 > 20 mm

Version 2 CE IIT, Kharagpur


Mox (Min. ecc.) = Pu(ex min) = (1700) (31) (10-3) = 52.7 kNm

Moy (Min. ecc.) = Pu(ey min) = (1700) (27.67) (10-3) = 47.04 kNm

Step 3: Additional eccentricities and additional moments

Method 1: Using Eq. 10.84

eax = D(lex/D)2/2000 = (450) (7000/450)2/2000 = 54.44 mm

eay = b(lex/b)2/2000 = (350) (6000/350)2/2000 = 51.43 mm

Max = Pu(eax) = (1700) (54.44) (10-3) = 92.548 kNm

May = Pu(eay) = (1700) (51.43) (10-3) = 87.43 kNm

Method 2: Table I of SP-16

For lex/D = 15.56, Table I of SP-16 gives:

eax/D = 0.1214, which gives eax = (0.1214) (450) = 54.63 mm

For ley/D = 17.14, Table I of SP-16 gives:

eay/b = 0.14738, which gives eay = (0.14738) (350) = 51.583 mm

It is seen that values obtained from Table I of SP-16 are comparable with
those obtained by Eq. 10.84 in Method 1.

Step 4: Primary moments and primary eccentricities (Eqs.10.88 and 89)

Mox = 0.6M2 – 0.4M1 = 0.6(70) – 0.4(30) = 30 kNm, which should be ≥


0.4 M2 (= 28 kNm). Hence, o.k.

Moy = 0.6M2 – 0.4M1 = 0.6(60) – 0.4(30) = 24 kNm, which should be ≥


0.4 M2 (= 24 kNm). Hence, o.k.

Primary eccentricities:

ex = Mox/Pu = (30/1700) (103) = 17.65 mm

ey = Moy/Pu = (24/1700) (103) = 14.12 mm

Version 2 CE IIT, Kharagpur


Since, both primary eccentricities are less than the respective minimum
eccentricities (see Step 2), the primary moments are revised to those of Step 2.
So, Mox = 52.7 kNm and Moy = 47.04 kNm.

Step 5: Modification factors

To determine the actual modification factors, the percentage of


longitudinal reinforcement should be known. So, either the percentage of
longitudinal reinforcement may be assumed or the modification factor may be
assumed which should be verified subsequently. So, we assume the modification
factors of 0.55 in both directions.

Step 6: Total factored moments

Mux = Mox + (Modification factor) (Max) = 52.7 + (0.55) (92.548)

= 52.7 + 50.9 = 103.6 kNm

Muy = Moy + (Modification factor) (May) = 47.04 + (0.55) (87.43)

= 47.04 + 48.09 = 95.13 kNm

Step 7: Trial section (Eq.10.61 of Lesson 26)

The trial section is determined from the design of uniaxial bending with Pu
= 1700 kN and Mu = 1.15 ( M ux2 + M uy2 )1 / 2 . So, we have Mu = (1.15){(103.6)2 +
(95.13)2}1/2 = 161.75 kNm. With these values of Pu (= 1700 kN) and Mu (= 161.75
kNm), we use chart of SP-16 for the d ′ / D = 0.134. We assume the diameters of
longitudinal bar as 25 mm, diameter of lateral tie = 8 mm and cover = 40 mm, to
get d ′ = 40 + 8 + 12.5 = 60.5 mm. Accordingly, d ′ / D = 60.5/450 = 0.134 and
d ′ / b = 60.5/350 = 0.173. We have:

Pu/fck bD = 1700(103)/(30)(350)(450) = 0.3598

Mu/fck bD2 = 161.75(106)/(30)(350)(450)(450) = 0.076

We have to interpolate the values of p/fck for d ′ / D = 0.134 obtained from


Charts 44 (for d ′ / D = 0.1) and 45 ( d ′ / D = 0.15). The values of p/fck are 0.05
and 0.06 from Charts 44 and 45, respectively. The corresponding values of p are
1.5 and 1.8 per cent, respectively. The interpolated value of p for d ′ / D = 0.134
is 1.704 per cent, which gives Asc = (1.704)(350)(450)/100 = 2683.8 mm2. We
use 4-25 + 4-20 (1963 + 1256 = 3219 mm2), to have p provided = 2.044 per cent
giving p/fck = 0.068.

Step 8: Calculation of balanced loads Pb

Version 2 CE IIT, Kharagpur


The values of Pbx and Pby are determined using Table 60 of SP-16. For
this purpose, two parameters k1 and k2 are to be determined first from the table.
We have p/fck = 0.068, d ′ / D = 0.134 and d ′ / b = 0.173. From Table 60, k1 =
0.19952 and k2 = 0.243 (interpolated for d ′ / D = 0.134) for Pbx. So, we have:
Pbx/fckbD = k1 + k2 (p/fck) = 0.19952 + 0.243(0.068) = 0.216044, which gives Pbx =
0.216044(30)(350)(450)(10-3) = 1020.81 kN.

Similarly, for Pby: d ′ / b = 0.173, p/fck = 0.068. From Table 60 of SP-16, k1


= 0.19048 and k2 = 0.1225 (interpolated for d ′ / b = 0.173). This gives Pby/fckbD =
0.19048 + 0.1225(0.068) = 0.19881, which gives Pby =
-3
(0.19881)(30)(350)(450)(10 ) = 939.38 kN.

Since, the values of Pbx and Pby are less than Pu, the modification factors
are to be used.

Step 9: Determination of Puz

Method 1: From Eq.10.59 of Lesson 26

Puz = 0.45 fck Ag + (0.75 fy – 0.45 fck) Asc

= 0.45(30)(350)(450) + {0.75(415) – 0.45(30)}(3219) = 3084.71 kN

Method 2: Using Chart 63 of SP-16

We get Puz/Ag = 19.4 N/mm2 from Chart 63 of SP-16 using p = 2.044 per
cent. Therefore, Puz = (19.4)(350)(450)(10-3) = 3055.5 kN, which is in good
agreement with that of Method 1.

Step 10: Determination of modification factors

Method 1: From Eqs.10.92 and 10.93

kax = (Puz – Pu)/(Puz – Pubx) … (10.92)

or kax = (3084.71 – 1700)/(3084.71 – 1020.81) = 0.671 and

kay = (Puz – Pu)/(Puz – Puby) … (10.93)

or kay = (3084.71 – 1700)/(3084.71 – 939.39) = 0.645

The values of the two modification factors are different from the assumed
value of 0.55 in Step 5. However, the moments are changed and the section is
checked for safety.

Version 2 CE IIT, Kharagpur


Method 2: From Chart 65 of SP-16

From Chart 65 of SP-16, for the two parameters, Pbx/Puz =


1020.81/3084.71 = 0.331 and Pu/Puz = 1700/3084.71 = 0.551, we get kax = 0.66.
Similarly, for the two parameters, Pby/Puz = 939.38/3084.71 = 0.3045 and Pu/Puz =
0.551, we have kay = 0.65. Values of kax and kay are comparable with those of
Method 1.

Step 11: Total moments incorporating modification factors

Mux = Mox (from Step 4) + (kax) Max (from Step 3)

= 52.7 + 0.671(92.548) = 114.8 kNm

Muy = Moy (from Step 4) + kay (May) (from Step 3)

= 47.04 + (0.645)(87.43) = 103.43 kNm.

Step 12: Uniaxial moment capacities

The two uniaxial moment capacities Mux1 and Muy1 are determined as
stated: (i) For Mux1, by interpolating the values obtained from Charts 44 and 45,
knowing the values of Pu/fckbD = 0.3598 (see Step 7), p/fck = 0.068 (see Step 7),
d ′ / D = 0.134 (see Step 7), (ii) for Muy1, by interpolating the values obtained from
Charts 45 and 46, knowing the same values of Pu/fckbD and p/fck as those of (i)
and d ′ / D = 0.173 (see Step 7). The results are given below:

(i) Mux1/fckbD2 = 0.0882 (interpolated between 0.095 and 0.085)

(ii) Muy1/fckbb2 = 0.0827 (interpolated between 0.085 and 0.08)

So, we have, Mux1 = 187.54 kNm and Muy1 = 136.76 kNm.

Step 13: Value of α n

Method 1: From Eq.10.60 of Lesson 26

We have Pu/Puz = 1700/3084.71 = 0.5511. From Eq.10.60 of Lesson 26,


we have α n = 0.67 + 1.67 (Pu/Puz) = 1.59.

Method 2: Interpolating the values between (Pu/Puz) = 0.2 and 0.6

The interpolated value of α n = 1.0 + (0.5511 – 0.2)/0.6 = 1.5852. Both the


values are comparable. We use α n = 1.5852.

Version 2 CE IIT, Kharagpur


Step 14: Checking of column for safety

Method 1: From Eq.10.58 of Lesson 26

We have in Lesson 26:

( M ux / M ux1 )α n + ( M uy / M uy1 ) α n ≤ 1 … (10.58)

Here, putting the values of Mux, Mux1, Muy, Muy1 and α n , we get:
(114.8/187.54)1.5452 + (103.43/136.76)1.5852 = 0.4593 + 0.6422 = 1.1015. Hence,
the section or the reinforcement has to be revised.

Method 2: Chart 64 of SP-16

The point having the values of (Mux/Mux1) = 114.8/187.54 = 0.612 and


(Muy/Muy1) = 103.43/136.76 = 0.756 gives the value of Pu/Pz more than 0.7. The
value of Pu/Puz here is 0.5511 (see Step 13). So, the section needs revision.

We revise from Step 7 by providing 8-25 mm diameter bars (= 3927 mm2,


p = 2.493 per cent and p/fck = 0.0831) as the longitudinal reinforcement keeping
the values of b and D unchanged. The revised section is checked furnishing the
repeated calculations from Step 8 onwards. The letter R is used before the
number of step to indicate this step as revised one.

Step R8: Calculation of balanced loads Pb

Table 60 of SP-16 gives k1 = 0.19952, and k2 = 0.243. We have p/fck =


0.0831 now. So, Pbx = {0.19952 + (0.243)(0.0831)} (30)(350)(450)(10-3) =
1038.145 kN. Similarly, k1 = 0.19048, k2 = 0.1225 and p/fck = 0.0831 give Pby =
{0.19048 + (0.1225)(0.0831)} (30)(350)(450)(10-3) = 948.12 kN.

The values of Pbx and Pby are less than Pu (= 1700 kN). So, modification
factors are to be incorporated.

Step R9: Determination of Puz (Eq. 10.59 of Lesson 26)

Puz = 0.45(30)(350)(450) + {0.75(415) – 0.45(30)}(3927) = 3295.514 kN.

Step R10: Determination of modification factors (Eqs.10.92 and 10.93)

kax = (3295.514 – 1700)/(3295.514 – 1038.145) = 0.707

kay = (3295.514 – 1700)/(3295.514 – 948.12) = 0.68

Step R11: Total moments incorporating modification factors

Version 2 CE IIT, Kharagpur


Mux = 52.70 + 0.707(92.548) = 118.13 kNm

Muy = 47.04 + 0.68(87.43) = 106.49 kNm

Step R12: Uniaxial moment capacities

Using Charts 44 and 45 for Mux1 and Charts 45 and 46 for Muy1, we get (i)
the coefficient 0.1032 (interpolating 0.11 and 0.10) and (ii) the coefficient 0.0954
(interpolating 0.1 and 0.09) for Mux1 and Muy1, respectively.

Mux1 = (0.1032)(30)(350)(450)(450)(10-6) = 219.429 kNm

Muy1 = (0.0954)(30)(450)(350)(350)(10-6) = 157.77 kNm

Step R13: Value of α n (Eq.10.60 of Lesson 26)

Pu/Puz = 1700/3295.514 = 0.5158 which gives

α n = 1 + (0.5158 – 0.2)/0.6 = 1.5263

Step R14: Checking of column for safety (Eq.10.58 of Lesson 26)

(118.13/219.424)1.5263 + (106.49/157.77)1.5263 = 0.3886 + 0.5488 =


0.9374 < 1.0

Hence, the revised reinforcement is safe. The section is shown in


Fig.10.27.18.

10.27.9 Practice Questions and Problems with Answers


Q.1: Define a slender column. Give three reasons for its increasing importance
and popularity.

A.1: See sec. 10.27.1.

Q.2: Explain the behaviour of a slender column subjected to concentric loading.


Explain Euler’s load.

A.2: See sec.10.27.3.

Q.3: Choose the correct answer.

(A) As the slenderness ratio increases, the strength of concentrically


loaded column:
(i) increases (ii) decreases
Version 2 CE IIT, Kharagpur
(B) For braced columns, the effective length is between

(i) l and 2l (ii) 0.5l and 2l (iii) 0.5l and l

(C) The critical load of a braced frame is

(i) always larger than that of an unbraced column

(ii) always smaller than that of an unbraced column

(iii) sometimes larger and sometimes smaller than that of an unbraced


column

A.3: A. (ii), B. (iii), C. (i)

Q.4: Explain the behaviour of slender columns under axial load and uniaxial
bending, bent in single curvature.

A.4: Part (A) of sec. 10.27.3.

Q.5: Explain the behaviour of slender columns under axial load and uniaxial
bending, bent in double curvature.

A.5: Part (B) of sec. 10.27.3.

Q.6: Explain the behaviour of columns in portal frame both braced and
unbraced.

A.6: Part (C) of sec. 10.27.3.

Version 2 CE IIT, Kharagpur


Q.7: Check the column of Fig.10.27.19, if subjected to an axial factored load of
Pu = 1500 kN only when the unsupported length of the column = l = 8.0 m,
lex = ley = 6.0 m, D = 400 mm, b = 300 mm, using concrete of M 20 and
steel grade in Fe 415.

A.7: Solution:

Step 1: Slenderness ratios

Lex/D = 6000/400 = 15 > 12

Ley/b = 6000/300 = 20 > 12

The column is slender about both the axes.

Step 2: Minimum eccentricities and moments due to minimum


eccentricities (Eq.10.3 of Lesson 21)

ex min = l/500 + D/30 = 8000/500 + 400/30 = 29.33 mm > 20 mm

ey min = 8000/500 + 300/30 = 26 mm > 20 mm

Mx due to min. ecc. = Pu (ex min) = 1500(29.33) = 43.995 kNm

My due to min. ecc. = Pu (ey min) = 1500(26.0) = 39.0 kNm

Step 3: Primary moments

Since the column is concentrically loaded, the primary moments are zero.
Therefore, the additional moments must be greater than the respective moments
due to minimum eccentricity.

Step 4: Additional eccentricities and moments (Eq.10.84)

eax = D(lex/D)2/2000 = 400(6000/400)2/2000 = 45 mm > ex min (= 29.23


mm)

eay = b(ley/b)2/2000 = 300(6000/300)2/2000 = 60 mm > ey min (= 26


mm)

Step 5: Calculation of balance loads Pbx and Pby

Given Asc = 3927 mm2 (8 bars of 25 mm diameter give p = 3.2725 per


cent. So, p/fck = 0.1636. Using 8 mm diameter lateral tie, d ′ = 40 + 8 + 12.5 =

Version 2 CE IIT, Kharagpur


60.5 mm giving d ′ /D = 60.5/400 = 0.15125 ≅ 0.15 and d ′ /b = 60.5/300 = 0.2017
≅ 0.20.

From Table 60 of SP-16, we get k1 = 0.196 and k2 = 0.061. Thus, we


have:

Pbx = {0.196 + (0.061)(0.1636)}(20)(300)(400)(10-3) = 494.35 kN

Similarly, for Pby: k1 = 0.184 and k2 = -0.011, we get

Pby = {0.184 - (0.011)(0.1636)}(20)(300)(400)(10-3) = 437.281 kN

Since, Pbx and Pby are less than Pu (= 1500 kN), modification factors are to
be incorporated.

Step 6: Determination of Puz (Eq.10.59 of Lesson 26)

Puz = 0.45(20)(300)(400) + {0.75(415) – 0.45(20)}(3927)(10-3) = 2266.94


kN

Step 7: Determination of modification factors

kax = (2266.94 – 1500)/(2266.94 – 494.35) = 0.433 and

kay = (2266.94 – 1500)/(2266.94 – 437.281) = 0.419

Step 8: Additional moments and total moments

Max = 1500(0.433)(45) = 29.2275 kNm

May = 1500(0.419)(60) = 37.71 kNm

Since, primary moments are zero as the column is concentrically loaded,


the total moment shall consist of the additional moments. But, as both the
additional moments are less than the respective moment due to minimum
eccentricity, the revised additional moments are: Max = 43.995 kNm and May =
39.0 kNm, which are the total moments also.

Thus, we have:

Mux = 43.995 kNm, Muy = 39.0 kNm and Pu = 1500 kN.

Step 9: Uniaxial moment capacities

Version 2 CE IIT, Kharagpur


We have, Pu/fck bD = {1500/(20)(300)(400)}(1000) = 0.625, p/fck = 0.1636
and d ′ /D = 0.15 for Mux1; and d ′ /b = 0.2 for Muy1. The coefficients are 0.11 (from
Chart 45) and 0.1 (from Chart 46) for Mux1 and Muy1, respectively. So, we get,

Mux1 = 0.11(20)(300)(400)(400)(10-6) = 225.28 kNm, and

Muy1 = 0.1(20)(300)(300)(400)(10-6) = 72.0 kNm

Step 10: Value of α n (Eq.10.60 of Lesson 26)

Here, Pu/Puz = 1500/2266.94 = 0.6617. So, we get

α n = 1.0 + (0.4617/0.6) = 1.7695

Step 11: Checking the column for safety (Eq.10.58 of Lesson 26)

( M ux / M ux1 )α n + ( M uy / M uy1 ) α n ≤ 1

Here, (43.995/225.28)1.7695 + (39.0/72.0)1.7695 = 0.0556 + 0.3379 =


0.3935 < 1

Hence, the column is safe to carry Pu = 1500 kN.

11.27.10 References
1. Reinforced Concrete Limit State Design, 6th Edition, by Ashok K. Jain,
Nem Chand & Bros, Roorkee, 2002.
2. Limit State Design of Reinforced Concrete, 2nd Edition, by P.C.Varghese,
Prentice-Hall of India Pvt. Ltd., New Delhi, 2002.
3. Advanced Reinforced Concrete Design, by P.C.Varghese, Prentice-Hall of
India Pvt. Ltd., New Delhi, 2001.
4. Reinforced Concrete Design, 2nd Edition, by S.Unnikrishna Pillai and
Devdas Menon, Tata McGraw-Hill Publishing Company Limited, New
Delhi, 2003.
5. Limit State Design of Reinforced Concrete Structures, by P.Dayaratnam,
Oxford & I.B.H. Publishing Company Pvt. Ltd., New Delhi, 2004.
6. Reinforced Concrete Design, 1st Revised Edition, by S.N.Sinha, Tata
McGraw-Hill Publishing Company. New Delhi, 1990.
7. Reinforced Concrete, 6th Edition, by S.K.Mallick and A.P.Gupta, Oxford &
IBH Publishing Co. Pvt. Ltd. New Delhi, 1996.
8. Behaviour, Analysis & Design of Reinforced Concrete Structural Elements,
by I.C.Syal and R.K.Ummat, A.H.Wheeler & Co. Ltd., Allahabad, 1989.

Version 2 CE IIT, Kharagpur


9. Reinforced Concrete Structures, 3rd Edition, by I.C.Syal and A.K.Goel,
A.H.Wheeler & Co. Ltd., Allahabad, 1992.
10. Textbook of R.C.C, by G.S.Birdie and J.S.Birdie, Wiley Eastern Limited,
New Delhi, 1993.
11. Design of Concrete Structures, 13th Edition, by Arthur H. Nilson, David
Darwin and Charles W. Dolan, Tata McGraw-Hill Publishing Company
Limited, New Delhi, 2004.
12. Concrete Technology, by A.M.Neville and J.J.Brooks, ELBS with
Longman, 1994.
13. Properties of Concrete, 4th Edition, 1st Indian reprint, by A.M.Neville,
Longman, 2000.
14. Reinforced Concrete Designer’s Handbook, 10th Edition, by C.E.Reynolds
and J.C.Steedman, E & FN SPON, London, 1997.
15. Indian Standard Plain and Reinforced Concrete – Code of Practice (4th
Revision), IS 456: 2000, BIS, New Delhi.
16. Design Aids for Reinforced Concrete to IS: 456 – 1978, BIS, New
Reinforced Concrete Limit State Design, 5th Edition, by Ashok K. Jain,
Nem Chand & Bros, Roorkee, 1999.

11.27.11 Test 27 with Solutions


Maximum Marks = 50, Maximum Time = 30 minutes

Answer all questions.

Version 2 CE IIT, Kharagpur


TQ.1: Determine the primary, additional and total moments of the column shown
in Fig.10.27.20 for the three different cases:

(i) When the column is braced against sidesway and is bent in single
curvature.

(ii) When the column is braced against sidesway and is bent in double
curvature.

(iii) When the column is unbraced.

Use the following data: Pu = 2000 kN, concrete grade = M 20, steel grade
= Fe 415, unsupported length l = 8.0 m, lex = 7.0 m, ley = 6.0 m, Asc = 6381 mm2
(12-25 mm diameter bars), lateral tie = 8 mm diameter @ 250 mm c/c, d ′ = 60.5
mm, D = 500 mm and b = 400 mm. The factored moments are: 70 kNm at top
and 40 kNm at bottom in the direction of larger dimension and 60 kNm at top and
30 kNm at bottom in the direction of shorter dimension.

A.TQ.1: Solution

The following are the common steps for all three cases.

Step 1: Slenderness ratios

lex/D = 7000/500 = 14 > 12 and ley/b = 6000/400 = 15 > 12

The column is slender about both axes.

Step 2: Minimum eccentricities and moments due to minimum


eccentricities (Eq.10.3 of Lesson 21)

ex min = l/500 + D/30 = 8000/500 + 500/30 = 32.67 mm > 20 mm, and

ey min = l/500 + b/30 = 8000/500 + 400/30 = 29.34 mm > 20 mm

Mx (min. ecc.) = 2000(32.67)(10-3) = 65.34 kNm, and

My (min. ecc.) = 2000(29.34)(10-3) = 58.68 kNm

Step 3: Additional eccentricities and moments due to additional


eccentricities (Eq.10.84)

eax = D(lex/D)2/2000 = 500(7000/500)2/2000 = 49 mm > ex min (= 32.67


mm)

Version 2 CE IIT, Kharagpur


eay = b(ley/b)2/2000 = 400(6000/400)2/2000 = 45 mm > ey min (= 29.34
mm)

Max = Pu(eax) = (2000)(49)(10-3) = 98 kNm, and

May = Pu(eay) = (2000)(45)(10-3) = 90 kNm

Step 4: Calculation of balanced loads

Using d ′ /D = 0.121 and p/fck = 3.1905/20 = 0.159525 in Table 60 of SP-


16, we have k1 = 0.20238 and k2 = 0.2755 (by linear interpolation). This gives

Pbx = {0.20238 + 0.2755(0.159525)}(20)(400)(500)(10-3) = 983.32 kN

Similarly, d ′ /b = 0.15125 and p/fck = 0.159525 in Table 60 of SP-16 gives


k1 = 0.1957 and k2 = 0.198625 (by linear interpolation). So, we get

Pby = {0.1957 + 0.198625(0.159525)}(20)(400)(500)(10-3) = 909.54 kN

Both Pbx and Pby are smaller than Pu (= 2000 kN). Hence, modification
factors are to be incorporated.

Step 5: Calculation of Puz (Eq.10.59 of Lesson 26)

Puz = 0.45 fck Ag + (0.75 fy – 0.45 fck) Asc

= 0.45(20)(400)(500) + {0.75(415) – 0.45(20)}(6381) = 3728.66 kN

Step 6: Modification factors and revised additional moments (Eqs.10.92


and 10.93)

kax = (3728.66 - 2000)/(3728.66 – 983.32) = 0.6297, and

kay = (3728.66 - 2000)/(3728.66 – 909.54) = 0.6132

The revised additional moments are:

Max = 98(0.6297) = 61.71 kNm, and

May = 90(0.6132) = 55.19 kNm

Now, the different cases are explained.

Case (i): Braced column in single curvature

Version 2 CE IIT, Kharagpur


Primary moments = 0.4 M1 + 0.6 M2, but should be equal to or greater
than 0.4 M2 and moment due to minimum eccentricities. So, we get,

Mox = largest of 58 kNm, 28 kNm and 65.34 kNm = 65.34 kNm

Moy = largest of 48 kNm, 24 kNm and 58.68 kNm = 58.68 kNm

Additional moments are Max = 61.71 kNm and May = 55.19 kNm
(incorporating the respective modification factors).

Total moments = Mux = Mox + Max = 65.34 + 61.71 = 127.05 kNm >
65.34 kNm (moment due to minimum eccentricity), and

Muy = Moy + May = 58.68 + 55.19 = 113.87 kNm > 58.68 kNm (moment
due to minimum eccentricity).

Case (ii): Braced column in double curvature

Primary moments = - 0.4 M1 + 0.6 M2, but should be equal to or greater


than 0.4M2 and the moment due to minimum eccentricity. So, we get,

Mox = largest of 26 kNm, 28 kNm and 65.34 kNm = 65.34 kNm

Moy = largest of 24 kNm, 24 kNm and 58.68 kNm = 58.68 kNm

Additional moments are Max = 61.71 kNm and May = 55.19 kNm

Final moments = Mux = Mox + Max = 65.34 + 61.71 = 127.05 kNm >
65.34 kNm (moment due to minimum eccentricity), and

Muy = 58.68 + 55.19 = 113.87 kNm > 58.68 kNm (moment due to
minimum eccentricity).

Case (iii): Unbraced column

Primary moments = M2 and should be greater than or equal to moment


due to minimum eccentricity.

Mox = 70 kNm > 65.34 kNm (moment due to minimum eccentricity), and

Moy = 60 kNm > 58.68 kNm (moment due to minimum eccentricity).

Additional moments are Max = 61.71 kNm and May = 55.19 kNm

Final moments = Mux = Mox + Max = 70.0 + 61.71 = 131.71 kNm >
65.34 kNm (moment due to minimum eccentricity), and

Version 2 CE IIT, Kharagpur


Muy = Moy + Max = 60.0 + 55.19 = 115.19 kNm > 58.68 kNm (moment
due to minimum eccentricity).

10.27.12 Summary of this Lesson


This lesson mentions the reasons of increasing importance and popularity
of slender columns and explains the behaviour of slender columns loaded
concentrically or eccentrically. The role of minimum eccentricity that cannot be
avoided in any practical column is explained for slender columns. The moments
due to minimum eccentricities in both directions should be taken into account for
a slender column loaded concentrically as it should be designed under biaxial
bending. On the other hand, the given primary moments are also to be checked
so that they are equal to or greater than the respective moments due to minimum
eccentricity for all slender columns.

Both braced and unbraced columns, bent in single or double curvatures,


are explained. The importance of modification factors of the additional moments
due to P- Δ effect is explained. Effective lengths and important parameter to
determine the slenderness ratios are illustrated for different types of support
conditions either in single column or when the column is a part of rigid frames.
Additional moment method, a simple method for the design of slender columns,
is explained, which is recommended in IS 456. Numerical problems in illustrative
example, practice problem and test questions will help in understanding and
applying the method for the design of slender columns, as stipulated in IS 456.
Direct computations from the given equations as well as use of design charts and
tables of SP-16 are illustrated for the design.

Version 2 CE IIT, Kharagpur


Module
11
Foundations - Theory
and Design
Version 2 CE IIT, Kharagpur
Lesson
28
Foundations - Theory
Version 2 CE IIT, Kharagpur
Instructional Objectives:
At the end of this lesson, the student should be able to:

• explain the two major and other requirements of the design of foundation,

• identify five points indicating the differences between the design of


foundation and the design of other elements of the superstructure,

• differentiate between footing and foundation,

• differentiate between shallow and deep foundations,

• identify the situations when a combined footing shall be used,

• explain the safe bearing capacity of soil mentioning the difference


between gross and net safe bearing capacities,

• determine the minimum depth of foundation,

• determine the critical sections of bending moment and shear in isolated


footings,

• draw the distributions of pressure of soil below the footing for concentric
and eccentric loads with e ≤ L/6 and e > L/6,

• determine the soil pressure in a foundation which is unsymmetrical.

11.28.1 Introduction
Till now we discussed the different structural elements viz. beams, slabs,
staircases and columns, which are placed above the ground level and are known
as superstructure. The superstructure is placed on the top of the foundation
structure, designated as substructure as they are placed below the ground level.
The elements of the superstructure transfer the loads and moments to its
adjacent element below it and finally all loads and moments come to the
foundation structure, which in turn, transfers them to the underlying soil or rock.
Thus, the foundation structure effectively supports the superstructure. However,
all types of soil get compressed significantly and cause the structure to settle.
Accordingly, the major requirements of the design of foundation structures are
the two as given below (see cl.34.1 of IS 456):

Version 2 CE IIT, Kharagpur


1. Foundation structures should be able to sustain the applied loads,
moments, forces and induced reactions without exceeding the safe bearing
capacity of the soil.

2. The settlement of the structure should be as uniform as possible and it


should be within the tolerable limits. It is well known from the structural analysis
that differential settlement of supports causes additional moments in statically
indeterminate structures. Therefore, avoiding the differential settlement is
considered as more important than maintaining uniform overall settlement of the
structure.

In addition to the two major requirements mentioned above, the foundation


structure should provide adequate safety for maintaining the stability of structure
due to either overturning and/or sliding (see cl.20 of IS 456). It is to be noted that
this part of the structure is constructed at the first stage before other components
(columns / beams etc.) are taken up. So, in a project, foundation design and
details are completed before designs of other components are undertaken.

However, it is worth mentioning that the design of foundation structures is


somewhat different from the design of other elements of superstructure due to
the reasons given below. Therefore, foundation structures need special attention
of the designers.

1. Foundation structures undergo soil-structure interaction. Therefore, the


behaviour of foundation structures depends on the properties of structural
materials and soil. Determination of properties of soil of different types itself is a
specialized topic of geotechnical engineering. Understanding the interacting
behaviour is also difficult. Hence, the different assumptions and simplifications
adopted for the design need scrutiny. In fact, for the design of foundations of
important structures and for difficult soil conditions, geotechnical experts should
be consulted for the proper soil investigation to determine the properties of soil,
strata wise and its settlement criteria.

2. Accurate estimations of all types of loads, moments and forces are


needed for the present as well as for future expansion, if applicable. It is very
important as the foundation structure, once completed, is difficult to strengthen in
future.

3. Foundation structures, though remain underground involving very little


architectural aesthetics, have to be housed within the property line which may
cause additional forces and moments due to the eccentricity of foundation.

4. Foundation structures are in direct contact with the soil and may be
affected due to harmful chemicals and minerals present in the soil and
fluctuations of water table when it is very near to the foundation. Moreover,

Version 2 CE IIT, Kharagpur


periodic inspection and maintenance are practically impossible for the foundation
structures.

5. Foundation structures, while constructing, may affect the adjoining


structure forming cracks to total collapse, particularly during the driving of piles
etc.

However, wide ranges of types of foundation structures are available. It is


very important to select the appropriate type depending on the type of structure,
condition of the soil at the location of construction, other surrounding structures
and several other practical aspects as mentioned above.

11.28.2 Types of Foundation Structures


Foundations are mainly of two types: (i) shallow and (ii) deep foundations.
The two different types are explained below:

(A) Shallow foundations

Shallow foundations are used when the soil has sufficient strength within a
short depth below the ground level. They need sufficient plan area to transfer the
heavy loads to the base soil. These heavy loads are sustained by the reinforced
concrete columns or walls (either of bricks or reinforced concrete) of much less
areas of cross-section due to high strength of bricks or reinforced concrete when
compared to that of soil. The strength of the soil, expressed as the safe bearing
capacity of the soil as discussed in sec.11.28.3, is normally supplied by the
geotechnical experts to the structural engineer. Shallow foundations are also
designated as footings. The different types of shallow foundations or footings are
discussed below.

Version 2 CE IIT, Kharagpur


1. Plain concrete pedestal footings

Plain concrete pedestal footings (Fig.11.28.1) are very economical for


columns of small loads or pedestals without any longitudinal tension steel (see
cls.34.1.2 and 34.1.3 of IS 456). In Fig.11.28.1, the angle α between the plane
passing through the bottom edge of the pedestal and the corresponding junction
edge of the column with pedestal and the horizontal plane shall be determined
from Eq. 11.3.

Version 2 CE IIT, Kharagpur


2. Isolated footings

Version 2 CE IIT, Kharagpur


Version 2 CE IIT, Kharagpur
Version 2 CE IIT, Kharagpur
These footings are for individual columns having the same plan forms of
square, rectangular or circular as that of the column, preferably maintaining the
proportions and symmetry so that the resultants of the applied forces and
reactions coincide. These footings, shown in Figs.11.27.2 to 11.27.4, consist of a
slab of uniform thickness, stepped or sloped. Though sloped footings are
economical in respect of the material, the additional cost of formwork does not
offset the cost of the saved material. Therefore, stepped footings are more
economical than the sloped ones. The adjoining soil below footings generates
upward pressure which bends the slab due to cantilever action. Hence, adequate
tensile reinforcement should be provided at the bottom of the slab (tension face).
Clause 34.1.1 of IS 456 stipulates that the sloped or stepped footings, designed
as a unit, should be constructed to ensure the integrated action. Moreover, the
effective cross-section in compression of sloped and stepped footings shall be
limited by the area above the neutral plane. Though symmetrical footings are
desirable, sometimes situation compels for unsymmetrical isolated footings
(Eccentric footings or footings with cut outs) either about one or both the axes
(Figs.11.28.5 and 6).

Version 2 CE IIT, Kharagpur


3. Combined footings

Version 2 CE IIT, Kharagpur


When the spacing of the adjacent columns is so close that separate isolated
footings are not possible due to the overlapping areas of the footings or
inadequate clear space between the two areas of the footings, combined footings
are the solution combining two or more columns. Combined footing normally
means a footing combining two columns. Such footings are either rectangular or
trapezoidal in plan forms with or without a beam joining the two columns, as
shown in Figs.11.28.7 and 11.28.8.

Version 2 CE IIT, Kharagpur


4. Strap footings

When two isolated footings are combined by a beam with a view to


sharing the loads of both the columns by the footings, the footing is known as
strap footing (Fig.11.28.9). The connecting beam is designated as strap beam.
These footings are required if the loads are heavy on columns and the areas of
foundation are not overlapping with each other.

Version 2 CE IIT, Kharagpur


5. Strip foundation or wall footings

These are in long strips especially for load bearing masonry walls or
reinforced concrete walls (Figs.11.28.10). However, for load bearing masonry
walls, it is common to have stepped masonry foundations. The strip footings
distribute the loads from the wall to a wider area and usually bend in transverse
direction. Accordingly, they are reinforced in the transverse direction mainly,
while nominal distribution steel is provided along the longitudinal direction.

Version 2 CE IIT, Kharagpur


6. Raft or mat foundation

These are special cases of combined footing where all the columns of the
building are having a common foundation (Fig.11.28.11). Normally, for buildings
with heavy loads or when the soil condition is poor, raft foundations are very
much useful to control differential settlement and transfer the loads not
exceeding the bearing capacity of the soil due to integral action of the raft
foundation. This is a threshold situation for shallow footing beyond which deep
foundations have to be adopted.

Version 2 CE IIT, Kharagpur


(B) Deep foundations

Version 2 CE IIT, Kharagpur


As mentioned earlier, the shallow foundations need more plan areas due
to the low strength of soil compared to that of masonry or reinforced concrete.
However, shallow foundations are selected when the soil has moderately good
strength, except the raft foundation which is good in poor condition of soil also.
Raft foundations are under the category of shallow foundation as they have
comparatively shallow depth than that of deep foundation. It is worth mentioning
that the depth of raft foundation is much larger than those of other types of
shallow foundations.

However, for poor condition of soil near to the surface, the bearing
capacity is very less and foundation needed in such situation is the pile
foundation (Figs.11.28.12). Piles are, in fact, small diameter columns which are
driven or cast into the ground by suitable means. Precast piles are driven and
cast-in-situ are cast. These piles support the structure by the skin friction
between the pile surface and the surrounding soil and end bearing force, if such
resistance is available to provide the bearing force. Accordingly, they are
designated as frictional and end bearing piles. They are normally provided in a
group with a pile cap at the top through which the loads of the superstructure are
transferred to the piles.

Piles are very useful in marshy land where other types of foundation are
impossible to construct. The length of the pile which is driven into the ground
depends on the availability of hard soil/rock or the actual load test. Another
advantage of the pile foundations is that they can resist uplift also in the same
manner as they take the compression forces just by the skin friction in the
opposite direction.

However, driving of pile is not an easy job and needs equipment and
specially trained persons or agencies. Moreover, one has to select pile
foundation in such a situation where the adjacent buildings are not likely to be
damaged due to the driving of piles. The choice of driven or bored piles, in this
regard, is critical.

Exhaustive designs of all types of foundations mentioned above are


beyond the scope of this course. Accordingly, this module is restricted to the
design of some of the shallow footings, frequently used for normal low rise
buildings only.

11.28.3 Safe Bearing Capacity of Soil


The safe bearing capacity qc of soil is the permissible soil pressure
considering safety factors in the range of 2 to 6 depending on the type of soil,
approximations and assumptions and uncertainties. This is applicable under
service load condition and, therefore, the partial safety factors λ f for different
load combinations are to be taken from those under limit state of serviceability

Version 2 CE IIT, Kharagpur


(vide Table 18 of IS 456 or Table 2.1 of Lesson 3). Normally, the acceptable
value of qc is supplied by the geotechnical consultant to the structural engineer
after proper soil investigations. The safe bearing stress on soil is also related to
corresponding permissible displacement / settlement.

Gross and net bearing capacities are the two terms used in the design.
Gross bearing capacity is the total safe bearing pressure just below the footing
due to the load of the superstructure, self weight of the footing and the weight of
earth lying over the footing. On the other hand, net bearing capacity is the net
pressure in excess of the existing overburden pressure. Thus, we can write

Net bearing capacity = Gross bearing capacity - Pressure due to overburden


soil (11.1)

While calculating the maximum soil pressure q, we should consider all the
loads of superstructure along with the weight of foundation and the weight of the
backfill. During preliminary calculations, however, the weight of the foundation
and backfill may be taken as 10 to 15 per cent of the total axial load on the
footing, subjected to verification afterwards.

11.28.4 Depth of Foundation


All types of foundation should have a minimum depth of 50 cm as per IS
1080-1962. This minimum depth is required to ensure the availability of soil
having the safe bearing capacity assumed in the design. Moreover, the
foundation should be placed well below the level which will not be affected by
seasonal change of weather to cause swelling and shrinking of the soil. Further,
frost also may endanger the foundation if placed at a very shallow depth.
Rankine formula gives a preliminary estimate of the minimum depth of foundation
and is expressed as

d = (qc/ λ ){(1 - sin φ )/(1 + sin φ )}2


(11.2)

where d = minimum depth of foundation


qc = gross bearing capacity of soil
λ = density of soil
φ = angle of repose of soil

Though Rankine formula considers three major soil properties qc, λ and
φ , it does not consider the load applied to the foundation. However, this may be
a guideline for an initial estimate of the minimum depth which shall be checked
subsequently for other requirements of the design.

Version 2 CE IIT, Kharagpur


11.28.5 Design Considerations
(a) Minimum nominal cover (cl. 26.4.2.2 of IS 456)

The minimum nominal cover for the footings should be more than that of
other structural elements of the superstructure as the footings are in direct
contact with the soil. Clause 26.4.2.2 of IS 456 prescribes a minimum cover of 50
mm for footings. However, the actual cover may be even more depending on the
presence of harmful chemicals or minerals, water table etc.

(b) Thickness at the edge of footings (cls. 34.1.2 and 34.1.3 of IS 456)

The minimum thickness at the edge of reinforced and plain concrete


footings shall be at least 150 mm for footings on soils and at least 300 mm above
the top of piles for footings on piles, as per the stipulation in cl.34.1.2 of IS 456.

For plain concrete pedestals, the angle α (see Fig.11.28.1) between the
plane passing through the bottom edge of the pedestal and the corresponding
junction edge of the column with pedestal and the horizontal plane shall be
determined from the following expression (cl.34.1.3 of IS 456)

tan α ≤ 0.9{(100 qa/fck) + 1}1/2


(11.3)

where qa = calculated maximum bearing pressure at the base of pedestal in


N/mm2, and

fck = characteristic strength of concrete at 28 days in N/mm2.

(c) Bending moments (cl. 34.2 of IS 456)

1. It may be necessary to compute the bending moment at several


sections of the footing depending on the type of footing, nature of loads and the
distribution of pressure at the base of the footing. However, bending moment at
any section shall be determined taking all forces acting over the entire area on
one side of the section of the footing, which is obtained by passing a vertical
plane at that section extending across the footing (cl.34.2.3.1 of IS 456).

2. The critical section of maximum bending moment for the purpose of


designing an isolated concrete footing which supports a column, pedestal or wall
shall be:

(i) at the face of the column, pedestal or wall for footing supporting a
concrete column, pedestal or reinforced concrete wall, (Figs.11.28.2,
3 and 10), and

Version 2 CE IIT, Kharagpur


(ii) halfway between the centre-line and the edge of the wall, for footing
under masonry wall (Fig.11.28.10). This is stipulated in cl.34.2.3.2 of
IS 456.

The maximum moment at the critical section shall be determined as


mentioned in 1 above.

For round or octagonal concrete column or pedestal, the face of the


column or pedestal shall be taken as the side of a square inscribed within the
perimeter of the round or octagonal column or pedestal (see cl.34.2.2 of IS 456
and Figs.11.28.13a and b).

(d) Shear force (cl. 31.6 and 34.2.4 of IS 456)

Footing slabs shall be checked in one-way or two-way shears depending


on the nature of bending. If the slab bends primarily in one-way, the footing slab
shall be checked in one-way vertical shear. On the other hand, when the bending
is primarily two-way, the footing slab shall be checked in two-way shear or
punching shear. The respective critical sections and design shear strengths are
given below:

1. One-way shear (cl. 34.2.4 of IS 456)

One-way shear has to be checked across the full width of the base slab on
a vertical section located from the face of the column, pedestal or wall at a
distance equal to (Figs.11.28.2, 3 and 10):

(i) effective depth of the footing slab in case of footing slab on soil, and

Version 2 CE IIT, Kharagpur


(ii) half the effective depth of the footing slab if the footing slab is on piles
(Fig.11.28.12).

The design shear strength of concrete without shear reinforcement is


given in Table 19 of cl.40.2 of IS 456.

2. Two-way or punching shear (cls.31.6 and 34.2.4)

Two-way or punching shear shall be checked around the column on a


perimeter half the effective depth of the footing slab away from the face of the
column or pedestal (Figs.11.28.2 and 3).

The permissible shear stress, when shear reinforcement is not provided,


shall not exceed ks τ c , where ks = (0.5 + β c ), but not greater than one, β c being
the ratio of short side to long side of the column, and τ c = 0.25(fck)1/2 in limit state
method of design, as stipulated in cl.31.6.3 of IS 456.

Normally, the thickness of the base slab is governed by shear. Hence, the
necessary thickness of the slab has to be provided to avoid shear reinforcement.

(e) Bond (cl.34.2.4.3 of IS 456)

The critical section for checking the development length in a footing slab
shall be the same planes as those of bending moments in part (c) of this section.
Moreover, development length shall be checked at all other sections where they
change abruptly. The critical sections for checking the development length are
given in cl.34.2.4.3 of IS 456, which further recommends to check the anchorage
requirements if the reinforcement is curtailed, which shall be done in accordance
with cl.26.2.3 of IS 456.

(f) Tensile reinforcement (cl.34.3 of IS 456)

The distribution of the total tensile reinforcement, calculated in accordance


with the moment at critical sections, as specified in part (c) of this section, shall
be done as given below for one-way and two-way footing slabs separately.

(i) In one-way reinforced footing slabs like wall footings, the reinforcement
shall be distributed uniformly across the full width of the footing i.e.,
perpendicular to the direction of wall. Nominal distribution reinforcement shall be
provided as per cl. 34.5 of IS 456 along the length of the wall to take care of the
secondary moment, differential settlement, shrinkage and temperature effects.

(ii) In two-way reinforced square footing slabs, the reinforcement


extending in each direction shall be distributed uniformly across the full
width/length of the footing.

Version 2 CE IIT, Kharagpur


(iii) In two-way reinforced rectangular footing slabs, the reinforcement in
the long direction shall be distributed uniformly across the full width of the footing
slab. In the short direction, a central band equal to the width of the footing shall
be marked along the length of the footing, where the portion of the reinforcement
shall be determined as given in the equation below. This portion of the
reinforcement shall be distributed across the central band:

Reinforcement in the central band = {2/( β +1)} (Total reinforcement in the short
direction)
(11.4)

where β is the ratio of longer dimension to shorter dimension of the footing slab
(Fig.11.28.14).

Each of the two end bands shall be provided with half of the remaining
reinforcement, distributed uniformly across the respective end band.

Version 2 CE IIT, Kharagpur


(g) Transfer of load at the base of column (cl.34.4 of IS 456)

All forces and moments acting at the base of the column must be
transferred to the pedestal, if any, and then from the base of the pedestal to the
footing, (or directly from the base of the column to the footing if there is no

Version 2 CE IIT, Kharagpur


pedestal) by compression in concrete and steel and tension in steel.
Compression forces are transferred through direct bearing while tension forces
are transferred through developed reinforcement. The permissible bearing
stresses on full area of concrete shall be taken as given below from cl.34.4 of IS
456:

σ br = 0.25fck, in working stress method, and


(11.5)

σ br = 0.45fck, in limit state method


(11.6)

It has been mentioned in sec. 10.26.5 of Lesson 26 that the stress of concrete is
taken as 0.45fck while designing the column. Since the area of footing is much
larger, this bearing stress of concrete in column may be increased considering
the dispersion of the concentrated load of column to footing. Accordingly, the
permissible bearing stress of concrete in footing is given by (cl.34.4 of IS 456):

σ br = 0.45fck (A1/A2)1/2
(11.7)

with a condition that

(A1/A2)1/2 ≤ 2.0
(11.8)

where A1 = maximum supporting area of footing for bearing which is


geometrically similar to and concentric with the loaded area A2, as
shown in Fig.11.28.15

A2 = loaded area at the base of the column.

The above clause further stipulates that in sloped or stepped footings, A1 may be
taken as the area of the lower base of the largest frustum of a pyramid or cone
contained wholly within the footing and having for its upper base, the area
actually loaded and having side slope of one vertical to two horizontal, as shown
in Fig.11.28.15.

If the permissible bearing stress on concrete in column or in footing is


exceeded, reinforcement shall be provided for developing the excess force
(cl.34.4.1 of IS 456), either by extending the longitudinal bars of columns into the
footing (cl.34.4.2 of IS 456) or by providing dowels as stipulated in cl.34.4.3 of IS
456 and given below:

(i) Sufficient development length of the reinforcement shall be provided to


transfer the compression or tension to the supporting member in accordance with
Version 2 CE IIT, Kharagpur
cl.26.2 of IS 456, when transfer of force is accomplished by reinforcement of
column (cl.34.4.2 of IS 456).

(ii) Minimum area of extended longitudinal bars or dowels shall be 0.5 per
cent of the cross-sectional area of the supported column or pedestal (cl.34.4.3 of
IS 456).

(iii) A minimum of four bars shall be provided (cl.34.4.3 of IS 456).

(iv) The diameter of dowels shall not exceed the diameter of column bars
by more than 3 mm.

(v) Column bars of diameter larger than 36 mm, in compression only can
be doweled at the footings with bars of smaller size of the necessary area. The
dowel shall extend into the column, a distance equal to the development length
of the column bar and into the footing, a distance equal to the development
length of the dowel, as stipulated in cl.34.4.4 of IS 456 and as shown in
Fig.11.28.16.

(h) Nominal reinforcement (cl. 34.5 of IS 456)

1. Clause 34.5.1 of IS 456 stipulates the minimum reinforcement and


spacing of the bars in footing slabs as per the requirements of solid slab
(cls.26.5.2.1 and 26.3.3b(2) of IS 456, respectively).

Version 2 CE IIT, Kharagpur


2. The nominal reinforcement for concrete sections of thickness greater
than 1 m shall be 360 mm2 per metre length in each direction on each face, as
stipulated in cl.34.5.2 of IS 456. The clause further specifies that this provision
does not supersede the requirement of minimum tensile reinforcement based on
the depth of section.

11.28.6 Distribution of Base Pressure

Version 2 CE IIT, Kharagpur


The foundation, assumed to act as a rigid body, is in equilibrium under the
action of applied forces and moments from the superstructure and the reactions
from the stresses in the soil. The distribution of base pressure is different for
different types of soil. Typical distributions of pressure, for actual foundations, in
sandy and clayey soils are shown in Figs.11.28.17 and 18, respectively.
However, for the sake of simplicity the footing is assumed to be a perfectly rigid
body, the soil is assumed to behave elastically and the distributions of stress and
stain are linear in the soil just below the base of the foundation, as shown in
Fig.11.28.19. Accordingly, the foundation shall be designed for the applied loads,
moments and induced reactions keeping in mind that the safe bearing capacity of
the soil is within the prescribed limit. It is worth mentioning that the soil bearing
capacity is in the serviceable limit state and the foundation structure shall be
designed as per the limit state of collapse, checking for other limit states as well
to ensure an adequate degree of safety and serviceability.

In the following, the distributions of base pressure are explained for (i)
concentrically loaded footings, (ii) eccentrically loaded footings and (iii)
unsymmetrical (about both the axes) footings.

Version 2 CE IIT, Kharagpur


(i) Concentrically loaded footings

Figure 11.28.20 shows rectangular footing symmetrically loaded with


service load P1 from the superstructure and P2 from the backfill including the
weight of the footing. The assumed uniformly distributed soil pressure at the base
of magnitude q is obtained from:

q = (P1 + P2)/A
(11.9)
where A is the area of the base of the footing.

In the design problem, however, A is to be determined from the condition


that the actual gross intensity of soil pressure does not exceed qc, the bearing
capacity of the soil, a known given data. Thus, we can write from Eq.11.9:
A = (P1 + P2)/qc
(11.10)
Version 2 CE IIT, Kharagpur
From the known value of A, the dimensions B and L are determined such
that the maximum bending moment in each of the two adjacent projections is
equal, i.e., the ratio of the dimensions B and L of the footing shall be in the same
order of the ratio of width b and depth D of the column.

(ii) Eccentrically loaded footings

In most of the practical situations, a column transfers axial load P and


moment M to the footing, which can be represented as eccentrically loaded
footing when a load P is subjected to an eccentricity e = M/P. This eccentricity
may also be there, either alone or in combined mode, when
• the column transfers a vertical load at a distance of e from the centroidal axis
of the footing, and

Version 2 CE IIT, Kharagpur


• the column or the pedestal transfers a lateral load above the level of
foundation, in addition to vertical loads.

Accordingly, the distribution of pressure may be of any one of the three


types, depending on the magnitude of the eccentricity of the load, as shown in
Figs.11.28.21b to d. The general expression of qa, the intensity of soil pressure at
a distance of y from the origin is:

qa = P/A ± (Pe/Ix)y
(11.11)

We would consider a rectangular footing symmetric to the column. Substituting


the values of A = BL, Ix = BL3/12 and y = L/2, we get the values of qa at the left
edge.

qa at the left edge = (P/BL) {1 - (6e/L)}


(11.12)

It is evident from Eq.11.12, that the three cases are possible:

(A) when e < L/6, qa at the left edge is compression (+),

(B) when e = L/6, qa at the left edge is zero, and

(C) when e > L/6, qa at the left edge is tension (-).

The three cases are shown in Figs.11.28.21b to d, respectively. It is to be noted


that similar three cases are also possible when eccentricity of the load is
negative resulting the values of qa at the right edge as compression, zero or
tension. Evidently, these soil reactions, in compression and tension, should be
permissible and attainable.

Case (A): when | e | ≤ L/6

Figures 11.28.21b and c show these two cases, when |e| < L/6 or |e| =
L/6, respectively. It is seen that the entire area of the footing is in compression
having minimum and maximum values of q at the two edges with a linear and
non-uniform variation. The values of q are obtained from Eq.11.11.

In the limiting case i.e., when |e| = L/6, the value of qa is zero at one edge
and the other edge is having qa = 2P/BL (compression) with a linear variation.
Similarly, when e = 0, the footing is subjected to uniform constant pressure of
P/BL. Thus, when |e| = L/6, the maximum pressure under one edge of the footing
is twice of the uniform pressure when e = 0.

Version 2 CE IIT, Kharagpur


In a more general case, as in the case of footing for the corner column of
a building, the load may have biaxial eccentricities. The general expression of qa
at a location of (x,y) of the footing, when the load is having biaxial eccentricities
of ex and ey is,

qa = P/A ± P exy/Ix ± P eyx/Iy


(11.13)

Similarly, it can be shown that the rectangular footing of width B and


length L will have no tension when the two eccentricities are such that

6ex/L + 6ey/B ≤ 1
(11.14)

Case (B): when | e | > L/6

Version 2 CE IIT, Kharagpur


The eccentricity of the load more than L/6 results in development of tensile
stresses in part of the soil. Stability, in such case, is ensured by either anchoring
or weight of overburden preventing uplift. However, it is to ensure that maximum
compressive pressure on the other face is within the limit and sufficient factor of
safety is available against over turning. Accordingly, the maximum pressure in
such a case can be determined considering the soil under compression part only.
Further, assuming the line of action of the eccentric load coincides with that of
resultant soil pressure (Fig.11.28.22) we have:

qmax = P/L'B + 12P(0.5 C)(1.5 C)/BL' = 2P/L'B


(11.15)

where L' = 3C
(11.16)

(iii) Unsymmetrical footings

It may be necessary to provide some cutouts in the foundation to reduce


the uplift pressure or otherwise. The footing in such cases becomes
unsymmetrical about both the axes. It is possible to determine the soil pressure
distribution using the structural mechanics principle as given below.

qa(x,y) = P/A ± {(MyIx - MxIxy)(x)/(IxIy - I xy2 )} + {(MxIy - MyIxy)(y)/(IxIy - I xy2 )}


(11.17)

where Mx = moment about x axis,

My = moment about y axis,

Ix = moment of inertia about x axis,

Iy = moment of inertia about y axis,

Ixy = product of inertia

11.28.7 Practice Questions and Problems with Answers


Q.1: (A) What are the two essential requirements of the design of foundation?

(B) Mention five points indicating the differences between the design of
foundation and the design of other elements of superstructure.

A.1: See sec. 11.28.1.

Q.2: Draw sketches of different shallow foundations.

Version 2 CE IIT, Kharagpur


A.2: Figure Nos. 11.28.1 to 11.

Q.3: Explain the difference between gross and net safe bearing capacities of soil.
Which one is used for the design of foundation?

A.3: See sec. 11.28.3.

Q.4: How would you determine the minimum depth of foundation?

A.4: See sec.11.28.4.

Q.5: What are the critical sections of determining the bending moment in
isolated footing?

A.5: See part (c)2 of sec.11.28.5.

Q.6: Explain the one-way and two-way shears of foundation slabs.

A.6: See part (d) of sec.11.28.5.

Q.7: Draw the actual distributions of base pressures of soil below the footing in
sandy and clayey soils. Draw the assumed distribution of base pressure
below the footing.

A.7: Figure Nos. 11.28.17 and 18.

Q.8: Draw the distributions of pressure in a footing for concentric and


eccentric loadings (e ≤ L/6 and e > L/6).

A.8: Figure Nos. 11.28.20 and 21.

Q.9: How would you determine the pressure at any point (x,y) of a foundation
which is unsymmetrical?

A.9: See part (iii) of sec.11.28.6.

11.28.8 References
1. Reinforced Concrete Limit State Design, 6th Edition, by Ashok K. Jain,
Nem Chand & Bros, Roorkee, 2002.
2. Limit State Design of Reinforced Concrete, 2nd Edition, by P.C.Varghese,
Prentice-Hall of India Pvt. Ltd., New Delhi, 2002.
3. Advanced Reinforced Concrete Design, by P.C.Varghese, Prentice-Hall of
India Pvt. Ltd., New Delhi, 2001.

Version 2 CE IIT, Kharagpur


4. Reinforced Concrete Design, 2nd Edition, by S.Unnikrishna Pillai and
Devdas Menon, Tata McGraw-Hill Publishing Company Limited, New
Delhi, 2003.
5. Limit State Design of Reinforced Concrete Structures, by P.Dayaratnam,
Oxford & I.B.H. Publishing Company Pvt. Ltd., New Delhi, 2004.
6. Reinforced Concrete Design, 1st Revised Edition, by S.N.Sinha, Tata
McGraw-Hill Publishing Company. New Delhi, 1990.
7. Reinforced Concrete, 6th Edition, by S.K.Mallick and A.P.Gupta, Oxford &
IBH Publishing Co. Pvt. Ltd. New Delhi, 1996.
8. Behaviour, Analysis & Design of Reinforced Concrete Structural Elements,
by I.C.Syal and R.K.Ummat, A.H.Wheeler & Co. Ltd., Allahabad, 1989.
9. Reinforced Concrete Structures, 3rd Edition, by I.C.Syal and A.K.Goel,
A.H.Wheeler & Co. Ltd., Allahabad, 1992.
10. Textbook of R.C.C, by G.S.Birdie and J.S.Birdie, Wiley Eastern Limited,
New Delhi, 1993.
11. Design of Concrete Structures, 13th Edition, by Arthur H. Nilson, David
Darwin and Charles W. Dolan, Tata McGraw-Hill Publishing Company
Limited, New Delhi, 2004.
12. Concrete Technology, by A.M.Neville and J.J.Brooks, ELBS with
Longman, 1994.
13. Properties of Concrete, 4th Edition, 1st Indian reprint, by A.M.Neville,
Longman, 2000.
14. Reinforced Concrete Designer’s Handbook, 10th Edition, by C.E.Reynolds
and J.C.Steedman, E & FN SPON, London, 1997.
15. Indian Standard Plain and Reinforced Concrete – Code of Practice (4th
Revision), IS 456: 2000, BIS, New Delhi.
16. Design Aids for Reinforced Concrete to IS: 456 – 1978, BIS, New Delhi.

11.28.9 Test 28 with Solutions


Maximum Marks = 50, Maximum Time = 30 minutes

Answer all questions.

TQ.1: (A) What are the two essential requirements of the design of foundation?
(5
marks)

(B) Mention five points indicating the differences between the design of
foundation and the design of other elements of superstructure.
(5 marks)

A.TQ.1: See sec. 11.28.1.

TQ.2: How would you determine the minimum depth of foundation? (10
marks)

Version 2 CE IIT, Kharagpur


A.TQ.2: See sec.11.28.4.

TQ.3: What are the critical sections of determining the bending moment in
isolated footing?
(10 marks)

A.TQ.3: See part (c)2 of sec.11.28.5.

TQ.4: Explain the one-way and two-way shears of foundation slabs. (10
marks)

A.TQ.4: See part (d) of sec.11.28.5.

TQ.5: Draw the distributions of pressure in a footing for concentric and


eccentric loadings (e ≤ L/6 and e > L/6).
(10 marks)

A.TQ.5: Figure Nos. 11.28.20 and 21.

10.26.11 Summary of this Lesson


This lesson explains the two major and other requirements of the
design of foundation structures. Various types of shallow foundations and pile
foundation are discussed explaining the distribution of pressure in isolated
footings loaded concentrically and eccentrically with e ≤ L/6 and e > L/6. The
gross and net safe bearing capacities are explained. The equation for
determining the minimum depth of the foundation is given. Various design
considerations in respect of minimum nominal cover, thickness at the edge of
footing, bending moment, shear force, bond, tensile reinforcement, transfer of
load at the base of the column, and minimum distribution reinforcement are
discussed, mentioning the codal requirements. The actual and the assumed
distributions of base pressure are discussed. The distributions of base pressure
for concentric and eccentric loads with eccentricity ≤ L/6 and > L/6 are
explained. Determination of bearing pressure of soil for unsymmetrical footing is
also discussed.

All the discussions are relevant in understanding the load carrying


mechanism of the foundation and the behaviour of soil. These understandings
are essential in designing the foundation structures which is taken up in the next
lesson.

Version 2 CE IIT, Kharagpur


Module
11
Foundations - Theory
and Design
Version 2 CE IIT, Kharagpur
Lesson
29
Design of Foundations
Version 2 CE IIT, Kharagpur
Instructional Objectives:
At the end of this lesson, the student should be able to:

• understand and apply the design considerations to satisfy the major and
other requirements of the design of foundations,

• design the plain concrete footings, isolated footings for square and
rectangular columns subjected to axial loads with or without the moments,
wall footings and combined footings, as per the stipulations of IS code.

11.29.1 Introduction
The two major and some other requirements of foundation structures are
explained in Lesson 28. Different types of shallow and deep foundations are
illustrated in that lesson. The design considerations and different codal provisions
of foundation structures are also explained. However, designs of all types of
foundations are beyond the scope of this course. Only shallow footings are taken
up for the design in this lesson. Several numerical problems are illustrated
applying the theoretical considerations discussed in Lesson 28. Problems are
solved explaining the different steps of the design.

11.29.2 Numerical Problems


Problem 1:

Design a plain concrete footing for a column of 400 mm x 400 mm


carrying an axial load of 400 kN under service loads. Assume safe bearing
capacity of soil as 300 kN/m2 at a depth of 1 m below the ground level. Use M 20
and Fe 415 for the design.

Solution 1:

Plain concrete footing is given in secs.11.28.2(A)1 and 11.28.5(b).

Step 1: Transfer of axial force at the base of column

It is essential that the total factored loads must be transferred at the base
of column without any reinforcement. For that the bearing resistance should be
greater than the total factored load Pu.

Here, the factored load Pu = 400(1.5) = 600 kN.

Version 2 CE IIT, Kharagpur


The bearing stress, as per cl.34.4 of IS 456 and given in Eqs.11.7 and 8 of
sec.11.28.5(g) of Lesson 28, is

σ br = 0.45 fck (A1/A2)1/2


(11.7)

with a condition that

(A1/A2)1/2 ≤ 2.0
(11.8)

Since the bearing stress σ br at the column-footing interface will be governed by


the column face, we have A1 = A2 = 400(400) = 160000 mm2. Using A1 = A2, in
Eq.11.7, we have

Pbr = Bearing force = 0.45 fck A1 = 0.45(20)(160000)(10-3) = 1440 kN > Pu (=


600 kN).

Thus, the full transfer of load Pu is possible without any reinforcement.

Step 2: Size of the footing

Version 2 CE IIT, Kharagpur


Let us assume the weight of footing and back fill soil as 15 per cent of Pu.
Then, the base area required = 400(1.15)/300 = 1.533 m2. Provide 1250 x 1250
mm (= 1.5625 m2) as shown in Fig.11.29.1. The bearing pressure qa =
400(1.15)/(1.25)(1.25) = 294.4 kN/m2.

Step 3: Thickness of footing

The thickness of the footing h is governed by Eq.11.3 of sec.11.28.5 of


Lesson 28. From Eq.11.3, we have

tan α ≤ 0.9{(100qa/fck) + 1}1/2 …. (11.3)

≤ 0.9[{100(0.2944)/20} + 1]1/2

≤ 1.415

We have from Fig.11.29.1a:

h = {(1250 - 400)/2}(tan α ) = 601.375 mm

Provide 1250 x 1250 x 670 mm block of plain concrete.

Step 4: Minimum reinforcement

The plain concrete block 1250 x 1250 x 670 shall be provided with the
minimum reinforcement 0.12 per cent for temperature, shrinkage and tie action.

Minimum Ast = 0.0012(1250)(670) = 1005.0 mm2.

Provide 9 bars of 12 mm diameter (= 1018 mm2) both ways as shown in


Fig.11.29.1b. The spacing of bars = (1250 - 50 - 12)/8 = 148.5 mm c/c. Provide
the bars @ 140 mm c/c.

Step 5: Check for the gross base pressure

Assuming unit weights of concrete and soil as 24 kN/m3 and 20 kN/m3

Service load = 400.00 kN

Weight of footing = (0.67)(1.25)(1.25)(24) = 25.125 kN

Weight of soil = (0.33)(1.25)(1.25)(20) = 10.3125 kN

Total = 435.4375 kN

qa = 435.4375/(1.25)(1.25) = 278.68 kN/m2 < 300 kN/m2

Version 2 CE IIT, Kharagpur


Hence, o.k.

Problem 2:

Design an isolated footing for a square column, 400 mm x 400 mm with


12-20 mm diameter longitudinal bars carrying service loads of 1500 kN with M 20
and Fe 415. The safe bearing capacity of soil is 250 kN/m2 at a depth of 1 m
below the ground level. Use M 20 and Fe 415.

Solution 2:

Step 1: Size of the footing

Version 2 CE IIT, Kharagpur


Given P = 1500 kN, qc = 250 kN/m2 at a depth of 1 m below the ground
level. Assuming the weight of the footing and backfill as 10 per cent of the load,
the base area required = 1500(1.1)/250 = 6.6 m2. Provide 2.6 m x 2.6 m, area =
6.76 m2 (Fig.11.29.2b).

Step 2: Thickness of footing slab based on one-way shear

Factored soil pressure = 1500(1.5)/(2.6)(2.6) = 0.3328 N/mm2, say, 0.333


2
N/mm .

Assuming p = 0.25% in the footing slab, for M 20 concrete τ c = 0.36


2
N/mm (Table 19 of IS 456). Vu = 0.36(2600)d and Vu (actual) =
0.333(2600)(1100 - d). From the condition that Vu should be more than or equal
to the actual Vu, we have (see Fig.11.29.2a, sec.11):

0.36(2600)d ≥ 0.333(2600) (1100 – d)

So, d ≥ 528.57 mm.

Provide d = 536 mm. The total depth becomes 536 + 50 + 16 + 8 (with 50 mm


cover and diameter of reinforcing bars = 16 mm) = 610 mm.

Step 3: Checking for two-way shear

The critical section is at a distance of d/2 from the periphery of the column.
The factored shear force (Fig.11.29.2b, sec. 2222) = 0.333{(2600)2 – (400 +
d)2}(10)-3 = 1959.34 kN. Shear resistance is calculated with the shear strength =
ks τ c = ks(0.25) (fck)1/2; where ks = 0.5 + β c (cl. 31.6.3 of IS 456). Here β c = 1.0,
ks = 1.5 >/ 1; so ks = 1.0. This gives shear strength of concrete = 0.25 (fck)1/2 =
1.118 N/mm2. So, the shear resistance = (1.118) 4 (936)(536) = 2243.58 kN >
1959.34 kN. Hence, ok.

Thus, the depth of the footing is governed by one-way shear.

Step 4: Gross bearing capacity

Assuming unit weights of concrete and soil as 24 kN/m3 and 20 kN/m3,


respectively:

Given, the service load = 1500 kN

Weight of the footing = 2.6(2.6)(0.61)(24) = 98.967 kN

Weight of soil = 2.6(2.6)(1.0-0.61)(20) = 52.728 kN (Assuming the depth of


the footing as 1.0 m).
Total = 1635.2 kN

Version 2 CE IIT, Kharagpur


Gross bearing pressure = 1635.2/(2.6)(2.6) = 241.893 kN/m2 < 250 kN/m2.
Hence, ok.

Step 5: Bending moment

The critical section (Fig.11.29.2b, sec.3.3), is at the face of the column.

Mu = 0.333(2600)(1100)(550) Nmm = 523.809 kNm

Moment of resistance of the footing = Rbd2 where R = 2.76 (Table 3.3 of Lesson
5).

Moment of resistance = 2.76(2600)(536)(536) = 2061.636 kNm > 523.809 kNm.

Area of steel shall be determined from Eq.3.23 of Lesson 5, which is:

Mu = 0.87 fy Ast d {1 – (Ast fy/fck bd)} …. (3.23)

Substituting Mu = 523.809 kNm, fy = 415 N/mm2, fck = 20 N/mm2, d = 536 mm,


b = 2600 mm, we have:

Ast2 - 67161.44578 Ast + 181.7861758 (105) = 6

Solving, we get Ast = 2825.5805 mm2.

Alternatively, we can use Table 2 of SP-16 to get the Ast as explained below:

Mu/bd2 = 523.809(106)/(2600)(536)(536) = 0.7013 N/mm2. Table 2 of


SP-16 gives p = 0.2034.

Ast = 0.2034(2600)(536)/100 = 2834.58 mm2.

This area is close to the other value = 2825.5805 mm2.

However, one-way shear has been checked assuming p = 0.25%. So, use
p = 0.25%. Accordingly, Ast = 0.0025(2600)(536) = 3484 mm2.

Provide 18 bars of 16 mm diameter (= 3619 mm2) both ways. The spacing of


bars = {2600 – 2(50) – 16}/17 = 146.117 mm. The spacing is 140 mm c/c
(Fig.11.29.2).

The bending moment in the other direction is also the same as it is a


square footing. The effective depth, however, is 16 mm more than 536 mm. But,
the area of steel is not needed to be determined with d = 552 mm as we are
providing 0.25 per cent reinforcement based on one-way shear checking.

Version 2 CE IIT, Kharagpur


Step 6: Development length

Ld = fs φ /4( τ bd ) = 0.87(415)(16)/4(1.6)(1.2) = 47(16) = 752 mm (cl.26.2.1 of IS


456).

Length available = 1100 – 50 = 1050 mm > 752 mm.

Step 7: Transfer of force at the base of the column

Pu = 1500(1.5) = 2250 kN

Compressive bearing resistance = 0.45 fck(A1/A2)1/2. For the column face A1/A2 =
1 and for the other face A1/A2 > 2 but should be taken as 2. In any case, the
column face governs.

Force transferred to the base through column at the interface =


0.45(20)(400)(400) = 1440 kN < 2250 kN. The balance force 2250 – 1440 = 810
kN has to be transferred by the longitudinal reinforcements, dowels or
mechanical connectors. As it is convenient, we propose to continue the
longitudinal bars (12-20 mm diameter) into the footing. The required development
length of 12-20 mm diameter bars, assuming a stress level of 0.87fy(810/2250) =
129.978 N/mm2, is 129.978(20)/4(1.6)(1.2)(1.25) = 270.8 mm. Here τ bd for M 20
= 1.2 N/mm2, increased factor of 1.6 is due to deformed bars and increased
factor of 1.25 is for the compression.

Length available = 610 – 50 – 16 – 16 – 16 = 512 mm > 270.8 mm


(Fig.11.29.2a). Hence, o.k. The arrangement is shown in Fig.11.29.2c.

Alternatively: Design of dowels

Version 2 CE IIT, Kharagpur


For the balance force 810 kN, the area of dowels = 810000/0.67(415) =
2913.15 mm2. Minimum area = 0.5(400)(400)/100 = 800 mm2 < 2913.15 mm2
(cl.34.4.3 of IS 456). Therefore, number of 16 mm dowels = 2913.15/201 = 15.
The development length of 16 mm dowels in compression =
0.87(415)(16)/4(1.6)(1.2)(1.25) = 601.76 mm. Available vertical embedment
length = 610 – 50 – 16 – 16 – 16 = 512 mm. So, the dowels will be extended by
another 100 mm horizontally, as shown in Fig.11.29.2c.

Problem 3:

Design a sloped footing for a square column of 400 mm x 400 mm with 16


longitudinal bars of 16 mm diameter carrying a service load of 1400 kN. Use M
20 and Fe 415 both for column and footing slab. The safe bearing capacity of soil
is 150 kN/m2.

Solution 3:

Version 2 CE IIT, Kharagpur


Step 1: Size of the footing

Given P = 1400 kN and qc = 150 kN/m2. Assuming the weight of the


footing and the back file as 10 per cent of the load, the required base area is:
1400(1.1)/150 = 10.27 m2. Provide 3400 x 3400 mm giving 11.56 m2
(Fig.11.29.3b).

Step 2: Thickness of footing slab based on one-way shear

Version 2 CE IIT, Kharagpur


Factored bearing pressure = 1400(1.5)/(3.4)(3.4) = 181.66 kN/m2 =
0.18166 N/mm2. Assuming 0.15 per cent reinforcement in the footing slab, Table
19 of IS 456 gives τ c for M 20 = 0.28 N/mm2. From the condition that the one-
way shear resistance ≥ one-way shear force, we have at a distance d from the
face of the column (sec.1-1of Figs.11.29.3a and b).

0.28(3400)d ≥ 0.18166(1500 – d)(3400)

or d ≥ 590.24 mm.

Provide total depth of footing as 670 mm, so that the effective depth = 670 – 50 –
16 – 8 = 596 mm. (The total depth is, however, increased to 750 mm in Step 7.)

Step 3: Checking for two-way shear (See sec.11.28.5.d-2)

At the critical section 2222 (Figs.11.29.3a and b), the shear resistance =
4(400 + 596)(596)(0.25)(fck)1/2 = 2654.73 kN.

The shear force = {(3.4)(3.4) – (0.996)(0.996)}0.18166 = 1919.78 kN <


2654.73 kN. Hence, o.k.

Step 4: Gross bearing capacity

Assuming unit weights of concrete and soil as 25 kN/m3 and 18 kN/m3,


respectively, we have:

Load on footing = 1400.00 kN

Weight of footing = (3.4)(3.4)(0.67)(25) = 193.63 kN

Weight of soil = (3.4)(3.4)(1.25 - 0.67)(18) = 120.69 kN (Assuming the


depth of the footing as 1.25 m).

Total = 1714.32 kN

Gross bearing capacity = 1714.32 / (3.4)(3.4) = 148.30 kN/m2 < 150


2
kN/m . Hence o.k.

Step 5: Bending moment

We have to determine the area of steel in one direction as it is a square


footing. So, we consider the lower effective depth which is 596 mm. The critical
section is sec.33 (Figs.11.29.3a and b), where we have

Mu = 3400(1500)(0.18166)(1500)/2 = 694.8495 kNm

Version 2 CE IIT, Kharagpur


Mu/bd2 = 694.8495(106)/(3400)(596)(596) = 0.575 N/mm2

Table 2 of SP-16 gives, p = 0.165%. Accordingly, area of steel =


0.165(3400)(596)/100 = 3343.56 mm2. Provide 30 bars of 12 mm diameter (=
3393 mm2), both ways.

Step 6: Development length

Development length of 12 mm diameter bars = 0.87(415)(12)/4(1.6)(1.2) =


564.14 mm. Hence, o.k.

Step 7: Providing slope in the footing slab

Since the three critical sections (.i.e., of bending moment, two-way shear
and one-way shear) are within a distance of 596 mm from the face of the column,
the full depth of the footing slab is provided up to a distance of 700 mm from the
face of the column. However, by providing slope the available section now is a
truncated rectangle giving some less area for the one-way shear. Accordingly,
the depth of the footing is increased from 670 mm to 750 mm. With a cover of 50
mm and bar diameter of 12 mm in both directions, the revised effective depth =
750 – 50 – 12 – 6 = 682 mm. Providing the minimum depth of 350 mm at the
edge, as shown in Figs.11.29.3a and b, we check the one-way shear again,
taking into account of the truncated rectangular cross-section at a distance of
682 mm from the face of the column.

One-way shear force = 0.18166(1500 – 682)(3400) = 505232.792 N

Area of truncated rectangle = 1800(682) + 1600(282) + 1600(682 – 282)/2

= 1998800 mm2

The shear stress = 505232.792/1998800 = 0.2527 N/mm2 < 0.28 N/mm2.


Hence, o.k.

Step 8: Revised area of steel

The bending moment in step 5 is 694.8495 kNm at the face of the column.
With d = 682 mm now, we have

Mu/bd2 = 694.8495(106)/(3400)(682)(682) = 0.4394 N/mm2

Table 2 of SP-16 gives, p is less than 0.15 per cent. Provide p = 0.15 per cent
due to the one-way shear. So, Ast = 0.15(3400)(682)/100 = 3478.2 mm2. Provide
31 bars of 12 mm (Ast = 3506 mm2), both ways. Effectively, the number of bars
has increased from 30 to 31 now.

Version 2 CE IIT, Kharagpur


Step 9: Transfer of force at the base of the column

Pu = 1400(1.5) = 2100 kN. Compressive bearing resistance = 0.45


fck(A1/A2)1/2 = 0.45(20)(1) = 9 N/mm2.

Force transferred at the base through the column = 9(400)(400)(10-3) =


1440 kN < 2100 kN.

Provide dowels for the excess (2100 – 1440) = 660 kN. The area of
dowels = 660(103)/(0.67)(415) = 2373.67 mm2. Minimum area of dowels =
0.5(400)(400)/100 = 800 mm2. Provide 12 dowels of 16 mm diameter (area =
2412 mm2).

The development length of 16 mm dowels =


0.87(415)(16)/4(1.6)(1.2)(1.25) = 601.76 mm.

The vertical length available 750 – 50 – 12 – 12 – 16 = 660 mm > 601.76


mm. Hence, o.k. The arrangement of reinforcement and dowels is shown in
Fig.11.29.3a and b.

Problem 4:

Design one isolated footing for a column 300 mm x 450 mm, having 20
bars of 20 mm diameter (Ast = 4021 mm2) of Problem 1 of sec.10.25.6 of Lesson
25 carrying Pu = 1620 kN and Mu = 170 kNm using M 25 and Fe 415. Assume
that the moment is reversible. The safe bearing capacity of the soil is 200 kN/m2
at a depth of 1 metre from ground level. Use M 25 and Fe 415 for the footing.

Solution 4:

Step 1: Size of the footing

Version 2 CE IIT, Kharagpur


Given Pu = 1620 kN and Mu = 170 kNm. The footing should be symmetric
with respect to the column as the moment is reversible. Assuming the weights of
footing and backfill as 15 per cent of Pu, the eccentricity of load Pu at the base is
e = Mu/P(1.15) = 170(106)/1620(1.15)(103) = 91.25 mm. This eccentricity may be
taken as < L/6 of the footing.

Version 2 CE IIT, Kharagpur


The factored bearing pressure is 200(1.5) = 300 kN/m2. For the footing of
length L and width B, we, therefore, have:

Pu/BL + 6M/BL2 ≤ 300

or, 1620(1.15)/BL + 6(170)/BL2 ≤ 300

or, BL2 – 6.21L – 3.4 ≤ 0


(1)

For the economic proportion, let us keep equal projection beyond the face
of the column in the two directions. This gives

(L – 0.45)/2 = (B – 0.3)/2

or, B = L – 0.15
(2)

Using Eq.(2) in Eq.(1), we have

(L – 0.15) L2 – 6.212 – 3.4 ≤ 0

or L3 – 0.15 L2 – 6.21 L – 3.4 ≤ 0

We have L = 2.8 m and B = 2.65 m. Let us provide L = 2.85 m and B =


2.70 m (Fig.11.29.4b). We get the maximum and minimum pressures as

1620(1.15)/(2.85)(2.70) ± 170(6)/(2.7)(2.85)(2.85) = 242.105 ± 46.51 = 288.615


kN/m2 and 195.595 kN/m2, respectively (Fig.11.29.4c). Both the values are less
than 300 kN/m2. Hence, o.k.

Step 2: Thickness of footing slab based on one-way shear

The critical section (sec.11 of Figs.11.29.4a and b) is at a distance d from


the face of the column. The average soil pressure at sec.11 is {288.615 –
(288.615 – 195.595)(1200 – d)/2850} = 249.449 + 0.0326d.

The one-way shear force at sec.11 = (2.7)(1.2 – 0.001d((249.449 +


0.0326d) kN. Assuming 0.15 per cent reinforcement in the footing slab, the shear
strength of M 25 concrete = 0.29 N/mm2. Hence, the shear strength of the
section = 2700(d)(0.29)(103) kN. From the condition that shear strength has to be
≥ shear force, we have

2700(d)(0.29)(10-3) = (2.7)(1.2 – 0.001d)(249.449 + 0.0326d)

This gives,

Version 2 CE IIT, Kharagpur


d2 + 15347.51534d – 9182171.779 = 0

Solving, we get d = 576.6198. Let us assume d = 600 mm

Step 3: Checking for two-way shear

At the critical section 2222 (Figs.11.29.4a and b), the shear resistance is
obtained cl.31.6.31 of IS 456, which gives τ c = (0.5 + 450/300)(0.25)(25)1/2 but
the multiplying factor (0.5 + 450/300) >/ 1.0. So, we have τ c = 0.25(25)1/2 = 1.25
N/mm2. Hence, the shear resistance = (1.25)(2){(300 + 600) + (450 + 600)}(600)
= 2925 kN.

Actual shear force is determined on the basis of average soil pressure at


the centre line of the cross-section which is (195.595 + 288.615)/2 = 242.105
kN/m2 (Fig.11.29.4c). So, the actual shear force = Vu = (242.105){(2.7)(2.85) –
(0.3 + 0.6)(0.45 + 0.6)} = 1634.209 kN < shear resistance (= 2925 kN). Hence,
the depth of the footing is governed by one-way shear. With effective depth = 600
mm, the total depth of footing = 600 + 50 (cover) + 16 (bar dia) + 8 (half bar dia)
= 674 mm.

Step 4: Gross bearing capacity

Assuming the unit weights of concrete and soil as 25 kN/m3 and 18 kN/m3,
respectively, we have the bearing pressure for (i) Pu = 1620 kN, (ii) Mu = 170
kNm and (iii) self weight of footing and backfill soil.

(i) Due to Pu = 1620 kN: pressure = 1620/(2.7)(2.85) = 210.53 kN/m2

(ii) Due to Mu = 170 kNm: pressure = ± 170(6)/(2.7)(2.85)(2.85) = ± 46.51


kN/m2

(iii) Self weight of footing of depth 674 mm and soil of (1000 – 674) = 326 mm:
pressure = 0.674(25) + 0.326(18) = 22.718 kN/m2

Thus, the maximum and minimum pressures are = 210.53 + 22.718 ± 46.51 =
279.758 kN/m2 and 186.738 kN/m2 < 300 kN/m2. Hence, o.k.

Step 5: Bending moment

(i) In the long direction (along the length = 2850 mm)

Bending moment at the face of column (sec.33 of Figs.11.29.4a and b) is


determined where the soil pressure = 288.615 – (288.615 – 195.595)(1200)/2850
= 249.45 kN/m2. So, the bending moment = 249.45(2.7)(1.2)(0.6) + (288.615 –
249.45)(2.7)(1.2)(2)/(2)(3) = 527.23 kNm.

Version 2 CE IIT, Kharagpur


M/Bd2 = 527.23(106)/(2700)(616)(616) = 0.515 N/mm2 < 3.45 N/mm2 for M 25
concrete.

Table 3 of SP-16 gives p = 0.1462 < 0.15 per cent as required for one-way
shear.

Thus, Ast = 0.15(2700)(616)/100 = 2494.8 mm2. Provide 13 bars of 16 mm


diameter (area = 2613 mm2), spacing = (2700 – 100 – 16)/12 = 215.33 mm, say
210 mm c/c.

(ii) In the short direction (B = 2700 mm)

The average pressure on soil between the edge and centre of the footing
= (288.615 + 242.105)/2 = 265.36 kN/m2. The bending moment is determined
with this pressure as an approximation.

Bending moment = (265.36)(1.2)(0.6)(2.85) kNm = 544.519 kNm

M/Ld2 = 544.519(106)/(2850)(600)(600) = 0.531

Table 3 of SP-16 gives p = 0.15068, which gives area of steel =


0.15068(2850)(600)/100 = 2576.628 mm2. Provide 13 bars of 16 mm diameter
(area = 2613 mm2) @ 210 mm c/c; i.e. the same arrangement in both directions.

Step 6: Development length

Development length of 16 mm diameter bars (M 25 concrete) =


0.87(415)(16)/4(1.6)(1.4) = 644.73 mm.

Length available = 1200 – 50 – 8 = 1142 mm > 644.73 mm. Hence, o.k.

Step 7: Transfer of force at the base of the column

Since the column is having moment along with the axial force, some of the
bars are in tension. The transfer of tensile force is not possible through the
column-footing interface. So, the longitudinal bars of columns are to be extended
to the footing. The required development length of 20 mm bars =
0.87(415)/4(1.4)(1.6) = 805.92 mm. Length available = 600 mm < 805.92 mm.
The bars shall be given 90o bend and then shall be extended by 200 mm
horizontally to give a total length of 600 + 8(20) (bend value) + 200 = 960 mm >
805.92 mm (Figs.11.29.4 a and b).

The arrangement of reinforcement is shown in Figs.11.29.4a and b.

Version 2 CE IIT, Kharagpur


Problem 5:

Design a combined footing for two columns C1, 400 mm x 400 mm with 8
bars of 16 mm diameter carrying a service load of 800 kN and C2, 300 mm x 500
mm with 8 bars of 20 mm diameter carrying a service load of 1200 kN
(Figs.11.29.5a and b). The column C1 is flushed with the property line. The
columns are at 3.0 m c/c distance. The safe bearing capacity of soil is 200 kN/m2
at a depth of 1.5 m below the ground level. Use M 20 and Fe 415 for columns
and footing.

Version 2 CE IIT, Kharagpur


Solution :

Step 1: Size of the footing

Assuming the weight of combined footing and backfill as 15 per cent of the
total loads of the columns, we have the required base area, considering qc = 200
kN/m2,

Area of the base = (800 + 1200)(1.15)/200 = 11.5 m2.


It is necessary that the resultant of the loads of two columns and the
centroid of the footing coincide so that a uniform distribution of soil pressure is
obtained. Thus, the distance of the centroid of the footing y from column C1
(Fig.11.29.5b) is:

y = 800(0) + 1200(3)/2000 = 1.8 m (Fig.11.29.5b). Since y is greater


than half the c/c distance of column, a rectangular footing has to be designed.
Let us provide 4 m x 3 m and the dimensions are shown in Fig.11.29.5b
coinciding the centroid of the footing and the resultant line of action of the two
loads, i.e. at a distance of 2 m from the left edge.

Version 2 CE IIT, Kharagpur


Step 2: Thickness of footing slab based on one-way shear

Considering the footing as a wide beam of B = 3 m in the longitudinal


direction, the uniformly distributed factored load = (800 + 1200)(1.5)/4 = 750
kN/m. Figures 11.29.6a, b and c present the column loads, soil pressure, shear
force and bending moment diagrams.

The critical section of one-way shear is sec.11 (at point K) of


Figs.11.29.5a and 11.29.6a, at a distance of d + 250 mm from G (the location of
column C2). The one-way shear force is

Shear force = (1600 – d – 250)1200/1600 = (1012.5 – 0.75d) kN

Assuming p = 0.15 per cent reinforcement in the footing slab, the shear strength
of M 20 concrete = 0.28 N/mm2. Hence, the shear strength of section 11 =
(3000)d(0.28)(10-3) kN. From the condition that shear strength ≥ shear force, we
have

(3000)d(0.28)(10-3) ≥ 1012.5 – 0.75d, which gives d ≥ 636.79 mm.


Provide d = 650 mm and the total depth = 650 + 50 + 16 + 8 = 724 mm
(assuming cover = 50 mm and the diameter of bars = 16 mm).

Step 3: Checking for two-way shear

(i) Around column C2

The effective depth along 4.0 m is 650 + 16 = 666 mm. The critical section
for the two-way shear around column C2 is at a distance of 666/2 = 333 mm from
the face of the column and marked by 2222 line in Fig.11.29.5b. The two-way
punching shear force, considering the soil pressure = 750/3 = 250 kN/m2, is

Vu = 1800 – (1.166)(0.966)(250) = 1518.411 kN

As per cl.31.6.3.1 of IS 456, here ks = 0.5 + (500/300) but >/ 1.0; so, ks = 1.0.
Therefore, shear strength of concrete = 0.25(20)1/2 (2){(300 + 666) + (500 +
666)}(666) = 3174.92 kN > 1518.411 kN. Hence, o.k.

(ii) Around column C1

The effective depth of footing is 666 mm. The critical section is marked by
3333 in Fig.11.29.5b. The two-way punching shear = 1200 – (1.066)(0.733)(250)
= 1004.65 kN. The resistance to two-way shear = 0.25(20)1/2 (2){(1066 +
733)}(666) = 2679.1 kN > 1004.65 kN. Hence, o.k.

Thus, the depth of the footing is governed by one-way shear.

Version 2 CE IIT, Kharagpur


Step 4: Gross bearing capacity

Assuming unit weights of concrete and soil as 25 kN/m3 and 18 kN/m3,


respectively, the gross bearing capacity under service load is determined below.

(i) Due to two loads: (800 + 1200)/(3)(4) = 166.67 kN/m2

(ii) Due to weight of the footing: With a total depth of the footing = 724 mm, the
pressure = 0.724(25) = 18.1 kN/m2.

(iii) Due to backfill of 1500 – 724 = 776 mm, the pressure = 0.776(18) = 13.968
kN/m2.

The total pressure = 166.67 + 18.1 + 13.968 = 198.738 kN/m2 < 200 kN/m2.
Hence, o.k.

Step 5: Bending moments (longitudinal direction)

(i) Maximum positive moment

Figure 11.29.6c shows the maximum positive bending moment = 720 kNm
at a distance of 1.4 m from the column C1 (at point J). With effective depth d =
666 mm, we have

M/Bd2 = 720(106)/(3000)(666)(666) = 0.541 N/mm2

Table 2 of SP-16 gives p = 0.1553 per cent.

Ast = 0.1553(3000)(666)/100 = 3102.894 mm2

Provide 16 bars of 16 mm diameter (area = 3217 mm2), spacing = (3000 – 50 –


16)/15 = 195.6 mm c/c, say 190 mm c/c.

Development length of 16 mm bars = 47.01(16) = 752.16 mm

Length available = 1600 – 50 – 16 = 1534 mm > 752.16 mm

Hence, o.k.

(ii) Maximum negative moment

Figure 11.29.6c shows the maximum negative moment = 240 kNm at a


distance of 800 mm from the right edge. With the effective depth = 666 mm, we
have

M/Bd2 = 240(106)/(3000)(666)(666) = 0.018 N/mm2

Version 2 CE IIT, Kharagpur


It is very nominal. So, provide 0.15 per cent steel, which gives Ast =
0.15(3000)(666)/100 = 2997 mm2. Provide 27 bars of 12 mm (area = 3053 mm2)
at spacing = (3000 – 50 – 12)/26 = 113 mm c/c; say 110 mm c/c.

Development length = 47.01(12) = 564 mm

Length available = 800 – 50 – 12 = 738 mm > 564 mm

Hence, o.k.

Step 6: Design of column strip as transverse beam

Figure 11.29.7 shows the two column strips under columns C1 and C2.

Version 2 CE IIT, Kharagpur


(i) Transverse beam under column C1

The width of the transverse beam is 0.75d from the face of column C1. The
effective depth is 666 – 6 – 8 = 652 mm, as the effective depth in the longitudinal
direction = 666 mm, bottom bar diameter in longitudinal direction = 12 mm and
assuming the bar diameter in the transverse direction as 16 mm. We have to
check the depth and reinforcement in the transverse direction considering one-
way shear and bending moment.

(A) One-way shear

The factored load for this transverse strip = 1200/3 = 400 kN/m. The
section of the one-way shear in sec.44 (Fig.11.29.7) at a distance of d = 652 mm
from the face of column C1. The width of the transverse strip = 400 + 0.75(652) =
889 mm.

One-way shear force in sec.44 = (1500 – 652 – 200)(400)(10-3) = 259.2


kN

Shear stress developed = 259.2(103)/(889)(652) = 0.447 N/mm2

To have the shear strength of concrete ≥ 0.447 N/mm2, the percentage of


reinforcement is determined by linear interpolation from Table 19 of IS 456, so
that the depth of footing may remain unchanged. Table 19 of IS 456 gives p =
0.43125. Accordingly, Ast = 0.43125(889)(652)/100 = 2499.65 mm2. Provide 13
bars of 16 mm (area = 2613 mm2), spacing = 889/12 = 74.08 mm c/c, say 70 mm
c/c. However, this area of steel shall be checked for bending moment
consideration also.

(B) Bending moment at the face of column C1 in the transverse strip under column

Bending moment = (1.3)(1.3)(400)/2 = 338 kNm. We, therefore, have

M/(width of strip)d2 = 338(106)/(889)(652)(652) = 0.89 N/mm2

Table 2 of SP-16 gives p = 0.2608 < 0.43125. Hence, the area of steel as
determine for one-way shear consideration is to be provided. Provide 13 bars of
16 mm @ 70 mm c/c in the column strip of width 889 mm under the column C1.

Development length of 16 mm bars = 47.01(16) = 752.16 mm

Length available = 1300 – 50 – 16 = 1234 mm > 752.16 mm

Hence, o.k.

(ii) Transverse beam under column C2

Version 2 CE IIT, Kharagpur


Figure 11.29.7 shows the strip of width = 500 + 0.75d + 0.75 d = 500 +
1.5(652) = 1478 mm, considering the effective depth of footing = 652 mm.

(A) One-way shear

The factored load for this transverse strip = 1800/3 = 600 kN/m. The one-
way shear section is marked by sec.5.5 in Fig.11.29.7 at a distance of d = 652
mm from the face of the column C2.

One-way shear in sec.55 (of width = 1478 mm) = (1500 – 652 –


150)(600)(10-3)

= 418.8 kN

The shear stress developed = 418.8(103)/(1478)(652) = 0.434 N/mm2

The corresponding percentage of area of steel, as obtained from Table 19


of IS 456 is, p = 0.404 per cent. Accordingly, Ast = 0.404(1478)(652)/100 =
3893.17 mm2. Provide 20 bars of 16 mm (area = 4021 mm2), spacing = 1478/19
= 77.78 mm c/c, say 75 mm c/c. However, this area of steel shall be checked for
bending moment consideration also.

(B) Bending moment at the face of column C2 in the transverse strip under column C2

The bending moment = (1.35)(1.35)(600)/2 = 546.75 kNm. We, therefore,


have

M/(width of strip)d2 = 546.75(106)/(1478)(652)(652) = 0.87 N/mm2

Table 2 of SP-16 gives: p = 0.2544 < 0.404 per cent as required for one-way
shear. So, provide 20 bars of 16 mm diameter @ 75 mm c/c in the column strip
of width 1478 under column C2.

Development length as calculated in Step (B) of col strip C1 = 752.16 mm

The length available = 1350 – 50 – 16 = 1284 mm > 752.16 mm

Hence, o.k.

Step 7: Transfer of forces at the base of the columns

(A) For column C1

The limiting bearing stress at the column face governs where the bearing
stress = 0.45fck = 9 N/mm2, since the column C1 is at the edge of the footing.

Version 2 CE IIT, Kharagpur


The force that can be transferred = 9(400)(400)(10-3) = 1440 kN > 1200
kN, load of column C1. Hence full transfer of force is possible without the need of
any dowels. However, four 16 mm nominal dowels are provided as shown in
Figs.11.29.5a and b.

(B) For column C2

Figure 11.29.8 shows the dimensions to determine A1 and A2 for the


column C2 and the footing. Accordingly,

A1 = (1600)(1400) mm2, and

A2 = (300)(500) mm2

(A1/A2)1/2 = {(16)(14)/(3)(5)}1/2 = 3.86 but limited to 2

Version 2 CE IIT, Kharagpur


The bearing stress at the column face = 0.45fck = 9 N/mm2, and the
bearing stress at the footing face = 0.45fck(2) = 18 N/mm2. However, the bearing
stress of 9 N/mm2 governs.

The force that can be transferred through the column C2 = 9(300)(500) =


1350 kN < 1800 kN. For the excess force (1800 – 1350) = 450 kN, dowels shall
be provided. The area of dowels = 450(103)/0.67(415) = 1618.414 mm2. The
minimum area of dowels = 0.5(300)(500)/100 = 750 mm2. Provide 6 dowels of 20
mm diameter (area = 1885 mm2).

The development length required in compression

= 0.97(415)(20)/4(1.6)(1.2)(1.25) = 752.2 mm.

The length available = 652 – 20 = 632 mm

Therefore, the dowels shall be given a 90o bend and shall be extended
horizontally by 100 mm to have a total length of 632 + 8(20) + 100 = 892 mm >
752.2 mm (Figs.11.29.5a and b).

Step 8: Distribution reinforcement

Nominal distribution reinforcement shall be provided at top and bottom


where the main reinforcement bars are not provided. The amount @ 0.12 per
cent comes to 0.12(1000)(652)/100 = 782.4 mm2/metre. Provide 12 mm diameter
bars @ 140 mm c/c (area = 808 mm2/m).

All the reinforcing bars and dowels are shown in Figs.11.29.5a and b.

Version 2 CE IIT, Kharagpur


11.29.3 Practice Questions and Problems with Answers

Version 2 CE IIT, Kharagpur


Q.1: Design an isolated footing for a rectangular column, 300 mm x 500 mm
with 16-20 mm diameter longitudinal bars carrying a service load of 3000
kN (Figs.11.29.9a and b). Assume safe bearing capacity of soil as 260
kN/m2 at a depth of 1 m below ground level. Use M 20 and Fe 415.

A.1:

Step 1: Size of the footing

Given P = 3000 kN and qc = 260 kN/m2. Assuming the weight of the


footing and backfill as 25 per cent of the load, the base area required =
3000(1.25)/260 = 14.42 m2. Provide 3 m x 5 m giving 15 m2 area.

Step 2: Thickness of footing slab based on one-way shear

(i) Along 5 m direction

The critical section of one-way shear is marked by sec.11 in Fig.11.29.9b.

Version 2 CE IIT, Kharagpur


Factored soil pressure = 3000(1.5)/15 = 300 kN/m2 = 0.3 N/mm2

Assuming 0.2 per cent reinforcement in the footing slab, Table 19 of IS


456 gives τ c for M 20 = 0.32 N/mm2. From the condition that the shear
resistance ≥ shear force, we have:

0.32(3000)d ≥ 0.3(2250 – d)(3000)

or d ≥ 1088.71 mm

Using 16 mm diameter bars with 50 mm cover the total depth = 1088.71 +


50 + 16 + 8 = 1162.71 mm. Provide total depth of 1165 mm, so that d = 1091
mm.

(ii) Along 3 m direction

The critical section of one-way shear is marked by sec.2-2 in Fig.11.29.9b.

Resistance shear force = 0.32(5000)(1091) = 1745.6 kN

Actual shear force = 0.3(259)(5000) = 388.5 kN < 1745.6 kN

Hence, o.k.

Step 3: Checking for two-way shear

The critical section is marked by 3-3-3-3 in Fig. 11.29.9b.

The permissible shear strength of concrete M 20

= (0.5 + β ) 0.25(fck)1/2 = 0.25(20)1/2 = 1.118 N/mm2

Total perimeter of section 3-3-3-3 = 2(500 + 1091 + 300 + 1091) = 5964


mm giving area = 5964(1091) = 6506724 mm2.

Actual shear force = {15 – (1.591)(1.391)}(300) = 3836.0757 kN

Shear stress = 3836.0757(103)/6506724 = 0.5896 N/mm2 < 1.118 N/mm2

Hence, o.k.

The depth of the footing slab is governed by one-way shear here.

Version 2 CE IIT, Kharagpur


Step 4: Gross bearing capacity

Assuming unit weights of concrete and soil as 25 kN/m3 and 20 kN/m3,


respectively, we have

Service load = 3000 kN

Weight of footing slab = 3(5)(1.165)(25) = 436.875 kN

Weight of backfill = 3(5)(1.165)(20) = 349.5

Total loads = 3786.375 kN

Gross bearing capacity = 3786.375/15 = 252.425 kN/m2 < 260 kN/m2

Hence, o.k.

Step 5: Bending moments

(i) In the long direction

The critical section of the bending moment in the long direction is marked
by sec.4-4, (at the face of column) in Fig.11.29.9b. With factored soil pressure q
= 0.3 N/mm2, the bending moment is 0.3(3000)(2250)(2250)/2 = 2278.125 kNm.

Moment of resistance of the section = RBd2 = 2.76(3000)(1091)(1091)(10-


6
) = 9855.53 kNm > 2278.125 kNm. Hence, o.k.

The area of steel is determined from Eq.3.23 of Lesson 5, which is,

Mu = 0.87 fy Ast d {1 – Ast fy/fck b d}

or 2278.125(106) = 0.87(415)(Ast)(1091) – 0.87(Ast2)(415)2/(20)(3000)

or Ast2 – 157734.9398 Ast + 912.249313(106) = 0,

which gives Ast = 6012.6245, i.e., p = 0.1837 per cent. However, we have to
provide 0.2 per cent, as required for the one-way shear. So, Ast =
0.2(3000)(1091)/100 = 6546 mm2. Provide 33 bars of 16 mm diameter (area =
6634 mm2, shown in Fig. 11.29.9c). The uniform spacing = (3000 – 50 – 16)/32 =
91.69 mm c/c, say @ 90 mm c/c. Alternatively,

Mu/Bd2 = 2278.125(106)/(3000)(1091)(1091) = 0.638

Version 2 CE IIT, Kharagpur


Table 2 of SP-16 gives p = 0.1834 per cent, which is very close to the computed
p = 0.1837 per cent. So, provide 33 bars of 16 mm diameter @ 90 mm c/c in the
long direction.

The development length of 16 mm bar is 47.01(16) = 752.16 mm

Length available = 2250 – 50 – 16 = 2184 mm > 752.16 mm

Hence, o.k.

(ii) In the short direction

The critical section is marked by sec.5-5 in Fig.11.29.9b, (at the face of


column) where the bending moment is:

0.3(5000)(1350)(1350)/2 = 1366.875 kNm

We get

Mu/Ld2 = 1366.875(106)/(5000)(1091)(1091) = 0.2296

This is a low value, which is not included in SP-16. So, provide 0.15 per cent to
have

Ast = 0.15(5000)(1091)/100 = 8182.5 mm2

However, we have to check for one-way shear as the p provided 0.15 per
cent is less than 0.2 per cent, required for the one-way shear and for which τ c =
0.28 N/mm2 (Table 19 of IS 456). The one-way shear force, as calculated in step
2(ii) is 388.5 kN. The resistance force of shear at this section = 0.28(5000)(1091)
= 1527.4 kN > 388.5 kN. Hence, o.k.

Further, the total area of steel 8182.5 mm2 has to be distributed in the
central band of width 3 m and two outer bands of width 0.75 m each as stipulated
in cl.34.3.1c of IS 456. The reinforcement in the central band is obtained as given
below:

Ast for central band = (2/ β + 1) (total area of steel)

where β = 5/3 = 1.67. Thus,

Area of steel in the central band = (2/2.67)(8182.5) = 6129.21 mm2

Provide 55 bars of 12 mm (area = 6220 mm2) at the spacing of 3000/54 = 55.55


mm c/c, say, @ 55 mm c/c. Each of two outer band needs (8182.5 – 6129.21)/2
= 1026.65 mm2. Provide 10 bars of 12 mm diameter (area = 1131 mm2). The

Version 2 CE IIT, Kharagpur


spacing = (1000 – 50)/10 = 95 mm c/c, say @ 90 mm c/c. Thus, the total area of
steel provided = 6220 + 1131 + 1131 = 8482 mm2 > 8182 mm2 (Fig. 11.29.9c).

The development length required for 12 mm bars = (47.01)(12) = 564.12


mm

Length available = 1350 – 50 – 16 = 1284 mm > 564.12 mm

Hence, o.k.

Step 6: Transfer of force at the base of the column

Factored load = 3000(1.5) = 4500 kN. From the limiting bearing stress at
the column-footing interface 0.45 fck(A1/A2)1/2, we have

(i) At the column face, where A1 = A2; bearing stress = 0.45 fck = 9.0 N/mm2

(ii) At the footing face, A1 = 15 m2 and A2 = (0.3)(0.5) = 0.15 m2. But, (A1/A2)1/2 >/ 2.
So, the bearing stress = 2(9) = 18 N/mm2.

Therefore, the column face governs. The force that can be transferred through
bearing = 9(500)(300) = 1350 kN. The excess force to be transferred by dowels =
4500 – 1350 = 3150 kN.

Area of dowels = 3150(103)/0.67(415) = 11328.9 mm2. Minimum area of


dowels = 0.5(300)(500)/100 = 750 mm2. So, the number of 20 mm dowels = 36.
Instead of providing 36 dowels, it is convenient to extend the sixteen column bars
of 20 mm diameter to the footing.

The development length required = 0.87(415)(20)/4(1.6)(1.25)(1.2) =


752.2 mm

Length available = 1091 – 74 = 1017 mm > 752.2 mm

Step 7: Nominal reinforcement

Clause 34.5.2 stipulates to provide @ 360 mm2 per metre length in each
direction on each force, when the thickness of footing slab is greater than one
metre. So, the minimum reinforcement is 8 mm bars @ 130 mm c/c (area = 387
mm2/m), wherever there is no other reinforcement.

The reinforcement bars are shown in Figs.11.29.9a and c.

Version 2 CE IIT, Kharagpur


Q.2: Design a reinforced concrete footing for a wall of 400 mm thickness
transmitting a load of 200 kN/m (Fig.11.29.10) under service condition. Assume
the safe bearing capacity of soil as 100 kN/m2 at a depth of 1 m below the
ground level. Use M 20 and Fe 415.

A.2:

Step 1: Size of the footing

Given axial load = 200 kN/m and safe bearing capacity of soil qc = 100
2
kN/m at a depth of 1 m below the ground level. Assuming the self weight of
footing and backfill as 10 per cent, the area of the base required = 200(1.1)/100 =
2.2 m2. Provide width of 2.2 m for every one metre to get the required area of 2.2
m 2.

Step 2: Thickness of footing slab based on shear

The critical section of shear is marked as sec.1-1 in Fig.11.29.10, at a


distance of effective depth d of the footing.

Version 2 CE IIT, Kharagpur


Factored soil pressure = 200(1.5)/2.2 = 136.36 kN/m2 = 0.13636 N/mm2.

Assuming 0.15 per cent reinforcement in the footing slab, the shear
strength of M 20 τ c = 0.28 N/mm2. From the condition that the shear strength of
the section ≥ actual shear force in sec.11, we have:

0.28(1000)d ≥ (0.13636)(1000)(900 – d)

This gives d ≥ 294.755 mm. Using cover of 50 mm and diameter of


reinforcing bar as 10 mm, the total depth of footing = 295 + 50 + 10 + 5 = 360
mm. This gives effective depth = 295 mm.

Step 3: Checking for the moment

The critical section for the bending moment is marked marked as sec.2-2
in Fig.11.29.10, where the bending moment (factored) is,

Mu = (1)(0.13636)(1000)(1000)/2 = 68.18 kNm/m

The capacity of the section = (2.76)(1000)(295)(295) = 240.189 kNm/m


> 68.18 kNm/m. Hence, o.k.

M/bd2 = 68.18(106)/1000(295)(295) = 0.784 N/mm2

Table 2 of SP-16 gives p = 0.2282 per cent. Accordingly, Ast =


2
0.2282(1000)(295)/100 = 673.19 mm /m. Provide 10 mm diameter bars @ 110
mm c/c which gives 714 mm2/m > 673.19 mm2/m.

Step 4: Development length

The development length of 10 mm bars = 47.01(10) = 470.1 mm

Length available = 900 – 50 = 850 > 542.88

Step 5: Distribution reinforcement

Minimum reinforcement @ 0.12 per cent should be provided longitudinally.

Ast = 0.12(2200)(360)/100 950.4 mm2

Provide 13 bars of 10 mm diameter (area = 1021 mm2). The spacing = (2200 –


50 – 10)/12 = 178.33 mm c/c, say @ 175 mm c/c.

Step 6: Transfer of loads at wall-footing base

Version 2 CE IIT, Kharagpur


Bearing stress = 0.45 fck (A1/A2)1/2 = 0.45(20)(1) = 9 N/mm2

The forces that can be transferred = 9(1000)(2200)(10-3) kN = 19800 kN


>> factored load of 200(1.5) = 300 kN. Hence, o.k.

11.29.4 References
1. Reinforced Concrete Limit State Design, 6th Edition, by Ashok K. Jain,
Nem Chand & Bros, Roorkee, 2002.
2. Limit State Design of Reinforced Concrete, 2nd Edition, by P.C.Varghese,
Prentice-Hall of India Pvt. Ltd., New Delhi, 2002.
3. Advanced Reinforced Concrete Design, by P.C.Varghese, Prentice-Hall of
India Pvt. Ltd., New Delhi, 2001.
4. Reinforced Concrete Design, 2nd Edition, by S.Unnikrishna Pillai and
Devdas Menon, Tata McGraw-Hill Publishing Company Limited, New
Delhi, 2003.
5. Limit State Design of Reinforced Concrete Structures, by P.Dayaratnam,
Oxford & I.B.H. Publishing Company Pvt. Ltd., New Delhi, 2004.
6. Reinforced Concrete Design, 1st Revised Edition, by S.N.Sinha, Tata
McGraw-Hill Publishing Company. New Delhi, 1990.
7. Reinforced Concrete, 6th Edition, by S.K.Mallick and A.P.Gupta, Oxford &
IBH Publishing Co. Pvt. Ltd. New Delhi, 1996.
8. Behaviour, Analysis & Design of Reinforced Concrete Structural Elements,
by I.C.Syal and R.K.Ummat, A.H.Wheeler & Co. Ltd., Allahabad, 1989.
9. Reinforced Concrete Structures, 3rd Edition, by I.C.Syal and A.K.Goel,
A.H.Wheeler & Co. Ltd., Allahabad, 1992.
10. Textbook of R.C.C, by G.S.Birdie and J.S.Birdie, Wiley Eastern Limited,
New Delhi, 1993.
11. Design of Concrete Structures, 13th Edition, by Arthur H. Nilson, David
Darwin and Charles W. Dolan, Tata McGraw-Hill Publishing Company
Limited, New Delhi, 2004.
12. Concrete Technology, by A.M.Neville and J.J.Brooks, ELBS with
Longman, 1994.
13. Properties of Concrete, 4th Edition, 1st Indian reprint, by A.M.Neville,
Longman, 2000.
14. Reinforced Concrete Designer’s Handbook, 10th Edition, by C.E.Reynolds
and J.C.Steedman, E & FN SPON, London, 1997.
15. Indian Standard Plain and Reinforced Concrete – Code of Practice (4th
Revision), IS 456: 2000, BIS, New Delhi.
16. Design Aids for Reinforced Concrete to IS: 456 – 1978, BIS, New
Reinforced Concrete Limit State Design, 5th Edition, by Ashok K. Jain,
Nem Chand & Bros, Roorkee, 1999.

Version 2 CE IIT, Kharagpur


11.29.5 Test 29 with Solutions
Maximum Marks = 50, Maximum Time = 30 minutes

Answer all questions.

Version 2 CE IIT, Kharagpur


TQ.1: Redesign the footing of Problem 4 of section 11.29.2 (Fig.11.29.4) to have
uniform base pressure considering that moments are due to dead load
and hence irreversible (Fig.11.29.11). Other data of Prolem 4 are: column
size = 300 mm x 450 mm, longitudinal bars of column = 20 bars of 20 mm
(Ast = 4021 mm2), Pu = 1620 kN, Mu = 170 kNm, safe bearing capacity of
soil = 200 kN/m2 at a depth of 1 metre below the ground level, grade of
concrete = M 25 and grade of steel = Fe 415.
(50
marks)

A.TQ.1:

Step 1: Size of the footing

The required eccentricity between the centroids of column and footing =


Mu/Pu = 170(106)/1620(103) = 104.938 mm, say 105 mm.

Assuming the weight of the footing and backfill as 15 per cent, the
required base area = 1620(1.15)/200(1.5) = 6.21 m2.

It is desirable that the cantilever projections in the two direction from the
respective column face should be equal. Accordingly, the footing is selected as
2.7 m x 2.64 m where the centre line of column is located at a distance of 105
mm left from the centre line of footing, as shown in Fig.11.29.11c. The
arrangement shows that the cantilever projections are equal (1200 mm).

Step 2: Thickness of footing based on one-way shear

Factored soil pressure = 1620/(2.7)(2.64) = 227.27 kN/m2 (Fig.11.29.11b).


The critical section of one-way shear is marked by sec.1-1 in Figs.11.29.11a and
c, at a distance of d from the face of the column, where the factored shear Vu is,

Vu = 0.22727(2700)(1200 – d) = (736354.8 – 613.629 d) N

Assuming 0.15 per cent reinforcement in the footing slab, Table 19 of IS 456
gives τ c = 0.29 for M 25 concrete. Accordingly, the resistance shear of the
section is 0.29(2700)d = 783d. From the condition that the resistance shear ≥
actual shear, we have

783 d ≥ 736354.8 – 613.629 d, which gives d ≥ 527.237. Let us use


full depth = 527.237 + 50 + 16 + 8 = 601.237 mm, say 605 mm, so that d = 605
– 50 – 8 = 547 mm in the long direction and 605 – 50 – 16 – 8 = 531 mm in the
short direction.

Step 3: Checking for two-way shear

Version 2 CE IIT, Kharagpur


The shear strength of concrete for two-way shear = 0.25(fck)1/2 (cl.31.6.3 of
IS 456) = 1.25 N/mm2. The resistance shear of the section is (sec.2222 of
Fig.11.29.11c):

(1.25)(2){(450 + 547) + (300 + 547)}(547) = 2521.67 kN

The actual shear force in sec.2222 (Fig. 11.29.11c) is

{(2.7)(2.64) – (0.997)(0.847)}(227.27) = 1428.06 kN < 2521.67 kN

Hence, o.k.

Step 4: Gross bearing capacity

Assuming unit weights of concrete and soil as 25 kN/m3 and 18 kN/m3, we


have the factored gross bearing capacity as

q = (1620)/(2.7)(2.64) + {0.605(25) + 0.395(18)}

= 249.509 kN/m2 < 300 kN/m2

Step 5: Bending moment

(i) In the short direction

The critical section is marked by sec.33 in Fig.11.29.11c, at the face of


column, where the bending moment is:

Mu = (2.7){(1.2)(1.2)/2}227.27 = 441.82 kNm

Mu/Ld2 = 441.82(106)/(2.7)(103)(547)(547) = 0.546 N/mm2 < 3.45 N/mm2


for M 25 and Fe 415.

Hence, the section has the capacity to resist Mu = 441.82 kNm.

Table 3 of SP-16 gives p = 0.15488 per cent. Accordingly,

Ast = 0.15488(2700)(547)/100 = 2287.42 mm2

Provide 12 bars of 16 mm diameter (area = 2412 mm2). The spacing = (2700 –


50 – 16)/11 = 239.45 mm c/c, say 220 mm c/c. Provide 12 bars of 16 mm
diameter @ 220 mm c/c.

Development length of 16 mm diameter bars

= 0.87(415)(16)/4(1.4)(1.6) = 644.73 mm

Version 2 CE IIT, Kharagpur


Length available = 1200 – 50 – 16 = 1134 mm > 644.73 mm

Hence, o.k.

(ii) In the long direction

The critical section is marked by sec.4-4 in Fig.11.29.11c, at the face of


column, where the moment Mu is,

Mu = (2.64){(1.2)(1.2)/2}(227.27) = 431.994 kNm

Mu/Bd2 = 431.994(106)/(2640)(531)(531) = 0.5803 N/mm2 < 3.45 N/mm2

Hence, the section can carry this moment. Table 3 of SP-16 gives p = 0.16509
per cent. Accordingly,

Ast = 0.16509(2640)(531)/100 = 2314.2976 mm2

Provide 12 bars of 16 mm diameter (area = 2412 mm2). The spacing = (2640 –


50 – 16)/11 = 234 mm c/c, say 220 mm c/c. Provide 12 bars of 16 mm diameter
@ 220 mm c/c.

Development length as calculated in (i) for 16 mm diameter = 644.73 mm

Length available = 1200 – 50 – 16 = 1134 mm > 644.73 mm

Hence, o.k.

Step 6: Transfer of force at the base of column

Since the column is having moment along with the axial force, some of the
bars are in tension. The transfer of tensile force is not possible through the
column-footing interface. So, the longitudinal bars of column are to extended to
the footing. The required development length of 20 mm bars =
0.87(415)/4(1.4)(1.6) = 805.92 mm. Length available = 531 mm < 805.92 mm.
The bars shall be given 90o bend and then shall be extended by 200 mm
horizontally to give a total length of 531 + 8(20) (bend value) + 200 = 891 mm >
805.92 mm.

The arrangement of reinforcing bars is shown in Fig.11.29.11a.

Version 2 CE IIT, Kharagpur


11.29.6 Summary of this Lesson
This lesson illustrates the applications of the requirements for the
design of foundations, explained in Lesson 28. Five illustrative examples, two
practice problems and one test problem include plain concrete footings, isolated
footings of square and rectangular columns transferring axial loads with or
without moments to the footings, wall footings and combined footings. The
solutions of illustrative problems and practice of the practice and test problems
will give a clear understanding of the design of footings under different
circumstances and soil conditions. All the designs are done following the
stipulations of IS codes. It is, however, worth mentioning that all the footings
explained in this lesson are shallow foundations. The design of deep foundations
is beyond the scope of this course.

Version 2 CE IIT, Kharagpur


Module
12
Yield Line Analysis for
Slabs
Version 2 CE IIT, Kharagpur
Lesson
30
Basic Principles, Theory and
One-way Slabs
Version 2 CE IIT, Kharagpur
Instructional Objectives:
At the end of this lesson, the student should be able to:

• state the limitations of elastic analysis of reinforced concrete slabs,

• explain the meaning of yield lines,

• explain the basic principle of yield line theory,

• state the assumptions of yield line analysis,

• state the rules for predicting yield patterns and locating the axes of rotation
of slabs with different plan forms and boundaries,

• state the upper and lower bound theorems,

• explain the two methods i.e., (i) method of segmental equilibrium and (ii)
method of virtual work,

• explain if the yield line analysis is a lower or upper bound method,

• analyse one-way slab problems to determine the location of yield lines and
determine the collapse load applying the theoretical formulations of (i)
method of segmental equilibrium and (ii) method of virtual work.

12.30.1 Introduction
The limit state of collapse method of design of beams and slabs considers
the actual inelastic behaviour of slabs when subjected to the factored loads.
Accordingly, it is desirable that the structural analysis of beams and slabs has to
be done taking the inelastic behaviour into account. Though the coefficients in
Annex D-1 of IS 456 to determine the bending moments for the design of two-
way slabs are based on inelastic analysis, the code also recommends the use of
linear elastic theory for the structural analysis in cl. 22.1. Moreover, IS 456 further
stipulates the use of coefficients of moments and shears for continuous beams
given in Tables 12 and 13 of cl. 22.5 in lieu of rigorous elastic analysis. These
coefficients of beams are also applicable for the design of one-way slabs, based
on linear elastic theory. Thus, there are inconsistencies between the methods of
analysis and design.

Version 2 CE IIT, Kharagpur


The above discussion clearly indicates the need of adopting the inelastic
analysis or collapse limit state analysis for all structures. However, there are
sufficient justifications for adopting the inelastic analysis for slabs as evident from
the following limitations of the elastic analysis of slabs:

(i) Slab panels are square or rectangular.

(ii) One-way slab panels must be supported along two opposite sides
only; the other two edges remain unsupported.

(iii) Two-way slab panels must be supported along two pairs of


opposite sides, supports remaining unyielding.

(iv) Applied loads must be uniformly distributed.

(v) Slab panels must not have large opening.

Version 2 CE IIT, Kharagpur


Therefore, for slabs of triangular, circular and other plan forms, for loads
other than uniformly distributed, for support conditions other than those specified
above and for slabs with large openings (Figs.12.30.1a to d); the collapse limit
state analysis has been found to be a powerful and versatile method.

Inelastic or limit analysis is similar to the plastic analysis of continuous


steel beams which is based on formation of plastic hinges to form a mechanism
of collapse. However, full plastic analysis of reinforced concrete beams and
frames is tedious and time consuming. One important advantage of the
reinforced concrete slabs over the reinforced concrete beams and frames is that
the slabs are mostly under-reinforced. This gives a large rotation capacity of
slabs, which may be taken as the presence of sufficient ductility.

12.30.2 Yield Line Theory


The yield line theory, thus, is an ultimate or factored load method of
analysis based on bending moment on the verge of collapse. At collapse loads,
the slab begins to crack as they are mostly under-reinforced, with the yielding of
reinforcement at points of high bending moment. With the propagation of cracks
the yield lines are developed gradually. Finally, a mechanism is formed when the
slab collapses due to uncontrolled rotation of members. Yield lines, therefore, are
lines of maximum yielding moments of the reinforcement of slab. The essence is
to find out the locations of the appropriate yield lines.

Yield line analysis, though first proposed by Ingerslev in 1923 (vide, “The
strength of rectangular slabs”, by A. Ingerslev, J. of Institution of Structural
Engineering, London, Vol. 1, No.1, 1923, pp. 3-14), Johansen is more known for
his large extension of the analysis (please refer to (i) Brutlinieteorier, Jul.
Gjellerups Forlag, Copenhagen, 1943, by K.W. Johansen, English translation,
Cement and Concrete Association London 1962; and (ii) Pladeformler, 2d ed.,
Polyteknisk Forening, Copenhagen, 1949, by K.W. Johansen, English
translation,” Yield Line formulas for slabs, Cement and Concrete Association,
London, 1972). Its importance is reflected in the recommendation of the use of
this method of analysis of slabs in the Note of cl. 24.4 of IS 456. It is to note that
only under-reinforced bending failure is considered in this theory ignoring the
effects due to shear, bond and deflection. Effect of in-plane forces developed is
also ignored.

Version 2 CE IIT, Kharagpur


12.30.3 Basic Principles

Version 2 CE IIT, Kharagpur


As mentioned earlier, reinforced concrete slabs are mostly under-
reinforced and they fail in flexure. Figures 12.30.2a and b show such a reinforced
concrete simply supported one-way slab subjected to short term loading. The
maximum bending moment is along CC, at a distance of L/2 from the supports,
where the deflection and curvature will be the maximum. The curvature φ is
expressed by (Fig. 12.30.2c):

φ = ∈max/x (12.1)

where ∈max is the compressive strain in concrete in the outermost fibre


and x is the depth of the neutral axis. Figure 12.30.2b shows the plastic curvature
φp distributed over a short length DE adjacent to the failure section and the
ignored elastic curvatures φe elsewhere on the slab.

Figure 12.30.3 shows the schematic moment-curvature diagrams of the


slab of Fig.12.30.2a. It is evident from Fig.12.30.3 that up to the point D, when
the crack first appears anywhere along the line CC of Figs.12.30.2a, the line OD
is elastic. Thereafter, the slope of the line changes gradually with the progress of
cracks. Accordingly, the stiffness of the slab is reduced. The reinforcement starts
yielding at point F, when the ultimate strength is almost reached. The two paths
FH for the mild steel reinforcing bars and FI for the deformed bars show marginal
increase in moment capacities. Two points H and I are the respective failure
points showing larger disproportionate deformations and curvatures when the
maximum moment capacity i.e., the resistance of the cross-section Mp is
reached. It may be noted that at these points (H and I), the plastic curvatures φp

Version 2 CE IIT, Kharagpur


are much larger than the elastic curvature φe. Neglecting these marginal moment
capacities in the regions beyond F, the moment curvature diagram is idealised as
OEG. The extended zone of increasing curvature at nearly constant moment and
considering φp as much larger than φe, it is justified to neglect the elastic
curvature φe totally. This amounts to shifting the point E to E′ in Fig. 12.30.3.

The first crack starting anywhere along CC of the one-way slab of


Fig.12.30.2a, after reaching the maximum moment capacity Mp, proceeds
forming plastic hinges. In the process, the crack line or yield line CC is formed
when the slab collapses forming a mechanism. It is worth mentioning that even at
collapse; the two segments AC and BC remain elastic and plane.

Version 2 CE IIT, Kharagpur


Version 2 CE IIT, Kharagpur
It us thus seen that only one yield line is needed to form a mechanism with
two real hinges for the collapse of the statically determinate one-way slab of
Fig.12.30.2a. In case of statically indeterminate slab, the clamped-clamped slab
of Fig.12.30.4a, however, can sustain the loads without collapse even after the
formation of one or more yield lines. Figures 12.30.4c to f show the bending
moment and deflection diagrams of the slab with increasing loads w1 < w2 < w3 <
w4. It is known that during the elastic range, the negative moment at the clamped
ends (w1l2/12) is twice of the positive moment at mid-span (w1l2/24), as shown in
Fig.12.30.4c. Assuming that the slab has equal reinforcement for positive and
negative moments, the highly stressed clamped ends start yielding first. Though
the support line hinges rotate, the restraining moments continue to act till the
mid-span moment becomes equal to the moment at the supports. Accordingly, a
third hinge is formed (Fig.12.30.4f). Now, the three hinges form the mechanism
and the slab collapses showing large deflection and curvature. The moment
diagram just before the collapse is shown in Fig.12.30.4f.

It is clear from the above discussion that such mechanism is possible with
the bending moment diagram of Fig.12.30.4f, if the slab is having adequate
reinforcement to resist equal moments at the support and mid-span. The elastic
bending moment ratio of 1:2 between the mid-span and support could be
increased to 1:1 by the redistribution of moment, which depends on the
reinforcement provided in the supports and mid-span sections and not on the
elastic bending moment diagram shown in Fig.12.30 4c.

12.30.4 Assumptions
The following are the assumptions of the yield line analysis of reinforced
concrete slabs.

1. The steel reinforcement is fully yielded along the yield lines at collapse.
Rotation following yield is at constant moment.

2. The slab deforms plastically at collapse and is separated into


segments by the yield lines. The individual segments of the slab
behave elastically.

3. The elastic deformations are neglected and plastic deformations are


only considered. The entire deformations, therefore, take place only
along the yield lines. The individual segments of the slab remain plane
even in the collapse condition.

4. The bending and twisting moments are uniformly distributed along the
yield lines. The maximum values of the moments depend on the
capacities of the section based on the amount of reinforcement
provided in the section.

Version 2 CE IIT, Kharagpur


5. The yield lines are straight lines as they are the lines of intersection
between two planes.

12.30.5 Rules for Yield Lines

The first requirement of the yield line analysis is to assume possible yield
patterns and locate the axes of rotation.

It has been observed that assuming the possible yield patterns and
locating the axes of rotation are simple to establish for statically determinate or
indeterminate (simply supported or clamped) one-way slabs. For other cases,
however, suitable guidelines are needed for drawing the yield lines and locating
the axes of rotation.

It is worth mentioning that other cases of two-way slabs will have sufficient
number of real or plastic hinges to form a mechanism while they will be on the
verge of collapse. The yield lines will divide the slabs into a number of segments,
which will rotate as rigid bodies about the respective axes of rotation. The axes of
rotations will be located along the lines of support or over columns, if provided as
point supports. The yield line between two adjacent slab segments is a straight
line, as the intersection of two-plane surfaces is always a straight line. The yield
line should contain the point of intersection, if any, of the two axes of rotation of
two adjacent segments as such point of intersection is common to the two
planes.

The two terms, positive and negative yield lines, are used in the analysis
to designate the yield lines for positive bending moments having tension at the
bottom and negative bending moments having tension at the top of the slab,
respectively.

The following are the guidelines for predicting the yield lines and axes of
rotation:

1. Yield lines between two intersecting planes are straight lines.

2. Positive yield line will be at the mid-span of one-way simply supported


slabs.

3. Negative yield lines will occur at the supports in addition to the positive
yield lines at the mid-span of one-way continuous slabs.

Version 2 CE IIT, Kharagpur


4. Yield lines will occur under point loads and they will be radiating
outward from the point of application of the point loads.

5. Yield line between two slab segments should pass through the point of
intersection of the axes of rotation of the adjacent slab segments.

6. Yield lines should end at the boundary of the slab or at another yield
line.

7. Yield lines represent the axes of rotation.

8. Supported edges of the slab will also act as axes of rotation. However,
the fixed supports provide constant resistance to rotation having
negative yield lines at the supported edges. On the other hand, axes of
rotation at the simply supported edges will not provide any resistance
to rotation of the segment.

9. Axis of rotation will pass over any column support, if provided, whose
orientation will depend on other considerations.

Version 2 CE IIT, Kharagpur


Version 2 CE IIT, Kharagpur
Version 2 CE IIT, Kharagpur
Some examples are taken up to illustrate the applications of the guidelines
for predicting the possible yield patterns in Figs.12.30.5a to g.

12.30.6 Upper and Lower Bound Theorems

According to the general theory of structural plasticity, the collapse load of


a structure lies in between the upper bound and lower bound of the true collapse
load. Therefore, the solution employing the theory of plasticity should ensure that
lower and upper bounds converge to the unique and correct values of the
collapse load.

The statements of the two theorems applied to slabs are given below:

(A) Lower bound theorem: The lower bound of the true collapse load is that
external load for which a distribution of moments can be found satisfying
the requirements of equilibrium and boundary conditions so that the
moments at any location do not exceed the yield moment.

(B) Upper bound theorem: The upper bound of the true collapse load is that
external load for which the internal work done by the slab for a small
increment of displacement, assuming that moment at every plastic hinge
is equal to the yield moment and satisfying the boundary conditions, is
equal to the external work done by that external load for the same amount
of small increment of displacement.

Version 2 CE IIT, Kharagpur


Thus, the collapse load satisfying the lower bound theorem is always
lower than or equal to the true collapse load. On the other hand, the
collapse load satisfying the upper bound theorem is always higher than or
equal to the true collapse load.

The yield line analysis is an upper bound method in which the predicted
failure load of a slab for given moment of resistance (capacity) may be higher
than the true value. Thus, the solution of the upper bound method (yield line
analysis) may result into unsafe design if the lowest mechanism could not be
chosen. However, it has been observed that the prediction of the most
probable true mechanism in slab is not difficult. Thus, the solution is safe and
adequate in most of the cases. However, it is always desirable to employ a
lower bound method, which is totally safe from the design point of view.

12.30.7 Methods of Analysis

After predicting the general yield pattern and locating the axes of rotation,
the specific pattern and locations of axes of rotation and the collapse load for the
slab can be determined by one of the two methods given below:

(1) Method of segmental equilibrium

(2) Method of virtual work.

The two methods are briefly explained below.

(1) Method of segmental equilibrium

In this method, equilibrium of the individual slab segments causing the


collapse forming the required mechanism is considered to arrive at a set
of simultaneous equations. The solutions of the simultaneous equations
give the values of geometrical parameters for finalising the yield pattern
and the relation between the load capacity and resisting moment.

Thus, in segmental equilibrium method, each segment of the slab is


studied as a free body (Fig.12.30.5b) which is in equilibrium at incipient
failure under the action of the applied loads, moment along the yield lines,
and reactions or shear along the support lines. Since, yield moments are
principal moments, twisting moments are zero along the yield lines.
Further, in most of the cases, shear forces are also zero. Thus, with
reference to Fig.12.30.5b, the vector sum of moments along yield lines AO

Version 2 CE IIT, Kharagpur


and OB (segment AOB) is equal to moments of the loads on the segment
AOB about the axis of rotation 1-1 just before the collapse.

It should be specially mentioned that equilibrium of a slab segment should


not be confused with the general equilibrium equation of the true
equilibrium method, which is lower bound. Strip method of slab design,
developed by A. Hillerborg, (vide “Equilibrium theory for reinforced
concrete slabs” (in Swedish), Betong, vol.4 No. 4, 1956, pp. 171-182) is a
true equilibrium method resulting in a lower bound solution of the collapse
load, which is safe and preferable too. The governing equilibrium equation
for a small slab element of lengths dx and dy is

∂ 2 Mx ∂ 2 Mxy ∂ 2 My
+ 2 + =−w (12.2)
∂x 2 ∂x∂y ∂y 2

where w is the external load per unit area; Mx and My are the bending
moments per unit width in x and y directions, respectively; and Mxy is the
twisting moment. However, strip method of analysis is beyond the scope
of this course. For more information about strip method, the reader may
refer to Chapter 15 of “Design of concrete structures” by A.H. Nilson, Tata-
McGraw – Hill Publishing Company Limited, New Delhi.

(2) Method of virtual work

This method is based on the principle of virtual work. After predicting the
possible yield pattern and the axes of rotation, the slab, which is in
equilibrium with the moments and loads on the structure, is given an
infinitesimal increase in load to cause the structure further deflection. The
principle of virtual work method is that the external work done by the loads
to cause a small virtual deflection should be equal to the internal work
done by the yield moments to cause the rotation in accommodating the
virtual deflection. The relation between the applied loads and the ultimate
resisting moments of the slab is obtained by equating the internal and
external works. As the elastic deflections and rotations are small
compared to the plastic deformations and rotations, they are neglected in
the governing equation. Further, the compatibility of deflection must be
maintained. The work equation is written as follows:

∑wΔ = ∑Mθ l (12.3)


where

w = collapse load,

Δ = vertical deflection through which the collapse load w moves,

Version 2 CE IIT, Kharagpur


M = moment capacity of the section per unit length,

θ = rotation of the slab segment satisfying the compatibility of deflection,


and

l = length of the yield line.

As mentioned earlier, both the methods of segmental equilibrium and virtual work
are upper bound methods. Therefore, the collapse load obtained by either
method of yield line analysis will be at the higher end of the true collapse load.
Accordingly, each of the two methods should be developed to get the correct
solution for predicted mechanism. However, the true collapse load will be
obtained only if the correct mechanism has been predicted.

Thus, the solution of any of the two upper bound methods has two essential
parts: (1) predicting the correct yield pattern, and (2) determining the geometric
parameters that define the exact location and orientation of the yield pattern and
solving for the relation between applied load and resisting moments.

12.30.8 Analysis of One-Way Slab

Version 2 CE IIT, Kharagpur


Version 2 CE IIT, Kharagpur
Figures12.30.6a and b show one-way continuous slab whose one span of width
one metre is considered. The factored moments of resistance are MA and MB at
the continuous supports A and B where negative yield lines have formed and MC
at C where the positive yield line has formed. All the moments of resistance are
for one metre width. Figure 12.30.6c shows the free body diagrams of the two
segments of slab, which are in equilibrium. We now apply the method of
segmental equilibrium in the free body diagrams of Fig.12.30.6c.

(1) Method of segmental equilibrium

The line CC, where the positive yield line has formed is at a distance of x from
AA. The shear V = 0 at C as the bending moment is the maximum (positive)
there. Here there are two unknowns x, the locations of the positive yield line and
w, the collapse load, which are determined from the two equations of equilibrium
∑V = 0 and ∑M = 0.

From the statical analysis, we know that the vertical reactions VA and VB at A and
B, respectively are, (assuming MA > MB):

VA = (wL/2) + (MA – MB) / L (12.4)


VB = (wL/2) – (MA – MB) / L (12.5)

Now, ∑V = 0 gives: VA – wx = 0
(12.6)

Substituting the expression of VA from Eq. 12.4 in Eq. 12.6, we have, wL/2 + (MA
– MB) / L – wx = 0, which gives:

w = -2 (MA – MB) / L (L-2x) (12.7)

∑M = 0 gives:
VA x – MA – wx2/2 – Mc = 0
(12.8)

Substituting the expression of VA from Eq. 12.4 in Eq. 12.8, we have,

{wL/2 + (MA – MB) / L}x – MA – wx2/2 – MC = 0


(12.9)

Substituting the expression of w from Eq. 12.7 into Eq. 12.9, we have,

(MB – MA) x2 + 2 (MA + MC) Lx – (MA + MC) L2 = 0 (12.10)

Equation 12.10 will give the values of x for known values of MA, MB and MC.
Equation12.7 will give the value of w after getting the value of x from Eq. 12.10.

Version 2 CE IIT, Kharagpur


(2) Method of virtual work

The work equation (Eq.12.3) is written equating the total work done by the
collapse loads during the rotation of slab segments, maintaining the deflection
compatibility to the total internal work done by bending and twisting moments on
all the yield lines. Figure12.30.6d presents the rigid body rotations of the two slab
segments. The left segment AACC rotates by θA in the clockwise direction and
the right segment BBCC rotates by θB in the anticlockwise direction, while
maintaining the deflection Δ compatible, as shown in Fig.12.30.6d.

External work done = wx (Δ/2) + w (L-x) Δ/2 = wLΔ/2


(12.11)

Internal work done = + MA θA + MB θB + MC (θA + θB)


(12.12)

From Fig.12.30.6d:

θA = Δ/x and θB = Δ/(L-x)


(12.13)

Substituting the values of θA and θB from Eq.12.13 into Eq.12.12, we have:

Internal work done = MA (Δ/x) + MB (Δ /L-x) + MC (Δ/x + Δ/L-x)


(12.14)

Equating the works, we have form Eqs.12.11 and 12.14, wL Δ/2 = MA (Δ/x) + MB
(Δ/L-x) + MC (Δ/x + Δ/L-x)

or, w = 2MA / Lx + 2MB / L (L-x) + 2 MC / x (L-x)


(12.15)

In the method of segmental equilibrium, we have two equations


(Eqs.12.7 and 12.9) for the two unknowns x and w. In the method virtual work,
however, we have only one equation (Eq.12.15) to determine the two unknowns
x and w. Accordingly; we generate another equation with the help of differential
calculus. Since the method of analysis is upper bound, we have to consider the
minimum value of w satisfying Eq.12.15. This can be obtained by differentiating
w of Eq.12.15 with respect to x and putting dw/dx = 0. Hence, we have:

dw/dx = (2MA / L) (-1 /x2) + (2 MB /L){1/(L-x)2}+ 2 MC [(1/L-x) (-1/x2) + (1/x){1/(L-


x)2}] = 0
(12.16)

After arranging the coefficients of x2, x and constant terms, we have:

Version 2 CE IIT, Kharagpur


(MB – MA) x2 + 2 (MA + MC) Lx – (MA + MC) L2 = 0
(12.10)

This is the same equation (Eq.12.10), as we obtained in the method of segmental


equilibrium.

In the method of virtual work, therefore, we get the values of x from the same
equation, Eq.12.10, and then we get w from Eq.12.15.

We consider two special cases of simply supported and clamped slabs from the
above equations.

Case (i): Simply Supported Slab

Here, we have MA = MB = 0. We get the values of x and w from Eqs.12.10 and


12.7, respectively, by the method of segmental equilibrium and from Eqs.12.10
and 12.15, respectively, by the method of virtual work.

Equation 12.10 becomes 2 MC Lx – MC L2 = 0, when MA = MB = 0, which gives x


= L/2.

However, Eq. 12.7 gives division by zero for the simply supported case, when MA
= MB = 0 and x = L/2. So, we use Eq.12.8 ( ∑M = 0) when MA = MB = 0 and x =
L/2. This gives w = 8MC / L2.

In the method of virtual work, Eq.12.10 (the same as in the method of segmental
equilibrium) gives x = L/2. Thereafter, Eq.12.15 is used for determining w, which
gives (when MA = MB = 0 and x = L/2),

w = 8MC / L2

Thus, we get the following values of x and w for the simply supported slab

x = L/2 and w = 8MC / L2


(12.17)

Case (ii) Clamped Slab with MA = MB = 2MC

Here, we get x = L/2 from Eq.12.10. Then, we use Eq.12.8 as Eq.12.7 involves
zero by zero case, as explained in case (i) above. This gives w = 24 MC / L2
(when MA = MB = 2MC and x = L/2). These values are by the method of
segmental equilibrium.

Similarly, by the method of virtual work, Eq.12.10 gives x = L/2. Then, Eq.12.15
gives, w = 24 MC / L2.

Version 2 CE IIT, Kharagpur


Thus, we get for the clamped slab, when MA = MB = 2 MC,
B

x = L/2 and w = 24 MC /L2


(12.18)

It is clear from the two cases that there will be positive yield line at the centre of
the slab when it is simply supported and there will be two negative yield lines at
the two supports in addition to the positive yield line at the centre of the slab
when the slab is clamped at both ends. The collapse loads are: w = 8 MC / L2 and
24 MC / L2 for the simply supported and clamped slabs, respectively. However, it
is worth mentioning that the formation of those specific yield lines and the
respective collapse loads are possible only if the slab is designed with adequate
positive and negative reinforcement, as assumed to get the solution.

In the practical cases of continuous one-way slabs, the negative moments


over the supports may be anywhere in the range of wl2/8 and wl2/12. Moreover, it
is not essential that the negative moment of resistances at the two ends should
be equal. Depending on the values of MA, MB and MC as per the reinforcement
provided, the values of x and w shall be determined using the respective
equations employing either the method of segmental analysis or the method of
virtual work.

We now take up numerical problems in the next section for the purpose of
illustration.

12.30.9 Numerical Problems

Version 2 CE IIT, Kharagpur


Problem 1. The factored moment capacities of the one-way continuous
reinforce concrete slab of Figs. 12.30.7 are MA = 32 kNm, MB = 36
kNm and MC = 30 kNm. The span of the slab is 5 m. Determine the
location of plastic hinges and the collapse load employing methods
of segmental equilibrium and virtual work.

Solution 1:
(A) Method of segmental equilibrium

We have two equations (Eqs.12.10 and 12.7) to determine the


location of positive yield line (distance x) and the collapse load
(Figs.12.30.6a to d).

Version 2 CE IIT, Kharagpur


(MB – MA) x2 + 2 (MA + MC) Lx – (MA + MC) L2=0
(12.10)

Using the given values of MA = 32 kNm, MB = 36 kNm and MC = 30


kNm in Eq.12.10, we have
4 x2 + 2(62) (5) x – (62) (25) = 0

or, x2 + 155 x – 387.5 = 0 gives x = 2.4609 m.

w = - 2 (MA – MB) / L (L-2x) (12.7)


Using the values of MA = 32 kNm, MB = 36 kNm and x = 2.4609 m
in Eq.12.7, we have, w = 20.4604 kN/m.

Therefore, the locations of the positive yield line is at a distance of


2.4609 m from the left support at A, the negative yield lines are at
the supports A and B, and the collapse load = 20.4604 kN/m.

(B) Method of virtual work

In this method, we have Eqs.12.10 and 12.15 to determine x and w


(Figs.12.30.6a to d). In the method of segmental equilibrium (A)
above, Eq.12.10 gives: x = 2.4609 m.

w = 2 MA/Lx + 2 MB / L(L-x) + 2 MC /x (L-x)


(12.15)

Using MA = 32 kNm, MB = 36 kNm, MC = 30 kNm and x = 2.4609 m,


in Eq.12.15 above, we get w = 20.4749 kN/m. Thus, we get the
same location of positive yield line i.e., at x = 2.4609 m from the left
support A, negative yield lines are at the support A and B and the
collapse load w = 20.4749 kN/m .

The marginal difference of the two collapse loads is due to the


truncation error in the calculation. Otherwise, both the values
should be the same. The bending moment and shear force
diagrams are shown in Figs.12.30.7b and c, respectively.

12.30.10 Practice Questions and Problems with Answers

Q.1: Name five limitations of elastic analysis of reinforced concrete slabs.

Version 2 CE IIT, Kharagpur


A.1: Second paragraph of sec.12.30.1

Q.2: What are yield lines and what is yield line theory?

A.2: First paragraph of sec.12.30.2

Q.3: Explain the basic principles of yield line theory.

A.3: Sec.12.30.3

Q.4: State the assumptions of yield line theory.

A.4: Sec.12.30.4

Q.5: What are the guidelines to draw the possible yield patterns and locate the
axes of rotations?

A.5: The last paragraph of sec.12.30.5

Q.6: State upper and lower bound theorems.

A.6: Sec.12.30.6

Q.7: Explain the two methods of yield line analysis.

A.7: Sec.12.30.7

Q.8: Compare the locations of positive and negative yield lines and the values
of the respective collapse loads of one 3 m × 6 m slab supported along 6
m direction (Figs.12.30.6a) carrying a total factored load of 20 kN/m for
the three cases: (i) the slab is simply supported, (ii) the slab is clamped
and (iii) the positive and negative reinforcements are identical to have the
equal resistance for the continuous slab. Discuss the results.

A.8: The moments MC at the mid-span for one simply supported slab is wl2/8 =
90 kNm and for another clamped supported slab MC = wl2 /24= 30 kNm.
The support moments for clamped slab MA = MB = -wl2/12 = - 60 kNm. For
the third case the magnitude of MA, MB and MC are equal for a continuous
slab. So, MA = MB = - 45 kNm and MC = + 45 kNm. Method of virtual work
is employed.

Version 2 CE IIT, Kharagpur


Case (i): From Eq. 12.17, we have the distance of positive yield line x =
6/2 = 3 m and w = 8 MC /L2 = 8 (90) / 6(6) = 20 kN/m. There are no
negative yield lines as the slab is simply supported.

Case (ii): From Eq. 12.18, we have the distance of positive yield line x =
6/2 = 3 m and w = 24 MC /L2 = 24 (30) / 6(6) = 20 kN/m. The negative yield
lines of the clamped slab are at the two sides AA and BB.

Case (iii): Employing method of virtual work

(MB – MA)x2 + 2(MA + MC)Lx – (MA + MC)L2 = 0


(12.10)

w = 2MA / Lx + 2MB/L (L-x) + 2MC / x (L-x)


(12.15)

Using MA = MB = MC = 45 kNm in Eq.12.10, we have, 2x – L = 0 or x = L/2 = 6/2


= 3 m.

Using x = 3 m and MA = MB = MC = 45 kNm in Eq. 12.15, we have w =


90{1/6(3) + 1/6(3) + 1/3(3)} = 20 kN/m. The negative yield lines are over the
supports of continuous slab i.e., along AA and BB.

Discussion of results: All the three slabs have x =3 m and w = 20 kN/m.


However, the third case is the most economic as the depth of the slab may be
provided for M = 45 kNm and the areas of positive and negative steel are the
same. The simply supported slab has no negative moment and the clamped slab
has large difference between the positive and negative moments and
accordingly, wide variation is seen in the magnitudes of positive and negative
steel.

12.30.11 References

1. Reinforced Concrete Limit State Design, 6th Edition, by Ashok K. Jain,


Nem Chand & Bros, Roorkee, 2002.
2. Limit State Design of Reinforced Concrete, 2nd Edition, by P.C. Varghese,
Prentice-Hall of India Pvt. Ltd., New Delhi, 2002.
3. Advanced Reinforced Concrete Design, by P.C. Varghese, Prentice-Hall
of India Pvt. Ltd., New Delhi, 2001.
4. Reinforced Concrete Design, 2nd Edition, by S. Unnikrishna Pillai and
Devdas Menon, Tata McGraw-Hill Publishing Company Limited, New
Delhi, 2003.

Version 2 CE IIT, Kharagpur


5. Limit State Design of Reinforced Concrete Structures, by P. Dayaratnam,
Oxford & I.B.H. Publishing Company Pvt. Ltd., New Delhi, 2004.
6. Reinforced Concrete Design, 1st Revised Edition, by S.N. Sinha, Tata
McGraw-Hill Publishing Company. New Delhi, 1990.
7. Reinforced Concrete, 6th Edition, by S.K. Mallick and A.P. Gupta, Oxford &
IBH Publishing Co. Pvt. Ltd. New Delhi, 1996.
8. Behaviour, Analysis & Design of Reinforced Concrete Structural Elements,
by I.C.Syal and R.K.Ummat, A.H.Wheeler & Co. Ltd., Allahabad, 1989.
9. Reinforced Concrete Structures, 3rd Edition, by I.C.Syal and A.K.Goel,
A.H.Wheeler & Co. Ltd., Allahabad, 1992.
10. Textbook of R.C.C, by G.S.Birdie and J.S.Birdie, Wiley Eastern Limited,
New Delhi, 1993.
11. Design of Concrete Structures, 13th Edition, by Arthur H. Nilson, David
Darwin and Charles W. Dolan, Tata McGraw-Hill Publishing Company
Limited, New Delhi, 2004.
12. Concrete Technology, by A.M.Neville and J.J.Brooks, ELBS with
Longman, 1994.
13. Properties of Concrete, 4th Edition, 1st Indian reprint, by A.M.Neville,
Longman, 2000.
14. Reinforced Concrete Designer’s Handbook, 10th Edition, by C.E.Reynolds
and J.C. Steedman, E & FN SPON, London, 1997.
15. Indian Standard Plain and Reinforced Concrete – Code of Practice (4th
Revision), IS 456: 2000, BIS, New Delhi.
16. Design Aids for Reinforced Concrete to IS: 456 – 1978, BIS, New Delhi.

12.30.12 Test 30 with Solutions

Maximum Marks = 50 Maximum Time = 30


minutes
Answer all questions.

TQ.1: Explain the basic principles of yield line theory.

A.TQ.1: Sec.12.30.3
[10 Marks]
TQ.2: State the assumptions of yield line theory.

A.TQ.2: Sec.12.30.4
[10 Marks]
TQ.3: What are the guidelines to draw the possible yield patterns and locate the
axes of rotations?

A.TQ.3: The last paragraph of sec.12.30.5

Version 2 CE IIT, Kharagpur


[10 Marks]
TQ.4: Determine the location of plastic hinges and collapse load of the one-way
slab supported at 4 m c/c (Fig.12.30.6a) having the positive and negative
reinforcements to have the factored moments of resistance MA = 25 kNm,
MB = 35 kNm and MC = 30 kNm. Use method of virtual work.
[20 Marks]

A.TQ.4: (MB – MA)x2 + 2(MA + MC)Lx – (MA + MC)L2 = 0


(12.10)

w = 2MA / Lx + 2MB/L (L-x) + 2MC / x (L-x)


(12.15)

Using MA = 25 kNm, MB = 35 kNm and MC = 30 kNm in Eq. 12.10, we


have, x = 1.9165 m and using the values of MA, MB, MC and x in Eq. 12.15,
we get w = 29.9478 kN/m2.

12.30.13 Summary of this Lesson

This lesson explains the basic principle of yield line analysis which is
required to remove the inconsistency between the elastic analysis and the design
by limit state method considering inelastic behaviour. Moreover, the limitations of
elastic analysis of slab are mentioned. The upper and lower bound theorems are
explained to show that the two methods of yield line analysis viz. (i) method of
segmental equilibrium and (ii) method of virtual work are upper bound methods.
The governing equations of both the methods are derived. The first requirement
of the yield line analysis is to predict the possible yield pattern and locate the
axes of rotation. Suitable guidelines are given for the same as the correctness of
the solution depends on the prediction of true yield line pattern and location of
the axes of rotation. Numerical problems are solved by both methods of analysis.

Version 2 CE IIT, Kharagpur


Module
12
Yield Line Analysis for
Slabs
Version 2 CE IIT, Kharagpur
Lesson
31
Nodal Forces and Two-way
Slabs
Version 2 CE IIT, Kharagpur
Instructional Objectives:
At the end of this lesson, the student should be able to:

• derive the expression for determining the work done by bending and
twisting moments when the yield lines are at angles with the directions of
reinforcing bars,

• state the need for considering the nodal forces and to estimate their values
when one yield line meets another yield line or a free edge,

• to select the possible yield pattern of a two-way slab supported at four sides
either by simple supports or fixed supports,

• to finalise the yield patterns and to evaluate the collapse loads of two-way
slabs, either simply supported or clamped at four sides,

• apply the theory in solving numerical problems of slabs to finalise the yield
pattern and to determine the collapse load employing (i) the method of
segmental equilibrium and (ii) the method of virtual work.

12.31.1 Introduction
Lesson 30 introduces the yield line analysis, which is an upper bound
method of analysis for slabs. The different rules for predicting the yield lines are
stated. The two methods i.e., (i) method of segmental equilibrium and (ii) method
of virtual work are explained. Applications of both the methods are illustrated
through numerical problems of one-way slabs – either simply supported or
continuous.

This lesson presents the derivations of the expressions for determining


bending and torsional moments when yield lines are at angles with the directions
of reinforcement. The need for the nodal forces and their determinations are
explained when yield line meets another yield line or the free edge. Thereafter,
different possible yield patterns of two-way slabs are explained. Numerical
illustrative problems of two-way slabs with or without nodal forces are illustrated.

Version 2 CE IIT, Kharagpur


12.31.2 Work Done by Yield Line Moments

Normally, the reinforcing bars are placed in two mutually perpendicular


directions parallel to the sides of rectangular and square slabs. However, the
yield lines may be at an angle with the direction of reinforcing bars as shown in
Fig. 12.31.1, in which the yield line AB of length L has bending moment Mb and
twisting moment Mt per unit length of the yield line. The slab segment is
undergoing rigid body rotation whose components are θx and θy. The horizontal
and vertical projections of the yield line are having moment capacities of Mx and
My per unit length, respectively. All moments and rotations are shown using the
right hand thumb rule. The following expression is derived for obtaining the
absolute values of the work done by Mb and Mt on the yield line AB.

With reference to Fig. 12.31.1, the total work done by the bending and
twisting moments Mb and Mt is,

W = Mb L (θx cos θ + θy sin θ) + Mt L (- θx sin θ + θy cos θ)


(12.19)

Version 2 CE IIT, Kharagpur


Equating the work done by bending and twisting moments Mb and Mt along the
yield line AB with the respective work done by their components along the
projections of the yield line, we have the following two equations:

Mb L = Mx L sin θ sin θ + My L cosθ cosθ , which gives

Mb = Mx sin2θ + My cos2θ
(12.20)

and
Mt L = Mx sin θ cos θ - My L cos θ sin θ , which gives

Mt = (Mx – My) sinθ cosθ


(12.21)

Substituting the expressions of Mb and Mt from Eqs.12.20 and 12.21 in Eq.12.19,


we have, W = (Mx sin2θ + My cos2θ) (θx L cosθ + θy L sinθ) + (Mx – My) sinθ cosθ
(-θx L sinθ + θy L cosθ), which gives

W = Mx L sinθ θy + My L cosθ θx
or
W = Mx Ly θy + My Lx θx
(12.22)

The two terms on the right hand side of Eq. 12.22 give the external work done by
the moments MxLy and MyLx acting on the horizontal and vertical projections of
the yield line.

It is thus seen that the expression of Eq.12.19 involving bending and twisting
moments Mb and Mt may be replaced by Mx and My of Eq.12.22 to get the same
work done by Eq. 12.22 as that of Eq. 12.19.

12.31.3 Special Conditions at Edges and Corners

Version 2 CE IIT, Kharagpur


Figures 12.30.5d, e and f of Lesson 30 present yield patterns of slabs
when positive yield lines intersect the free edges at angles. In actual case,
however, both bending and twisting moments are zero at the free or simply
supported edges. The directions of the principal stresses are, therefore, parallel
and perpendicular to the respective edge. Accordingly, the yield lines should
enter the edge perpendicular to it, which is confirmed by experimental tests also.
In Fig. 12.31.2a, it is shown that the yield line ADC normally turns quite close to
the edge, say at D, which is at a distance s. The yield line is approximated by
extending it in a straight line AB to the edge introducing a pair of concentrated
shear force, (+) V and (-) V, as shown in Fig. 12.31.2b. These shear forces,
which are parallel, equal and opposite forces for the equilibrium, are designated
as nodal forces acting upward at an obtuse corner, marked by (-)ve sign on the
left of the yield line AB and acting downward at an acute corner, marked by (+)ve
sign on the right of the yield line AB, as shown in Fig. 12.31.2b.

They are, in fact, the static equivalent of twisting moments and shear
forces near the edge. We now establish the required expression for determining
the magnitude of these nodal forces V.

Figure 12.31.3a shows an element ACB of a slab where the yield line AB
is making an angle α with the free edge CB. The angle between the yield line AB
and the element side AC (i.e., angle CAB) is dα. The bending and twisting
moments Mb and Mt on the yield lines AB and AC are shown in Fig. 12.31.3a,
neglecting their differential increments. The free body diagram is shown in Fig.
12.31.3b. The distances BD, AD and BC are Lx, Ly and dLx, respectively. Taking
moments about the line AC and equating it to zero, we get My (Lx +dLx) cos (α -

Version 2 CE IIT, Kharagpur


dα) – My Lx cos (α - dα) + Mx Ly sin (α - dα) Mx Ly sin (α - dα) – V dLx sin (α - dα)
=0
or V sin (α - dα) = My cos (α - dα)

or V (sin α cos dα - cos α sin dα) = My (cos α cos dα + sin α sin dα)

For small values of dα, sin dα = 0 and cos dα = 1. So, we have V sin α = My cos
α

or V = My cot α
(12.23)

Equation 12.23 gives the magnitude of the nodal force V where α is


measured anticlockwise. When α = 90 degree, V = 0, i.e., the nodal force is zero
if the yield line intersects the free edge at an angle of 90 degree.

The nodal force V shall be used in the method of segmental equilibrium as


we consider the equilibrium of individual segment. In the method of virtual work,
however, the work done is determined for the entire slab involving all the
elements, when the total work done by the positive and negative nodal forces is
zero. Hence, it is not needed to consider the nodal force in the method of virtual
work.

Version 2 CE IIT, Kharagpur


Version 2 CE IIT, Kharagpur
Version 2 CE IIT, Kharagpur
Figures 12.31.4a and b present two typical yield patterns of two-way slabs
clamped at four sides and Fig. 12.31.4c and d show two typical yield patterns of
a slab simply supported on three sides and free at one side. In all the yield
patterns, it is assumed that the yield lines enter the corners between the two
intersecting sides. It may not be the case always. Sometimes, yield lines fork
before they reach the corner as shown in Fig. 12.31.4e and thus form corner
lever. The triangular element EFB in Fig. 12.31.4e will pivot about the axis EF
and lift off the supports if the corner is not held down. It has been observed that
such yield patterns with corner levers are more critical than those without them.
However, these patterns are generally neglected. It should be mentioned that the
introduction of corner levers makes the analysis more complicated and does not
produce much error by neglecting them. This is illustrated in Lesson 32.

12.31.4 Two-way Slabs of Yield Pattern 1 of Figure


12.31.4a

Version 2 CE IIT, Kharagpur


Version 2 CE IIT, Kharagpur
We consider the two-way slab of Fig. 12.31.4a subjected to uniformly
distributed collapse load of intensity w kN/m2. Yield pattern 1 divides the slab into
four segments marked by 1, 2, 3 and 4 in Fig. 12.31.5a, where the positive and
negative yield lines are shown along with the positive and negative nodal forces.
The slab undergoes a displacement Δ at the center point O. The free body
diagrams of segments 1 and 2 are shown in Figs. 12.31.5b and c, respectively.
The positive and negative moments along x and y directions are designated by
Mpx, Mpy, Mnx and Mny in these two figures. We are employing both (A) the
method of segmental equilibrium and (B) the method of virtual work to determine
the magnitude of the collapse load w of the slab.

(A) Method of segmental equilibrium

At the equilibrium, the moment of all the forces and moments of segment
1 about the left edge AB is zero. This gives (Fig. 12.31.5b): (1/2) (Lx /2) (Ly)
w(Lx/6) – V(Lx/2) – Mnx Ly – Mpx Ly= 0

or wLxLy – 12 V = 24 (Mpx + Mnx) (Ly / Lx)


(12.24)

Similarly, at the equilibrium, the moment of all forces and moments of


segment 2 about the bottom edge BC is zero. This gives (Fig. 12.31.5c): (1/2) Lx
(Ly/2) w (Ly/6) + V (Ly/2) – Mny Lx – Mpy Lx = 0

or, wLxLy + 12 V = 24 (Mpy + Mny) (Lx / Ly)


(12.25)

Eliminating V from Eqs. 12.24 and 12.25 by adding the two equations, we have

2w Lx Ly = 24 {(Mpx + Mnx)(Ly / Lx)+ (Mpy + Mny) (Lx / Ly)}

or w = 12 {(Mpx + Mnx) / Lx2 + (Mpy + Mny) / Ly2}


(12.26)

The collapse load w is determined from Eq. 12.26 from known values of Mpx, Mnx
Mpy and Mny.

(B) Method of virtual work

The total external work done by the load in causing the segments to
undergo deflection and the total internal work done by the moments in rotating all
the four segments are computed. As mentioned in sec. 12.31.3, the effect of all
the nodal forces is zero. Accordingly, the total external work done (TEW) is,

TEW = 4 (1/2) Lx (Ly/2) w (Δ/3) = wLx Ly (Δ/3)


(12.27)

Version 2 CE IIT, Kharagpur


The total internal work done (TIW) is,

TIW = 2 (Mpx + Mnx) Ly θx + 2 (Mpy + Mny) Lx θy


= 4Δ {(Mpx + Mnx) (Ly /Lx) + (Mpy + Mny) (Lx / Ly)}
(12.28)

Equating the two works from Eqs. 12.27 and 12.28

w Lx Ly (Δ/3) = 4Δ{(Mpx + Mnx) (Ly /Lx) + (Mpy + Mny) (Lx/Ly)}

or w = 12 {(Mpx + Mnx) / Lx2 + (Mpy+Mny) / Ly2}


(12.26)

The above equation is the same as obtained by the method of segmental


equilibrium to get the value of the collapse load w.

We now consider the two cases below, first the square and simply supported at
all edges and secondly the square and clamped at all edges.

Case (i) Square and simply supported slab

For square and simply supported slab, Lx = Ly = L, Mnx = Mny = 0 and let us
assume Mpx = Mpy = Mp. Using the conditions mentioned above in Eq. 12.26, we
get

w = 24 Mp / L2
(12.29)

or, Mp = w L2/24
(12.30)

Case (ii) Square and clamped slab

For square and clamped slab, let us assume Mpx =Mpy = Mnx = Mny = Mp. Using
these conditions in Eq. 12.26, we get

w = 48 Mp / L2
(12.31)

or Mp = wL2/48
(12.32)

The values of w and Mp in the two cases above reveal that the factored load
intensity of a clamped slab is twice of that of simply supported slab having the
same moment carrying capacity Mp. Further, we observe that for the same
factored load intensity w, a clamped slab would have half the factored moment of
simply supported slab and, therefore, would be economic.
Version 2 CE IIT, Kharagpur
12.31.5 Two-way Slabs of Yield Pattern 2 of Figure
12.31.4b

Version 2 CE IIT, Kharagpur


We now consider the two-way slab having the yield pattern 2 as shown in
Fig. 12.31.4b and subjected to uniformly distributed collapse load of intensity w
kN/m2. Yield pattern 2 divides the slab into four segments marked by 1, 2, 3 and
4 in Fig. 12.31.6a, where the positive and negative yield lines are shown. The
slab undergoes a displacement of Δ along the yield line EF. The free body
diagrams of segments 1 and 2 are shown in Figs. 12.31.6b and c, respectively.
The positive and negative moments along x and y directions are designated by
Mpx, Mpy, Mnx and Mny in these two figures. We are employing both (A) method of
segmental equilibrium and (B) method of virtual work to determine the distance x
and the magnitude of the collapse load w of the slab.

(A) Method of segmental equilibrium

At the equilibrium, the moment of all the forces and moments of segment 1 about
the left edge AB is zero. This gives (Fig. 12.31.6b): (Ly/2) x w (x/3) – (Mpx + Mnx)
=0

or w = 6 (Mpx + Mnx) / x2
(12.33)

Similarly, at the equilibrium, the moment of all forces and moments of segment 2
about the bottom edge BC is zero. This gives (Fig. 12.31.6c): (1/2) x (Ly/2) (Ly/6)
2w + w(Lx – 2x) (Ly/2) (Ly/4) – (Mpy + Mny)Lx = 0

or, w = 24 (Mpy + Mny) Lx /{2x Ly2 + 3Ly2 (Ly –2x)}


(12.34)

Equating the two expressions of w from Eqs. 12.33 and 12.34, we have

6( M px + M nx ) 24( M py + M ny ) Lx
2
= 2 2
x 2 xL y + 3L y ( Lx − 2 x)

or, 4 (Mpy + Mny) Lxx2 + 4 (Mpx + Mnx) Ly2 x – 3 (Mpx + Mnx) Lx Ly2 = 0
(12.35)

Equation 12.35 is used to determine the values of x when Mpx, Mpy, Mnx and Mny
are known. Thereafter, Eq. 12.34 is used to determine the magnitude of the
collapse load w.

(B) Method of virtual work

The total external work done by the load in causing deflection of the four
segments of Fig. 12.31.6a, TEW is:

Version 2 CE IIT, Kharagpur


TEW = 2W1 + 2 (W21 + W22 + W23), where W1 is the work done for the segment 1
and the work done of segment 2 is subdivided into three parts as W21, W22 and
W23. Noting that W21 = W23, we get the TEW as,

TEW = 2w{(1/2)x Ly (Δ/3)} + 2w {(1/2)x (Ly/2) (Δ/3) + 2w {(Lx-2x) (Ly/2) (Δ/2)}

= (wΔ/6) (3 Lx Ly – 2x Ly)
(12.36)

The total internal work done by the yield moments (TIW) is,

TIW = 2 (Mpx + Mnx) θx Ly + 2 (Mpy + Mny) θy Lx


= 2 (Mpx + Mnx) (Δ/x) Ly + 2 (Mpy + Mny) (2Δ/Ly) Lx
(12.37)

Equating the two works from Eqs. 12.36 and 12.37, we have,

(w Δ/6) (3Lx Ly - 2x Ly) = 2 (Mpx + Mnx) (Δ/x) + 2 (Mpy + Mny) (2Δ/Ly) Lx


2
12( M px + M nx ) L y + 24 ( M py + M ny ) xL x
or w = 2
L y (3 xL x − 2 x 2 )
(12.38)

To get the minimum collapse load, we put dw/dx = 0, which gives

Ly2 (3x Lx – 2x2) {24 Lx (Mpy + Mny)} – 12 [(Mpx + Mnx) Ly2 + 24 x Lx (Mpy + Mny)]
(3Lx – 4x) Ly2 = 0

or 4 (Mpy + Mny) Lx x2 + 4 (Mpx + Mnx) Ly2 x – 3 (Mpx + Mnx) LxLy2 = 0


(12.35)

The above equation is the same as obtained by the method of segmental


equilibrium to determine the values of x. Thereafter, Eq. 12.38 is used to get the
value of w, the collapse load.

We observe two points from secs. 12.31.4 and 12.31.5. They are as follows:

(i) Only one equation (Eq. 12.26) is needed for determining the value of the
collapse load w for the yield pattern 1 of Fig. 12.31.5a. On the other hand, two
equations are need for the yield pattern 2. This is because the yield pattern 1 is
already determined but the yield pattern 2 is determined only after finding the
values of x. Therefore, two equations are needed for determining the two
unknowns x and w in the case of yield pattern 2.

Version 2 CE IIT, Kharagpur


(ii) The one equation needed for the yield pattern 1 is the same equation (Eq.
12.26) by the two methods, viz. method of segmental equilibrium and method of
virtual work. Out of the two equations needed for the yield pattern 2, only one
equation (Eq. 12.35) is the same by both the methods. After getting the values of
x from Eq. 12.35, the values of collapse load w is determined from Eq. 12.33 by
the method of segmental equilibrium and Eq. 12.38 by the method of virtual work.

Let us now take up Eq. 12.35, which is used to determine the values of x in the
case of yield pattern 2. Substituting the value of x = Lx/2 in Eq. 12.35, we get

4 (Mpy + Mny) (Lx3 )(1/4) + 4 (Mpx + Mnx) Ly2 (Lx/2) – 3 (Mpx + Mnx) Lx Ly2 = 0

M px + M nx Lx 2
or, =
M py + M ny Ly 2
(12.39)

Noting that, x = L/2 gives the yield pattern 1 of Fig.12.31.5a, Eq.12.39 is used to
check if the slab has the yield pattern 1 from the known values of Mpx, Mnx, Mpy,
Mny, Lx and Ly of the slab.

Similarly, it can be shown that for the yield pattern 2, i.e., when x < Lx/2, the
required condition is:

2
M px + M nx ⎛L ⎞
<⎜ x ⎟
M py + M ny ⎜L ⎟
⎝ y ⎠
(12.40)

We now take up numerical problems in the next section for the purpose of
illustration.

12.31.6 Numerical Problems

Version 2 CE IIT, Kharagpur


Problem 1. Determine the yield pattern and collapse load of the trapezoidal slab
of Fig.12.31.7a having clamped edges along BA, AF and FE while the
edge BE is free. Given that Mn = Mp = 70 kNm/m. Employ the method
of segmental equilibrium.

Solution 1. Here, in this problem the yield pattern drawn in Fig.12.31.7a shows
that there is one additional unknown x to finalise the yield pattern.
Thus, we have two unknowns x and the collapse load w.

The yield pattern divides the slab into three segments marked by 1, 2
and 3. Yield lines AC and FD meet the free edge BE. So, we have to

Version 2 CE IIT, Kharagpur


consider the nodal forces (+)V and (-)V, as shown on the left and right
of C. Since the problem is symmetrical, segments 1 and 3 are
identical. We first consider the equilibrium of segment 1 having
positive moment Mp along yield line AC and negative moment Mn
along yield line AB (Fig. 12.31.7b). The nodal force V at C of segment
1 is negative, i.e., acting downward.

Equation 12.23 gives the magnitude of the nodal force V = Mn cot α,


where α is the angle ACB.

Geometric properties:

In triangle ABK, the side AB = (BK2 + AK2)1/2 = 6 m. The angle ABC =


600. Assuming the angle BAK = θ, we have sinθ = BK/BA = 0.5 giving
θ = 300. The line CJ is perpendicular to AB. The angle JCB = 300
gives BJ = BC cos 60 = 0.5x. Therefore, AJ = 6 - 0.5x. The distance
CJ = BC sin 60 =31/2 (x/2). From triangle ACK, we have cotα = CK/AK
= (x-3)/3(3)1/2. Area of the triangle ABC = (1/2) (BC) (AK) = (x/2)
3(3)1/2. The load of segment 1 is acting at a distance of CJ/3 from the
side AB, which is equal to 05x/(3)1/2. Taking moments of load of the
segment 1, nodal force V and considering Mn and Mp about the edge
AB, we have:

Mp (AJ) + Mn (AB) – V (CJ) – w (area of segment ABC) (CJ/3) = 0

Substituting the values of Mp, Mn, V, AJ, AB, CJ and the area of
triangle ABC, we have

70 (6 –0.5x) + 70 (6) – {70 (x – 3) / 3 (3)1/2} (3)1/2 (x/2) – (x/2) 3(3)1/2 w


(3)1/2 (x/2) / 3 = 0

or w = 4(2520 – 35x2) / 9x2 (i)

Now, we take up segment 2 for writing the equilibrium equation. The


segment is subdivided into three parts marked by 21, 22 and 23. The sub-parts
21 and 23 are identical. The area of the triangle ACH = (1/2) (x - 3) 3 (3)1/2 and
the load is considered at a distance of CH/3 = (3)1/2 m from the edge AF. The
area of rectangle CDGH = (12 - 2x) 3(3)1/2 and the load is considered at a
distance of (3/2) (3)1/2 from the edge AF. The equilibrium equation of segment 2
is obtained by taking moments of loads on this segment, two nodal forces of (+)
V at C and D about the edge AF and considering Mp along AC and FD and Mn
along AF (Fig.12.31.7c). This gives 2Mp (AH) + Mn(AF) + 2V (CH) – 2w(area of
sub segment AHC) (HC/3) – w (area of rectangle CDGH) (HC/2) = 0. Substituting
the values of Mp, Mn, V, AH, AF, CH and the areas of triangle AHC and rectangle
CDGH, we have,

Version 2 CE IIT, Kharagpur


2 (70) (x – 3) + 70 (6) + 2{(70) (x-3) / 3 (3)1/2} 3(3)1/2 – 2 w (1/2) (x –3) (3) (3)1/2
(3)1/2 + w [{6 – 2(x – 3)} 3 (3)1/2 (3/2) (3)1/2 ] = 0

This gives

w = (280 x – 420) / (135 – 18 x) (ii)

Equating the two expressions of w from Eqs. (i) and (ii), we have:

4 (2520 – 35x2) / 9x2 = (280x – 420) / (135 – 18x)

This gives

x2 + 12x – 90 = 0 (iii)

Solving we get x = 5.2249 m

From Eq. (i) w = 4 (2520 – 35x2) / 9x2 = 25.465 kN/m2


From Eq. (ii) w = (280x – 420) / (135 – 18x) = 25.4695 kN/m2

So, we get the same value of w from Eqs. (i) and (ii). Thus, the yield pattern is
finalised when x = 5.2249 m and the collapse load of the slab is 25.4695 kN/m2.

Problem 2. Determine the yield pattern and the collapse load of the two-way slab
of Fig.12.31.8 which is having clamped edges along four sides.
Assume that Mpx = Mnx = 50 kNm/m and Mpy = Mny = 70 kNm/m.

Version 2 CE IIT, Kharagpur


Solution 2. In this problem, we have to find out whether yield pattern 1 or 2 will
be the governing from Eqs.12.39 and 12.40 of sec.12.31.5.

From the given data, we have:

(Mpx + Mnx) / (Mpy + Mny) = 100/140 = 0.714 and Lx2 /Ly2 = 64/36 = 1.77.
Therefore, yield pattern 2 will govern (see Eq. 12.40).

Thus, we have two unknowns: (i) the value of x for finalising the yield
pattern and (ii) the value of the collapse load w. Since, the governing
equations of yield pattern 2 are derived in sec.12.31.5, the problem is
solved by direct application of the equations.

We have Eq.12.35 to determine the value of x. Using the values of


Mpx, Mnx, Mpy, Mny, Lx and Ly in Eq. 12.35, we have:

4 (Mpy + Mny) Lx x2 + 4 (Mpx + Mnx) Ly2 x – 3 (Mpx + Mnx) Lx Ly2 = 0


(12.35)

or 4 (140) (8) x2 + 4 (100) (36) x – 3 (100) (8) (36) = 0


or 14 x2 + 45 x – 270 = 0, which gives x = 3.069 m

Now, we use the three equations (Eqs.12.33 and 12.34 by the method
of segmental equilibrium and Eq. 12.38 by the method of virtual work).
Using the values of Mpx, Mpy, Mnx, Mny, Lx, Ly and x in those three
equations, we determine the value of w for comparing them.

(i) Eq.12.33 (i.e., w = 6 (Mpx + Mnx) x2 ) gives w = 63.69 kN/m2

24( M py + M ny ) Lx
(ii) Eq. 12.34 (i.e., w = 2 2
) gives w = 63.69 kN/m2
2 xL y + 3L y ( L x − 2 x)
2
12( M px + M nx ) L y + 24( M py + M ny ) xL x
(iii) Eq.12.38 (i.e., w= 2
) gives
L y (3 xL x − 2 x 2 )
w = 63.69 kN/m2.

Thus, we observe that the collapse load is the same from the three
equations. Usually, they may differ marginally depending on the truncation of the
value of x.

12.31.7 Practice Questions and Problems with Answers.


Q.1: Establish the work done by the yield line moments Mb and Mt when the
yield line is at an angle with the two orthogonal directions of the
reinforcement.

Version 2 CE IIT, Kharagpur


A.1: See sec. 12.31.2

Q.2: Explain the nodal force and derive the expression to determine its value.

A.2: See sec. 12.31.3

Q.3: Draw the possible two yield patterns of a two-way slab clamped at four
sides. Derive the equation to find out which one will govern in a particular
case given the values of Mpx, Mnx, Mpy, Mny, Lx and Ly.

A.3: Figures 12.31.4a and b are the two possible yield patterns. For finding the
governing yield pattern, the equations (Eqs. 12.39 and 12.40) are
established in sec. 12.31.5.

Version 2 CE IIT, Kharagpur


Q.4: (a) Establish the general equations for determining the yield pattern and
uniformly distributed collapse load of an isosceles triangular slab, shown
in Fig. 12.31.9 having the positive and negative moment capacity of Mp
and Mn, respectively. Use the method of virtual work.

Version 2 CE IIT, Kharagpur


(b) Determine the specific yield pattern and uniformly distributed collapse
load of such a slab when B = 6 m, 2L = 12 m, Mp = 9 kNm/m and Mn =
12 kNm/m, as shown in Fig.12.31.10. Use method of virtual work.

A.4: (a): The possible yield pattern is drawn in Fig.12.31.9 having unknown y.
Thus, we have two unknowns y and the collapse load w of the slab. The
yield pattern divides the slab into three segments of which segments 2
and 3 are symmetrical.

Geometric properties.

The angle DEJ = θ = tan-1 (L/B). The perpendicular distance from G to ED


is GH = EG sinθ = (B – y) {sin tan-1 (L/B)}. The length of side DE = (B2 +
L2)1/2.

Let us assume the displacement of the slab at point G = Δ. The rotation of


the segment 1 = θ1 = Δ/y and the rotation of the segments 2 and 3 = θ2 =
Δ/GH = Δ / [(B-y){sin tan-1 (L/B)}].

The total external work (TEW) done by the loads of three segments is
obtained considering the displacement of the centroid of all three
segments as Δ/3.
Thus,
TEW = (1/2) (2L) (B) w (Δ/3) = B L w Δ/3
(1)

The internal work done by the negative yield lines DF, DE and EF is:
Mn {(DF) (θ1) + 2(DE) (θ2)}
(2A)

For the positive yield moment along DG, let us project DG along DE and
DF. The projected lengths are DH and DJ, respectively. Similarly,
projecting the moments of positive yield lines of EG and FG along the
sides of the triangle DEF, we have the total internal works done by the
three positive yield lines = Mp {(DF) (θ1) + 2(DE)(θ2)}
(2B)

Therefore, the total internal work done (TEW) is obtained adding the two
expressions of 2A and 2B as:

TIW = (Mp + Mn) {(DF) (θ1) + 2 (DE) (θ2)}


(2)

Equating TEW and TIW from Eqs. 1 and 2, we have

Version 2 CE IIT, Kharagpur


(B)(L) w Δ/3 = (Mp + Mn) {(DF) (θ1) + 2 (DE) (θ2)}
(3)

Substituting the values of DF, DE, θ1 and θ2, we have from Eq. 3

(B)(L) w Δ/3 = (Mp + Mn) [(2L) (Δ/y) + 2 (B2 + L2)1/2 Δ / [(B-y) {sin tan-1
(L/B)}]]

3( M p + M n ) ⎡ 2 L 2( B 2 + L2 )1 / 2 ⎤
or, w = ⎢ + −1 ⎥
( B)( L) ⎣ y ( B − y ){sin tan ( L / B)} ⎦
(4)

Equation 4 is the only one equation from the method of virtual work to
determine y and w. Therefore, we differentiate w with respect to y to get
the lowest value of the load and determine y. Thereafter, Eq. 4 shall be
used to find the value of w.

dw 2L − 2( B 2 + L2 )1 / 2
= 0 gives : − 2 =
dy y ( B − y ){sin tan -1 ( L / B)}

or [L{sin tan-1 (L/B)} – (B2 + L2)1/2] y2 – [2 (L) (B) {sin tan-1 (L/B)}] y + LB2
{sin tan-1 (L/B)} = 0
(5)

Thus, Eqs. 4 and 5 are the general equations to determine y and w of the
slab.

A 4. (b): For the specific case when 2L = 12 m, B = 6 m, Mp = 9 kNm/m and Mn


= 12 kNm/m, we have angle DEJ = θ = 450. The distance GH = EG sinθ =
(6-y) / (2)1/2. The length of side DE = EF = (36 + 36)1/2 = 6(2)1/2 m. The
rotations θ1 = Δ/y and θ2 = Δ/[(B-y) {sin tan-1 (L/B)}] = Δ(2)1/2 / (6-y).

Equating the TEW and TIW from Eqs. 1 and 2 of A.4a, we have:

( M p + M n ) ⎧12 24 ⎫
w= ⎨ + ⎬
12 ⎩ y 6− y⎭
(6)

dw
= 0 gives : y2 + 12y – 36 = 0
dy
(7)

Version 2 CE IIT, Kharagpur


Solution of Eq. 7 gives y = 2.485 m. Using the value of y in Eq. 6, we get w
= 20.399 kN/m2.

12.31.8 References
1. Reinforced Concrete Limit State Design, 6th Edition, by Ashok K. Jain,
Nem Chand & Bros, Roorkee, 2002.
2. Limit State Design of Reinforced Concrete, 2nd Edition, by P.C. Varghese,
Prentice-Hall of India Pvt. Ltd., New Delhi, 2002.
3. Advanced Reinforced Concrete Design, by P.C. Varghese, Prentice-Hall
of India Pvt. Ltd., New Delhi, 2001.
4. Reinforced Concrete Design, 2nd Edition, by S. Unnikrishna Pillai and
Devdas Menon, Tata McGraw-Hill Publishing Company Limited, New
Delhi, 2003.
5. Limit State Design of Reinforced Concrete Structures, by P. Dayaratnam,
Oxford & I.B.H. Publishing Company Pvt. Ltd., New Delhi, 2004.
6. Reinforced Concrete Design, 1st Revised Edition, by S.N. Sinha, Tata
McGraw-Hill Publishing Company. New Delhi, 1990.
7. Reinforced Concrete, 6th Edition, by S.K. Mallick and A.P. Gupta, Oxford &
IBH Publishing Co. Pvt. Ltd. New Delhi, 1996.
8. Behaviour, Analysis & Design of Reinforced Concrete Structural Elements,
by I.C.Syal and R.K.Ummat, A.H.Wheeler & Co. Ltd., Allahabad, 1989.
9. Reinforced Concrete Structures, 3rd Edition, by I.C.Syal and A.K.Goel,
A.H.Wheeler & Co. Ltd., Allahabad, 1992.
10. Textbook of R.C.C, by G.S.Birdie and J.S.Birdie, Wiley Eastern Limited,
New Delhi, 1993.
11. Design of Concrete Structures, 13th Edition, by Arthur H. Nilson, David
Darwin and Charles W. Dolan, Tata McGraw-Hill Publishing Company
Limited, New Delhi, 2004.
12. Concrete Technology, by A.M.Neville and J.J.Brooks, ELBS with
Longman, 1994.
13. Properties of Concrete, 4th Edition, 1st Indian reprint, by A.M.Neville,
Longman, 2000.
14. Reinforced Concrete Designer’s Handbook, 10th Edition, by C.E.Reynolds
and J.C. Steedman, E & FN SPON, London, 1997.
15. Indian Standard Plain and Reinforced Concrete – Code of Practice (4th
Revision), IS 456: 2000, BIS, New Delhi.
16. Design Aids for Reinforced Concrete to IS: 456 – 1978, BIS, New Delhi.

12.31.9 Test 31 with Solutions

Version 2 CE IIT, Kharagpur


Maximum Marks = 50 Maximum Time = 30
minutes

Answer all questions.

Version 2 CE IIT, Kharagpur


TQ.1: Determine the yield pattern and the uniformly distributed collapse load w
kN/m2 of the triangular slab shown in Fig. 12.31.11a having simple
supports along AB and AC and the edge BC is free. The reinforcing bars

Version 2 CE IIT, Kharagpur


along x and y directions have the moment capacities Mx = 50 kNm/m and
My = 60 kNm/m, respectively. Use both the methods i.e., (i) method of
segmental equilibrium and (ii) method of virtual work.
[25×2 = 50]

A.TQ.1: (i) Method of segmental equilibrium. The yield pattern of the slab is
drawn in Fig.12.31.11a involving x as unknown. Thus, we have two
unknowns x and w here.

The yield pattern divides the slab into two segments 1 and 2, whose free
body diagrams are shown in Figs. 12.31.11b and c, respectively. The
nodal force V has the magnitude (90 – 15x)/ 2 obtained from Eq.12.23 ( V
= My cotα).

Taking moment of all forces and moments of segment 1 about AB and


equating it to zero gives

(1/2) x (8) w (DE)/3 – V (DE) – 8 Mx sin β- My (6-x) cosβ = 0, which


gives

w = (-18 x2 + 1608) / (3.2) x2


(1)

Similarly, taking moment of all forces and moments about AC of segment


2 and equating it to zero gives: (1/2) (6-x) w (8) (6-x) / 3 + V (CD) – 8 Mx =
0, which gives,

w = (-22.5 x2 + 270 x + 390) / (144 + 4x2 – 48x)


(2)

Equating Eqs. 1 and 2

− 18 x 2 + 1608 − 22.5 x 2 + 270 x + 390


=
3.2 x 2 144 + 4 x 2 − 48 x

or, 27 x2 – 804 x + 2412 = 0


(3)

which gives x = 3.3847 m.

From Eq. 1: w = (1608 – 18x2) / 3.2 x2 = 38.2369 kN/m2 and


From Eq.2: w = (-22.5 x2+ 270 x + 390) / (144 + 4 x2 – 48 x) = 38.2369
kN/m2.

(ii) Method of virtual work

Version 2 CE IIT, Kharagpur


Total external work TEW = {w (Δ) / 3 (2)} {8x + (6 – x) 8} = 8 wΔ
(4)

Total internal work TIW = 8Mx (θ1) sin β + (6-x) My θ1 cos β + (6-x) My θ2
cos 90 + 8 Mx θ2

= 8 (50) (Δ / DE) (0.8) + (6 – x) (60) (Δ/DE) (0.6) + 0 + 8 (50) Δ/(6-


x)}

⎡ 400 45( 6 − x ) 400 ⎤


= Δ⎢ + +
⎣ x x ( 6 − x ) ⎥⎦
(5)

Equating Eqs. 4 and 5

⎡ 400 45( 6 − x ) 400 ⎤


8 wΔ = Δ ⎢ + +
⎣ x x 6 − x ⎥⎦

1 ⎡ 670 400 ⎤
or, w= ⎢ − 45 +
8⎣ x 6 − x ⎥⎦
(6)

Equation 6 is the only equation to determine x and w. Differentiating w


with respect to x and equating it to zero will give the lowest value of w.
Thus, dw/dx = 0 gives:

(-) 670/(x2) + 400 / (6 –x)2 = 0

or, 27 x2 – 804 x + 2412 = 0


(7)

Equation 7 is the same as Eq. 3, obtained by the method of segmental


equilibrium. The solution of Eq.7 is the same as that of Eq.3 and so, we
get x = 3.3847 m.

Using the value of x in Eq.6, we get

1 ⎡ 670 400 ⎤
w= ⎢ − 45 + ⎥ = 38.2369 kN/m2
8⎣ x 6 − x⎦

Thus, we get the same values of x and w by both methods.

Version 2 CE IIT, Kharagpur


12.31.10 Summary of this Lesson
This lesson presents the derivations of the expressions for determining
the work done by bending and twisting moments when yield lines are at
angles with the two orthogonal directions of the reinforcing bars. The
need for the nodal forces and their determination are explained when one
yield line meets another yield line or the free edge. Different possible
yield patterns of two-way slabs are explained. Numerical problems are
solved for the purpose of illustration taking examples with or without
nodal forces employing both (i) method of segmental equilibrium and (ii)
method of virtual work. Illustrative examples, practice problems and
problem of test will give a clear understanding of analysing the slabs by
the two methods.

Version 2 CE IIT, Kharagpur


Module
12
Yield Line Analysis for
Slabs
Version 2 CE IIT, Kharagpur
Lesson
32
Two-way Rectangular,
Square, Triangular and
Circular Slabs
Version 2 CE IIT, Kharagpur
Instructional Objectives:
At the end of this lesson, the student should be able to:

• analyse rectangular slabs simply supported at three edges and free at the
other edge considering the two possible yield patterns, employing (i) the
method of segmental equilibrium and (ii) the method of virtual work,
• analyse square slab with forking yield patterns when the corners are having
inadequate reinforcement,
• predict yield lines of fan pattern for slabs in case this may be a possibility,
• analyse the fan pattern of yield lines to determine the collapse loads of
triangular and circular slabs with different support conditions,
• analyse the fan pattern of yield lines to determine the collapse loads of
circular slabs clamped along the circumference and having a column support
at the centre.

12.32.1 Introduction
Rectangular / square slabs may have different yield patterns depending on
the support conditions and type of loads. Simply supported slabs at three edges
and free at the other edge may have two types of yield patterns depending on the
ratio of moment resisting capacities and the aspect ratio. This lesson first takes
up such slabs to determine the condition for selecting a particular one out of the
two possible yield patterns. The case of a square slab having forking yield
pattern is explained when the corner reinforcement is inadequate. Several cases
of triangular and circular slabs with or without a central column support are taken
up to explain the yield lines of fan pattern.

All the expressions are derived by employing the method of either


segmental equilibrium or virtual work. In some cases both the methods are taken
up to compare the values.

12.32.2 Rectangular Slabs Simply Supported at Three


Edges and Free at the Other Edge Considering Yield
Pattern 1

Version 2 CE IIT, Kharagpur


Figure 12.31.4c of Lesson 31 shows the yield pattern 1 of such slabs
involving one additional unknown y. So, there are two unknowns – y and w to be
determined. The yield pattern divides the slab into three segments marked by 1,
2 and 3 (Fig.12.32.1a). The slab carrying uniformly distributed load of w kN/m2,
undergoes deflection of Δ at point E. The free body diagrams of segments 1 and
2 are shown in Figs.12.32.1b and c, respectively. Due to symmetry, segments 1
and 3 are identical. Segment 1 is subdivided into two parts as 11 and 12. We
employ the method of segmental equilibrium first.

(A) Method of segmental equilibrium

Here, the nodal forces are zero since the moment capacities of three
intersecting yield lines are identical. The equilibrium equation of segment 1 is
developed taking the moments of loads and moments of segment 1 about AB,

Version 2 CE IIT, Kharagpur


and equating the same to zero. Thus, we get: (Lx/2) (Ly – y) w (Lx / 4) + (Lx /4)y
w (Lx / 6) – Mx Ly = 0,

or w = (24 Mx Ly) / {3Lx2 (Ly – y) + Lx2 y}


(12.41)

Similarly, the equilibrium equation of segment 2 is developed by taking moment


of loads and moments of segment 2 about the base BC and equating the same
to zero, which gives: (Lx / 2) y w (y/3) – My Lx = 0

or w = 6 My / y2
(12.42)

Equating the expression of w from Eqs.12.41 and 12.42, we get:


24 Mx Ly / {3Lx2 (Ly – y) + Lx2 y} = 6My / y2

or 4 Mx Ly y2 + 2My Lx2 y – 3 My Lx2 Ly = 0


(12.43)

The solution of Eq. 12.43 is

y = [-2 My Lx2 + {4 My2 Lx4 + 48 Mx My Lx2 Ly2}1/2] / 8Mx Ly


(12.44)

After getting the value of y from Eq. 12.44, the value of the collapse load w is
obtained from either Eq. 12.41 or Eq. 12.42. The condition that y < Ly gives:
(- 2)(My/Mx) Lx2 + {(4) (My/Mx)2 Lx4 + 48(My / Mx) Lx2 Ly2} < 8Ly2

or My / Mx < 4 (Ly/Lx)2
(12.45)

(B) The method of virtual work

Referring to Figs.12.32.1a, b and c, total external work done by the loads of three
segments TEW is as follows: TEW = 2w [(Lx/2) (Ly-y) (Δ/2) + (Lx/2)y (Δ/3)]

or TEW = w Lx (3Ly – y) (Δ/6)


(12.46)

Total internal work done by the yield moments TIW is: TIW = 2 Mx (Ly) θx + My (y)
θy.
Using θx = 2Δ / Lx and θy = Δ/y, we have

Version 2 CE IIT, Kharagpur


TIW = Δ {4Mx (Ly/Lx) + My (Lx/y)}
(12.47)

From the two equations of TEW and TIW (Eqs. 12.46 and 12.47), we have

w = (24 Mx Ly y + (My Lx2) / {Lx2 (3y Ly – y2)}


(12.48)

This is the only equation in the method of virtual work to determine y and the
collapse load w. Differentiating w with respect to y and equating that to zero to
get the lowest w, we have: Lx2 (3y Ly – y2) (24 Mx Ly) – (24 Mx Ly y + 6My Lx2) Lx2
(3Ly – 2y) = 0

or 4 Mx Ly y2 + 2 My Lx2 y – 3My Lx2 Ly = 0


(12.43)

Thus, the method of virtual work gives the same equation (Eq. 12.43) as
that obtained by the method of segmental equilibrium. After getting the value of y
from Eq. 12.44, the value of the collapse load w is obtained from Eq. 12.48.

12.32.3 Rectangular Slabs Simply Supported at Three


Edges and Free at the Other Edge Considering Yield Pattern
2

Version 2 CE IIT, Kharagpur


Version 2 CE IIT, Kharagpur
Figure 12.31.4d of Lesson 31 shows that yield pattern 2 of such slabs
involving one additional unknown x. So, there are two unknowns – x and w to
determine. The yield pattern divides the slab into three segments marked by 1, 2
and 3. The slab, carrying the uniformly distributed load of w kN/m2, undergoes
deflection of Δ along EF (Fig.12.32.2a). Free body diagrams of segments 1 and 2
are shown in Figs.12.32.2b and c, respectively. Due to symmetry, segments 1
and 3 are identical. Segment 2 is further subdivided into three parts 21, 22 and
23, of which sub-segments 21 and 23 are also symmetrical. We consider the
method of segmental equilibrium first.

(A) Method of segmental equilibrium

The nodal forces on the left and right of point E of the yield line BE are (+)
V, acting downward and (-) V, acting upward, respectively. Similarly, nodal forces
on the left and right of point F of yield line CF are (-) V, acting upward and (+) V,
acting downward, respectively. The magnitude of the nodal force, as given in
Eq.12.23 of Lesson 31, is

V = My cot α = My (AE/AB) = My (x/Ly)


(12.49)

Version 2 CE IIT, Kharagpur


We now develop the equilibrium equation of segment 1 taking moments of loads
and moments of segment 1 about AB and equating the same to zero.

w (x/2) Ly (x/3) + Vx – Mx Ly = 0, which gives,

w = 6 (MxLy2 – My x2) / (x2 Ly2)


(12.50)

Similarly, the equilibrium equation of segment 2 is developed by taking moment


of loads and forces of segment 2 about BC and equating the same to zero.

2 w (x/2) (Ly) (Ly/3) + w (Lx – 2x) Ly (Ly/2) – 2V Ly – 2My x = 0 , which gives,

w = (24 My x) / { Ly2 (3Lx – 4x)}


(12.51)

Equating the two expressions of w from Eqs.12.50 and 12.51, we have,

6( M x L y 2 − M y x 2 ) 24 M y x
=
x2 Ly 2 L y 2 ( 3L x − 4 x )

or 3(My / Mx) Lx x2 + 4 Ly2 x – 3Ly2 Lx = 0


(12.52)

The solution of the above equations is:

x = [- 4Ly2 + {16 Ly4 + 36 (My / Mx) Lx2 Ly2}1/2] / {6 Lx (My / Mx)}


(12.53)

From Fig. 12.32.2a, it is evident that x < Lx / 2. So, we get from Eq. 12.53 for the
condition that x < Lx/2,

- 4 Ly2 + {16 Ly4 + 36 (My / Mx) Lx2 Ly2}1/2 〈 (Lx / 2) (6 Lx) (My /Mx), which
finally gives

(My / Mx) > (4/3) (Ly / Lx)2


(12.54)

Therefore, Eq. 12.54 shall be used to confirm if yield pattern 2 is possible or not.
After getting the value of x from Eq. 12.53 (the solution of Eq. 12.52), the
collapse load w is determined either from Eq. 12.50 or from Eq. 12.51.

Version 2 CE IIT, Kharagpur


(B) Method of virtual work

Referring to Figs. 12.32.2a, b and c, the total external work done by the loads of
three segments TEW is as follows: TEW = 2 (W1) + 2 (W21) + W22 = 2{(x/2) Ly
w (Δ/3)} + 2 {(x/2) Ly w (Δ/3)}+ (Lx – 2x) Ly w (Δ/2)

or TEW = w Ly (Δ/6) (3 Lx – 2x)


(12.55)

The total internal work done by the yield moments TIW is: TIW = 2 Mx Ly θx + 2
My x θy. Using θx = Δ/x and θy = Δ/Ly, we have: TIW = 2Mx Ly (Δ/x) + 2 Myx (Δ/Ly),

or TIW = Δ{2Mx Ly/x + 2 My x/Ly)


(12.56)

From the two expressions of TEW and TIW (Eqs. 12.55 and 12.56), we have: w
(Ly/6) (3Lx – 2x) = 2Mx (Ly/x) + 2My (x/Ly),

or w = {12 Mx Ly2 + 12 My x2} / {Ly2 (3x Lx – 2x2)


(12.57)

The above is the only equation to determine x and the collapse load w in
the method of virtual work. Differentiating w with respect to x and setting that to
zero shall give the lowest load. Hence, we have: Ly2 (3 x Lx – 2x2) (24 My x) – (12
Mx Ly2 + 12 My x2) Ly2 (3Lx – 4x) = 0,

or 3 (My / Mx) Lx x2 + 4 Ly x2 – 3Ly2 Lx = 0


(12.52)

Thus, the method of virtual work gives the same equation (Eq.12.52) as
that obtained by the method of segmental equilibrium. After getting the value of y
from Eq.12.52 (or Eq.12.53, the solution of Eq.12.52), the collapse load w is
determined from Eq. 12.57.

12.32.4 Special Cases for Predicting Yield Patterns


Let us examine the two conditions of Eqs.12.45 and 12.54 for determining
the correct yield pattern.

Equation 12.45 specifies that for the yield pattern 1, (My / Mx) < 4 (Ly /Lx)2,
while Eq. 12.54 specifies that for the yield pattern 2, My/Mx > 4/3 (Ly/Lx)2.
However, for slabs when both the conditions are satisfied, it appears that both
the yield patterns are possible. In such cases, both the yield patterns should be
considered and the one giving the lowest load (either 1 or 2) is the correct yield
pattern. We explain the above taking two specific cases of square slabs having

Version 2 CE IIT, Kharagpur


simply supported edges on three sides and free at the other side, as shown in
Fig.12.32.1a, with two different ratios of My / Mx.

(i) Case 1: Ly / Lx = 1 and My / Mx = 1.5

Here, (My/Mx) is less than 4 (Ly /Lx)2 (= 4). So, yield pattern 1 is possible.
Similarly, (My/Mx) is greater than (4/3) (Ly/Lx)2 (= 4/3). So, yield pattern 2 is also
possible. Therefore, we should consider both the yield patterns and select the
one giving the lowest value of the collapse load.

For the yield pattern 1, the value of the collapse load is obtained from any of the
three equations (Eqs.12.41, 12.42 and 12.48), after determining the value of y
from Eq.12.43. The results are as follows.

4 Mx Ly y2 + 2My Lx2 y – 3 My Lx2 Ly = 0


(12.43)

Using My = 1.5 Mx and Ly = Lx, we have

4y2 + 3Lx y – 4.5 Lx2 = 0


(12.58)

from which y = 0.75 Lx


(12.59)

Using the value of y in the three equations, we get the value of w as:

24M x Ly
From Eq.12.41: w = = 16( M x / Lx 2 )
3Lx ( Ly − y ) + ( Lx 2 y )
2

(12.60)

From Eq. 12.42: w = 6My / y2 = 16 (Mx / Lx2)


(12.60)

24 M x L y y + 6 M y L x 2
From Eq. 12.48: w = = 16( M x / L x 2 )
2 2
L x ( 3 yL y − y )
(12.60)

We now consider yield pattern 2 in which the unknown distance x is obtained


from Eq. 12.52, which is

3 (My / Mx) Lx x2 + 4 Ly2 x – 3Ly2 Lx = 0


(12.52)

Version 2 CE IIT, Kharagpur


When My = 1.5 Mx and Ly = Lx, the above equation becomes

4.5 x2 + 4 Lx x – 3 Lx2 = 0
(12.61)

which gives x = 0.4852 Lx


(12.62)

Using the value of x in Eqs. 12.50, 12.51 and 12.57, we get the value of w as
given below.

6( M x Ly 2 − M y x 2 )
From Eq. 12.50: w = 2 2
= 16.488( M x / Lx 2 )
x Ly
(12.63)

24( M y )x
From Eq. 12.51: w = = 16.488( M x / L x 2 )
2
L y ( 3L x − 4 x )
(12.63)

12 M x L y 2 + 12 M y x 2
From Eq. 12.57: w = = 16.488( M x / L x 2 )
2 2
L y ( 3 xL x − 2 x )
(12.63)

Computing the values of w from Eqs. 12.60 and 12.63, it is evident that yield
pattern 1 is the correct yield pattern.

(ii) Case 2: Ly / Lx =1 and My/Mx = 3.5

Here also both the conditions of Eqs. 12.45 and 12.54 are satisfied. So, we
consider them separately. Proceeding in the same manner, we determine the
values of y and w for yield pattern 1, and x and w for yield pattern 2. Only the
final results are given below.

(a) For the yield pattern 1, y = 0.966 Lx


(12.64)

and w = 22.487 (Mx /Lx2)


(12.65)

(b) For the yield pattern 2, x = 0.377 Lx


(12.66)

Version 2 CE IIT, Kharagpur


and w = 21.222 (Mx / Lx2)
(12.67)

Thus, in this case the yield pattern 2 is the correct yield pattern.

12.32.5 Square Slabs with Forking Yield Pattern

Version 2 CE IIT, Kharagpur


Yield lines of two-way slabs having inadequate corner reinforcement fork
out before they reach the corners, as shown in Fig.12.31.4e of Lesson 31. We
analyse a square slab having simply supported edges in all four sides subjected
to uniformly distributed load w kN/m2 and having yield pattern as shown in
Fig.12.32.3a. Yield lines divide the slab into symmetrical segments, of which we
consider segments 1 and 2 employing the method of segmental equilibrium.

The yield lines have two unknown parameters x and r (Fig.12.32.3a),


where x is the distance AE and r is the perpendicular distance GV from point G to
the yield line EF. In segment 1, EF is the negative yield line, and EG and FG are
positive yield lines. Figure 12.32.3b shows the free body diagram of segment 1.
The negative and positive moments are represented by Mnb and Mpb,
respectively. These moments are resolved into their respective components Mnx,
Mny, Mpx and Mpy along the sides of the slab, as shown in the figure. Segment 2 is
shown in Fig.12.32.3c which is bounded by yield lines FG, GO, OH and HI, and
the side FI. All the yield lines of segment 2 are having positive moment Mpb which
are resolved as Mpx and Mpy along the sides of the square slab. Segment 2 is
further subdivided into four sub-segments marked by 21, 22, 23 and 24 of which
sub-segments 21 and 23 are symmetrical.

Version 2 CE IIT, Kharagpur


Before we take up the equilibrium of the two segments, let us determine
the dimensions of different lengths of the two segments. With reference to
Figs.12.32.3a, b and c,

AE = AF = BI = BU = x (assumed)
GV = r (assumed)
AK = KG = GJ = AJ = {(x /√2) + r}/ √2 = r/√2 + x/ 2
FK = EJ = AJ – AE = r/√2 – x/2

We further assume that Mnx = Mny = Mn and Mpx = Mpy = Mp.

The area of triangle EFG = (1/2) (EF) (FG) = x r /√2.

Taking moments of the loads and moments of segment 1 about EF and equating
the same to zero, we get: w (area EFG) (r /3) + (- Mnx AF – Mny AE + Mpy EJ –
Mpx JG – Mpy GK + Mpx KF) cos 450 = 0

or r = {6(Mn + Mp) / w}½


(12.68)

We now consider the equilibrium of segment 2 taking moment of loads and


moment about FI. For easy understanding, the contributions of different sub-
segments are computed separately and then added with the contribution of the
moments. The contributions of different sub-segments and intermediate addition
are:

(i) For the sub-segment 21:


2
⎛ r x⎞ ⎛ r x⎞
(1 / 2 )( FK )( KG ) w ( KG / 3) = (1 / 2 ) ⎜ − ⎟ ⎜ + ⎟ ( w / 3)
⎝ 2 2⎠ ⎝ 2 2⎠

(ii) For the sub-segment 22:

⎧ x⎞ ⎫⎛ r
2
⎛ r x⎞
( KS )( KG ) 2 ( w / 2 ) = ⎨ L − 2 ⎜ + ⎟ ⎬⎜ + ⎟ (w / 2 )
⎩ ⎝ 2 2⎠ ⎭⎝ 2 2⎠

(iii) For the sub-segment 24: (1/2) (GH) {(L/2) – KG} {KG + {(L/2 – KG)/3}
w

= w ⎧⎨ L − 2 ⎛⎜ r + x ⎞⎟ ⎫⎬ ⎧⎨ L − ⎛⎜ r + x ⎞⎟ ⎫⎬ ⎡⎢ ⎛⎜ r + x ⎞⎟ + ⎧L ⎛ r
⎨ −⎜
x ⎞⎫ ⎤
+ ⎟ ⎬ / 3⎥
2 ⎩ ⎝ 2 2 ⎠⎭ ⎩ 2 ⎝ 2 2 ⎠ ⎭ ⎢⎣ ⎝ 2 2⎠ ⎩2 ⎝ 2 2 ⎠ ⎭ ⎥⎦

w ⎧⎪ L2 ⎫⎪ ⎡ ⎛ r x⎫ ⎤
2
= ⎛ r x⎞ ⎛ r x⎞ ⎛ r x⎞ x⎞ ⎧L r
⎨ − L⎜ + ⎟ − L⎜ + ⎟ + 2⎜ + ⎟ ⎬ ⎢⎜ + ⎟+⎨ − − ⎬ / 3⎥
2 ⎪⎩ 2 ⎝ 2 2⎠ ⎝ 2 2⎠ ⎝ 2 2⎠ ⎪⎭ ⎣ ⎝ 2 2 ⎠ ⎩ 2 2 2⎭ ⎦

Version 2 CE IIT, Kharagpur


= ⎧⎪ wL 2 ⎛ r x⎞ ⎛ r x ⎞ ⎫⎪ ⎡ r
2
x L r x⎤
⎨ − wL ⎜
⎜ + ⎟
⎟ + w ⎜
⎜ + ⎟
⎟ ⎬ ⎢ + + − − ⎥
⎪⎩ 4 ⎝ 2 2 ⎠ ⎝ 2 2 ⎠ ⎪⎭ ⎣ 2 2 6 3 2 6 ⎦

⎧⎪ wL 2 ⎛ r x⎞ ⎛ r x ⎞ ⎫⎪ ⎧ 2 r x L ⎫
2
= ⎨ − ⎜ + ⎟ + ⎜ + ⎟ + + ⎬
wL ⎜ ⎟ w ⎜ ⎟ ⎬ ⎨
⎪⎩ 4 ⎝ 2 2⎠ ⎝ 2 2 ⎠ ⎪⎭ ⎩ 3 3 6⎭

⎧ wL 2 x ⎞⎫ ⎧ 2r x L ⎫ x ⎞ ⎧ 2r x L ⎫
2
= ⎛ r ⎛ r
⎨ − wL ⎜⎜ + ⎟⎟ ⎬ ⎨ + + ⎬ + w ⎜⎜ + ⎟⎟ ⎨ + + ⎬
⎩ 4 ⎝ 2 2 ⎠⎭ ⎩ 3 3 6 ⎭ ⎝ 2 2 ⎠ ⎩ 3 3 6⎭

⎛ 2 r x ⎞ wL 3 x ⎞ ⎧ 2r x L ⎫ x ⎞ ⎧ 2r x L ⎫
2
= wL 2 ⎛ r ⎛ r
⎜ ⎟
⎜ 3 + 3 ⎟ + 24 - wL ⎜⎜ 2 + 2 ⎟⎟ ⎨ 3 + 3 + 6 ⎬ + w ⎜⎜ 2 + 2 ⎟⎟ ⎨ 3 + 3 + 6 ⎬
4 ⎝ ⎠ ⎝ ⎠⎩ ⎭ ⎝ ⎠ ⎩ ⎭

(iv) The addition of {2 (sub-segment 21) + (sub-segment 22)}

x⎞ ⎛ w ⎞ ⎧ x ⎞⎫⎛ r
2 2
= ⎛ r x ⎞⎛ r ⎛ r x⎞ ⎛ w ⎞
⎜ − ⎟⎜ + ⎟ ⎜ ⎟ + ⎨ L − 2⎜ + ⎟⎬⎜ + ⎟ ⎜ ⎟
⎝ 2 2 ⎠⎝ 2 2 ⎠ ⎝ ⎠ ⎩
3 ⎝ 2 2 ⎠⎭⎝ 2 2 ⎠ ⎝ 2 ⎠

2
= w ⎛⎜ r + x ⎞⎟ ⎛⎜ r −
x
+
L

r x⎞
− ⎟
⎝ 2 2⎠ ⎝3 2 6 2 2 2⎠

= w ⎛⎜ r + x ⎞⎟ ⎛⎜ − 2 r − 2 x + L ⎞⎟
2

⎝ 2 2 ⎠ ⎜⎝ 3 3 2 ⎟⎠

(v) Therefore, {2 (sub-segment 21) + (sub-segment 22) + (sub-segment 24)}

⎛ L ⎞
2
⎛ r x ⎞
⎜−
2r 2x L 2r x
⎟ +
= w⎜ + ⎟ ⎜ − + + + +
⎝ 2 2 ⎠ ⎝ 3 3 2 3 3 6 ⎟⎠
wL 2
⎛ 2r x ⎞ wL 3 ⎛ r x ⎞⎧ 2r x L ⎫
⎜ + ⎟ + − wL ⎜ + ⎟⎨ + + ⎬
4 ⎜ 3 ⎟
3 ⎠ 24 ⎝ 2 ⎠⎩ 3 3 6 ⎭
⎝ 2

⎛ L ⎞
2
⎛ r x ⎞ ⎛ x 2L ⎞ wL 2 r wL 2 x wL 3 ⎛ r x ⎞

2r x

= w⎜ + ⎟ ⎜− + ⎟+ + + - wL ⎜ + ⎟ ⎜ + +
⎝ 2 2 ⎠ ⎝ 3 3 ⎠ 6 2 12 24 ⎝ 2 2 ⎠ ⎝ 3 2 6 ⎟⎠

x ⎞ ⎛ 2 rL L2 ⎞
2
⎛ r x ⎞ ⎛ x ⎞ ⎛ r xL 2 rL xL
= w⎜ + ⎟ ⎜− ⎟ + w⎜ + ⎟ ⎜ + − − − ⎟
⎝ 2 2 ⎠ ⎝ 3 ⎠ ⎝ 2 2 ⎠ ⎜⎝ 3 2 3 3 3 6 ⎟⎠
wL 2 r wL 2 x wL 3
+ + +
6 2 12 24

wr 2 x wx 3 wrx 2 wL3
=− − − +
6 12 3 2 24

(vi) Including the contributions of moment, the equilibrium equation of segment 2


becomes

Version 2 CE IIT, Kharagpur


r 2 x wx 3 wrx 2 wL3
−w − − + − M p ( L − 2 x) = 0
6 12 3 2 24
(12.69)

Using the expression of r from Eq.12.68 in the above equation, we have: - w


6(Mn + Mp) x / 6w – wx3/12 – (w x2/3√2) {6(Mn + Mp)/w}1/2 + (w L3/24) – Mp (L-2x)
=0

or w x3/ 12 + x2{w (Mn + Mp) / 3}1/2 + (Mn – Mp)x + MpL – wL3/24 = 0


(12.70)

Setting dw/dx = 0, Eq.12.70 finally gives:

wx2/4 + 2x{ (Mn + Mp) w/3}1/2 + (Mn-Mp) = 0


(12.71)

which has the solution of x as

x = 2{(Mn + 7Mp) / 3 w}1/2 – 4 {(Mn + Mp) / 3w}1/2


(12.72)

Thus, the three unknowns x, r and w are determined from the three equations,
Eqs. 12.68, 12.70 and 12.72.

We now consider two cases below: (i) with adequate corner reinforcement and
(ii) with inadequate corner reinforcement.

Case (i): When the corner reinforcement is adequate

The value of x = 0 when the corner reinforcement is adequate as there will be no


forking of the yield line. Putting x = 0 in Eq. 12.70, we have: MpL – wL3 / 24 = 0

or w = 24 Mp / L2
(12.73)

Using x = 0 in Eq. 12.72, we get: Mn + 7Mp = 4 (Mn + Mp), which gives:


Mn = Mp
(12.74)

Using w = 24 Mp /L2 and Mn = Mp from Eqs 12.73 and 12.74 in Eq. 12.68, we
have:

r = L / √2
(12.75)

Version 2 CE IIT, Kharagpur


Case (ii): When the corner reinforcement is inadequate

In this case, Mn = 0 and only Mp is present. Putting Mn = 0 in Eq. 12.71, we have:

x = 2(7Mp / 3w)1/2 – 4 (Mp /3w)1/2 = 0.746 (Mp/w)1/2


(12.76)

Using Mn = 0 and substituting the value of x from Eq.12.76 in Eq.12.70, we have:


w x3/12 + x2 (w Mp/3)1/2 – Mpx + MpL – wL3 / 24 =0

or wL3 – 24 MpL + 9.3624 Mp (Mp/w)1/2 = 0


(12.77)

The numerical solution of w of Eq.12.77 is:

w = 22 M /L2
(12.78)

The two values of w of the two cases (i) and (ii) reveal the reduction of w by
about 8.33 per cent (= 24 – 22 / 0.24) when the corner reinforcement is such that
Mn = 0. Therefore, it is justified to reduce the load carrying capacity by ten per
cent rather than involving in the complicated analysis.

Further, if the negative moment carrying capacity Mn is larger than Mp, the value
of x becomes negative. In that case, the yield pattern is the same as that with
adequate corner reinforcement.

12.32.6 Yield Lines of Fan Pattern


Triangular or circular slabs subjected to uniformly distributed loads or point
loads may have yield lines of fan pattern. We explain below different cases of
such slabs to estimate the collapse loads.

Version 2 CE IIT, Kharagpur


(A) Triangular slab subjected to a point load

Figure 12.32.4a shows a triangular slab clamped along the three edges
and subjected to a point load P away from the edges and corners.
Negative yield line of approximately circular pattern is formed with positive
yield lines radiating outward from the point of application of the load.
Assuming the resisting moment capacities of Mp and Mn per unit length for
positive and negative moments, respectively, we consider one segment
ODE, as shown in Fig.12.32.4b. The angle DOE is dθ and the resultant of
the positive moments Mp along DO and OE, is Mp rdθ along DE, as shown
in Fig.12.32.4c. The resultant Mp rdθ is in the same direction of the
negative moment Mn rdθ along DE. The fractional part of the total load P
acting on the segment is P dθ / 2π.

Version 2 CE IIT, Kharagpur


Taking moment of the load and moments of the segment DOE about DE
and equating the same to zero, we have: (Mp + Mn) r dθ - P(dθ / 2π) r = 0,
which gives:

P = 2π (Mp + Mn)
(12.79)

If we assume Mn = kMp, Eq.12.79 gives:

P = 2π Mp (1 + k)
(12.80)

When Mn = Mp i.e., k =1, we have:

P = 4π Mp
(12.81)

Equations12.79 to 12.81 reveal that the collapse load P is independent of


the radius of the fan patterns of yield lines. Thus, at the collapse load, the
triangular slab clamped along three edges may have complete fan pattern of
yield lines of any radius without any change of the collapse load. However, it is
necessary that the boundaries must have the resisting moment capacities Mn at
all points.

(B) Simply supported circular slabs subjected to uniformly distributed load


w kN/m2

Version 2 CE IIT, Kharagpur


Figure 12.32.5a shows the positive yield lines of radiating type having the
moment resistance capacity of Mp per unit length. Figure 12.32.5b shows the free
body diagram of the segment DOE making an angle dθ at the center of the slab.

Equating the moment of the load and moment about DE to zero, we have:
MpR dθ = (1/2) (R dθ) R w (R/3), which gives:

w = 6 Mp / R2
(12.82)

(C) Clamped circular slab subjected to uniformly distributed load w kN/m2

Figure 12.32.6a shows the circular slab clamped at the periphery having
negative yield lines along the periphery of moment resisting capacity of Mn per
unit length. Positive yield lines radiating from the centre of the slab are also
shown in the figure having moment resisting capacity of Mp per unit length.
Figure 12.32.6b shows the free body diagram of one segment DOE where dθ is

Version 2 CE IIT, Kharagpur


the angle made by the yield lines DO and OE at the centre. As explained earlier,
the resultant of the two positive moments Mp of magnitude MpR dθ and the
negative moment Mn are in the same direction as shown in the figure.
Equating the moment of load and moments of segment DOE about DE to zero,
we have: Mp r dθ + Mn r dθ - (1/2) (rdθ) r w (r/3) = 0. This gives:

w = 6(Mn + Mp) / R2
(12.83)

Assuming Mn = kMp, Eq.12.83 gives:

w = 6 Mp (1 + k) / R2
(12.84)

(D) Circular slabs clamped along the circumference and having a column
support at the centre

Version 2 CE IIT, Kharagpur


Version 2 CE IIT, Kharagpur
In many practical examples of circular slabs, a column support is provided
at the centre. The yield pattern of a circular slab clamped along the periphery and
having a central column support, subjected to uniformly distributed load of w
kN/m2 is shown in Fig.12.32.7a.

Negative yield lines of moment resisting capacity Mn are along the


periphery and radiating from the centre of the slab. Positive yield line is shown in
approximately circular pattern having a radius of r. Figure 12.32.7b shows the
free body diagram of a segment OCD where dθ is the angle made by the yield
lines OC and OD at the centre.

Figure 12.32.7c shows the deflection profile of the segment OCD having a
deflection of Δ along AB. The segment is further divided into four sub-segments
11, 12, 13 and 14, of which sub-segments 13 and 14 are symmetrical. The
deflection of the centroid of the sub-segment 11 (OAB) is 2 Δ/3 at G and the
deflection of the centroid of the sub-segments 13 and 14 is Δ/3. The deflection of
the centroid of the sub-segment 12 is Δ/2. We employ the method of virtual work
here. The external work done by the loads of the four sub-segments are
presented in Table 12.1 giving the respective area of the sub-segment and the
deflection at the centroid. By summing them considering that the work done by
the load of segment ACE is the same as that of segment BDF, the total external
work done TEW is as follows:

TEW = (w Δdθ) {r2 / 3 + r (R –r) / 2 + (R – r)2 / 6}


(12.85)

The internal works done by different positive and negative yield lines of the
segment OCD are furnished in Table 12.2 giving the respective moment and
rotation. The total internal work (TIW) is then obtained by summing them
considering that the work done by the moment of yield line AC is the same as
that of yield line BD. Thus, the expression of TIW is,

TIW = Δ{Mn (AB) / r + 2 Mn (CE) / (R – r) + Mn (CD) / (R – r) + Mp (AB) / r + Mp


(AB) / (R – r)}
(12.86)

Table 12.1: Total External Work (TEW) by loads of segment OCD

Sl. Sub-Segment Area Deflection at External work done


No. the centroid
1. OAB (11) (r/2) r dθ 2 Δ/3 wΔ dθ (r2 / 3)
2. AEFB (12) r dθ (R – r) Δ/2 (wΔ dθ ) {r (R – r)/2}
3. ACE and BFD (R – r) (R – r) (dθ Δ/3 (wΔ dθ ) {(R – r)2 / 6}
(13 and 14) /2)
Adding, we get: TEW = (wΔ dθ ) {(r2 / 3) + r (R – r) / 2 + (R – r)2 / 6} … Eq.
(12.85)
Version 2 CE IIT, Kharagpur
Table 12.2: Total Internal Work (TIW) by positive and negative yield lines of
segment OCD

Sl. Yield line Moment Rotation Internal work done


No.
1. OA and Mn (AB) Δ/r Mn (AB) (Δ/r)
OB
2. AC and BD 2 Mn Δ/(R – r) 2 Mn (CE) {Δ/(R – r)}
(CE)
3. CD Mn (CD) Δ/(R – r) Mn (CD) {Δ/(R – r)}
4. AB Mp (AB) {Δ/r + Δ/(R – r)} Mp (AB) {Δ/r + Δ/(R – r)}
Adding, we get: TIW = Δ{Mn (AB)/r + 2Mn (CE)/(R – r) + Mn(CD)/(R – r) +
Mp(AB) /r + Mp (AB)/(R – r)} … Eq. (12.86)

Equating the two works TEW and TIW form Eqs.12.85 and 12.86, and assuming
that Mn = Mp = M, we get: w dθ { r2/3 + r(R-r) / 2 + (R – r)2/6} = M { 2(AB)/ r =
(2 CE + CD + AB) / (R –r)}

or w dθ { R (R + r) / 6} = M{2 (dθ) + 2R (dθ) / (R-r)}

or w = {12M (2 R – r)} / {R (R2 – r2)}


(12.87)

Setting dw/dr = 0, we have the most critical layout of the mechanism and then

r2 – 4 R r + R2 = 0
(12.88)

which has a solution,

r = 0.2679 R
(12.89)

Using the value of r from Eq. 12.89 in Eq. 12.87, we have

w = 22.39 (M / R2)
(12.90)

or M = w R2 / 22.39
(12.91)

12.32.7 Practice Questions and Problems with Answers

Version 2 CE IIT, Kharagpur


Q.1: Determine the collapse load for the two possible yield patterns of a
rectangular slab, simply supported at three edges and free at the other
edge, when subjected to uniformly distributed loads employing (i) method of
segmental equilibrium and (ii) method of virtual work.

A.1: secs. 12.32.2 and 3

Q.2: When do the yield lines of square slabs fork before reaching the corners?
What should be the approach to estimate the uniformly distributed collapse
load in such cases?

A.2: sec. 12.32.5

Q.3: Determine the collapse point load in a triangular slab clamped along the
three edges.

A.3: sec. 12.32.6 – part A

Q.4: Determine the uniformly distributed collapse load of a circular slab clamped
along the periphery.

A.4: sec. 12.32.6 – part C

Q.5: Determine the uniformly distributed collapse load of a circular slab having
clamped support along the periphery and a column support at the centre of
the slab.

A.5: sec.12.32.6 – part D

12.32.8 References
1. Reinforced Concrete Limit State Design, 6th Edition, by Ashok K. Jain,
Nem Chand & Bros, Roorkee, 2002.
2. Limit State Design of Reinforced Concrete, 2nd Edition, by P.C. Varghese,
Prentice-Hall of India Pvt. Ltd., New Delhi, 2002.
3. Advanced Reinforced Concrete Design, by P.C. Varghese, Prentice-Hall
of India Pvt. Ltd., New Delhi, 2001.
4. Reinforced Concrete Design, 2nd Edition, by S. Unnikrishna Pillai and
Devdas Menon, Tata McGraw-Hill Publishing Company Limited, New
Delhi, 2003.
5. Limit State Design of Reinforced Concrete Structures, by P. Dayaratnam,
Oxford & I.B.H. Publishing Company Pvt. Ltd., New Delhi, 2004.
6. Reinforced Concrete Design, 1st Revised Edition, by S.N. Sinha, Tata
McGraw-Hill Publishing Company. New Delhi, 1990.

Version 2 CE IIT, Kharagpur


7. Reinforced Concrete, 6th Edition, by S.K. Mallick and A.P. Gupta, Oxford &
IBH Publishing Co. Pvt. Ltd. New Delhi, 1996.
8. Behaviour, Analysis & Design of Reinforced Concrete Structural Elements,
by I.C.Syal and R.K.Ummat, A.H.Wheeler & Co. Ltd., Allahabad, 1989.
9. Reinforced Concrete Structures, 3rd Edition, by I.C.Syal and A.K.Goel,
A.H.Wheeler & Co. Ltd., Allahabad, 1992.
10. Textbook of R.C.C, by G.S.Birdie and J.S.Birdie, Wiley Eastern Limited,
New Delhi, 1993.
11. Design of Concrete Structures, 13th Edition, by Arthur H. Nilson, David
Darwin and Charles W. Dolan, Tata McGraw-Hill Publishing Company
Limited, New Delhi, 2004.
12. Concrete Technology, by A.M.Neville and J.J.Brooks, ELBS with
Longman, 1994.
13. Properties of Concrete, 4th Edition, 1st Indian reprint, by A.M.Neville,
Longman, 2000.
14. Reinforced Concrete Designer’s Handbook, 10th Edition, by C.E.Reynolds
and J.C. Steedman, E & FN SPON, London, 1997.
15. Indian Standard Plain and Reinforced Concrete – Code of Practice (4th
Revision), IS 456: 2000, BIS, New Delhi.
16. Design Aids for Reinforced Concrete to IS: 456 – 1978, BIS, New Delhi.

12.32.9 Test 32 with Solutions


Maximum Marks = 50, Maximum Time = 30 minutes

Answer all questions.

TQ.1: Choose the correct answer for each of the following statements.
(4 × 5 = 20 Marks)
(A) Rectangular slabs simply supported at three edges and free at the other
edge will have yield pattern 1 (Fig.12.32.1a),

(i) when (Mx/ My) < 4 (Ly / Lx)


(ii) when (My / Mx) < 4 (Ly / Lx)2
(iii) when (Mx /My) < 4 (Ly/ Lx)2
(iv) when (My / Mx) > 4 (Ly/ Lx)2

A.TQ.1 (A): (ii).

(B) Nodal forces are to be considered when

(i) the slab is subjected to vertical loads at the corners of the slab
(ii) the slab is one-way
(iii) the slab is polygonal having more than five corner node points

Version 2 CE IIT, Kharagpur


(iv) when the yield line meets a free edge

A.TQ.1 (B): (iv)

(C) Rectangular slabs simply supported at three edges and free at the other
edge will have yield pattern 2 (Fig.12.32.2a),

(i) when (My/ Mx) > (4/3) (Ly / Lx)2


(ii) when (Mx/ My) > (4/3) (Ly / Lx)2
(iii) when (Mx/ My) = (4/3) (Ly / Lx)2
(iv) when (My/ Mx) = (4/3) (Ly / Lx)2

A.TQ.1 (C): (i)

(D) Yield lines of fan pattern will occur in

(i) any slab having a central cut out


(ii) any slab subjected to unsymmetrical loading
(iii) any slab having point load
(iv) any slab having torsional moment

A.TQ.1 (D): (iii)

TQ.2: Determine the uniformly distributed collapse load of a circular slab having
clamped support along the periphery and a column support at the centre of
the slab.

(30 marks)
A.TQ.2: sec.12.32.6 – part D

12.32.10 Summary of this Lesson


This lesson explains two different yield patterns of two-way slabs simply
supported on three edges and free at the other edge. The expressions of
determining the respective collapse loads are derived both by the method of
segmental equilibrium and method of virtual work. The forking out type of yield
pattern of square / rectangular slabs having inadequate corner reinforcement is
explained. Slabs subjected to point loads or circular slabs having yield lines of
fan pattern are taken up to determine the collapse loads. Practical circular slabs
supported at the periphery and with a central column support is also taken up in
this lesson.

Version 2 CE IIT, Kharagpur


Module
12
Yield Line Analysis for
Slabs
Version 2 CE IIT, Kharagpur
Lesson
33
Numerical Examples
Version 2 CE IIT, Kharagpur
Instructional Objectives:
At the end of this lesson, the student should be able to:

• apply the theory of segmental equilibrium for the solution of different types of
slab problems,
• apply the theory of virtual work for the solution of different types of slab
problems.

12.33.1 Introduction
The theoretical formulations of the yield line analysis of slabs are
explained in earlier Lessons 30 to 32, considering (i) the method of segmental
equilibrium and (ii) the method of virtual work. Illustrative examples are solved in
Lesson 30. This lesson includes different types of illustrative examples of slabs
with different combinations of support conditions. Two types of loadings, viz., (i)
point loads and (ii) uniformly distributed loads, are considered. In some cases,
two or three possible yield patterns are examined to select the correct one and
the corresponding loads are determined. The problems include different types of
rectangular or square two-way slabs, triangular, quadrantal and circular slabs.
Circular slabs also include one practical example where it is supported by a
central column in addition to clamped support along the periphery. Some practice
problems and test problem are also included. Understanding the numerical
problems and solving practice and test problems will give a better understanding
of the theories of yield line analysis and their applications.

12.33.2 Illustrative Examples

Version 2 CE IIT, Kharagpur


Version 2 CE IIT, Kharagpur
Problem 1. Determine the uniformly distributed collapse load w kN/m2 of a
square slab (L m × L m), simply supported at three edges and the
other edge is free. Assume My = 2Mx = 2M.

Solution 1. The slab is shown in Fig.12.33.1a. Given data are:


Lx = Ly = L and My = 2Mx = 2M.

Step 1. To examine the possibility of yield patterns 1, 2 or both

Equation12.45 of Lesson 32 stipulates that (My/Mx) 〈 4 (Ly /Lx)2 for


yield pattern 1 and Eq.12.54 of Lesson 32 gives the condition that
(My/Mx) 〉 (4/3) (Ly/Lx)2 for yield pattern 2.

Here, Ly/Lx = 1 and My/Mx = 2. So, (My/Mx) 〈 4 (Ly /Lx)2 and (My/Mx) 〉
(4/3) (Ly /Lx)2. Thus, both the yield patterns are to be examined.

Step 2. Value of y for yield pattern 1

The value of y is obtained from Eq.12.43 of Lesson 32, which is

4 Mx Ly y2 + 2 My Lx2 y – 3 My Lx2 Ly = 0
(12.43)

or 2y2 +2L y – 3L2 = 0


(1)

which gives y = 0.823L


(2)

Step 3. Determination of collapse load w kN/m2

Equations 12.41 and 12.42 of the method of segmental equilibrium


of Lesson 32 and Eq.12.48 of the method of virtual work of Lesson
32 shall be used to determine w. The results are given below.

(i) From Eq. 12.41: w = (24ML)/{3L2 (L– y) + L2y}= 17.722 (M /L2)


(3)
(ii) From Eq. 12.42: w = 12M/y2 = 17.722 (M/L2) (3)
2
(iii) From Eq. 12.48: w = {12 M (2y + L)} / {L(3yL – y )} = 17.722
(M/L2)
(3)

Step 4. Value of x for yield pattern 2

Version 2 CE IIT, Kharagpur


The value of x is obtained from Eq. 12.52 of Lesson 32, which is
6x2 + 4Lx – 3L2 = 0 (4)
which gives: x = 0.448L (5)

Step 5. Determination of collapse load w kN/m2

Equations 12.50 and 12.51 of the method of segmental equilibrium


of Lesson 32 and Eq. 12.57 for the method of virtual work of Lesson
32 shall be used to determine w. The results are given below:

(i) From Eq. 12.50: w = {6M(L2 – 2x2)} / x2L2 = 17.841 (M /L2) (6)
2 2
(ii) From Eq.12.51: w = 24 Mx / {L (3L-4x)}= 17.841 (M/L ) (6)
2 2 2
(iii) From Eq.12.57: w = (M/L ) {12 (L + 2x )}/ {x (3L – 2x)} = 17.841
(M/L2)
(6)

Step 6: Correct yield pattern and collapse load

Comparison of results of steps 3 and 5 reveals that yield pattern 1 is


the correct one giving the lower value of w. For this yield pattern, y =
0.823 L and w = 17.722 (M/L2).

Problem 2: Determine the uniformly distributed collapse load w kN/m2 of a


rectangular slab whose Ly/Lx = 3/4, simply supported at three edges
and free at the other edge. Assume My = 2Mx.

Solution 2: The slab is shown in Fig.12.33.1a. Given data are: Ly/Lx = 3/4 and
My/Mx = 2.

Step 1: To examine the possibility of yield patterns 1, 2 or both

Equation 12.45 of Lesson 32 stipulates that (My/Mx) 〈 4 (Ly/Ln)2 for


yield pattern 1 and Eq. 12.54 of Lesson 32 gives the condition that
(My/Mx) 〉 (4/3) (Ly/Lx)2 for yield pattern 2.

Here, Ly/Lx = 3/4 and My/Mx = 2 give (My/Mx) 〈 4 (Ly/Lx)2 and (My/Mx)
〉 (4/3) (Ly/Lx)2. Thus, both the yield patterns are to be examined.

Step 2: Value of y for yield pattern 1

The value of y is obtained from Eq.12.43 of Lesson 32, which gives:

Version 2 CE IIT, Kharagpur


6y2 + 8Lx – 9Lx2= 0 (7)

The solution of Eq. 7 is: y = 0.728 Lx (8)

Step 3: Determination of collapse load w kN/m2

Equations 12.41 and 12.42 of the method of segmental equilibrium


of Lesson 32 and Eq. 12.48 of the method of virtual work of Lesson
32 shall be used to determine w. The results are given below:

(i) From Eq. 12.41: w = 24Mx Ly/{3Lx2(Ly - y)+ Lx2y}= 22.657(Mx/Lx2)


(9)
(ii) From Eq. 12.42: w = 6My / y2 = 22.657 (Mx/Lx2)
(9)
(iii) From Eq. 12.48: w = (24MxLy y + 6My Lx2) / {Lx2 (3y Ly - y2)}
= 22.657 (Mx/Lx2) (9)

Step 4: Value of x for yield pattern 2

The value of x is obtained from Eq. 12.52 of Lesson 32, which is


6x2 + 2.25 Lx x – 1.6875 Lx2 = 0 (10)
The solution of the above equation is
x = 0.375 Lx (11)

Step 5: Determination of collapse load w kN/m2

Equations 12.50 and 12.51 of the method of segmental equilibrium


of Lesson 32 and Eq. 12.57 of the method of virtual work of Lesson
32 shall be used to determine w. The results are:

(i) From Eq. 12.50: w = 6(Mx Ly2 – My x2) /(x2Ly2)= 21.334 (Mx/Lx2)
(12)
(ii) From Eq. 12.51: w = 24 My x /{Ly2 (3Lx – 4x)}= 21.334 (Mx/Lx2)
(12)
(iii) From Eq. 12.57: w = 12{Mx Ly2+Myx2}/{Ly2 x(3Lx - 2x)}
= 21.334 (Mx/Lx2) (12)

Step 6: Correct yield pattern and collapse load

Comparison of results of steps 3 and 5 reveals that yield pattern 2 is


the correct one for which w is lower than that of yield pattern 1. For
yield pattern 2, x = 0.375 Lx and w = 21.334 (Mx / Lx2).

Version 2 CE IIT, Kharagpur


Problem 3: Determine the uniformly distributed collapse load w kN/m2 of a
square slab (L m × L m), simply supported at three edges and free
at the other edge. Assume My = Mx = M.

Solution 3: The slab is shown in Fig. 12.33.1a. Given data are:


Lx = Ly = L and My = Mx = M.

Step 1. To examine the possibility of yield patterns 1, 2 or both

From Eq. 12.45 of Lesson 32, it is seen that (My/Mx) < 4 (Ly/Ln)2. So,
yield pattern 1 has to be considered. But, (My/Mx) is not greater than
(4/3) (Ly/Lx)2 (vide Eq. 12.54 of Lesson 32). So, yield pattern 2 is not
to be considered.

Step 2. Value of y for yield pattern 1

Equation 12.43 of Lesson 32 gives: 4 y2 + 2Ly – 3L2 = 0 (13)


The solution of Eq.13 is: y = 0.651 L (14)

Step 3. Collapse load w kN/m2

(i) From Eq.12.41 of Lesson 32: w = (M/L2){(24)/(3 –2y)}=14.141(M/L2)


(15)
(ii) From Eq. 12.42 of Lesson 32: w = 6M/y2 = 14.141 (M/L2)
(15)
(iii) From Eq. 12.48 of Lesson 32: w = (M/ L2) {(24 y + 6)/{3y-y2)}=
14.141(M/L2) (15)

Problem 4: Determine the uniformly distributed collapse load w kN/m2 of a


rectangular slab whose Ly/Lx = 0.4, simply supported at three edges
and free at the other edge. Assume My/Mx = 1.

Solution 4: The slab is shown in Fig.12.33.1a. Given data are: Ly/Lx = 0.4 and
My/Mx = 1.

Step 1: To examine the possibility of yield patterns 1, 2 or both

Version 2 CE IIT, Kharagpur


Here, (My/Mx) is not less than 4 (Ly/Lx)2. So, yield pattern 1 is not to
be considered. However, (My/Mx) is 〉 (4/3) (Ly/Lx)2. So, yield pattern
2 has to be considered (vide Eqs. 12.45 and 12.54 of Lesson 32).

Step 2: Value of x for yield pattern 2

Equation 12.52 (Lesson 32) gives: 3x2 + 0.64 Lx2 – 0.48 Lx2 = 0 (16)
The solution of Eq. 16 is: x = 0.3073 Lx (17)

Step 3: Collapse load w kN/m2

(i) Eq.12.50 gives: w = (Mx/Lx2) {6(0.16-x2}/0.16x2} = 26.032 (Mx/Lx2)


(18)
(ii) Eq.12.51 gives: w = (Mx/Lx2){24x / 0.16(3-4x)}= 26.032 (Mx / Lx2)
(18)
(iii) Eq. 12.57 gives: w = (Mx/Lx2){12 (0.16+x2)/0.16(3x – 2x2)}
= 26.032 (Mx / Lx2) (18)
(Equations 12.50, 12.51 and 12.57 are from Lesson 32.)

Problem 5: Determine the correct yield pattern and the collapse point load P of
the isosceles triangular slab of Q.4b of sec.12.31.7 of Lesson 31
(Fig.12.31.10), if the load is applied at the centroid of the slab. The
slab is also shown in Fig. 12.33.2a having B = 6 m and 2L = 12 m.
Assume Mp = 9 kNm/m and Mn = 12 kNm/m. Use the method of
virtual work.

Version 2 CE IIT, Kharagpur


Version 2 CE IIT, Kharagpur
Solution 5: The slab is shown in Fig. 12.32.2a. Given data are: 2L=12 m, B = 6
m, Mp = 9 kNm/m and Mn = 12 kNm/m. In this problem, two possible
yield patterns, as shown in Figs.12.33.2b and c, are to be
considered. The lower of the two loads shall be taken, as the correct
collapse load and the corresponding yield pattern is the correct one.

Step 1: Yield pattern 1

Yield pattern 1, shown in Fig. 12.33.2b, divides the slab into three
segments. Assuming the deflection of the slab at the centroid G = Δ,
the rotation θ1 of segment DFG = θ1 = Δ/2 (as the distance GJ = 2
m). The length of the side DE = 6√2 m and the perpendicular

Version 2 CE IIT, Kharagpur


distance from G to ED is GH = EG sin 450 = 2√2 m. The rotation of
the segment DEG = rotation of the segment FEG = Δ/GH = Δ/2√2.

Step 2: TIW and TEW due to moments and loads

TIW = (Mn + Mp) {DF (θ1) + 2DE (θ2)} = 21{12(Δ/2) + 2(6√2) (Δ/2√2)}
= 252 Δ
So, TIW = 252 Δ (19)
TIW = P Δ (20)

Step 3: Collapse load P

Equating TIW and TEW from Eqs. 19 and 20, we have, P = 252 kN
(21)
Thus, the collapse load for the yield pattern 1 is 252 kN.

Step 4: Yield pattern 2

Yield pattern 2 divides the segment into a large number of sub-


segments as shown in Fig. 12.33.2c. The free body diagram of a
typical segment GSR is shown in Fig. 12.33.2d. The internal work
done by moments Mp and Mn of the segment GSR is given below.

Internal work done by moments


= Mp r dθ (Δ/r) + Mn r dθ (Δ/r) = (Mn + Mp) dθ Δ (22)
Total number of such segment = 2π/dθ (23)

So, TIW = (Mn + Mp) dθ Δ (2π/dθ) = (Mn + Mp) 2π (Δ) (24)

Total external work done by the load:

TEW = PΔ (25)

Equating TIW and TEW from Eqs. 24 and 25, we have


(Mn + Mp) 2π (Δ) = PΔ

which gives
P = (Mn + Mp) 2π = 42 π = 119.428 kN (26)

Step 5: Correct yield pattern and the collapse load

Version 2 CE IIT, Kharagpur


The correct yield pattern is the second one as it gives the lower
collapse load P =119.428 kN.

Problem 6: A circular slab of 6 m radius is shown in Fig. 12.33.3 which is


clamped along the circumference and has a column support at the
centre. Determine the yield pattern and uniformly distributed
collapse load of the slab if the positive and negative moments of
resistance Mp and Mn are 30 kNm/m. Use the method of virtual work.

Solution 6: Figure 12.33.3 shows the slab. Given data are R = 6 m and Mn = Mp
= 30 kNm/m.

The analysis of this type of slab is explained in sec. 12.32.6D of


Lesson 32 by the method of virtual work. We have Eqs.12.89 and
12.90 to determine the values of r and w (Fig. 12.32.7a to c). Using
those equations with reference to Fig. 12.32.7 of Lesson 32, we
have

(i) From Eq. 12.89: r = 0.2679 R = 1.6074 m (27)


(ii) From Eq. 12.90: w = 22.39 (M/R2) = 18.658 kN/m2 (28)

Version 2 CE IIT, Kharagpur


Problem 7: Determine the yield pattern and uniformly distributed collapse load
of the quadrantal slab of radius R = 6 m as shown in Fig.12.33.4,
which is clamped along the straight edges OA and OB, and free
along the curved edge ACB. Assume Mn = Mp= 30 kNm/m. Use the
method of segmental equilibrium for this problem.

Solution 7:

Step 1: Yield pattern

The slab and the yield pattern are shown in Fig. 12.33.4. Here, the
negative yield lines are along OA and OB, and the positive yield line
OC divides the slab into two symmetrical segments. The point G is
the centroid of the segment COA. The angle GOA = π/8. We know,

OG = (2R/3) (sin π/8 / π/8), GD = OG sin π/8 and area of the


segment COA = πR2/8
(29)

Step 2: Segmental equilibrium equation

Taking moment of the load of segment COA about OA and


considering Mn and Mp, we have
Mn (OA) + Mp (OC cos450) – w (π R2/8) (GD) = 0

Version 2 CE IIT, Kharagpur


or Mn (R) + Mp (R/√2) – w (πR2/8) (2R/3) (sin π/8 / π/8) sin π/8 = 0

or Mn +Mp/√2 – (2 w R2/3) sin 2 (π/8) = 0 (30)

The value of the collapse load w can be determined from known


values of Mn , Mp and R from Eq.30.

Let us assume:
Mn = k Mp (31)

Equation 30 then gives


w = {3 (√2 k +1) Mp} /{R2(√2 –1)} (32)

For this particular problem k = 1 and R = 6 m.


Thus, from Eq. 32, we have:

w = 14.571 kN/m2 (33)

12.33.3 Practice Questions and Problems with Answers


Q.1: Determine the uniformly distributed collapse load w kN/m2 of a
circular slab of radius = 6 m simply supported along the periphery, if
Mp = 30 kNm/m. Use the method of segmental equilibrium.

A.1: The analysis of such slabs is explained in sec. 12.32.6B of Lesson


32 employing the method of segmental equilibrium with reference to
Fig. 12.32.5a and b. Equation 12.82 gives the value of the uniformly
distributed load, which is

w = 6Mp/R2 = 6(30)/(36) = 5 kN/m2 (34)

Therefore, the collapse load of this slab is 5 kN/m2.

Q.2: Determine the uniformly distributed collapse load w kN/m2 of a


circular slab of radius = 6 m and clamped along the periphery, if Mp
= Mn = 30 kNm/m. Use the method of segmental equilibrium.

A.2: The analysis of such slabs is explained in sec. 12.32.6C of Lesson


32 employing the method of segmental equilibrium with reference to
Figs.12.32.6a and b. Equation 12.83 gives the value of the uniformly
distributed load, which is

Version 2 CE IIT, Kharagpur


w = 6 (Mn + Mp) / R2 = 60(60)/36 = 10 kN/m2 (35)
Therefore, the collapse load of this slab is 10 kN/m2.

Q.3: Determine the uniformly distributed collapse load of a 6 m × 6 m


square slab clamped along four edges and has the moment of
resistance Mpx = Mpy = Mnx = Mny = Mp = Mn = 30 kNm/m.

Version 2 CE IIT, Kharagpur


A.3: For this problem, we have to examine three yield patterns 1, 2 and
3, shown in Figs.12.33.5a, b and c, respectively.

Step 1: Selection of yield patterns

In this problem, (Ly / Lx)2 = 1 and (Mpy + Mny)/ (Mpx +Mnx) = 1. So,
yield pattern 1 has to be considered. Since, (Mpy + Mny) / (Mpx + Mnx)
is not greater than (4/3) (Ly/Lx)2, yield pattern 2 is not possible. Yield
pattern 3 is also possible. So, we have to examine yield patterns 1
and 3.

Step 2: Yield pattern 1

Equation 12.26 of Lesson 31 gives the value of w for yield pattern 1,


which is

w = 12{(Mpx + Mnx)/(Lx)2 + (Mpy + Mny)/(Ly)2


(12.26)

= 12 {60/36 + 60/36} = 40 kN/m2

Thus, w = 40 kN/m2 for yield pattern 1

Step 3: Yield pattern 3

Version 2 CE IIT, Kharagpur


Equation 12.83 of Lesson 32 gives the value of load w for yield
pattern 3, which is:

w = 6(Mn + Mp) / R2 (12.83)

Using Mn = Mp = 30 kNm/m as R = 3 m, we have w = 40 kN/m2.

Therefore, the collapse load of this slab is 40 kN/m2 either for the
yield pattern 1 or for the yield pattern 3.

12.33.4 Reference
1. Reinforced Concrete Limit State Design, 6th Edition, by Ashok K. Jain,
Nem Chand & Bros, Roorkee, 2002.
2. Limit State Design of Reinforced Concrete, 2nd Edition, by P.C. Varghese,
Prentice-Hall of India Pvt. Ltd., New Delhi, 2002.
3. Advanced Reinforced Concrete Design, by P.C. Varghese, Prentice-Hall
of India Pvt. Ltd., New Delhi, 2001.
4. Reinforced Concrete Design, 2nd Edition, by S. Unnikrishna Pillai and
Devdas Menon, Tata McGraw-Hill Publishing Company Limited, New
Delhi, 2003.
5. Limit State Design of Reinforced Concrete Structures, by P. Dayaratnam,
Oxford & I.B.H. Publishing Company Pvt. Ltd., New Delhi, 2004.
6. Reinforced Concrete Design, 1st Revised Edition, by S.N. Sinha, Tata
McGraw-Hill Publishing Company. New Delhi, 1990.
7. Reinforced Concrete, 6th Edition, by S.K. Mallick and A.P. Gupta, Oxford &
IBH Publishing Co. Pvt. Ltd. New Delhi, 1996.
8. Behaviour, Analysis & Design of Reinforced Concrete Structural Elements,
by I.C.Syal and R.K.Ummat, A.H.Wheeler & Co. Ltd., Allahabad, 1989.
9. Reinforced Concrete Structures, 3rd Edition, by I.C.Syal and A.K.Goel,
A.H.Wheeler & Co. Ltd., Allahabad, 1992.
10. Textbook of R.C.C, by G.S.Birdie and J.S.Birdie, Wiley Eastern Limited,
New Delhi, 1993.
11. Design of Concrete Structures, 13th Edition, by Arthur H. Nilson, David
Darwin and Charles W. Dolan, Tata McGraw-Hill Publishing Company
Limited, New Delhi, 2004.
12. Concrete Technology, by A.M.Neville and J.J.Brooks, ELBS with
Longman, 1994.
13. Properties of Concrete, 4th Edition, 1st Indian reprint, by A.M.Neville,
Longman, 2000.
14. Reinforced Concrete Designer’s Handbook, 10th Edition, by C.E.Reynolds
and J.C. Steedman, E & FN SPON, London, 1997.

Version 2 CE IIT, Kharagpur


15. Indian Standard Plain and Reinforced Concrete – Code of Practice (4th
Revision), IS 456: 2000, BIS, New Delhi.
16. Design Aids for Reinforced Concrete to IS: 456 – 1978, BIS, New Delhi.

12.33.5 Test 33 with Solutions


Maximum Marks = 50, Maximum Time = 30 minutes

Answer all questions.

TQ.1: Determine the uniformly distributed collapse load of a rectangular


slab (Figs. 12.33.1a, b and c) having Ly/Lx = 0.5, simply supported at
three edges and free at the other edge. Assume My = 0.5 Mx.

A.TQ.1: The slab is shown in Figs. 12.33.1a, b and c. Given data are: Ly/Lx =
0.5 and My/Mx = 0.5.

Step 1: To examine the possibility of yield patterns 1, 2 or both

Here, (My/Mx) 〈 4(Ly/Lx)2. So, yield pattern 1 is possible. Again


(My/Mx) 〉 (4/3) (Ly/Lx)2 . So, yield pattern 2 is also possible. Thus,
both yield patterns 1 and 2 are to be examined.

Step 2: Value of y for yield pattern 1

Equation 12.43 of Lesson 32 gives: 2y2 + Lx y – 0.75 Lx2 = 0, which


gives,
y = 0.4114 Lx (36)

Step 3: Determination of collapse load w kN/m2

(i) Eq. 12.41 gives: w = (Mx/Lx2) {12/(1.5-2y)} = 17.722(Mx/Lx2)


(37)
(ii) Eq. 12.42 gives: w = (Mx/Lx2) (3/y2) = 17.722 (Mx / Lx2)
(37)
(iii) Eq. 12.48 gives: w = (Mx/Lx2) {(12y + 3) / y(1.5-y)}
= 17.722 (Mx/Lx2) (37)
(Equations 12.41, 12.42 and 12.48 are from Lesson 32.)

Step 4: Value of x for yield pattern 2

The value of x is obtained from Eq. 12.52 of Lesson 32, which is,
1.5 x2 + Lx x – 0.75 Lx2 = 0 (38)

Version 2 CE IIT, Kharagpur


The solution of Eq. 38 is:

x = 0.448 Lx (39)

Step 5: Determination of collapse load w kN/m2

(i) Eq. 12.50 of Lesson 32 gives: w = 17.841 (Mx/Lx2)


(40)
(ii) Eq. 12.51 of Lesson 32 gives: w = 17.841 (Mx/Lx2)
(40)
(iii) Eq. 12.57 of Lesson 32 gives: w = 17.841 (Mx/Lx2) (40)

Step 6: Correct yield pattern and collapse load

Comparison of results of Steps 3 and 5 reveals that the yield


pattern 1 is the correct one giving lower value of w = 17.722
(Mx/Lx2).

12.33.6 Summary of this Lesson


This lesson explains the different types of yield pattern of triangular,
rectangular, quadrantal and circular slabs through several numerical problems.
Both the methods of segmental equilibrium and virtual work are employed. The
loadings considered are either point load or uniformly distributed load. All
possible yield patterns of a particular problem are examined to select the correct
one and the corresponding lower/lowest load for the slab problems. The
moments of resistance of the slab in the two directions are also varied in the
problems. Different types of support conditions are taken up in the numerical
problems. In addition to illustrative examples, practice problems and test problem
will help in understanding the theories and their applications in analysing slab
problems with different types of support conditions and loadings.

Version 2 CE IIT, Kharagpur


Module
13
Working Stress Method
Version 2 CE IIT, Kharagpur
Lesson
34
Rectangular Beams under
Flexure
Version 2 CE IIT, Kharagpur
Instructional Objectives:
At the end of this lesson, the student should be able to:

• explain the philosophy of the design by working stress method,

• explain the concepts of permissible stresses in concrete and steel and the
factors of safety in this method,

• state the limit of increase of permissible stresses under specific loading


considerations,

• state the assumptions for the design of flexural members employing working
stress method,

• explain the concept of modular ratio,

• derive the governing equations of rectangular, balanced and under-reinforced


(singly-reinforced) sections,

• name the two types of problems for the designer,

• write down the steps separately for the two types of problems in the working
stress method,

• justify the need for doubly-reinforced beams, and

• derive the governing equations of the doubly-reinforced rectangular flexural


members.

13.34.1 Introduction
Design of reinforced concrete structures started in the beginning of this
century following purely empirical approach. Thereafter came the so called
rigorous elastic theory where it is assumed that concrete is elastic and reinforcing
steel bars and concrete act together elastically. The load-deflection relation is
linear and both concrete and steel obey Hooke’s law.

The method is designated as working stress method as the loads for the
design of structures are the service loads or the working loads. The failure of the
structure will occur at a much higher load. The ratio of the failure loads to the
working loads is the factor of safety. Accordingly, the stresses of concrete and
steel in a structure designed by the working stress method are not allowed to
exceed some specified values of stresses known as permissible stresses. The

Version 2 CE IIT, Kharagpur


permissible stresses are determined dividing the characteristic strength fck of
the material by the respective factor of safety. The values of the factor of safety
depend on the grade of the material and the type of stress. Thus, for concrete in
bending compression, the permissible stress of grade M 20 is 7 N/mm2, which is
obtained by dividing the characteristic strength fck of M 20 concrete by a number
3 and then rationalising the value to 7. This permissible stress is designated by
σcbc, the symbol σ stands for permissible stress and the letters c, b and c mean
concrete in bending compression, respectively.

13.34.2 Permissible Stresses in Concrete


The permissible stress of concrete in direct tension is denoted by σtd. The
values of σtd for member in direct tension for different grades of concrete are
given in cl. B-2.1.1 of IS 456. However, for members in tension, full tension is to
be taken by the reinforcement alone. Though full tension is taken by the
reinforcement only, the actual tensile stress of concrete ftd in such members shall
not exceed the respective permissible values of σtd to prevent any crack. Table
13.1 presents the values of σtd for selective grades of concrete as a ready
reference. It may be worth noting that the factor of safety of concrete in direct
tension is from 8.5 to 9.5.

The permissible stresses of concrete in bending compression σcbc, in


direct compression σcc and the average bond for plain bars in tension τbd are
given in Table 21 of IS 456 for different grades of concrete. However, Table 13.1
presents these values for selective grades of concrete, as a ready reference. The
factors of safety of concrete in bending compression, direct compression and
average bond for plain bars are 3, 4 and from 25 to 35, respectively. For plain
bars in compression, the values of average bond stress are obtained by
increasing the respective value in tension by 25 percent, as given in the note of
Table 21 of IS 456. For deformed bars, the values of Table 21 are to be
increased by sixty per cent, as stipulated in cl. B-2.1.2 of IS 456.

Table 13.1 Permissible stresses in concrete

Grade of Direct tension Bending Direct Average bond


concrete σtd (N/mm2) compression compression τbd for plain
σcbc (N/mm2) σcc (N/mm2) bars in
tension
(N/mm2)
M 20 2.8 7.0 5.0 0.8
M 25 3.2 8.5 6.0 0.9
M 30 3.6 10.0 8.0 1.0

Version 2 CE IIT, Kharagpur


M 35 4.0 11.5 9.0 1.1
M 40 4.4 13.0 10.0 1.2

From the values of the permissible stresses and the respective


characteristic strengths for different grades of concrete, it may be seen that the
factors of safety of concrete in direct tension, bending compression, direct
compression and average bond for plain bars in tension are from 8.5 to 9.5, 3, 4
and from 25 to 35, respectively.

13.34.3 Permissible Stresses in Steel Reinforcement


Permissible stresses in steel reinforcement for different grades of steel,
diameters of bars and the types of stress in steel reinforcement are given in
Table 22 of IS 456. Selective values of permissible stresses of steel of grade Fe
250 (mild steel) and Fe 415 (high yield strength deformed bars) in tension (σst
and σsh) and compression in column (σsc) are furnished in Table 13.2 as a ready
reference. It may be noted from the values of Table 13.2 that the factor of safety
in steel for these stresses is about 1.8, much lower than concrete due to high
quality control during the production of steel in the industry in comparison to
preparing of concrete.

Table 13.2: Permissible stresses in steel reinforcement

Type of stress in steel Mild steel bars, Fe 250, High yield strength
reinforcement (N/mm2) deformed bars, Fe 415,
(N/mm2)
Tension σst or σss
(a) up to and including 140 230
20 mm diameter
(b) over 20 mm diameter 130 230
Compression in column 130 190
bars σsc

13.34.4 Permissible Shear Stress in Concrete τc


Permissible shear stress in concrete in beams without any shear
reinforcement depends on the grade of concrete and the percentage of main
tensile reinforcement in beams. Table 23 of IS 456 furnishes the values of τc for
wide range of percentage of tensile steel reinforcement for different grades
concrete. Other relevant clauses regarding the permissible shear stress of
concrete are given in cls.B-5.2.1.1, B-5.2.2 and B-5.2.3 of IS 456.

Version 2 CE IIT, Kharagpur


13.34.5 Increase in Permissible Stresses
Clause B-2.3 of IS 456 recommends the increase of permissible stresses
of concrete and steel given in Tables 21 to 23 up to a limit of 33.33 per cent,
where stresses due to wind (or earthquake), temperature and shrinkage effects
are combined with those due to dead, live and impact loads.

13.34.6 Assumptions for Design of Members by Working


Stress Method
As mentioned earlier, the working stress method is based on elastic
theory, where the following assumptions are made, as specified in cl. B-1.3 of IS
456.

(a) Plane sections before bending remain plane after bending.

(b) Normally, concrete is not considered for taking the tensile stresses except
otherwise specifically permitted. Therefore, all tensile stresses are taken
up by reinforcement only.

(c) The stress-strain relationship of steel and concrete is a straight line under
working loads.

(d) The modular ratio m has the value of 280/3σcbc, where σcbc is the
permissible compressive stress in concrete due to bending in N/mm2. The
values of σcbc are given in Table 21 of IS 456. The modular ratio is
explained in the next section.

13.34.7 Modular Ratio m


In the elastic theory, structures having different materials are made
equivalent to one common material. In the reinforced concrete structure,
concrete and reinforcing steel are, therefore, converted into one material. This is
done by transformation using the modular ratio m which is the ratio of modulus of
elasticity of steel and concrete. Thus, m = Es/Ec. where Es is the modulus of
elasticity of steel which is 200000 N/mm2. However, concrete has different
moduli, as it is not a perfectly elastic material.
2
The short-term modulus of concrete Ec = 5000 f ck in N/mm , where fck is
the characteristic strength of concrete. However, the short-term modulus does
not take into account the effects of creep, shrinkage and other long-term effects.
Accordingly, the modular ratio m is not computed as m = Es/Ec = 200000/(5000

Version 2 CE IIT, Kharagpur


f ck ). The value of m, as given in sec. 13.34.6 d, i.e., 280/3σcbc, partially takes
into account long-term effects. This is also mentioned in the note of cl. B-1.3 of IS
456.

13.34.8 Flexural Members – Singly Reinforced Sections

A simply supported beam subjected to two point loads shall have pure
moment and no shear in the middle-third zone, as shown in Fig. 1.1.1 of Lesson
1.The cross-sections of the beam in this zone are under pure flexure. Figures
13.34.1a, b and c show the cross-section of a singly-reinforced beam, strain
profile and stress distribution across the depth of the beam, respectively due to
the loads applied on the beam. Most of the symbols are used in earlier lessons.
The new symbols are explained below.

x = kd = depth of the neutral axis, where k is a factor,

fcbc = actual stress of concrete in bending compression at the top fibre which
should not exceed the respective permissible stress of concrete in bending
compression σcbc,

fst = actual stress of steel at the level of centroid of steel which should not
exceed the respective permissible stress of steel in tension σst,

jd = d(1-k/3) = lever arm i.e., the distance between lines of action of total
compressive and tensile forces C and T , respectively.

Version 2 CE IIT, Kharagpur


Figures 13.34.1b and c show linear strain profile and stress distribution,
respectively. However, the value of the stress at the level of centroid of steel of
Fig. 13.34.1 c is fst /m due to the transformation of steel into equivalent concrete
of area mAst.

13.34.9 Balanced Section – Singly-Reinforced

In a balanced cross-section both fcbc and fst reach their respective


permissible values of σcbc and σst at the same time as shown in Fig.13.34.2c. The
depth of neutral axis is xb = kbd. From the stress distribution of Fig.13.34.2c, we
have

σst /m = σcbc (1-kb) / kb (13.1)

An expression of kb is obtained by substituting the expression of m as

m = 280 / 3σcbc (13.2)

into Eq. 13.1. This gives

kb = 93.33 / (σst + 93.33) (13.3)

Equation 13.3 shows that the value of kb for balanced section depends only on
σst . It is independent of σcbc.

The lever arm, jb d = (1- kb/3) d (13.4)

Version 2 CE IIT, Kharagpur


The expressions of total compressive and tensile forces, C and T are:

C = (1/2) σcbc b xb = (1/2) σcbc b kb d


(13.5)

T = Ast σst (13.6)

The total compressive force is acting at a depth of xb/3 from the top fibre of the
section.

The moment of resistance of the balanced cross-section, Mb is obtained by


taking moment of the total compressive force C about the centroid of steel or
moment of the tensile force T about the line of action of the total compressive
force C. Thus,

M b = C( jb d ) = ( 1 / 2 )σ cbc kb jb ( bd 2 ) (13.7)
or, M b = T( jb d ) = σ st Ast jb d = ( pt ,bal /100)σ st jb (bd )
2
(13.8)
as Ast = pt ,bal ( bd / 100 ) (13.9)

where pt,bal = balanced percentage of steel


From Eqs.13.7 and 13.8, we can write

Mb = Rb bd2
(13.10)

where Rb = (1/2) σcbc kb jb = (pt,bal /100) σst jb


(13.11)

and jb = 1- (kb/3)
(13.12)

The expression of the balanced percentage of steel pt,bal is obtained by


equating the total compressive force C to the tensile force T from Eqs. 13.5 and
13.6. This gives,

Ast σst = (σcbc/2) b kb d, which gives: pt,bal (bd/100) σst = (σcbc/2) b kbd

or pt,bal = 50 kb(σcbc/σst)
(13.13)

It is always desirable, though may not be possible in most cases, to


design the beam as balanced since the actual stresses of concrete fcbc at the top
compression fibre and steel at the centroid of steel should reach their respective
permissible stresses σcbc and σst in this case only. The procedure of the design is
given below.

Version 2 CE IIT, Kharagpur


Treating the design moment as the balanced moment of resistance Mb
and assuming the width, b of the beam as 250 mm, 300 mm or 350 mm, the
effective depth d is obtained from Eq.13.10. The required balanced area of steel
Ast is then obtained from Eq. 13.9 getting the values of kb from Eq. 13.3 and then
pt,bal from Eq. 13.13.

The values of Rb, the moment of resistance factor Mb/bd2 are obtained
from Eq. 13.11 for different values of σcbc and σst (different grades of concrete
and steel) and are presented in Table 13.3. Similarly, the values of balanced
percentage of tensile reinforcement, pt, bal obtained form Eq. 13.13 for different
grades of concrete and steel are given in Table 13.4.

Table 13.3 Moment of resistance factor Rb in N/mm2 for balanced rectangular


section.

σcbc σst (N/mm2)


(N/mm2) 140 230 275
7.0 1.21 0.91 0.81
8.5 1.47 1.11 0.99
10.0 1.73 1.30 1.16

Table 13.4 Percentage of tensile reinforcement pt,bal for singly-reinforced


balanced section.

σcbc σst (N/mm2)


(N/mm2) 140 230 275
7.0 1.0 0.44 0.32
8.5 1.21 0.53 0.39
10.0 1.43 0.63 0.46

Values of Rb and pt.bal from Tables 13.3 and 13.4 reveal the following:

1. For given values of width and effective depth, b and d of a rectangular


section, the balanced moment of resistance Mb increases with higher
grade of concrete for a particular grade of steel. However, the balanced
moment of resistance decreases with higher grade of steel for a particular
grade of concrete.

2. The balanced percentage of steel, pt.bal increases with the increase of


grade of concrete for a particular grade of steel for given values of width
and effective depth of a rectangular section. On the other hand, the

Version 2 CE IIT, Kharagpur


balanced percentage of steel, pt.bal decreases with the increased grade of
steel for a particular grade of concrete.

As mentioned earlier, it may not be possible to design a balanced section


since the area of steel required for the balanced condition is difficult to satisfy
with available bar diameters. In such cases, it is essential that the beam should
be provided with the steel less than the balanced steel so that the actual stress of
steel in tension reaches the permissible value σst and the actual stress of
concrete fcbc is less than the permissible value. Such sections are designated as
under-reinforced sections and moment of resistance shall be governed by the
tensile stress of steel σst, which is known. The depth of the neutral axis will be
less than the balanced depth of the neutral axis, as shown in Fig. 13.34.3b. The
relevant equations for the design of under-reinforced section are established in
the next section.

13.34.10 Under-reinforced Section -- Singly Reinforced


Figure 13.34.3a shows the cross-section where x (= kd) is the depth of the
neutral axis. The depth of the neutral axis is determined by taking moment of the
area of concrete in compression (= bx) and the transformed area of steel (= mAst)
about the neutral axis, which gives

b kd (kd/2) = m (pt bd/100) (d-kd)

Version 2 CE IIT, Kharagpur


or k2 + (pt mk/50) - (pt m/50) = 0
(13.14)

Equation 13.14 has two roots of k given by

k= - (pt m/100) ± {(pt m/100)2+ (pt m/50)}1/2


(13.15)

Since k cannot be negative, we have the positive root to be considered as

k= - (ptm /100) +{(pt m /100)2 + (pt m/50)}1/2


(13.16)

The moment of resistance of the under-reinforced section is obtained from

M = T (lever arm) = Ast σst d(1 - k/3) = (pt bd /100) σst d (1 - k/3)

Therefore, we have:

M = (pt/100)σst (1 - k/3) bd2


(13.17)

which can also be expressed as

M = R bd2
(13.18)

where
R = (pt/100) σst (1 – k/3)
(13.19)
is the moment of resistance factor M/bd2. The values of R are obtained for given
values of pt for different grades of steel and concrete. Tables 68 to 71 of SP-16
furnish the values of R for four grades concrete and five values of σst.

The actual stress of concrete at the top fibre fcbc shall not reach σcbc in
under- reinforced sections. The actual stress fcbc is determined from the equation
C=T as explained below:

With reference to Fig. 13.34.3c, the compressive force C of concrete and


tensile for T of steel are:

C = (1/2) fcbc b kd
(13.20)

T = σst Ast (13.6)

Version 2 CE IIT, Kharagpur


For the tensile force, the actual stress of steel fst shall reach the value of
σst. So, we are using Eq. 13.6, the same equation as for the balanced section.
Equating C and T from Eqs. 13.20 and 13.6, we get

(1/2)fcbc b kd = σst Ast

or, fcbc = 2 σst Ast / b k d


(13.21)
Expressing Ast = pt bd /100
(13.22)

and using Eq. 13.22 in Eq. 13.21, we get

fcbc = pt σst / 50 k
(13.23)

The two types of problems: (i) Analysis type and (ii) Design type are now taken
up in the following sections.

13.34.11 Analysis Type of Problems


For the purpose of analysing a singly-reinforced beam where the working
loads, area of steel, b and d of the cross section are given, the actual stresses of
concrete at the top fibre and steel at the centroid of steel are to be determined in
the following manner.

Step 1: To determine the depth of the neutral axis kd from Eq. 13.16.

Step 2: The beam is under-reinforced, balanced or over-reinforced, if k is


less than, equal to or greater than kb, to be obtained from Eq. 13.3.

Step 3: The actual compressive stress of concrete fcbc and tensile stress of
steel at the centroid of steel fst are determined in the following manner for
the three cases of Step 2.

Case (i) When k < kb (under-reinforced section)

From the moment equation, we have

M= Ast fst d(1 - k/3)

or fst =M /{Ast d(1 - k/3)}


(13.24)

Version 2 CE IIT, Kharagpur


where M is obtained from the given load, Ast and d are given, and k is
determined in Step 1.

Equating C = T, we have: (1/2) fcbc b(kd) = Ast fst

or fcbc = (2 Ast fst) / (b kd)


(13.25)

where Ast, b and d are given and k and fst are determined in steps 1 and 2,
respectively.

Case (ii) When k = kb (balanced section)

In the balanced section fcbc = σcbc and fst = σst.

Case (iii) When k > kb (over-reinforced section)

Such beams are not to be used as in this case fcbc shall reach σcbc while fst
shall not reach σst. These sections are to be redesigned either by increasing the
depth of the beam or by providing compression reinforcement. Beams with
compression and tension reinforcement are known as doubly-reinforced beam
and is taken up in sec. 13.34.13.

13.34.12 Design Type of Problems


It has been explained earlier in sec.13.34.9 that it is difficult, though
desirable, to design a balanced section, as the concrete and steel should attain
their respective permissible values at the same time. Therefore, a practical
design shall preferably be under-reinforced. However, the given data are the
working load, span and support conditions of the beam. These data shall enable
to get the design moment of the section. The breadth b, effective depth d and
area of steel are to be determined in these problems. The steps to be followed
are given below.

Step 1. Selection of preliminary dimension b and D

The width is normally taken as 250 mm, 300 mm or 350 mm so that the
reinforcing bars can be accommodated without difficulty. The total depth of the
beam may be supplied in architectural drawings or may be estimated from the
span to depth ratios, as stiputed in cl. 23.2.1 of IS 456 for the control of
deflection. However, these values should be revised, if needed.

Step 2. Determination of d

Version 2 CE IIT, Kharagpur


The effective depth of a balanced section is first obtained from Eq. 13.10,
where the value of Rb is taken from Table 13.3 after finalising the grades of
concrete and steel. This balanced effective and total depths of beam, as given in
the architectural drawings or may be needed for the control of deflection, give
some idea while finalising the depth of the beam, higher than the balanced depth,
so that the beam becomes under-reinforced.

Step 3. Determination of Ast

First, the balanced percentage of steel pt,bal is taken from Table 13.4 for
the selected grades of concrete and steel. The amount of increase of the depth
of the beam of step 2 may also depend on the amount pt, bal. In fact, there are
sets of values of depth and the respective area of steel, to be selected judiciously
on the basis of practical considerations.

Step 4. Checking for the stresses

After finalising the depth and area of steel (depth larger than the balanced
depth and area of steel lower than the balanced area of steel), the section is to
be analysed as an analysis type of problem, as explained in sec. 13.34.11.

Alternatively, appropriate table from Tables 63 to 71 of SP-16 may be


used for getting several possible combinations quickly.

13.34.13 Doubly-Reinforced Beams


In some situations, it may not be possible to provide the required depth of
the beam as singly-reinforced either due to restrictions given by the architects or
due to functional requirements. In such cases, the designer has to satisfy the
structural requirement with a depth less than that of a singly-reinforced section.
This is done by providing steel reinforcing bars in the compression side along
with the tensile steel in the tension side of the beam. These beams are
designated as doubly-reinforced beams.

Version 2 CE IIT, Kharagpur


Figures 13.34.4a to c show the cross-section, strain profile and stress
distribution of a doubly-reinforced section. Since, the design moment is more
than the balanced moment of resistance of the section, we have

M = Mb + M′
(13.26)

The additional moment M′ is resisted by providing compression


reinforcement Asc (= pcbd/100) and additional tensile reinforcement Ast2.
The modular ratio of the compression steel is taken as 1.5 m, where m is the
modular ratio as explained in sec. 13.34.7.

Figure 13.34.4c shows that the stress of concrete at the level of


compression steel is σcbc (kbd - d′) / kdd. Accordingly, the stress in the
compression steel reinforcement is 1.5m σcbc (kbd - d′) / kbd.

Version 2 CE IIT, Kharagpur


Figure 13.34.4d and e present separate stress distribution for the
balanced beam (shown in Fig. 13.34.2c) and the compressive and tensile forces
of compressive and tensile reinforcing bars C2 and T2, respectively. The
expression of the additional moment M′ is obtained by multiplying C2 and T2 with
the lever arm (d - d′ ), where d′ is the distance of the centroid of compression
steel from the top fibre. We have, therefore,

C2 = Asc (1.5m –1) σcbc (kd - d′) / kd


(13.27)

T2 = (pt – pt, bal) (bd / 100) σst


(13.28)

M′ = C2 (d - d′) = (pc bd /100) (1.5m – 1) σcbc (kd - d′ )/ kd (d- d′ )

or, M′ = (pc / 100)(1.5m – 1) σcbc (1- d′ / kd)(1- d′/d)bd2


(13.29)

also, M′ = T2 (d - d′ ) d = (pt –pt, bal) (bd / 100)σst (d - d′ )

or, M′ = (pt – pt, bal) / 100 σst (1- d′/d) bd2


(13.30)

Equating T2 = C2 from Eqs. 13.28 and 13.27, we have

(pt – pt, bal) σst = pc (1.5m – 1) σcbc (1- d′ / kd)


(13.31)

The total moment M is obtained by adding Mbal and M′, as given below:

M = Mbal + (pt – pt, bal) / 100 σst (1- d′ / d) bd2


(13.32)

The total tensile reinforcement Ast has two components Ast1 + Ast2 for Mbal and M′,
respectively. The equation of Ast is:

Ast = Ast 1 + Ast 2


(13.33)

where Ast1 = pt bal (bd / 100)


(13.34)

and Ast 2 = M′ / σst (d - d′)


(13.35)

Version 2 CE IIT, Kharagpur


The compression reinforcement Asc is expressed as a ratio of additional
tensile reinforcement Ast2, as given below:

(Asc / Ast2) = {pc /(pt – pt bal )}

or, (Asc / Ast2) = σst / {σcbc (1.5m – 1) (1 - d′ / kd)}


(13.36)

Table M of SP-16 presents the values of Ast /Ast2 for different values of d′ /
d and σcbc for two values of σst =140 N/mm2 and 230 N/mm2. Selective values are
furnished in Table 13.5 as a ready reference. Tables 72 to 79 of SP-16 provide
values of pt and pc for four values of d′ / d against M/bd2 for four grades of
concrete and two grades of steel.

Table 13.5: Values of Asc / Ast 2

d′ / d
σst σcbc
(N/mm2) (N/mm2) 0.05 0.10 0.15 0.20
7.0 1.20 1.40 1.68 2.11
140
8.5 1.22 1.42 1.70 2.13
10.0 1.23 1.44 1.72 2.15
7.0 2.09 2.65 3.60 5.54
230 8.5 2.12 2.68 3.64 5.63
10.0 2.14 2.71 3.68 5.76

13.34.14 Practice Questions and Problems with Answers


Q.1: Justify the name working stress method of design.

A.1: Paragraph 2 of Sec. 13.34.1

Q.2: How are the permissible stresses of concrete in direct tension, bending
compression, direct compression and average bond for plain bars in
tension related to the factor of safety in the working stress method of
design?

A.2: Sec. 13.14.2

Q.3: How is the permissible stress of steel in tension related to the factor of
safety in the working stress method of design?

Version 2 CE IIT, Kharagpur


A.3: Sec. 13.34.3

Q.4: Is it possible to increase the permissible stress? If yes, when is it done?

A.4: Sec. 13.34.5

Q.5: State the assumption for the design of members by working stress
method.

A.5: Sec. 13.34.6

Q.6: Explain the concept of modular ratio.

A.6: Sec. 13.34.7

Q.7: Draw a cross-section of a singly-reinforced rectangular beam, the strain


and stress distributions along the depth of the section.

A.7: Figs. 13.34.1a, b and c

Q.8: What do you mean by balanced rectangular beam? Establish the


equations for determining the moment of resistance and percentage of
tension steel in a balanced rectangular beam.

A.8: Sec. 13.34.9

Q.9: Establish the equations for determining the depth of neutral axis, moment
of resistance and area of tension steel of an under-reinforced rectangular
beam.

A.9: Sec. 13.34.10

Q.10: Write down the steps for solving the analysis type of problems of singly-
reinforced rectangular beams.

A.10: Sec. 13.34.11

Q.11: Write down the steps for solving the design type of problems of singly-
reinforced rectangular beams.

A.11: Sec. 13.34.12

Q.12: When do we go for doubly-reinforced beams? Establish the equation for


determining areas of steel for the doubly reinforced rectangular beams.

A.12: Sec. 13.34.13

Version 2 CE IIT, Kharagpur


13.34.15 References
1. Reinforced Concrete Limit State Design, 6th Edition, by Ashok K. Jain,
Nem Chand & Bros, Roorkee, 2002.
2. Limit State Design of Reinforced Concrete, 2nd Edition, by P.C. Varghese,
Prentice-Hall of India Pvt. Ltd., New Delhi, 2002.
3. Advanced Reinforced Concrete Design, by P.C. Varghese, Prentice-Hall
of India Pvt. Ltd., New Delhi, 2001.
4. Reinforced Concrete Design, 2nd Edition, by S. Unnikrishna Pillai and
Devdas Menon, Tata McGraw-Hill Publishing Company Limited, New
Delhi, 2003.
5. Limit State Design of Reinforced Concrete Structures, by P. Dayaratnam,
Oxford & I.B.H. Publishing Company Pvt. Ltd., New Delhi, 2004.
6. Reinforced Concrete Design, 1st Revised Edition, by S.N. Sinha, Tata
McGraw-Hill Publishing Company. New Delhi, 1990.
7. Reinforced Concrete, 6th Edition, by S.K. Mallick and A.P. Gupta, Oxford &
IBH Publishing Co. Pvt. Ltd. New Delhi, 1996.
8. Behaviour, Analysis & Design of Reinforced Concrete Structural Elements,
by I.C.Syal and R.K.Ummat, A.H.Wheeler & Co. Ltd., Allahabad, 1989.
9. Reinforced Concrete Structures, 3rd Edition, by I.C.Syal and A.K.Goel,
A.H.Wheeler & Co. Ltd., Allahabad, 1992.
10. Textbook of R.C.C, by G.S.Birdie and J.S.Birdie, Wiley Eastern Limited,
New Delhi, 1993.
11. Design of Concrete Structures, 13th Edition, by Arthur H. Nilson, David
Darwin and Charles W. Dolan, Tata McGraw-Hill Publishing Company
Limited, New Delhi, 2004.
12. Concrete Technology, by A.M.Neville and J.J.Brooks, ELBS with
Longman, 1994.
13. Properties of Concrete, 4th Edition, 1st Indian reprint, by A.M.Neville,
Longman, 2000.
14. Reinforced Concrete Designer’s Handbook, 10th Edition, by C.E.Reynolds
and J.C. Steedman, E & FN SPON, London, 1997.
15. Indian Standard Plain and Reinforced Concrete – Code of Practice (4th
Revision), IS 456: 2000, BIS, New Delhi.
16. Design Aids for Reinforced Concrete to IS: 456 – 1978, BIS, New Delhi.

13.34.16 Test 34 with Solutions


Maximum Marks = 50, Maximum Time = 30 minutes

Answer all questions.

Version 2 CE IIT, Kharagpur


TQ.1: State the assumption for the design of members by working stress
method. (15 marks)

A.TQ.1: Sec. 13.34.6

TQ.2: Establish the equations for determining the depth of neutral axis, moment
of resistance and area of tension steel of an under-reinforced rectangular
beam.
(15 marks)
A.TQ.2: Sec. 13.34.10

TQ.3: When do we go for doubly-reinforced beams? Establish the equation for


determining areas of steel for the doubly-reinforced rectangular beams.
(20 marks)
A.TQ.3: Sec. 13.34.13

13.34.17 Summary of this Lesson


This lesson explains the philosophy of the analysis and design of singly-
reinforced rectangular flexural members employing working stress method. The
concepts of permissible stresses of concrete and reinforcement and the factors
of safety are explained. The basic assumptions of the working stress method are
mentioned. Balanced and under-reinforced sections are explained establishing
the governing equations for the analysis and design of such members. The
solution procedures for the two types of problems, viz., analysis and design
types, are given in steps. Due to the architect’s restrictions or for the functional
requirements, the doubly-reinforced rectangular beams are needed to be
designed. The governing equations of doubly-reinforced rectangular beams are
established.

Version 2 CE IIT, Kharagpur


Module
13
Working Stress Method
Version 2 CE IIT, Kharagpur
Lesson
35
Numerical Problems
Version 2 CE IIT, Kharagpur
Instructional Objectives:
At the end of this lesson, the student should be able to:

• employ the equations established for the analysis and design of singly and
doubly-reinforced rectangular beams,

• use tables of SP-16 for the analysis and design of singly and doubly-
reinforced rectangular beams,

• understand the economy in the design by limit state of collapse.

13.35.1 Introduction
Lesson 34 explains the equations for the analysis and design of singly and
doubly-reinforced rectangular beams. The applications of the equations are
illustrated in this lesson through the solutions of several numerical problems.
Direct computation method and use of tables of SP-16 are the two approaches
for the analysis and design of singly and doubly-reinforced rectangular beams. In
direct computation method, the derived equations are employed directly, while
the use of tables of SP-16 gives the results quickly with several alternatives
avoiding tedious calculations. Some of the numerical problems are solved using
both methods. Problems having the same width and effective depth but of
different grades of steel are solved to compare the results. Moreover, problems
of singly and doubly-reinforced rectangular beams, solved earlier by limit state of
collapse method are also taken up here to compare the results. It is shown that
the beams designed by limit state of collapse method are economical. In addition
to illustrative examples, practice and test problems are also given in this lesson.
Understanding the solved numerical examples and solving the practice and test
problems will help in following the applications of the equations and the use of
tables of SP-16 in analysing and designing singly and doubly-reinforced
rectangular beams.

Version 2 CE IIT, Kharagpur


13.35.2 Numerical Problems

Problem 1. (a) Determine the moment of resistance of the rectangular beam of


Fig. 13.35.1 having b = 350 mm, d = 600 mm, D = 650 mm, Ast = 804 mm2 (4-
16T), σcbc = 7 N/mm2 and σst = 230 N/mm2. (b) Determine the balanced moment
of resistance of the beam and the balanced area of tension steel. (c) Determine
the actual compressive stress of concrete fcbc and tensile stress of steel fst when
60 kNm is applied on the beam. Use direct computation method for all three
parts.

Solution 1.

1 (a): Given data are: b = 350 mm, d = 600 mm, Ast = 804 mm2, σcbc = 7 N/mm2
and σst = 230 N/mm2.

We have: pt = Ast (100) / bd = 80400 / (350) (600) = 0.383 per cent and m = 93.33
/σcbc = 93.33 /7 = 13.33. We determine the value of k from Eq. 13.16.

k = - (pt m /100) + {(pt m/100)2 + (pt m/50)}1/2


(13.16)
= - 0.051 + 0.323 = 0.272

j = 1- k/3 = 0.909

Equation 13.17 gives the moment of resistance of the beam as,

M = (pt/100) σst (1 – k/3) bd2


(13.17)

Version 2 CE IIT, Kharagpur


= (0.383/100) (230) (0.909) (350) (600) (600) = 100.89 kNm.

1 (b): The balanced moment of resistances and pt, bal of the beam is obtained
from Eqs. 13.10, 13.11 and 13.13.

Mb = Rbbd2
(13.10)
Rb = (1/2) σcbc kb jb = (pt, bal /100) σst jb
(13.11)
pt, bal = 50 kb (σcbc /σst )
(13.13)

where kb = 93.33 /(σst + 93.33) (13.3)


jb = 1- kb /3
(13.12)

The values of Rb and pt, bal may also be taken from Tables 13.3 and 13.4,
respectively of Lesson 34.

Here, kb = 93.33 / (230 + 93.33) = 0.288 and jb = 1- 0.288/3 = 0.904.

pt, bal = 50 kb (σcbc /σst) = 50 (0.288) (7/230) = 0.438


pt, bal is 0.44 from Table 13.4.

Therefore, Mb = (1/2)σcbc kb jb (bd2) = 114.81 kNm,


and Mb = (pt, bal /100) σst jb (bd2) = 114.75 kNm

Taking Rb from Table 13.3, we get:


Mb = Rb (bd2) = 0.91 (350) (600)2 = 114.66 kNm

The balanced area of steel = pt,bal (bd /100) = 0.438 (350) (600) /100 = 919.8
mm2. The beam has 4-16 mm diameter bars having 804 mm2. So, additional
amount of steel = 919.8 – 804 = 115.8 mm2 is needed to make the beam
balanced.

1`(c): The actual stresses in steel and concrete fst and fcbc are obtained from
Eqs. 13.24 and 13.25, respectively.

fst = M /{Ast d (1- k/3)}


(13.24)
fcbc = (2 Ast fst) / ( b k d)
(13.25)

where Ast = 804 mm2 and k = 0.272 (obtained in part a of the solution of this
problem). Thus, we have:

Version 2 CE IIT, Kharagpur


fst = 60000000 / 804 (600) (0.909) = 136.83 N/mm2 〈 230 N/mm2
fcbc = 2(804) (136.83) / (350) (0.272) (600) = 3.85 N/mm2 〈 7 N/mm2

Problem 2. Solve part a of Problem 1 (Fig. 13.35.1) by employing table of SP-


16.

Solution 2.

Given data are: pt = 0.383 per cent, σcbc = 7 N/mm2 and σst = 230 N/mm2. Table
69 of SP-16 gives R = 0.7956 (by linear interpolation). Accordingly, the moment
of resistance of the beam is M = R bd2 = (0.7956) (350) (600) (600) = 100.246
kNm.
The moment of resistance of the beam is 100.89 kNm by direct computation, as
obtained in part a of Problem 1.

Problem 3. Solve Problem 1 (Fig. 13.35.1) when σcbc = 7 N/mm2 and σst = 140
N/mm2 (mild steel) for all three parts. For part c, the applied moment is 40 kNm.
Other values/data remain unchanged.

Solution 3.

3 (a): Given data are: b = 350 mm, d = 600 mm, Ast= 804 mm2, σcbc 7 N/mm2 and
σst = 140 N/mm2. We have from Problem 1: pt = 0.383 per cent, m = 13.33, k =
0.272, and j = 0.909.

From Eq. 13.17, we get:

M = (pt / 100) σst (1 – k/3) bd2


(13.17)
= (0.383 / 100) (140) (0.909) (350) (600) (600) = 61.41 kNm

3 (b): Here kb = 93.33 / (140 + 93.33) = 0.4; jb = 0.87,


pt,bal = 50 kb (σcbc /σst) = 50 (0.4) (7/140) = 1.0 (Table 13.4 also gives pt,bal
= 1).

Therefore, Mb = (1/2) σcbc kb jb (bd2)


(13.10)
= (1/2) (7) (0.4) (0.87) (350) (600) (600) = 153.47 kNm

Mb = (pt, bal/100) σst (jb) (bd2)


(13.11)
= (1/100) (140) (0.87) (350) (600) (600) = 153.47 kNm

Version 2 CE IIT, Kharagpur


and, taking Rb from Table 13.3, we have from Eq. 13.10,
Mb = R bd2 = (1.21) (350) (600) (600) = 152.46 kNm.

The balanced area of steel is obtained from taking pt, bal from Table 13.4.
Accordingly, we get balanced area of steel = 1 (350) (600)/100 = 2100 mm2. The
beam is having 4-16 mm diameter bars of area 804 mm2. The additional area of
steel = 1296 mm2 is needed to make the beam balanced.

3 (c): fst = M / {Ast d (1- k/3)}


(13.24)
This gives fst = 40000000 / (804) (600) (0.909) = 91.22 N/mm2 〈 140 N/mm2

fcbc = (2 Ast fst) / b kd


(13.25)

So, fcbc = 2 (804) (91.22) / (350) (0.272) (600) = 2.57 N/mm2 〈 7 N/mm2

Problem 4. Solve part a of Problem 3 (Fig. 13.35.1) by using table of SP-16.

Solution 4.

Given data are: pt = 0.383 per cent, σcbc = 7 N/mm2 and σst = 140 N/mm2. Table
69 of SP-16 gives R = 0.48436 (by linear interpolation). Accordingly, the moment
of resistance of the beam is
M = R b d2 = (0.48436) (350) (600) (600) = 61.03 kNm.
The moment of resistance of this beam is 61.41 kNm by direct
computation, as obtained in part a of Problem 3.

Version 2 CE IIT, Kharagpur


Problem 5. Design a singly-reinforced rectangular beam to carry the design
moment = 100 kNm using M 25 concrete and Fe 415 grade of steel. Use b = 300
mm and d = 700 mm (Fig. 13.35.2) as preliminary dimensions.

Solution 5.

Steps 1 and 2 are not needed as the preliminary dimension of b = 300 mm and d
= 700 mm are given.

Step 3. The balanced moment of resistance of the beam = Mb = Rb d2= (1.11)


(300) (700) (700) = 163.17 kNm (taking Rb = 1.11 from Table 13.3 for σcbc = 8.5
N/mm2 and σst = 230 N/mm2). The balanced area of steel = pt, bal (bd) / 100 =
(0.53) (300) (700) / 100 = 1113 mm2 (taking pt, bal = 0.53 per cent from Table 13.4
for σcbc = 8.5 N/mm2 and σst = 230 N/mm2). We provide 4-16 mm diameter bars
of Ast = 804 mm2 〈 1113 mm2 to have pt = 804 (100) / (300) (700) = 0.383 per
cent. This is more than the minimum tensile steel = 0.85 bd / fy = 430.12 mm2, as
stipulated in cl. 26.5.1.1 of IS 456.

Step 4. Now, the section has to be checked for the stresses in concrete and
steel, fcbc and fst, respectively. For this problem m = 93.33 /σcbc = 10.98 and pt =
0.383 per cent, obtained in Step 3.

Equation 13.16 gives: k = - (pt m /100) + {(pt m/100)2 + (pt m /50)}1/2 = 0.251.
So, j = 1 – k/3 = 1 – 0.251/3 = 0.916.

Equation 13.24 gives: fst = M / {Ast d (1-k/3)}= 100 (106) / (804) (700) (0.916) =
193.98 N/mm2 〈 230 N/mm2.

Equation 13.25 gives: fcbc = 2 Ast fst / b kd = 2(804) (193.98) / (300) (0.251) (700)
= 5.92 N/mm2 〈 8.5 N/mm2

Therefore, the beam having b = 300 mm, d = 700 mm and Ast = 4-16 mm
diameter bars is safe.

Problem 6. Design the singly-reinforced rectangular beam of Problem 5 (Fig.


13.35.2) by using tables of SP-16.

Solution 6.

Given data are: M = 100 kNm, b = 300 mm and d = 700 mm. M/bd2 = 100 (106) /
(300) (700) (700) = 0.6803. Using Table 70 of SP-16, we get pt = 0.321, which
gives Ast = 0.321 (300) (700)/100 = 674 mm2. This area of steel is the minimum
Version 2 CE IIT, Kharagpur
amount of steel required to resist the design moment of 100 kNm. In fact, we
have several alternatives of selecting area of steel from 674 mm2 to the balanced
area of steel = 1113 mm2, as obtained in Step 3 of Problem 5. While solving
Problem 5 by direct computation method, steel area of 804 mm2 is selected (4-16
mm diameter bars), as the amount is less than the balanced area of steel = 1113
mm2 . However, selecting area of 674 mm2 is difficult as 3-16 mm diameter bars
have the area of steel 603 mm2. Only one 12 mm diameter is to be added to give
additional area of 113 mm2, which satisfies the requirement. However, 12 mm
bars are normally not preferred in beams. So, for the practical considerations, we
prefer 4-16 mm diameter bars, as assumed in the direct computation method.

The other alternatives are changing the depth of the beam. For studying
all these options, use of tables of SP-16 is very much time-saving.

Moreover, in the direct computation method, the selected area of steel has
to be checked so that it is more than the minimum area of steel, as stipulated in
cl. 26.5.1.1 of IS 456. On the other hand, the areas obtained from the table of
SP-16 already take care so that they are more than the minimum area of steel.

Problem 7. Determine the area of steel of the beam of Problem 3 to carry a


design moment of 200 kNm. The relevant data of Problem 3 are: b = 350 mm, d
= 600 mm, D = 650 mm (Fig. 13.35.1) σcbc = 7 N/mm2 and σst = 140 N/mm2.

Solution 7.

The moment of resistance of the balanced beam = 152.46 kNm obtained by


using Rb = 1.21 in the equation Mb = Rb bd2, in part b of the solution of Problem
3. The balanced area of the steel = 2100 mm2, which is taken from the solution of
part b of Problem 3. The values of m = 13.33, kb = 0.4, jb = 0.87 and pt, bal = 1
are also taken from the solution of part b of Problem 3.

It is evident that the design moment (= 200 kNm) being greater than
balanced moment (= 152.46 kNm), a doubly-reinforced beam has to be
designed. The additional moment M′ is obtained from Eq.13.26, as

M′ = M - Mb = 200 – 152.46 = 47.54 kNm.

The additional tensile steel, obtained from Eq.13.35, is: Ast2 = M′ /σst (d-d′) =
47.54 (106)/140 (600–50) = 617.4 mm2 (assuming d′ = 50 mm).

The amount of compression steel, obtained from Equation 13.36 is: Asc = (Ast2) /
(σst)/{σcbc(1.5m -1)(1- d′/kd)} = (617.4)(140)/7(18.995)(0.792) = 820.79 mm2.

Version 2 CE IIT, Kharagpur


Total area of steel, obtained from Eq.13.33 is Ast = Ast1 + Ast2 = 2100 + 617.4 =
2717.4 mm2.

Problem 8. Solve Problem 7 (Fig. 13.35.1) using tables of SP-16.

Solution 8.

The data of Problem 7 are: b = 350 mm, d = 600 mm, D = 650 mm, σcbc = 7
N/mm2 and σst = 140 N/mm2. We assume d′ = 50 mm, as in Problem 7.

The design of doubly-reinforced beam is vary simple with the help of


tables of SP-16. There are two ways. One is the direct solution using the
appropriate table from Tables 72 to 79. The other option is to determine Ast1 and
Ast2 from the equations and then to obtain Asc from Table M of SP-16. Both the
methods are explained below.

Method 1: Here, the appropriate table is Table 73. From the values of M/bd2 and
d′ /d, we get the percentages of tensile (total) and compressive reinforcement Ast
and Asc. Thereafter, the total tensile area of steel and compression area of steel
are determined.

M/bd2 = 200(106) / (350) (600) (600) = 1.59 and d′/d = 50/600 = 0.083.

All tables from 72 to 79 of SP-16 have step values of M/bd2 and d′/d. So, the
linear interpolations are to be done three times. The adjacent M/bd2 and d′/d are
1.55 and 1.60, and 0.05 and 0.1. First, we carry out linear interpolation for d′/d =
0.5. For M/bd2 = 1.59 (when d′/d = 0.5)

pt = 1.253 + (0.038 / 0.05) 0.04 = 1.2834


and pc = 0.305 + (0.045 /0.05) 0.04 = 0.341

Again for M/bd2 = 1.59, when d′/d = 0.1


pt = 1.267 + (0.04/0.05) (0.04) = 1.299
and pc = 0.375 + (0.056 / 0.05) (0.04) = 0.4198.

Now, the linear interpolation is done for M/bd2 = 1.59 and d′/d = 0.083. The
results are given below:
pt = 1.2834 + (0.033) (1.299 – 1.2834) / 0.05 = 1.2936
pc = 0.341 + (0.033) (0.4198 – 0.341) / 0.05 = 0.404

Version 2 CE IIT, Kharagpur


Thus, we have area of total tensile steel, Ast = (1.2936) (350) (600)/100 =
2716.56 mm2 and the area of compression steel, Asc = 0.404 (350) (600)/100 =
848.4 mm2. In the above calculations, more number of digits are taken to
minimise the truncation error.

We get almost the same values of Ast and Asc for Problem 7 by direct
computation method (same as those of Problem 8). They are as follows: Ast =
2717.4 mm2 and Asc = 820.79 mm2. The values are reasonably comparable.

Method 2: Here, in this method the values of Ast1 and Ast2 are obtained by direct
computation as in Problem 7 giving Ast1 = 2100 mm2 and Ast2 = 617.4 mm2. Table
M of SP-16 gives the ratio of Asc to Ast2 for different d′/d ratios. For d′/d = 0.083,
by linear interpolation, we get

Asc / Ast2 = 1.2 + (0.2) (0.333)/(0.05) = 1.332

So, Asc = (1.332) (617.4) = 822.37 mm2. This value is close to that of direct
computation method as the ratio is depending on Ast2 obtained by direct
computation method.

13.35.3 Practice Questions and Problems with Answers

Q.1: Design the beam of Problem 5 of sec. 13.35.2 (Fig. 13.35.2) using σcbc = 8.5
N/mm2 and σst = 140 N/mm2. The given data of Problem 5 are: b = 300 mm,
d = 700 mm and M = 100 kNm.

A.1: Steps 1 and 2 are not needed as preliminary dimensions of b and d are
given.

Step 3.

Mb = Rb bd2 = (1.47) (300) (700) (700) = 216.09 kNm (Using Rb = 1.47 from
Table 13.3). Ast, bal = pt, bal (bd/100) = (1.21) (300) (700) / 100 = 2541 mm2 (using
pt, bal = 1.21 from Table 13.4). We provide 4-20 + 2-16 (=1256 + 402 = 1658 mm2)
as Ast, giving pt = 1658 (100) / (300) (700) = 0.79 per cent. The minimum area of
steel = 0.85 bd / fy = (0.85) (300) (700) / 250 = 714 mm2 〈1658 mm2. Hence, ok.

Step 4. Check for stresses

With the modular ratio m = 93.33 /8.5 = 10.98 and pt = 0.79; Eq. 13.16 gives:
k = - (pt m/100) + {(pt m/100)2 + (pt m/50)}1/2 = 0.339 ; and j = 1- k/3 = 0.887.

Version 2 CE IIT, Kharagpur


Equation 13.24 gives: fst = M /{Ast d (1 – k/3)} = 100(106) / (1658) (700) (0.887) =
97.14 N/mm2 〈 140 N/mm2

Equation 13.25 gives: fcbc = 2 Ast fst / bkd = 2(1658) (97.14) / (300) (0.339) (700)
= 4.53 N/mm2 〈 8.5 N/mm2.

Hence, this design is ok.

In fact, there are several possible alternatives as explained in the solution of


Problem 5. These alternatives are obtained quickly using tables of SP-16.

Q.2: Design the beam of Q.1 (Fig. 13.35.2) using tables of SP-16.

A.2: Steps 1 and 2 are not needed as the given data are: b = 300 mm, d = 700
mm and M = 100 kNm.

Step 3.

M/bd2 = 100(106) / (300) (700) (700) = 0.6803. Table 70 of SP-16 gives pt =


0.53775 by linear interpolation. Accordingly, Ast = (0.53775) (300) (700)/100 =
1129.275 mm2. Provide 4-20 mm diameter bars of area 1256 mm2, which has pt
= 1256 (100) / (300) (700) = 0.598

Step 4. Check for stresses

The modular ratio m = 10.98, k = 0.339 and j = 0.887 from A.1 of Q.1. Equation
13.24 gives: fst = M/Ast d(j) = 100 (106) / (1256) (700) (0.887) = 128.23 N/mm2 〈
140 N/mm2. Equation 13.25 gives: fcbc = 2Ast fst / b kd = (2) (1256) (128.23) /
(300) (0.339) (700) = 4.52 N/mm2 〈 8.5 N/mm2 . Hence, the design is ok.

Q.3: The problem of Q.4 of sec. 3.6.11 of Lesson 6 (Fig. 3.6.4) has the following
data: b = 300 mm, d = 550 mm, D = 600 mm, Mu, lim = 220.45 kNm, Ast, lim =
1107.14 mm2, M 20 and Fe 500.

(a) Determine the balanced moment of resistance and Ast,b in working stress
method.
(b) Determine the areas of steel required to resist the design moment = Mu,lim
/1.5 = 146.97 kNm.

A.3: (a) To determine Mb and Ast, b

Version 2 CE IIT, Kharagpur


Mb = Rb bd2 = (0.99) (300) (550) (550) = 89.84 kNm (using Rb = 0.99 from Table
13.3).
Using pt, bal = 0.39 from Table 13.4, Ast, b = (0.39) bd/100 = 643.5 mm2.

(b) Design of doubly-reinforced beam

From Eq.13.26: M′ = 146.97 – 89.84 = 57.13 kNm. From Eq.13.34: Ast1 = 643.5
mm2 for
the balanced moment of 89.84 kNm (see sol.a of this question when pt, bal =
0.39).

From Eq.13.35: Ast2 = M′/σst (d - d′ ) = (57.13) (106)/ (275) (500) = 415.49


mm2 (assuming d′ = 50 mm).

Total Ast = Ast1 + Ast2 = 643.5 + 415.49 = 1058.99 mm2.

From Eq.13.36, Asc = (Ast2) (σst) / {(σcbc) (1.5 m – 1) (1- d′ /kd)} =


(415.49)(275) / {(7) (18.995) (0.636)} = 1350.36 mm2.

The beam requires Ast = 1058.99 mm2 and Asc = 1350.36 mm2. In the limit
state method the values are: Ast = 1107.14 mm2 and Asc = 0, i.e., only singly-
reinforced beam with almost the same amount of Ast is sufficient. This shows the
economy in the design when done by employing limit state method.

This problem, however, cannot be solved by using tables of SP-16 as the


tables of SP-16 do not have the values for Fe 500 grade of steel.

13.35.4 Limitations of Working Stress Method

The basis of the analysis by the working stress method is very simple. This
method was used for the design of steel and timber structures also. However,
due to the limitations of the method, now the limit state methods are being used.
The limitations of working stress method are the following:

i) The assumptions of linear elastic behaviour and control of stresses within


specially defined permissible stresses are unrealistic due to several
reasons viz., creep, shrinkage and other long term effects, stress
concentration and other secondary effects.

ii) The actual factor of safety is not known in this method of design. The
partial safety factors in the limit state method is more realistic than the

Version 2 CE IIT, Kharagpur


concept of permissible stresses in the working stress method to have
factor of safety in the design.

iii) Different types of load acting simultaneously have different degrees of


uncertainties. This cannot be taken into account in the working stress
method.

Accordingly, the working stress method is gradually replaced by the limit


state method. The Indian code IS 456 has given working stress method in Annex
B to give greater emphasis to limit state design. Moreover, cl. 18.2.1 of IS 456
specifically mentions of using limit state method normally for structures and
structural elements. However, cl.18.2.2 recommends the use of working stress
method where the limit state method cannot be conveniently adopted. Due to its
simplicity in the concept and applications, better structural performance in service
state and conservative design, working stress method is still being used for the
design of reinforced concrete bridges, water tanks and chimneys. In fact, design
of tension structures and liquid retaining structures are not included in IS 456 for
the design guidelines in the limit state method of design.

Accordingly, Lessons 34 and 35 include the basic concept of this method.


The design of T-beam in flexure, rectangular and T-beams under shear, torsion
and other topics of the limit state method, covered in different lessons, are not
included adopting working stress method. However, the designs of tension
structures and liquid retaining structures are taken up in the next lesson, as these
are not included by the limit state method.

13.35.5 References
1. Reinforced Concrete Limit State Design, 6th Edition, by Ashok K. Jain,
Nem Chand & Bros, Roorkee, 2002.
2. Limit State Design of Reinforced Concrete, 2nd Edition, by P.C. Varghese,
Prentice-Hall of India Pvt. Ltd., New Delhi, 2002.
3. Advanced Reinforced Concrete Design, by P.C. Varghese, Prentice-Hall
of India Pvt. Ltd., New Delhi, 2001.
4. Reinforced Concrete Design, 2nd Edition, by S. Unnikrishna Pillai and
Devdas Menon, Tata McGraw-Hill Publishing Company Limited, New
Delhi, 2003.
5. Limit State Design of Reinforced Concrete Structures, by P. Dayaratnam,
Oxford & I.B.H. Publishing Company Pvt. Ltd., New Delhi, 2004.
6. Reinforced Concrete Design, 1st Revised Edition, by S.N. Sinha, Tata
McGraw-Hill Publishing Company. New Delhi, 1990.
7. Reinforced Concrete, 6th Edition, by S.K. Mallick and A.P. Gupta, Oxford &
IBH Publishing Co. Pvt. Ltd. New Delhi, 1996.
8. Behaviour, Analysis & Design of Reinforced Concrete Structural Elements,
by I.C.Syal and R.K.Ummat, A.H.Wheeler & Co. Ltd., Allahabad, 1989.

Version 2 CE IIT, Kharagpur


9. Reinforced Concrete Structures, 3rd Edition, by I.C.Syal and A.K.Goel,
A.H.Wheeler & Co. Ltd., Allahabad, 1992.
10. Textbook of R.C.C, by G.S.Birdie and J.S.Birdie, Wiley Eastern Limited,
New Delhi, 1993.
11. Design of Concrete Structures, 13th Edition, by Arthur H. Nilson, David
Darwin and Charles W. Dolan, Tata McGraw-Hill Publishing Company
Limited, New Delhi, 2004.
12. Concrete Technology, by A.M.Neville and J.J.Brooks, ELBS with
Longman, 1994.
13. Properties of Concrete, 4th Edition, 1st Indian reprint, by A.M.Neville,
Longman, 2000.
14. Reinforced Concrete Designer’s Handbook, 10th Edition, by C.E.Reynolds
and J.C. Steedman, E & FN SPON, London, 1997.
15. Indian Standard Plain and Reinforced Concrete – Code of Practice (4th
Revision), IS 456: 2000, BIS, New Delhi.
16. Design Aids for Reinforced Concrete to IS: 456 – 1978, BIS, New Delhi.

13.35.6 Test 35 with Solutions


Maximum Marks = 50, Maximum Time = 30 minutes

Answer all questions.

TQ.1: Problem 4.1 of sec. 4.9.2 of Lesson 4 has the following data: (Fig. 4.9.1) b
= 300 mm, d = 630 mm, d′ = 70 mm, D = 700 mm, M 20, Fe 415, Mu, lim =
492.96 kNm, Ast1 =1809.14 mm2, Ast2 = 783.621 mm2 (Ast = 2572.834
mm2), Asc = 806.517 mm2 (by direct computation method). Tension steel
reinforcement = 4-25 mm diameter bars + 2-20 mm diameter bars (= 2591
mm2) and compression steel reinforcement = 2-20 mm diameter bars + 2-
12 mm diameter bars (= 854 mm2).

Determine the areas of steel by working stress method when the design
moment = Mu, lim / 1.5 = 328.64 kNm. Use the same dimensions of the
beam as mentioned above. Use direct computation method and also tables
of SP-16.
(25+25 = 50 marks)

A.TQ.1: (1) Direct computation method

Given data are: b = 300 mm, d = 630 mm, σcbc = 7 N/mm2 and σst = 230 N/mm2.

Mb = Rb bd2 = 0.91 (300) (630) (630) = 108.35 kNm (using Rb = 0.91 from
Table 13.3). Therefore, M′ = M – Mb = 328.64 – 108.35 = 220.29 kNm.

Version 2 CE IIT, Kharagpur


Ast b = pt, bal (bd/100) = 0.44 (300) (630) / 100 = 831.6 mm2 (using pt, bal =
0.44 from Table 13.4) and Ast2 = M′ / {σst (d-d′ )} = 220.29 (106) / {(230)
(630 – 70)} = 1710.32 mm2. Therefore, Ast = 831.6 + 1710.32 = 2541.92
mm2.

The modular ratio m = 93.33 / σcbc = 13.33 and kb = 93.33/(σst +93.33) =


0.289.

Asc =(Ast2) (σst) / {σcbc (1.5 m –1) (1-d′/kd) = 4806.37 mm2 , giving pc = Asc
(100)/ bd = (4806.37) (100) / (300) (630) = 2.54 per cent 〈 maximum
percentage allowed (4 per cent) as per cl.26.5.1.2 of IS 456.

So, Ast = 2541.92 mm2 = (4-25 + 2-20 giving 2591 mm2)


and Asc =4806.37 mm2 = (8-25 + 3-20 giving 4869 mm2)

(2) Using tables of SP-16

M/bd2 = 328.64 (106) / (300) (630) (630) = 2.76


d′/d = 70/630 = 0.11

Table 77 has the values of pt and pc for different values of M/bd2 and d′/d.
Therefore, the final values of pt and pc are to be determined by carrying out
the linear interpolation three times, as explained in the solution of Problem
8. The results are given below:

(a) For Mu/bd2 = 2.75, d′/d = 0.11

pt = 1.327 + 0.052 (0.01) / 0.05 = 1.3374


pc = 2.348 + 1.03 (0.01) / 0.005 = 2.555

(b) For Mu/bd2 = 2.8 and d′/d = 0.11

pt = 1.351 + (0.053) (0.01) / 0.05 = 1.3616


pc = 2.412 + (1.063) (0.01) / 0.05 = 2.6246

(c) Therefore, for Mu/bd2 = 2.76 and d′/d = 0.11

pt = 1.3374 + (0.0342) (0.01) / (0.05) = 1.34424


pc = 2.555 + (0.0696) (0.01) / (0.05) = 2.5689

So, Ast = (1.34424) (300) (630) / 100 = 2540.6 mm2

Version 2 CE IIT, Kharagpur


and Asc = (2.5689) (300) (630) / 100 = 4855.2 mm2.

The values of Ast = 2541.92 mm2 and Asc = 4806.37 mm2, obtained by direct
computation method in part (1) of this problem, are reasonably comparable
with the values obtained by using tables of SP-16.

13.35.7 Summary of this Lesson

This lesson illustrates the applications of the equations developed in Lesson


34 for the analysis and design of singly and doubly-reinforced rectangular beams
by working stress method. For this purpose, several numerical problems are
solved. Some of the problems have the same dimensions of width and effective
depth but with different grades of steel so that the results are compared.
Moreover, problems solved earlier on singly and doubly-reinforced beams are
also taken up to compare the results. The method of direct computation i.e.,
using the appropriate equations directly and the use of tables of SP-16 are
employed for some of the problems to illustrate the simplicity in using the tables
of SP-16. Several practice problems, test problem and illustrative examples will
help in understanding the applications for the analysis and design of singly and
doubly-reinforced rectangular beams.

Version 2 CE IIT, Kharagpur


Module
14
Tension Members
Version 2 CE IIT, Kharagpur
Lesson
36
Structural
Requirements, Code
Stipulations and
Governing Equations
Version 2 CE IIT, Kharagpur
Instructional Objectives:
At the end of this lesson, the student should be able to:

• state the need to design the tension structures using reinforced concrete
though concrete is very weak in tension,
• justify the use of working stress method for designing reinforced concrete
tension members,
• name the two essential requirements for the design of reinforced concrete
tension structures,
• explain why some amount of cracking may be allowed in such structures,
• explain why porous aggregates are not allowed in the reinforced concrete
tension structures,
• state the basis of design of reinforced concrete tension members,
• state the different cases of reinforced concrete tension members, and
• establish the governing equations for all the cases of tension members to
determine the stresses of concrete and steel following the working stress
method of design.

14.36.1 Introduction
Reinforced concrete is used in the design of several practical members of
tension structures though it is not a potential material due to very inadequate
tensile strength of concrete compared to its compression strength. In such cases,
reinforced concrete develops tensile stresses either due to direct tension force or
combined with bending. Tie members of trusses and arches, walls of rectangular
tanks and bunkers, suspended roofs, cylindrical pipes, walls of liquid retaining
structures are some of the examples where tension stresses also develop.
Traditional elastic approach (working stress method) and the limit state method
can be applied for the design of such structures. Based on elastic theory, the
working stress method is simpler in concept and applications, while the limit state
method, based on cracking behaviour of concrete, is not fully developed yet.

Some of the structures or members mentioned above may not be in direct


contact with any liquid. On the other hand, some may be in direct contact with
liquid. Though control of cracking is important for all types of structures, it is an
essential requirement for liquid retaining structures. Impermeability of structure is
not only for preventing leakage but also for improving durability, resistance to
leaching, chemical action, erosion, frost damage and protecting the
reinforcement from corrosion.

Version 2 CE IIT, Kharagpur


IS 456 gives due considerations for compression either direct or
associated with bending and shear when limit state method is employed.
However, it is almost silent on tension members. IS 3370, dealing with liquid
retaining structures, is still adopting working stress method. Thus, the
applications of limit state method for the design of tension members are still in
the development stage. Accordingly, the working stress method shall be
employed in this lesson to explain the design of such members.

14.36.2 Resistance to Cracking and Strength

Figures 14.36.1a and b show one rectangular section under flexural


tension and the distribution of stresses, respectively. The symmetric section of
depth D is having the neutral axis at a depth of 0.5 D. Assuming that the actual
tension stress of M 25 concrete (say) in bending ftb reaches the permissible value
of σtb = 1.8 N/mm2 for resistance to cracking (see Table 14.1), the tensile stress
of steel fst at the level of steel shall be modular ratio times 0.8 σtb or 15.81 N/mm2
{(10.98) (0.8) (1.8) N/mm2, where modular ratio of M 25 concrete m = 93.33 / 8.5
= 10.98 and d′ = 0.1 D}. The value of fst is very low when compared with the
values of the permissible tension stress of steel (115 N/mm2 for Fe 250 and 150
N/mm2 for Fe 415 as given in Table 14.2. Therefore, some amount of cracking
may be permitted when the structure is not in contact with any liquid at all.
Moreover, the remote face of the liquid retaining structures, which is not in
contact with the liquid, also is a possible place of allowing minor cracks.
However, it is also important to note that sufficient depth is available to prevent
the corrosion of the reinforcement.

Version 2 CE IIT, Kharagpur


Allowing minor cracks in such cases means increasing the values of
permissible stresses of concrete and steel reinforcement specifying the particular
cases. IS 3370 Part II recommends the permissible stresses of concrete and
reinforcement for different types of situations, which are given in Tables 14.1 and
14.2.

Hence, the design of tension members shall provide adequate control of


cracking in addition to normal requirements of strength. There are many other
issues involved in the design of tension structures. IS 3370 Parts I to IV deal with
such issues in detail. The relevant important aspects are briefly mentioned in the
following section.

14.36.3 IS Code Stipulations


(a) Porous aggregates – (cl.2.1.a of IS 3370, Part I)

Use of porous aggregates, such as burnt clay and broken bricks or tiles is not
allowed for parts of structure either in contact with the liquids on any face or
enclosing the space above the liquid.

(b) Concrete mix - (cl.3.1 a of IS 3370, Part I)

Concrete mix weaker than M 20 shall not be used except in thick sections
(thickness > 450 mm) and parts of structure neither in contact with the liquid
on any face nor enclosing the space above the liquid.

(c) Basis of design - (cl.3.2 of IS 3370, Part II)

(i) The parts of the structure not in contact with the liquid shall be designed in
accordance with the requirements of IS 456.

(ii) Design of members other than those in (i) above shall be based on
consideration of adequate resistance to cracking as well as adequate
strength.

(iii) Calculation of stresses shall be based on the following assumptions in


addition to the general assumptions given in IS 456.

1. The whole section of concrete including the cover together with the
reinforcement shall be taken into account in calculating both flexure and
direct tension (or combination of both) relating to resistance to cracking.

2. Total shear stress = Q/(b jd), shall not exceed the permissible value given
in Table 14.1, where Q, b and jd are the total shear, breadth and lever
arm, respectively.

Version 2 CE IIT, Kharagpur


3. Concrete has no tensile strength in strength calculations.

(d) Permissible stresses in concrete - (cl.3.3 of IS 3370 Part II)

(i) For resistance to cracking

Table 14.1 furnishes the permissible stresses in tension σtd and σtb (direct
and due to bending) and shear τsh for calculations relating to the
resistance of members to cracking. The permissible tensile stresses due
to bending apply to the face of the member in contact with the liquid. In
members less than 225 mm thick and in contact with the liquid on one
side, these permissible stresses in bending apply also to the face remote
from the liquid.

Table 14.1: Permissible stresses in concrete in calculations relating to


resistance to cracking

Permissible stresses (N/mm2)


Grade of
concrete
Direct Tension, Tension due to Shear τsh (Q / b
σtd Bending σtb jd)
M 15 1.1 1.5 1.5
M 20 1.2 1.7 1.7
M 25 1.3 1.8 1.9
M 30 1.5 2.0 2.2
M 35 1.6 2.2 2.5
M 40 1.7 2.4 2.7

(ii) For strength calculations

In strength calculations, the permissible concrete stresses shall be in


accordance with IS 456 (Table 13.1 of Lesson 34). Where the calculated
shear stress in concrete alone exceeds the permissible value,
reinforcement acting in conjunction with diagonal compression in the
concrete shall be provided to take the whole of the shear.

(e) Permissible stresses in steel - (cl.3.4 of IS 3370, Part II)

(i) For resistance to cracking

When steel and concrete are assumed to act together for checking the
tensile stress in concrete for avoidance of cracks, the tensile stress in the
steel will be limited by the requirement that the permissible tensile stress
in concrete is not exceeded, so that the tensile stress in steel shall be

Version 2 CE IIT, Kharagpur


equal to the product of modular ratio of steel and concrete and the
corresponding allowable tensile stress in concrete.

(ii) For strength calculations

For strength calculations the permissible stresses in steel reinforcement


are given in Table 14.2.

Table 14.2: Permissible stresses in steel reinforcement for strength


calculations

Permissible stresses σst N/mm2


Sl. No Types of stress in steel Fe 250 Fe 415
reinforcement
(i) Tensile stress in members under 115 150
direct tension
(ii) Tensile stress in members in
bending 115 150
(a) On liquid retaining face of
members 115 150
(b) On face away from liquid for
members less than 225 mm
in thickness
125 190
(c) On face away from liquid for
members 225 mm or more in
thickness
(iii) Tensile stress in shear
reinforcement: 115 150
(a) For members less than 225
mm in thickness 125 175
(b) For members 225 mm or
more in thickness
(iv) Compressive stress in columns 125 175
subjected to direct load

Note: Stress limitations for liquid retaining faces shall also apply to the
following:

a) Other faces within 225 mm of the liquid retaining face.


b) Outside or external faces of structures away from the liquid but placed in
water logged soils up to the level of the highest subsoil water.

Version 2 CE IIT, Kharagpur


14.36.4 Types of Tension Structures

Figure 14.36.2 summarises the two types of tension structures made of


reinforced concrete indicating the different cases. The two types are (i) structures
having no contact with the liquid and (ii) structures in direct contact with the liquid
or liquid retaining structures. The different cases are first due to the three
sources of tension stresses: (i) due to direct tension, (ii) pure flexure and (iii)
direct tension associated with the flexure. Secondly, for the liquid retaining
structures if the tension is on the (i) liquid face or (ii) on the remote face. Further
two cases are there for the structures having tension on the remote face
depending on, if the depth of section is (i) < 225 mm or (ii) ≥ 225 mm.

It is worth mentioning that understanding of the behaviour of tension


structures and developing the governing equations of different cases are possible
by taking up three main types: (i) when subjected to axial tension only, (ii) when
subjected to bending moment only and (iii) when subjected to combined axial
tension and bending moment. Accordingly, we take up these three main types in
the following sections.

Version 2 CE IIT, Kharagpur


14.36.5 Members Subjected to Axial Tension only
Based on the discussion made so far, the criteria of design of such tension
members are:

(i) Reinforcement alone shall take the full tension force.

(ii) The tension stress in steel shall not exceed the recommended
permissible stress values given in Table 22 of IS 456 and Table 13.2 of
Lesson 34.

(iii) The tensile stress of concrete shall be determined transforming the


steel into equivalent concrete.

(iv) The tensile stress in the transformed section shall not exceed the
stipulated permissible values given in cl. B-2.1.1 of IS 456 (Table 13.1
of Lesson 34). This will regulate the cracking of concrete.

Therefore, we can determine the actual tensile stress of steel fst and
concrete ftd from the following equations:

fst = Ft / Ast (14.1)

and ftd = Ft /(Ac + m Ast)


(14.2)

where

Ft = total tension force on the member minus pretension in steel, if any,


before concreting,
Ast = cross-sectional area of reinforcing steel in tension,
Ac = cross-sectional area of concrete excluding any finishing material and
reinforcing steel, and
m = modular ratio

14.36.6 Members Subjected to Pure Flexure


For this, we will consider two cases: (i) when the section is uncracked and
(ii) when the section is cracked. The first case is applicable when either the
tension is on the liquid face or when the reinforcement is away from the liquid by
a distance less than 225 mm. The second case is applicable when there is no
direct contact of the structure with the liquid. The two cases are explained below.

Version 2 CE IIT, Kharagpur


(A) Uncracked Section

Figures 14.36.3a and b show the uncracked section and the stress
distribution of such section subjected to bending moment alone. The maximum
bending stresses in compression or tension are given by the expression.

fcb = M x / Iyy (14.3)

and ftb = M (D-x) / Iyy (14.4)

where M = Applied moment on the member


x and (D - x) = maximum distances of the compression and tension
fibres from the neutral axis, x is also the depth of the
neutral axis.
Iyy = moment of inertia of the uncracked section about the
neutral axis of the section transforming area of steel
into equivalent concrete.
The depth of the neutral axis x is obtained by taking moment of the transformed
section about the top face, which gives: bD2/2 + (m-1) pt b Dd/100 = {bD + (m-1)
pt bD / 100}x

or x = {b D2/2 + (m –1) pt b Dd/100} / {bD + (m-1) pt bD / 100} (14.5)

The moment of inertia of the transformed section about the neutral axis is
obtained from:
:

Version 2 CE IIT, Kharagpur


Iyy = b x3 / 3 + b (D – x)3 / 3 + (m –1) pt bD (D-x)2/100 (14.6)

The values of fcb and ftb are then obtained from Eqs. 14.3 and 14.4, respectively.

The moment of resistance of the section is determined from Eq.14.4 as ftb is the
governing stress of the section. This gives

M = ftb Iyy / (D – x) (14.7)

(B) Cracked Section

Figures 14.36.4a and b show the section and stress distribution of the
cracked section when subjected to moment alone. This particular case has been
discussed in secs. 13.34.8 and 13.34.9 when the section is balanced and under-
reinforced, respectively. Therefore, the equations are not derived here and only
the final equations are given for the balanced and under-reinforced sections
separately. The only difference is the notation of compressive stress of concrete
in bending compression, which is fcbc in secs. 13.34.8 and 9, while the same is
denoted by fcb here.

(i) Equations for the under-reinforced sections

The depth of the neutral axis is determined from x = kd, where

k = - (pt m / 100) + {(ptm / 100)2 + (pt m / 50)}1/2


(13.16)

The compressive and tensile forces C and T are obtained from:

Version 2 CE IIT, Kharagpur


C = (1/2) fcb b kd
(13.20)

T = fst Ast
(13.6)

The moment of resistance with respect to steel is obtained from:

M = Ast fst d(1 – k/3)


(13.24)

M = (pt /100)fst (1 – k/3) bd2


(13.19)

(ii) Equations for the balanced section are as follows:

The depth of the balanced neutral axis is determined from: xb = kb d, where

kb = 93.33 / (σst + 93.33) (13.3)

The expressions of compressive and tensile forces C and T are:

C = (1/2) (σcb) b kb d (13.5)

T = Ast σst (13.6)

The expressions of the balanced moment of resistance with respect to concrete


and steel are:

Mb = (1/2) σcb kb jb (b d 2) (13.7)

Mb = (pt, bal / 100) σst jb (b d2)


(13.8)

14.36.7 Members Subjected to Combined Axial Tension and


Moment.
Here also, we will consider two cases: (i) when the section is uncracked
and (ii) when the section is cracked.

Version 2 CE IIT, Kharagpur


(A) Uncracked Section

If tension is there on the liquid face, the tensile stress actually developed,
i.e., ftd and ftb in direct tension and bending tension, respectively should satisfy
the following condition:

(ftd /σtd) + (ftb / σtb)≤ 1.0 (14.8)

where σtd and σtb are the permissible stresses of concrete in direct tension and
bending tension, respectively and are to be taken from Table 14.1, and ftd and ftb
are the actual stresses of concrete in direct tension and bending tension,
respectively and are determined from Eqs.14.2 and 14.4, respectively.

(B) Cracked Section

This case is classified under two groups: (i) when the eccentricity e of the
tension force Ft is small; i.e., tensile force is large and (ii) when the eccentricity e
of the tension force is large, i.e., tension force is small. We are explaining them
separately.

(i) When the eccentricity is small, i.e., Ft is large

Figures 14.36.5a and b show the cracked section subjected to tension


force at a small eccentricity e within the depth of the section. As the eccentricity
is small, measured from the center of gravity of the two areas of steel, As1 and
As2, the whole section is in tension and hence the entire concrete is ineffective.

Version 2 CE IIT, Kharagpur


We have two equations of equilibrium: (i) equilibrium of forces and (ii)
equilibrium of moments. They are expressed as follows:

Ft = Ts1 + Ts2 = fst1 As1 + fst2 As2 (14.9)

Moment of the forces about the bottom steel gives: Ft (d1 – e) – Ts2 (d1 + d2) = 0

or Ft (d1 – e) – fst2 As2 (d1 + d2) = 0


(14.10)

Substituting the expression of Ft from Eq.14.9 into Eq.14.10, fst2 can be


computed. The value of fst1 can then be computed from Eq.14.9.

(ii) When the eccentricity is large, i.e., Ft is small

Figure 14.36.6a and b show the section and stress distribution of cracked
section subjected to tension force Ft having large value of the eccentricity e
measured from the mid-depth of the section. The line of action of Ft is outside the
section. The areas of compression and tension steel are denoted by Asc and Ast,
respectively. Due to large eccentricity, the direct and bending stresses are
equally dominant. The forces of compression and tension steel are represented
by Csc and Ts, respectively, while the force of compressive concrete is
represented by Cc. The distance of the neutral axis is x.

Version 2 CE IIT, Kharagpur


The expressions of these three forces are:

Csc = fsc Asc


(14.11)

Ts = fst Ast
(14.12)

Cc = 0.5 fcb bx
(14.13)

The expressions of fsc and fst, obtained from the stress distribution of Fig. 14.36.6
b, are:

fsc = (mc –1) fcb (x - d′)/x


(14.14)

fst = m fcb (d – x) / x
(14.15)

Using the expression of fsc and fst from Eqs. 14.14 and 14.15 in Eqs. 14.11 and
14.12, we have:

Csc = (mc – 1) Asc fcb (x - d′) / x


(14.16)

Ts = m Ast fcb (d – x) / x
(14.17)

The equation of the equilibrium of forces is:

Ft – Ts + Cc + Cse = 0
(14.18)

Taking moment of all forces about the line of action of Ft, the equation of
equilibrium of moment is:

Ts (e + 0.5D - d) – Cc (e + 0.5D – x/3) – Csc (e + 0.5D - d′ ) = 0


(14.19)

Substituting the expressions of Cc, Csc and Ts from Eqs. 14.13, 14.16 and 14.17,
respectively into Eqs. 14.18 and 14.19, we have:

Ft = fcb {m Ast (d – x) / x – 0.5 bx – (mc – 1)Asc (x -d′) / x}


(14.20)

Version 2 CE IIT, Kharagpur


m Ast (d –x) (e + 0.5D –d) / x = 0.5 bx (e + 0.5D – x/3) + (mc – 1) Asc (x-d′)
(e + 0.5D - d′) / x
(14.21)

Equations 14.20 and 14.21 have seven unknowns b, d, D, x, fcb, Ast and
Asc of which b, d and D are assumed or taken from the preliminary dimensions.
So, we have four unknowns x, fcb, Ast and Asc to be determined from two
equations (Eqs. 14.20 and 14.21). Therefore, trial and error method has to be
used.

As a particular case, when Asc = 0, Eqs. 14.20 and 14.21 are reduced to:

Ft = fcb {m Ast (d –x) / x – 0.5 bx}


(14.22)

and m Ast (d – x) (e + 0.5 D – d) / x = 0.5 bx (e + 0.5 D – x/3)


(14.23)

Here also the three unknowns x, fcb and Ast are determined from Eqs.
14.22 and 14.23 by trial and error.

However, for such problems of singly-reinforced section, the initial value of


Ast can be determined as explained below.

We use the equation of equilibrium of forces i.e., Eq. 14.24 as such and
write the equation of equilibrium of moment by taking moment about Ts, which
gives

Cc (jd) – Ft (e + 0.5D – d) = 0
(14.24)

or Cc = Ft (e + 0.5 D – d)/jd
(14.25)

Using the expression of Cc from Eq. 14.25 in Eq. 14.18, when Asc = 0, we get:

Ft = Ts - Cc = Ts – Ft (e + 0.5 D – d)/ jd

or Ts =Ft {1 + (e + 0.5 D –d)/ jd}


(14.26)

Version 2 CE IIT, Kharagpur


Assuming Ts = Ast σst in Eq. 14.26, we have:

Ast = (Ft / σst){1+ (e + 0.5 D – d)/ jd}


(14.27)

Equation 14.27 is used to get the starting value of Ast assuming j as 0.87
(say). This value of Ast now can be used in Eq. 14.23 to get the value of x. Using
that value of x in Eq. 14.22, the value of fcb can be determine. If fcb >σcb, either
the value of Ast shall be increased keeping D as the same or the value of D may
be increased keeping Ast as the same. The third option is to increase both Ast
and D, as appropriate.

Numerical problems are taken up in Lesson 37 for tension structures


having no contact with liquid and for liquid retaining structures covering all the
cases explained in Fig. 14.36.2.

14.36.8 Practice Questions and Problems with Answers


Q.1: Why is it needed to design the reinforced concrete tension structures when
concrete is not good in tension?

A.1: Sec. 14.36.1, Paragraph 1.

Q.2: Name the two criteria of design of reinforced concrete tension structures.

A.2: Sec. 14.36.2

Q.3: State the basis of design of reinforced concrete tension structures.

A.3: Sec. 14.36.3, Part (c)

Q.4 Draw a schematic diagram of different cases of tension structures made of


reinforced concrete.

A.4: Fig. 14.36.2

Q.5: Establish the equations to determine the tensile stresses of concrete and
steel when the member is subjected to axial force Ft alone.

A.5: Sec. 14.36.5

Q.6: Establish the equations to determine the stresses in concrete due to applied
moment only when the section is uncracked.

A.6: Sec. 14.36.6, Part A

Version 2 CE IIT, Kharagpur


Q.7: Answer Q.6, if the section is cracked.

A.7: Sec. 14.36.6, Part B

Q.8: Answer Q.6. if the section is uncracked and subjected to combined axial
tension and moment.

A.8: Sec. 14.36.7, Part A

Q.9: Answer Q.6, if the section is cracked and subjected to combined axial
tension and moment. Assume the eccentricity of the tension force Ft is
small.

A.9: Sec. 14.36.7, Part B (i)

Q.10: Answer Q.6, if the section is cracked and subjected to combined axial
tension and moment. Assume the eccentricity of the tension force Ft is
large.

A.10: Sec. 14.36.7, Part B, (ii)

14.36.9 References
1. Reinforced Concrete Limit State Design, 6th Edition, by Ashok K. Jain,
Nem Chand & Bros, Roorkee, 2002.
2. Limit State Design of Reinforced Concrete, 2nd Edition, by P.C. Varghese,
Prentice-Hall of India Pvt. Ltd., New Delhi, 2002.
3. Advanced Reinforced Concrete Design, by P.C. Varghese, Prentice-Hall
of India Pvt. Ltd., New Delhi, 2001.
4. Reinforced Concrete Design, 2nd Edition, by S. Unnikrishna Pillai and
Devdas Menon, Tata McGraw-Hill Publishing Company Limited, New
Delhi, 2003.
5. Limit State Design of Reinforced Concrete Structures, by P. Dayaratnam,
Oxford & I.B.H. Publishing Company Pvt. Ltd., New Delhi, 2004.
6. Reinforced Concrete Design, 1st Revised Edition, by S.N. Sinha, Tata
McGraw-Hill Publishing Company. New Delhi, 1990.
7. Reinforced Concrete, 6th Edition, by S.K. Mallick and A.P. Gupta, Oxford &
IBH Publishing Co. Pvt. Ltd. New Delhi, 1996.
8. Behaviour, Analysis & Design of Reinforced Concrete Structural Elements,
by I.C.Syal and R.K.Ummat, A.H.Wheeler & Co. Ltd., Allahabad, 1989.
9. Reinforced Concrete Structures, 3rd Edition, by I.C.Syal and A.K.Goel,
A.H.Wheeler & Co. Ltd., Allahabad, 1992.

Version 2 CE IIT, Kharagpur


10. Textbook of R.C.C, by G.S.Birdie and J.S.Birdie, Wiley Eastern Limited,
New Delhi, 1993.
11. Design of Concrete Structures, 13th Edition, by Arthur H. Nilson, David
Darwin and Charles W. Dolan, Tata McGraw-Hill Publishing Company
Limited, New Delhi, 2004.
12. Concrete Technology, by A.M.Neville and J.J.Brooks, ELBS with
Longman, 1994.
13. Properties of Concrete, 4th Edition, 1st Indian reprint, by A.M.Neville,
Longman, 2000.
14. Reinforced Concrete Designer’s Handbook, 10th Edition, by C.E.Reynolds
and J.C. Steedman, E & FN SPON, London, 1997.
15. Indian Standard Plain and Reinforced Concrete – Code of Practice (4th
Revision), IS 456: 2000, BIS, New Delhi.
16. Design Aids for Reinforced Concrete to IS: 456 – 1978, BIS, New Delhi.

14.36.10 Test 36 with Solutions


Maximum Marks = 50, Maximum Time = 30 minutes

Answer all questions.

TQ.1: Draw a schematic diagram of different cases of tension structures made of


reinforced concrete.
(20
marks)

A.TQ.1: Fig. 14.36.2

TQ.2: Establish the equations to determine the stresses in concrete due to


applied moment only when the section is uncracked.
(15 marks)

A.TQ.2: Sec. 14.36.6, Part A

TQ.3: Answer TQ.2, if the section is cracked and subjected to combined axial
tension and moment. Assume the eccentricity of the tension force Ft is
small.
(15 marks)
A.TQ.3: Sec. 14.36.7, Part B (i)

Version 2 CE IIT, Kharagpur


14.36.11 Summary of this Lesson
This lesson presents the analysis and design of tension structures
following the working stress method as stipulated in IS 456 and IS 3370. The two
requirements i.e., the resistance to cracking and strength must be considered for
all types of tension structures whether they are in direct contact with the liquid or
not. The respective permissible stresses of concrete in direct tension, bending
tension, bending compression and shear are taken from the IS Codes for both
cases: when no cracking is allowed and when some cracking may be allowed.
The different cases of tension structures are classified depending on if they are in
direct contact with the liquid or not, if the tension is on the liquid face or not and
on the depth of the section. Understanding of the philosophy and concept of
design and the different equations established for different cases will help to
apply them in numerical problems. The numerical problems are taken up in Lesson
37.

Version 2 CE IIT, Kharagpur


Module
14
Tension Members
Version 2 CE IIT, Kharagpur
Lesson
37
Numerical Problems
Version 2 CE IIT, Kharagpur
Instructional Objectives:
At the end of this lesson, the student should be able to:

• select the category of the problem if it is a liquid retaining structure or a


structure having no contact with the liquid,

• identify the appropriate equations to be applied for the solution of a


particular problem belonging to specific category,

• select the permissible stresses in concrete and steel depending on the


category of the problems,

• apply the appropriate equations for the solution of the problems,

• solve the problems by establishing the equations from the first principle,

• revise the sections, if needed, to satisfy the requirements as per IS codes.

14.37.1 Introduction

The requirements for the design of tension structures / members are


discussed in Lesson 36. The stipulations of Indian Standard Codes are also
mentioned. Tension structures / members are classified depending on (i) if they
are liquid retaining or not having any contact with the liquid, (ii) if the tensile
stresses are developed due to pure axial force, pure moment or combined effects
of axial force and moment, (iii) if the tension is developed on the liquid face or on
the remote face and (iv) if the depth of the member is less than, equal to or
greater than 225 mm when the tension is developed in the remote face.
Accordingly, the permissible stresses of concrete and steel are different. The
equations are also different based on if the section is assumed to be cracked or
uncracked. All these issues are explained in Lesson 36.

This lesson explains the applications of the established equations for the
different cases as mentioned above through several numerical problems. The
problems are solved in step-by-step mostly utilising the equations. However, a
few problems are solved from the first principle for better understanding.
Understanding the solutions of illustrative examples and solutions of the practice
and test problems will give confidence in applying the appropriate equations and
selecting the permissible stresses for analysing and designing tension structures
or members using working stress method and following the stipulations of Indian
Standard Codes.

Version 2 CE IIT, Kharagpur


14.37.2 Numerical Problems

Problem 1. Determine the dimensions and area of tension steel of a


reinforced concrete rectangular tie member of a truss for
carrying a direct tensile force Ft = 470 kN using M 25 and Fe 415.

Solution 1. The theory of such tensile members made of reinforced concrete is


discussed in sec.14.36.5 of Lesson 36. We have Eqs. 14.1 and 14.2
to determine the required parameters. The modular ratio m of M 25
concrete is 93.33/8.5 = 10.98, fst of Fe 415 is 230 N/mm2 and ftd ≤ 3.2
N/mm2 of M 25 concrete (Table 13.1 of Lesson 34).

Step 1. Determination of Ast


From Eq. 14.1 of Lesson 36, we have: Ast = Ft / fst assuming that fst,
the actual stress will reach the value of σst = 230 N/mm2, which gives
Ast = 470,000/230 = 2043.48 mm2. Provide 4 bars of 20 mm diameter
and 4 bars of 16 mm diameter (= 2060 mm2).

Step 2. Determination of width and depth of the rectangular tie member


From Eq. 14.2 of Lesson 36, we have: Ac = (Ft /ftd) – m Ast
or Ag = (Ft / ftd) – (m – 1)Ast, where Ag = gross area of the tie member.
Substituting the values of Ft =470 kN, ftd = σtd = 3.2 N/mm2, m = 10.98
and Ast = 2060 mm2, we get Ac = 126316.2 mm2. Provide 300 mm x
425 mm (= 127500 mm2) as the width and depth of the tie member.

Step 3. Checking for the tensile stress in concrete


From Eq. 14.2 of Lesson 36, we have:
ftd = Ft / (Ac + m Ast) = Ft / {Ag + (m – 1)Ast}
= 3.17 N/mm2 < 3.2 N/mm2. Hence, ok

Therefore, the tie member of 300 mm x 425 mm with 4 bars of 20 mm


diameter and 4 bars of 16 mm diameter is the solution.

Problem 2. Design a bunker wall of 250 mm depth for resisting a moment of


33 kNm using M 20 and Fe 415.

Solution 2. Let us assume the section as cracked. The necessary equations of a


cracked section subjected to pure flexure are given in sec.14.36.6B.
For M 20 grade of concrete, m = 93.33/7 = 13.33 and kb = 93.33 /
(σst + 93.33), as per Eq. 13.3 of Lesson 36. Here, kb = 0.288, using

Version 2 CE IIT, Kharagpur


σst = 230 N/mm2 and assume d = 225 mm from the given value of D
= 250 mm.

Step 1. Determination of preliminary area of steel


The lever arm = jd = (1 – kb / 3) d = 0.904 (225) = 203.4 mm.
Therefore, from Eq. 13.8 of Lesson 34, preliminary Ast = M/{σst (lever
arm)} = 33 (106) / (230) (203.4) = 705.399 mm2. Let us provide 12
mm diameter bars @ 160 mm c/c to have 707 mm2/m.

Step 2. Determination of depth of neutral axis

The depth of neutral axis can be determined from Eq. 13.16 of


Lesson 34. Alternatively, we can determine x by taking moment of
the area of concrete and steel, converting it into equivalent concrete.
With reference to Fig. 14.37.1, taking moment of the two areas about
the neutral axis, 1000 (x2/2) = 13.33 (707) (225-x) or x2 + 18.84862 x
– 4240.9395 = 0, which gives x = 56.38 mm. This gives the lever arm
= 225 – 56.38/3 = 206.21 mm.

Step 3. Determination of tensile stress of steel


Tensile stress of steel fst = M/{(Ast) (lever arm) = 33 (106) / {(707)
(206.21)} = 226.35 N/mm2 < 230 N/mm2. Hence, the section is ok.

Step 4. Checking of the maximum bending compression stress in


concrete

Version 2 CE IIT, Kharagpur


Figure 14.37.2a shows the section. The maximum bending
compression stress of concrete is determined either form the stress
distribution diagram of Fig. 14.37.2b or after determining the moment
of inertia of the section about the neutral axis. From the stress
distribution diagram of Fig. 14.37.2b, we have: fcb = (fst / m) {x / (d-x)}
= (226.35 / 13.33) (56.38 / 168.62) = 5.678 N/mm2 〈 7 N/mm2.

The moment of inertia about the neutral axis is:

Iyy = 1000 (56.38)3 / 3 + (13.33) (707) (168.62)2 = 0.3277 (109) mm4.


Then, fcb = 33 (106) (56.38) / {0.3277 (109)} = 5,678 N/mm2 〈 7
N/mm2. Hence, the section, as shown in Fig. 14.37.2a, is ok.

Problem 3. Determine the area of tensile steel of a singly-reinforced bunker


wall of depth 250 mm subjected to Ft = 50 kN/m and M = 35
kNm/m at the horizontal level. Use M 20 and Fe 415 grade of
steel.

Solution 3. In this problem, the eccentricity of the tension force Ft is M/Ft =


35(103) / 50 = 700 mm, i.e., the tension force is acting outside the
section (Figs. 14.36.6a and b). The governing equations of such
section are given in sec. 14.36.7.B(ii), assuming the section as
cracked. Given D = 250 mm, so d = 225 mm. The value of the
modular ratio m = 93.33 / 7 = 13.33. For M 20 concrete fcb ≤ 7
N/mm2. Let us assume j = 0.87.

Version 2 CE IIT, Kharagpur


Step 1. Preliminary area of tension steel
From Eq. 14.27 of Lesson 36, we have:

Ast =(Ft /σst) {1 + (e + 0.5 D – d) / jd}


(14.27)

or Ast = (5000/230) {1 + (700 + 125 – 225) / (0.87) (225)} = 883.72 mm2.


Provide 12 mm diameter bars @ 120 mm c/c to have 942 mm2.

Step 2. Determination of the depth of the neutral axis


From Eq. 14.23 of Lesson 36, we get
m(Ast) (d-x) (e+0.5 D – d) = 0.5 bx2(e + 0.5D – x/3)
or 13.33 (942) (225 – x) (700+125-225) = 500 x2 (700 +125-x/3)
or x3 – 2475 x2 – 45204.696 x + 1017105.6 = 0

By Newton’s method, we get x = 56.171 mm.

Step 3. Determination of stresses of concrete and steel


From Eq. 14.22 of Lesson 36, we have:
fcb = Ft /{m Ast (d –x)/x – (0.5 b x)}
= 50000 / {(13.33) (942) (225 – 56.171)/56.171 – 0.5 (1000)
(56.171)}
= 5.18 N/mm2 〈 7.00 N/mm2

From Eq. 14.15 of Lesson 35, we have:


fst = m fcb (d – x) / x = 13.33 (5.18) (225-56.171)/56.171
= 207.54 N/mm2 〈 230 N/mm2.
Hence, the design is ok.

Problem 4. Design an interior slab panel of dimensions 4000 mm x 4000


mm of a flat bottom water tank to support 3 m of water. Use M
20 concrete and Fe 415 steel.

Solution 4. This is a slab panel of liquid retaining structure. We have to


determine the depth of the slab and area of steel by trial and error.
Let us assume D = 250 mm, and d = 200 mm giving a cover of 50
mm in the top liquid surface. The cover at the bottom surface is
assumed to be 30 mm.

Version 2 CE IIT, Kharagpur


Step 1. Evaluation of loads

Loads of 3 m height of water = 30000 N/m2


Loads of plastering (assumed) = 500 N/m2
Weight of reinforced concrete slab (of depth
= 250 mm) = 25000 (250) (10-3) = 6250 N/m2
__________
Total loads = 36750 N/m2

Step 2. Calculation of bending moments


Table 26 of IS 456 gives the values of coefficients ∝x = 0.032 for the
negative moment and ∝y = 0.024 for the positive moment. Thus,
we have, negative moment = (0.032) (36750) (4) (4) = 18.816 kNm
and positive moment = (0.024) (36750) (4) (4) = 14.112 kNm.

Step 3. Preliminary areas of steel


Assuming fst = σst = 150 N/mm2, kb = 93.33/(150+93.33) = 0.38,
Lever arm = jd = (0.87) (200) = 174 mm for the negative moment
and lever arm = jd = 0.87(220) = 191.4 mm for the positive moment
at mid-span, the areas of steel are as follows:
At the support = M/σst jd = 18816000 / (150) (174) = 720.92 mm2
At the mid-span = M/σst jd = 14112000 / (150) (191.4) = 491.54
mm2.
Provide 10 mm diameter bars @ 100 mm c/c to give 785 mm2 at
the support and 10 mm diameter bars @ 150 mm c/c at the mid-
span to give 524 mm2.

Step 4. Checking for stresses and moment of resistance of the


section

(A) At the support

Version 2 CE IIT, Kharagpur


Figure 14.37.3 shows the section at the support, which is assumed
as uncracked. The depth of neutral axis x is determined by taking
moment of the areas of concrete and steel about the bottom, which
gives:
x = {(1000) (250) (125) + (12.33) (785) (200)} / {(1000) (250) +
(12.33) (785)} = 127.79 mm; D – x = 250 – 127.79 = 122.21 mm.
The moment of inertia about the neutral axis is:
Iyy =(1000) (122.21)3 / 3 + (1000) (127.79)3 / 3 + (12.33) (785)
(122.21 – 50)2 = (1.35) (109) mm4

The moment of resistance of the section is determined using ftb =


σtb = 1.7 N/mm2, as given below:
M =ftb (Iyy) / (D-x) = 1.7 (1.35) (109) / 122.21 = 18.7 kNm < 18.816
kNm. Hence, the design has to be revised. Accordingly, the
checking of stresses at the mid-span for positive moment is not
carried out.

Step 5. Revised Section

Version 2 CE IIT, Kharagpur


(A) The depth of the section is increased to 270 mm. Accordingly, d =
220 mm for support section and d = 240 mm for mid-span section.
With j = 0.87, the lever arm at the support section = 0.87 (220)
=191.4 mm and at the mid-span section jd = 0.87 (240) = 208.8
mm. We have to add the extra self-weight of slab.

Earlier total loads in Step 1 = 36750 N/m2


Additional load of 20 mm thickness = 500
(25000) (20) / 103 = 500 N/m2
___________________
Total revised loads = 37250 N/m2

Version 2 CE IIT, Kharagpur


Revised negative moment = (0.032) (37250) (16) = 19.072 kNm
Revised positive moment = (0.024) (37250) (16) = 14.304 kNm

Revised negative steel = (19072000) / (150) (191.4) = 664.298 mm2


and Revised positive steel = (14304000) / (150) (208.8) = 456.7 mm2.

Provide 10 mm diameter bars @100 mm c/c (= 785 mm2) at the support and 10
mm diameter bars @ 150 mm c/c (= 524 mm2) at the mid-span, as shown in
Figs. 14.37.4a and 14.37.5a, respectively.

Step 6. Checking of stresses and moment of resistance

(A) At the support (Fig. 14.37.4b)

The depth of neutral axis x = {1000 (270) (135) + (112.33) (785)


(220)} / {(1000) (270) + (12.33) (785)} = 137.94 mm.
(D-x) = 270 – 137.94 = 132.06 mm and (d-x) = 220 -137.94 = 82.06
mm.

Iyy = {(1000) (137.94)3 / 3} + {(1000) (132.06)3 / 3} + {(12.33) (785)


(82.06)2} = 1.71 (109) mm4

Moment of resistance = (1.7) (1.71) (103) / 132.06 = 22.01 kNm 〉


19.072 kNm.

ftb = 19.072 (106) (132.06) / (1.71) (109) = 1.47 N/mm2 〈 1.7 N/mm2.
fcb = 19.072 (106) (137.94) / (1.71) (109) = 1.54 N/mm2 〈 7 N/mm2.
fst = (13.33)(19.072) (106)(82.06) / (1.71)(109) = 12.2 N/mm2 〈 230
N/mm2

Hence, the section at the support is ok.

(B) At the mid-span (Fig. 14.37.5b)

The depth of neutral axis x = {(1000) (270) (135) + (12.33) (524)


(240)} / {(1000) (270) + (12.33) (524)} = 137.45 mm, D - x = 132.55
mm and d-x = 102.55 mm.

Version 2 CE IIT, Kharagpur


Iyy = {(1000) (137.45)3 / 3}+ {(1000) (132.55)3 / 3} + {(12.33) (524)
(102.55)2} = 1.71 (109) mm4.

Moment of resistance of the section (when ftb = 1.7 N/mm2) = 1.7


(1.71) (103)/(132.55) = 21.93 kNm 〉14.304 kNm.

When ftb = 1.7 N/mm2, fcb = 1.7 (137.45) / (132.55) = 1.7642 N/mm2.
The moment of resistance (when fcb = 1.7642 N/mm2) = 1.7642
(1.71) (103)/(137.45) = 21.94 kNm 〉 14.304 kNm.

For the positive moment of 14.304 kNm, ftb = (14.304) (132.55) (106)
/ (1.71) (109) = 1.109 N/mm2 〈 1.7 N/mm2 and fcb = (14.304)
(137.45) (106) / (1.71) (109) = 1.145 N/mm2 〈 7 N/mm2

Problem 5. Determine the moment of resistance of the slab of Fig. 14.37.6


subjected to moment alone using M 20 and Fe 415.

Solution 5. Since tension is on the remote face and the depth of the slab is
more than 225 mm, we use cracked section. The value of m =
93.33/7 = 13.33.

Taking moment of area of concrete and steel about the top face.

500 x2 = (13.33) (942) (250 - x)


or, x2+ 25.11372 x – 6278.43 = 0
which gives x = 67.67 mm. Accordingly, d – x = 250 - 67.67 =
182.33 mm and D –x = 280 – 67.67 = 212.33 mm.
Version 2 CE IIT, Kharagpur
Moment of resistance from steel is determined using fst = σst = 190
N/mm2. Then M = σst Ast (d –x/3) = 190(942) (250 – 67.67/3) (10-6)
= 40.71 kNm.

When fst = σst = 190 N/mm2, fcb = 190 (67.67) / (182.33) (13.33) =
5.29 N/mm2 〈 7 N/mm2.
The moment of resistance from compression concrete = 0.5(67.67)
(5.29) (1000) (250 - 67.67/3) = 40.707 kNm.

Problem 6. Design the wall of a water tank using concrete of grade M 20


and steel of grade Fe 415, when subjected to Ft = 60 kN/m and
M = 25 kNm/m.

Solution 6. Since both Ft and M are acting on the wall of a liquid retaining
structure, the design must satisfy Eq. 14.8 of Lesson 36, i.e.,
(ftd /σtd) + (ftb /σtb) ≤ 1.0 (14.8)
where σtd = 1.2 N/mm and σtb = 1.7 N/mm for M 20 concrete, as
2 2

given in Table 14.1 of Lesson 36, when the tension is on the liquid
face. The section is considered as uncracked.

Step 1. Preliminary values of d, D, Ast1 for moment and Ast2 axial


force

Assuming σst = 150 N/mm2, we get kb = 93.33/(150+93.33) = 0.38


and j = 1 - 0.38/3 = 0.87. Assuming pt = 0.4 per cent, we get from:
Version 2 CE IIT, Kharagpur
M = Ast1 σst jd = (pt jσst / 100) (bd2)
or, d = (100 M/σst pt j b)1/2 = {(100) (25000 000)/(150) (0.4) (0.87)
(1000)}1/2 = 218.8 mm. Provide d = 250 mm with cover of 50 mm, D
= 300 mm.

Ast1 for moment (0.4 per cent) = 0.4 (300) (1000) / 100 = 1200 mm2.
Ast2 for Ft = 60000 / 150 = 400 mm2.

Total Ast = Ast1 + Ast2 = 1200 + 400 = 1600 mm2. Provide 16 mm


diameter bars @ 120 mm c/c to give 1675 mm2, as shown in Fig.
14.37.7a.

Step 2. Sectional properties

Figure 14.37.7a shows the section.


x = {1000 (300) (150) + (12.33) (1675) (250)} / {1000 (300) +
(12.33) (1675)} = 156.44 mm.

D – x = 300 – 156.44 = 143.56 mm, and d – x = 93.56 mm.


Iyy = 1000 (143.56)3 / 3 + 1000 (156.44)3 / 3 + (12.33) (1675)
(93.56)2
= 2.443 (109) mm4.
A = area of the section = 300 (1000) + (12.33) (1675) = 320652.75
mm2.

Step 3. Values of stresses and checking of Eq. 14.8

ftb = (25000 000) (143.56) / (2.443) (109) = 1.469 N/mm2 〈 1.7


N/mm2.
ftd = 60000 / 320652.75 = 0.187 N/mm2 〈 1.2 N/mm2
(ftb / σtb) + (ftd / σtd) = (1.469 / 1.7) + (0.187 / 1.2) = 1.02 〉 1.0.
Hence, the section has to be revised.

The revision can be done in two ways: (i) increasing Ast1 to 0.5 per
cent keeping the depth of the section = 300 mm, and / or (ii)
increasing the depth of the section by 20 mm. We try with the first
one in the next step.

Step 4. Revised section

Version 2 CE IIT, Kharagpur


With D as 300 mm, Ast1 = 0.5 (300) (1000) / 100 = 1500 mm2 and
Ast2 = 400 mm2 (as in earlier). So, Ast = 1900 mm2. Provide 20 mm
diameter bars @ 160 mm c/c to give 1963 mm2 as shown in Fig.
14.37.8a.

x ={(1000) (300) (150) + (12.33) (1963) (250)}/{(1000) (300) +


(12.33) (1963)} = 157.47 mm.
D – x = 300 – 157.47 = 142.53 mm and d –x = 250 – 157.47 =
92.53 mm.
I = 1000 (157.47)3/3 + 1000 (142.53)3/3 + (12.33) (1963) (92.53)2 =
2.47(109) mm4
A = (300) (1000) + (12.33) (1963) = 324203.79 mm2.
ftb = 25(106) (142.53) / 2.47(109) = 1.44 N/mm2
ftd = 6000 /324203.79 = 0.185 N/mm2

(ftb / σtb) + (ftd / σtd) = (1.44 / 1.7) + (0.185/1.2) = 1.0012 〉 1.0.


So, we further increase the depth of the concrete and repeat the
calculations of Step 4 in Step 5.

Step 5. Further revision

Version 2 CE IIT, Kharagpur


Now, the revised section has: D = 320 mm, d = 270 mm and Ast
=1963 mm2 (20 mm diameter bars @ 160 mm c/c) as shown in
Figs. 14.37.9a and b. The following are the results:

x = 167.77 mm
D –x = 152.23 mm
I = (3.003) (109) mm4
A = 344203.79 mm2
ftb = 1.267 N/mm2
ftd = 0.174 N/mm2

and (ftb /σtb) + (ftd / σtd) =0.745 + 0.145 = 0.89 〈 1.0

Hence, the revised section of Fig. 14.37.9a is ok.

Problem 7. Determine the area of tensile steel of the wall of a water tank of
250 mm depth and subjected to M = 35 kNm/m only. Use M 20
and Fe 415. Redesign, if needed.

Solution 7. Given data are D = 250 mm, d = 225 mm, M 20 and Fe 415. Since,
the tension face is at a distance of 225 mm away, we consider fst
≤190 N/mm2 and assume the section as cracked. We determine the
preliminary area of steel first.

Version 2 CE IIT, Kharagpur


Step 1. Determination of preliminary area of steel

For M 20 concrete, m = 93.33/7 = 13.33, kb = 93.33 / (σst + 93.33)


= 93.33 / (190 + 93.33) = 0.329 and j = 1 – kb / 3 = 1 – 0.329/3 =
0.89.

Preliminary Ast = M/σst jd = 35(106) / (190) (0.89) (225) = 919.9


mm2. Provide 12 mm diameter bars @ 120 mm c/c to have 942
mm2, as shown in Fig. 14.37.10a.

Step 2. Properties of section

Taking moment of the area about the top surface (Fig. 14.37.10a),
we have:
500 x2 = (13.33) (942) (225 – x)
or, x2 + 25.11372 x – 5650.587 = 0. The solution is x = 76.21 mm.

Iyy = 1000 (76.21)3/3 + (13.33) (942) (225 – 76.21)2 = 0.425 (109)


mm4.

Step 3. Checking for the stresses

Version 2 CE IIT, Kharagpur


Compression stress in concrete in bending fcb = Mx/Iyy = 35 (106)
(76.21) / (0.425) (109) = 6.28 N/mm2. Tensile stress in steel = fst =
m(fcb) (d-x) / x = (13.33) (6.28)(148.79)/76.21 = 163.43 N/mm2 〈 190
N/mm2. Hence, the section is ok.

Problem 8. Check the acceptability of the design of the base slab of a


water tank as shown in Fig. 14.37.11, which is subjected to Ft =
60 kN/m and M = 6 kNm/m. Assume the section as uncracked.
Use M 20 and Fe 415.

Solution 8. For M 20 concrete m = 93.33/7 = 13.33. Equations 14.2 and 14.4 of


Lesson 36 are used in determining ftd and ftb, respectively. Since the
section is symmetrically reinforced, the depth of the neutral axis =
85 mm.

Iyy = 1000 (170)3 / 12 + (12.33) (503) (2) (50)2 = 0.4404 (109) mm4
ftd = Ft /{(1000) (170) + (12.33) (2) (503)} = 0.329 N/mm2
ftb = 6(106) (85) / (0.4404) (109) = 1.158 N/mm2

(ftd / σtd) + (ftb / σtb) = (0.329 /1.2) + (1.158/1.7) = 0.224+0.681 =


0.955 〈 1.0. Hence, the design is ok.

Version 2 CE IIT, Kharagpur


14.37.3 Practice Questions and Problems with Answers

Q.1: Solve Problem 1 of sec.14.37.2 when Fe 250 is used.

A.1: Given data are Ft = 470 kN, concrete grade is M 25 and steel grade is Fe
250. Modular ratio m = 10.98 (see solution 1 of sec. 14.37.2). The values
of fst of Fe 250 ≤ 140 N/mm2 and ftd ≤ 3.2 N/mm2.

Step 1. Determination of Ast

From Eq. 14.1 of Lesson 36, we have Ast = Ft /σst = 470000/140 = 3357.14
mm2. Provide 6 bars of 25 mm diameter and 4 bars of 12 mm diameter to
have Ast = 2945 + 452 = 3397 mm2.

Step 2. Determination of width and depth of the section

From Ag = (Ft / ftd) – (m – 1) Ast, we have Ag = 112972.94 mm2. Provide


300 mm × 380 mm section.

Step 3. Checking for tensile stress of concrete

From Eq. 14.2 of Lesson 36, we have:


ftd =Ft / (Ac + m Ast) = Ft / {Ag + (m –1) Ast} = 3.18 N/mm2 〈 3.2 N/mm2.
Hence, the section of 300 mm × 380 mm with 6 bars of 25 mm diameter
and 4 bars of 12 mm diameter is ok.

Version 2 CE IIT, Kharagpur


Q.2: Determine the moment of resistance of the section shown in Fig. 14.37.12.
Use M 20 and Fe 415.

A.2: The section is considered uncracked as the liquid face is having tension.
The modular ration m = 93.33 / 7 = 13.33 and ftb ≤ 1.7 N/mm2.

Step 1. Sectional properties

The depth of the neutral axis, as shown in Fig. 14.37.12, is obtained by


taking moment of the areas about the top, which gives:

x = {(300) (900) (450) + (1100) (290) (610 + 145) + (12.33) (2463) (900 –
85) + (1.5) (12.33) (942) (60)} / {(300) (900) + (1100)(290) + (12.33)
(2463) + (1.5) (12.33) (942) = 609.53 mm,
D – x = 900 – 609.53 = 290.47 mm.

The moment of inertia about the neutral axis yy is:


Iyy = (300) (609.53)3 / 3 + (300) (0.47)3/3 + 1400 (290)3 / 12 + (1400) (290)
(145 + 0.47)2 + (12.33) (2463) (290.47 – 85)2 + (1.5) (12.33) (942) (609.53
– 60)2 = (40.626) (109) mm4.

Moment of resistance = (1.7) (40.626) (109) / 290.47 = 237.767 kNm/m.

Q.3: Solve Problem 5 of sec. 14.37.2 (Fig. 14.37.6a) using Fe 250.

Version 2 CE IIT, Kharagpur


A.3: Given data are: D = 280 mm, d = 250 mm, Ast = 12 mm diameter bars @
120 mm c/c = 942 mm2, M 20 and Fe 250 (Fig. 14.37.6a).

Modular ratio m = 93.33 / 7 = 13.33, x = 67.67 (as obtained in the solution


of Problem 5), σst = 125 N/mm2 and (d – x) = 250 – 67.67 = 182.33 mm.
Therefore, M = σst Ast (d – x/3) = (125) (942) (250 – 67.67/3) = 26.78 kNm.

fcb = σst (67.67) / (13.13) (182.33) = 3.48 N/mm2. The moment of


resistance from compression concrete = 125 (942) (250 – 67.67/3) =
26.78 kNm 〈 40.71 kNm of the slab of Problem 5.

It is thus seen that though the slab of Problem 5 and this one have x =
67.67 mm but moment of resistance of this slab is much less when Fe 250
is used.

14.37.4 Reference

1. Reinforced Concrete Limit State Design, 6th Edition, by Ashok K. Jain,


Nem Chand & Bros, Roorkee, 2002.
2. Limit State Design of Reinforced Concrete, 2nd Edition, by P.C. Varghese,
Prentice-Hall of India Pvt. Ltd., New Delhi, 2002.
3. Advanced Reinforced Concrete Design, by P.C. Varghese, Prentice-Hall
of India Pvt. Ltd., New Delhi, 2001.
4. Reinforced Concrete Design, 2nd Edition, by S. Unnikrishna Pillai and
Devdas Menon, Tata McGraw-Hill Publishing Company Limited, New
Delhi, 2003.
5. Limit State Design of Reinforced Concrete Structures, by P. Dayaratnam,
Oxford & I.B.H. Publishing Company Pvt. Ltd., New Delhi, 2004.
6. Reinforced Concrete Design, 1st Revised Edition, by S.N. Sinha, Tata
McGraw-Hill Publishing Company. New Delhi, 1990.
7. Reinforced Concrete, 6th Edition, by S.K. Mallick and A.P. Gupta, Oxford &
IBH Publishing Co. Pvt. Ltd. New Delhi, 1996.
8. Behaviour, Analysis & Design of Reinforced Concrete Structural Elements,
by I.C.Syal and R.K.Ummat, A.H.Wheeler & Co. Ltd., Allahabad, 1989.
9. Reinforced Concrete Structures, 3rd Edition, by I.C.Syal and A.K.Goel,
A.H.Wheeler & Co. Ltd., Allahabad, 1992.
10. Textbook of R.C.C, by G.S.Birdie and J.S.Birdie, Wiley Eastern Limited,
New Delhi, 1993.
11. Design of Concrete Structures, 13th Edition, by Arthur H. Nilson, David
Darwin and Charles W. Dolan, Tata McGraw-Hill Publishing Company
Limited, New Delhi, 2004.

Version 2 CE IIT, Kharagpur


12. Concrete Technology, by A.M.Neville and J.J.Brooks, ELBS with
Longman, 1994.
13. Properties of Concrete, 4th Edition, 1st Indian reprint, by A.M.Neville,
Longman, 2000.
14. Reinforced Concrete Designer’s Handbook, 10th Edition, by C.E.Reynolds
and J.C. Steedman, E & FN SPON, London, 1997.
15. Indian Standard Plain and Reinforced Concrete – Code of Practice (4th
Revision), IS 456: 2000, BIS, New Delhi.
16. Design Aids for Reinforced Concrete to IS: 456 – 1978, BIS, New Delhi.

14.37.5 Test 37 with Solutions

Maximum Marks = 50 Maximum Time = 30


minutes

TQ.1: The base slab of the water tank, as shown in Fig. 14.37.13a, is subjected
to Ft = 60 kN/m and M = 1.2 kNm/m. Determine the stresses in steel and
concrete. Use M 20 and Fe 415.
[20 marks]
2 2
A.TQ.1:Given D = 200 mm, As1 = 785 mm , As2 = 785 mm , M 20 and Fe 415.

Version 2 CE IIT, Kharagpur


The eccentricity of Ft = e = 1.2 (103) /60 = 20 mm, i.e., within the section.
The whole section is in tension and the section is cracked. We have Eqs.
14.9 and 14.10 as
Ft = fst1 As1 + fst2 As2 (14.9)

Ft (d1 – e) – fst2 As2 (d1 + d2) = 0


(14.10)

From Eq. 14.10, we have: fst2 = Ft (d1 – e) / {As2 (d1 + d2)} = (60000) (50 -
20)/(785) (100) = 22.93 N/mm2 〈 150 N/mm2.
Using the value of fst2 in Eq. 14.9, we have fst1 = (Ft - fst2 As2) / As1 = {60000
– (22.93) (785)}/785 = 53.50 N/mm2 〈 150 N/mm2.

Modular ratio m for M20 = 93.33 / 7 = 13.33.

ftd = Ft /{1000(200) + (13.33 –1) (785) (2)} = 0.27 N/mm2


ftb = fst1 / m = 53.50 / 13.33 = 4.0 N/mm2 〉 1.7 N/mm2. It shows that the section is
cracked.

TQ.2: Determine the area of tensile steel of a singly-reinforced bunker wall of


depth 300 mm subjected to Ft = 60 kN/m and M = 42 kNm/m at the
horizontal level. Use M 20 and Fe 415 grade of steel.
(30 marks)

A.TQ.2: In this problem, the eccentricity of the tensile force Ft = M/Ft =42000 / 60
= 700 mm, i.e., e is outside the section. Given data are: D = 300 mm, d =
275 mm. For M 20 concrete, m = 93.33/7 = 13.33, fcb ≤ 7 N/mm2. Let us
assume j = 0.87.

Step 1. Preliminary area of tension steel

From Eq. 14.27 of Lesson 36, we have: Ast = (Ft /σst) {1 + (e + 0.5 D - d) /
jd} = (60000 /230) {1 + (700 + 150 - 275) / 0.87 (275)} = 887.83 mm2.
Provide 12 mm diameter bars @ 120 mm c/c to give 942 mm2.

Step 2. Depth of the neutral axis

Eq. 14.23 of Lesson 36 gives: m (Ast) (d – x) (e + 0.5 D – d) = 0.5 bx2 (e +


0.5 D – x/3), which gives: (13.33) (942) (275 – x) (700 +150-275) = 500 x2
(700 + 150 – x/3)

Version 2 CE IIT, Kharagpur


or x3 – 2550 x2 – 43321.167 x + 11913320.93 = 0.The solution of the
equation is x = 61.0265 mm by trial and error.

Step 3. Determination of stresses of concrete and steel

Equation 14.22 gives: fcb = Ft / {m Ast (d – x) / x – (0.5 bx)} = 60000 / {(13.33)


(942) (275 – 61.0265) / 61.0265 – 500 (61.0265)} = 4.44 N/mm2 〈 7 N/mm2.

Equation 14.15 gives: fst = m fcb (d – x) / x = (13.33) (4.44) (275 – 61.0265) /


61.0265 = 207.52 N/mm2 〈 230 N/mm2 . Hence, the solution is o.k.

14.37.6 Summary of this Lesson

This lesson illustrates the applications of the equations explained in


Lesson 36 for the analysis and design of tension members employing
working stress method as stipulated in IS Codes. These structures may be
either liquid retaining structures or may not have any contact with the
liquid. The permissible stresses in concrete and steel depend on several
factors as explained in Lesson 36. The section may be considered
cracked or uncracked. Such tension structures may be subjected to axial
tension only, moment only or combinations of them. Illustrative examples
cover most of the categories of problems. For better understanding, some
of the problems are solved from the first principle instead of using the
equations directly. Understanding the illustrative examples and solutions
of the practice and test problems will help in applying the equations in
analysing and designing tension structures as per the stipulations of IS
Codes.

Version 2 CE IIT, Kharagpur


Module
15
Redistribution of
Moments
Version 2 CE IIT, Kharagpur
Lesson
38
Redistribution of
Moments – Theory and
Numerical Problems
Version 2 CE IIT, Kharagpur
Instructional Objectives:
At the end of this lesson, the student should be able to:

• explain the concept of redistribution of moments in the design of statically


indeterminate reinforced concrete structures,

• explain the behaviour of statically indeterminate reinforced concrete


structures with increasing loads till the collapse of the structures,

• state the advantages of redistributing the moments in statically


indeterminate reinforced concrete structures,

• identify structures when such redistributions are to be made based on the


analysis of structures,

• mention the reasons why full redistribution is not allowed in reinforced


concrete structures,

• specify the stipulations of IS Codes about the redistribution separately when


the structure is designed by working stress method and by limit state
method,

• apply the theories in solving numerical problems of statically indeterminate


beams of one or more spans as per the stipulations of IS Code.

15.38.1 Introduction
Statically indeterminate structures made of reinforced concrete like fixed
ended one span beams, continuous beams and frames are designed considering
internal forces like bending moment, shear force and axial thrust obtained from
structural analysis. Either one or several sections of these structures may have
peak values of the internal forces, which are designated as critical sections.
These sections are dimensioned and reinforced accordingly. Flexural members,
however, do not collapse immediately as soon as the loads at a particular section
cause bending moment exceeding the maximum resisting moment capacity of
that section. Instead, that section starts rotating at almost constant moment. This
is known as formation of plastic hinge at that section reaching its maximum
resisting moment capacity. The section then transfers loads to other sections if
the applied loads are further increased. This process continues till the structures

Version 2 CE IIT, Kharagpur


have plastic binges at sufficient sections to form a failure mechanism when it
actually collapses. However, significant transfer of loads has occurred before the
collapse of the structure. This transfer of loads after the formation of first plastic
hinge at section having the highest bending moment till the collapse of the
structure is known as redistribution of moments. By this process, therefore, the
structure continues to accommodate higher loads before it collapses.

The elastic bending moment diagram prior to the formation of first plastic
hinge and the final bending moment diagram just before the collapse are far
different. The ratio of the negative to positive elastic bending moments is no
more valid. The development of plastic hinges depends on the available plastic
moment capacity at critical sections. It is worth mentioning that the redistribution
of moment is possible if the section forming the plastic hinge has the ability to
rotate at constant moment, which depends on the amount of reinforcement
actually provided at that section. The section must be under-reinforced and
should have sufficient ductility.

This phenomenon is well known in steel structures. However, the


redistribution of moment has also been confirmed in reinforced concrete structure
by experimental investigations. It is also a fact that reinforced concrete structures
have comparatively lower capacity to rotate than steel structures. Yet, this
phenomenon is drawing the attention of the designers. Presently, design codes
of most of the countries allow the redistribution up to a maximum limit because of
the following advantages:

1) It gives a more realistic picture of the actual load carrying capacity of


the indeterminate structure.

2) Structures designed considering the redistribution of moment (though


limited) would result in economy as the actual load capacity is higher
than that we determine from any elastic analysis.

3) The designer enjoys the freedom of modifying the design bending


moments within limits. These adjustments are sometimes helpful in
reducing the reinforcing bars, which are crowded, especially at
locations of high bending moment.

Version 2 CE IIT, Kharagpur


15.38.2 Two Span Beam

Version 2 CE IIT, Kharagpur


Let us take up a two-span beam of uniform cross-section, as shown in Fig.
15.38.1a, with the following assumptions:

a) The ultimate moments of resistance of sections at B and D are –Mun


and +Mup, respectively.

b) There will not be any premature shear failure so that the sections can
attain the respective ultimate moment capacity.

c) The moment-curvature relationship for the ductile sections is the


idealised bilinear as shown in Fig. 15.38.2. It may be noted that for all
practical purposes only a limited rotation will take place.

d) All sections of the beam have the same constant flexural rigidity up to
the ultimate moment and moment remains constant at the ultimate
moment with increasing curvature.

e) The self-weight of the beam is neglected.

The beam is subjected to two point loads of magnitude P each and acting
at distances of l/2 from the two supports A and C, as shown in Fig. 15.38.1a. The
elastic bending moment diagram is shown in Fig. 15.38.1b. On increasing the
loads P, the ultimate moment of resistance Mun of cross-section at B will be
reached at the support first before reaching at other sections, which is shown in
Fig. 15.38.1d. The plastic hinge will be formed at the cross-section B (Fig.
15.38.1c). The two point loads P can still be increased as long as the plastic
hinge at B will rotate sufficiently. If the cross-section at B is brittle, the load will

Version 2 CE IIT, Kharagpur


decrease fast (see Fig. 15.38.2) and the beam will fail suddenly. Hence, the
beam will not carry any additional load. The cross-section at B has to be ductile
so that it undergoes rotation at the constant moment of resistance, which will
enable the beam to carry additional loads. This increase of load will continue until
the maximum positive moments in the span (at D and E) reach Mup, as shown in
Fig. 15.38.1f, when plastic hinges will form at D and E also. The three plastic
hinges at B, D and E will form the collapse mechanism (Fig. 15.38.1e). The
structure will fail at this stage carrying much higher loads. It is important to note
that the requirement of equilibrium will be satisfied at all stages, i.e., Mp and Mn ,
during the elastic phase, will follow the equation:

Mp + 0.5 Mn = Pl/4 (15.1)

From the structural analysis of the beam of Fig. 15.38.1a, we know that:

Mn = 6 Pl / 32 and Mp = 5 Pl / 32.

Substituting these values in Eq. 15.1, we find that the equation is satisfied where
Pl/4 is the maximum positive moment of a simply supported beam having a load
P at the centre of the beam. From the above values of Mn and Mp (= 6 Pl/32 and
5 Pl/32, respectively), we also note that:

Mn / Mp = 1.2 (15.2)

Version 2 CE IIT, Kharagpur


As the loads are increased till the formation of the first plastic hinge at B, this
ratio of Mn /Mp as 1.2 will be maintained, as shown in Fig. 15.38.3. This phase is
known as the elastic phase when the bending moments increases with
increasing loads maintaining the ratio of Mn / Mp as 1.2.

With further increase of the loads, the plastic hinge at B will rotate at the
constant moment Mun and positive moments at D and E will increase as shown in
Fig. 15.38.3. This phase is known as moment redistribution phase as the loads
are now transferred to sections, which have less moment. However, the cross-
section at B must have the ability to sustain the required plastic rotation at this
stage with increasing loads. As the value of Mp is now increasing when Mn is
remaining constant, the ratio of Mn / Mp will not be 1.2 any more. Thus, in the
redistribution phase, the additional moments at higher loads are to be
redistributed to the support and mid-span in such a manner that we get a similar
equation like Eq. 15.1, i.e.

Mup + 0.5 Mun =Pl/4 (15.3)

which means that, after the redistribution

Mun = Mn –M (15.4)

where M is some amount of moment by which negative moment at the support is


reduced. From Eqs. 15.3 and 15.4, we have: Mup = Pl/4 – 0.5 Mun = Pl/4 - 0.5
(Mn-M) = (Pl/4 – 0.5 Mn) + 0.5 M = Mp + 0.5 M, which we get by substituting Mp
= Pl/4 – 0.5 Mn from Eq. 15.1. Therefore, we have:

Mup = Mp + 0.5 M (15.5)

Thus, in the moment redistribution, if we reduce some amount M from the


negative moment Mun at support as in Eq. 15.4, we have to add 0.5 M to the span
moment Mp to get the Mup as in Eq. 15.5. The redistributed moments Mun and Mup
satisfy the equilibrium condition of Eq. 15.3. Similar equilibrium condition is also
satisfied at the elastic stage by the Mn and Mp as in Eq. 15.1.

The amount of moment M which will be reduced from the negative support
moment depends on the rotational capacity as per the actual reinforcement
provided in the cross-section. Furthermore, the deformation of the support should
be within acceptable limits under service loads.

In the extreme case, if the negative moment at the support Mun is reduced
to zero, i.e., M = Mn , we have from Eq. 15.5:

Mup = Mp + 0.5 Mn = Pl/4 (15.6)

Version 2 CE IIT, Kharagpur


That is, the unreinforced central support will crack and the continuous
beam will behave as two simply supported beams.

Thus, the choice of the bending moment diagram after the redistribution
should satisfy the equilibrium of internal forces and external loads. Moreover, it
must ensure the following:

1. The plastic rotations required at the critical sections should not


exceed the amount the sections can sustain.

2. The extent of cracking or the amount of deformation should not


make the performance unsatisfactory under service loads.

The redistribution of moments is permitted if the analysis of forces and


moments is done following linear elastic behaviour. Analysis by linear elastic
behaviour is a logical procedure in the working stress method of design where
the design concept is based on the assumptions of linear elastic behaviour of
materials up to the level of recommended safe stresses. However, analysis by
linear elastic theory and design by the limit state of collapse appear to be
somewhat contradictory. At the stress levels of 0.87 fy for steel and 0.67 fck for
concrete, the strains are not linearly related. Unfortunately, till now there is no
such analysis, which takes into account the complex behaviour of reinforced
concrete just before the collapse. Accordingly, the elastic theory of analysis of
forces and moments cannot be avoided at present except for the slabs.

Moreover, unlike steel structures, cross-section forming the first plastic


hinge in reinforced concrete members undergoes limited rotations, which is
insufficient for other sections to attain the desired Mup. Therefore, full
redistribution has been restricted in the design of reinforced concrete members.
These restrictions and stipulations of IS Codes are taken up in the next section.

15.38.3 Recommendations of IS 456


IS 456 recommends the redistribution of moments during the analysis
provided the analysis is done by linear elastic theory. Such redistribution of
moments is not permitted when either simplified analysis is done or coefficients
of cl.22.5 of IS 456 are used to determine the moments.

In the working stress method, cl. B-1.2 of IS 456 stipulates that the
moments over the supports for any assumed arrangement of loading, including
the dead load moments may each be increased or decreased by not more than
15 per cent, provided that these modified moments over the supports are used
for the calculation of the corresponding moments in the spans.

Version 2 CE IIT, Kharagpur


In the limit state of collapse method, cl.37.1 of IS 456 recommends the
redistribution of the calculated moment in continuous beams and frames
satisfying the following conditions:

1. Equilibrium between the internal forces and the external loads should be
maintained.

2. The ultimate moment of resistance provided at any cross-section of a


member after the redistribution should not be less than 70 per cent of the
moment at that cross-section obtained from an elastic maximum moment
diagram covering all appropriate combinations of loads.

3. The elastic moment at any cross-section in a member due to a particular


combination of loads shall not be reduced by more than 30 per cent of the
numerically largest moment given anywhere by the elastic maximum
moment diagram for the particular member, covering all appropriate
combinations of loads.

4. Cross-sections having moment capacity after redistribution less than that


of the elastic maximum moment shall satisfy the relationship:

(xu / d) + (δM / 100) ≤ 0.6 (15.7)

where

xu = depth of the neutral axis,


d = effective depth, and
δM = percentage reduction in moment.

5. Thirty per cent reduction in moment permitted in 3 above shall be


restricted to ten per cent for structures in which the structural frames
provide lateral stability.

15.38.4 Explanations of the Conditions Stipulated in IS Code


The first condition of maintaining the equilibrium between internal forces
and external loads is explained in sec.15.38.2 taking up a two-span continuous
beam. For the other conditions, let us use the following notations for the sake of
brevity:

Mew = elastic bending moment under working loads,


Meu = elastic bending moment under design loads i.e., the factored loads
considering partial safety factor of loads γf,
Mdu = design factored moment after redistribution, and

Version 2 CE IIT, Kharagpur


δM = percentage reduction in Meu.

Since γf = 1.5, we can write

Meu = 1.5 Mew


(15.8)

The second condition of cl.37.1 of IS 456, as given in sec. 15.38.3 of this lesson,
is expressed as

Mdu ≥ 0.7 Meu (15.9)

Beeby has reported that the redistribution as given in Eq. 15.8 is possible
before the lower bound rotation capacity is reached (Beeby, A.W, “The analysis
of beams in plane frames according to CP 110” Cement and Concrete
Association, U.K. Development Report No. 1, October 1978). Thus, Eq. 15.9
ensures the first criterion of the redistribution of moments in reinforced concrete
structures as given in sec. 15.38.2. Equation 15.9 also satisfies other
requirements as explained below.

Version 2 CE IIT, Kharagpur


Using the value of Meu from Eq. 15.8 into Eq. 15.9, we have

Mdu ≥ 0.7 (1.5 Mew) ≥ 1.05 Mew


(15.10)

This ensures that the design factored moments are greater than the
elastic moments everywhere in the structures. Figure 15.38.4 presents three
bending moment diagrams: (i) elastic moment under working loads (Mew), (ii)
elastic moment under ultimate loads (Meu) and (iii) design moment under ultimate
loads (Mdu) of the two-span beam of Fig. 15.38.1a. The moment diagram after
the redistribution (Mdu) satisfies the condition of Eq. 15.1 such that

b + 0.5 a = b′ + 0.5 a′
(15.11)

Version 2 CE IIT, Kharagpur


However, Mdu diagram is not satisfying the requirement of Eq. 15.10 as in the
zone GH of length x (Fig.15.38.4), the negative moment after redistribution is
less than the negative moment under working loads. To satisfy Eq. 15.10, the
bending moment in the zone should be as per Mew.

We now take up the fourth condition of cl.37.1 of IS 456 and given in sec. 15.38.3
of this lesson, which is expressed in Eq. 15.7 as

(xu/d) + (δM / 100) ≤ 0.6 (15.7)

This equation ensures that the cross-section of the member is under-reinforced,


as explained below.

Allowing the maximum redistribution of thirty per cent as per the third
condition of sec. 15.38.3, we have δM/100 = 0.3. Substituting this in Eq. 15.7, we
get xu/d ≤0.3. For (δM/100) less than thirty per cent, the corresponding xu/d can
be determined from Eq. 15.7. The values of xu,max/d for the three grades of steel
are given in Table 3.2 of Lesson 5. It is easy to verify that permitting the
redistribution of thirty per cent, the cross-sections remain under-reinforced. This
ensures low amount of steel reinforcement to have smaller value of xu which will
give higher value of the rotation φ as the rotation φ = 0.0035/xu.

It is evident, therefore, that for the analysis of determining the values of


bending moments after the redistribution, bending moment envelopes are to be
drawn satisfying the requirements discussed above. This is explained with the
help of illustrative examples in the next section.

Version 2 CE IIT, Kharagpur


15.38.5 Illustrative Examples

Version 2 CE IIT, Kharagpur


Problem 1. Draw the design bending moment diagram of the beam of Fig.
15.38.5a, clamped at both ends and carrying ultimate uniformly
distributed load of 24 kN/m with full redistribution of 30 per cent as
per IS 456.

Solution 1.

Step 1: Elastic bending moment diagram

The beam (Fig.15.38.5a) is statically indeterminate. The elastic


bending moment diagram is shown in Fig. 15.38.5b. The negative
and positive moments at the supports (A and B) and mid-span (C),
respectively, are:

MA = MB = - w l2 / 12 = - 24 (8) (8) / 12 = - 128 kNm


B

MC = + w l2/24 = + 24 (8) (8) / 24 = + 64 kNm

The distance x where the bending moment is zero is obtained by


taking moments of forces of the free body diagram (Fig. 15.38.5c)
about D after determining the vertical reaction at A.

Vertical reaction at A, VA = 24 (8) / 2 = 96 kN. At section D, Mx =


96x – 128 – 12x2. So, the value of x when Mx = 0 is obtained from
96x – 128 – 12x2 = 0 or, 3x2 – 24x + 32 = 0. The solution of the
equation is x =1.69 m and 6.31 m.

Step 2. Redistributed bending moment diagram

From Fig.15.38.5e, the maximum negative moment at A after the


full redistribution of 30 per cent = 0.7 (128) = 89.6 kNm. So, the
maximum positive moment at the mid-span C = w l2 / 8 – 89.6 =
102.4 kNm. The vertical reaction at A is: VA = 96 kN.

The point of inflection, i.e., the point where bending moment is


zero, is obtained by taking moment of forces about E of the free
body diagram shown in Fig. 15.38.5d. Thus, we have: 96x – 12x2 –
89.6 = 0, which gives x = 1.08 m and 6.92 m. The bending moment
diagram after the redistribution is shown in Fig. 15.38.5e.

Step 3. Envelope of bending moment diagrams

At x =1.08 m, the elastic bending moment is Mx = 96x – 128 – 12x2


= - 38.32 kNm, which becomes 0.7 (- 38.32) = - 26.824 kNm after
the redistribution. The envelope of the bending moment diagram is
shown in Fig. 15.38.5f.

Version 2 CE IIT, Kharagpur


Version 2 CE IIT, Kharagpur
Problem 2. Draw the design bending moment diagram of the beam of Fig.
15.38.6a, clamped at both ends and carrying two point loads of 30
kN each at distances of 3 m from the supports. Assume full
redistribution of 30 per cent as per IS 456.

Solution 2.

Step 1. Elastic bending moment diagram

The beam (Fig.15.38.6a) is statically indeterminate. The elastic


bending moment diagram is shown in Fig. 15.38.6b. The negative
and positive moments at the supports (A and B) and at mid-span
(E), respectively, are:

MA = MB = - 2 P l/9 = -2 (30) (9)/ 9 = - 60 kNm


ME = 90 – 60 = 30 kNm.

Vertical reaction at A = VA = 30 kN

Taking moment of all the forces about F at a distance of x (0 ≤ x ≤ 3


m) from A of the free body diagram shown in Fig. 15.38.6c, we
have 30 x – 60 = 0, which gives x = 2 m, where the bending
moment is zero.

Step 2. Redistributed bending moment diagram

Redistributed negative moment at A = 0.7 (60) = 42 kNm


Redistributed positive moment at E = 90 - 42 = 48 kNm

The point of zero moment (point G) is obtained by taking moments


of forces about G of the free body diagram shown in Fig. 15.38.6d,
when x ≤ 0 ≤ 3 m; gives 30x – 42 = 0. Therefore, x = 1.4 m.

The redistributed moment diagram is shown in Fig. 15.38.6e.

Step 3. Envelope of bending moment diagram

At G when x = 1.4 m, elastic moment = 30(1.4) – 60 = - 18 kNm.


The redistributed moment at G = 0.7 (-18) = -12.6 kNm. The
envelope of the bending moment diagrams is shown in Fig.
15.38.6f.

Version 2 CE IIT, Kharagpur


Version 2 CE IIT, Kharagpur
Problem 3. Draw envelope of the design moments of the two-span continuous
beam shown in Fig. 15.38.7a, carrying characteristic live load of 35
kN/m in addition to its characteristic self-weight. The cross-section
of the beam is 300 mm × 700 mm.

Solution 3. The two-span continuous beam of Fig. 15.38.7a is statically


indeterminate and the method of moment distribution is employed
for the analysis. Clause 22.4.1 of IS 456 stipulates the
arrangements of live loads, which are:

1) Design dead load on all spans with full design imposed loads on
two adjacent spans to get the maximum negative moment over
the support, and

2) Design dead load on all spans with full design imposed loads on
alternate spans to get the maximum span moment (positive
moment).

In this case of two-span continuous beam, therefore, we have the


following three load cases:

(i) For the maximum negative moment at the support B,


both the spans are loaded with self-weight and live
loads,

Version 2 CE IIT, Kharagpur


(ii) For the maximum positive moment in span AB, only
the span AB is loaded with live load and self-weight of
the beam in both the spans, and

(iii) For the maximum positive moment in span BC, only


the span BC is loaded with live loads and self-weight
of the beam in both the spans.

However, we are considering only cases (ii) and (iii) as they are the
mirror image of each other.

Step 1. Evaluation of design loads

Characteristic dead load of the beam = (0.3) (0.7) (25) = 5.25 kN/m.
Therefore, the design dead load with γf = 1.5 is (1.5) (5.25) = 7.875
kN/m. The design live load = 1.5 (35) = 52.5 kN/m.

Step 2. Load case (i): Elastic bending moment diagram

Figure 15.38.7b shows the load case (i) of the beam. The
calculations of the moment distribution are presented in Table 15.1.
Figure 15.38.7c presents the elastic bending moment diagram and
Fig. 15.38.7d presents the free body diagram and the calculations
of determining the reactions. The final values of the vertical
reactions are: VA = + 181.125 kN, VB1 = + 301.875 kN, VB2 = +
301.875 kN and VC = +181.125 kN (+ means upward). Shear force
is zero at D (x = 3 m), where the moment is 271.6875 kNm,
obtained from the following equations:

(a) 181.125 – 60.375 x = 0 gives x = 3 m, and


(b) Mx (at x = 3.0 m) = 181.125 (3) – 60.375 (3) (3) /2 = 271.6875
kNm.
At point E (mid-span), Mx = 181.125 (4) – 60.373 (4) (4)/2 = 241.5
kNm.
At point G, where x = 6 m, the bending moment is zero as obtained
from equation 181.125x – 30.1875x2 = 0.

For the span BC, the bending moment diagram is the mirror image
of the bending moment diagram of span AB due to symmetrical
beams and loadings.

Table 15.1 Moment Distribution for Load Case (i)

Joint A B C
Member AB BA BC CB

Version 2 CE IIT, Kharagpur


D.F 1.0 0.5 0.5 1.0
F.E.M + 322.0 - 322.0 + - 322.0
322.0
Balance moment - 322.0 - - + 322.0
Carry over moment - - 161.0 + -
161.0
Total balanced moment 0 - 483.0 + 0
483.0

Note: D.F. = Distribution Factors; F.E.M = Fixed End Moments

Step 3. Load Case (i): Redistributed bending moment diagram

Redistributed moment at support B = 0.7 (483) = 338.1 kNm.


Vertical reaction at A = 199.2375 kN (Fig. 15.38.7f).

Shear is zero at F where x = 3.3 m as obtained from: 199.2375 –


60.375 x = 0.

Maximum bending moment at F (x = 3.3 m) is: 199.2375 (3.3) -


60.375 (3.3) (3.3) /2 = 328.742 kNm.

At the mid-span E (x = 4 m), the moment is 60.373 (8) (8) / 8 –


338.1 /2 = 313.95 kNm.

Moment is zero at H where x = 6.6 m is obtained from: 199.2375 x


– 60.373 x2/2 = 0.

Calculations for the span BC are not performed due to symmetrical


loadings. The redistributed bending moment diagram is shown in
Fig. 15.38.7e.

Version 2 CE IIT, Kharagpur


Version 2 CE IIT, Kharagpur
Step 4. Load Case (ii): Elastic bending moment diagram

This load case is for the maximum positive moment in span AB.
The design loads consist of factored dead load and factored live
load = 1.5 (5.25 + 35) = 60.375 kN/m for the span AB. For the span
BC, we consider only the dead load with a load factor of one only.
So, the load in span BC is 5.25 kN/m. The loads are shown in Fig.
15.38.8a. Bending moments are determined by moment distribution
method and presented in Table 15.2. Bending moment diagram is
shown in Fig. 15.38.8b and the vertical reactions are shown in Fig.
15.38.8c.

Table 15.2 Moment Distribution for Load Case (ii)

Joint A B C
Member AB BA BC CB
D.F 1.0 0.5 0.5 1.0
F.E.M + 322.0 - 322.0 + 28.0 - 28.0
Balanced Moment - 322.0 + 147.0 + 147.0 + 28.0
Carry over Moment - - 161.0 + 14.0 -
Balanced Moment - + 73.5 + 73.5 -
Total Moment 0.0 - 262.5 + 262.5 0.0

Note: D.F. = Distribution Factors, F.E.M = Fixed End Moments

Final values of the reactions are: VA = + 208.6875 kN, VB1 = +


274.3125 kN, VB2 = + 53.8125 kN and VC = - 11.8125 kN (+
means upward).

In the span AB, shear force is zero at I where x = 3.46 m and the
maximum moment is 360.666 kNm, obtained from the following
equations:

(a) 208.6875 – 60.375 x = 0, gives x = 3.46 m and


(b) Mx (at x = 3.46 m) = 208.6875 (3.46) – 60.375 (3.46) (3.46)/2 =
360.666 kNm.

At point E (mid-span), Mx = 208.6875 (4) – 60.375(4) (4)/(2) =


351.75 kNm.
The bending moment is zero at J (x = 6.91 m), obtained from the
equation 208.6875 x – 60.375 (x2/2) = 0 where 0 ≤ x ≤ 8 m in the
span AB. In the span BC, the bending moment is zero only at C (x
= 8 m from B), as obtained from the equation: - 262.5 – 5.25 (x2/2)
+ 53.8125 x = 0, where 0 ≤ x ≤ 8 m in the span BC. The other value

Version 2 CE IIT, Kharagpur


of x where the negative moment is zero is x = 12.5 m. This point is
outside the span 8m of BC. Thus, the bending moment is negative
in the whole span BC.

Step 5. Load case (ii): Redistributed bending moment diagram

While redistributing the moments for the load case (ii), we have to
take care of the static equilibrium of span AB regarding the
redistributed moments. Figure 15.38.7d shows that the redistributed
positive moment is the maximum at F (x = 3.3 m) and the value is
328.742 kNm. The maximum redistributed negative moment is
338.1 kNm at B. So, let us reduce the elastic bending moment at F
(x = 3.3 m) to 328.742 kNm for the load case (ii) also. It should be
checked if this reduction is within the maximum limit of 30 per cent
or not.

Elastic bending moment at F (x = 3.3 m) for the load case (ii) = VA x


– wx2/2 = 208.6875 (3.3) – 60.375 (3.3) (3.3)/2 = 359.93 kNm (Fig.
15.38.8b). Hence, the reduction of 359.93 kNm to 328.742 kNm is
(100) (359.93 – 328.742) / 359.93 = 8.665 per cent, which is within
the limit of 30 per cent.

To have the redistributed moment at F = 328.742 kNm, the


magnitude of VA is determined from the equation:

VA (3.3) – w (3.3) (3.3)/2 = 328.742 kNm, which gives VA = 199.237


kN.

At F where x = 3.3 m, the shear force = 199.237 – 60.375 (3.3) = 0.


Therefore, the moment at F is the maximum positive redistributed
moment of magnitude = 328.742 kNm.

At B where x = 8 m, the redistributed moment for the load case (ii)


is obtained from: (199.237) (8) – 60.375 (8) (8) / 2 = - 338.1 kNm,
which is the same moment at B in the load case (i).

At E, (mid-span) where x = 4 m, the moment is: (199.237) 4 –


60.375 (4) (4)/2 = 313.95 kNm. The value of x where the
redistributed bending moment = 0, in the span AB, is obtained from:
199.37 x – 60.375 (x2/2) = 0, which gives x = 6.6 m.

For the span BC, the redistributed negative moment is – 338.1


kNm, as obtained in the span AB for this load case. The vertical
reaction VB2 in the span BC is (5.25) (4) + 338.1/8 = 63.2625 kN.
The location of the point where the moment is zero in the span BC
is obtained from the equation: – MB - wx2/2 + VB2 x = 0 or – 338.1 –

Version 2 CE IIT, Kharagpur


5.25 (x2/2) + 63.2625 x = 0, which gives x = 8 m. The other value of
x where moment is zero is 16.1 m (outside the span BC).
Therefore, the redistributed moment is negative in the entire span
BC. The redistributed bending moment diagram is shown in Fig.
15.38.8d and calculations of reaction are shown in Fig. 15.38.8e.

As mentioned earlier, the load case (iii) is not separately taken up


since it is the mirror image of load case (ii).

Version 2 CE IIT, Kharagpur


Step 6. Superimposition of the elastic bending moment
diagrams

The envelope of the positive and negative bending moments of


elastic analyses is to be prepared considering all three load cases
from Figs. 15.38.7c and 15.38.8b. The bending moment diagram
for the third load case is the mirror image of Fig. 15.38.8b. The
positive moment is to be taken from the diagram of span AB of Fig.
15.38.8b for both the spans as that is more than the values of Fig.
15.38.7c.

For the negative moment in span BC, the diagram for the load case
(i) will intersect that of the load case (ii). Similarly, for the negative
moment in span AB, the diagram of load case (i) will intersect that
of load case (iii). Due to the mirror image, the point of intersection is
determined for span BC only.

For the load case (i) in span BC, the bending moment at a distance
of x from the support B is given by:

Mx = - 483 + 301.875 x – 60.375 (x2/2)

Similarly, for the load case (ii) in span BC, the bending moment at a
distance of x from the support B is given by:

Mx = - 262.5 + 53.8125 x – 5.25 (x2/2)

We can find the value of x, where these two negative moments are
the same by equating the two expressions:

- 483 + 301.875x – 60.375 (x2/2) = - 262.5 + 53.8125x – 5.25


(x2/2)

or 27.5625 x2 – 248.0625 x + 220.5 = 0

The values of x are 1.0 m and 8.0 m. So, at a distance of 1.0 m


from the support B, the two negative moments intersect. The value
of the moment is obtained from either of the two equations, which
comes out to be 211.3125 kNm. Accordingly, Fig 15.38.9a presents
the envelope of the elastic bending moment diagrams making use
of symmetry of the load in the two spans.

Version 2 CE IIT, Kharagpur


Step 7. Superposition of redistributed bending moment
diagrams

The procedure is exactly the same as in Step 6. The positive


bending moment diagram is taken from that of span AB of load
case (ii) and the negative moment envelope is obtained combining
the negative bending moments of span BC for the two load cases.
In this redistributed negative moments, there is no intersection of
the diagram as in Step 6. Accordingly, Fig. 15.38.9b presents the
envelope of the redistributed bending moments using symmetry of
the loads in the two spans.

15.38.6 Practice Questions and Problems with Answers


Q.1: Explain the concept of redistribution of moments in statically indeterminate
reinforced concrete structures.

A.1: Paragraphs 1 and 2 of sec. 15.38.1

Q.2: Mention three advantages of considering the redistribution of moments for


the design of statically indeterminate reinforced concrete structures.

A.2: Paragraph 3 of sec. 15.38.1

Q.3: State the assumptions of considering the redistribution of moments in the


design of statically indeterminate reinforced concrete structures.

A.3: Paragraph 1 of sec. 15.38.2

Q.4: What are the recommendations of IS 456 regarding the redistribution of


moment in the design of statically indeterminate structures employing
working stress and limit state methods?

A.4: Section 15.38.3 is the full answer.

Version 2 CE IIT, Kharagpur


Q.5: Solve Problem 1 of sec. 15.38.5 (Fig. 15.38.5a) when the redistribution is
limited to 20 per cent.

A.5: Step 1 is the same as that of Problem 1 of sec. 15.38.5. Please refer to
Figs. 15.38.5b and c for the elastic bending moment diagram and
free body diagram, respectively.

Step 2. Redistributed bending moment diagram and envelope of


moment diagrams.

Version 2 CE IIT, Kharagpur


Maximum negative moment at A after the redistribution of 20 per cent =
0.8 (-128) = - 102.4 kNm. So, the maximum positive moment at the mid-span C
= 192-102.4 = 89.6 kNm (Please refer to Fig. 15.38.10b). VA = 96 kN.

The point of inflection is obtained from: 96 x – 12x2 – 102.4 = 0, which


gives x = 1.27 m (Fig. 15.38.10a). At x = 1.27 m, elastic moment = 96x – 12x2 –
128 = - 25.435 kNm. The redistributed moment at E (x = 1.27 m) = 0.8 (- 25.435)
= - 20.35 kNm.

The redistributed bending moment diagram is shown in Fig. 15.38.10b and


the envelope of the moment diagram is shown in Fig. 15.38.10c.

15.38.7 References
1. Reinforced Concrete Limit State Design, 6th Edition, by Ashok K. Jain,
Nem Chand & Bros, Roorkee, 2002.
2. Limit State Design of Reinforced Concrete, 2nd Edition, by P.C. Varghese,
Prentice-Hall of India Pvt. Ltd., New Delhi, 2002.
3. Advanced Reinforced Concrete Design, by P.C. Varghese, Prentice-Hall
of India Pvt. Ltd., New Delhi, 2001.
4. Reinforced Concrete Design, 2nd Edition, by S. Unnikrishna Pillai and
Devdas Menon, Tata McGraw-Hill Publishing Company Limited, New
Delhi, 2003.
5. Limit State Design of Reinforced Concrete Structures, by P. Dayaratnam,
Oxford & I.B.H. Publishing Company Pvt. Ltd., New Delhi, 2004.
6. Reinforced Concrete Design, 1st Revised Edition, by S.N. Sinha, Tata
McGraw-Hill Publishing Company. New Delhi, 1990.
7. Reinforced Concrete, 6th Edition, by S.K. Mallick and A.P. Gupta, Oxford &
IBH Publishing Co. Pvt. Ltd. New Delhi, 1996.
8. Behaviour, Analysis & Design of Reinforced Concrete Structural Elements,
by I.C.Syal and R.K.Ummat, A.H.Wheeler & Co. Ltd., Allahabad, 1989.
9. Reinforced Concrete Structures, 3rd Edition, by I.C.Syal and A.K.Goel,
A.H.Wheeler & Co. Ltd., Allahabad, 1992.
10. Textbook of R.C.C, by G.S.Birdie and J.S.Birdie, Wiley Eastern Limited,
New Delhi, 1993.
11. Design of Concrete Structures, 13th Edition, by Arthur H. Nilson, David
Darwin and Charles W. Dolan, Tata McGraw-Hill Publishing Company
Limited, New Delhi, 2004.
12. Concrete Technology, by A.M.Neville and J.J.Brooks, ELBS with
Longman, 1994.
13. Properties of Concrete, 4th Edition, 1st Indian reprint, by A.M.Neville,
Longman, 2000.
14. Reinforced Concrete Designer’s Handbook, 10th Edition, by C.E.Reynolds
and J.C. Steedman, E & FN SPON, London, 1997.

Version 2 CE IIT, Kharagpur


15. Indian Standard Plain and Reinforced Concrete – Code of Practice (4th
Revision), IS 456: 2000, BIS, New Delhi.
16. Design Aids for Reinforced Concrete to IS: 456 – 1978, BIS, New Delhi.

15.38.8 Test 38 with Solutions


Maximum Marks = 50 Maximum Time = 30
minutes

TQ.1: Mention three advantages of considering the redistribution of moments for


the design of statically indeterminate reinforced concrete structures.
(10 marks)
A.TQ.1: Paragraph 3 of sec. 15.38.1

TQ.2: What are the recommendations of IS 456 regarding the redistribution of


moment in the design of statically indeterminate structures employing
working stress and limit state methods?
(10 marks)
A.TQ.2: Section 15.38.3 is the full answer

TQ.3: Solve Problem 2 of sec. 15.38.5 (Fig. 15.38.6a) considering the


redistribution up to (a) 20 per cent and (b) 10 per cent, separately.
(30 marks)

Version 2 CE IIT, Kharagpur


A.TQ.3: Step 1 is the same as that of Problem 2 of sec. 15.38.5. Please
refer to Figs. 15.38.6b and c for the elastic bending moment diagram and
free body diagram, respectively.

Step 2. Redistributed bending moment diagram (20 per cent


redistribution) and envelope of the moment diagrams

Redistributed negative moment at A = 0.8 (- 60) = - 48 kNm.


Redistributed positive moment at E = 90 – 48 = 42 kNm

The point of zero moment (point G) is obtained by taking moment of forces


of the free body diagram about G, shown in Fig. 15.38.11a, when 0 ≤ x ≤3
m, gives 30x – 48 = 0 or x = 1.6 m. The redistributed moment diagram is
shown in Fig. 15.38.11b.

At G where x = 1.6 m, the elastic moment = 30 (1.6) – 60 = - 12kNm.


Therefore, the redistributed moment at G = 0.8 (-12) = - 9.6 kNm. The
envelope of the moment diagrams is shown in Fig. 15.38.11c.

Step 3. Redistributed bending moment diagram (10 per cent


redistribution) and envelope of the moment diagrams.

Redistributed negative moment at A = 0.9 (- 60) = - 54 kNm.


Redistributed positive moment at E = 90 - 54 = 36 kNm.

Version 2 CE IIT, Kharagpur


The point of zero moment (point G) is obtained by taking moment of forces
about G of free body diagram shown in Fig. 15.38.12a, when 0 ≤ x ≤ 3 m,
gives 30x – 54 = 0, or x = 1.8 m. The redistributed moment diagram is
shown in Fig. 15.38.12b.

At G, where x = 1.8 m, the elastic moment = 30 (1.8) – 60 = - 6 kNm.


Therefore, the redistributed moment at G = 0.9 (- 6) = - 5.4 kNm. The
envelope of the moment diagrams is shown in Fig. 15.38.12c.

15.38.9 Summary of this Lesson


This lesson explains the concept of redistribution of moments in statically
indeterminate reinforced concrete structures designed either by the
working stress or by the limit state methods. The behaviour of such
structures with increasing loads after reaching the respective design loads
is important to understand the inclusion of redistribution of moments. The
advantages of such inclusion are mentioned in this lesson. The formations
of first plastic hinge and subsequently other plastic hinges leading to the
formation of collapse mechanism will not only increase the load carrying
capacity of the structure but also help the designer to have flexibility in
reducing or increasing moments to a limited extent. By this, the detailing of
reinforcement helps to avoid congestion of the bars at sections having
high moments. Due to the limited rotation capacities of reinforced concrete
cross-sections, the reductions / additions of moments are somewhat
restricted, which are also explained as stipulated in IS 456. Several
numerical problems are solved for the purpose of illustration considering
full or partial redistribution in clamped or two-span continuous beams
taking up different combinations of dead and live loads. Understanding the
illustrative examples and solving the practice and test problems will help in
utilising the redistribution to get the advantages in designing statically
indeterminate reinforced concrete structures.

Version 2 CE IIT, Kharagpur


Module
16
Earthquake Resistant
Design of Structures
Version 2 CE IIT, Kharagpur
Lesson
39
Seismic Effects,
Material Behaviour and
General Principles of
Earthquake Resistant
Design of Structures
Version 2 CE IIT, Kharagpur
Instructional Objectives:

At the end of this lesson, the student should be able to:

• state the direct and indirect seismic effects,

• explain the behaviour of plain concrete, steel and reinforced concrete with
high intensity repeated axial cyclic loads,

• define common terminologies needed for earthquake engineering,

• name different standards of BIS regarding the earthquake resistant design


of structures,

• state the assumptions of earthquake resistant design of structures,

• determine the horizontal seismic coefficient, design seismic base shear and
distribution of design force,

• state the steps of static elastic design of such structures,

• mention the objectives of earthquake resistant design of structures,

• explain the need for ductility and ductile detailing of reinforcement in the
earthquake resistant design of structures.

16.39.1 Introduction
Earthquakes, Tsunamis, Seiches, Landslides, Floods and Fires are
natural calamities causing severe damage and sufferings to persons by
collapsing the structures, cutting off transport systems, killing or trapping
persons, animals etc. Such natural disasters are challenges to the progress of
development. However, civil engineers as designers have a major role to play in
minimising the damages by proper designing the structures or taking other useful
decisions. Because of the vastness of the topic, “Disaster management and
mitigation”, this module includes understanding the earthquakes, behaviour of
the materials of construction and structures and the extent to which structural
engineers make use of the knowledge in taking proper decisions in designing the
structures made of reinforced concrete.

Version 2 CE IIT, Kharagpur


Earthquakes have many other effects besides vibrating the structures in
response to ground shaking at its foundation. These other effects may even
exceed that due to vibration. Unfortunately, the procedure of their estimation and
the required steps for the design are considered outside the scope of structural
engineering. Different seismic resistant design codes have provisions to take into
account the vibration of structures. But, these codes do not have any provision to
take care of other effects. However, structural engineers should be aware of the
intensity of the hazards with a view to taking precautionary measures either in
the design of structures, advising clients in selecting proper sites in such zones
or making them aware of the importance of proper maintenance of the structures
and other considerations the clients should follow up while using the designed
structures. The different direct and indirect effects of earthquakes are mentioned
in the following section.

16.39.2 Direct and Indirect Seismic Effects


A. Direct effects.

The following are the direct seismic effects:


1. Damage due to surface faulting
The damage due to surface faulting varies widely. It may totally demolish
houses, rupture the foundations, tilt the foundation slabs and walls or may
cause minor damage to the houses.

2. Damage due to liquefaction

Liquefaction of soil may cause instability due to internal seismic waves


and thereby may significantly damage in form of settlement, tilting and
rupture of the structures. The extent of damage depends on properties of
soils of different layers, depth of the water table and the intensity,
magnitude and duration of the earthquake. Accordingly, there may be
either large settlement or differential settlement of the ground surface.
Please refer to sec. 16.39.4(h) of this lesson for the definition of
liquefaction as per IS Code.

It is always desirable to avoid construction in such areas than to design


the structures following codal provisions, which may be insufficient, though
ensure effective design against vibration of structures due to shaking at
the foundation level.

3. Damage due to ground shaking

As the state of the art of this subject is still developing, integrated field
inspection of structures damaged by earthquakes and their analyses are
useful in further adding/improving the expert knowledge in the seismic

Version 2 CE IIT, Kharagpur


resistant design and construction. Earlier inspections and analyses
established the types of foundation, configurations of structures, materials
of construction and design and detailing of construction. Such data are
being continuously updated.

4. Damage due to sliding of superstructure on its foundation

It is essential that the whole structure and the foundation should work as a
unit especially for the seismic resistant design and construction of
structures. For this the superstructures should be anchored properly to the
foundation.

5. Damage due to structural vibration

The extent of the damage due to structural vibration depends on the


materials of construction. Wood, reinforced concrete and steel are widely
used in civil engineering structures. It is well-known that inertia forces are
developed as vibration response of a structure due to earthquake ground
shaking. The intensity of such inertia forces is directly proportional to the
product of mass and acceleration. Hence, reduction of mass is very
effective to minimise the inertia forces. In this respect, timber has the
maximum advantage as a potential construction material due to its low
mass. Concrete, though a heavy material when reinforced with steel bars,
has good strengths both in compression and in tension. Accordingly,
reinforced concrete can be used effectively by providing proper amount of
reinforcement and correct detailing of them as they play significant roles in
the seismic resistant design of reinforced concrete structures. Steel has
the additional advantages of ductility, strength and toughness per unit
weight than concrete. The major problem of steel is the buckling.
Behaviour of plain and reinforced concrete and steel with high intensity
repeated axial cyclic loads is taken up in the next section.

B. Indirect effects

Tsunamis, seiches, landslides, floods and fires are the indirect effects of
earthquakes. These may occur either alone or in combinations to add to
the damages during an earthquake.

16.39.3 Behaviour of Concrete and Steel with High Intensity


Repeated Axial Cyclic Loads.
Earthquakes cause ground motions in a random fashion in all directions
having significant horizontal and vertical ground accelerations as function of time.
Structures subjected to ground motions respond in a vibratory fashion. The
maximum response acceleration during the elastic stage depends on the natural
frequency of vibration of the structure and the magnitude of the damping. The

Version 2 CE IIT, Kharagpur


maximum inertia loads acting on a structure during an earthquake is determined
by multiplying the mass by the acceleration.

In order to understand the behaviour of reinforced concrete structure


under such repeated cyclic loads, we first discuss the behaviour of plain concrete
and reinforcing steel below.

(a) Plain concrete

Plain concrete has tensile strength less than twenty per cent of its
compressive strength. The tensile strength of plain concrete is normally not
obtained from the direct tension test because of difficulties in holding the
specimens and the uncertainties of developing secondary stresses due to
holding devices. Hence, this is measured indirectly either by split cylinder test or
by bending test conducted on plain concrete prisms. Prism tests give the tensile
strength of concrete in flexure, known as the modulus of rupture. Plain concrete
specimens are not tested under repeated axial cyclic tensile loads.

However, earlier experiments on plain concrete cylinders with high


intensity repeated axial compressive loads at slow rate of straining give the
stress-strain curve as shown in Fig. 16.39.1. The stress-strain curve shows
pronounced hysteresis effect and the envelope curve is almost identical to that of
a single continuous load. Furthermore, such a curve obtained from the static test
is also the same as that of the first cycle. Figure 16.39.1 shows that the slope of
the stress-strain curve and the maximum attainable stress decrease with the
number of cycles indicating the reduction of strength and stiffness of concrete
due to the formation of cracks. With the increase of rate of loading, the
compressive strength of concrete increases while the strain at the maximum
stress decreases.

Version 2 CE IIT, Kharagpur


(b) Reinforcing steel

Figure 16.39.2 represents a typical stress-strain curve based on


experiments with steel rods with repeated axial loads in tension and
compression. Initially, the curve is similar to that obtained in static test in tension.
On reversing the load after reaching the yield strain in tension, the unloading
curve is seen to be curvilinear forming loops instead of the straight line as shown
in Fig. 16.39.2, showing Bauschinger effect. The loop formed by one complete
cycle is known as hysteresis loop, the area of which is the energy absorbed by
the specimen in that cycle. Practically the same path is repeated in subsequent
cycles. Thus, we find that the stress-strain curve for steel is cycle independent.
The loop, therefore, continues for number of cycles till the specimen buckles or
fails due to fatigue. It is also worth mentioning that the same hysteresis loops are
obtained for specimens loaded first in compression, unloaded and reloaded in
tension. However, the yield strength of steel is dependent on the rate of loading.

Version 2 CE IIT, Kharagpur


(c) Reinforced Concrete

Similar tests of loading and unloading in tension and compression on


doubly-reinforced cantilever beam loaded with a point load P at the free end
reveal the following:

(i) Formation of large cracks when the load P acting downward


causes post-elastic range of stress in tension steel (Fig.
16.39.3a).

Version 2 CE IIT, Kharagpur


(ii) Due to residual plastic strains in the steel, these cracks do not
completely close on unloading but they remain open (Fig.
16.39.3b).

(iii) On loading the beam in a reverse direction (upward), as shown


in Fig. 16.39.3c, the resistance to rotation is decreased with
respect to that during the first loading. This reduction is due to
the presence of open cracks in the compression zone. As a
result, the whole of compression is carried by compression steel.
Thus, the flexural rigidity of the section is only that of steel.

(iv) At higher load (upward) when Bauschinger effect starts and


behaves inelastically, the flexural rigidity further reduces (Fig.
16.39.3d). The cracks in the compression zone may close
depending on the magnitude of the load and the relative
amounts of tension and compression steel. As a result, the
stiffness of the member increases since concrete now can carry
some compression load due to the closing of the cracks.
However, if the cracks do not close and the member is unloaded,
the cracks may be throughout the whole depth at critical
sections. The width of these full-depth cracks depends on the
amount of yielding and the effectiveness of the bond.

(v) If the member is again loaded downward, the member behaves


as a steel beam initially as the concrete is not in contact with the
steel at the face of the crack.

Version 2 CE IIT, Kharagpur


Due to such reversal of loading, opening and closing of cracks in
the alternate tension and compression zones will gradually
reduce the compressive strength of concrete either due to less or
no contact because of slight relative lateral movement or
presence of debris in the crack. This along with Bauchinger
effect of steel gives the moment-curvature relation for the doubly-
reinforced section as shown in Fig. 16.39.4. This diagram is far
different from the idealised moment-curvature relation as shown
in Fig. 16.39.5. The rounding and pinching of the loops of Fig.
16.39.4 clearly show smaller area than that of Fig. 16.39.5.
Therefore, less energy will be dissipated per cycle in the actual
case than the assumed idealised one. Accordingly, the response
of frames to severe earthquake motions will be influenced by
such effects.

Furthermore, the presence of high shear causing large shear


displacement will split the concrete longitudinally along the
flexural steel bars leading to further loss of bond and stiffness.

Thus, the influential factors of load-displacement relationship of


reinforced concrete members during severe earthquake
subjected to reversed inelastic deformations are summarised as:
Version 2 CE IIT, Kharagpur
1. Inelastic behaviour of steel reinforcement,

2. The extent of cracking of concrete,

3. Effectiveness of bond and anchorage, and

4. Presence of high shear.

Accordingly, realistic dynamic analyses shall be based on more


accurate moment-curvature loops. The moment-curvature
relationships, shown in Figs. 16.39.4 and 16.39.5, reveal that the
deformation changes from cycle to cycle after the yielding starts.

16.39.4 Terminology for Earthquake Engineering


(a) Design Basis Earthquake (DBE)

It is an earthquake which can reasonably be expected to occur at least


once during the design life of the structure, as defined in cl.3.6 of IS 1893
(Part 1): 2002.

(b) Epicentre

Epicentre is the geographical point on the surface of earth vertically above


the focus of the earthquake (cl.3.10 of IS 1893 (Part 1): 2002).

(c) Focus

Focus is the source of the elastic waves of the originating earthquake


inside the earth which cause shaking of ground (cl. 3.13 of IS 1893 (Part
1): 2002).

(d) Intensity of earthquake

The intensity of an earthquake indicates the strength of shaking during the


earthquake and is expressed by a number according to the modified
Mercalli Scale or M.S.K Scale of seismic intensities (cl. 3.15 of IS 1893
(Part 1): 2002).

(e) Magnitude of earthquake (Richter’s Magnitude)

The magnitude of an earthquake is expressed by a number, which is a


measure of the energy released in an earthquake. The magnitude of an
earthquake is defined as logarithm to the base 10 of the maximum trace
amplitude, expressed in micron, which the standard short-period torsion

Version 2 CE IIT, Kharagpur


seismometer (with a period of 0.8 second, magnification of 2800 and
damping nearly critical) would register due to the earthquake at an
epicentral distance of 100 km (cl. 3.18 of IS 1893 (Part 1): 2002).

(f) Critical damping

Critical damping is the damping beyond which the free vibration motion
will not be oscillatory (cl. 3.3 of IS 1893 (Part 1): 2002).

(g) Maximum Considered Earthquake (MCE)

Maximum Considered Earthquake is the most severe earthquake whose


effects are considered by IS 1893 (Part 1): 2002, as given in cl. 3.19 of
this standard.

(h) Liquefaction

Liquefaction is a state in saturated cohesionless soil wherein the effective


shear strength is reduced to negligible value for all engineering purposes
due to pore pressure caused by vibrations during an earthquake when
they approach the total confining pressure. In this condition the soil tends
to behave like a fluid mass (cl. 3.16 of IS 1893 (Part 1): 2002).

16.39.5 Bureau of Indian Standards for Earthquake Design


In our country, several major earthquakes have occurred in the
Himalayan-Nagalushai region, Indo-Gangetic Plain, Western India, Kutch and
Kathiawar regions. Taking into account seismic data from studies of these Indian
earthquakes, Bureau of Indian Standard first published IS 1893
“Recommendations for earthquake resistant design of structures” in 1962 and
revised in 1966. Considering the local seismology, accepted level of seismic risk,
building topologies and materials and methods used in construction, presently
the Bureau of Indian Standards has the following seismic codes:

1. IS 1893 (Part 1), 2002; Indian Standard Criteria for Earthquake


Resistant Design of Structures (5th Revision),

2. IS 1893 has other four parts: (a) Part 2 for liquid retaining tanks–
elevated and ground supported, (b) Part 3 for bridges and retaining
walls, (c) Part 4 for industrial structures including stack like structures
and (d) Part 5 for dams and embankments. However, they are yet to
be finalised. Hence, provisions of Part 1 will be read along with
relevant clauses of IS 1893: 1984 for structures other than buildings.

Version 2 CE IIT, Kharagpur


3. IS 4326: 1993, Indian Standard Code of Practice for Earthquake
Resistant Design and Construction of Buildings, (2nd Revision),

4. IS 13827: 1993, Indian Standard Guidelines for Improving Earthquake


Resistance of Earthen Buildings,

5. IS 13828: 1993, Indian Standard Guidelines for Improving Earthquake


Resistance of Low Strength Masonary Buildings,

6. IS 13920: 1993, Indian Standard Code of Practice for Ductile Detailing


of Reinforced Concrete Structures Subjected to Seismic Forces, and

7. IS 13935: 1993, Indian Standard Guidelines for Repair and Seismic


Strengthening of Buildings.

The regulations of these standards will not result in structures having no


damage during earthquake of all magnitudes. However, the regulations shall
ensure that, as far as possible, structures will be able to respond without
structural damage to shocks of moderate intensities and without total collapse
to shocks of heavy intensities.

16.39.6 General Principles of Earthquake Resistant


Design of Structures

a) Ground motion

The characteristic parameters like intensity, duration etc. of seismic


ground vibrations depend upon the magnitude of the earthquake, its depth
of focus, distance from the epicentre, properties of soil or medium through
which the seismic waves travel and the soil strata where the structure
stands. The random earthquake motions can be resolved in any three
mutually perpendicular directions. The horizontal direction is normally the
prominent direction. Vertical acceleration is considered in large-span
structures.

The response of a structure to ground vibrations depends on the nature of


foundation soil, form, material, size and mode of construction of structures
and the duration and characteristics of ground motion.

(b) Assumptions

The following are the assumptions in the earthquake resistant design of


structures:
Version 2 CE IIT, Kharagpur
1. Impulsive ground motions of earthquake are complex, irregular in
character, changing in period and time and of short duration. They,
therefore, may not cause resonance as visualised under steady-state
sinusoidal excitations, except in tall structures founded on deep soft
soils.

2. Wind, maximum flood or maximum sea waves will not occur


simultaneously with the earthquake.

3. For static analysis, elastic modulus of materials shall be taken unless


otherwise mentioned.

16.39.7 Design Lateral Forces

(a) Design horizontal seismic coefficient

The design horizontal seismic coefficient Ah (cl. 6.4.2 of IS 1893 (Part 1): 2002)
for structure is determined from

Ah = (Z I Sa / 2 R g) (16.1)

where Z = the zone factor as given in Table 2 of IS 1893 (Part 1): 2002, based on
classifying the country in four seismic zones,

I = Importance factor, depending upon the functional use of the structure


as given in Table 6 of IS 1893 (Part 1): 2002,

R = Response reduction factor, depending on the perceived seismic


damage, performance of the structure, characterised by ductile or
brittle deformations, as given in Table 7 of IS 1893 (Part 1): 2002.
However, the value of I/R shall not be greater than 1.0, and

Sa/g = Average response acceleration coefficient for rock or soil sites as


given in Fig. 2 and Table 3 of IS 1893 (Part 1): 2002.

It is further stipulated in cl.6.4.2 of IS 1893 (Part 1): 2002, that for any
structure with undamped natural period of vibration of the structure (in seconds)
T ≤ 0.1 second, the value of Ah will not be taken less than Z/2 whatever be the
value of I/R.

Version 2 CE IIT, Kharagpur


(b) Design seismic base shear

The total design lateral force or design seismic base shear VB along any
principal direction (cl.7.5.3 of IS 1893 (Part 1): 2002) shall be determined from
the following equation:

VB = Ah W (16.2)

where Ah is as given in Eq. 16.1, and W is the seismic weight of the building
as given in cl. 7.4.2 of IS 1893 (Part 1): 2002.

(c) Distribution of design force

The design base shear VB of Eq. 16.2 shall be distributed along the height of
B

the building as per the following equation (cl.7.7 of IS 1893 (Part 1): 2002):

n
Qi = VB (Wi hi2 ) / ∑ Wi h 2j (16.3)
j =1

where

Qi = Design lateral force at floor i,


Wi = Seismic weight of floor i,
hi = Height of floor i measured from base, and
n = Number of storeys in the building at which the masses are located

16.39.8 Static Elastic Design

The various steps, based on the elastic design are:

1. Dead load and imposed loads are considered as per cl.6.3 of IS 1893
(Part 1): 2002.

2. The horizontal loads are determined as explained in sec. 16.39.7.

3. Bending moments and shear forces are determined on columns and floor
beams in a manner similar to that of frames subjected to wind loads.

4. All parts must be efficiently bonded together so that the structure acts as a
unit.

Version 2 CE IIT, Kharagpur


5. Panel walls, finishes and ornaments should be permanently attached to
the frame to ensure that they do not collapse independently in the event of
a shock.

6. Separate footings of columns shall be connected by ties to resist the


compressive thrust or tensile pull of magnitude 1/10th of the load on the
footing.

Structures designed on this simple elastic principle may even survive when
subjected to severe earthquake due to the following:

a) Yielding at critical sections increases the period of vibration and helps to


absorb greater amounts of input energy.

b) Assistance of non-structural partitions and the dissipated energy are


helpful as they crack.

c) Yielding of foundations helps to reduce the predicted response.

16.39.9 Dynamic Analysis

Though static elastic analysis is considered sufficient for smaller building,


dynamic analyses shall be performed to determine the seismic force and its
distribution to different levels for regular and irregular buildings, as defined in cl.
7.1 of IS 1893 (Part 1): 2002, following the recommendations of cl. 7.8 of the
same IS Code.

(a) Regular buildings – Those greater than 40 m in height in Zones IV and V


and those greater than 90 m in height in Zones II and III,

(b) Irregular buildings – All framed buildings higher than 12 m in Zones IV and
V and those higher than 40 m in Zones II and III.

Though not mandatory, the code also recommends dynamic analysis for
buildings lesser than 40 m for irregular building in Zones II and III (see Note
below cl.7.8.1 of IS 1893 (Part 1): 2002).

The dynamic analysis may be carried out either by Time History Method or by
Response Spectrum Method. More about the dynamic analysis is beyond the
scope of this module.

Version 2 CE IIT, Kharagpur


16.39.10 Objectives of Earthquake Resistant Design of
Structures

It is uneconomical to design structures to withstand major earthquakes


elastically. Therefore, the trend of the design is that the structure should have
sufficient strength and ductility to withstand large tremors elastically. For this the
interconnections of the members must be designed particularly to ensure
sufficient ductility.

Accordingly, the design approach adopted in IS 1893 (Part 1): 2002 is stated
in cl 6.1.3 of the standard which is as follows:

The design approach is to ensure the following:

(a) that structures possess at least a minimum strength to withstand minor


earthquakes (〈DBE), which occur frequently, without damage;

(b) that structures resist moderate earthquakes (DBE) without significant


structural damage though some non-structural damage may occur; and

(c) that structures withstand major earthquakes (MCE) without collapse.

16.39.11 Ductility and Ductile Detailing of Reinforcement

Actual forces caused by earthquakes on structures are much greater


than the design forces determined following IS 1893 (Part 1): 2002. However,
ductility arising from inelastic behaviour of reinforced concrete and steel, as
explained in sec.16.39.3 and detailing of reinforcement and over-strength due to
reserve strength in structures beyond the design strength are to be relied upon
to take care of this difference between the actual and design lateral loads.

Thus, ductility and ductile detailing of reinforced concrete structures are


very important when the structures are subjected to seismic forces. Accordingly,
IS 13920: 1993 stipulates the recommendations of ductile detailing of reinforced
concrete structures subjected to seismic forces. These two aspects, therefore,
are taken up in the next lesson.

16.39.12 Practice Questions and Problems with Answers

Q.1: State the direct and indirect seismic effects.

Version 2 CE IIT, Kharagpur


A.1: Section 16.39.2 is the complete answer

Q.2: Explain the behaviour of plain concrete with high intensify repeated axial
cyclic loads.

A.2: Part (a) of sec. 16.39.3

Q.3: Explain the behaviour of steel with high intensity repeated axial cyclic loads.

A.3: Part (b) of sec. 16.39.3

Q.4: Explain the behaviour of reinforced concrete with high intensity repeated
axial cyclic loads.

A.4: Part (c) of sec. 16.39.3

Q.5: Define the following terminologies as per IS 1893 (Part 1): 2002: (a) Design
Basis Earthquake (DBE), (b) Epicentre, (c) Focus, (d) Intensify of
earthquake, (e) Magnitude of earthquake, (f) critical damping, (g) Maximum
Considered Earthquake and (h) Liquefaction.

A.5: Section 16.39.4 is the complete answer.

Q.6: Name the different BIS Standards for earthquake design.

A.6: Section 16.39.5 is the complete answer.

Q.7: Write the expression for determining (a) Horizontal seismic coefficient, (b)
Design seismic base shear and (c) Distribution of design force.

A.7: Section 16.39.7 is the complete answer.

Q.8: Write the steps of performing static elastic design of earthquake resistant
structures.

A.8: Section 16.39.8 is the complete answer.

Version 2 CE IIT, Kharagpur


Q.9: When should we have to perform dynamic analysis?

A.9: Section 16.39.9 is the complete answer.

Q.10: State the objectives of earthquake resistant design of structures.

A.10: Section 16.39.10 is the complete answer.

Q.11: Explain the need of ductility and ductile detailing of reinforced concrete
structures subjected to seismic forces.

A.11: Section 16.39.11 is the complete answer.

16.39.13 References

1. Reinforced Concrete Limit State Design, 6th Edition, by Ashok K. Jain,


Nem Chand & Bros, Roorkee, 2002.
2. Limit State Design of Reinforced Concrete, 2nd Edition, by P.C. Varghese,
Prentice-Hall of India Pvt. Ltd., New Delhi, 2002.
3. Advanced Reinforced Concrete Design, by P.C. Varghese, Prentice-Hall
of India Pvt. Ltd., New Delhi, 2001.
4. Reinforced Concrete Design, 2nd Edition, by S. Unnikrishna Pillai and
Devdas Menon, Tata McGraw-Hill Publishing Company Limited, New
Delhi, 2003.
5. Limit State Design of Reinforced Concrete Structures, by P. Dayaratnam,
Oxford & I.B.H. Publishing Company Pvt. Ltd., New Delhi, 2004.
6. Reinforced Concrete Design, 1st Revised Edition, by S.N. Sinha, Tata
McGraw-Hill Publishing Company. New Delhi, 1990.
7. Reinforced Concrete, 6th Edition, by S.K. Mallick and A.P. Gupta, Oxford &
IBH Publishing Co. Pvt. Ltd. New Delhi, 1996.
8. Behaviour, Analysis & Design of Reinforced Concrete Structural Elements,
by I.C.Syal and R.K.Ummat, A.H.Wheeler & Co. Ltd., Allahabad, 1989.
9. Reinforced Concrete Structures, 3rd Edition, by I.C.Syal and A.K.Goel,
A.H.Wheeler & Co. Ltd., Allahabad, 1992.
10. Textbook of R.C.C, by G.S.Birdie and J.S.Birdie, Wiley Eastern Limited,
New Delhi, 1993.
11. Design of Concrete Structures, 13th Edition, by Arthur H. Nilson, David
Darwin and Charles W. Dolan, Tata McGraw-Hill Publishing Company
Limited, New Delhi, 2004.
12. Concrete Technology, by A.M.Neville and J.J.Brooks, ELBS with
Longman, 1994.

Version 2 CE IIT, Kharagpur


13. Properties of Concrete, 4th Edition, 1st Indian reprint, by A.M.Neville,
Longman, 2000.
14. Reinforced Concrete Designer’s Handbook, 10th Edition, by C.E.Reynolds
and J.C. Steedman, E & FN SPON, London, 1997.
15. Indian Standard Plain and Reinforced Concrete – Code of Practice (4th
Revision), IS 456: 2000, BIS, New Delhi.
16. Design Aids for Reinforced Concrete to IS: 456 – 1978, BIS, New Delhi.
17. Structural Engineering Slide Library, W.G.Godden, Editor; Set J:
Earthquake Engineering V.V.Bertero, National Information Service for
Earthquake Engineering, University of California, Berkeley, USA. (
http://nisee.berkeley.edu/bertero/html/damage_due_to_ground_failure.htm
l)

16.39.14 Test 39 with Solutions

Maximum Marks = 50 Maximum Time = 30


minutes

Answer all questions.

TQ.1: State the direct and indirect seismic effects.


(10 Marks)
A.TQ.1: Section 16.39.2 is the complete answer

TQ.2: Explain the behaviour of reinforced concrete with high intensity repeated
axial cyclic loads.
(10 Marks)
A.TQ.2: Part (c) of sec. 16.39.3

TQ.3: Name the different BIS Standards for earthquake design.


(15 Marks)
A.TQ.3: Section 16.39.5 is the complete answer.

TQ.4: Write the steps of performing static elastic design of earthquake resistant
structures.
(10 Marks)
A.TQ.4: Section 16.39.8 is the complete answer.

TQ.5: State the objectives of earthquake resistant design of structures.


(5 Marks)

Version 2 CE IIT, Kharagpur


A.TQ.5: Section 16.39.10 is the complete answer.

16.39.15 Summary of this Lesson

This lesson explains the direct and indirect effects of earthquakes. Behaviour
of plain concrete, steel bars and reinforced concrete are discussed when
subjected to high intensity repeated axial loads. Some common terminologies are
defined as given in IS 1893 (Part 1): 2002. Different BIS standards pertaining to
earthquake design are given. General principles and assumptions are given in
this lesson. Determinations of horizontal seismic coefficient, design seismic base
shear and distribution of design force are explained. Steps of the static elastic
design are given mentioning the particular situations when dynamics analysis
should be done. The objectives of earthquake resistant design of structures are
explained. The need for ductility and ductile detailing of reinforcement is
explained in this lesson.

Version 2 CE IIT, Kharagpur


Module
16
Earthquake Resistant
Design of Structures
Version 2 CE IIT, Kharagpur
Lesson
40
Ductile Design and
Detailing of Earthquake
Resistant Structures
Version 2 CE IIT, Kharagpur
Instructional Objectives:

At the end of this lesson, the student should be able to:

• define and explain ductility factor with respect to displacement, curvature or


rotation,

• state the advantages of ductility in the design of reinforced concrete


members,

• derive expressions of ductility factor of singly and doubly-reinforced


rectangular beams,

• mention the factors influencing ductility,

• give general specification of materials for the ductile design of reinforced


concrete members,

• state the general guidelines in the design and detailing of structures having
sufficient strength and ductility,

• identify the situation when special confining reinforcement is needed,

• to determine the ductility factor of singly and doubly-reinforced rectangular


beams applying the expressions,

• to apply the knowledge in designing and detailing beams, columns and


beam-column joints as per IS 13920:1993.

16.40.1 Introduction
As mentioned in sec. 16.39.10 of Lesson 39, it is uneconomical to design
structures to withstand major earthquakes. However, the design should be done
so that the structures have sufficient strength and ductility. This lesson explains
the requirements and advantages of ductility in the design of reinforced concrete
members which can be expressed with respect to displacement, curvature or
rotation of the member. The expressions of ductility of singly and doubly-
reinforced beams with respect to curvature are derived. The influencing
parameters of the ductility are explained. Several aspects of design for ductility
are explained mentioning detailing for ductility, as stipulated in IS 13920:1993, for

Version 2 CE IIT, Kharagpur


flexural members and columns. Illustrative examples are solved to determine the
ductility with respect to curvature of singly and doubly-reinforced beams.
Moreover, numerical problems are solved to illustrate the design of beams,
columns and beam-column joints.

16.40.2 Displacement Ductility

Version 2 CE IIT, Kharagpur


It is essential that an earthquake resistant structure should be capable of
deforming in a ductile manner when subjected to lateral loads in several cycles in
Version 2 CE IIT, Kharagpur
the inelastic range. Let us take a single degree of freedom oscillator, as shown in
Fig. 16.40.1. In the elastic response, the oscillator has the maximum response at
a. The area oab represents the potential energy stored when maximum
deflection occurs. The energy is converted into kinetic energy when the mass
returns to zero position. Figure 16.40.2 shows the oscillator forming a plastic
hinge at a much lower response c when the deflection response continues along
cd, d being the maximum response. The potential energy at the maximum
response is now represented by the area ocde. When the mass returns to zero
position, the part of the potential energy converted to kinetic energy is
represented by fde, while the other energy under the area ocdf is dissipated by
the plastic hinge by being transferred into heat and other forms of energy, which
are irrecoverable. It is thus evident that, elastically, the full potential energy is
returned to kinetic energy, while elastoplastically a part of the energy is
converted into kinetic energy. Hence, the potential energy stored in the
elastoplastic structure may not be equal to that in elastic structure and the
maximum deflection of the elastoplastic structure may not be equal to that of
elastic structure. Figures 16.40.3 and 4 present equal maximum deflection and
equal maximum potential energy responses of two structures.

The displacement ductility factor μ, a measure of ductility of a structure, is


defined as the ratio of Δu, and Δy, where Δu, and Δy are the respective lateral
deflections at the end of post elastic range and when the yield is first reached.
Thus, we have

μ (with respect to displacement) = Δu / Δy


(16.4)
The values of displacement ductility factor should range from 3 to 5.

16.40.3 Curvature Ductility


The curvature ductility factor μ is defined as the ratio of φu and φy, where
φu and φy are the respective curvatures at the end of postelastic range and at the
first yield point of tension steel, as stipulated in cl. 3.3 of IS 13920:1993. Thus,
we have

μ (with respect to curvature) = φu / φy, (16.5)

It should be noted that the curvature ductility factor is significantly different


from the displacement ductility factor. At the start of yielding in a frame, the
deformations concentrate at the positions of plastic hinge. Therefore, when a
frame is deflected laterally in the postelastic range, the φu / φy, ratio in a plastic
hinge may be greater than Δu, / Δy ratio.

Version 2 CE IIT, Kharagpur


16.40.4 Rotational Ductility
In a similar manner, the rotational ductility factor μ is defined as the ratio of
θu and θy, where θu and θy are the respective rotations of at the end of postelastic
range and at the first yield point of tension steel. Thus, we have

μ (with respect to rotation) = θu / θy


(16.6)

Thus, there are three methods of defining the ductility. In general, it can be
stated that the ductility is the ratio of absolute maximum deformation at the end
of postelastic range to the yield deformation. Accordingly, the ductility can be
defined with respect to strain, rotation, curvature or deflection. Rotation and
curvature based ductility factors take into account shape and size of the member.
Though there is no special advantage of one or the other definition, we will
consider curvature ductility, as stipulated in cl. 3.3 of IS 13920:1993 and
explained in sec. 16.40.3 (Eq. 16.5).

16.40.5 Advantages of Ductility


The following are the advantages of a reinforced concrete structure having
sufficient ductility:

(i) A ductile reinforced concrete structure may take care of


overloading, load reversals, impact and secondary stresses due
to differential settlement of foundation.

(ii) A ductile reinforced concrete structure gives the occupant


sufficient time to vacate the structure by showing large
deformation before its final collapse. Accordingly, the loss of life
is minimised with the provision of sufficient ductility.

(iii) Properly designed ductile joints are capable of resisting forces


and deformations at the yielding of steel reinforcement.
Therefore, these sections can reach their respective moment
capacities, which is one of the assumptions in the design of
reinforced concrete structures by limit state method.

16.40.6 Expressions of Ductility of Reinforced Concrete


Rectangular Beams
(A) Singly-reinforced rectangular section

Version 2 CE IIT, Kharagpur


igures 16.40.5a, b and c show the cross-section of a singly-reinforced
rectangular beam, strain profile and curvature at yield and ultimate stages. From
Fig. 16.40.5b, we have:

φy = ∈y / (d – kd) (16.7)
where ∈y = fy/Es (16.8)

k = - (mp/100) + {(mp/100)2 + 2 (mp/100)}1/2 (16.9)

m = 280 / 3σcbc
(16.10)

and σcbc = permissible stress of concrete in bending compression.

From Fig. 16.40.5c, we have:

φu = ∈uc / xu
(16.11)

where ∈uc = 0.0035, as given in cl. 38.1b of IS 456:2000.

From Eq. 3.17 of Lesson 5, we have for singly-reinforced sections,

xu / d = 0.87 fy p / (36) (fck)


(16.12)

Version 2 CE IIT, Kharagpur


where p = 100 Ast / bd
(16.13)

It is also known that xu / d ≤ xu, max / d for under-reinforced sections. Substituting


Eqs. 16.7 and 16.11 in Eq. 16.5, we have:

Curvature ductility μ = (∈uc / ∈y) {(1 – k) / (xu/d)}


(16.14)

Substituting ∈y, k and (xu/d) from Eqs.16.8, 9 and 12, respectively, and using
∈uc = 0.0035 and Ey = 200000 N/mm2, we have:

μ = (25200/0.87 fy) (fck / p fy) ⎧⎨1 + ( mp / 100 ) − ( mp / 100 )2 + 2( mp / 100 ) ⎫⎬


⎩ ⎭
(16.15)

From Fig. 16.40.5c, we also have

xu /(d-xu) = ∈uc / ∈ust


which gives

xu / d =∈uc / (∈uc + ∈ust)


(16.16)

where ∈ust = maximum strain in tension steel = μs ∈y, where μs = Ductility in steel
with respect to strain.

Thus, the curvature ductility μ = φu/φy (Eq. 16.5) can be determined in any
of the following ways:

(i) Employing Eq. 16.14 using ∈uc = 0.0035 and determining ∈y, k and (xu/d)
from Eqs. 16.8, 16.9 and 16.12, respectively from the given fck, fy and p.

(ii) Employing Eq. 16.15 from the given fck , fy and p.

The value of k can be determined by finding the depth of the neutral axis
taking moment of the compression concrete and the tensile steel about the
neutral axis directly in place of employing Eq. 16.9. Numerical problems are
solved in sec. 16.40.12 to illustrate the determination of ductility factor with
respect to curvature.

The value of xu/d can also be determined from Eq.16.16 if the value of μs,
ductility in steel with respect to strain is given.

Version 2 CE IIT, Kharagpur


(B) Doubly-reinforced rectangular sections.

Doubly-reinforced rectangular sections have the area of compression


steel = Asc in addition to the area of tension steel = Ast. Accordingly, the value of
k is obtained after determining the depth of the neutral axis by taking moment of
the compression concrete, compression steel and tension steel about the neutral
axis. The expression of xu/d is derived below. Equating the total compressive
force C due to concrete (= 0.36 fck b xu) and compression steel (= fscAsc) with the
tensile force T (= 0.87 fy Ast) for a doubly-reinforced rectangular section, we
have:

0.36 fck b xu + fsc Asc = 0.87 fy Ast


(16.17)

which gives

xu 0.87 f y Ast f sc Asc


= −
d 0.36 f ck bd 0.36 f ck bd
(16.18)

Using Ast = p bd/100 and Asc = pc bd/100 in Eq. 16.18, we have:

(xu/d) = (fy /36fck) {0.87p – fsc pc / fy}


(16.19)

When fsc = 0.87 fy


(16.20)

we have from Eq. 16.19:

(xu/d) = (0.87 fy/36 fck) (p – pc) ≤ (xu, max/d)


(16.21)

Accordingly, the ductility factor with respect to curvature for the doubly-reinforced
rectangular sections can be determined as explained through illustrative
examples in sec. 16.40.12.

16.40.7 Factors Influencing Ductility


The following factors influence the ductility of reinforced concrete sections.

Version 2 CE IIT, Kharagpur


(i) Figure 16.40.6 shows the plots of ductility μ with respect to
curvature versus p or (p – pc) for singly / doubly-reinforced
rectangular sections for Fe 415 steel and three grades of concrete.
It is evident from the plots that the ductility decreases with the
increase of p or (p – pc). Accordingly, cl. 6.2.2 of IS 13920:1993
stipulates the maximum percentage of steel on any face at any
section as 2.5. Providing high percentage of tension steel will cause
brittle failure by crushing of concrete. Thus, the sections should be
designed as under-reinforced.

(ii) Equation 16.15 reveals that the ductility decreases with increasing
grade of steel for a particular grade of concrete and specific
percentage of steel. Hence, Fe 250 (mild steel) is the most
preferred steel from the point of ductility of reinforced concrete
sections. Clause 5.3 of IS 13920:1993 recommends steel
reinforcement of grade Fe 415 or less. However, Fe 500 and Fe
550, produced by thermo-mechanical treatment process are also
recommended by the code as they have elongation more than 14.5
per cent provided they conform to other requirement of IS
1786:1985.

Version 2 CE IIT, Kharagpur


(iii) Equation 16.15 and Fig. 16.40.6 show that the ductility increases
with increasing grade of concrete for a particular grade of steel and
specific percentage of steel. Accordingly, cl.5.2 of IS 13920:1993
prescribes the minimum grade of concrete as M 20 for all buildings
which are more than three storeys in height.

(iv) Lower values of k and xu will have higher values of ductility as


evident from Eq. 16.14. Accordingly, T beams have more ductility
than that of rectangular beams because of the reduced depth of
neutral axis due to the presence of enlarged compression flange.

(v) The presence of lateral reinforcement prevents premature shear


failure to enable the sections undergoing sufficient deformation.
Moreover, lateral reinforcement in the compression zone arrests
the buckling of compression reinforcement. Thus, lateral
reinforcement, though cannot increase the ductility directly, helps to
reach the attainable ductility as per the design.

16.40.8 Design for Ductility

The objectives of the ductile design of reinforced concrete members are to


ensure both strength and ductility for the designed structures or members.
Strength of members can be assured by proper design of the sections following
limit state method as explained earlier. However, for ensuring ductility, specific
recommendations are to be followed as given in IS 13920:1993 regarding the
materials, dimensions, minimum and maximum percentages of reinforcement.
Further, detailing of reinforcement plays an important role. Accordingly, some of
the major steps to be followed in the design are given below which will ensure
sufficient ductility in the design.

(i) General specification of materials

(a) The minimum grade of concrete shall be M 20 for all buildings,


which are more than three storeys in height (cl. 5.2 of IS
13920:1993).

(b) Steel reinforcing bars of grade Fe 415 or less shall be used.


However, steel bars of grades Fe 500 and Fe 550 may be used if
they are produced by thermo-mechanical treatment process
having elongation more than 14.5 per cent (cl. 5.3 of IS
13920:1993).

Version 2 CE IIT, Kharagpur


(ii) General guidelines in the design and detailing

(a) Simple and regular layout should be made avoiding any offsets of
beams to columns or offsets of columns from floor to floor.
Changes of stiffness of columns should be gradual from floor to
floor.
(b) In a reinforced concrete frame, beams and columns should be
designed such that the inelasticity is confined to beams only,
while the columns should remain elastic. To satisfy this
requirement, the sum of the moment capacities of the columns at
a beam-column joint for the design axial loads should be greater
than the sum of the moment capacities of the beams along each
principal plane. Therefore,

∑ Mcolumn 〉 1.2 ∑ Mbeams


(16.22)

Version 2 CE IIT, Kharagpur


The moment capacities of beams and columns are such that the column
moments oppose the beam moments as shown in Fig. 16.40.7.

(c) The beams and columns should be designed taking into account
the reversal of stresses due to the nature of earthquake forces.

(d) Beam-column connections should be made monolithic.

(e) The following are the requirements of flexural members:

• The factored axial stress on the member due to earthquake


loading shall not exceed 0.1 fck (cl. 6.1.1 of IS 13920:1993).

• The width to depth ratio of the member shall preferably be


more than 0.3 (cl. 6.1.2 of IS 13920:1993).

Version 2 CE IIT, Kharagpur


• The width of the member should not be less than 200 mm
(cl. 6.1.3 of IS 13920:1993).

• The total depth D of the member shall preferably be not


more than 1/4 of the clear span (cl. 6.1.4 of IS 13920:1993).

• The minimum percentage of tension steel on any face at any


section is 24 f ck / f y , where fck and fy are in N/mm2 (cl.
6.2.1b of IS 13920:1993).

• The maximum percentage of steel on any face at any


section is 2.5 (cl. 6.2.2 of IS 13920:1993).

• The positive steel at a joint face must be at least equal to


half the negative steel at that face (cl. 6.2.3 of IS
13920:1993).

• The redistribution of moments shall be used only for vertical


load moments and not for lateral load moments (cl. 6.2.4 of
IS 13920:1993).

Version 2 CE IIT, Kharagpur


• The anchorage length of top and bottom bars of the beam in
an external joint shall be measured beyond the inner face of
the column and should be equal to development length in
tension plus 10 times the bar diameter minus the allowance
for 90 degree bends (Fig. 16.40.8). In an internal joint, both
face bars of the beam shall be taken continuously through
the column (cl. 6.2.5 of IS 13920:1993).

• Longitudinal bars shall be spliced if hoops are provided over


the entire splice length at a spacing not exceeding 150 mm,
as shown in Fig. 16.40.9. The lap length shall be at least
equal to the development length in tension. However, lap
splices should not be provided (a) within a joint, (b) within a
distance of 2d from the face of the joint, and (c) within a
quarter length of the member where flexural yielding may
generally occur under the effect of earthquake forces.
Moreover, at one section not more than 50 per cent of the
bars should be spliced (cl. 6.2.6 of IS 13920:1993).

16.40.9 Design for Shear in Flexural Members

Version 2 CE IIT, Kharagpur


Web reinforcement shall consist of either (a) vertical hoops or (b) U-
stirrup with a 135 degree hook and a 10 diameter extension (but not 〈 75 mm) at
each end and a crosstie, as shown in Figs. 16.40.10a and b, respectively. The
crosstie is a bar having a 135 degree hook with a 10 diameter extension (but not
〈 75 mm) at each end (cl. 6.3.1 of IS 13920:1993). Normally, vertical hoops shall
be used except in compelling circumstances when U-stirrups and crosstie shall
be used.

The minimum diameter of the bar forming a hoop shall be 6 mm, except
for beams with clear span exceeding 5 m, where the minimum diameter of the
bar forming a hoop shall be 8 mm (cl. 6.3.2 of IS 13920:1993).

Version 2 CE IIT, Kharagpur


As per cl. 6.3.3 of IS 13920:1993, the shear force to be resisted by the
vertical hoops shall be the maximum of:

(i) calculated factored shear force as per analysis, and


(ii) shear force due to formation of plastic hinges at both ends of the
beam plus the factored gravity load on the span. The expressions
are given below (Fig. 16.40.11 a and b):

(a) for sway to right:

⎡ M As + M Bh ⎤
Vu ,a = VaD + L − 1.4 ⎢
u ,lim u ,lim ⎥
⎢ L AB ⎥
⎣ ⎦
(16.23)
and

Version 2 CE IIT, Kharagpur


D+ L
⎡ M uAs,lim + M uBh,lim ⎤
Vu ,b = V b + 1.4 ⎢ ⎥ , and
⎣ L AB ⎦

(b) for sway to left

D+ L
⎡ M uAh,lim + M uBs,lim ⎤
Vu ,a = V a + 1.4 ⎢ ⎥
⎣ L AB ⎦
(16.24)
and

D+ L
⎡ M uAh,lim + M uBs,lim ⎤
Vu ,b = V b − 1.4 ⎢ ⎥
⎣ L AB ⎦

where

M uAs Ah Bs Bh
,lim , M u ,lim and M u ,lim , M u ,lim = sagging and hogging moments of
resistance of the beam section at ends A and B, respectively, LAB = clear span of
the beam, VaD + L and VbD + L = shear forces at ends A and B, respectively, due
to vertical loads with a partial safety factor of 1.2 on loads.

Out of the two values of Vu,a , the design shear at the end A shall be the
larger. Similarly, out of the two values of Vu,b , the design shear at the end B shall
be the larger.

The expressions of Eqs. 16.23 and 16.24 are based on the assumption
that the ratio of the actual ultimate tensile stress to the actual tensile yield
strength of steel is not less than 1.25. Accordingly, 1.25 times the yield strength
of steel divided by 0.87 is 1.43, which has been taken as 1.4 in Eqs. 16.23 and
16.24.

Version 2 CE IIT, Kharagpur


The contributions of bent up bars and inclined hoops, if any, shall not be
considered to shear resistance of the section, as stipulated in cl. 6.3.4 of IS
13920:1993. Figure 16.40.12 shows the spacing of hoops and the following
recommendations of cl. 6.3.5 of IS 13920:1993 are to be followed:

• Over a length of 2d at either end of the beam, the spacing of hoops shall
not exceed (a) d/4, and (b) 8 times the diameter of the smallest
longitudinal bar. However, the spacing shall not be less than 100 mm.

• The first hoop shall be at a distance not exceeding 50 mm from the joint
face.

• Vertical hoops at the same spacing as mentioned above shall also be


provided over a length of 2d on either side of a section where flexural
yielding may occur under the effect of earthquake forces.

• At other places, the beam shall have vertical hoops at a spacing not
exceeding d/2.

Version 2 CE IIT, Kharagpur


16.40.10 Column and Frame Members Subjected to
Bending and Axial Load

The following requirements are applicable to frame members having


factored axial stress more than 0.1 fck under the effect of earthquake forces.

The dimension of the member should be at least 200 mm. However, in


frames having beams with centre-to-centre span exceeding 5 m or columns of
unsupported length exceeding 4 m, the shortest dimension of the column shall be
at least 300 mm (cl. 7.1.2 of IS 13920:1993).

(A) Longitudinal reinforcement

Longitudinal reinforcing bars shall be spliced only in the central half of the
member length. Hoops shall be provided over the entire splice length at spacing
not exceeding 150 mm centre to centre. A maximum of 50 per cent of the bars
shall be spliced at one section (cl. 7.2.1 of IS 13920:1993).

The detailing of any area of column extending more than 100 mm beyond
the confined core due to architectural requirement (cl. 7.2.2 of IS 13920:1993)
shall be as follows:

Version 2 CE IIT, Kharagpur


(a) If the contribution of this area to strength is considered, then it
should be the minimum longitudinal and transverse
reinforcement as per IS 13920:1993.

(b) If this area is treated as non-structural, minimum longitudinal


and transverse reinforcement shall be provided as per IS
456:2000, as shown in Fig. 16.40.13.

(B) Transverse reinforcement

Transverse reinforcement shall consist of spiral or circular hoops for


circular columns and rectangular hoops for rectangular columns. Rectangular
hoops shall be a closed stirrup having a 135 degree hook with a 10 diameter
extension (but not 〈 75 mm) at each end, as shown in Fig. 16.40.14 (cl. 7.3.1 of
IS 13920:1993).

Version 2 CE IIT, Kharagpur


The parallel legs of rectangular hoop shall be spaced not exceeding 300
mm c/c. For spacing more than 300 mm, a crosstie shall be provided as shown in

Version 2 CE IIT, Kharagpur


Fig. 16.40.15. Alternatively, a pair of overlapping hoops may be provided within
the column, as shown in Fig. 16.40.16. Hooks shall engage peripheral
longitudinal bars (cl. 7.3.2 of IS 13920: 1993).

The maximum spacing of hoops shall be half the least lateral dimension
of the column, except where spiral confining reinforcement is provided as per cl.
7.4 of IS 13920:1993 (cl. 7.3.3 of IS 13920:1993).

Version 2 CE IIT, Kharagpur


The design shear force for columns, as recommended in cl. 7.3.4 of IS
13920:1993, shall be the maximum of:

(i) calculated factored shear force as per analysis, and


(ii) a factored shear force given by

⎡ M bL + M ubR,lim ⎤
Vu = 1.4 ⎢ u ,lim ⎥
⎣ hst ⎦
(16.25)

where
M ubL,lim and M ubR
,lim = moments of resistance, of opposite sign, of beams
framing into the column from opposite faces, as shown in Fig. 16.40.17, and

hst = storey height.

Version 2 CE IIT, Kharagpur


16.40.11 Special Confining Reinforcement

Version 2 CE IIT, Kharagpur


The following requirement shall be met with, unless a larger amount of
transverse reinforcement is required from shear strength considerations.

Special confining reinforcement shall be provided over a length lo from


each joint face towards mid-span, and on either side of any section, where
flexural yielding may occur under the effect of earthquake forces, as shown in
Fig. 16.40.18. The length lo shall not be less than (a) larger lateral dimension of
the member at the section where yielding occurs, (b) 1/6 of clear span of the
member, and (c) 450 mm (cl. 7.4.1 of IS 13920:1993).

Special confining reinforcement shall extend at least 300 mm into the


footing or mat when a column terminates into a footing or mat, as shown in Fig.
16.40.19 (cl. 7.4.2 of IS 13920:1993).

Special confining reinforcement shall be provided over the full height of


the column for the following situations (cls. 7.4.3 to 5 of IS 13920:1993):

(i) when the calculated point of contra-flexure, under the effect of


gravity and earthquake loads, is not within the middle half of the
member clear height,

(ii) columns supporting reactions from discontinued stiff members,


such as walls, and

(iii) columns which have significant variation in stiffness along their


heights.

Version 2 CE IIT, Kharagpur


The maximum spacing of hoops used as special confining reinforcement
shall be 1/4 of minimum member dimension. However, the spacing of hoops
should not be less than 75 mm nor more than 100 mm (cl. 7.4.6 of IS 13920:
1993).

The area of cross-section Ash of the bar forming circular hoops or spiral,
to be used as special confining reinforcement, shall not be less than (cl.7.4.7 of
IS 13920:1993).

f ck ⎡ Ag ⎤
Ash = 0.09 S Dk ⎢ − 1⎥
f y ⎣ Ak ⎦
(16.26)

where

Ash = area of the bar cross-section,


S = pitch of spiral or spacing of hoops,
Dk = diameter of core measured to the outside of the spiral or
hoop,
fck = characteristic compressive strength of concrete cube,
fy = yield stress of steel (of circular hoop or spiral),
Ag = gross area of column cross-section, and
Ak = area of concrete core = (π/4) D2k

The area of cross-section Ash of the bar forming rectangular hoop, to be used as
special confining reinforcement, shall not be less than (cl.7.4.8 of IS
13920:1993):

f ck ⎡ Ag ⎤
Ash = 0.18 S h ⎢ − 1 .0 ⎥
f y ⎣ Ak ⎦
(16.27)

where

h = longer dimension of the rectangular confining hoop


measured to its outer face. It shall not exceed 300 mm (see
Figs. 16.40.14, 15 and 16), and

Ak = area of confined concrete core in the rectangular hoop


measured to its outside dimensions.

Version 2 CE IIT, Kharagpur


16.40.12 Numerical Problems

Problem 1: Determine the ductility with respect to curvature of the beam shown
in Fig. 16.40.20 using M 20 and Fe 250.

Solution 1: This is a singly-reinforced rectangular beam of b = 300 mm, d =


540 mm, D = 600 mm, Ast = 942 mm2 (3-20T), fck = 20 N/mm2 and
fy = 250 N/mm2.

Step 1: Checking for minimum and maximum percentages of


reinforcement (sec. 16.40.8 (ii) e)

Minimum percentage of Ast = 100 (0.24) f ck / fy = 0.429


Maximum percentage of Ast = 2.5
For this problem p = 94200/(300) (540) = 0.581. Hence, Ast is within
minimum and maximum percentages i.e., 0.429 < 0.581 < 2.5).

Step 2: Determination of ∈y, k and xu/d (Eqs. 16.8, 16.9 and 16.12)

From Eq. 16.8, ∈y = fy / Es = 250/200000 (1)


2 1/2
From Eq. 16.9, k = - (mp/100) + {(mp/100) + 2 (mp/100)} where
m = 280/3σcbc = 280/3(7) = 13.33 and p = 0.581. Therefore, k =
0.3236.

Taking moment of compression concrete and tension steel about


the neutral axis, we have: 300 (x2/2) = 942 (13.33) (d - x),

Version 2 CE IIT, Kharagpur


or x2 + 83.7124x – 45204.696 = 0, which gives x = 174.742 mm.
Therefore, k = x/d = 0.3236.

From Eq. 16.12, xu/d = 0.87 fy p/(36 - fck) ≤ (xu, max/d) , which gives:
xu/d = 0.87(250) (0.581)/(36) (20) = 0.1755 < 0.53, (as xu, max/d = 0.53).

Step 3: Determination of ductility with respect to curvature (Eq. 16.14)

From Eq. 16.14, μ = (∈uc /∈y) {(1 – k) / (xu/d)}. Using ∈uc = 0.0035
and substituting the values of ∈y, k and (xu/d) from step 2,

μ = {0.0035 (200000)/250} {(1 –0.3236) / 0.1755} = 10.79

Hence, the ductility of this beam μ = 10.79.

Problem 2: Compare the ductility with respect to curvature of the cross-section


of the beam of Fig. 16.40.21 using (a) M 20 and Fe 250, and (b) M
20 and Fe 415.

Solution 2: This is a doubly-reinforced rectangular beam of b = 300 mm, d =


540 mm, D = 600 mm, d′ = 50 mm, Ast = 1884 mm2 (6-20T), Asc =
942 mm2 (3-20T), fck = 20 N/mm2 and fy = 250 N/mm2 for (a); and
415 N/mm2 for (b). The amounts of steel are within minimum and
maximum percentages. Here, p = 1.162 and pc = 0.581. The
modular ratio m for M 20 for both (a) and (b) parts of the problem =
13.33 as in Problem 1.

Version 2 CE IIT, Kharagpur


Step 1: Determination of k for both parts a and b
Taking moment of compression concrete, compression steel and
tension steel about the neutral axis, we have:

300 (x2/2) + 942 (1.5 m – 1) (x – 50) = 1884 (13.33) (d –x)


or x2 + 286.7134x – 96373.822 = 0, which gives: x = 198.5862272
mm. Hence, k = x/d = 198.5862272/540 = 0.3677

Step 2: Determination of xu/d (Eq. 16.21)

Equation 16.21 is: xu/d = (0.87 fy/36 fck) (p – pc) ≤ (xu, max/d)

For (a), when fy = 250 N/mm2:


xu/d = {0.87 (250)/36(20)} (0.581) = 0.1755 < 0.53, (as xu,max/d =
0.53)

For (b), when fy = 415 N/mm2:


xu/d = {0.87 (415)/36(20)} (0.581) = 0.2913 < 0.48, (as xu,max/d =
0.48)

Step 3: Determination of ductility with respect to curvature (Eq. 16.14)

Equation 16.14 is: μ = (∈uc / (∈y) {(1 – k) / (xu / d)}. Using ∈uc =
0.0035, ∈y = fy/Es, and substituting the values of k and xu/d from
steps 1 and 2, we have,

For (a): μ = {(0.0035) (200000)/250} {(1 – 0.3677) / 0.1755} =


10.088
For (b): μ = {(0.0035) (200000)/415} {(1 – 0.3677) / 0.2913} = 3.66

Hence, the ductility of this doubly-reinforced beam is 10.088 when


Fe 250 is used and is 3.66 when Fe 415 is used. The loss of
ductility when opting for Fe 415 in place of Fe 250 is noted.

Problem 3: Design an inner beam of span 6 m of one reinforced concrete


frame for ductility using M 25 and Fe 415. The beam has negative
bending moment of 300 kNm and shear force of 250 kN at the face
of beam-column joint due to gravity and earthquake loads.

Solution 3:

Step 1: Selection of dimensions and determining areas of steel

Version 2 CE IIT, Kharagpur


Let us assume the dimensions b = 300 mm and D = 600 mm. With
effective cover of 55 mm, d = 545 mm and d′ /D = 0.1 and using
partial safety factor for load as 1.2, we have: Mu/bd2 =
1.2(300)106/300 (545) (545) = 4.04 N/mm2

The design for ductility has been given in sec. 16.40.8 (ii) following
the general guidelines in the design and detailing. Following the
general guidelines, Design Tables 20.2 and 20.3 have been
presented in the book, “Reinforced Concrete Limit State Design”
(6th Edition) by A.K. Jain. These tables give the percentages of
tension and compression steel for M 20 and M 25 grades up to p/pc
≤ 2.0 for four different values of d′/D as 0.05, 0.10, 0.15 and 0.20,
and Mu/bd2. Accordingly, we refer to Table 20.3 for this problem,
when Mu/bd2 = 4.04 N/mm2 and d′/D = 0.1 to get p = 1.2554 and pc
= 0.6276.

The minimum percentage of steel = 100 (0.24) 25 / 415 = 0.289


and maximum percentage of steel = 2.5. Thus, the determined
percentages are within the respective limits. So, Ast = 1.2554 (300)
(545) / 100 = 2052.58 mm2. Provide 2-28T + 2-25 T (1231 + 981 =
2212 mm2) giving p provided = 1.353 per cent. The compression
steel Asc = 0.6276 (300) (545)/100 = 1026.13 mm2. Provide 2-28T
(1232 mm2), giving pc provided = 0.753 per cent.

Step 2: Design for shear

With partial safety factor for loads as 1.2, the factored shear force
Vu = 1.2 (250) = 300 kN.

τv = Vu/bd = 300/(0.3) (545) = 1.83 N/mm2,


τc = (from Table 19 of IS 456 with p = 1.353%) = 0.716 N/mm2,
and τmax (from Table 20 of IS 456) = 3.1 N/mm2

Since τc 〈τv 〈τmax, we have to provide vertical hoops (stirrups). Providing 10 mm–
2 legged hoops (Asv = 157 mm2), we have the spacing of hoops as

Sv = 0.87 fy Asv d/(Vu - τc bd) = 0.87 fy Asv / (τv - τc)b = 0.87(415)


(157) / (1.83 – 0.716) 300 = 169.61 mm.

Version 2 CE IIT, Kharagpur


The arrangement, minimum and maximum spacing of hoops are given in sec.
16.40.9. The maximum spacing is (a) d/4 = 136.25 mm, (b) 8 (25) = 200 mm and
the spacing shall not be less than 100 mm, for a length of 2 d at either end of the
beam. So, provide 10 mm – 2 legged hoops @130 mm c/c up to 1200 mm from
the face of the column and then @ 260 mm c/c up to the centre-line, to be placed
symmetrically. It may be noted that the maximum spacing at other places is d/2 =
272 mm. The first hoop shall be provided at a distance of 30 mm (〈 50 mm as
per the code) from the face of the column. There will be ten hoops in 1200 mm.
Figure 14.40.22 shows the cross-section of the beam.

Problem 4: Check (i) if the inner beam-column joint of a reinforced concrete


frame satisfies weak girder–strong column proportion and (ii) shears

Version 2 CE IIT, Kharagpur


in beam and column, using M 25, Fe 415 and other details as given
below:

Clear span of the left beam = 5 m, clear span of the right beam = 4
m, slab thickness = 120 mm, finish on slab = 50 mm, live loads on
floor = 2.0 kN/m2, axial load on column = 800 kN, beam dimensions
= 300 mm × 600 mm, steel bars at top on either side = 4-25T + 2-
20T (1.6 per cent), steel bars at bottom on either side = 2-25T + 1-
20T (0.8 per cent), column dimensions = 300 mm × 600 mm, column
reinforcement bars = 8-28T + 4 -25T (3.827 per cent), and height of
storey = 3.6 m (Fig. 16.40.23).

Solution 4:

(i) Beam-column Joint

As given in sec. 16.40.8 (ii), the requirement of the design is to


satisfy Eq. 16.22, i.e.,

∑ Mcolumn 〉 1.2 ∑ Mbeam

along each principal plane. We consider the joint in bending about


the weaker axis and use tables of SP-16 for the computation.

Assuming d′ / d = 0.1 and with p = 1.6%, Table 51 of SP-16 gives


Mu/bd2 = 4.77 for doubly-reinforced section. This gives the hogging
moment capacity as Mu = 4.77 (300) (545) (545) = 425.04 kNm.
The sagging moment capacity of singly-reinforced section is
obtained from Table 3 of SP-16, where Mu / bd2 = 2.503 for p =
0.8%. Thus, Mu = 2.503 (300) (545) (545) = 223.04 kNm. Therefore,

∑ Mbeam = 425.04 + 223.04 = 648.08 kNm

For columns Pu = 1.2 (800) = 960 kN gives Pu/fck bD = 1.2 (800) /


(25) (3) (60) = 0.213 and p / fck = 3.827/25 = 0.153. Chart 32 of SP-
16 gives Mu/fck bD2 = 0.256. The column moment Mu = 0.256 (25)
(300) (600) (600) = 691.2 kNm. Therefore,

∑ Mcolumn = 2 (691.2) = 1382.4 kNm.

Here, ∑ Mcolumn 〉 (1.2) ∑ Mbeam (= 1.2 (648.08) = 777.7 kNm)


Hence, the requirement of Eq. 16.22 is satisfied.

(ii) Shear in beam and column

Step 1: Shear capacity of left beam having spacing of 4m c/c

Version 2 CE IIT, Kharagpur


Live loads = 4 m (2 kN/m2) = 8 kN/m
Dead load of slab = 4 (0.12 + 0.05) (25) = 17.0 kN/m
Dead load of web of beam = 0.3 (0.48) (25) = 3.6 kN/m

Total dead load = 20.6 kN/m.

Factored shear force due to gravity loads = 1.2 (20.6 + 8) (5)/ 2 =


85.8 kN.
Factored shear force due to plastic hinge = 1.4 (648.08) / 5 =
181.46 kN
Thus, Vu = 85.8 + 181.46 = 267.26 kN

τv = 267.26 / 300 (0.545) = 1.63 N/mm2


τc (from Table 19 of IS 456 for p = 1.6 %) = 0.756 N/mm2.
τcmax (from Table 20 of IS 456) = 3.1 N/mm2.

Since τc 〈τv 〈τcmax, we provide 8 mm-2 legged hoops (Asv = 100


mm2) of spacing sv where sv is as follows:

sv = 0.87 fy Asv / (τv - τc)b = 0.87 (415) (100) / (1.63 – 0.756) (300) =
137.7 mm c/c.

For a distance of 2 d (= 1090 mm) from the face of the column, the
maximum spacing is: (a) 0.25 (545) = 136.25 mm and (b) 8 (20) =
160 mm. The spacing should not be less than 100 mm. Beyond
1090 mm, the spacing = d/2 = 272.5 mm.

Let us change the diameter of hoop as 10 mm beyond 1090 mm,


for which the spacing is 0.87 (415) (157)/(1.63 – 0.756) (300) =
216.36 mm c/c.

So, let us provide 8 mm-2 legged hoops @ 130 mm c/c up to 1.2 m


from the face of the column at either end. The first hoop shall be at
a distance of 30 mm from the face of the joint. We need ten hoops.
Beyond 1.2 m, provide 10 mm-2 legged hoops @ 200 mm c/c,
symmetrically.

Step 2: Checking of column for storey height = 3600 mm

Factored shear in column Vu = 1.4 (648.08)/3.6 = 252.03 kN. This


gives τv = 252.03/3 (60) = 1.4 N/mm2.

τc (from Table 19 of IS 456: 2000 with 1.913% Ast at each face) =


0.806 N/mm2. This shall be multiplied by a factor = 1+3 (1.2)
(800)/(30) (6) (25) = 1.64. However, the multiplying factor is limited

Version 2 CE IIT, Kharagpur


to 1.5 (see cl. 40.2.2 of IS 456: 2000). So, τc = 1.5 (0.806) = 1.209
N/mm2.

τcmax (from Table 20 of IS 456: 2000) = 3.1 N/mm2. Since τc 〈τv


〈τcmax, we provide 8 mm – 2 legged hoop of mild steel (Fe 250), for
which the spacing is

sv = 0.87fyAsv/(τv - τc)b = 0.87(250)(100)/(1.4 – 1.209) (300) =


379.58 mm.

The maximum spacing is 0.5 (300) = 150 mm (cl. 7.3.3 of IS


13920:1993). Hence, provide 8 mm-2 legged hoops of Fe 250
@150 mm c/c except in the confining steel zone.

Step 3: Confining steel

Here, axial stress = 1.2 (800) / 30(6) = 5.33 N/mm2 〉 0.1 fck (=2.5
N/mm2). So, we provide confining steel of rectangular closed loops
of spacing lesser of (a) 0.25 (300) and (b) 100 mm (cl.7.4.6 of IS
13920: 1993). So, the spacing is 75 mm c/c. From Fig. 16.40.23,
we have h = 200 mm.

From Eq. 16.27, we have: Ash = 0.18 S h {(Ag / Ak) –1} (fck / fy)
= 0.18 (75) (200) [(300) (600) / (184) (504)}-1](25/415) = 153.05
mm2

Provide 2-10 mm rectangular hoops (157 mm2). These confining hoops shall be
provided for a distance greater of: (i) 600 mm, (ii) 3600/6 = 600 mm and (c) 450
mm. So, we provide 8 numbers of 2 legged 10 mm diameter hoops @ 75 mm c/c
up to 630 mm from the joint. The first one is at a distance of 30 mm from the joint
(see sec.16.40.11).

16.40.13 Practice Questions and Problems with Answers


Q.1: Define and explain ductility with respect to (a) displacement, (b) curvature
and (c) rotation of a reinforced concrete structure.

A.1: Secs. 16.40.2, 3 and 4 are the respective answers of part (a), (b) and (c).

Q.2: State the advantages of ductility in reinforced concrete structures.

A.2: Sec. 16.40.5 is the complete answer.

Q.3: Derive the expressions of ductility of (a) singly-reinforced and (b) doubly-
reinforced concrete beams.

Version 2 CE IIT, Kharagpur


A.3: Part (A) and Part (B) of sec. 16.40.6 are the answers of (a) and (b),
respectively.

Q.4: State the factors influencing ductility of reinforced concrete sections.

A.4: Sec. 16.40.7 is the complete answer.

Q.5: Compare the ductility with respect to curvature of the cross-section of the
beam of Fig. 16.40.23 using (a) M 25 and Fe 250, and (b) M 25 and Fe
415.

A.5: This is a doubly-reinforced rectangular beam of b = 300 mm, d = 540 mm,


D = 600 mm, d′ = 50 mm, Ast = 3694 mm2 (6-28T), Asc = 1847 mm2 (3-
28T), fck = 25 N/mm2 and fy = 250 N/mm2 for (a) and 415 N/mm2 for (b).
The percentages of tension steel p = 2.28% and pc = 1.14%. The
reinforcing steel is within the range of minimum and maximum
percentages. The modular ratio m = 280/3σcbc = 280/3(8.5) = 10.98.

Step 1: Determination of k for both parts a and b

Taking moment of compression concrete, compression steel and tension


steel about the neutral axis, we have:

300 (x2/2) + 1847(1.5 m – 1) (x-50) = 3694 (m) (d-x)


or x2 + 460.8880667x – 155540.7953 = 0
or x = 226.333 mm giving k = x/d = 0.419

Step 2: Determination of xu/d (Eq. 16.21)

Equation 16.21 is: xu/d = (0.87 fy/36 fck) (p – pc) ≤ (xu, max /d)

Version 2 CE IIT, Kharagpur


For (a), when fy = 250 N/mm2: xu/d = {0.87 (250) / 36(25)} {1.14} = 0.2755
〈0.53
(as xu,max / d = 0.53)

For (b), when fy = 415 N/mm2: xu/d = {0.87 (415) / 36(25)} {1.14} = 0.457
〈0.48
(as xu,max / d = 0.48)

Step 3: Determination of ductility with respect to curvature (Eq. 16.14)

Equation 16.14 is: μ = (∈uc / ∈y) {(1- k)/(xu / d)}


Using ∈uc = 0.0035, ∈y = fy / Es and substituting the values of k and xu /d
from steps 1 and 2, we have

For (a): μ = {(0.0035) (200000)/250} {(1 – 0.419) / (0.2755)} = 5.905


For (b): μ = {(0.0035) (200000)/415} {(1 – 0.419) / (0.457)} = 2.144

Hence, the ductility of this doubly-reinforced beam is 5.905 when Fe 250


is used and is 2.144 when Fe 415 is used.

Q.6: Design the column of a mulitstoreyed building for ductility with M 25 and
Fe 415 subjected to an axial force of 2000 kN and bending moment of
416.67 kNm.

A.6:

Version 2 CE IIT, Kharagpur


Step 1: Dimensions and area of steel of column

Let us take the column dimensions as 500 mm × 500 mm. Considering


partial safety factor for loads = 1.2, we have:

Mu = (1.2) (416.67) = 500 kNm


Pu = (1.2) (2000) = 2400 kN

Pu/fck bD = 2400 / (25) (5) (50) = 0.384


Mu/fckbD2 = 500 / (25) (125) = 0.16
d′/D = 75/500 = 0.15

Chart 45 of SP-16 gives p/fck = 0.16. With p = 0.16 (25)= 4%, we have Ast
= 4(500)(500)/100 = 10000 mm2. Provide 4 - 32T + 8 – 28T + 4 – 25T (=
3217 + 4926 + 1963 = 10106 mm2) as shown in Fig. 16.40.25.

Step 2: Lateral ties

Diameter of lateral tie is the greater of 32/4 = 8 mm or 6 mm. Let us take 8


mm diameter bars.

Version 2 CE IIT, Kharagpur


Maximum pitch is 0.5 (500) = 250 mm (cl. 7.3.3 of IS 13920:1993). So,
provide 8 mm ties @ 250 mm c/c.

Step 3: Confining reinforcement

Here, Pu/A = 2400 (103)/(500) (500) = 9.6 N/mm2 〉 0.1 fck (=2.5 N/mm2).
Hence, confining reinforcement shall be provided.

From Eq. 16.27, we have

f ck ⎡ Ag ⎤
Ash = 0.18 S h ⎢ − 1⎥
f y ⎣ Ak ⎦

Ash = 113 mm2 for 12 mm diameter hoop.


h = 500 – 2 (40 + 12) = 396 mm 〉300 mm.
So, h is revised as 396/2 = 198 mm 〈 300 mm.
Ag = 500 (500) = 250000 mm2
Ak = {500 – 2 (40 + 12 + 8)} (380) = 144400 mm2 and {(Ag/Ak) – 1} =
0.7313.

Hence, S = 113 (415)/{0.18(198) (0.7313) (25)} = 71.9 mm

Provide confining hoops of 12 mm diameter bar @ 70 mm c/c, as shown


in Fig. 16.40.25.

The distance of the confining reinforcement is the largest of: (i) clear span
of column / 6 = 3600/6 = 600 mm, (ii) larger lateral dimension = 500 mm
and (iii) 450 mm. So, provide confining hoops for a distance of 700 mm
from the face of the joint (eleven numbers of hoop).

16.40.14 References
1. Reinforced Concrete Limit State Design, 6th Edition, by Ashok K. Jain,
Nem Chand & Bros, Roorkee, 2002.
2. Limit State Design of Reinforced Concrete, 2nd Edition, by P.C. Varghese,
Prentice-Hall of India Pvt. Ltd., New Delhi, 2002.
3. Advanced Reinforced Concrete Design, by P.C. Varghese, Prentice-Hall
of India Pvt. Ltd., New Delhi, 2001.
4. Reinforced Concrete Design, 2nd Edition, by S. Unnikrishna Pillai and
Devdas Menon, Tata McGraw-Hill Publishing Company Limited, New
Delhi, 2003.
5. Limit State Design of Reinforced Concrete Structures, by P. Dayaratnam,
Oxford & I.B.H. Publishing Company Pvt. Ltd., New Delhi, 2004.

Version 2 CE IIT, Kharagpur


6. Reinforced Concrete Design, 1st Revised Edition, by S.N. Sinha, Tata
McGraw-Hill Publishing Company. New Delhi, 1990.
7. Reinforced Concrete, 6th Edition, by S.K. Mallick and A.P. Gupta, Oxford &
IBH Publishing Co. Pvt. Ltd. New Delhi, 1996.
8. Behaviour, Analysis & Design of Reinforced Concrete Structural Elements,
by I.C.Syal and R.K.Ummat, A.H.Wheeler & Co. Ltd., Allahabad, 1989.
9. Reinforced Concrete Structures, 3rd Edition, by I.C.Syal and A.K.Goel,
A.H.Wheeler & Co. Ltd., Allahabad, 1992.
10. Textbook of R.C.C, by G.S.Birdie and J.S.Birdie, Wiley Eastern Limited,
New Delhi, 1993.
11. Design of Concrete Structures, 13th Edition, by Arthur H. Nilson, David
Darwin and Charles W. Dolan, Tata McGraw-Hill Publishing Company
Limited, New Delhi, 2004.
12. Concrete Technology, by A.M.Neville and J.J.Brooks, ELBS with
Longman, 1994.
13. Properties of Concrete, 4th Edition, 1st Indian reprint, by A.M.Neville,
Longman, 2000.
14. Reinforced Concrete Designer’s Handbook, 10th Edition, by C.E.Reynolds
and J.C. Steedman, E & FN SPON, London, 1997.
15. Indian Standard Plain and Reinforced Concrete – Code of Practice (4th
Revision), IS 456: 2000, BIS, New Delhi.
16. Design Aids for Reinforced Concrete to IS: 456 – 1978, BIS, New Delhi.

16.40.15 Test 40 with Solutions

Maximum Marks = 50 Maximum Time = 30


minutes

Answer all questions.

TQ.1: State the advantages of ductility in reinforced concrete structures.


(10 Marks)
A.TQ.1: Sec. 16.40.5 is the complete answer.

TQ.2: Derive the expressions of ductility of (a) singly-reinforced and (b) doubly-
reinforced concrete beams.
(15 Marks)
A.TQ.2: Part (A) and Part (B) of sec. 16.40.6 are the answers of (a) and (b),
respectively.

Version 2 CE IIT, Kharagpur


TQ.3: Compare the ductility with respect to curvature of the cross section of the
beam of Fig. 16.40.25 using (a) M 30 and Fe 250 and (b) M 30 and Fe
415.

A.TQ.3: This is a doubly-reinforced rectangular beam of b = 300 mm, d = 540


mm, D = 600 mm, d′ = 50 mm, Ast = 1206 mm2 (6 – 16T), Asc = 603 mm2 (
3 – 16T ), fck = 30 N/mm2 and fy = 250 N/mm2 for (a) and 415 N/mm2 for
(b). The modular ratio m = 280/3(10) = 9.33, minimum percentage of steel
= 24 30 / 250 = 0.526 for (a) and = 24 30 / 415 = 0.317 for (b). Here, p =
120600/300(540) = 0.7444 per cent. Thus, percentage of Ast is within the
range of minimum and maximum limits (2.5%).

Step 1: Determination of k for both parts (a) and (b)

Taking moment of compression steel and tension steel about the neutral
axis, we have:
300 (x2/2) + 603(1.5 m-1) (x - 50) = 1206 (m) (d - x)
or x2 + 127.2531x – 43119.123 = 0
which gives x = 153.554 mm, which gives: k = x/d = 0.284.

Step 2: Determination of xu/d (Eq. 16.21)

Equation 16.21 is: xu/d = (0.87 fy /36 fck) (p – pc) ≤ (xu, max/d)

For (a), when fy = 250 N/mm2: xu/d = {0.87 (250) / 36(30)} (0.3722) =
0.0749 〈
0.53 (as xu,max / d = 0.53)

Version 2 CE IIT, Kharagpur


For (b), when fy = 415 N/mm2: xu/d = {0.87 (415) / 36(30)} (0.3722) =
0.124 〈
0.48 (as xu,max / d = 0.48)

Step 3: Determination of ductility with respect to curvature (Eq. 16.14)

Equation 16.14 is: μ = (∈uc / ∈y) {(1 – k) / (xu/d)}

Using ∈uc = 0.0035, ∈y = fy /Es and substituting the values of k and xu / d


from Steps 1 and 2, we have:

For (a): μ = {(0.0035) (200000)/250} {(1 – 0.284) / (0.0749)} = 26.766


For (b): μ = {(0.0035) (200000)/415} {(1 – 0.284) / (0.124)} = 9.74

Hence, the ductility of this doubly-reinforced beam (Fig. 16.40.26) is


26.766 when Fe 250 is used and is 9.74 when Fe 415 is used.

16.40.16 Summary of this Lesson


This lesson defines and explains the ductility factor with respect
to displacement, curvature and rotation. Stating and explaining the
advantages of ductility in the design of reinforced concrete members,
expressions of ductility factor are derived for singly and doubly-reinforced
concrete rectangular beams. Factors influencing the ductility are
explained. General specifications of materials are given and the general
guidelines in the design and detailing of structures are stated as stipulated
in IS 13920:1993. The situations requiring special confining reinforcement
are explained. Several illustrative examples are solved determining the
ductility of singly and doubly-reinforced rectangular beams. Furthermore,
numerical problems are also solved to explain the design of beams,
columns and beam-column joints as per IS 13920:1993.

Version 2 CE IIT, Kharagpur

S-ar putea să vă placă și